Combined Peds/OB Final Exam Set

Ace your homework & exams now with Quizwiz!

A 2-year-old girl has excessive tearing and corneal haziness. The nurse knows that these symptoms may indicate: a. Viral conjunctivitis. c. Congenital cataract. b. Paralytic strabismus. c. Congenital cataract. d. Infantile glaucoma.

d. Infantile glaucoma.

Which screening tests should the school nurse perform for the adolescent (select all that apply)? a. Glucose b. Vision c. Hearing d. Cholesterol e. Scoliosis

B, C, E The school nurse should perform vision, hearing, and scoliosis screening tests according to the school district's required schedule. Glucose and cholesterol screening would be performed in the medical clinic setting.

In terms of cognitive development, the 5-year-old child would be expected to: a. Use magical thinking. b. Think abstractly. c. Understand conservation of matter. d. Be able to comprehend another person's perspective.

A Magical thinking is believing that thoughts can cause events. Abstract thought does not develop until school-age years. The concept of conservation is the cognitive task of school-age children ages 5 to 7 years. Five-year-olds cannot understand another's perspective.

5. Because of the effect of cyclic ovarian changes on the breast, the best time for breast self-examination (BSE) is: A. 5 to 7 days after menses ceases. B. Day 1 of the endometrial cycle. C. mid-menstrual cycle. D. any time during a shower or bath.

ANS: A The physiologic alterations in breast size and activity reach their minimal level about 5 to 7 days after menstruation stops. All women should perform BSE during this phase of the menstrual cycle.

30. Abdominal thrusts (the Heimlich maneuver) are recommended for airway obstruction in children older than: a. 1 year. c. 8 years. b. 4 years. d. 12 years.

ANS: A The Heimlich maneuver is recommended for airway obstruction in children older than 1 year. In children younger than 1 year, back blows and chest thrusts are administered.

2. A woman has just moved to the United States from Mexico. She is 3 months pregnant and has arrived for her first prenatal visit. During her assessment interview, you discover that she has not had any immunizations. Which immunizations should she receive at this point in her pregnancy? (Select all that apply.) A. Tetanus B. Diphtheria C. Chickenpox D. Rubella E. Hepatitis B

ANS: A, B, E Immunization with live or attenuated live viruses is contraindicated during pregnancy because of potential teratogenicity. Vaccines consisting of killed viruses may be used. Immunizations that may be administered during pregnancy include tetanus, diphtheria, recombinant hepatitis B, and rabies vaccines. Live-virus vaccines include those for measles (rubeola and rubella), chickenpox, and mumps.

MULTIPLE RESPONSE 37.The nurse is caring for an infant with a suspected urinary tract infection. Which clinical manifestations would be observed (Select all that apply)? a. Vomiting b. Jaundice c. Failure to gain weight d. Swelling of the face e. Back pain f. Persistent diaper rash

ANS: A, C, F Vomiting, failure to gain weight, and persistent diaper rash are clinical manifestations observed in an infant with a urinary tract infection. Jaundice, swelling of the face, and back pain would not be observed in an infant with a urinary tract infection.

26. A child has liver cancer. The most recent results for the alpha-fetoprotein level show it has been reduced by 50%. Which statement by the nurse to the parents and child is most appropriate at this time? A. "Once the level gets to normal, we can resect the tumor." B. "This shows the cancer is responding to therapy." C. "Unfortunately, the chemotherapy is not working." D. "Your child will need a liver transplant soon."

ANS: B Alpha-fetoprotein (AFP) is a protein produced by both hepatoblastomas and hepatocellular carcinomas. Falling levels of AFP indicate that treatment is working. The other responses are not correct.

41. Which consideration is the most important in managing tuberculosis (TB) in children? a. Skin testing annually c. Adequate nutrition b. Pharmacotherapy d. Adequate hydration

ANS: B Drug therapy for TB includes isoniazid, rifampin, and pyrazinamide daily for 2 months and 2 or 3 times a week for the remaining 4 months. Although skin testing and adequate nutrition and hydration are important, pharmacotherapy is the most important intervention for TB.

12. Which statement best describes fear in school-age children? a. They are increasingly fearful for body safety. b. Most of the new fears that trouble them are related to school and family. c. They should be encouraged to hide their fears to prevent ridicule by peers. d. Those who have numerous fears need continuous protective behavior by parents to eliminate these fears.

ANS: B During the school-age years, children experience a wide variety of fears, but new fears related predominantly to school and family bother children during this time. During the middle-school years, children become less fearful of body safety than they were as preschoolers. Parents and other persons involved with children should discuss their fear with them individually or as a group activity. Sometimes school-age children hide their fears to avoid being teased. Hiding the fears does not end them and may lead to phobias.

31. Which diagnostic test is used to confirm a suspected diagnosis of breast cancer? A. Mammogram B. Ultrasound C. Needle aspiration D. CA 15.3

ANS: C When a suspicious mammogram is noted or a lump is detected, diagnosis is confirmed by needle aspiration, core needle biopsy, or needle localization biopsy. Mammography is a clinical screening tool that may aid early detection of breast cancers. Transillumination, thermography, and ultrasound breast imaging are being explored as methods of detecting early breast carcinoma. CA 15.3 is a serum tumor marker that is used to test for residual disease.

Infants of mothers with diabetes (IDMs) are at higher risk for developing: a. Anemia. b. Hyponatremia. c. Respiratory distress syndrome. d. Sepsis.

ANS: C IDMs are at risk for macrosomia, birth injury, perinatal asphyxia, respiratory distress syndrome, hypoglycemia, hypocalcemia, hypomagnesemia, cardiomyopathy, hyperbilirubinemia, and polycythemia. They are not at risk for anemia, hyponatremia, or sepsis. PTS: 1 DIF: Cognitive Level: Comprehension REF: 682 OBJ: Nursing Process: Planning MSC: Client Needs: Physiologic Integrity

Which is the initial clinical manifestation of generalized seizures? a. Being confused b. Feeling frightened c. Losing consciousness d. Seeing flashing lights

ANS: C Loss of consciousness is a frequent occurrence in generalized seizures and is the initial clinical manifestation. Being confused, feeling frightened, and seeing flashing lights are clinical manifestations of a complex partial seizure.

12. A child diagnosed with hypoparathyroidism is receiving vitamin D therapy. The parents should be advised to watch for which sign of vitamin D toxicity? a. Headache and seizures b. Physical restlessness and voracious appetite without weight gain c. Weakness and lassitude d. Anorexia and insomnia1

ANS: C Vitamin D toxicity can be a serious consequence of therapy. Parents are advised to watch for signs including weakness, fatigue, lassitude, headache, nausea, vomiting, and diarrhea. Renal impairment is manifested through polyuria, polydipsia, and nocturia. Headaches may be a sign of vitamin D toxicity, but seizures are not. Physical restlessness and a voracious appetite with weight loss are manifestations of hyperthyroidism. Anorexia and insomnia are not characteristic of vitamin D toxicity.

The nurse is caring for a neonate with suspected meningitis. Which clinical manifestations should the nurse prepare to assess if meningitis is confirmed? (Select all that apply.) a. Headache b. Photophobia c. Bulging anterior fontanel d. Weak cry e. Poor muscle tone

ANS: C, D, E Assessment findings in a neonate with meningitis include bulging anterior fontanel, weak cry, and poor muscle tone. Headache and photophobia are signs seen in an older child.

18. Nurses should be aware that the induction of labor: a. can be achieved by external and internal version techniques. b. is also known as a trial of labor (TOL). c. is almost always done for medical reasons. d. is rated for viability by a Bishop score.

ANS: D Induction of labor is likely to be more successful with a Bishop score of 9 or higher for first-time mothers and 5 or higher for veterans. Version is turning of the fetus to a better position by a physician for an easier or safer birth. A trial of labor is the observance of a woman and her fetus for several hours of active labor to assess the safety of vaginal birth. Two thirds of cases of induced labor are elective and are not done for medical reasons.

4. The hormone responsible for maturation of mammary gland tissue is: a. estrogen. b. testosterone. c. prolactin. d. progesterone.

ANS: D Progesterone causes maturation of the mammary gland tissue, specifically acinar structures of the lobules. Estrogen increases the vascularity of the breast tissue. Testosterone has no bearing on breast development. Prolactin is produced after birth and released from the pituitary gland. It is produced in response to infant suckling and emptying of the breasts.

Because of their striving for independence and productivity, which age-group of children is particularly vulnerable to events that may lessen their feeling of control and power? a. Infants b. Toddlers c. Preschoolers d. School-age children

ANS: D When a child is hospitalized, the altered family role, physical disability, loss of peer acceptance, lack of productivity, and inability to cope with stress usurp individual power and identity. This is especially detrimental to school-age children, who are striving for independence and productivity and are now experiencing events that lessen their control and power. Infants, toddlers, and preschoolers, although affected by loss of power, are not as significantly affected as are school-age children.

17. The nurse is caring for a child with acute renal failure. What clinical manifestation should he or she recognize as a sign of hyperkalemia? a. Dyspnea b. Seizure c. Oliguria d. Cardiac arrhythmia

ANS: D Hyperkalemia is the most common threat to the life of the child. Signs of hyperkalemia include electrocardiographic anomalies such as prolonged QRS complex, depressed ST segments, peaked T waves, bradycardia, or heart block. Dyspnea, seizure, and oliguria are not manifestations of hyperkalemia.

19. Which comment is most developmentally typical of a 7-year-old boy? a. "I am a Power Ranger, so don't make me angry." b. "I don't know whether I like Mary or Joan better." c. "My mom is my favorite person in the world." d. "Jimmy is my best friend."

ANS: D School-age children form friendships with peers of the same sex, those who live nearby, and other children who have toys that they enjoy sharing. Magical thinking is developmentally appropriate for the preschooler. Opposite-sex friendships are not typical for the 7-year-old child. Seven-year-old children socialize with their peers, not their parents.

10. A child has been diagnosed with chronic myelogenous leukemia (CML). Which statement by the nurse to the parents is most appropriate? A. "Radiation therapy is the standard treatment." B. "The prognosis for this disease is extremely poor." C. "There are lots of good medications for nausea." D. "We need to test siblings for a bone marrow match."

ANS: D The preferred treatment for CML is a bone marrow or stem cell transplant from a matching sibling, which can be curative in up to 80% of patients. Radiation therapy is not used. Although there are many good medications for nausea, this statement is not the best choice, because it is not specific to this child's condition.

What is descriptive of the preschooler's understanding of time? a. Has no understanding of time b. Associates time with events c. Can tell time on a clock d. Uses terms like "yesterday" appropriately

B In a preschooler's understanding, time has a relation with events such as, "We'll go outside after lunch." Preschoolers develop an abstract sense of time at age 3 years. Children can tell time on a clock at age 7 years. Children do not fully understand use of time-oriented words until age 6 years.

A normal characteristic of the language development of a preschool-age child is: a. Lisp. c. Echolalia. b. Stammering. d. Repetition without meaning.

B Stammering and stuttering are normal dysfluencies in preschool-age children. Lisps are not a normal characteristic of language development. Echolalia and repetition are traits of toddlers' language.

A nurse is caring for an adolescent hospitalized for cellulitis. The nurse notes that the adolescent experiences many "mood swings" throughout the day. The nurse interprets this behavior as: a. Requiring a referral to a mental health counselor. b. Requiring some further lab testing. c. Normal behavior. d. Related to feelings of depression.

C Adolescents vacillate in their emotional states between considerable maturity and childlike behavior. One minute they are exuberant and enthusiastic; the next minute they are depressed and withdrawn. Because of these mood swings, adolescents are frequently labeled as unstable, inconsistent, and unpredictable, but the behavior is normal. The behavior would not require a referral to a mental health counselor or further lab testing. The mood swings do not indicate depression.

6. A pregnant woman reports that she is still playing tennis at 32 weeks of gestation. The nurse would be most concerned that during and after tennis matches this woman consumes: a. Several glasses of fluid. b. Extra protein sources such as peanut butter. c. Salty foods to replace lost sodium. d. Easily digested sources of carbohydrate.

a. Several glasses of fluid.

The school nurse is caring for a child with a penetrating eye injury. Emergency treatment includes: a. Applying a regular eye patch. b. Applying a Fox shield to the affected eye and any type of patch to the other eye. c. Applying ice until the physician is seen. d. Irrigating the eye copiously with a sterile saline solution.

b. Applying a Fox shield to the affected eye and any type of patch to the other eye.

25. The labor and delivery nurse is preparing a bariatric patient for an elective cesarean birth. Which piece of "specialized" equipment is unnecessary when providing care for this pregnant woman. a. Extra long surgical instruments b. Wide surgical table c. Temporal thermometer d. Increased diameter blood pressure cuff

c. Temporal thermometer

8. A pregnant woman's diet history indicates that she likes the following list of foods. The nurse would encourage this woman to consume more of which food to increase her calcium intake? a. Fresh apricots b. Canned clams c. Spaghetti with meat sauce d. Canned sardines

d. Canned sardines

21. While taking a diet history, the nurse might be told that the expectant mother has cravings for ice chips, cornstarch, and baking soda. This represents a nutritional problem known as: a. preeclampsia. b. pyrosis. c. pica. d. purging.

ANS: C The consumption of foods low in nutritional value or of nonfood substances (e.g., dirt, laundry starch) is called pica.

22. The most common neurologic disorder accompanying pregnancy is: A. eclampsia. B. Bell's palsy. C. epilepsy. D. multiple sclerosis.

ANS: C The effects of pregnancy on epilepsy are unpredictable. Eclampsia sometimes may be confused with epilepsy, which is the most common neurologic disorder accompanying pregnancy. Bell's palsy is a form of facial paralysis. Multiple sclerosis is a patchy demyelinization of the spinal cord that does not affect the normal course of pregnancy or birth.

The nurse is reviewing Erikson's theory about the autonomy versus shame and doubt stage. The nurse is trying to correlate it to Freud's psychosexual theory. Which stage would the nurse review in Freud's theory? 1 Oral 2 Anal 3 Phallic 4 Latency

2 When the nurse is reviewing the autonomy versus shame and doubt stage in Erikson theory, it refers to a toddler. The corresponding level in Freud's theory for the toddler's psychosexual developmental stage is the anal stage, when the toddler is toilet trained. The oral stage in Freud's theory represents infancy, from birth to 1 year, and is the trust versus mistrust stage in Erikson's theory. The phallic stage in Freud's theory represents early childhood, 3-6 years of age, or initiative versus guilt in Erikson's theory. Latency in Freud's theory represents middle childhood, 6-12 years, or industry versus inferiority in Erikson's theory.

A child with a temporary visual impairment was admitted to the hospital for treatment. What nursing intervention would make the child feel most comfortable in the hospital? The nurse: 1 Explains the different departments of the hospital. 2 Understands the child's behavior and daily routine. 3 Describes the surroundings of the room and the unit. 4 Asks the cleaner to move the furnishings around

3 The nurse helps the child become familiar with the room so that the child knows the layout in order to avoid injury while moving around the room. Explaining to the child about the hospital departments is not necessary to make the child feel comfortable. Understanding the child's daily routine is necessary to plan activities for the child but does not increase comfort. The cleaning personnel are asked to maintain the décor of the room to avoid accidents; therefore changes should be avoided.

9. A 7-year-old child presents to the emergency department, where the parent reports a 3-week history of pale skin, extreme fatigue, and dizziness. Which laboratory value would the nurse correlate with the patient's current condition? A. Hematocrit: 33% B. Hemoglobin: 13.2 g/dL C. Red blood cell count: 2.8/mm3 D. White blood cell count: 12.3/mm3

ANS: C For a child of this age, a normal RBC count is 4-5.2/mm3. Low RBCs can lead to pallor, fatigue, headaches, and dizziness, as tissues are not being oxygenated. The other laboratory values are normal.

7. In their role of implementing a plan of care for infertile couples, nurses should: A. be comfortable with their sexuality and nonjudgmental about others to counsel their patients effectively. B. know about such nonmedical remedies as diet, exercise, and stress management. C. be able to direct patients to sources of information about what herbs to take that might help and which ones to avoid. D. do all of the above plus be knowledgeable about potential drug and surgical remedies.

ANS: D Nurses should be open to and ready to help with a variety of medical and nonmedical approaches.

The nurse caring for the postpartum woman understands that breast engorgement is caused by: a. Overproduction of colostrum. b. Accumulation of milk in the lactiferous ducts and glands. c. Hyperplasia of mammary tissue. d. Congestion of veins and lymphatics.

ANS: D Breast engorgement is caused by the temporary congestion of veins and lymphatics. Breast engorgement is not the result of overproduction of colostrum. Accumulation of milk in the lactiferous ducts and glands does not cause breast engorgement. Hyperplasia of mammary tissue does not cause breast engorgement. PTS: 1 DIF: Cognitive Level: Knowledge REF: 502 OBJ: Nursing Process: Assessment MSC: Client Needs: Health Promotion and Maintenance

27.Which intervention is appropriate when examining a male infant for cryptorchidism? a. Cooling the examiner's hands b. Taking a rectal temperature c. Eliciting the cremasteric reflex d. Warming the room

ANS: D For the infant's comfort, the infant should be examined in a warm room with the examiner's hands warmed. Testes can retract into the inguinal canal if the infant is upset or cold. Examining the infant with cold hands is uncomfortable for the infant and likely to cause the infant's testes to retract into the inguinal canal. It may also cause the infant to be uncooperative during the examination. A rectal temperature yields no information about cryptorchidism. Testes can retract into the inguinal canal if the cremasteric reflex is elicited. This can lead to an incorrect diagnosis.

5. The parent of an infant with nasopharyngitis should be instructed to notify the health care professional if the infant: a. Becomes fussy. c. Has a fever over 99 F. b. Has a cough. d. Shows signs of an earache.

ANS: D If an infant with nasopharyngitis has a fever over 101 F, there is early evidence of respiratory complications. Irritability and a slight fever are common in an infant with a viral illness. Cough can be a sign of nasopharyngitis

34. Which information should the nurse teach workers at a day care center about respiratory syncytial virus (RSV)? a. RSV is transmitted through particles in the air. b. RSV can live on skin or paper for up to a few seconds after contact. c. RSV can survive on nonporous surfaces for about 60 minutes. d. Frequent hand washing can decrease the spread of the virus

ANS: D Meticulous hand washing can decrease the spread of organisms. RSV infection is not airborne. It is acquired mainly through contact with contaminated surfaces. RSV can live on skin or paper for up to 1 hour and on cribs and other nonporous surfaces for up to 6 hours

10. The nurse closely monitors the temperature of a child with nephrosis. The purpose of this is to detect an early sign of: a. Infection. b. Hypertension. c. Encephalopathy. d. Edema.

ANS: A Infection is a constant source of danger to edematous children and those receiving corticosteroid therapy. An increased temperature could be an indication of an infection, but it is not an indication of hypertension or edema. Encephalopathy is not a complication usually associated with nephrosis. The child will most likely have neurologic signs and symptoms.

A nurse is instructing a nursing assistant on techniques to facilitate lipreading with a hearing-impaired child who lip-reads. Which techniques should the nurse include (select all that apply)? a. Speak at eye level. b. Stand at a distance from the child. c. Speak words in a loud tone. d. Use facial expressions while speaking. e. Keep sentences short.

A, D, E

26. Parents of a 3-year-old child with congenital heart disease are afraid to let their child play with other children because of possible overexertion. The nurse's reply should be based on which of the following? 1. Child needs opportunities to play with peers. 2. Child needs to understand that peers' activities are too strenuous. 3. Parents can meet all the child's needs. 4. Constant parental supervision is needed to avoid overexertion.

ANS: 1 1. The child needs opportunities for social development. Children usually limit their activities if allowed to set their own pace.

37. Therapeutic management of the child with rheumatic fever includes: 1. Administration of penicillin. 2. Avoid salicylates (aspirin). 3. Strict bed rest for 4 to 6 weeks. 4. Administration of corticosteroids if chorea develops.

ANS: 1 1. The goal of medical management is the eradication of the hemolytic streptococci. Penicillin is the drug of choice

19. Which of the following is a common sign of digoxin toxicity? 1. Seizures 2. Vomiting 3. Bradypnea 4. Tachycardia

ANS: 2 2. Vomiting is a common sign of digoxin toxicity

1. A chest radiograph film is ordered for a child with suspected cardiac problems. The child's parent asks the nurse, "What will the radiograph show about the heart?" The nurse's response should be based on knowledge that the x-ray film will do which of the following? 1. Show bones of chest but not the heart 2. Measure electrical potential generated from heart muscle 3. Provide permanent record of heart size and configuration 4. Provide computerized image of heart vessels and tissues

ANS: 3 3. A chest radiograph will provide information on the heart size and pulmonary blood-flow patterns. It will be provide a baseline for future comparisons

30. The nurse is caring for a child after heart surgery. Which of the following should the nurse do if evidence is found of cardiac tamponade? 1. Increase analgesia. 2. Apply warming blankets. 3. Immediately report this to physician. 4. Encourage child to cough, turn, and breathe deeply.

ANS: 3 3. If evidence is noted of cardiac tamponade, blood or fluid in the pericardial space constricting the heart, the physician is notified immediately of this life-threatening complication

29. Which of the following is an important nursing consideration when suctioning a young child who has had heart surgery? 1. Perform suctioning at least every hour. 2. Suction for no longer than 30 seconds at a time. 3. Administer supplemental oxygen before and after suctioning. 4. Expect symptoms of respiratory distress when suctioning.

ANS: 3 3. If suctioning is indicated, supplemental oxygen is administered with a manual resuscitation bag before and after the procedure to prevent hypoxia

The nurse suspects tissue injury in an infant on intravenous therapy. What parameters will the nurse assess to determine tissue injury? Select all that apply. 1 The amount of redness 2 Blanching 3 The amount of swelling 4 Quality of pulses above infiltration 5 Coolness of the area

1, 2, 3, 5 The nurse adheres to certain guidelines available for determining the severity of tissue injury. Staging characteristics, such as the amount of redness, blanching, the amount of swelling, pain, capillary refill, and warmth or coolness of the area, are used to determine severity. The quality of pulses below infiltration is assessed and not above it.

The nurse is assessing a child with autism. What characteristic features of autism does the nurse expect to find in the child? Select all that apply. 1 Verbal impairment 2 Stereotyped behavior patterns 3 Hearing and visual impairment 4 Nonrepetitive behavioral patterns 5 Decreased involvement in play

1, 2, 5 Children with autism usually have verbal impairment caused by poor language development. Autistic children exhibit stereotyped behavioral patterns caused by impaired neuromuscular function. Such children show decreased interest in functional play activities. Autistic children do not usually have hearing and visual impairment. Autistic children exhibit repetitive behavioral patterns.

3. The nurse is assessing the knowledge of new parents with a child born with maple syrup urine disease (MSUD). This is an autosomal recessive inherited disorder, which means that: A. both genes of a pair must be abnormal for the disorder to be expressed. B. only one copy of the abnormal gene is required for the disorder to be expressed. C. the disorder occurs in males and heterozygous females. D. the disorder is carried on the X chromosome.

ANS: A MSUD is a type of autosomal recessive inheritance disorder in which both genes of a pair must be abnormal for the disorder to be expressed. MSUD is not an X-linked dominant or recessive disorder or an autosomal dominant inheritance disorder.

2. Prenatal testing for human immunodeficiency virus (HIV) is recommended for: A. all women, regardless of risk factors. B. a woman who has had more than one sexual partner. C. a woman who has had a sexually transmitted infection. D. a woman who is monogamous with her partner.

ANS: A Testing for the antibody to HIV is strongly recommended for all pregnant women. A HIV test is recommended for all women, regardless of risk factors. Women who test positive for HIV can be treated, reducing the risk of transmission to the fetus.

19. A child has a chronic, nonproductive cough and diffuse wheezing during the expiratory phase of respiration. This suggests: a. Asthma. c. Bronchiolitis. b. Pneumonia. d. Foreign body in the trachea

ANS: A Children with asthma usually have these chronic symptoms. Pneumonia appears with an acute onset and fever and general malaise. Bronchiolitis is an acute condition caused by respiratory syncytial virus. Foreign body in the trachea will manifest with acute respiratory distress or failure and maybe stridor

12. Women with an inadequate weight gain during pregnancy are at higher risk of giving birth to an infant with: A. spina bifida. B. intrauterine growth restriction. C. diabetes mellitus. D. Down syndrome.

ANS: B Both normal-weight and underweight women with inadequate weight gain have an increased risk of giving birth to an infant with intrauterine growth restriction. Spina bifida, diabetes mellitus, and Down syndrome are not associated with inadequate maternal weight gain.

17. The least common cause of long, difficult, or abnormal labor (dystocia) is: a. midplane contracture of the pelvis. b. compromised bearing-down efforts as a result of pain medication. c. disproportion of the pelvis. d. low-lying placenta.

ANS: C The least common cause of dystocia is disproportion of the pelvis.

3. A pregnant woman's diet consists almost entirely of whole grain breads and cereals, fruits, and vegetables. The nurse would be most concerned about this woman's intake of: A. calcium. B. protein. C. vitamin B12. D. folic acid.

ANS: C This diet is consistent with that followed by a strict vegetarian (vegan). Vegans consume only plant products. Because vitamin B12 is found in foods of animal origin, this diet is deficient in vitamin B12.

The school nurse tells adolescents in the clinic that confidentiality and privacy will be maintained unless a life-threatening situation arises. This practice is: a. Not appropriate in a school setting. b. Never appropriate because adolescents are minors. c. Important in establishing trusting relationships. d. Suggestive that the nurse is meeting his or her own needs.

C Health professionals who work with adolescents should consider the adolescents' increasing independence and responsibility while maintaining privacy and ensuring confidentiality. However, circumstances may occur in which they are not able to maintain confidentiality, such as self-destructive behavior or maltreatment by others. Confidentiality and privacy are necessary to facilitate trust with this age group. The nurse must be aware of the limits placed on confidentiality by local jurisdiction.

The first stage of labor is often a time of introspection. In light of this, which of the following would guide your planning of nursing care? a) A woman should be left entirely alone during this period. b) A woman may spend time thinking about what is happening to her. c) No nursing care is needed to be done during this time. d) A woman will rarely speak or laugh during this period.

B (A woman may spend time thinking about what is happening to her. Rationale: Women need a support person with them during all stages of labor.)

All statements about normal labor are true except: a) A single fetus presents by vertex b) It is completed within 8 hours c) A regular progression of contractions, effacement, dilation, and descent occurs d) No complications are involved

B (It is completed within 8 hours)

The nurse is caring for an adolescent brought to the hospital with acute drug toxicity. Cocaine is believed to be the drug involved. Data collection should include the: a. Mode of administration. b. Actual content of the drug. c. Function the drug plays in the adolescent's life. d. Adolescent's level of interest in rehabilitation.

A When the drug is questionable or unknown, every effort must be made to determine the type, amount of drug taken, the mode and time of administration, and factors relating to the onset of presenting symptoms. Because the actual content of most street drugs is highly questionable, this information would be difficult to obtain. It is helpful to know the pattern of use but not essential during this emergency. This is an inappropriate time for an evaluation about the level of interest in rehabilitation.

In planning for the care of a 30-year-old woman with pregestational diabetes, the nurse recognizes that the most important factor affecting pregnancy outcome is the: A. mother's age. B. number of years since diabetes was diagnosed. C. amount of insulin required prenatally. D. degree of glycemic control during pregnancy.

D. degree of glycemic control during pregnancy. Although advanced maternal age may pose some health risks, for the woman with pregestational diabetes the most important factor remains the degree of glycemic control during pregnancy. The number of years since diagnosis is not as relevant to outcomes as the degree of glycemic control. The key to reducing risk in the pregestational diabetic woman is not the amount of insulin required but rather the level of glycemic control. Women with excellent glucose control and no blood vessel disease should have good pregnancy outcomes.

11. Which of the following structural defects constitute tetralogy of Fallot? 1. Pulmonary stenosis, ventricular septal defect, overriding aorta, right ventricular hypertrophy 2. Aortic stenosis, ventricular septal defect, overriding aorta, right ventricular hypertrophy 3. Aortic stenosis, atrial septal defect, overriding aorta, left ventricular hypertrophy 4. Pulmonary stenosis, ventricular septal defect, aortic hypertrophy, left ventricular hypertrophy

ANS: 1 1. Tetralogy of Fallot has these four characteristics: pulmonary stenosis, ventricular septal defect, overriding aorta, and right ventricular hypertrophy.

3. José is a 4-year-old child scheduled for a cardiac catheterization. Preoperative teaching should be: 1. Directed at his parents, because he is too young to understand. 2. Detailed in regard to the actual procedures so he will know what to expect. 3. Done several days before the procedure so that he will be prepared. 4. Adapted to his level of development so that he can understand.

ANS: 4 4. Preoperative teaching should always be directed at the child's stage of development. The caregivers also benefit from the same explanations.

9. Surgical closure of the ductus arteriosus would do which of the following? 1. Stop the loss of unoxygenated blood to the systemic circulation 2. Decrease the edema in legs and feet 3. Increase the oxygenation of blood 4. Prevent the return of oxygenated blood to the lungs

ANS: 4 4. The ductus arteriosus allows blood to flow from the higher-pressure aorta to the lower-pressure pulmonary artery, causing a right-to-left shunt. If this is surgically closed, no additional oxygenated blood (from the aorta) will return to the lungs through the pulmonary artery

7. Which statement made by a lactating woman would lead the nurse to believe that the woman might have lactose intolerance? a. "I always have heartburn after I drink milk." b. "If I drink more than a cup of milk, I usually have abdominal cramps and bloating." c. "Drinking milk usually makes me break out in hives." d. "Sometimes I notice that I have bad breath after I drink a cup of milk."

b. "If I drink more than a cup of milk, I usually have abdominal cramps and bloating."

2. Which meal would provide the most absorbable iron? a. Toasted cheese sandwich, celery sticks, tomato slices, and a grape drink b. Oatmeal, whole wheat toast, jelly, and low-fat milk c. Black bean soup, wheat crackers, orange sections, and prunes d. Red beans and rice, cornbread, mixed greens, and decaffeinated tea

c. Black bean soup, wheat crackers, orange sections, and prunes

What represents the major stressor of hospitalization for children from middle infancy throughout the preschool years? a. Separation anxiety b. Loss of control c. Fear of bodily injury d. Fear of pain

ANS: A The major stress for children from infancy through the preschool years is separation anxiety, also called anaclitic depression. This is a major stressor of hospitalization. Loss of control, fear of bodily injury, and fear of pain are all stressors associated with hospitalization. However, separation from family is a primary stressor in this age-group.

11. A 9 year old often comes to the school nurse complaining of stomach pains. The teacher says that the child has lately been somewhat aggressive and stubborn in the classroom. What should the school nurse recognize as the possible trigger for these behaviors? a. Signs of stress b. Developmental delay c. A physical problem causing emotional stress d. Lack of adjustment to the school environment

ANS: A Signs of stress include stomach pains or headache, sleep problems, bed-wetting, changes in eating habits, aggressive or stubborn behavior, reluctance to participate, or regression to early behaviors. This child is exhibiting signs of stress, not developmental delay, a physical problem, or lack of adjustment.

The nurse caring for the newborn should be aware that the sensory system least mature at the time of birth is: a. Vision. b. Hearing. c. Smell. d. Taste.

ANS: A The visual system continues to develop for the first 6 months. As soon as the amniotic fluid drains from the ear (minutes), the infants hearing is similar to that of an adult. Newborns have a highly developed sense of smell. The newborn can distinguish and react to various tastes. PTS: 1 DIF: Cognitive Level: Knowledge REF: 579 OBJ: Nursing Process: Planning MSC: Client Needs: Health Promotion and Maintenance

7. A child is being admitted with an infratentorial brain tumor. Which anatomical regions of the brain does the nurse know this tumor might include? (Select all that apply.) A. Brainstem B. Cerebellum C. Cerebrum D. Frontal lobe E. Parietal lobe

ANS: A, B Brain tumors in children are classified as either supratentorial or infratentorial. Infratentorial tumors are located in the posterior third of the brain, below the tentorium, and involve the brainstem and cerebellum. The cerebrum, frontal, and parietal lobes are located in the supratentorial section of the brain, which is the anterior two-thirds of the brain structure.

1. Signs and symptoms that a woman should report immediately to her health care provider include: (Select all that apply.) A. vaginal bleeding. B. rupture of membranes. C. heartburn accompanied by severe headache. D. decreased libido. E. Urinary frequency.

ANS: A, B, C Vaginal bleeding, rupture of membranes, and severe headaches all are signs of potential complications in pregnancy. Patients should be advised to report these signs to the health care provider. Decreased libido and urinary frequency are common discomforts of pregnancy that do not require immediate health care interventions.

9. The nurse is teaching parents about the importance of good nutrition for their child who has cancer. Which components does the nurse include as important for this child's diet? (Select all that apply.) A. High calories B. High carbohydrates C. High vitamins D. Low minerals E. Low protein

ANS: A, C The child with cancer needs optimal nutrition, including a diet high in calories, fatty acids, vitamins, protein, and minerals.

1. The diagnosis of pregnancy is based on which positive signs of pregnancy? (Select all that apply.) A. Identification of fetal heartbeat B. Palpation of fetal outline C. Visualization of the fetus D. Verification of fetal movement E. Positive hCG test

ANS: A, C, D Identification of fetal heartbeat, visualization of the fetus, and verification of fetal movement are all positive, objective signs of pregnancy. Palpation of fetal outline and a positive hCG test are probable signs of pregnancy. A tumor also can be palpated. Medication and tumors may lead to false-positive results on pregnancy tests.

39.A school-age child is admitted to the hospital with acute glomerulonephritis and oliguria. Which dietary menu items should be allowed for this child (Select all that apply)? a. Apples b. Bananas c. Cheese d. Carrot sticks e. Strawberries

ANS: A, D, E Moderate sodium restriction and even fluid restriction may be instituted for children with acute glomerulonephritis. Foods with substantial amounts of potassium and sodium are generally restricted during the period of oliguria. Apples, carrot sticks, and strawberries would be items low in sodium and allowed. Bananas are high in potassium and cheese is high in sodium. Those items would be restricted.

31. Concerning the third stage of labor, nurses should be aware that: a. the placenta eventually detaches itself from a flaccid uterus. b. an expectant or active approach to managing this stage of labor reduces the risk of complications. c. it is important that the dark, roughened maternal surface of the placenta appear before the shiny fetal surface. d. the major risk for women during the third stage is a rapid heart rate.

ANS: B Active management facilitates placental separation and expulsion, thus reducing the risk of complications. The placenta cannot detach itself from a flaccid (relaxed) uterus. Which surface of the placenta comes out first is not clinically important. The major risk for women during the third stage of labor is after birth hemorrhage.

13. A 3-year-old girl's mother is 6 months pregnant. What concern is this child likely to verbalize? A. How the baby will "get out"? B. What the baby will eat? C. Whether her mother will die? D. What color eyes the baby has?

ANS: B By age 3 or 4, children like to be told the story of their own beginning and accept its comparison with the present pregnancy. They like to listen to the fetal heartbeat and feel the baby move. Sometimes they worry about how the baby is being fed and what it wears. School-age children take a more clinical interest in their mother's pregnancy and may want to know, "How did the baby get in there?" and "How will it get out?" Whether her mother will die does not tend to be the focus of a child's questions about the impending birth of a sibling. The baby's eye color does not tend to be the focus of children's questions about the impending birth of a sibling.

4. In presenting to obstetric nurses interested in genetics, the genetic nurse identifies the primary risk(s) associated with genetic testing as: A. anxiety and altered family relationships. B. denial of insurance benefits. C. high false-positive results associated with genetic testing. D. ethnic and socioeconomic disparity associated with genetic testing.

ANS: B Decisions about genetic testing are shaped by socioeconomic status and the ability to pay for the testing. Some types of genetic testing are expensive and are not covered by insurance benefits. Anxiety and altered family relationships, high false-positive results, and ethnic and socioeconomic disparity are factors that may be difficulties associated with genetic testing, but they are not risks associated with testing.

What is a common postoperative complication of anesthesia? 1 Respiratory tract infections 2 Cardiac arrest 3 Infection of the joints 4 Resistance to anesthetic agents

4

B (Giving a bottle of milk or juice at naptime or bedtime predisposes the child to this syndrome.)

Which statement about early childhood caries (ECC) is correct? A. The syndrome is distinguished by protruding upper front teeth, resulting from sucking on a hard nipple. B. Giving a bottle of milk or juice at naptime or bedtime predisposes the child to this syndrome. C. The syndrome can be prevented by breastfeeding. D. Giving the child juice in the bottle instead of milk at bedtime prevents this syndrome.

A young child who has an intelligence quotient (IQ) of 45 would be described as: a. Within the lower limits of the range of normal intelligence. b. Mildly cognitively impaired but educable. c. Moderately cognitively impaired but trainable. d. Severely cognitively impaired and completely dependent on others for care.

c. Moderately cognitively impaired but trainable.

The factors that affect the process of labor and birth, known commonly as the five Ps, include all except: a. Passenger. b. Powers. c. Passageway. d. Pressure.

D (The five Ps are passenger (fetus and placenta), passageway (birth canal), powers (contractions), position of the mother, and psychologic response.)

A new mother asks the nurse when the "soft spot" on her son's head will go away. The nurse's answer is based on the knowledge that the anterior fontanel closes after birth by _____ months. a. 2 b. 12 c. 8 d. 18

D (The larger of the two fontanels, the anterior fontanel, closes by 18 months after birth.)

During the second stage of labor, a woman is generally a) very aware of activities immediately around her. b) anxious to have people around her. c) no longer in need of a support person. d) turning inward to concentrate on body sensations.

D (Turning inward to concentrate on body sensations. Rationale: Second-stage contractions are so unusual that most women are unable to think of things other than what is happening inside their body.)

In girls, the initial indication of puberty is: a. Menarche. c. Growth of pubic hair. b. Growth spurt. d. Breast development.

D In most girls, the initial indication of puberty is the appearance of breast buds, an event known as the larche. The usual sequence of secondary sexual characteristic development in girls is breast changes, rapid increase in height and weight, growth of pubic hair, appearance of axillary hair, menstruation, and abrupt deceleration of linear growth.

Which common childhood communicable disease may cause severe defects in the fetus when it occurs in its congenital form? a. Erythema infectiosum c. Rubeola b. Roseola d. Rubella

D Rubella causes teratogenic effects on the fetus. There is a low risk of fetal death to those in contact with children affected with fifth disease. Roseola and rubeola are not dangerous to the fetus.

B (take a thorough, detailed history of usual daily events.)

The exhausted parents of a 2-month-old infant with colic ask the nurse what is the best method to promote comfort and sleep for the infant. The nurse's initial action is to: A. advise the mother to follow a milk-free diet for 3 to 5 days. B. take a thorough, detailed history of usual daily events. C. administer simethicone drops to provide relief from gas pains. D. explain that the parents need to stay calm so the infant will remain calm.

C (Inability to conserve)

The nurse notices that a toddler is more cooperative when taking medicine from a small cup rather than from a large cup. This is an example of which characteristic of preoperational thought? A. Egocentrism B. Irreversibility C. Inability to conserve D. Transductive reasoning

Which is the most significant factor in distinguishing those who commit suicide from those who make suicidal attempts or threats? a. Social isolation c. Degree of depression b. Level of stress d. Desire to punish others

A Social isolation is a significant factor in distinguishing adolescents who will kill themselves from those who will not. It is also more characteristic of those who complete suicide than of those who make attempts or threats. Level of stress, degree of depression, and desire to punish others are contributing factors in suicide, but they are not the most significant factor in distinguishing those who complete suicide from those who attempt suicide.

When using intermittent auscultation (IA) to assess uterine activity, the nurse should be cognizant that: a. The examiner's hand should be placed over the fundus before, during, and after contractions. b. The frequency and duration of contractions is measured in seconds for consistency. c. Contraction intensity is given a judgment number of 1 to 7 by the nurse and client together. d. The resting tone between contractions is described as either placid or turbulent.

A The assessment is done by palpation; duration, frequency, intensity, and resting tone must be assessed. The duration of contractions is measured in seconds; the frequency is measured in minutes. The intensity of contractions usually is described as mild, moderate, or strong. The resting tone usually is characterized as soft or relaxed.

A 14-year-old boy and his parents are concerned about bilateral breast enlargement. The nurse's discussion of this should be based on knowing that: a. This is usually benign and temporary. b. This is usually caused by Klinefelter's syndrome. c. Administration of estrogen effectively reduces gynecomastia. d. Administration of testosterone effectively reduces gynecomastia.

A The male breast responds to hormone changes. Some degree of bilateral or unilateral breast enlargement occurs frequently in boys during puberty. This is not a manifestation of Klinefelter's syndrome. Administration of estrogen or testosterone will have no effect on the reduction of breast tissue and may aggravate the condition.

C (explain to the parent that this is not harmful.)

A 2-year-old child has recently started having temper tantrums during which she holds her breath and sometimes faints. The nurse should: A. refer the child for respiratory evaluation. B. refer the child for psychologic evaluation. C. explain to the parent that this is not harmful. D. explain to the parent that the child is spoiled.

Signs that precede labor include (Select all that apply): a. Lightening. b. Exhaustion. c. Bloody show. d. Rupture of membranes. e. Decreased fetal movement.

A, C, D (Signs that precede labor may include lightening, urinary frequency, backache, weight loss, surge of energy, bloody show, and rupture of membranes. Many women experience a burst of energy before labor. A decrease in fetal movement is an ominous sign that does not always correlate with labor.)

A collection of blood between the skull bone and its periosteum is known as a cephalhematoma. To reassure the new parents whose infant develops such a soft bulge, it is important that the nurse be aware that this condition: a. May occur with spontaneous vaginal birth. b. Happens only as the result of a forceps or vacuum delivery. c. Is present immediately after birth. d. Will gradually absorb over the first few months of life.

ANS: A Bleeding may occur during a spontaneous vaginal delivery as a result of the pressure against the maternal bony pelvis. The soft, irreducible fullness does not pulsate or bulge when the infant cries. Low forceps and other difficult extractions may result in bleeding. However, cephalhematomas can also occur spontaneously. The swelling may appear unilaterally or bilaterally and is usually minimal or absent at birth. It increases over the first 2 to 3 days of life. Cephalhematomas disappear gradually over 2 to 3 weeks. A less common condition results in calcification of the hematoma, which may persist for months. PTS: 1 DIF: Cognitive Level: Knowledge REF: 571 OBJ: Nursing Process: Planning MSC: Client Needs: Health Promotion and Maintenance

Which drug should the nurse expect to administer to a preschool child who has increased intracranial pressure (ICP) resulting from cerebral edema? a. Mannitol (Osmitrol) b. Epinephrine hydrochloride (Adrenalin) c. Atropine sulfate (Atropine) d. Sodium bicarbonate (Sodium bicarbonate)

ANS: A For increased ICP, mannitol, an osmotic diuretic, administered intravenously, is the drug used most frequently for rapid reduction. Epinephrine hydrochloride, atropine sulfate, and sodium bicarbonate are not used to decrease ICP.

In many hospitals new mothers are routinely presented with gift bags containing samples of infant formula. This practice: a. Is inconsistent with the Baby Friendly Hospital Initiative. b. Promotes longer periods of breastfeeding. c. Is perceived as supportive to both bottle-feeding and breastfeeding mothers. d. Is associated with earlier cessation of breastfeeding.

ANS: A Infant formula should not be given to mothers who are breastfeeding. Such gifts are associated with earlier cessation of breastfeeding. Baby-Friendly USA prohibits the distribution of any gift bags or formula to new mothers. PTS: 1 DIF: Cognitive Level: Comprehension REF: 502 OBJ: Nursing Process: Assessment MSC: Client Needs: Safe and Effective Care Environment

An adolescent boy is brought to the emergency department after a motorcycle accident. His respirations are deep, periodic, and gasping. There are extreme fluctuations in blood pressure. Pupils are dilated and fixed. The nurse should suspect which type of head injury? a. Brainstem b. Skull fracture c. Subdural hemorrhage d. Epidural hemorrhage

ANS: A Signs of brainstem injury include deep, rapid, periodic or intermittent, and gasping respirations. Wide fluctuations or noticeable slowing of the pulse, widening pulse pressure, or extreme fluctuations in blood pressure are consistent with a brainstem injury. Skull fracture, subdural hemorrhage, and epidural hemorrhage are not consistent with brainstem injuries.

A child is upset because, when the cast is removed from her leg, the skin surface is caked with desquamated skin and sebaceous secretions. Which should the nurse suggest to remove this material? a. Soak in a bathtub. b. Vigorously scrub leg. c. Apply powder to absorb material. d. Carefully pick material off leg.

ANS: A Simple soaking in the bathtub is usually sufficient for the removal of the desquamated skin and sebaceous secretions. It may take several days to eliminate the accumulation completely. The parents and child should be advised not to scrub the leg vigorously or forcibly remove this material because it may cause excoriation and bleeding. Oil or lotion, but not powder, may provide comfort for the child.

A careful review of the literature on the various recreational and illicit drugs reveals that: a. More longer-term studies are needed to assess the lasting effects on infants when mothers have taken or are taking illegal drugs. b. Heroin and methadone cross the placenta; marijuana, cocaine, and phencyclidine (PCP) do not. c. Mothers should discontinue heroin use (detox) any time they can during pregnancy. d. Methadone withdrawal for infants is less severe and shorter than heroin withdrawal.

ANS: A Studies on the effects of marijuana and cocaine use by mothers are somewhat contradictory. More long-range studies are needed. Just about all these drugs cross the placenta, including marijuana, cocaine, and PCP. Drug withdrawal is accompanied by fetal withdrawal, which can lead to fetal death. Therefore, detoxification from heroin is not recommended, particularly later in pregnancy. Methadone withdrawal is more severe and more prolonged than heroin withdrawal. PTS: 1 DIF: Cognitive Level: Analysis REF: 679 OBJ: Nursing Process: Evaluation MSC: Client Needs: Physiologic Integrity

The nurse is using the Ballard scale to determine the gestational age of a newborn. Which assessment finding is consistent with a gestational age of 40 weeks? a. Flexed posture b. Abundant lanugo c. Smooth, pink skin with visible veins d. Faint red marks on the soles of the feet

ANS: A Term infants typically have a flexed posture. Abundant lanugo usually is seen on preterm infants. Smooth, pink skin with visible veins is seen on preterm infants. Faint red marks usually are seen on preterm infants. PTS: 1 DIF: Cognitive Level: Comprehension REF: 587 OBJ: Nursing Process: Diagnosis MSC: Client Needs: Health Promotion and Maintenance

Which newborn reflex is elicited by stroking the lateral sole of the infants foot from the heel to the ball of the foot? a. Babinski b. Tonic neck c. Stepping d. Plantar grasp

ANS: A The Babinski reflex causes the toes to flare outward and the big toe to dorsiflex. The tonic neck reflex (also called the fencing reflex) refers to the posture assumed by newborns when in a supine position. The stepping reflex occurs when infants are held upright with their heel touching a solid surface and the infant appears to be walking. Plantar grasp reflex is similar to the palmar grasp reflex: when the area below the toes is touched, the infants toes curl over the nurses finger. PTS: 1 DIF: Cognitive Level: Knowledge REF: 576 OBJ: Nursing Process: Assessment MSC: Client Needs: Health Promotion and Maintenance

24. A parent confides to the nurse that a friend, who is 32, has been diagnosed with Hodgkin's disease. The parent says "I thought only children get that!" What response by the nurse is the most appropriate? A. "No, there are both young adult and older adult forms." B. "Usually people over the age of 50 do not get this." C. "Yes, only children under the age of 10 are affected." D. "You are right; your friend must have misspoken."

ANS: A Three groups are affected by Hodgkin's disease: children younger than 14, young adults 15-34 years of age, and older adults 55-74 years of age. The parent's friend could certainly be correct about the diagnosis.

27. What food choice by the parent of a 2-year-old child with celiac disease indicates a need for further teaching? a. Oatmeal b. Rice cake c. Corn muffin d. Meat patty

ANS: A The child with celiac disease is unable to fully digest gluten, the protein found in wheat, barley, rye, and oats. Oatmeal contains gluten and is not an appropriate food selection. Rice is an appropriate choice because it does not contain gluten. Corn is digestible because it does not contain gluten. Meats do not contain gluten and can be included in the diet of a child with celiac disease.

16. What condition is characterized by a chronic inflammatory process that may involve any part of the gastrointestinal (GI) tract from mouth to anus? a. Crohn's disease b. Ulcerative colitis c. Meckel's diverticulum d. Irritable bowel syndrome

ANS: A The chronic inflammatory process of Crohn's disease involves any part of the GI tract from the mouth to the anus but most often affects the terminal ileum. Ulcerative colitis, Meckel's diverticulum, and irritable bowel syndrome do not affect the entire GI tract.

3. At least five factors affect the process of labor and birth. These are easily remembered as the five Ps. Which factors are included in this process? (Select all that apply.) a. Passenger b. Passageway c. Powers d. Pressure e. Psychologic response

ANS: A, B, C, E The five Ps are passenger (fetus and placenta), passageway (birth canal), powers (contractions), position of the mother, and psychologic response. Pressure is not one of the five Ps.

4. Because of its size and rigidity, the fetal head has a major effect on the birth process. Which bones comprise the structure of the fetal skull? (Select all that apply.) a. Parietal b. Temporal c. Fontanel d. Occipital e. Femoral

ANS: A, B, D The fetal skull has two parietal bones, two temporal bones, an occipital bone, and a frontal bone. The fontanels are membrane-filled spaces.

The treatment of brain tumors in children consists of which therapies? (Select all that apply.) a. Surgery b. Bone marrow transplantation c. Chemotherapy d. Stem cell transplantation e. Radiation f. Myelography

ANS: A, C, E Treatment for brain tumors in children may consist of surgery, chemotherapy, and radiotherapy alone or in combination. Bone marrow and stem cell transplantation therapies are used for leukemia, lymphoma, and other solid tumors where myeloablative therapies are used. Myelography is a radiographic examination after an intrathecal injection of contrast medium. It is not a treatment.

When taking the history of a child hospitalized with Reye syndrome, the nurse should not be surprised that a week ago the child had recovered from: a. measles. b. varicella. c. meningitis. d. hepatitis.

ANS: B Most cases of Reye syndrome follow a common viral illness such as varicella or influenza. Measles, meningitis, and hepatitis are not associated with Reye syndrome.

11. The diet of a child with nephrosis usually includes: a. High protein. b. Salt restriction. c. Low fat. d. High carbohydrate.

ANS: B Salt is usually restricted (but not eliminated) during the edema phase. The child has very little appetite during the acute phase. Favorite foods are provided (with the exception of high-salt ones) in an attempt to provide nutritionally complete meals.

8. A neutropenic child is admitted to the hospital and placed in protective isolation. Which instruction does the nurse give the family to help maintain a safe environment for the child? A. Do not let the child have chewing gum B. Flowers, plants, and produce are not allowed C. The child can only have one visitor at a time D. Toys and items from home cannot be brought in

ANS: B The neutropenic child should not have fresh flowers, plants, fruits, or vegetables because they can harbor infectious microorganisms. The other instructions are not needed.

21. One of the clinical manifestations of chronic renal failure is uremic frost. What best describes this term? a. Deposits of urea crystals in urine b. Deposits of urea crystals on skin c. Overexcretion of blood urea nitrogen d. Inability of body to tolerate cold temperatures

ANS: B Uremic frost is the deposition of urea crystals on the skin, not in the urine. The kidneys are unable to excrete blood urea nitrogen, leading to elevated levels. There is no relation between cold temperatures and uremic frost.

29. A child has cancer, is unresponsive, and is doing poorly. Which action by the nursing student causes the faculty to intervene? A. Allows the parents to hold the child B. Places the child on NPO status C. Takes the child's rectal temperature D. Turns the child even if she moans

ANS: C The nurse avoids the rectal route for anything: temperatures, suppositories, and enemas are not allowed, as the rectal mucosa is fragile and prone to injury, which can lead to infection. The other actions are appropriate.

A pregnant woman at 37 weeks of gestation has had ruptured membranes for 26 hours. A cesarean section is performed for failure to progress. The fetal heart rate (FHR) before birth is 180 beats/min with limited variability. At birth the newborn has Apgar scores of 6 and 7 at 1 and 5 minutes and is noted to be pale and tachypneic. On the basis of the maternal history, the cause of this newborns distress is most likely to be: a. Hypoglycemia. b. Phrenic nerve injury. c. Respiratory distress syndrome. d. Sepsis.

ANS: D The prolonged rupture of membranes and the tachypnea (before and after birth) both suggest sepsis. An FHR of 180 beats/min is also indicative. This infant is at high risk for sepsis. PTS: 1 DIF: Cognitive Level: Comprehension REF: 666 OBJ: Nursing Process: Assessment MSC: Client Needs: Physiologic Integrity

28. A nurse works on the pediatric oncology floor. After receiving the handoff report, which child does the nurse assess first? A. Child on protective isolation B. 4 hours post-bone marrow biopsy C. Not eating an hour after chemotherapy D. Temperature of 101.5°F (38.5°C)

ANS: D This fever indicates a probable infection. The nurse will see this child first and provide report to the physician, if this has not already been done. This child is the sickest and should be seen first; one might be tempted to see the child in protective isolation first to avoid cross-contamination, but by following isolation precautions, this risk is minimized. Not eating after chemotherapy is not cause for concern, and the child 4 hours post-bone marrow biopsy should be stable.

A macrosomic infant is born after a difficult forceps-assisted delivery. After stabilization the infant is weighed, and the birth weight is 4550 g (9 pounds, 6 ounces). The nurses most appropriate action is to: a. Leave the infant in the room with the mother. b. Take the infant immediately to the nursery. c. Perform a gestational age assessment to determine whether the infant is large for gestational age. d. Monitor blood glucose levels frequently and observe closely for signs of hypoglycemia.

ANS: D This infant is macrosomic (more than 4000 g) and is at high risk for hypoglycemia. Blood glucose levels should be monitored frequently, and the infant should be observed closely for signs of hypoglycemia. Observation may occur in the nursery or in the mothers room, depending on the condition of the fetus. Regardless of gestational age, this infant is macrosomic. PTS: 1 DIF: Cognitive Level: Application REF: 668 OBJ: Nursing Process: Implementation MSC: Client Needs: Physiologic Integrity

Premature infants who exhibit 5 to 10 seconds of respiratory pauses followed by 10 to 15 seconds of compensatory rapid respiration are: a. Suffering from sleep or wakeful apnea. b. Experiencing severe swings in blood pressure. c. Trying to maintain a neutral thermal environment. d. Breathing in a respiratory pattern common to premature infants.

ANS: D This pattern is called periodic breathing and is common to premature infants. It may still require nursing intervention of oxygen and/or ventilation. Apnea is a cessation of respirations for 20 seconds or longer. It should not be confused with periodic breathing. PTS: 1 DIF: Cognitive Level: Comprehension REF: 684 OBJ: Nursing Process: Assessment MSC: Client Needs: Health Promotion and Maintenance

Which nursing action is most appropriate to correct a boggy uterus that is displaced above and to the right of the umbilicus? a. Notify the physician of an impending hemorrhage. b. Assess the blood pressure and pulse. c. Evaluate the lochia. d. Assist the patient in emptying her bladder.

ANS: D Urinary retention may cause overdistention of the urinary bladder, which lifts and displaces the uterus. Nursing actions need to be implemented before notifying the physician. It is important to evaluate blood pressure, pulse, and lochia if the bleeding continues; however, the focus at this point in time is to assist the patient in emptying her bladder. PTS: 1 DIF: Cognitive Level: Application REF: 498 OBJ: Nursing Process: Implementation MSC: Client Needs: Health Promotion and Maintenance

A child is unconscious after a motor vehicle accident. The watery discharge from the nose tests positive for glucose. The nurse should recognize that this suggests: a. diabetic coma. b. brainstem injury. c. upper respiratory tract infection. d. leaking of cerebrospinal fluid (CSF).

ANS: D Watery discharge from the nose that is positive for glucose suggests leaking of CSF from a skull fracture and is not associated with diabetes or respiratory tract infection. The fluid is probably CSF from a skull fracture and does not signify whether the brainstem is involved.

14. The school nurse has been asked to begin teaching sex education in the 5th grade. Which statement should be the foundation for the information the nurse should present? a. Children in 5th grade are too young for sex education. b. Children should be discouraged from asking too many questions. c. Correct terminology should be reserved for children who are older. d. Sex can be presented as a normal part of growth and development.

ANS: D When sex information is presented to school-age children, sex should be treated as a normal part of growth and development. Fifth graders are usually 10 to 11 years old. This age is not too young to speak about physiologic changes in their bodies. They should be encouraged to ask questions. Preadolescents need precise and concrete information.

A (playing peek-a-boo Because object permanence is a new achievement, peek-a-boo is an excellent activity to practice this new skill for visual stimulation. Playing pat-a-cake and showing how to clap hands will help with kinesthetic stimulation. Imitating animal sounds will help with auditory stimulation.)

An appropriate play activity for a 7-month-old infant to encourage visual stimulation is: a. Playing peek-a-boo. b. Playing pat-a-cake. c. Imitating animal sounds d. Showing how to clap hands.

B (Sweets should be consumed with meals so the teeth can be cleaned afterward. This decreases the amount of time that the sugar is in contact with the teeth. Raisins, honey, and molasses are highly cariogenic and should be avoided.)

An appropriate recommendation in preventing tooth decay in young children is to: a. Substitute raisins for candy. b. Serve sweets after a meal. c. Use honey or molasses instead of refined sugar. d. Serve sweets between meals.

D (Make a follow-up home visit to parents as soon as possible after the infant's death. A competent, qualified professional should visit the family at home as soon as possible after the death and provide the family with printed information about SIDS. An explanation of how SIDS could have been predicted and prevented is inappropriate. SIDS cannot be prevented or predicted. Discussions about the cause will only increase parental guilt. The parents should be asked only factual questions to determine the cause of death. Parents should be allowed and encouraged to make a last visit with their infant.)

An important nursing responsibility when dealing with a family experiencing the loss of an infant from sudden infant death syndrome (SIDS) is to: a. Explain how SIDS could have been predicted and prevented. b. Interview parents in depth concerning the circumstances surrounding the infant's death. c. Discourage parents from making a last visit with the infant. d. Make a follow-up home visit to parents as soon as possible after the infant's death.

C ("What time did you find the infant?" During a SIDS incident, if the infant is not pronounced dead at the scene, he or she may be transported to the emergency department to be pronounced dead by a physician. While they are in the emergency department, the parents are asked only factual questions, such as when they found the infant, how he or she looked, and whom they called for help. The nurse avoids any remarks that may suggest responsibility, such as "Why didn't you go in earlier?" "Didn't you hear the infant cry out?" or "Was the head buried in a blanket?")

An infant has been pronounced dead from sudden infant death syndrome (SIDS) in the emergency department. Which is an appropriate question to ask the parents? a. "Did you hear the infant cry out?" b. "Why didn't you check on the infant earlier?" c. "What time did you find the infant?" d. "Was the head buried in a blanket?"

The primary difference between the labor of a nullipara and that of a multipara is the: a. Amount of cervical dilation. b. Total duration of labor. c. Level of pain experienced. d. Sequence of labor mechanisms.

B (In a first-time pregnancy, descent is usually slow but steady; in subsequent pregnancies, descent is more rapid, resulting in a shorter duration of labor. Cervical dilation is the same for all labors. Level of pain is individual to the woman, not to the number of labors she has experienced. The sequence of labor mechanisms is the same with all labors.)

A woman with severe preeclampsia is being treated with an intravenous infusion of magnesium sulfate. This treatment is considered successful if: A. blood pressure is reduced to prepregnant baseline. B. seizures do not occur. C. deep tendon reflexes become hypotonic. D. diuresis reduces fluid retention

B (Magnesium sulfate is a central nervous system (CNS) depressant given primarily to prevent seizures . A temporary decrease in blood pressure can occur; however, this is not the purpose of administering this medication. Hypotonia is a sign of an excessive serum level of magnesium. It is critical that calcium gluconate be on hand to counteract the depressant effects of magnesium toxicity. Diuresis is not an expected outcome of magnesium sulfate administration.)

One of the theories about the onset of labor is the prostaglandin theory. While not being conclusively proven that the action of prostaglandins initiate labor, it is known that prostaglandins do play a role in labor. What is an action of prostaglandins? a) Stimulates uterine muscle to relax b) Softens cervix c) Initiates relaxation of perineum d) Initiates cervical dilation

B (Softens cervix Rationale: The prostaglandin theory is another theory of labor initiation. Prostaglandins influence labor in several ways, which include softening the cervix and stimulating the uterus to contract. However, evidence supporting the theory that prostaglandins are the agents that trigger labor to begin is inconclusive.)

A pregnant patient has a systolic blood pressure that exceeds 160 mm Hg. Which action should the nurse take for this patient? A. Administer magnesium sulfate intravenously. B. Obtain a prescription for antihypertensive medications. C. Restrict intravenous and oral fluids to 125 mL/hr. D. Monitor fetal heart rate (FHR) and uterine contractions (UCs).

B (Systolic blood pressure exceeding 160 mm Hg indicates severe hypertension in the patient. The nurse should alert the health care provider and obtain a prescription for antihypertensive medications, such as nifedipine (Adalat) and labetalol hydrochloride (Normodyne). Magnesium sulfate would be administered if the patient was experiencing eclamptic seizures. Oral and intravenous fluids are restricted when the patient is at risk for pulmonary edema. Monitoring FHR and UCs is a priority when the patient experiences a trauma so that any complications can be addressed immediately.)

From 4% to 8% of pregnant women have asthma, making it one of the most common preexisting conditions of pregnancy. Severity of symptoms usually peaks: A. in the first trimester. B. between 24 to 36 weeks of gestation. C. during the last 4 weeks of pregnancy. D. immediately postpartum

B. between 24 to 36 weeks of gestation. Women often have few symptoms of asthma during the first trimester. The severity of symptoms peaks between 24 and 36 weeks of gestation. Asthma appears to be associated with intrauterine growth restriction and preterm birth. During the last 4 weeks of pregnancy symptoms often subside. The period between 24 and 36 weeks of pregnancy is associated with the greatest severity of symptoms. Issues have often resolved by the time the woman delivers.

A (6 to 8 weeks The bones surrounding the posterior fontanel fuse and close by age 6 to 8 weeks. Ten weeks or longer is too late.)

By what age does the posterior fontanel usually close? a. 6 to 8 weeks b. 10 to 12 weeks c. 4 to 6 months d. 8 to 10 months

A fetus is assessed at 2 cm above the ischial spines. The nurse would document fetal station as: a) +2 b) 0 c) -2 d) +4

C (-2 Rationale: When the presenting part is above the ischial spines, it is noted as a negative station. Since the measurement is 2 cm, the station would be -2. A 0 station indicates that the fetal presenting part is at the level of the ischial spines. Positive stations indicate that the presenting part is below the level of the ischial spines.)

When assessing a woman in labor, the nurse is aware that the relationship of the fetal body parts to one another is called fetal: a. Lie. b. Presentation. c. Attitude. d. Position.

C (Attitude is the relation of the fetal body parts to one another. Lie is the relation of the long axis (spine) of the fetus to the long axis (spine) of the mother. Presentation refers to the part of the fetus that enters the pelvic inlet first and leads through the birth canal during labor at term. Position is the relation of the presenting part to the four quadrants of the mother's pelvis.)

Which of the following would alert the nurse that the client is in the transition phase of labor? a) Reduction of rectal pressure b) Decrease in the bloody show c) Beginning urge to bear down d) Enthusiasm in the client

C (Beginning urge to bear down Rationale: Starting of the urge to bear down is a feature associated with the transition phase of labor. The transition phase is the last phase of the first stage of labor. In this phase the process of cervical dilatation is completed. During this phase the client experiences an increase in rectal pressure, an increase in the bloody show and an urge to bear down. The contractions are stronger and hence the client feels irritable, restless and nauseous. The client feels enthusiastic during the latent phase and not the transition phase.)

The skull is the most important factor in relation to the labor and birth process. The fetal skull must be small enough to travel through the bony pelvis. What feature of the fetal skull helps to make this passage possible? a) Vertex presentation b) Caput succedaneum c) Molding d) Cephalohematoma

C (Molding Rationale: The cartilage between the bones allows the bones to overlap during labor, a process called molding that elongates the fetal skull thereby reducing the diameter of the head.)

Which position would the nurse suggest for second-stage labor if the pelvic outlet needs to be increased? a) Semirecumbent b) Sitting c) Squatting d) Side-lying

C (Squatting)

A post delivery CBC has noted an elevated WBC count of 22,000/mm3. Which rationale is accurate regarding the elevated WBC count? a) Abnormal finding and she needs antibiotics b) Might be a false result, recommend re-testing c) This is a normal variation due to stress of labor d) Occurs in patients who have cesarean birth, from the trauma of surgery

C (This is a normal variation due to stress of labor Rationale: An elevation of WBC up to 30,000mm/3 can be normal variation for any woman after delivery. This is related to the stress on her body from labor and delivery. Antibiotics are not indicated as this is a normal response to intense stress. The increase in WBC is not related to cesarean birth. Retesting would be wasteful as it is known that this is a normal response to any stress.)

In order to care for obstetric patients adequately, the nurse understands that labor contractions facilitate cervical dilation by: a. Contracting the lower uterine segment. b. Enlarging the internal size of the uterus. c. Promoting blood flow to the cervix. d. Pulling the cervix over the fetus and amniotic sac.

D (Effective uterine contractions pull the cervix upward at the same time that the fetus and amniotic sac are pushed downward. The contractions are stronger at the fundus. The internal size becomes smaller with the contractions; this helps to push the fetus down. Blood flow decreases to the uterus during a contraction.)

9. The role of the nurse with regard to informed consent is to: a. inform the patient about the procedure and have her sign the consent form. b. act as a patient advocate and help clarify the procedure and the options. c. call the physician to see the patient. d. witness the signing of the consent form.

ANS: B Nurses play a part in the informed consent process by clarifying and describing procedures or by acting as the woman's advocate and asking the primary health care provider for further explanations. The physician is responsible for informing the woman of her options, explaining the procedure, and advising the patient about potential risk factors. The physician must be present to explain the procedure to the patient. However, the nurse's responsibilities go further than simply asking the physician to see the patient. The nurse may witness the signing of the consent form. However, depending on the state's guidelines, the woman's husband or another hospital health care employee may sign as witness.

11. Which type of croup is always considered a medical emergency? a. Laryngitis c. Spasmodic croup b. Epiglottitis d. Laryngotracheobronchitis (LTB)

ANS: B Epiglottitis is always a medical emergency needing antibiotics and airway support for treatment. Laryngitis is a common viral illness in older children and adolescents, with hoarseness and upper respiratory infection symptoms. Spasmodic croup is treated with humidity. LTB may progress to a medical emergency in some children

The nurse is teaching nursing students about childhood fractures. Which describes a compound skull fracture? a. Involves the basilar portion of the occipital bone b. Bone is exposed through the skin c. Traumatic separations of the cranial sutures d. Bone is pushed inward, causing pressure on the brain

ANS: B A compound fracture has the bone exposed through the skin. A basilar fracture involves the basilar portion of the frontal, ethmoid, sphenoid, temporal, or occipital bone. Diastatic skull fractures are traumatic separations of the cranial sutures. A depressed fracture has the bone pushed inward, causing pressure on the brain.

7. A child is admitted and is scheduled to receive intravenous asparginase (Elspar). Which action by the nurse is most important when administering this medication? A. Arranging an outpatient hearing test B. Having emergency drugs on hand C. Monitoring the child's intake and output D. Providing anti-emetic drugs as needed

ANS: B Anaphylaxis is a possible side effect of this drug. Emergency medications should be readily available. Ototoxicity can be caused by carboplatin (Paraplatin). Monitoring intake and output is important for any child on IV therapy. Anti-emetic drugs are important for any child receiving chemotherapy.

Kristin, age 10 years, sustained a fracture in the epiphyseal plate of her right fibula when she fell off of a tree. When discussing this injury with her parents, the nurse should consider which statement? a. Healing is usually delayed in this type of fracture. b. Growth can be affected by this type of fracture. c. This is an unusual fracture site in young children. d. This type of fracture is inconsistent with a fall.

ANS: B Detection of epiphyseal injuries is sometimes difficult, but fractures involving the epiphysis or epiphyseal plate present special problems in determining whether bone growth will be affected. Healing of epiphyseal injuries is usually prompt. The epiphysis is the weakest point of the long bones. This is a frequent site of damage during trauma.

Parents ask the nurse for advice when telling their 4-year-old about a grandmother's death. The nurse's best response involves teaching the parents that the child's concept of death is: 1 temporary. 2 permanent. 3 personified in various forms. 4 inevitable at some age.

1 Death is seen as a temporary departure. Death is thought of as not permanent; life and death can change places with each other. Personification is typical of school-age children. Children 9 to 10 years old have this understanding of death.

Cardiovascular changes that cause the foramen ovale to close at birth are a direct result of: a. Increased pressure in the right atrium. b. Increased pressure in the left atrium. c. Decreased blood flow to the left ventricle. d. Changes in the hepatic blood flow.

ANS: B With the increase in the blood flow to the left atrium from the lungs, the pressure is increased, and the foramen ovale is functionally closed. The pressure in the right atrium decreases at birth. It is higher during fetal life. Blood flow increases to the left ventricle after birth. The hepatic blood flow changes, but that is not the reason for the closure of the foramen ovale. PTS: 1 DIF: Cognitive Level: Knowledge REF: 558 OBJ: Nursing Process: Assessment MSC: Client Needs: Physiologic Integrity

10. The nurse providing care for a woman with gestational diabetes understands that a laboratory test for glycosylated hemoglobin Alc: A. is now done for all pregnant women, not just those with or likely to have diabetes. B. is a snapshot of glucose control at the moment. C. would be considered evidence of good diabetes control with a result of 5% to 6%. D. is done on the patient's urine, not her blood.

ANS: C A score of 5% to 6% indicates good control. This is an extra test for diabetic women, not one done for all pregnant women. This test defines glycemic control over the previous 4 to 6 weeks. Glycosylated hemoglobin level tests are done on the blood.

8. When nurses help their expectant mothers assess the daily fetal movement counts, they should be aware that: A. alcohol or cigarette smoke can irritate the fetus into greater activity. B. "kick counts" should be taken every half hour and averaged every 6 hours, with every other 6-hour stretch off. C. the fetal alarm signal should go off when fetal movements stop entirely for 12 hours. D. obese mothers familiar with their bodies can assess fetal movement as well as average-size women.

ANS: C No movement in a 12-hour period is cause for investigation and possibly intervention. Alcohol and cigarette smoke temporarily reduce fetal movement. The mother should count fetal activity ("kick counts") two or three times daily for 60 minutes each time. Obese women have a harder time assessing fetal movement.

31. A woman who is older than 35 years may have difficulty achieving pregnancy primarily because: A. personal risk behaviors influence fertility. B. she has used contraceptives for an extended time. C. her ovaries may be affected by the aging process. D. prepregnancy medical attention is lacking.

ANS: C Once the mature woman decides to conceive, a delay in becoming pregnant may occur because of the normal aging of the ovaries. Older adults participate in fewer risk behaviors than younger adults. The past use of contraceptives is not the problem. Prepregnancy medical care is both available and encouraged.

A 4-week pregnant patient is undergoing an ultrasound. The report shows an absence of fetal heart activity. What does the nurse infer about the fetus from the report? 1 Normal finding 2 Congenital abnormality 3 impaired growth 4 Cardiac disorder

1 Fetal heart activity begins around 6 weeks, so 4 weeks is too early to detect fetal heart activity, and this is a normal finding. Absence of fetal heart activity at an advanced gestational age may indicate congenital anomalies, impaired growth, or cardiac disorders.

The nurse finds that the parent of a terminally ill child is unusually optimistic about the child's condition. What should the nurse interpret from this? The parent: 1 Is in the denial stage of grief. 2 Believes the diagnosis is wrong. 3 Normally has a cheerful nature. 4 Is optimistic about the treatment

1 Parents may be in a state of shock when first hearing about the terminal illness of their child. The parent's abnormal optimism about the child's condition indicates that the parent is in denial and has not accepted the child's illness. It is unlikely that the parent believes that the diagnosis is wrong. The diagnosis of a terminally ill condition in a child is depressing for any parent, even one with a cheerful nature. Being optimistic when there is no cure indicates the stage of shock and denial.

A child is scheduled for a tonsillectomy and is afraid of the surgery. The child asks the nurse, "Will I need another operation when I have a sore throat again?" Which response should the nurse give to the child? 1 "Once your tonsils are taken out, you will not need the surgery again." 2 "You will need to repeat the surgery when you have another infection." 3 "You will need to have another surgery when you turn 14 years old." 4 "Once your tonsils are fixed, you will not have any more sore throats."

1 The child does not have enough knowledge about the tonsillectomy. Therefore the child may have fear about the surgery. The nurse should explain to the child that once the tonsils are removed, they do not need "fixing" again. It helps relieve the child's fear about the operation, and the child may feel comfortable. Once the tonsillectomy has been done in the child, a second operation is not required after another throat infection. There will actually not be a need for repeating the operation at any age. The child needs to be instructed that there may be other sore throats in the future. However, the child needs to be reassured that future sore throats will not require surgery.

The nurse is reviewing lab values to determine Rh incompatibility between mother and fetus. Which specific lab result should the nurse assess? 1 Indirect Coombs test 2 Hemoglobin level 3 hCG level 4 Maternal serum alpha-fetoprotein (MSAFP

1 The indirect Coombs test is a screening tool for Rh incompatibility. If the maternal titer for Rh antibodies is greater than 1:8, amniocentesis for determination of bilirubin in amniotic fluid is indicated to establish the severity of fetal hemolytic anemia. Hemoglobin reveals the oxygen carrying capacity of the blood. hCG is the hormone of pregnancy. Maternal serum alpha-fetoprotein (MSAFP) levels are used as a screening tool for NTDs in pregnancy.

A couple who is going through a divorce asks the nurse how to disclose this news to their 4-year-old child. What would be the nurse's best response? 1 "You should sit down and calmly explain the situation to your child." 2 "You should not discuss it with the child until after the divorce is final." 3 'You should ask the grandparents or another relative to break the news." 4 "You should take your child to see a psychiatrist to break the news."

1 The parents should set aside time and explain the separation to their child. They should answer the child's questions and give the child time to absorb the information. The parents should not hide the information from the child because it can damage trust between the parents and the child. It is healthier when the child finds this out from parents rather than somebody else, whether it is the grandparents or a psychiatrist.

The community health nurse is performing an assessment of an adolescent. Which group does the nurse ask the adolescent about when assessing the primary social group? 1 Peers 2 Church group 3 Work colleagues 4 Professional seniors

1 There are two types of social groups, primary and secondary. The primary group consists of peers and family who have a direct contact with and influence on the person. Secondary groups are indirectly related to the person. The examples include social organizations such as the church group, and professional groups such as office colleagues and professional seniors. Test-Taking Tip: Read carefully and answer the question asked; pay attention to specific details in the question.

A patient in the first trimester of pregnancy undergoes a triple marker screening test. On reviewing the report, the nurse infers that the fetus may have Down syndrome. What clinical findings are noted by the nurse in the test reports? Select all that apply. 1 High levels of beta-human chorionic gonadotropin (β-hCG) 2 Low levels of pregnancy-associated placental protein (PAPP-A) 3 Low levels of inhibin-A in the fetal blood 4 Low levels of maternal serum alpha-fetoprotein (MSAFP) 5 Low levels of unconjugated estriol in the fetal blood

1, 2 Triple marker screening, which is performed in the first trimester of pregnancy, includes the measurement of two maternal biomarkers: PAPP-A and free β-hCG. High levels of free β-hCG and low levels of PAPP-A in the first trimester indicate that the fetus has Down syndrome, or trisomy 21.Inhibin-A is a placental hormone. Low levels of inhibin-A also indicate the possibility of Down syndrome, but inhibin-A levels are not measured in the triple marker screen; these levels are measured in quad screening. A low level of MSAFP and unconjugated estriol also indicate Down syndrome, but these can be measured only in the second and third trimesters.

The nurse is explaining the health care bill of rights for children to parents and children in a pediatric ward. What key information does the nurse discuss with the group? Select all that apply. 1 Quality health care 2 Economic assistance 3 Respect and personal dignity 4 Making choices and decisions 5 Complex information

1, 3, 4 The bill of rights emphasizes the quality of health care to the children and explains that children and teens should be treated with respect and dignity. The bill also states that children have the right to make choices and decisions in their health care. Children have right to get emotional support from the health care professionals. Children cannot always expect economic assistance from the hospital. The nurse need not explain complicated information such as pathologic process of the disease to the child since the child will not be able to understand. Such information should be provided to the caregivers of the child.

Autism is a complex developmental disorder. The diagnostic criteria for autism include delayed or abnormal functioning in which area(s) with onset before age 3 years (select all that apply)? a. Language as used in social communication b. Gross motor development c. Growth below the 5th percentile for height and weight d. Symbolic or imaginative play e. Social interaction

A, D, E

The nurse is discharging a young child from the hospital. The nurse should instruct the parents to look for which posthospital child behaviors? Select all the apply. 1 Tendency to cling to parents 2 Jealousy toward others 3 Demands for parents' attention 4 Anger toward parents 5 New fears such as nightmares

1, 3, 5 Young children's posthospital behaviors include: They show initial aloofness toward parents; this may last from a few minutes (most common) to a few days. This is frequently followed by dependency behaviors: tendency to cling to parents; demands for parents' attention; vigorous opposition to any separation (e.g., staying at preschool or with a babysitter). Other negative behaviors include: new fears (e.g., nightmares); resistance to going to bed, night waking; withdrawal and shyness; hyperactivity; temper tantrums; food peculiarities; attachment to blanket or toy; regression in newly learned skills (e.g., self-toileting). Posthospital behaviors for older children include negative behaviors: emotional coldness followed by intense, demanding dependence on parents; anger toward parents; jealousy toward others (e.g., siblings).

The nurse is teaching a group of student nurses about developmental patterns of neonates. Which information should the nurse include in the teaching plan? Select all that apply. Infants: 1 use their eyes before they use their hands. 2 gain control of their feet before their hands. 3 use their hands to observe things around them. 4 stand erect first to get control of their back. 5 have structural control of the head before the trunk

1, 5 Postnatal development happens in a cephalocaudal pattern, from the head to the feet. Infants use or develop their eyes before gaining control of their hands. Infants are able to first hold their head up, and then they can stabilize their trunk. In this development pattern, infants have control of their hands first and then gain control of their feet. Infants observe objects first using their eyes and then with their hands. Infants must be able to control their back before they can stand erect.

After reviewing the Doppler umbilical flow reports of a pregnant patient, the nurse advises the patient to quit smoking immediately. Which finding in the report could be the reason for this instruction? 1 High amniotic fluid volume (AFV) 2 High systolic-to-diastolic (S/D) ratio 3 Low amniotic fluid volume (AFV) 4 Low systolic-to-diastolic (S/D) ratio

2 Exposure to nicotine from maternal smoking has been reported to increase the fetal S/D ratio. An elevated S/D ratio indicates a poorly perfused placenta. To improve the blood supply to the placenta, the patient should quit smoking as soon as possible. The AFV cannot be assessed through Doppler umbilical blood flow study. Moreover, smoking does not affect amniotic fluid volume. Smoking increases the S/D ratio; it does not decrease it.

The nurse is caring for a child dying from cancer. Physical signs that the child is approaching death include: 1 rapid pulse. 2 change in respiratory pattern. 3 sensation of cold, although body feels hot. 4 loss of hearing followed by loss of other senses.

2 In the final hours of life the respiratory pattern may become labored, with periods of apnea. The pulse becomes weak and slowed. There is a sensation of heat, although the body feels cold. Hearing is the last sense to fail.

The pediatric nurse has recorded the birth weight, head circumference, axillary temperature, and crown to rump length of a newborn baby. What does the nurse consider while assessing the risk for mortality in this infant? 1 Measure head circumference on alternate weeks. 2 Use the birth weight for the assessment of infant mortality rate. 3 Prefer rectal temperature to axillary temperature in the new born. 4 Crown to rump length is the best indicator of infant mortality rate

2 Low birth weight is associated with high mortality rate so birth weight is considered in predicting the infant mortality rate. Head circumference, rectal temperature or axillary temperature, and crown to rump length are generally not preferred indicators as compared to birth weight. Large head may increase the susceptibility of the child to acquire a head injury. Body temperature fluctuates due to many physiologic and pathological reasons. Alterations in rectal or axillary temperature of a newborn do not indicate that the child has high infant mortality rate. The height (crown-rump length) of the newborn is not an indicator of infant mortality rate. Test-Taking Tip: Start by reading each of the answer options carefully. Usually at least one of them will be clearly wrong. Eliminate this one from consideration. Now you have reduced the number of response choices by one and improved the odds. Continue to analyze the options. If you can eliminate one more choice in a four-option question, you have reduced the odds to 50/50. While you are eliminating the wrong choices, recall often occurs. One of the options may serve as a trigger that causes you to remember what a few seconds ago had seemed completely forgotten.

Several nurses tell their nursing supervisor that they want to be able to attend the funeral of a child for whom they had cared. They say they felt especially close to both the child and the family. The supervisor should recognize that attending the funeral is: 1 appropriate because families expect this expression of concern. 2 appropriate because it can assist in the resolution of personal grief. 3 inappropriate because it is unprofessional. 4 inappropriate because it increases burnout.

2 Nurses should attend the funeral of a child if they felt closeness with the family. This will help the nurses grieve and gain closure. Families may or may not expect this expression of concern. The behavior is appropriate if a relationship existed between the nurses and family. This may prevent burnout.

A nursing professor is teaching a group of graduate nurses about anticipatory grief reactions. What information should be included in this presentation? 1 It may last for weeks or up to a few months. 2 It can happen when the death is expected. 3 It happens more than 1 year after the death. 4 It can result in feelings of excessive loneliness.

2 Nurses should be able to identify family members having anticipatory grief reactions. When death is expected, family members may experience anticipatory grief. Anticipatory grief may be manifested in varying behaviors and intensities and may include denial, anger, depression, and other psychological and physical symptoms. Anticipatory grief lasts while the loved one is preparing to die. There is no standard time frame for anticipatory grief. Complicated grief reactions are seen more than 1 year after the loss, whereas anticipatory grief happens before the death. The patient with complicated grief can display symptoms such as unusual sleep disturbances and feelings of excessive loneliness or emptiness.

The nurse is teaching the nursing students about functions of play in the hospital. Which statement made by the nursing student indicates the need for further teaching? "Play: 1 Can lessen the stress of separation from the family." 2 Makes the child nervous in a strange environment." 3 Helps the child develop a positive attitude for others." 4 Provides an expressive outlet for the child's creative ideas."

2 Play is one of the most important aspects of a child's life and one of the most effective tools for managing stress. It is helpful for the child to relieve stress. It is also essential for the child's mental, emotional, and social well-being. Play does not make the child anxious in an unfamiliar environment. It helps the child feel more secure in a strange environment. Play lessens the stress of separation from the family because the child is busy. During play, the child communicates with others, which helps develop a positive attitude toward others. It also stimulates thinking in the child by allowing the child to express creative ideas.

The nurse is caring for a 2-day-old neonate who is healthy but has a low body temperature. The nurse instructs the infant's mother to place the unclothed infant on her bare chest. Which finding in the infant indicates ineffective management of the infant's condition? 1 Hyperglycemia 2 Metabolic acidosis 3 Body weight of 21 lbs 4 Body weight of 7.5 lbs

2 The neonate has hypothermia, and therefore the nurse instructs the mother to perform kangaroo care, in which the unclothed infant is placed on the mother's bare chest. This ensures improved thermoregulation and improves the complications of hypothermia. The presence of metabolic acidosis is a symptom of hypothermia. Hypoglycemia is caused by hypothermia in the infant. The normal weight of a healthy neonate at birth is approximately 7.5 lbs. By one year of age, the infant's weight normally triples to 21 lbs. Test-Taking Tip: Be alert for details. Details provided in the stem of the item such as behavioral changes or clinical changes (or both) within a certain time period can provide a clue to the most appropriate response or, in some cases, responses.

The primary health care provider advises a pregnant woman to undergo a Doppler blood flow analysis after reviewing the amniocentesis reports. What clinical condition in the fetus could be the reason for this referral? 1 Down syndrome 2 Hemolytic anemia 3 Potter syndrome 4 Fetal hydrops

2 The presence of bilirubin in the amniotic fluid indicates the possibility of hemolytic anemia in the fetus. The degree of hemolytic anemia can be determined by using Doppler blood flow analysis. The presence of the placental hormone inhibin-A in the quad screen indicates Down syndrome. The amniotic fluid index values are used to detect Potter syndrome. Fetal hydrops is caused by polyhydramnios, which can be assessed by ultrasound scanning.

The nurse is caring for a child who is in the terminal stage of acute lymphocytic leukemia. What should be the goals of palliative care in the child? Select all that apply. 1 To assess the severity of disease symptoms 2 To provide effective pain management 3 To strive for the best possible quality of life 4 To mandate that the family join in the care 5 To support the family and the child

2, 3, 5 Palliative care focuses on relieving pain and managing symptoms related to a terminal disease. It also addresses the psychological, social, and spiritual problems of children living with terminal conditions. The goal of palliative care is to achieve the best quality of life for patients and their families, consistent with their values. Palliative care seeks to improve the quality of life for a patient with a terminal condition. The nurse will assess the severity of the child's symptoms, but this is not the goal of palliative care. The nurse should encourage family members to participate in the child's care but should not mandate their help.

After reviewing the triple marker screen reports of a patient who is in the second trimester of pregnancy, the nurse concludes that the fetus has trisomy 18. What factors in the report led to the nurse's conclusion? Select all that apply. 1 Low level of inhibin-A in the maternal serum 2 Low level of unconjugated estriol in serum 3 Elevated nuchal translucency (NT) in the fetus 4 Low level of maternal human chorionic gonadotrophin (hCG) 5 Low level of maternal serum alpha-fetoprotein (MSAFP)

2, 4, 5 The triple marker screen measures the levels of three maternal serum markers: unconjugated estriol, hCG, and MSAFP. Low values of unconjugated estriol, hCG, and MSAFP indicate that the fetus has trisomy 18. The quad screen has an additional serum marker: inhibin-A. A low inhibin-A level indicates the possibility of Down syndrome. NT is not a serum marker protein. Moreover, elevated NT indicates that the fetus has a chromosomal abnormality but does not specifically indicate that the fetus has trisomy 18.

The Kohlberg moral development theory states that children are concerned with conformity and loyalty at a stage of their growth. When this stage is correlated with the cognitive development of children, what would the age group be? 1 0-2 years 2 2-7 years 3 7-11 years 4 11-15 years

3 According to Kohlberg's moral development theory, when children are concerned with conformity and loyalty, they are at the conventional level. Children at this level are considered to be working on concrete operations of cognitive development, where children 7-11 years old are included. Infants between 0 and 2 years of age are included in the sensory motor level of cognitive development. Children between 2 and 7 years old are in the preoperational stage of cognitive development. Children between 11 and 15 years are considered to be in the formal operation stage of cognitive development.

According to Piaget, the adolescent is in the fourth stage of cognitive development, or period of: a. Formal operations. c. Conventional thought. b. Concrete operations. d. Postconventional thought.

A Cognitive thinking culminates with capacity for abstract thinking. This stage, the period of formal operations, is Piaget's fourth and last stage. The concrete operations stage usually develops between ages 7 and 11 years. Conventional and postconventional thought refer to Kohlberg's stages of moral development.

The nurse is caring for an 8-year-old child who has a chronic illness. The child has a tracheostomy, and a parent is rooming-in during this hospitalization. The parent insists on providing almost all of the child's care and tells the nurses how to care for the child. When planning the child's care, the primary nurse should recognize that the parent is: 1 controlling and demanding. 2 assuming the nurse's role. 3 the expert in care of the child. 4 not allowing nurses to function independently.

3 Because these parents care for this child with complex health needs at home, they are most familiar with the care requirements and routine. The nurse's role includes assessment and evaluation, not just the implementation phase. The nurse recognizes that the philosophy of family-centered care states that the parents are the experts in the care of their child. The nurse functions collaboratively with the family.

The nurse is speaking to a group in the community about psychosocial development according to Erikson's life-span approach. The nurse instructs the group not to impose too many expectations on a child because the child may develop an inferiority complex. What age group of children is nurse referring to here? 1 1-3 years 2 3-6 years 3 6-12 years 4 12-18 years

3 Erikson's life-span approach categorized childhood into five stages. Industry versus Inferiority is the fourth stage of development the crucial stage attained by children 6-12 years of age. Children at this stage are workers and producers, and they initiate and complete work aiming at real achievement. The child may feel inferior if parents impose many expectations on the child. The second stage is autonomy versus shame and doubt (1-3 years), when children increase their ability to control their bodies and their environment and use their mental powers in decision making. Negative feelings develop when children are made to feel low and when others shame them. Initiative versus guilt (3-6 years) is when children explore the physical world with all their senses and powers and may feel guilt when parents make their child feel their behaviors are bad. Identity versus role confusion (12-18 years) is the stage when rapid and marked physical changes occur. Adolescents struggle to fit the roles they have played and those they expect to play. When the ability to resolve these conflicts fails, it leads to role confusion.

Nurses play an important role in current issues and trends in health care. What is a current trend in pediatric nursing and health care today? 1 The patient is the unit of care for the health care provider. 2 Discharge planning begins when the physician writes the order. 3 Health promotion resources enable children to achieve their full potential. 4 The focus of pediatric health care is trending toward acute hospital care.

3 Health promotion provides opportunities to reduce differences in current health status among members of different groups and provides a better chance to achieve the fullest health potential. The patient and family is the unit of care for the health care provider. Discharge planning begins when the patient is admitted. The focus of pediatric health care is trending away from acute hospital settings.

The nurse is caring for a Buddhist patient. The nurse finds that the patient has not touched the food tray. The food tray holds chicken, bread, soup, and fruits. What is the most appropriate action by the nurse? 1 Start an intravenous (IV) infusion because the patient is not eating. 2 Advise the patient to eat the food because it is essential for health. 3 Ask the patient whether there is a problem with the food provided. 4 Ignore the behavior.

3 Many Buddhists are strictly vegetarian. This may explain why the patient does not touch the meal tray. The nurse should ask the patient what his or her dietary preferences are and provide what the patient wants. However, the nurse should not provide food choices that go against any dietary restrictions placed by the health care provider. It is inappropriate to start an IV line because IV fluids do not provide nutrition. The nurse should not force the patient to eat or ignore the patient.

A preschooler is diagnosed with terminal stage bronchial carcinoma. What would the child consider the most probable reason for the condition? 1 The child will blame the parents for acquiring the disease. 2 The child will blame the grandparents for acquiring this condition. 3 The child would consider it to be a punishment for his or her own actions. 4 The child would understand the pathophysiology of the disease.

3 Preschoolers tend to believe that their thoughts or actions are sufficient to cause death or disease. Therefore, the child is most likely to think that the disease is a punishment for the child's action. This may result in the child feeling guilty. The nurse should help parents to understand this kind of reaction of their child and encourage them to be with the child. The child will not blame the parents or grandparents for the condition. The child is too young to understand the pathophysiology of the disease.

Which statement describes a reconstituted family? 1 A child who lives with another family that has legal guardianship 2 A child staying with grandparents, away from the parents 3 A child staying with the father and the new stepmother 4 A child living in a family with both parents working for a living

3 Reconstituted families are families made up of stepfathers or stepmothers with or without their respective children. Children staying with another family with legal guardianship are in foster care. Children staying with their grandparents are in kinship care. Families where both parents work are called dual-earner families.

The nurse is assessing a child. The nurse asks the parents, "Has your child started sleeping less lately?" Which attribute of temperament is the nurse assessing? 1 Adaptability 2 Distractibility 3 Rhythmicity 4 Activity

3 Rhythmicity is an attribute of temperament that refers to regularity in the timing of physiologic functions such as sleep and hunger. Because the nurse is asking about changes in the child's duration of sleep, the nurse is assessing rhythmicity. Adaptability is an attribute of temperament that refers to the child's ability to adapt to new situations. Distractibility is an attribute of temperament that refers to the ease with which a child's attention is diverted. Activity is an attribute of temperament that refers to the levels of physical activity such as sleeping and eating.

The nurse develops a plan of care based on the information documented in a child's admission assessment. The nurse instructs the health care team that they should not leave the room until the child falls asleep. What information documented under the self-perception-self-concept pattern would necessitate this nursing intervention? The child has: 1 Nightmares. 2 Disturbed sleep patterns. 3 A fear of sleeping alone. 4 The habit of bed-wetting.

3 The child has fear of sleeping alone in the room. This information is usually noted under the self-perception—self-concept pattern of the nursing history. The nurse tries to comfort the child's fear of sleeping alone by being present until the child falls asleep. Nightmares can be managed by comforting the child and preventing specific fears. Nightmares and disturbed sleep may be brought on by hospitalization and may improve once the child adapts himself or herself to the new environment. Bed-wetting is common in younger children but needs further evaluation in older children.

Parents who are divorcing are worried about how the divorce will affect their child. Which symptom indicates unhealthy coping in the child? 1 Body aches 2 Increased sleep 3 Loss of appetite 4 Body rash

3 The most common manifestation of distress in children is loss of appetite. The child is emotionally distressed by the parents' separation, and this stress can be manifested by a loss of appetite. Body aches, increased sleep, and rash over the body are physical symptoms that are very rarely seen in emotional distress. These symptoms may be caused by a viral infection.

The nurse notices that a child's spleen is quite large. To which age group does the child belong? 1 0-12 months 2 1-6 years 3 6-12 years 4 12-18 years

3 The spleen is easily palpated between the ages of 6 and 12 years. If it is palpated at any other age, such as 0-12 months, 1-6 years, or 12-18 years, this finding must be reported to the primary health care provider.

While reviewing the ultrasound reports of a patient, the nurse notices a floating fetus in the scanned image. What potential fetal risks should the nurse interpret from this finding? Select all that apply: 1 Renal agenesis 2 Growth restriction 3 Neural tube defects 4 Gastrointestinal obstruction 5 Cardiac disease.

3, 4 A floating fetus is seen in cases of elevated amniotic fluid volume, or polyhydramnios. Polyhydramnios is associated with neural tube defects and gastrointestinal obstruction. Renal agenesis and severe intrauterine growth restriction are associated with oligohydramnios, or low amniotic fluid volume. A low amount of fluid may not result in a floating fetus in the scanned image. The amniotic fluid level is unrelated to cardiac disease in the fetus.

The nursing instructor is explaining the risk factors and pathogenesis of Down syndrome to a group of nursing students. What information should the nurse include in the explanation? Select all that apply. 1 It is caused by a mutation of chromosomes. 2 It is more likely to occur if the paternal age is more than 35 years. 3 It is more likely to occur if the maternal age is more than 35 years. 4 It is caused by acquisition of an extra sex chromosome. 5 It is caused by acquisition of an extra autosomal chromosome.

3, 5 Maternal age more than 35 years increases the risk of having babies with Down syndrome. Down syndrome is caused by the presence of an extra autosomal chromosome. Down syndrome is not caused by a mutation of chromosomes. Advanced paternal age is not a risk factor for Down syndrome. There is no extra sex chromosome in children with Down syndrome.

The nurse is caring for a child who is in the final stages of death. What can the nurse do to provide comfort for the parents? 1 Explain to the parents that births and deaths are normal. 2 Inform the family that their child will soon be at peace. 3 Ask the parents to stay away from the child at this time. 4 Have the parents focus on the quality of the child's remaining life.

4 Acknowledging grief is often the first step toward facing the reality of the child's illness. It is important to have the parents spend the time that is left with their child. Such acceptance may help parents focus on the quality of the child's remaining life. The nurse should not be so blunt as to say that births and deaths are normal. It is not showing compassion. Even though the child will be at peace when the child dies, this is not the best response because it does not provide comfort. The nurse should not ask the parents to stay away from their child in their final stages of life.

During a prenatal checkup, the patient who is 7 months pregnant reports that she is able to feel about two kicks in an hour. The nurse refers the patient for an ultrasound. What is the primary reason for this referral? To check: 1 For fetal anomalies 2 Gestational age 3 Fetal position 4 For fetal well-being

4 Fetal kick count is a simple method to determine the presence of complications related to fetal oxygenation and activity level. The fetal kick count during the third trimester of pregnancy is approximately 30 kicks an hour; a count lower than that is an indication of poor health of the fetus. Fetal anomalies may not affect the oxygenation levels of the fetus. The nurse already knows the gestational age of the fetus; therefore the nurse need not refer the woman for ultrasonography to find the gestational age. Fetal position does not affect the activity level of the fetus.

The parents of a child are grieving the loss of their child. The mother of the child has disturbed sleep. The father informs the nurse that the mother can hear their child talk. The mother wants to be left alone and does not speak to any of the family members. What should be the most appropriate response of the nurse? 1 Tell the father that his wife may have mental breakdown in near future. 2 Tell the father that his wife is coping very poorly and needs professional help. 3 Inform the father that his wife has hallucinations and needs psychiatric referral. 4 Inform the father that it is a normal part of grief and would subside over time.

4 Grief reactions vary from person to person and it is required for the healing to occur. It is important for the nurse to inform the husband that hearing the dead person talk and distancing from others is very common reaction. It does not indicate any impending mental breakdown, poor coping, or psychiatric problem. Therefore, the husband should be reassured that his wife is trying to cope with the grief and she is normal.

The role of the pediatric nurse is influenced by trends in health care. The greatest trend in health care is: 1 primary focus on treatment of disease or disability. 2 national health care planning on a distributive or episodic basis. 3 accountability to professional codes and international standards. 4 shift of focus to prevention of illness and maintenance of health.

4 Prevention is the current focus of health care, one in which nursing plays a major role. Traditionally, treating disease or disability is the role of the physician. National health care planning is not a major trend. Accountability to professional codes is an established responsibility, not a trend.

A normal uterine activity pattern in labor is characterized by: a. Contractions every 2 to 5 minutes. b. Contractions lasting about 2 minutes. c. Contractions about 1 minute apart. d. A contraction intensity of about 1000 mm Hg with relaxation at 50 mm Hg.

A Contractions normally occur every 2 to 5 minutes and last less than 90 seconds (intensity 800 mm Hg) with about 30 seconds in between (20 mm Hg or less).

The nurse is evaluating the quality of evidence of research found in the field of infectious diseases in infants. The nurse found unusually strong evidence from unbiased observational studies. What grade should be given to this research according to the GRADE criteria? 1 Low 2 High 3 Very low 4 Moderate

4 The GRADE criteria are used for grading the evidence of research by nurses. According to this, if evidence is found to be unusually strong from unbiased observational studies, it should be graded as moderate. Low grade is given to the evidence with at least one critical outcome from the observation studies. High grade is given to the studies with exceptionally strong evidence from unbiased observational studies and very low grade is given to studies with at least one critical outcome from very indirect evidence.

A 40-year-old woman with a high body mass index is 10 weeks pregnant. Which diagnostic tool is appropriate to suggest to her at this time? 1 Biophysical profile 2 Amniocentesis 3 Maternal serum alpha-fetoprotein (MSAFP) 4 Transvaginal ultrasound

4 Transvaginal ultrasound is useful for obese women whose thick abdominal layers cannot be penetrated adequately with the abdominal approach. A biophysical profile is a method of biophysical assessment of fetal well-being in the third trimester. An amniocentesis is performed after the fourteenth week of pregnancy. A MSAFP test is performed from week 15 to week 22 of the gestation (weeks 16 to 18 are ideal). An ultrasound is the method of biophysical assessment of the infant that is performed at this gestational age.

A nurse is teaching a group of nursing students about the role of progesterone in labor. Which of the following should the nurse explain as the function of progesterone? a) Suppresses the uterine irritability throughout pregnancy b) Promotes oxytocin production from the posterior pituitary c) Sensitizes the uterus to effects of oxytocin on the myometrium d) Stimulates smooth muscle contraction in the uterus

A (Suppresses the uterine irritability throughout pregnancy Rationale: The function of progesterone is to suppress uterine irritability throughout pregnancy. The function of estrogen is to promote oxytocin production and to sensitize the uterus to the effects of oxytocin. Prostaglandin, and not progesterone, stimulates the smooth muscle contractions in the uterus.)

The pediatric nurse is working on a project to contribute to research and evidence-based practice. What should the nurse do when caring for patients of different age groups? Arrange the following steps in the correct order. 1. Develop a care plan. 2. Evaluate the effectiveness of intervention 3. Collect information. 4. Identify specific questions.

4, 3, 1, 2 The responses of the nurse to health and illness have to be followed in an order. The nurse has to identify specific questions to collect appropriate information and develop a care plan to implement. Identification of specific questions to be asked to the patient would help to formulate a clear and precise assessment plan. Collection of subjective and objective information of the patient would be helpful in determining the needs of the patient. Developing a care plan of the patient would help in establish the desired outcomes and the interventions required to achieve those outcomes. Finally, the nurse should evaluate the effectiveness of the intervention to determine if the care plan designed for the patient was successful. Test-Taking Tip: In this Question Type, you are asked to prioritize (put in order) the options presented. For example, you might be asked the steps of performing a nursing action or skill such as those involved in medication administration.

B (The blanket is an important transitional object.)

A hospitalized toddler clings to a worn, tattered blanket. She screams when anyone tries to take it away. What is the nurse's BEST explanation to the parents for the child's attachment to the blanket? A. The blanket encourages immature behavior. B. The blanket is an important transitional object. C. She has not mastered the developmental task of individuation-separation. D. She has not bonded adequately with her mother.

B (Vitamin D)

A mother is bringing her 4-month-old infant into the clinic for a routine well-baby check. The mother is exclusively breastfeeding. There are no other liquids given to the infant. What vitamin does the nurse anticipate the provider will prescribe for this infant? A. Vitamin B B. Vitamin D C. Vitamin C D. Vitamin K

B, C, D (The characteristics of preoperational thinking that occur for the toddler include egocentrism (views everything in relation to self), animism (believes that inert objects are alive), and magical thinking (believes that thinking something causes that event). Concrete thinking is seen in school-age children and ability to reason is seen with adolescents.)

A nurse is planning care for a hospitalized toddler in the preoperational thinking stage. Which characteristics should the nurse expect in this stage (select all that apply)? a. Concrete thinking b. Egocentrism c. Animism d. Magical thinking e. Ability to reason

16. A woman asks the nurse, "What protects my baby's umbilical cord from being squashed while the baby's inside of me?" The nurse's best response is: A. "Your baby's umbilical cord is surrounded by connective tissue called Wharton jelly, which prevents compression of the blood vessels and ensures continued nourishment of your baby." B. "Your baby's umbilical floats around in blood anyway." C. "You don't need to worry about things like that." D. "The umbilical cord is a group of blood vessels that are very well protected by the placenta."

ANS: A "Your baby's umbilical cord is surrounded by connective tissue called Wharton jelly, which prevents compression of the blood vessels and ensures continued nourishment of your baby" is the most appropriate response. "Your baby's umbilical floats around in blood anyway" is inaccurate. "You don't need to worry about things like that" is an inappropriate response. It negates the patient's need for teaching and discounts her feelings. The placenta does not protect the umbilical cord. The cord is protected by the surrounding Wharton jelly.

6. A key finding from the Human Genome Project is: A. approximately 20,500 genes make up the genome. B. all human beings are 80.99% identical at the DNA level. C. human genes produce only one protein per gene; other mammals produce three proteins per gene. D. single gene testing will become a standardized test for all pregnant patients in the future.

ANS: A Approximately 20,500 genes make up the human genome; this is only twice as many as make up the genomes of roundworms and flies. Human beings are 99.9% identical at the DNA level. Most human genes produce at least three proteins. Single gene testing (e.g., alpha-fetoprotein) is already standardized for prenatal care.

15. The nurse caring for the laboring woman should know that meconium is produced by: A. fetal intestines. B. fetal kidneys. C. amniotic fluid. D. the placenta.

ANS: A As the fetus nears term, fetal waste products accumulate in the intestines as dark green-to-black, tarry meconium.

12. Compared with contraction stress test (CST), nonstress test (NST) for antepartum fetal assessment: A. has no known contraindications. B. has fewer false-positive results. C. is more sensitive in detecting fetal compromise. D. is slightly more expensive.

ANS: A CST has several contraindications. NST has a high rate of false-positive results, is less sensitive than the CST, and is relatively inexpensive.

The nurse is planning a staff in-service on childhood spastic cerebral palsy. Spastic cerebral palsy is characterized by: a. hypertonicity and poor control of posture, balance, and coordinated motion. b. athetosis and dystonic movements. c. wide-based gait and poor performance of rapid, repetitive movements. d. tremors and lack of active movement.

ANS: A Hypertonicity and poor control of posture, balance, and coordinated motion are part of the classification of spastic cerebral palsy. Athetosis and dystonic movements are part of the classification of dyskinetic (athetoid) cerebral palsy. Wide-based gait and poor performance of rapid, repetitive movements are part of the classification of ataxic cerebral palsy. Tremors and lack of active movement may indicate other neurologic disorders.

20. A woman has a thick, white, lumpy, cottage cheese-like discharge, with patches on her labia and in her vagina. She complains of intense pruritus. The nurse practitioner would order which preparation for treatment? a. Miconazole b. Tetracycline c. Clindamycin d. Acyclovir

ANS: A Miconazole and clotrimazole are the drugs of choice to treat candidiasis. Tetracycline is used to treat syphilis. Clindamycin is used to treat bacterial vaginosis. Acyclovir is used to treat genital herpes.

4. Screening at 24 weeks of gestation reveals that a pregnant woman has gestational diabetes mellitus (GDM). In planning her care, the nurse and the woman mutually agree that an expected outcome is to prevent injury to the fetus as a result of GDM. The nurse identifies that the fetus is at greatest risk for: A. macrosomia. B. congenital anomalies of the central nervous system. C. preterm birth. D. low birth weight.

ANS: A Poor glycemic control later in pregnancy increases the rate of fetal macrosomia. Poor glycemic control during the preconception time frame and into the early weeks of the pregnancy is associated with congenital anomalies. Preterm labor or birth is more likely to occur with severe diabetes and is the greatest risk in women with pregestational diabetes. Increased weight, or macrosomia, is the greatest risk factor for this woman.

31. The nurse knows that proper placement of the tocotransducer for electronic fetal monitoring is located: a. over the uterine fundus. b. on the fetal scalp. c. inside the uterus. d. over the mother's lower abdomen.

ANS: A The tocotransducer monitors uterine activity and should be placed over the fundus, where the most intensive uterine contractions occur. The tocotransducer is for external use.

A 10-year-old girl needs to have another intravenous (IV) line started. She keeps telling the nurse, Wait a minute, and, Im not ready. The nurse should recognize that: a. This is normal behavior for a school-age child. b. This behavior is usually not seen past the preschool years. c. The child thinks the nurse is punishing her. d. The child has successfully manipulated the nurse in the past.

ANS: A This school-age child is attempting to maintain control. The nurse should provide the girl with structured choices about when the IV will be inserted. This can be characteristic behavior when an individual needs to maintain some control over a situation. The child is trying to have some control in the hospital experience.

3. While developing an intrapartum care plan for the patient in early labor, it is important that the nurse recognize that psychosocial factors may influence a woman's experience of pain. These include: (Select all that apply.) a. culture. b. anxiety and fear. c. previous experiences with pain. d. intervention of caregivers. e. support systems.

ANS: A, B, C, E Culture: A woman's sociocultural roots influence how she perceives, interprets, and responds to pain during childbirth. Some cultures encourage loud and vigorous expressions of pain, whereas others value self-control. The nurse should avoid praising some behaviors (stoicism) while belittling others (noisy expression). Anxiety and fear: Extreme anxiety and fear magnify sensitivity to pain and impair a woman's ability to tolerate it. Anxiety and fear increase muscle tension in the pelvic area, which counters the expulsive forces of uterine contractions and pushing efforts. Previous experiences with pain: Fear and withdrawal are a natural response to pain during labor. Learning about these normal sensations ahead of time helps a woman suppress her natural reactions of fear regarding the impending birth. If a woman previously had a long and difficult labor, she is likely to be anxious. She may also have learned ways to cope and may use these skills to adapt to the present labor experience. Support systems: An anxious partner is less able to provide help and support to a woman during labor. A woman's family and friends can be an important source of support if they convey realistic and positive information about labor and delivery. Although the intervention of caregivers may be necessary for the well-being of the woman and her fetus, some interventions add discomfort to the natural pain of labor (i.e., fetal monitor straps, intravenous lines).

8. A child has been admitted with a paraspinal Ewing's sarcoma. The nursing instructor questions the student about assessing for signs of spinal cord compression. Which manifestations does this include? (Select all that apply.) A. Burning pain down the legs B. Difficulty with swallowing C. "Foot drop," causing a limp D. Respiratory depression E. Weakness in the hands

ANS: A, C, E Common manifestations of spinal cord compression include burning pain, often down the legs; foot drop causing difficulty with ambulation; and weakness in the hands. Other manifestations include numbness, cramping, and loss of sensation in the feet, and sexual dysfunction in the older patient. Swallowing problems and respiratory depression are not related.

4. The nurse should expect to assess which clinical manifestations in an adolescent with Cushing's syndrome? (Select all that apply.) a. Hyperglycemia b. Hyperkalemia c. Hypotension d. Cushingoid features e. Susceptibility to infections

ANS: A, D, E In Cushing's syndrome, physiologic disturbances seen are cushingoid features, hyperglycemia, susceptibility to infection, hypertension, and hypokalemia.

11. With regard to chromosome abnormalities, nurses should be aware that: A. they occur in approximately 10% of newborns. B. abnormalities of number are the leading cause of pregnancy loss. C. Down syndrome is a result of an abnormal chromosome structure. D. unbalanced translocation results in a mild abnormality that the child will outgrow.

ANS: B Aneuploidy is an abnormality of number that also is the leading genetic cause of mental retardation. Chromosome abnormalities occur in less than 1% of newborns. Down syndrome is the most common form of trisomal abnormality, an abnormality of chromosome number (47 chromosomes). Unbalanced translocation is an abnormality of chromosome structure that often has serious clinical effects.

14. With regard to a pregnant woman's anxiety and pain experience, nurses should be aware that: a. even mild anxiety must be treated. b. severe anxiety increases tension, which increases pain, which in turn increases fear and anxiety, and so on. c. anxiety may increase the perception of pain, but it does not affect the mechanism of labor. d. women who have had a painful labor will have learned from the experience and have less anxiety the second time because of increased familiarity.

ANS: B Anxiety and pain reinforce each other in a negative cycle. Mild anxiety is normal for a woman in labor and likely needs no special treatment other than the standard reassurances. Anxiety increases muscle tension and ultimately can build sufficiently to slow the progress of labor. Unfortunately, an anxious, painful first labor is likely to carry over, through expectations and memories, into an anxious and painful experience in the second pregnancy.

13. During a health history interview, a woman states that she thinks that she has "bumps" on her labia. She also states that she is not sure how to check herself. The correct response would be to: A. reassure the woman that the examination will not reveal any problems. B. explain the process of vulvar self-examination to the woman and reassure her that she should become familiar with normal and abnormal findings during the examination. C. reassure the woman that "bumps" can be treated. D. reassure her that most women have "bumps" on their labia.

ANS: B During the assessment and evaluation, the responsibility for self-care, health promotion, and enhancement of wellness is emphasized. The pelvic examination provides a good opportunity for the practitioner to emphasize the need for regular vulvar self-examination. Providing reassurance to the woman concerning the "bumps" would not be an accurate response.

27. A 25-year-old single woman comes to the gynecologist's office for a follow-up visit related to her abnormal Papanicolaou (Pap) smear. The test revealed that the patient has human papillomavirus (HPV). The patient asks, "What is that? Can you get rid of it?" Your best response is: A. "It's just a little lump on your cervix. We can freeze it off." B. "HPV stands for 'human papillomavirus.' It is a sexually transmitted infection (STI) that may lead to cervical cancer. There is no known cure but symptoms are treated." C. "HPV is a type of early human immunodeficiency virus (HIV). You will die from this." D. "You probably caught this from your current boyfriend. He should get tested for this."

ANS: B It is important to inform the patient about STIs and the risks involved with HPV. The health care team has a duty to provide proper information to the patient, including information related to STIs. HPV and HIV are both viruses that can be transmitted sexually, but they are not the same virus. The onset of HPV can be insidious. Often STIs go unnoticed. Abnormal bleeding frequently is the initial symptom. The patient may have had HPV before her current boyfriend. You cannot make any deductions from this limited information.

Amy, age 6 years, needs to be hospitalized again because of a chronic illness. The clinic nurse overhears her school-age siblings tell her, We are sick of Mom always sitting with you in the hospital and playing with you. It isnt fair that you get everything and we have to stay with the neighbors. The nurses best assessment of this situation is that: a. The siblings are immature and probably spoiled. b. Jealousy and resentment are common reactions to the illness or hospitalization of a sibling. c. The family has ineffective coping mechanisms to deal with chronic illness. d. The siblings need to better understand their sisters illness and needs.

ANS: B Siblings experience loneliness, fear, worry, anger, resentment, jealousy, and guilt. The siblings experience stress equal to that of the hospitalized child. These are not uncommon responses by normal siblings. There is no evidence that the family has maladaptive coping or that the siblings lack understanding.

16. After an emergency birth, the nurse encourages the woman to breastfeed her newborn. The primary purpose of this activity is to: a. facilitate maternal-newborn interaction. b. stimulate the uterus to contract. c. prevent neonatal hypoglycemia. d. initiate the lactation cycle.

ANS: B Stimulation of the nipples through breastfeeding or manual stimulation causes the release of oxytocin and prevents maternal hemorrhage. Breastfeeding facilitates maternal-newborn interaction, but it is not the primary reason a woman is encouraged to breastfeed after an emergency birth. The primary intervention for preventing neonatal hypoglycemia is thermoregulation. Cold stress can result in hypoglycemia. The woman is encouraged to breastfeed after an emergency birth to stimulate the release of oxytocin, which prevents hemorrhage. Breastfeeding is encouraged to initiate the lactation cycle, but it is not the primary reason for this activity after an emergency birth.

23. As relates to the structure and function of the placenta, the maternity nurse should be aware that: A. as the placenta widens, it gradually thins to allow easier passage of air and nutrients. B. as one of its early functions, the placenta acts as an endocrine gland. C. the placenta is able to keep out most potentially toxic substances such as cigarette smoke to which the mother is exposed. D. optimal blood circulation is achieved through the placenta when the woman is lying on her back or standing.

ANS: B The placenta produces four hormones necessary to maintain the pregnancy. The placenta widens until week 20 and continues to grow thicker. Toxic substances such as nicotine and carbon monoxide readily cross the placenta into the fetus. Optimal circulation occurs when the woman is lying on her side.

19. The measurement of lecithin in relation to sphingomyelin (L/S ratio) is used to determine fetal lung maturity. Which ratio reflects maturity of the lungs? a. 1.4:1 b. 1.8:1 c. 2:1 d. 1:1

ANS: C A ratio of 2:1 indicates a two-to-one ratio of L/S, an indicator of lung maturity. Ratios of 1.4:1, 1.8:1, and 1:1 indicate immaturity of the fetal lungs.

21. It is paramount for the obstetric nurse to understand the regulatory procedures and criteria for admitting a woman to the hospital labor unit. Which guideline is an important legal requirement of maternity care? a. The patient is not considered to be in true labor (according to the Emergency Medical Treatment and Active Labor Act [EMTALA]) until a qualified health care provider says she is. b. The woman can have only her male partner or predesignated "doula" with her at assessment. c. The patient's weight gain is calculated to determine whether she is at greater risk for cephalopelvic disproportion (CPD) and cesarean birth. d. The nurse may exchange information about the patient with family members.

ANS: C According to EMTALA, a woman is entitled to active labor care and is presumed to be in "true" labor until a qualified health care provider certifies otherwise. A woman can have anyone she wishes present for her support. The risk for CPD is especially great for petite women or those who have gained 16 kg or more. All patients should have their weight and BMI calculated on admission. This is part of standard nursing care on a maternity unit and not a regulatory concern. According to the Health Insurance Portability and Accountability Act (HIPAA), the patient must give consent for others to receive any information related to her condition.

17. When evaluating a patient for sexually transmitted infections (STIs), the nurse should be aware that the most common bacterial STI is: a. gonorrhea. b. syphilis. c. chlamydia. d. candidiasis.

ANS: C Chlamydia is the most common and fastest spreading STI among American women, with an estimated 3 million new cases each year. Gonorrhea and syphilis are bacterial STIs, but they are not the most common ones among American women. Candidiasis is caused by a fungus, not by bacteria.

9. A nulliparous woman who has just begun the second stage of her labor would most likely: a. experience a strong urge to bear down. b. show perineal bulging. c. feel tired yet relieved that the worst is over. d. show an increase in bright red bloody show.

ANS: C Common maternal behaviors during the latent phase of the second stage of labor include feeling a sense of accomplishment and optimism because "the worst is over." During the latent phase of the second stage of labor, the urge to bear down often is absent or only slight during the acme of contractions. Perineal bulging occurs during the transition phase of the second stage of labor, not at the beginning of the second stage. An increase in bright red bloody show occurs during the descent phase of the second stage of labor.

23.The diet of a child with chronic renal failure is usually characterized as: a. High in protein. b. Low in vitamin D. c. Low in phosphorus. d. Supplemented with vitamins A, E, and K.

ANS: C Dietary phosphorus is controlled to prevent or control the calcium/phosphorus imbalance by the reduction of protein and milk intake. Protein should be limited in chronic renal failure to decrease intake of phosphorus. Vitamin D therapy is administered in chronic renal failure to increase calcium absorption. Supplementation with vitamins A, E, and K is not part of dietary management in chronic renal disease.

29. Nursing intervention for the pregnant diabetic patient is based on the knowledge that the need for insulin: A. increases throughout pregnancy and the after birth period. B. decreases throughout pregnancy and the after birth period. C. varies depending on the stage of gestation. D. should not change because the fetus produces its own insulin.

ANS: C Insulin needs decrease during the first trimester, when nausea, vomiting, and anorexia are a factor. They increase during the second and third trimesters, when the hormones of pregnancy create insulin resistance in maternal cells. Insulin needs increase during the second and third trimesters, when the hormones of pregnancy create insulin resistance in maternal cells. The insulin needs change throughout the different stages of pregnancy.

Iron dextran is ordered for a young child with severe iron deficiency anemia. Nursing considerations include to: a. administer with meals. b. administer between meals. c. inject deeply into a large muscle. d. massage injection site for 5 minutes after administration of drug.

ANS: C Iron dextran is a parenteral form of iron. When administered intramuscularly, it must be injected into a large muscle. Iron dextran is for intramuscular or intravenous (IV) administration. The site should not be massaged to prevent leakage, potential irritation, and staining of the skin.

1. When assessing a patient for amenorrhea, the nurse should be aware that this is unlikely to be caused by: A. anatomic abnormalities. B. type 1 diabetes mellitus. C. lack of exercise. D. hysterectomy.

ANS: C Lack of exercise is not a cause of amenorrhea. Strenuous exercise may cause amenorrhea. Anatomic abnormalities, type 1 diabetes mellitus, and hysterectomy all are possible causes of amenorrhea.

40. At 1 minute after birth, the nurse assesses the newborn to assign an Apgar score. The apical heart rate is 110 bpm, and the infant is crying vigorously with the limbs flexed. The infant's trunk is pink, but the hands and feet are blue. What is the correct Apgar score for this infant? a. 7 b. 8 c. 9 d. 10

ANS: C The Apgar score is 9 because 1 point is deducted from the total score of 10 for the infant's blue hands and feet. The baby received 2 points for each of the categories except color. Because the infant's hands and feet were blue, this category is given a grade of 1.

12. A woman's cousin gave birth to an infant with a congenital heart anomaly. The woman asks the nurse when such anomalies occur during development. Which response by the nurse is most accurate? A. "We don't really know when such defects occur." B. "It depends on what caused the defect." C. "They occur between the third and fifth weeks of development." D. "They usually occur in the first 2 weeks of development."

ANS: C The cardiovascular system is the first organ system to function in the developing human. Blood vessel and blood formation begins in the third week, and the heart is developmentally complete in the fifth week. "We don't really know when such defects occur" is an inaccurate statement. Regardless of the cause, the heart is vulnerable during its period of development, the third to fifth weeks. "They usually occur in the first 2 weeks of development" is an inaccurate statement.

Which interventions should the nurse plan when caring for a child with a visual impairment (select all that apply)? a. Touch the child upon entering the room before speaking. b. Keep items in the room in the same location. c. Describe the placement of the eating utensils on the meal tray. d. Use color examples to describe something to a child who has been blind since birth. e. Identify noises for the child.

B, C, E

8. A 27-year-old pregnant woman had a preconceptual body mass index (BMI) of 18.0. The nurse knows that this woman's total recommended weight gain during pregnancy should be at least: A. 20 kg (44 lbs). B. 16 kg (35 lbs). C. 12.5 kg (27.5 lbs). D. 10 kg (22 lbs).

ANS: C This woman has a normal BMI and should gain 11.5 to 16 kg during pregnancy. A weight gain of 20 kg would be unhealthy for most women. A weight gain 35 lbs is the high end of the range of weight this woman should gain in her pregnancy. A weight gain of 22 lbs would be appropriate for an obese woman.

2. During pregnancy, many changes occur as a direct result of the presence of the fetus. Which of these adaptations meet this criterion? (Select all that apply.) A. Leukorrhea B. Development of the operculum C. Quickening D. Ballottement E. Lightening

ANS: C, D, E Leukorrhea is a white or slightly gray vaginal discharge that develops in response to cervical stimulation by estrogen and progesterone. Quickening is the first recognition of fetal movements or "feeling life." Quickening is often described as a flutter and is felt earlier in multiparous women than in primiparas. Lightening occurs when the fetus begins to descend into the pelvis. This occurs 2 weeks before labor in the nullipara and at the start of labor in the multipara. Mucus fills the cervical canal creating a plug otherwise known as the operculum. The operculum acts as a barrier against bacterial invasion during the pregnancy. Passive movement of the unengaged fetus is referred to as ballottement.

The nurse is caring for a preschool child with a cast applied recently for a fractured tibia. Which assessment findings indicate possible compartment syndrome? (Select all that apply.) a. Palpable distal pulse b. Capillary refill to extremity less than 3 seconds c. Severe pain not relieved by analgesics d. Tingling of extremity e. Inability to move extremity

ANS: C, D, E Indications of compartment syndrome are severe pain not relieved by analgesics, tingling of extremity, and inability to move extremity. A palpable distal pulse and capillary refill to the extremity less than 3 seconds are expected findings.

The nurse is preparing a child for possible alopecia from chemotherapy. Which should be included? a. Explain to child that hair usually regrows in 1 year. b. Advise child to expose head to sunlight to minimize alopecia. c. Explain to child that wearing a hat or scarf is preferable to wearing a wig. d. Explain to child that when hair regrows, it may have a slightly different color or texture.

ANS: D Alopecia is a side effect of certain chemotherapeutic agents. When the hair regrows, it may be a different color or texture. The hair usually grows back within 3 to 6 months after cessation of treatment. The head should be protected from sunlight to avoid sunburn. Children should choose the head covering they prefer.

8. Although remarkable developments have occurred in reproductive medicine, assisted reproductive therapies are associated with numerous legal and ethical issues. Nurses can provide accurate information about the risks and benefits of treatment alternatives so couples can make informed decisions about their choice of treatment. Which issue would not need to be addressed by an infertile couple before treatment? A. Risks of multiple gestation. B. Whether or how to disclose the facts of conception to offspring. C. Freezing embryos for later use. D. Financial ability to cover the cost of treatment.

ANS: D Although the method of payment is important, obtaining this information is not the responsibility of the nurse. Many states have mandated some form of insurance to assist couples with coverage for infertility. Risk of multiple gestation is a risk of treatment of which the couple needs to be aware. To minimize the chance of multiple gestation, generally only three or fewer embryos are transferred. The couple should be informed that there may be a need for multifetal reduction. Nurses can provide anticipatory guidance on this matter. Depending on the therapy chosen, there may be a need for donor oocytes, sperm, embryos, or a surrogate mother. Couples who have excess embryos frozen for later transfer must be fully informed before consenting to the procedure. A decision must be made regarding the disposal of embryos in the event of death or divorce or if the couple no longer wants the embryos at a future time.

3. When a nulliparous woman telephones the hospital to report that she is in labor, the nurse initially should: a. tell the woman to stay home until her membranes rupture. b. emphasize that food and fluid intake should stop. c. arrange for the woman to come to the hospital for labor evaluation. d. ask the woman to describe why she believes she is in labor.

ANS: D Assessment begins at the first contact with the woman, whether by telephone or in person. By asking the woman to describe her signs and symptoms, the nurse can begin the assessment and gather data. The amniotic membranes may or may not spontaneously rupture during labor. The patient may be instructed to stay home until the uterine contractions become strong and regular. The nurse may want to discuss the appropriate oral intake for early labor such as light foods or clear liquids, depending on the preference of the patient or her primary health care provider. Before instructing the woman to come to the hospital, the nurse should initiate the assessment during the telephone interview.

23. As part of their participation in the gynecologic portion of the physical examination, nurses should: A. take a firm approach that encourages the patient to facilitate the examination by following the physician's instructions exactly. B. explain the procedure as it unfolds and continue to question the patient to get information in a timely manner. C. take the opportunity to explain that the trendy vulvar self-examination is only for women at risk for cancer. D. Help the woman relax through proper placement of her hands and proper breathing during the examination.

ANS: D Breathing techniques are important relaxation techniques that can help the patient during the examination. The nurse should encourage the patient to participate in an active partnership with the care provider. Explanations during the procedure are fine, but many women are uncomfortable answering questions in the exposed and awkward position of the examination. Vulvar self-examination on a regular basis should be encouraged and taught during the examination.

The nurse is conducting a staff in-service on childhood blood disorders. Which describes the pathology of idiopathic thrombocytopenic purpura? a. Bone marrow failure in which all elements are suppressed b. Deficiency in the production rate of globin chains c. Diffuse fibrin deposition in the microvasculature d. An excessive destruction of platelets

ANS: D Idiopathic thrombocytopenic purpura is an acquired hemorrhagic disorder characterized by an excessive destruction of platelets, discolorations caused by petechiae beneath the skin, and a normal bone marrow. Aplastic anemia refers to a bone marrowfailure condition in which the formed elements of the blood are simultaneously depressed. Thalassemia major is a group of blood disorders characterized by deficiency in the production rate of specific hemoglobin globin chains. Disseminated intravascular coagulation is characterized by diffuse fibrin deposition in the microvasculature, consumption of coagulation factors, and endogenous generation of thrombin and plasma.

22. The standard of care for obstetrics dictates that an internal version may be used to manipulate the: a. fetus from a breech to a cephalic presentation before labor begins. b. fetus from a transverse lie to a longitudinal lie before cesarean birth. c. second twin from an oblique lie to a transverse lie before labor begins. d. second twin from a transverse lie to a breech presentation during vaginal birth.

ANS: D Internal version is used only during vaginal birth to manipulate the second twin into a presentation that allows it to be born vaginally. For internal version to occur, the cervix needs to be completely dilated.

19. For the labor nurse, care of the expectant mother begins with any or all of these situations, with the exception of: a. the onset of progressive, regular contractions. b. the bloody, or pink, show. c. the spontaneous rupture of membranes. d. formulation of the woman's plan of care for labor.

ANS: D Labor care begins when progressive, regular contractions begin; the blood-tinged mucoid vaginal discharge appears; or fluid is discharged from the vagina. The woman and nurse can formulate their plan of care before labor or during treatment.

10. Which fetal heart rate (FHR) finding would concern the nurse during labor? a. Accelerations with fetal movement b. Early decelerations c. An average FHR of 126 beats/min d. Late decelerations

ANS: D Late decelerations are caused by uteroplacental insufficiency and are associated with fetal hypoxemia. They are considered ominous if persistent and uncorrected. Accelerations in the FHR are an indication of fetal well-being. Early decelerations in the FHR are associated with head compression as the fetus descends into the maternal pelvic outlet; they generally are not a concern during normal labor.

20. When using intermittent auscultation (IA) for fetal heart rate, nurses should be aware that: a. they can be expected to cover only two or three patients when IA is the primary method of fetal assessment. b. the best course is to use the descriptive terms associated with electronic fetal monitoring (EFM) when documenting results. c. if the heartbeat cannot be found immediately, a shift must be made to EFM. d. ultrasound can be used to find the fetal heartbeat and reassure the mother if initial difficulty was a factor.

ANS: D Locating fetal heartbeats often takes time. Mothers can be reassured verbally and by the ultrasound pictures if ultrasound is used to help locate the heartbeat. When used as the primary method of fetal assessment, auscultation requires a nurse-to-patient ratio of one to one. Documentation should use only terms that can be numerically defined; the usual visual descriptions of EFM are inappropriate.

3. Which symptom is considered a warning sign and should be reported immediately by the pregnant woman to her health care provider? A. Nausea with occasional vomiting B. Fatigue C. Urinary frequency D. Vaginal bleeding

ANS: D Signs and symptoms that must be reported include severe vomiting, fever and chills, burning on urination, diarrhea, abdominal cramping, and vaginal bleeding. These symptoms may be signs of potential complications of the pregnancy. Nausea with occasional vomiting, fatigue, and urinary frequency are normal first-trimester complaints. Although they may be worrisome or annoying to the mother, they usually are not indications of pregnancy problems.

24. With regard to the noncontraceptive medical effects of combined oral contraceptive pills (COCs), nurses should be aware that: A. COCs can cause toxic shock syndrome if the prescription is wrong. B. hormonal withdrawal bleeding usually is a bit more profuse than in normal menstruation and lasts a week. C. COCs increase the risk of endometrial and ovarian cancer. D. the effectiveness of COCs can be altered by some over-the-counter medications and herbal supplements.

ANS: D The effectiveness of COCs can be altered by some over-the-counter medications and herbal supplements. Toxic shock syndrome can occur in some diaphragm users, but it is not a consequence of taking oral contraceptive pills. Hormonal withdrawal bleeding usually is lighter than in normal menstruation and lasts a couple of days. Oral contraceptive pills offer protection against the risk of endometrial and ovarian cancers.

1. A new mother asks the nurse when the "soft spot" on her son's head will go away. What is the nurse's best response, based upon her understanding of when the anterior frontal closes? a. 2 months b. 8 months c. 12 months d. 18 months

ANS: D The larger of the two fontanels, the anterior fontanel, closes by 18 months after birth. The posterior fontanel closes at 6 to 8 weeks. The remaining three options are too early for the anterior fontanel to close.

20. The stimulated release of gonadotropin-releasing hormone and follicle-stimulating hormone is part of the: A. menstrual cycle. B. endometrial cycle. C. ovarian cycle. D. hypothalamic-pituitary cycle.

ANS: D The menstrual, endometrial, and ovarian cycles are interconnected. However, the cyclic release of hormones is the function of the hypothalamus and pituitary glands.

6. A woman in the active phase of the first stage of labor is using a shallow pattern of breathing, which is about twice the normal adult breathing rate. She starts to complain about feeling light-headed and dizzy and states that her fingers are tingling. The nurse should: a. notify the woman's physician. b. tell the woman to slow the pace of her breathing. c. administer oxygen via a mask or nasal cannula. d. help her breathe into a paper bag.

ANS: D This woman is experiencing the side effects of hyperventilation, which include the symptoms of lightheadedness, dizziness, tingling of the fingers, or circumoral numbness. Having the woman breathe into a paper bag held tightly around her mouth and nose may eliminate respiratory alkalosis. This enables her to rebreathe carbon dioxide and replace the bicarbonate ion.

9.A common side effect of corticosteroid therapy is: a. Fever. b. Hypertension. c. Weight loss. d. Increased appetite.

ANS: D Side effects of corticosteroid therapy include an increased appetite. Fever is not a side effect of therapy. It may be an indication of infection. Hypertension is not usually associated with initial corticosteroid therapy. Weight gain, not weight loss, is associated with corticosteroid therapy.

C ("we will check the monitor several times a day to be sure the alarm is working." The parents should check the monitor several times a day to be sure the alarm is working and that it can be heard from room to room. The parents should not adjust the monitor to eliminate false alarms. Adjustments could compromise the monitor's effectiveness. The monitor should be placed on a firm surface away from the crib and drapes. The parents should not sleep in the same bed as the monitored infant.)

An infant experienced an apparent life-threatening event and is being placed on home apnea monitoring. The parents have understood the instructions for use of a home apnea monitor when they state: a. "We can adjust the monitor to eliminate false alarms." b. "We should sleep in the same bed as our monitored infant." c. "We will check the monitor several times a day to be sure the alarm is working." d. "We will place the monitor in the crib with our infant."

Diabetes in pregnancy puts the fetus at risk in several ways. Nurses should be aware that: A. with good control of maternal glucose levels, sudden and unexplained stillbirth is no longer a major concern. B. the most important cause of perinatal loss in diabetic pregnancy is congenital malformations. C. infants of mothers with diabetes have the same risks for respiratory distress syndrome because of the careful monitoring. D. at birth, the neonate of a diabetic mother is no longer at any greater risk.

B. the most important cause of perinatal loss in diabetic pregnancy is congenital malformations. Even with good control, sudden and unexplained stillbirth remains a major concern. Congenital malformations account for 30% to 50% of perinatal deaths. Infants of diabetic mothers are at increased risk for respiratory distress syndrome. The transition to extrauterine life often is marked by hypoglycemia and other metabolic abnormalities.

C (By age 12 months, most children are eating the same food that is prepared for the rest of the family. Using a spoon usually is not mastered until age 18 months. The parents should be engaged in weaning a child from a bottle if that is the source of liquid. Toddlers should be encouraged to drink from a cup at the first birthday and weaned totally by 14 months. The child should be weaned from a milk/formula-based diet to a balanced diet that includes iron-rich sources of food.)

Developmentally, most children at age 12 months: a. Use a spoon adeptly. b. Relinquish the bottle voluntarily. c. Eat the same food as the rest of the family. d. Reject all solid food in preference to the bottle.

C (Changing the infant's position frequently. Changing the infant's position frequently may be beneficial. The parent can walk holding the infant face down and with the infant's chest across the parent's arm. The parent's hand can support the infant's abdomen, applying gentle pressure. Gently massaging the abdomen is effective in some infants. Pacifiers can be used for meeting additional sucking needs. The infant should not be placed where monitoring cannot be done. The infant can be placed in the crib and allowed to cry. Periodically, the infant should be picked up and comforted.)

Parent guidelines for relieving colic in an infant include: a. Avoiding touching the abdomen. b. Avoiding using a pacifier. c. Changing the infant's position frequently. d. Placing the infant where the family cannot hear the crying.

B (storing poisonous substances in a locked cabinet.)

Poisoning in toddlers can best be prevented by: A. consistently using safety caps. B. storing poisonous substances in a locked cabinet. C. keeping ipecac syrup in the home. D. storing poisonous substances out of reach.

D (Able to grasp object voluntarily.)

The nurse expects which characteristic of fine motor skills in a 5-month-old infant? A. Strong grasp reflex B. Neat pincer grasp C. Able to build a tower of two cubes D. Able to grasp object voluntarily

C (Those using yogurt as primary source of milk.)

The nurse in the pediatric clinic identifies which infants at risk for developing vitamin D-deficient rickets? A. Lacto-ovo vegetarians B. Those who are breastfed exclusively C. Those using yogurt as primary source of milk D. Those exposed to daily sunlight

A, B, C, D, E (A. allow for catch-up growth. B. correct nutritional deficiencies. C. achieve ideal weight for height. D. restore optimum body composition. E. educate the parents or primary caregivers on child's nutritional requirements.)

The primary goals in the nutritional management of children with failure to thrive (FTT) are: (Select all that apply.) A. allow for catch-up growth. B. correct nutritional deficiencies. C. achieve ideal weight for height. D. restore optimum body composition. E. educate the parents or primary caregivers on child's nutritional requirements. F. educate the parents or primary caregivers that the child will need tube feedings first.

B, C (Usually by age 30 months, the primary dentition of 20 teeth is completed, and the child has sphincter control in preparation for bowel and bladder control. A doubling of birth weight, opening of the anterior fontanel, and doubling of length are not characteristic of the physical development of a 30-month-old child.)

Which are characteristic of the physical development of a 30-month-old child (select all that apply)? a. Birth weight has doubled. b. Primary dentition is complete. c. Sphincter control is achieved. d. Anterior fontanel is open. e. Length from birth is doubled.

D (The 24-month-old child can go up and down stairs alone with two feet on each step. Skipping and the ability to broad jump are skills acquired at age 3. Tricycle riding is achieved at age 4.)

Which characteristic best describes the gross motor skills of a 24-month-old child? a. Skips b. Rides tricycle c. Broad jumps d. Walks up and down stairs

When providing an infant with a gavage feeding, which of the following should be documented each time? a. The infants abdominal circumference after the feeding b. The infants heart rate and respirations c. The infants suck and swallow coordination d. The infants response to the feeding

ANS: D Documentation of a gavage feeding should include the size of the feeding tube, the amount and quality of the residual from the previous feeding, the type and quantity of the fluid instilled, and the infants response to the procedure. Abdominal circumference is not measured after a gavage feeding. Vital signs may be obtained before feeding. However, the infants response to the feeding is more important. Some older infants may be learning to suck, but the important factor to document would be the infants response to the feeding (including attempts to suck). PTS: 1 DIF: Cognitive Level: Application REF: 700 OBJ: Nursing Process: Evaluation MSC: Client Needs: Physiologic Integrity

The nurse is assessing a 14-year-old child who has recently discovered he or she is adopted. What may be a sign that the child is not handling this information well? 1 Nausea and vomiting 2 Decline in school grades 3 Increased thirst and polyuria 4 Changes in elimination

2 Teenagers do not always verbalize their feelings, so one sign may be a decline in school grades. The child's temper and social behaviors may also be altered. Nausea and vomiting may be a sign of a viral infection. Increased thirst and polyuria are signs diabetes mellitus. Disturbances in elimination are common in infants after stress; however, they are uncommon in adolescence. Test-Taking Tip: The computerized NCLEX exam is an individualized testing experience in which the computer chooses your next question based on the ability and competency you have demonstrated on previous questions. The minimum number of questions will be 75 and the maximum 265. You must answer each question before the computer will present the next question, and you cannot go back to any previously answered questions. Remember that you do not have to answer all of the questions correctly to pass.

2. Which of the following is a complication that may occur after a cardiac catheterization? 1. Cardiac arrhythmia 2. Hypostatic pneumonia 3. Congestive heart failure 4. Rapidly increasing blood pressure

ANS: 1 1. Because a catheter is introduced into the heart, a risk exists of catheter-induced dysrhythmias occurring during the procedure. These are usually transient.

39. When discussing hyperlipidemia with a group of adolescents, the nurse should explain that high levels of what are thought to protect against cardiovascular disease? 1. Cholesterol 2. Triglycerides 3. Low-density lipoproteins (LDLs) 4. High-density lipoproteins (HDLs)

ANS: 4 4. HDLs contain very low concentrations of triglycerides, relatively little cholesterol, and high levels of proteins. It is thought that HDLs protect against cardiovascular disease.

31. A pregnant patient would like to know a good food source of calcium other than dairy products. Your best answer is: A. legumes. B. yellow vegetables. C. lean meat. D. whole grains.

ANS: A Although dairy products contain the greatest amount of calcium, it is also found in legumes, nuts, dried fruits, and some dark green leafy vegetables. Yellow vegetables are rich in vitamin A. Lean meats are rich in protein and phosphorus. Whole grains are rich in zinc and magnesium.

Latex allergy is suspected in a child with spina bifida. Appropriate nursing interventions include which action? a. Avoid using any latex product. b. Use only nonallergenic latex products. c. Administer medication for long-term desensitization. d. Teach family about long-term management of asthma.

ANS: A Care must be taken that individuals who are at high risk for latex allergies do not come in direct or secondary contact with products or equipment containing latex at any time during medical treatment. There are no nonallergenic latex products. At this time, desensitization is not an option. The child does not have asthma. The parents must be taught about allergy and the risk of anaphylaxis.

The nurse is teaching parents about the importance of iron in a toddlers diet. Which explains why iron deficiency anemia is common during toddlerhood? a. Milk is a poor source of iron. b. Iron cannot be stored during fetal development. c. Fetal iron stores are depleted by age 1 month. d. Dietary iron cannot be started until age 12 months.

ANS: A Children between the ages of 12 and 36 months are at risk for anemia because cows milk is a major component of their diet and it is a poor source of iron. Iron is stored during fetal development, but the amount stored depends on maternal iron stores. Fetal iron stores are usually depleted by age 5 to 6 months. Dietary iron can be introduced by breastfeeding, iron-fortified formula, and cereals during the first 12 months of life.

Olivia, age 5 years, tells the nurse that she needs a Band-Aid where she had an injection. The best nursing action is to: a. Apply a Band-Aid. b. Ask her why she wants a Band-Aid. c. Explain why a Band-Aid is not needed. d. Show her that the bleeding has already stopped.

ANS: A Children in this age-group still fear that their insides may leak out at the injection site, even if the bleeding has stopped. Provide the Band-Aid. No explanation should be required.

18. A normal uterine activity pattern in labor is characterized by: a. contractions every 2 to 5 minutes. b. contractions lasting about 2 minutes. c. contractions about 1 minute apart. d. a contraction intensity of about 1000 mm Hg with relaxation at 50 mm Hg.

ANS: A Contractions normally occur every 2 to 5 minutes and last less than 90 seconds (intensity 800 mm Hg) with about 30 seconds in between (20 mm Hg or less).

7. A woman is experiencing back labor and complains of intense pain in her lower back. An effective relief measure would be to use: a. counterpressure against the sacrum. b. pant-blow (breaths and puffs) breathing techniques. c. effleurage. d. conscious relaxation or guided imagery.

ANS: A Counterpressure is a steady pressure applied by a support person to the sacral area with the fist or heel of the hand. This technique helps the woman cope with the sensations of internal pressure and pain in the lower back. The pain-management techniques of pant-blow, effleurage, and conscious relaxation or guided imagery are usually helpful for contractions per the gate-control theory.

Nurses can prevent evaporative heat loss in the newborn by: a. Drying the baby after birth and wrapping the baby in a dry blanket. b. Keeping the baby out of drafts and away from air conditioners. c. Placing the baby away from the outside wall and the windows. d. Warming the stethoscope and the nurses hands before touching the baby.

ANS: A Because the infant is wet with amniotic fluid and blood, heat loss by evaporation occurs quickly. Heat loss by convection occurs when drafts come from open doors and air currents created by people moving around. If the heat loss is caused by placing the baby near cold surfaces or equipment, it is referred to as a radiation heat loss. Conduction heat loss occurs when the baby comes in contact with cold objects or surfaces. PTS: 1 DIF: Cognitive Level: Comprehension REF: 561 OBJ: Nursing Process: Assessment MSC: Client Needs: Physiologic Integrity

Knowing that the condition of the new mothers breasts will be affected by whether she is breastfeeding, nurses should be able to tell their clients all the following statements except: a. Breast tenderness is likely to persist for about a week after the start of lactation. b. As lactation is established, a mass may form that can be distinguished from cancer by its position shift from day to day. c. In nonlactating mothers colostrum is present for the first few days after childbirth. d. If suckling is never begun (or is discontinued), lactation ceases within a few days to a week.

ANS: A Breast tenderness should persist for 24 to 48 hours after lactation begins. That movable, noncancerous mass is a filled milk sac. Colostrum is present for a few days whether the mother breastfeeds or not. A mother who does not want to breastfeed should also avoid stimulating her nipples. PTS: 1 DIF: Cognitive Level: Comprehension REF: 487 OBJ: Nursing Process: Planning, Implementation MSC: Client Needs: Health Promotion and Maintenance

10. An infants parents ask the nurse about preventing otitis media (OM). What should the nurse recommend? a. Avoid tobacco smoke. b. Use nasal decongestant. c. Avoid children with OM. d. Bottle-feed or breastfeed in supine position.

ANS: A Eliminating tobacco smoke from the childs environment is essential for preventing OM and other common childhood illnesses. Nasal decongestants are not useful in preventing OM. Children with uncomplicated OM are not contagious unless they show other upper respiratory infection symptoms. Children should be fed in an upright position to prevent OM

9. A goiter is an enlargement or hypertrophy of which gland? a. Thyroid b. Adrenal c. Anterior pituitary d. Posterior pituitary

ANS: A A goiter is an enlargement or hypertrophy of the thyroid gland. Goiter is not associated with the adrenals or the anterior and posterior pituitaries.

1. Nurses must be alert for increased fluid requirements when a child presents with which possible concern? a. Fever b. Mechanical ventilation c. Congestive heart failure d. Increased intracranial pressure (ICP)

ANS: A Fever leads to great insensible fluid loss in young children because of increased body surface area relative to fluid volume. Respiratory rate influences insensible fluid loss and should be monitored in the mechanically ventilated child. Congestive heart failure is a case of fluid overload in children. ICP does not lead to increased fluid requirements in children.

In a variation of rooming-in, called couplet care, the mother and infant share a room, and the mother shares the care of the infant with: a. The father of the infant. b. Her mother (the infants grandmother). c. Her eldest daughter (the infants sister). d. The nurse.

ANS: D In couplet care the mother shares a room with the newborn and shares infant care with a nurse educated in maternity and infant care. PTS: 1 DIF: Cognitive Level: Knowledge REF: 492 OBJ: Nursing Process: Implementation MSC: Client Needs: Health Promotion and Maintenance

7. The nurse is administering an IV chemotherapeutic agent to a child diagnosed with leukemia. The child suddenly begins to wheeze and have severe urticaria. Which is the most appropriate nursing action? a. Stop drug infusion immediately. b. Recheck rate of drug infusion. c. Observe child closely for next 10 minutes. d. Explain to child that this is an expected side effect.

ANS: A If an allergic reaction is suspected, the drug should be immediately discontinued. Any drug in the line should be withdrawn, and a normal saline infusion begun to keep the line open. Rechecking the rate of drug infusion, observing the child closely for next 10 minutes, and explaining to the child that this is an expected side effect can all be done after the drug infusion is stopped and the child is evaluated.

17. The nurse is taking care of an adolescent with osteosarcoma. The parents ask the nurse about treatment. The nurse should make which accurate response about treatment for osteosarcoma? a. Treatment usually consists of surgery and chemotherapy. b. Amputation of the affected extremity is rarely necessary. c. Intensive irradiation is the primary treatment. d. Bone marrow transplantation offers the best chance of long-term survival.

ANS: A The optimal therapy for osteosarcoma is a combination of surgery and chemotherapy. Intensive irradiation and bone marrow transplantation are usually not part of the therapeutic management.

The nurse is caring for an infant with developmental dysplasia of the hip. Which clinical manifestations should the nurse expect to observe? (Select all that apply.) a. Positive Ortolani click b. Unequal gluteal folds c. Negative Babinski sign d. Trendelenburg sign e. Telescoping of the affected limb f. Lordosis

ANS: A, B A positive Ortolani test and unequal gluteal folds are clinical manifestations of developmental dysplasia of the hip seen from birth to 2 to 3 months. Unequal gluteal folds, negative Babinski sign, and Trendelenburg sign are signs that appear in older infants and children. Telescoping of the affected limb and lordosis are not clinical manifestations of developmental dysplasia of the hip.

The nurse is evaluating the laboratory results on cerebral spinal fluid (CSF) from a 3-year-old child with bacterial meningitis. Which findings confirm bacterial meningitis? (Select all that apply.) a. Elevated white blood cell (WBC) count b. Decreased glucose c. Normal protein d. Elevated red blood cell (RBC) count

ANS: A, B The cerebrospinal fluid analysis in bacterial meningitis shows elevated WBC count, decreased glucose, and increased protein content. There should not be RBCs evident in the CSF fluid.

Risk factors associated with necrotizing enterocolitis (NEC) include (Select all that apply): a. Polycythemia. b. Anemia. c. Congenital heart disease. d. Bronchopulmonary dysphasia. e. Retinopathy.

ANS: A, B, C Risk factors for NEC include asphyxia, respiratory distress syndrome, umbilical artery catheterization, exchange transfusion, early enteral feedings, patent ductus arteriosus, congenital heart disease, polycythemia, anemia, shock, and gastrointestinal infection. Bronchopulmonary dysphasia and retinopathy are not associated with NEC. PTS: 1 DIF: Cognitive Level: Comprehension REF: 693 OBJ: Nursing Process: Assessment MSC: Client Needs: Physiologic Integrity

1. Intrauterine growth restriction (IUGR) is associated with numerous pregnancy-related risk factors. (Select all that apply.) A. Poor material weight gain B. Chronic maternal infections C. Gestational hypertension D. Premature rupture of membranes E. Smoking

ANS: A, B, C, E Poor material weight gain, chronic infections disease, gestational hypertension, and smoking are all risk factors associated with IUGR. Premature rupture of membranes is associated with preterm labor, not IUGR.

1. Nursing care of a child diagnosed with syndrome of inappropriate antidiuretic hormone (SIADH) should include which interventions? (Select all that apply.) a. Weigh daily b. Encourage fluids c. Turn frequently d. Maintain nothing by mouth e. Restrict fluids

ANS: A, E Increased secretion of ADH causes the kidney to resorb water, which increases fluid volume and decreases serum osmolarity with a progressive reduction in sodium concentration. The immediate management of the child is to restrict fluids. The child should also be weighed at the same time each day. Encouraging fluids, turning frequently, and maintaining nothing by mouth are not associated with SIADH since they are not associated with managing/monitoring for fluid retention.

3. Which symptom described by a patient is characteristic of premenstrual syndrome (PMS)? A. "I feel irritable and moody a week before my period is supposed to start." B. "I have lower abdominal pain beginning the third day of my menstrual period." C. "I have nausea and headaches after my period starts, and they last 2 to 3 days." D. "I have abdominal bloating and breast pain after a couple days of my period."

ANS: APMS is a cluster of physical, psychologic, and behavioral symptoms that begin in the luteal phase of the menstrual cycle and resolve within a couple of days of the onset of menses. Complaints of lower abdominal pain, nausea and headaches, and abdominal bloating all are associated with PMS. However, the timing reflected is inaccurate.

17. A new patient and her partner arrive in the labor, delivery, recovery, and after birth unit for the birth of their first child. You apply the electronic fetal monitor (EFM) to the woman. Her partner asks you to explain what is printing on the graph, referring to the EFM strip. He wants to know what the baby's heart rate should be. Your best response is: a. "Don't worry about that machine; that's my job." b. "The top line graphs the baby's heart rate. Generally the heart rate is between 110 and 160. The heart rate will fluctuate in response to what is happening during labor." c. "The top line graphs the baby's heart rate, and the bottom line lets me know how strong the contractions are." d. "Your doctor will explain all of that later."

ANS: B "The top line graphs the baby's heart rate. Generally the heart rate is between 110 and 160. The heart rate will fluctuate in response to what is happening during labor" educates the partner about fetal monitoring and provides support and information to alleviate his fears. "Don't worry about that machine; that's my job" discredits the partner's feelings and does not provide the teaching he is requesting. "The top line graphs the baby's heart rate, and the bottom line lets me know how strong the contractions are" provides inaccurate information and does not address the partner's concerns about the fetal heart rate. The EFM graphs the frequency and duration of the contractions, not the intensity. Nurses should take every opportunity to provide patient and family teaching, especially when information is requested.

26. The nerve block used in labor that provides anesthesia to the lower vagina and perineum is called: a. an epidural. b. a pudendal. c. a local. d. a spinal block.

ANS: B A pudendal block anesthetizes the lower vagina and perineum to provide anesthesia for an episiotomy and use of low forceps if needed. An epidural provides anesthesia for the uterus, perineum, and legs. A local provides anesthesia for the perineum at the site of the episiotomy. A spinal block provides anesthesia for the uterus, perineum, and down the legs.

23. After change-of-shift report the nurse assumes care of a multiparous patient in labor. The woman is complaining of pain that radiates to her abdominal wall, lower back, and buttocks and down her thighs. Before implementing a plan of care, the nurse should understand that this type of pain is: a. visceral. b. referred. c. somatic. d. afterpain.

ANS: B As labor progresses the woman often experiences referred pain. This occurs when pain that originates in the uterus radiates to the abdominal wall, the lumbosacral area of the back, the gluteal area, and thighs. The woman usually has pain only during a contraction and is free from pain between contractions. Visceral pain is that which predominates in the first stage of labor. This pain originates from cervical changes, distention of the lower uterine segment, and uterine ischemia. Visceral pain is located over the lower portion of the abdomen. Somatic pain is described as intense, sharp, burning, and well localized. This results from stretching of the perineal tissues and the pelvic floor. This occurs during the second stage of labor. Pain experienced during the third stage of labor or afterward during the early after birth period is uterine. This pain is very similar to that experienced in the first stage of labor. P

14. While providing care in an obstetric setting, the nurse should understand that after birth care of the woman with cardiac disease: A. is the same as that for any pregnant woman. B. includes rest, stool softeners, and monitoring of the effect of activity. C. includes ambulating frequently, alternating with active range of motion. D. includes limiting visits with the infant to once per day.

ANS: B Bed rest may be ordered, with or without bathroom privileges. Bowel movements without stress or strain for the woman are promoted with stool softeners, diet, and fluid. Care of the woman with cardiac disease in the after birth period is tailored to the woman's functional capacity. The woman will be on bed rest to conserve energy and reduce the strain on the heart. Although the woman may need help caring for the infant, breastfeeding and infant visits are not contraindicated.

18. The viral sexually transmitted infection (STI) that affects most people in the United States today is: A. herpes simplex virus type 2 (HSV-2). B. human papillomavirus (HPV). C. human immunodeficiency virus (HIV). D. cytomegalovirus (CMV).

ANS: B HPV infection is the most prevalent viral STI seen in ambulatory health care settings. HSV-2, HIV, and CMV all are viral STIs but are not the most prevalent viral STIs.

3. The nurse teaches a pregnant woman about the presumptive, probable, and positive signs of pregnancy. The woman demonstrates understanding of the nurse's instructions if she states that a positive sign of pregnancy is: A. a positive pregnancy test. B. fetal movement palpated by the nurse-midwife. C. Braxton Hicks contractions. D. quickening.

ANS: B Positive signs of pregnancy are attributed to the presence of a fetus, such as hearing the fetal heartbeat or palpating fetal movement. A positive pregnancy test and Braxton Hicks contractions are probable signs of pregnancy. Quickening is a presumptive sign of pregnancy.

18. The body part that both protects the pelvic structures and accommodates the growing fetus during pregnancy is the: A. perineum. B. bony pelvis. C. vaginal vestibule. D. fourchette.

ANS: B The bony pelvis protects and accommodates the growing fetus. The perineum covers the pelvic structures. The vaginal vestibule contains openings to the urethra and vagina. The fourchette is formed by the labia minor.

In follow-up appointments or visits with parents and their new baby, it may be useful if the nurse can identify parental behaviors that can either facilitate or inhibit attachment. Which one is a facilitating behavior? a. The parents have difficulty naming the infant. b. The parents hover around the infant, directing attention to and pointing at the infant. c. The parents make no effort to interpret the actions or needs of the infant. d. The parents do not move from fingertip touch to palmar contact and holding.

ANS: B Hovering over the infant and obviously paying attention to the baby are facilitating behaviors. Inhibiting behaviors include difficulty naming the infant, making no effort to interpret the actions or needs of the infant, and not moving from fingertip touch to palmar contact and holding. PTS: 1 DIF: Cognitive Level: Comprehension REF: 511 OBJ: Nursing Process: Assessment MSC: Client Needs: Health Promotion and Maintenance

With regard to small for gestational age (SGA) infants and intrauterine growth restrictions (IUGR), nurses should be aware that: a. In the first trimester diseases or abnormalities result in asymmetric IUGR. b. Infants with asymmetric IUGR have the potential for normal growth and development. c. In asymmetric IUGR weight is slightly more than SGA, whereas length and head circumference are somewhat less than SGA. d. Symmetric IUGR occurs in the later stages of pregnancy.

ANS: B IUGR is either symmetric or asymmetric. The symmetric form occurs in the first trimester; SGA infants have reduced brain capacity. The asymmetric form occurs in the later stages of pregnancy. Weight is less than the 10th percentile; head circumference is greater than the 10th percentile. Infants with asymmetric IUGR have the potential for normal growth and development. PTS: 1 DIF: Cognitive Level: Comprehension REF: 664 OBJ: Nursing Process: Planning MSC: Client Needs: Health Promotion and Maintenance

6. A nurse is caring for a child who is scheduled to have intrathecal chemotherapy today. Which action by the nurse is most important when providing care to this patient and family? A. Educating family on side effects of chemotherapy B. Ensuring a signed consent is on the chart C. Providing distraction techniques during the process D. Reassuring the child the parents will be present

ANS: B Intrathecal chemotherapy (introducing chemotherapy into the subarachnoid space of the spinal cord) is an invasive procedure and requires a signed consent. Although all actions are important for this child, the priority is ensuring the consent is executed appropriately and on the chart.

An examiner who discovers unequal movement or uneven gluteal skin folds during the Ortolani maneuver would then: a. Tell the parents that one leg may be longer than the other, but they will equal out by the time the infant is walking. b. Alert the physician that the infant has a dislocated hip. c. Inform the parents and physician that molding has not taken place. d. Suggest that, if the condition does not change, surgery to correct vision problems may be needed.

ANS: B The Ortolani maneuver is a technique for checking hip integrity. Unequal movement suggests that the hip is dislocated. The physician should be notified. PTS: 1 DIF: Cognitive Level: Application REF: 572 OBJ: Nursing Process: Implementation MSC: Client Needs: Health Promotion and Maintenance

3.Which factor predisposes a child to urinary tract infections? a. Increased fluid intake b. Short urethra in young girls c. Prostatic secretions in males d. Frequent emptying of the bladder

ANS: B The short urethra in females provides a ready pathway for invasions of organisms. Increased fluid intake and frequent bladder emptying offer protective measures against urinary tract infections. Prostatic secretions have antibacterial properties that inhibit bacteria.

21. b-Adrenergic agonists and methylxanthines are often prescribed for a child with an asthma attack. What is their action? a. Liquefy secretions c. Reduce inflammation of the lungs b. Dilate the bronchioles d. Reduce infection

ANS: B These medications work to dilate the bronchioles in acute exacerbations. These medications do not liquefy secretions or reduce infection. Corticosteroids and mast cell stabilizers reduce inflammation in the lungs.

12. A child is admitted with acute glomerulonephritis. The nurse would expect the urinalysis during this acute phase to show: a. Bacteriuria and hematuria. b. Hematuria and proteinuria. c. Bacteriuria and increased specific gravity. d. Proteinuria and decreased specific gravity.

ANS: B Urinalysis during the acute phase characteristically shows hematuria and proteinuria. Bacteriuria and changes in specific gravity are not usually present during the acute phase.

Which description of postpartum restoration or healing times is accurate? a. The cervix shortens, becomes firm, and returns to form within a month postpartum. b. Vaginal rugae reappear by 3 weeks postpartum. c. Most episiotomies heal within a week. d. Hemorrhoids usually decrease in size within 2 weeks of childbirth.

ANS: B Vaginal rugae reappear by 3 weeks postpartum; however, they are never as prominent as in nulliparous women. The cervix regains its form within days; the cervical os may take longer. Most episiotomies take 2 to 3 weeks to heal. Hemorrhoids can take 6 weeks to decrease in size. PTS: 1 DIF: Cognitive Level: Comprehension REF: 485 OBJ: Nursing Process: Planning MSC: Client Needs: Health Promotion and Maintenance

14. Chronic adrenocortical insufficiency is also referred to as what? a. Graves' disease b. Addison's disease c. Cushing's syndrome d. Hashimoto's disease

ANS: B Addison's disease is chronic adrenocortical insufficiency. Graves' and Hashimoto's diseases involve the thyroid gland. Cushing's syndrome is a result of excessive circulation of free cortisol.

27. Which laboratory finding confirms that a child with type 1 diabetes is experiencing diabetic ketoacidosis? a. No urinary ketones b. Low arterial pH c. Elevated serum carbon dioxide d. Elevated serum phosphorus

ANS: B Severe insulin deficiency produces metabolic acidosis, which is indicated by a low arterial pH. Urinary ketones, often in large amounts, are present when a child is in diabetic ketoacidosis. Serum carbon dioxide is decreased in diabetic ketoacidosis. Serum phosphorus is decreased in diabetic ketoacidosis.

32. Which vaccine is now recommended for the immunization of all newborns? a. Hepatitis A vaccine b. Hepatitis B vaccine c. Hepatitis C vaccine d. Hepatitis A, B, and C vaccines

ANS: B Universal vaccination for hepatitis B is now recommended for all newborns. A vaccine is available for hepatitis A, but it is not yet universally recommended. No vaccine is currently available for hepatitis C. Only hepatitis B vaccine is recommended for newborns.

What are modes of heat loss in the newborn (Select all that apply)? a. Perspiration b. Convection c. Radiation d. Conduction e. Urination

ANS: B, C, D Convection, radiation, evaporation, and conduction are the four modes of heat loss in the newborn. Perspiration and urination are not modes of heat loss in newborns. PTS: 1 DIF: Cognitive Level: Analysis REF: 561 OBJ: Nursing Process: Diagnosis MSC: Client Needs: Health Promotion and Maintenance

1. Complications and risks associated with cesarean births include: (Select all that apply.) a. placental abruption. b. wound dehiscence. c. hemorrhage. d. urinary tract infections. e. fetal injuries.

ANS: B, C, D, E Placental abruption and placenta previa are both indications for cesarean birth and are not complications thereof. Wound dehiscence, hemorrhage, urinary tract infection, and fetal injuries are all possible complications and risks associated with delivery by cesarean section.

14. A nurse has a RN preceptor student working on the pediatric oncology unit. When teaching the student about oncological crises, what disorders does the nurse include? (Select all that apply.) A. Inferior vena cava infarction B. Neurogenic shock C. Perirectal abscess D. Pleural effusion E. Superior vena cava syndrome

ANS: B, C, D, E There are many oncologic emergencies, including neurogenic shock (and other types of shock), perirectal abscess, pleural effusion, and superior vena cava syndrome. Inferior vena cava infarction is not on the list of emergent conditions.

The nurse should recommend medical attention if a child with a slight head injury experiences: a. sleepiness. b. vomiting, even once. c. headache, even if slight. d. confusion or abnormal behavior.

ANS: D Medical attention should be sought if the child exhibits confusion or abnormal behavior, loses consciousness, has amnesia, has fluid leaking from the nose or ears, complains of blurred vision, or has an unsteady gait. Sleepiness alone does not require evaluation. If the child is difficult to arouse from sleep, medical attention should be obtained. Vomiting more than three times requires medical attention. Severe or worsening headache or one that interferes with sleep should be evaluated.

1. A tiered system of categorizing FHR has been recommended by regulatory agencies. Nurses, midwives, and physicians who care for women in labor must have a working knowledge of fetal monitoring standards and understand the significance of each category. These categories include: (Select all that apply.) a. reassuring. b. Category I. c. Category II. d. nonreassuring. e. Category III.

ANS: B, C, E The three tiered system of FHR tracings include Category I, II, and III. Category I is a normal tracing requiring no action. Category II FHR tracings are indeterminate. This category includes tracings that do not meet Category I or III criteria. Category III tracings are abnormal and require immediate intervention.

15. A pediatric patient is receiving asparaginase (Elspar). What manifestations would lead the nurse to determine that the child is having a possible side effect from this drug? (Select all that apply.) A. Blistering at infusion site B. Increased PT and INR C. Potassium of 2.7 mEq/L D. Seizures E. Shortness of breath

ANS: B, D Some common side effects of Elspar include seizures, hyperglycemia, nausea/vomiting, rashes, coagulation abnormalities, hepatotoxicity, pancreatitis, and anaphylaxis. Blistering is common with daunorubicin (Daunomycin). Hypokalemia is seen with carboplatin (Paraplatin). Shortness of breath could be seen with bleomycin (Blenoxane), which causes pulmonary fibrosis and pneumonitis.

5. A nurse is caring for several patients with acute lymphocytic leukemia (ALL). Which children does the nurse understand have the best prognosis? A. Infant B. < 10 years of age C. > 25% abnormal cells in bone marrow aspirate D. White count 4,200/mm3 E. White count 25,000/mm3

ANS: B, D The best prognosis for ALL occurs in children 2 to 9 years of age and in children whose initial white blood cell count is < 5,000/mm3. Children 10 and older and whose initial white blood cell counts are ?= 50,000/mm3 have worse prognoses. Infants have a very poor prognosis.

The nurse is monitoring an infant for signs of increased intracranial pressure (ICP). Which are late signs of increased intracranial pressure (ICP) in an infant? (Select all that apply.) a. Tachycardia b. Alteration in pupil size and reactivity c. Increased motor response d. Extension or flexion posturing e. Cheyne-Stokes respirations

ANS: B, D, E Late signs of ICP in an infant or child include bradycardia, alteration in pupil size and reactivity, decreased motor response, extension or flexion posturing, and Cheyne-Stokes respirations.

19. With regard to the initial visit with a patient who is beginning prenatal care, nurses should be aware that: A. the first interview is a relaxed, get-acquainted affair in which nurses gather some general impressions. B. if nurses observe handicapping conditions, they should be sensitive and not enquire about them because the patient will do that in her own time. C. nurses should be alert to the appearance of potential parenting problems, such as depression or lack of family support. D. because of legal complications, nurses should not ask about illegal drug use; that is left to physicians.

ANS: C Besides these potential problems, nurses need to be alert to the woman's attitude toward health care. The initial interview needs to be planned, purposeful, and focused on specific content. A lot of ground must be covered. Nurses must be sensitive to special problems, but they do need to inquire because discovering individual needs is important. People with chronic or handicapping conditions forget to mention them because they have adapted to them. Getting information on drug use is important and can be done confidentially. Actual testing for drug use requires the patient's consent.

2. The nurse is providing genetic counseling for an expectant couple who already have a child with trisomy 18. The nurse should: A. tell the couple they need to have an abortion within 2 to 3 weeks. B. explain that the fetus has a 50% chance of having the disorder. C. discuss options with the couple, including amniocentesis to determine whether the fetus is affected. D. refer the couple to a psychologist for emotional support.

ANS: C Genetic testing, including amniocentesis, would need to be performed to determine whether the fetus is affected. The couple should be given information about the likelihood of having another baby with this disorder so that they can make an informed decision. A genetic counselor is the best source for determining genetic probability ratios. The couple eventually may need emotional support, but the status of the pregnancy must be determined first.

What is the best action for the nurse to take when a 5-year-old child who requires another 2 days of intravenous (IV) antibiotics cries, screams, and resists having the IV restarted? a. Exit the room and leave the child alone until he stops crying. b. Tell the child big boys and girls dont cry. c. Let the child decide which color arm board to use with the IV. d. Administer a narcotic analgesic for pain to quiet the child.

ANS: C Giving the preschooler some choice and control, while maintaining boundaries of treatment, supports the childs coping skills. Leaving the child alone robs the child of support when a coping difficulty exists. Crying is a normal response to stress. The child needs time to adjust and support to cope with unfamiliar and painful procedures during hospitalization. Although administration of a topical analgesic is indicated before restarting the childs IV, a narcotic analgesic is not indicated.

7. The nurse caring for the woman in labor should understand that maternal hypotension can result in: a. early decelerations. b. fetal dysrhythmias. c. uteroplacental insufficiency. d. spontaneous rupture of membranes.

ANS: C Low maternal blood pressure reduces placental blood flow during uterine contractions and results in fetal hypoxemia. Maternal

18. Which heart condition is not a contraindication for pregnancy? A. Peripartum cardiomyopathy B. Eisenmenger syndrome C. Heart transplant D. All of these contraindicate pregnancy.

ANS: C Pregnancy is contraindicated for peripartum cardiomyopathy and Eisenmenger syndrome. Women who have had heart transplants are successfully having babies. However, conception should be postponed for at least 1 year after transplantation.

26. The nurse should know that once human immunodeficiency virus (HIV) enters the body, seroconversion to HIV positivity usually occurs within: A. 6 to 10 days. B. 2 to 4 weeks. C. 6 to 12 weeks. D. 6 months.

ANS: C Seroconversion to HIV positivity usually occurs within 6 to 12 weeks after the virus has entered the body.

27. The microscopic examination of scrapings from the cervix, endocervix, or other mucous membranes to detect premalignant or malignant cells is called: A. bimanual palpation. B. rectovaginal palpation. C. a Papanicolaou (Pap) test. D. a four As procedure.

ANS: C The Pap test is a microscopic examination for cancer that should be performed regularly, depending on the patient's age. Bimanual palpation is a physical examination of the vagina. Rectovaginal palpation is a physical examination performed through the rectum. The four As is an intervention procedure to help a patient stop smoking.

A postpartum woman overhears the nurse tell the obstetrics clinician that she has a positive Homans sign and asks what it means. The nurses best response is: a. You have pitting edema in your ankles. b. You have deep tendon reflexes rated 2+. c. You have calf pain when the nurse flexes your foot. d. You have a fleshy odor to your vaginal drainage.

ANS: C Discomfort in the calf with sharp dorsiflexion of the foot may indicate deep vein thrombosis. Edema is within normal limits for the first few days until the excess interstitial fluid is remobilized and excreted. Deep tendon reflexes should be 1+ to 2+. A fleshy odor, not a foul odor, is within normal limits. PTS: 1 DIF: Cognitive Level: Knowledge REF: 494 OBJ: Nursing Process: Implementation MSC: Client Needs: Physiologic Integrity

While assessing the integument of a 24-hour-old newborn, the nurse notes a pink, papular rash with vesicles superimposed on the thorax, back, and abdomen. The nurse should: a. Notify the physician immediately. b. Move the newborn to an isolation nursery. c. Document the finding as erythema toxicum. d. Take the newborns temperature and obtain a culture of one of the vesicles.

ANS: C Erythema toxicum (or erythema neonatorum) is a newborn rash that resembles flea bites. This is a normal finding that does not require notification of the physician, isolation of the newborn, or any additional interventions. PTS: 1 DIF: Cognitive Level: Application REF: 568 OBJ: Nursing Process: Assessment MSC: Client Needs: Health Promotion and Maintenance

As related to laboratory tests and diagnostic tests in the hospital after birth, nurses should be aware that: a. All states test for phenylketonuria (PKU), hypothyroidism, cystic fibrosis, and sickle cell diseases. b. Federal law prohibits newborn genetic testing without parental consent. c. If genetic screening is done before the infant is 24 hours old, it should be repeated at age 1 to 2 weeks. d. Hearing screening is now mandated by federal law.

ANS: C If done very early, genetic screening should be repeated. States all test for PKU and hypothyroidism, but other genetic defects are not universally covered. Federal law mandates newborn genetic screening, but not screening for hearing problems (although more than half the states do mandate hearing screening). PTS: 1 DIF: Cognitive Level: Knowledge REF: 609 OBJ: Nursing Process: Planning MSC: Client Needs: Physiologic Integrity

Anxiety disorders are the most common mental disorders that affect women. While providing care to the maternity patient, the nurse should be aware that one of these disorders is likely to be triggered by the process of labor and birth. This disorder is: a. Phobias. b. Panic disorder. c. Post-traumatic stress disorder (PTSD). d. Obsessive-compulsive disorder (OCD).

ANS: C In PTSD, women perceive childbirth as a traumatic event. They have nightmares and flashbacks about the event, anxiety, and avoidance of reminders of the traumatic event. Phobias are irrational fears that may lead a person to avoid certain objects, events, or situations. Panic disorders include episodes of intense apprehension, fear, and terror. Symptoms may manifest themselves as palpitations, chest pain, choking, or smothering. OCD symptoms include recurrent, persistent, and intrusive thoughts. The mother may repeatedly check and recheck her infant once he or she is born, even though she realizes that this is irrational. OCD is best treated with medications. PTS: 1 DIF: Cognitive Level: Knowledge REF: 552 OBJ: Nursing Process: Assessment MSC: Client Needs: Psychosocial Integrity

The nurse is teaching a family how to care for their infant in a Pavlik harness to treat developmental dysplasia of the hip. Which should be included? a. Apply lotion or powder to minimize skin irritation. b. Remove harness several times a day to prevent contractures. c. Return to clinic every 1 to 2 weeks. d. Place diaper over harness, preferably using a superabsorbent disposable diaper that is relatively thin.

ANS: C Infants have a rapid growth pattern. The child needs to be assessed by the practitioner every 1 to 2 weeks for possible adjustments. Lotions and powders should not be used with the harness. The harness should not be removed, except as directed by the practitioner. A thin disposable diaper can be placed under the harness.

The nurse practicing in the perinatal setting should promote kangaroo care regardless of an infants gestational age. This intervention: a. Is adopted from classical British nursing traditions. b. Helps infants with motor and central nervous system impairment. c. Helps infants to interact directly with their parents and enhances their temperature regulation. d. Gets infants ready for breastfeeding.

ANS: C Kangaroo care is skin-to-skin holding in which the infant, dressed only in a diaper, is placed directly on the parents bare chest and then covered. The procedure helps infants interact with their parents and regulates their temperature, among other developmental benefits. PTS: 1 DIF: Cognitive Level: Knowledge REF: 699 OBJ: Nursing Process: Implementation MSC: Client Needs: Health Promotion and Maintenance

With regard to basic care of the breastfeeding mother, nurses should be able to advise her that she: a. Will need an extra 1000 calories a day to maintain energy and produce milk. b. Can go back to prepregnancy consumption patterns of any drinks, as long as she ingests enough calcium. c. Should avoid trying to lose large amounts of weight. d. Must avoid exercising because it is too fatiguing.

ANS: C Large weight loss would release fat-stored contaminants into her breast milk. It would also likely involve eating too little and/or exercising too much. A breastfeeding mother need add only 200 to 500 extra calories to her diet to provide extra nutrients for the infant. The mother can go back to her consumption patterns of any drinks as long as she ingests enough calcium, only if she does not drink alcohol, limits coffee to no more than two cups (caffeine in chocolate, tea, and some sodas), and reads the herbal tea ingredients carefully. The mother needs her rest, but moderate exercise is healthy. PTS: 1 DIF: Cognitive Level: Comprehension REF: 651 OBJ: Nursing Process: Planning MSC: Client Needs: Physiologic Integrity

A meconium stool can be differentiated from a transitional stool in the newborn because the meconium stool is: a. Seen at age 3 days. b. The residue of a milk curd. c. Passed in the first 12 hours of life. d. Lighter in color and looser in consistency.

ANS: C Meconium stool is usually passed in the first 12 hours of life, and 99% of newborns have their first stool within 48 hours. If meconium is not passed by 48 hours, obstruction is suspected. Meconium stool is the first stool of the newborn and is made up of matter remaining in the intestines during intrauterine life. Meconium is dark and sticky. PTS: 1 DIF: Cognitive Level: Knowledge REF: 563 OBJ: Nursing Process: Assessment MSC: Client Needs: Health Promotion and Maintenance

A new mother wants to be sure that she is meeting her daughters needs while feeding her commercially prepared infant formula. The nurse should evaluate the mothers knowledge about appropriate infant care. The mother meets her childs needs when she: a. Adds rice cereal to her formula at 2 weeks of age to ensure adequate nutrition. b. Warms the bottles using a microwave oven. c. Burps her infant during and after the feeding as needed. d. Refrigerates any leftover formula for the next feeding.

ANS: C Most infants swallow air when fed from a bottle and should be given a chance to burp several times during a feeding and after the feeding. Solid food should not be introduced to the infant for at least 4 to 6 months after birth. A microwave should never be used to warm any food to be given to an infant. The heat is not distributed evenly, and this may pose a risk of burning the infant. Any formula left in the bottle after the feeding should be discarded because the infants saliva has mixed with it. PTS: 1 DIF: Cognitive Level: Comprehension REF: 656 OBJ: Nursing Process: Evaluation MSC: Client Needs: Health Promotion and Maintenance

Which options for saying goodbye would the nurse want to discuss with a woman who is diagnosed with having a stillborn girl? a. The nurse shouldnt discuss any options at this time; there is plenty of time after the baby is born. b. Would you like a picture taken of your baby after birth? c. When your baby is born, would you like to see and hold her? d. What funeral home do you want notified after the baby is born?

ANS: C Mothers and fathers may find it helpful to see the infant after delivery. The parents wishes should be respected. Interventions and support from the nursing and medical staff after a prenatal loss are extremely important in the healing of the parents. Although this may be an intervention, the initial intervention should be related directly to the parents wishes with regard to seeing or holding their dead infant. Although information about funeral home notification may be relevant, it is not the most appropriate option at this time. Burial arrangements can be discussed after the infant is born. PTS: 1 DIF: Cognitive Level: Application REF: 553 OBJ: Nursing Process: Planning MSC: Client Needs: Psychosocial Integrity

With regard to parents early and extended contact with their infant and the relationships built, nurses should be aware that: a. Immediate contact is essential for the parent-child relationship. b. Skin-to-skin contact is preferable to contact with the body totally wrapped in a blanket. c. Extended contact is especially important for adolescents and low-income women because they are at risk for parenting inadequacies. d. Mothers need to take precedence over their partners and other family matters.

ANS: C Nurses should encourage any activity that optimizes family extended contact. Immediate contact facilitates the attachment process but is not essential; otherwise, adopted infants would not establish the affectionate ties they do. The mode of infant-mother contact does not appear to have any important effect. Mothers and their partners are considered equally important. PTS: 1 DIF: Cognitive Level: Comprehension REF: 514 OBJ: Nursing Process: Planning MSC: Client Needs: Psychosocial Integrity

The nurse has documented that a child's level of consciousness is obtunded. Which describes this level of consciousness? a. Slow response to vigorous and repeated stimulation b. Impaired decision making c. Arousable with stimulation d. Confusion regarding time and place

ANS: C Obtunded describes a level of consciousness in which the child is arousable with stimulation. Stupor is a state in which the child remains in a deep sleep, responsive only to vigorous and repeated stimulation. Confusion is impaired decision making. Disorientation is confusion regarding time and place.

An appropriate nursing intervention when caring for an unconscious child should be to: a. change the child's position infrequently to minimize the chance of increased ICP. b. avoid using narcotics or sedatives to provide comfort and pain relief. c. monitor fluid intake and output carefully to avoid fluid overload and cerebral edema. d. give tepid sponge baths to reduce fever because antipyretics are contraindicated.

ANS: C Often comatose patients cannot cope with the quantity of fluids that they normally tolerate. Overhydration must be avoided to prevent fatal cerebral edema. The child's position should be changed frequently to avoid complications such as pneumonia and skin breakdown. Narcotics and sedatives should be used as necessary to reduce pain and discomfort, which can increase ICP. Antipyretics are the method of choice for fever reduction.

To initiate the milk ejection reflex (MER), the mother should be advised to: a. Wear a firm-fitting bra. b. Drink plenty of fluids. c. Place the infant to the breast. d. Apply cool packs to her breast.

ANS: C Oxytocin, which causes the MER reflex, increases in response to nipple stimulation. A firm bra is important to support the breast; however, will not initiate the MER reflex. Drinking plenty of fluids is necessary for adequate milk production, but this alone will not initiate the MER reflex. Cool packs to the breast will decrease the MER reflex. PTS: 1 DIF: Cognitive Level: Comprehension REF: 638 OBJ: Nursing Process: Implementation MSC: Client Needs: Health Promotion and Maintenance

To provide adequate postpartum care, the nurse should be aware that postpartum depression (PPD) without psychotic features: a. Means that the woman is experiencing the baby blues. In addition she has a visit with a counselor or psychologist. b. Is more common among older, Caucasian women because they have higher expectations. c. Is distinguished by irritability, severe anxiety, and panic attacks. d. Will disappear on its own without outside help.

ANS: C PPD is also characterized by spontaneous crying long after the usual duration of the baby blues. PPD, even without psychotic features, is more serious and persistent than postpartum baby blues. It is more common among younger mothers and African-American mothers. Most women need professional help to get through PPD, including pharmacologic intervention. PTS: 1 DIF: Cognitive Level: Comprehension REF: 546 OBJ: Nursing Process: Assessment MSC: Client Needs: Psychosocial Integrity

A woman who has recently given birth complains of pain and tenderness in her leg. On physical examination the nurse notices warmth and redness over an enlarged, hardened area. The nurse should suspect___ and should confirm the diagnosis by ______. a. Disseminated intravascular coagulation; asking for laboratory tests b. von Willebrand disease; noting whether bleeding times have been extended c. Thrombophlebitis; using real-time and color Doppler ultrasound d. Coagulopathies; drawing blood for laboratory analysis

ANS: C Pain and tenderness in the extremities, which show warmth, redness, and hardness, likely indicate thrombophlebitis. Doppler ultrasound is a common noninvasive way to confirm diagnosis. PTS: 1 DIF: Cognitive Level: Analysis REF: 539 OBJ: Nursing Process: Assessment MSC: Client Needs: Physiologic Integrity

A first-time dad is concerned that his 3-day-old daughters skin looks yellow. In the nurses explanation of physiologic jaundice, what fact should be included? a. Physiologic jaundice occurs during the first 24 hours of life. b. Physiologic jaundice is caused by blood incompatibilities between the mother and infant blood types. c. The bilirubin levels of physiologic jaundice peak between the second and fourth days of life. d. This condition is also known as breast milk jaundice.

ANS: C Physiologic jaundice becomes visible when the serum bilirubin reaches a level of 5 mg/dL or greater, which occurs when the baby is approximately 3 days old. This finding is within normal limits for the newborn. Pathologic jaundice occurs during the first 24 hours of life. Pathologic jaundice is caused by blood incompatibilities, causing excessive destruction of erythrocytes, and must be investigated. Breast milk jaundice occurs in one third of breastfed infants at 2 weeks and is caused by an insufficient intake of fluids. PTS: 1 DIF: Cognitive Level: Knowledge REF: 565 OBJ: Nursing Process: Implementation MSC: Client Needs: Health Promotion and Maintenance

Which hormone remains elevated in the immediate postpartum period of the breastfeeding woman? a. Estrogen b. Progesterone c. Prolactin d. Human placental lactogen

ANS: C Prolactin levels in the blood increase progressively throughout pregnancy. In women who breastfeed, prolactin levels remain elevated into the sixth week after birth. Estrogen and progesterone levels decrease markedly after expulsion of the placenta and reach their lowest levels 1 week into the postpartum period. Human placental lactogen levels decrease dramatically after expulsion of the placenta. PTS: 1 DIF: Cognitive Level: Comprehension REF: 486 OBJ: Nursing Process: Assessment MSC: Client Needs: Health Promotion and Maintenance

Which type of traction uses skin traction on the lower leg and a padded sling under the knee? a. Dunlop b. Bryant c. Russell d. Buck extension

ANS: C Russell traction uses skin traction on the lower leg and a padded sling under the knee. The combination of longitudinal and perpendicular traction allows realignment of the lower extremity and immobilizes the hips and knees in a flexed position. Dunlop traction is an upper-extremity traction used for fractures of the humerus. Bryant traction is skin traction with the legs flexed at a 90-degree angle at the hip. Buck extension traction is a type of skin traction with the legs in an extended position. It is used primarily for short-term immobilization, preoperatively with dislocated hips, for correcting contractures, or for bone deformities such as Legg-Calvé-Perthes disease.

A child is brought to the emergency department after experiencing a seizure at school. There is no previous history of seizures. The father tells the nurse that he cannot believe the child has epilepsy. The nurse's best response is: a. "Epilepsy is easily treated." b. "Very few children have actual epilepsy." c. "The seizure may or may not mean that your child has epilepsy." d. "Your child has had only one convulsion; it probably won't happen again."

ANS: C Seizures are the indispensable characteristic of epilepsy; however, not every seizure is epileptic. Epilepsy is a chronic seizure disorder with recurrent and unprovoked seizures. The treatment of epilepsy involves a thorough assessment to determine the type of seizure the child is having and the cause, followed by individualized therapy to allow the child to have as normal a life as possible. The nurse should not make generalized comments regarding the incidence of epilepsy until further assessment is made.

If a woman is at risk for thrombus and is not ready to ambulate, nurses may intervene by performing a number of interventions. Which intervention should the nurse avoid? a. Putting the patient in antiembolic stockings (TED hose) and/or sequential compression device (SCD) boots. b. Having the patient flex, extend, and rotate her feet, ankles, and legs. c. Having the patient sit in a chair. d. Notifying the physician immediately if a positive Homans sign occurs.

ANS: C Sitting immobile in a chair will not help. Bed exercise and prophylactic footwear may. TED hose and SCD boots are recommended. Bed exercises, such as flexing, extending, and rotating her feet, ankles, and legs, are useful. A positive Homans sign (calf muscle pain or warmth, redness, or tenderness) requires the physicians immediate attention. PTS: 1 DIF: Cognitive Level: Comprehension REF: 500 OBJ: Nursing Process: Implementation MSC: Client Needs: Physiologic Integrity

15. A clinic nurse notes that a child brought in for a physical has swelling and bruising around the eyes. The patient denies any trauma and the parent reports no environmental allergies. Which assessment is most important? A. Auscultate lungs bilaterally. B. Inspect skin on the back. C. Palpate abdomen and neck. D. Percuss abdomen and flank.

ANS: C Swelling and bruising around the face and eyes is often seen in children with neuroblastoma. Most commonly the tumor can be found by palpation of the abdomen or neck, where the tumor will present as a hard, painless mass that crosses the midline.

22. A parent whose two school-age children have asthma asks the nurse in what sports, if any, they can participate. The nurse should recommend: a. Soccer. c. Swimming. b. Running. d. Basketball.

ANS: C Swimming is well tolerated in children with asthma because they are breathing air fully saturated with moisture and because of the type of breathing required in swimming. Exercise-induced bronchospasm is more common in sports that involve endurance, such as soccer, running, and basketball. Prophylaxis with medications may be necessary

In the classification of newborns by gestational age and birth weight, the appropriate for gestational age (AGA) weight would: a. Fall between the 25th and 75th percentiles for the infants age. b. Depend on the infants length and the size of the head. c. Fall between the 10th and 90th percentiles for the infants age. d. Be modified to consider intrauterine growth restriction (IUGR).

ANS: C The AGA range is large: between the 10th and the 90th percentiles for the infants age. The infants length and size of the head are measured, but they do not affect the normal weight designation. IUGR applies to the fetus, not the newborns weight. PTS: 1 DIF: Cognitive Level: Comprehension REF: 601 OBJ: Nursing Process: Diagnosis MSC: Client Needs: Health Promotion and Maintenance

While examining a newborn, the nurse notes uneven skin folds on the buttocks and a click when performing the Ortolani maneuver. The nurse recognizes these findings as a sign that the newborn probably has: a. Polydactyly. b. Clubfoot. c. Hip dysplasia. d. Webbing

ANS: C The Ortolani maneuver is used to detect the presence of hip dysplasia. Polydactyly is the presence of extra digits. Clubfoot (talipes equinovarus) is a deformity in which the foot turns inward and is fixed in a plantarflexion position. Webbing, or syndactyly, is a fusing of the fingers or toes. PTS: 1 DIF: Cognitive Level: Knowledge REF: 571 OBJ: Nursing Process: Diagnosis MSC: Client Needs: Health Promotion and Maintenance

While assessing the newborn, the nurse should be aware that the average expected apical pulse range of a full-term, quiet, alert newborn is: a. 80 to 100 beats/min. b. 100 to 120 beats/min. c. 120 to 160 beats/min. d. 150 to 180 beats/min.

ANS: C The average infant heart rate while awake is 120 to 160 beats/min. The newborns heart rate may be about 85 to 100 beats/min while sleeping. The infants heart rate typically is a bit higher when alert but quiet. A heart rate of 150 to 180 beats/min is typical when the infant cries. PTS: 1 DIF: Cognitive Level: Comprehension REF: 559 OBJ: Nursing Process: Assessment MSC: Client Needs: Health Promotion and Maintenance

In administering vitamin K to the infant shortly after birth, the nurse understands that vitamin K is: a. Important in the production of red blood cells. b. Necessary in the production of platelets. c. Not initially synthesized because of a sterile bowel at birth. d. Responsible for the breakdown of bilirubin and prevention of jaundice.

ANS: C The bowel is initially sterile in the newborn, and vitamin K cannot be synthesized until food is introduced into the bowel. Vitamin K is necessary to activate blood clotting factors. The platelet count in term newborns is near adult levels. Vitamin K is necessary to activate prothrombin and other clotting factors. PTS: 1 DIF: Cognitive Level: Comprehension REF: 560 OBJ: Nursing Process: Implementation MSC: Client Needs: Physiologic Integrity

A breastfeeding woman develops engorged breasts at 3 days postpartum. What action would help this woman achieve her goal of reducing the engorgement? The woman: a. Skips feedings to let her sore breasts rest. b. Avoids using a breast pump. c. Breastfeeds her infant every 2 hours. d. Reduces her fluid intake for 24 hours.

ANS: C The mother should be instructed to attempt feeding her infant every 2 hours while massaging the breasts as the infant is feeding. Skipping feedings may cause further swelling and discomfort. If the infant does not feed adequately and empty the breast, the mother may pump to extract the milk and relieve some of the discomfort. Dehydration further irritates swollen breast tissue. PTS: 1 DIF: Cognitive Level: Comprehension REF: 653 OBJ: Nursing Process: Implementation MSC: Client Needs: Physiologic Integrity

With regard to the newborns developing cardiovascular system, nurses should be aware that: a. The heart rate of a crying infant may rise to 120 beats/min. b. Heart murmurs heard after the first few hours are cause for concern. c. The point of maximal impulse (PMI) often is visible on the chest wall. d. Persistent bradycardia may indicate respiratory distress syndrome (RDS).

ANS: C The newborns thin chest wall often allows the PMI to be seen. The normal heart rate for infants who are not sleeping is 120 to 160 beats/min. However, a crying infant temporarily could have a heart rate of 180 beats/min. Heart murmurs during the first few days of life have no pathologic significance; an irregular heart rate past the first few hours should be evaluated further. Persistent tachycardia may indicate RDS; bradycardia may be a sign of congenital heart blockage. PTS: 1 DIF: Cognitive Level: Comprehension REF: 559 OBJ: Nursing Process: Assessment MSC: Client Needs: Health Promotion and Maintenance

A mother with mastitis is concerned about breastfeeding while she has an active infection. The nurse should explain that: a. The infant is protected from infection by immunoglobulins in the breast milk. b. The infant is not susceptible to the organisms that cause mastitis. c. The organisms that cause mastitis are not passed to the milk. d. The organisms will be inactivated by gastric acid.

ANS: C The organisms are localized in the breast tissue and are not excreted in the breast milk. The mother is just producing the immunoglobulin from this infection, so it is not available for the infant. Because of an immature immune system, infants are susceptible to many infections; however, this infection is in the breast tissue and is not excreted in the breast milk. The organism will not enter the infants gastrointestinal system. This patient should be encouraged to empty her breasts fully every 2 hours, either by pumping or by breastfeeding. PTS: 1 DIF: Cognitive Level: Comprehension REF: 540 OBJ: Nursing Process: Implementation MSC: Client Needs: Physiologic Integrity

A 5-year-old girl sustained a concussion when she fell out of a tree. In preparation for discharge, the nurse is discussing home care with her mother. Which statement made by the mother indicates a correct understanding of the teaching? a. "I should expect my child to have a few episodes of vomiting." b. "If I notice sleep disturbances, I should contact the physician immediately." c. "I should expect my child to have some behavioral changes after the accident." d. "If I notice diplopia, I will have my child rest for 1 hour."

ANS: C The parents are advised of probable posttraumatic symptoms that may be expected. These include behavioral changes and sleep disturbances. If the child has these clinical signs, they should be immediately reported for evaluation. Sleep disturbances are to be expected.

28. A pregnant woman's diet may not meet her need for folates. A good source of this nutrient is: A. chicken. B. cheese. C. potatoes. D. green leafy vegetables.

ANS: D Sources of folates include green leafy vegetables, whole grains, fruits, liver, dried peas, and beans. Chicken and cheese are excellent sources of protein but are poor in folates. Potatoes contain carbohydrates and vitamins and minerals but are poor in folates.

Which is beneficial in reducing the risk of Reye syndrome? a. Immunization against the disease b. Medical attention for all head injuries c. Prompt treatment of bacterial meningitis d. Avoidance of aspirin to treat fever associated with influenza

ANS: D Although the etiology of Reye syndrome is obscure, most cases follow a common viral illness, either varicella or influenza. A potential association exists between aspirin therapy and the development of Reye syndrome, so use of aspirin is avoided. No immunization currently exists for Reye syndrome. Reye syndrome is not correlated with head injuries or bacterial meningitis.

Which is a type of skin traction with legs in an extended position? a. Dunlop b. Bryant c. Russell d. Buck extension

ANS: D Buck extension traction is a type of skin traction with the legs in an extended position. It is used primarily for short-term immobilization, preoperatively with dislocated hips, for correcting contractures, or for bone deformities such as Legg-Calvé-Perthes disease. Dunlop traction is an upper-extremity traction used for fractures of the humerus. Bryant traction is skin traction with the legs flexed at a 90-degree angle at the hip. Russell traction uses skin traction on the lower leg and a padded sling under the knee. The combination of longitudinal and perpendicular traction allows realignment of the lower extremity and immobilizes the hips and knees in a flexed position.

The nurse is performing a Glasgow Coma Scale on a school-age child with a head injury. The child opens eyes spontaneously, obeys commands, and is oriented to person, time, and place. Which is the score the nurse should record? a. 8 b. 11 c. 13 d. 15

ANS: D The Glasgow Coma Scale (GCS) consists of a three-part assessment: eye opening, verbal response, and motor response. Numeric values of 1 through 5 are assigned to the levels of response in each category. The sum of these numeric values provides an objective measure of the patient's level of consciousness (LOC). A person with an unaltered LOC would score the highest, 15. The child who opens eyes spontaneously, obeys commands, and is oriented is scored at a 15.

As part of their teaching function at discharge, nurses should educate parents regarding safe sleep. Which statement is incorrect? a. Prevent exposure to people with upper respiratory tract infections. b. Keep the infant away from secondhand smoke. c. Avoid loose bedding, water beds, and beanbag chairs. d. Place the infant on his or her abdomen to sleep.

ANS: D The infant should be laid down to sleep on his or her back for better breathing and to prevent sudden infant death syndrome. Infants are vulnerable to respiratory infections; infected people must be kept away. Secondhand smoke can damage lungs. Infants can suffocate in loose bedding, and furniture that can trap them. Per AAP guidelines, infants should always be placed back to sleep and allowed tummy time to play, to prevent plagiocephaly. PTS: 1 DIF: Cognitive Level: Comprehension REF: 622 OBJ: Nursing Process: Planning MSC: Client Needs: Safe and Effective Care Environment

Early this morning, an infant boy was circumcised using the PlastiBell method. The nurse tells the mother that she and the infant can be discharged after: a. The bleeding stops completely. b. Yellow exudate forms over the glans. c. The PlastiBell rim falls off. d. The infant voids.

ANS: D The infant should be observed for urination after the circumcision. Bleeding is a common complication after circumcision. The nurse will check the penis for 12 hours after a circumcision to assess and provide appropriate interventions for prevention and treatment of bleeding. Yellow exudates cover the glans penis in 24 hours after the circumcision. This is part of normal healing and not an infective process. The PlastiBell remains in place for about a week and falls off when healing has taken place. PTS: 1 DIF: Cognitive Level: Comprehension REF: 615 OBJ: Nursing Process: Planning MSC: Client Needs: Health Promotion and Maintenance

A primiparous woman is delighted with her newborn son and wants to begin breastfeeding as soon as possible. The nurse can facilitate the infants correct latch-on by helping the woman hold the infant: a. With his arms folded together over his chest. b. Curled up in a fetal position. c. With his head cupped in her hand. d. With his head and body in alignment.

ANS: D The infants head and body should be in correct alignment with the mother and the breast during latch-on and feeding. Holding the infant with his arms folded together over his chest, curled up in a fetal position, or with his head cupped in her hand are not ideal positions to facilitate latch-on. PTS: 1 DIF: Cognitive Level: Comprehension REF: 645 OBJ: Nursing Process: Implementation MSC: Client Needs: Health Promotion and Maintenance

The nurse is conducting teaching to parents of a 7-year-old child who fractured an arm and is being discharged with a cast. Which instruction should be included in the teaching? a. Swelling of the fingers is to be expected for the next 48 hours. b. Immobilize the shoulder to decrease pain in the arm. c. Allow the affected limb to hang down for 1 hour each day. d. Elevate casted arm when resting and when sitting up.

ANS: D The injured extremity should be kept elevated while resting and in a sling when upright. This will increase venous return. Swelling of the fingers may indicate neurovascular damage and should be reported immediately. Permanent damage can occur within 6 to 8 hours. Joints above and below the cast on the affected extremity should be moved. The affected limb should not hang down for any length of time.

While completing a newborn assessment, the nurse should be aware that the most common birth injury is: a. To the soft tissues. b. Caused by forceps gripping the head on delivery. c. Fracture of the humerus and femur. d. Fracture of the clavicle.

ANS: D The most common birth injury is fracture of the clavicle (collarbone). It usually heals without treatment, although the arm and shoulder may be immobilized for comfort. PTS: 1 DIF: Cognitive Level: Knowledge REF: 665 OBJ: Nursing Process: Assessment MSC: Client Needs: Physiologic Integrity

Because of the premature infants decreased immune functioning, what nursing diagnosis should the nurse include in a plan of care for a premature infant? a. Delayed growth and development b. Ineffective thermoregulation c. Ineffective infant feeding pattern d. Risk for infection

ANS: D The nurse needs to understand that decreased immune functioning increases the risk for infection. Growth and development, thermoregulation, and feeding may be affected, although only indirectly. PTS: 1 DIF: Cognitive Level: Application REF: 684 OBJ: Nursing Process: Planning MSC: Client Needs: Physiologic Integrity

22. Which type of hernia has an impaired blood supply to the herniated organ? a. Hiatal hernia b. Incarcerated hernia c. Omphalocele d. Strangulated hernia

ANS: D A strangulated hernia is one in which the blood supply to the herniated organ is impaired. A hiatal hernia is the intrusion of an abdominal structure, usually the stomach, through the esophageal hiatus. An incarcerated hernia is a hernia that cannot be reduced easily. Omphalocele is the protrusion of intraabdominal viscera into the base of the umbilical cord. The sac is covered with peritoneum and not skin.

10. Therapeutic management of most children with Hirschsprung's disease is primarily: a. daily enemas. b. low-fiber diet. c. permanent colostomy. d. surgical removal of affected section of bowel.

ANS: D Most children with Hirschsprung's disease require surgical rather than medical management. Surgery is done to remove the aganglionic portion of the bowel, relieve obstruction, and restore normal bowel motility and function of the internal anal sphincter. Preoperative management may include enemas and low-fiber, high-calorie, high-protein diet until the child is physically ready for surgery. The colostomy that is created in Hirschsprung's disease is usually temporary.

32. The parent of a child diagnosed with diabetes mellitus asks the nurse when urine testing will be necessary. The nurse should explain that urine testing is necessary for which reason? a. Glucose is needed before administration of insulin. b. Glucose is needed 4 times a day. c. Glycosylated hemoglobin is required. d. Ketonuria is suspected.

ANS: D Urine testing is still performed to detect evidence of ketonuria. Urine testing for glucose is no longer indicated for medication administration because of the poor correlation between blood glucose levels and glycosuria. Glycosylated hemoglobin analysis is performed on a blood sample.

A (Cardiopulmonary resuscitation (CPR))

Apnea of infancy has been diagnosed in an infant who will soon be discharged with home monitoring. When teaching the parents about the infant's care, what is the most important information the nurse should include in the discharge teaching plan? A. Cardiopulmonary resuscitation (CPR) B. Administration of intravenous (IV) fluids C. Reassurance that the infant cannot be electrocuted during monitoring D. Advice that the infant not be left with other caretakers such as baby-sitters

B (2 months At age 2 months, the infant has a social, responsive smile. A reflex smile is usually present at age 1 month. The 3-month-old can recognize familiar faces. At age 4 months, the infant can enjoy social interactions.)

At what age should the nurse expect an infant to begin smiling in response to pleasurable stimuli? a. 1 month b. 2 months c. 3 months d. 4 months

D (Earlier-than-normal tooth eruption This is earlier than expected. Most infants at age 6 months have two teeth. Six teeth at 6 months is not delayed; it is early tooth eruption. Although unusual, it is not dangerous.)

Austin, age 6 months, has six teeth. The nurse should recognize that this is: a. Normal tooth eruption. b. Delayed tooth eruption. c. Unusual and dangerous d. Earlier-than-normal tooth eruption.

There are four essential components of labor. The first is the passageway. It is composed of the bony pelvis and soft tissues. What is one component of the passageway? a) Perineum b) Cervix c) False pelvis d) Uterus

B (Cervix Rationale: The cervix and vagina are soft tissues that form the part of the passageway known as the birth canal.)

The most prevalent clinical manifestation of abruptio placentae (as opposed to placenta previa) is: A. bleeding. B. intense abdominal pain. C. uterine activity. D. cramping

B (Pain is absent with placenta previa but may be agonizing with abruptio placentae. Bleeding may be present in varying degrees for both placental conditions. Uterine activity may be present with both placental conditions. Cramping is a form of uterine activity that may be present in both placental conditions.)

Which intervention does the nurse implement for a patient immediately after a severe abdominal trauma? A. Prep the patient for cesarean birth. B. Send the patient for pelvic computed tomography (CT) scanning. C. Provide fluids to the patient as part of the protocol for ultrasound examination. D. Prepare to administer Rho(D) immunoglobulin.

B (Pelvic CT scanning helps visualize extraperitoneal and retroperitoneal structures and the genitourinary tract. The nurse needs to prepare the patient for cesarean birth if there is no evidence of a maternal pulse. Ultrasound examination is not as effective as electronic fetal monitoring for determining placental abruption in the patient after the trauma. Therefore the nurse prepares the patient for a CT scan after a severe abdominal trauma. The nurse needs to administer Rho(D) immunoglobulin in an Rh-negative pregnant trauma patient. This helps protect the patient from isoimmunization.)

A 32-year-old woman presents to the labor-and-delivery suite in active labor. She is multigravida, relaxed, and talking with her husband. When examined by the nurse, the fetus is found to be in a cephalic presentation. His occiput is facing toward the front and slightly to the right of the mother's pelvis, and he is exhibiting a flexed attitude. How does the nurse document the position of the fetus? a) LOA b) ROA c) LOP d) ROP

B (Rationale: Document the fetal position in the clinical record using abbreviations (Box 8-1). The first letter describes the side of the maternal pelvis toward which the presenting part is facing ("R" for right and "L" for left). The second letter or abbreviation indicates the reference point ("O" for occiput, "Fr" for frontum, etc.). The last part of the designation specifies whether the presenting part is facing the anterior (A) or the posterior (P) portion of the pelvis, or whether it is in a transverse (T) position.)

To teach patients about the process of labor adequately, the nurse knows that which event is the best indicator of true labor? a. Bloody show b. Cervical dilation and effacement c. Fetal descent into the pelvic inlet d. Uterine contractions every 7 minutes

B (The conclusive distinction between true and false labor is that contractions of true labor cause progressive change in the cervix. Bloody show can occur before true labor. Fetal descent can occur before true labor. False labor may have contractions that occur this frequently; however, this is usually inconsistent.)

The nurse is caring for a hospitalized 4-year-old boy, Ryan. His parents tell the nurse that they will be back to visit at 6 PM. When Ryan asks the nurse when his parents are coming, the nurse's best response is: a. "They will be here soon." b. "They will come after dinner." c. "Let me show you on the clock when 6 PM is." d. "I will tell you every time I see you how much longer it will be."

B A 4-year-old understands time in relation to events such as meals. Children perceive "soon" as a very short time. The nurse may lose the child's trust if his parents do not return in the time he perceives as "soon." Children cannot read or use a clock for practical purposes until age 7 years. This answer assumes that the child understands the concept of hours and minutes, which is not developed until age 5 or 6 years.

The psychologic effects of being obese during adolescence include: a. Sexual promiscuity. b. Poor body image. c. Memory having no effect on eating behavior. d. Accurate body image but self-deprecating attitude.

B Common emotional consequences of obesity include poor body image, low self-esteem, social isolation, and feelings of depression and isolation. Sexual promiscuity is an unlikely effect of obesity. The obese adolescent often substitutes food for affection. Eating behaviors are closely related to memory. Memory and appetite are closely linked and can be modified over time with treatment. Obese adolescents most often have a very poor self-image.

Which behavior suggests appropriate psychosocial development in the adolescent? a. The adolescent seeks validation for socially acceptable behavior from older adults. b. The adolescent is self-absorbed and self-centered and has sudden mood swings. c. Adolescents move from peers and enjoy spending time with family members. d. Conformity with the peer group increases in late adolescence.

B During adolescence, energy is focused within. Adolescents concentrate on themselves in an effort to determine who they are or who they will be. Adolescents are likely to be impulsive and impatient. Parents often describe their teenager as being "self-centered" or "lazy." The peer group validates acceptable behavior during adolescence. Adolescents move from family and enjoy spending time with peers. Adolescents also spend time alone; they need this time to think and concentrate on themselves. Conformity becomes less important in late adolescence.

Which predisposes the adolescent to feel an increased need for sleep? a. An inadequate diet b. Rapid physical growth c. Decreased activity that contributes to a feeling of fatigue d. The lack of ambition typical of this age group

B During growth spurts, the need for sleep is increased. Rapid physical growth, the tendency toward overexertion, and the overall increased activity of this age contribute to fatigue.

The nurse is performing an assessment on a child and notes the presence of Koplik's spots. In which communicable disease are Koplik's spots present? a. Rubella c. Chickenpox (varicella) b. Measles (rubeola) d. Exanthema subitum (roseola)

B Koplik's spots are small, irregular red spots with a minute, bluish white center found on the buccal mucosa 2 days before systemic rash. Koplik's spots are not present with rubella, varicella, or roseola.

The mean age of menarche in the United States is: a. 11.5 years c. 13.5 years b. 12.5 years d. 14 years

B The average age of menarche is 12 years and 4 months in North American girls, with a normal range of 10.5 to 15 years.

According to Erikson, the psychosocial task of adolescence is developing: a. Intimacy. c. Initiative. b. Identity. d. Independence.

B Traditional psychosocial theory holds that the developmental crises of adolescence lead to the formation of a sense of identity. Intimacy is the developmental stage for early adulthood. Initiative is the developmental stage for early childhood. Independence is not one of Erikson's developmental stages.

Braxton Hicks contractions are termed "practice contractions" and occur throughout pregnancy. When the woman's body is getting ready to go into labor, it begins to show anticipatory signs of impending labor. Among these signs are Braxton Hicks contractions that are more frequent and stronger in intensity. What differentiates Braxton Hicks contractions from true labor? a) Braxton Hicks contractions get closer together with activity b) Braxton Hicks contractions cause "ripening" of the cervix. c) Braxton Hicks contractions usually decrease in intensity with walking d) Braxton Hicks contractions do not last long enough to be true labor

C (Braxton Hicks contractions usually decrease in intensity with walking Rationale: Braxton Hicks contractions occur more frequently and are more noticeable as pregnancy approaches term. These irregular, practice contractions usually decrease in intensity with walking and position changes.)

Which type of play is most typical of the preschool period? a. Solitary c. Associative b. Parallel d. Team

C Associative play is group play in similar or identical activities but without rigid organization or rules. Solitary play is that of infants. Parallel play is that of toddlers. School-age children play in teams.

During pregnancy, alcohol withdrawal may be treated using: A. disulfiram (Antabuse). B. corticosteroids. C. benzodiazepines. E. aminophylline.

C. benzodiazepines. Disulfiram is contraindicated in pregnancy because it is teratogenic. Corticosteroids are not used to treat alcohol withdrawal. Symptoms that occur during alcohol withdrawal can be managed with short-acting barbiturates or benzodiazepines. Aminophylline is not used to treat alcohol withdrawal.

A (Avoidance of eye contact. One of the clinical manifestations of nonorganic failure to thrive is the child's avoidance of eye contact with the health professional. A malabsorption defect would result in a physiologic problem, not behavioral. Weight (but not height) below the 5th percentile is indicative of failure to thrive. Developmental delays, including social, motor, adaptive, and language, exist.)

Clinical manifestations of failure to thrive caused by behavioral problems resulting in inadequate intake of calories include: a. Avoidance of eye contact. b. An associated malabsorption defect. c. Weight that falls below the 15th percentile. d. Normal achievement of developmental landmarks.

Nurses can advise their clients that all are signs that precede labor except: a) A return of urinary frequency as a result of increased bladder pressure b) Persistent low backache from relaxed pelvic joints c) Stronger and more frequent uterine (Braxton Hicks) contractions d) A decline in energy, as the body stores up for labor

D (A decline in energy, as the body stores up for labor)

A new OB/GYN physician has just finished evaluating her one hundredth patient. In reviewing the documentation from all patients thus far, which types of pelvis would you assume the physician has seen the most and the least? a) Android and platypelloid, respectively b) Gynecoid and android, respectively c) Anthropoid and gynecoid, respectively d) Gynecoid and platypelloid, respectively

D (Gynecoid and platypelloid, respectively Rationale: The gynecoid is the typical female pelvis shape; platypelloid pelvis is the least common type of pelvis in women. Women with anthropoid pelvic shapes are able to deliver normally one third of the time, and are somewhat rare. An android pelvis is similar to a male pelvis, and is seen in 16% of nonwhite women.)

A patient reports excessive vomiting in the first trimester of the pregnancy, which has resulted in nutritional deficiency and weight loss. The urinalysis report of the patient indicates ketonuria. Which disorder does the patient have? A. Preeclampsia B. Hyperthyroid disorder C. Gestational hypertension D. Hyperemesis gravidarum

D (Hyperemesis gravidarum is characterized by excessive vomiting during pregnancy, which causes nutritional deficiency and weight loss. The presence of ketonuria is another indication of this disorder. Preeclampsia refers to hypertension and proteinuria in patients after 20 weeks' gestation. Hyperthyroid disorder may be one of the causes of hyperemesis gravidarum. Gestational hypertension also develops after 20 weeks' gestation.)

Which fetal risk is associated with an ectopic pregnancy? A. Miscarriage B. Fetal anemia C. Preterm birth D. Fetal deformity

D (In an ectopic pregnancy, the risk for fetal deformity is high because of the pressure deformities caused by oligohydramnios. There may be facial or cranial asymmetry, various joint deformities, limb deficiency, and central nervous system (CNS) anomalies. Miscarriage is not likely to happen in an ectopic pregnancy. Instead, the patient is at risk for pregnancy-related death resulting from ectopic rupture. Fetal anemia is a risk associated with placenta previa. Preterm birth is not possible because the pregnancy is dissolved when it is diagnosed or a surgery is performed to remove the fetus.)

When going through the transition phase of labor women often feel out of control. What do women in the transition phase of labor need? a) Their significant other beside them b) Intense nursing care c) Just to be left alone d) Positive reinforcement

D (Positive reinforcement Rationale: Any women, even ones who have had natural childbirth classes, have a difficult time maintaining positive coping strategies during this phase of labor. Many women describe feeling out of control during this phase of labor. A woman in transition needs support, encouragement, and positive reinforcement.)

When using intermittent auscultation (IA) for fetal heart rate, nurses should be aware that: a. They can be expected to cover only two or three clients when IA is the primary method of fetal assessment. b. The best course is to use the descriptive terms associated with electronic fetal monitoring (EFM) when documenting results. c. If the heartbeat cannot be found immediately, a shift must be made to EFM. d. Ultrasound can be used to find the fetal heartbeat and reassure the mother if initial difficulty was a factor.

D Locating fetal heartbeats often takes time. Mothers can be reassured verbally and by the ultrasound pictures if ultrasound is used to help locate the heartbeat. When used as the primary method of fetal assessment, auscultation requires a nurse-to-client ratio of one to one. Documentation should use only terms that can be numerically defined; the usual visual descriptions of EFM are inappropriate.

During a physical assessment of an at-risk client, the nurse notes generalized edema, crackles at the base of the lungs, and some pulse irregularity. These are most likely signs of: A. euglycemia. B. rheumatic fever. C. pneumonia. D. cardiac decompensation.

D. cardiac decompensation. Euglycemia is a condition of normal glucose levels. These symptoms indicate cardiac decompensation. Rheumatic fever can cause heart problems, but it does not present with these symptoms, which indicate cardiac decompensation. Pneumonia is an inflammation of the lungs and would not likely generate these symptoms, which indicate cardiac decompensation. Symptoms of cardiac decompensation may appear abruptly or gradually.

B, C, D (B. who sleep prone C. who were premature D. with prenatal drug exposure)

Infants most at risk for sudden infant death syndrome (SIDS) are those: (Select all that apply.) A. who sleep supine B. who sleep prone C. who were premature D. with prenatal drug exposure E. with a cousin that died of SIDS

B (It is now recommended that all infants and toddlers ride in rear-facing car safety seats until they reach the age of 2 years or the height or weight recommended by the car seat manufacturer. Children 2 years old and older who have outgrown the rear-facing height or weight limit for their car safety seat should use a forward-facing car safety seat with a harness up to the maximum height or weight recommended by the manufacturer. One year is too young to switch to a forward-facing position.)

Kimberly's parents have been using a rearward-facing, convertible car seat since she was born. The parents should be taught that most car seats can be safely switched to the forward-facing position when the child reaches which age? a. 1 year b. 2 years c. 3 years d. 4 years

D. (control of anal and urethral sphincters.)

Myelination of the spinal cord is almost complete by 2 years of age. As a result of this, the toddler can gradually achieve: A. throwing a ball without falling. B. slowing of gastrointestinal transit time. C. visual acuity of 20/20. D. control of anal and urethral sphincters.

B (bowel control is accomplished before bladder control, so the parent should focus on bowel training first.)

One of the major tasks of toddlerhood is toilet training. In teaching the parents about a child's readiness for toilet training, it is important for the nurse to emphasize that: A. nighttime bladder control develops first, so parents should focus on that in the initial teaching with their toddler. B. bowel control is accomplished before bladder control, so the parent should focus on bowel training first. C. the toddler must have the gross motor skill to climb up to the adult toilet before training is begun. D. the universal age for toilet training to begin is 2 years, and the universal age for completion is 4 years.

C (Parents need to be taught to always use the restraint even for short trips. Further teaching is needed if they make this statement. Parents have understood the teaching if they encourage the child to help attach buckles, straps, and shields but always double-check fastenings; do not start the car until everyone is properly restrained; and anchor the car safety seat securely to the car's anchoring system and apply the harness snugly to the child.)

Parents need further teaching about the use of car safety seats if they make which statement? a. "Even if our toddler helps buckle the straps, we will double-check the fastenings." b. "We won't start the car until everyone is properly restrained." c. "We won't need to use the car seat on short trips to the store." d. "We will anchor the car seat to the car's anchoring system."

A (Animism is the attribution of lifelike qualities to inanimate objects. By scolding the stairs, the toddler is attributing human characteristics to them. Ritualism is the need to maintain sameness and reliability. It provides a sense of comfort to the toddler. Irreversibility is the inability to reverse or undo actions initiated physically. Steven is acting in an age-appropriate manner.)

Steven, 16 months old, falls down a few stairs. He gets up and "scolds" the stairs as if they caused him to fall. This is an example of which of the following? a. Animism b. Ritualism c. Irreversibility d. Delayed cognitive development

C (give child a frozen teething ring to relieve inflammation.)

The MOST appropriate recommendation for relief of teething pain is to instruct the parents to: A. rub gums with aspirin to relieve inflammation. B. apply hydrogen peroxide to gums to relieve irritation. C. give child a frozen teething ring to relieve inflammation. D. have child chew on a warm teething ring to encourage tooth eruption.

A (Injuries are the single most common cause of death in children ages 1 through 4 years. It is the period of highest death rate from injuries of any childhood age-group except adolescence. Infectious and childhood diseases are less common cause of deaths in this age-group. Congenital disorders are the second leading cause of death in this age-group.)

The leading cause of death during the toddler period is: a. Injuries. b. Infectious diseases. c. Congenital disorders. d. Childhood diseases.

A (trust.)

The nurse educator instructs a nursing student that according to Erikson, infancy is concerned with acquiring a sense of: A. trust. B. industry. C. initiative. D. separation.

C (12 months The American Academy of Pediatrics does not recommend the use of cow's milk for children younger than 12 months. At 6 and 9 months, the infant should be receiving commercial infant formula or breast milk. At age 18 months, milk and formula are supplemented with solid foods, water, and some fruit juices.)

The nurse should teach parents that which age is safe to give infants whole milk instead of commercial infant formula? a. 6 months b. 9 months c. 12 months d. 18 months

B (Play develops from the solitary play of infancy to the parallel play of toddlers. The toddler plays alongside other children, not with them. This typical behavior of the toddler is not intentionally aggressive. Shared play is not within their cognitive development. Toddlers do not conceptualize shared play. Because the toddler cannot view the situation from the perspective of the other child, it is okay to take the toy. Therefore, no right or wrong is associated with taking a toy.)

Two toddlers are playing in a sandbox when one child suddenly grabs a toy from the other child. The best interpretation of this behavior is that: a. This is typical behavior because toddlers are aggressive. b. This is typical behavior because toddlers are egocentric. c. Toddlers should know that sharing toys is expected of them. d. Toddlers should have the cognitive ability to know right from wrong.

C (Being persistent through 10 to 15 minutes of food refusal)

Which strategy might be recommended for an infant with failure-to-thrive to increase caloric intake? A. Using developmental stimulation by a specialist during feedings B. Avoiding solids until after the bottle is well accepted C. Being persistent through 10 to 15 minutes of food refusal D. Varying schedule of routine activities on a daily basis

A (Push-pull toys encourage large muscle activity and are appropriate for toddlers. Nesting blocks are more appropriate for a 12- to 15-month-old child. A bicycle with training wheels is appropriate for a preschool or young school-age child. A computer can be appropriate as early as the preschool years.)

Which toy is the most developmentally appropriate for an 18- to 24-month-old child? a. A push-pull toy b. Nesting blocks c. A bicycle with training wheels d. A computer

A (Place the infant prone for 30 to 60 minutes per day. Prevention of positional plagiocephaly may begin shortly after birth by implementing prone positioning or "tummy time" for approximately 30 to 60 minutes per day when the infant is awake. Soft mattresses or sleeping with parents (co-sleeping) are not recommended because they put the infant at a higher risk for a sudden infant death incident. To prevent plagiocephaly, prolonged placement in car safety seats should be avoided.)

With the goal of preventing plagiocephaly, the nurse should teach new parents to: a. Place the infant prone for 30 to 60 minutes per day. b. Buy a soft mattress. c. Allow the infant to nap in the car safety seat. d. Have the infant sleep with the parents.

3. Which nutrient's recommended dietary allowance (RDA) is higher during lactation than during pregnancy? a. Energy (kcal) b. Iron c. Vitamin A d. Folic acid

a. Energy (kcal)

26. To help a woman reduce the severity of nausea caused by morning sickness, the nurse might suggest that she: a. Try a tart food or drink such as lemonade or salty foods such as potato chips. b. Drink plenty of fluids early in the day. c. Brush her teeth immediately after eating. d. Never snack before bedtime.

a. Try a tart food or drink such as lemonade or salty foods such as potato chips.

9. A 27-year-old pregnant woman had a preconceptual body mass index (BMI) of 18.0. The nurse knows that this woman's total recommended weight gain during pregnancy should be at least: a. 20 kg (44 lb). b. 16 kg (35 lb). c. 12.5 kg (27.5 lb). d. 10 kg (22 lb).

c. 12.5 kg (27.5 lb).

34. If a patient's normal prepregnancy diet contains 45 g of protein daily, how many more grams of protein should she consume per day during pregnancy? a. 5 b. 10 c. 25 d. 30

c. 25

15. Pregnant adolescents are at high risk for _____ because of lower body mass indices (BMIs) and "fad" dieting. a. Obesity b. Diabetes c. Low-birth-weight babies d. High-birth-weight babies

c. Low-birth-weight babies

23. While taking a diet history, the nurse might be told that the expectant mother has cravings for ice chips, cornstarch, and baking soda. This represents a nutritional problem known as: a. Preeclampsia. b. Pyrosis. c. Pica. d. Purging.

c. Pica.

An appropriate nursing diagnosis for a child with a cognitive dysfunction who has a limited ability to anticipate danger is: a. Impaired Social Interaction. b. Deficient Knowledge. c. Risk for Injury. d. Ineffective Coping.

c. Risk for Injury.

The nurse is talking with a 10-year-old boy who wears bilateral hearing aids. The left hearing aid is making an annoying whistling sound that the child cannot hear. The most appropriate nursing action is to: a. Ignore the sound. b. Ask him to reverse the hearing aids in his ears. c. Suggest that he reinsert the hearing aid. d. Suggest that he raise the volume of the hearing aid.

c. Suggest that he reinsert the hearing aid

4. A pregnant woman's diet consists almost entirely of whole grain breads and cereals, fruits, and vegetables. The nurse would be most concerned about this woman's intake of: a. Calcium. b. Protein. c. Vitamin B12. d. Folic acid.

c. Vitamin B12.

31. A pregnant woman's diet may not meet her need for folates. A good source of this nutrient is: a. Chicken b. Cheese c. Potatoes d. Green leafy vegetables

d. Green leafy vegetables

1. A 22-year-old woman pregnant with a single fetus has a preconception body mass index (BMI) of 24. When she was seen in the clinic at 14 weeks of gestation, she had gained 1.8 kg (4 lb) since conception. How would the nurse interpret this? a. This weight gain indicates possible gestational hypertension. b. This weight gain indicates that the woman's infant is at risk for intrauterine growth restriction (IUGR). c. This weight gain cannot be evaluated until the woman has been observed for several more weeks. d. The woman's weight gain is appropriate for this stage of pregnancy.

d. The woman's weight gain is appropriate for this stage of pregnancy.

21. Which vitamins or minerals can lead to congenital malformations of the fetus if taken in excess by the mother? a. Zinc b. Vitamin D c. Folic acid d. Vitamin A

d. Vitamin A

29. Which pregnant woman should restrict her weight gain during pregnancy? a. Woman pregnant with twins b. Woman in early adolescence c. Woman shorter than 62 inches or 157 cm d. Woman who was 20 pounds overweight before pregnancy

d. Woman who was 20 pounds overweight before pregnancy

How should the nurse respond to a 53-year-old woman who appears anxious and distressed who wants to have children and approaches the nurse for counseling? 1 Refer her to local adoption agencies. 2 Advise the woman to attend prenatal classes. 3 Explain the consequences of having children later in life. 4 Explain the process of having children through surrogate mother.

1 A 53-year-old woman is likely to have attained menopause and may be able to have children only through adoption. Therefore, the nurse should refer the woman to a local adoption agency. Prenatal classes are recommended for women who are of childbearing age. Explaining the woman's inability to have children after menopause might not solve the problem. The woman likely will have attained menopause and would be unable to produce ovum, so surrogacy is not possible. Test-Taking Tip: Look for options that are similar in nature. If all are correct, either the question is poor or all options are incorrect; the latter of which is more likely. Example: If the answer you are seeking is directed to a specific treatment and all but one option deal with signs and symptoms, you would be correct in choosing the treatment-specific option.

A pregnant patient with a normal fetal kick count has come for a regular nonstress testing session. The nurse notices that there are no heart accelerations after 40 minutes of testing. What diagnostic testing will the nurse include in the plan of care? 1 Contraction stress test 2 Biophysical profile test 3 Maternal serum analysis 4 Doppler blood flow test

1 A normal fetal kick count is an indication of fetal activity. The patient has undergone nonstress testing, which may have high false-positive rates. Therefore the patient may be scheduled for a contraction stress test. Biophysical profile testing allows detailed assessment of the physical and physiologic characteristics of the fetus. Because the kick count is normal, biophysical profile testing is not needed. Maternal serum analysis is done to determine fetal abnormalities. It is not advised in this case, because the fetal kick count indicates adequate fetal activity. Doppler blood flow analysis is a noninvasive test for analyzing fetal circulation. It cannot be used to assess the fetal heart rate.

A preschool child watches a nurse pour medication from a tall, thin glass to a short, wide glass. Which statement is appropriate developmentally for this age group? 1 The amount of medicine is less. 2 The amount of medicine did not change, only its appearance. 3 Pouring medicine makes the medicine hot. 4 The glass changed shape to accommodate the medicine

1 A preschool child does not have the ability to understand the concept of conservation . This concept is not developed until school age. Understanding conservation occurs between 7 to 10 years of age, when a child begins to realize that physical factors, such as volume, weight, and number, remain the same even though outward appearances are changed. Children are able to deal with a number of different aspects of a situation simultaneously. This is not an expected response by a child. A preschool child will not typically believe the glass changed shape to accommodate the medicine but rather that the amount of medicine is less in the short, wide glass.

After reviewing the reports of a pregnant patient, the nurse infers that there might be a high risk for intrauterine growth restriction (IUGR). What could be the reason for this? The amniotic fluid index (AFI) is: 1 Less than 5 cm. 2 Equal to or more than 10 cm. 3 Between 5 and 10 cm. 4 More than 25 cm

1 An AFI less than 5 cm indicates oligohydramnios. Oligohydramnios is associated with intrauterine growth restriction and congenital anomalies. An AFI of 10 cm or greater indicates that the fetus is normal. AFI values between 5 and 10 cm are considered low normal, indicating a comparatively low risk for congenital anomalies. An AFI greater than 25 cm indicates polyhydramnios. This is associated with neural tube defects and obstruction of the fetal gastrointestinal tract.

A mother tells the nurse that she will visit her 2-year-old son tomorrow about noon. During the child's bath, he asks for mommy. The nurse's best reply is: 1 "Mommy will be here after lunch." 2 "Mommy always comes back to see you." 3 "Your mommy told me yesterday that she would be here today about noon." 4 "Mommy had to go home for a while, but she will be here today."

1 Because toddlers have a limited concept of time, the nurse should translate the mother's statement about being back around noon by linking the arrival time to a familiar activity that takes place at that time. Saying that the child's mother will always return does not give the child any information about when his mother will visit. Twelve noon is a meaningless concept for a toddler. Saying generally that the child's mother will visit does not give the child specific information about when his mother will visit.

Biochemical examination of the amniotic fluid of a pregnant patient yields the following results: lecithin-to-sphingomyelin (L/S) ratio, 2:1; surfactant-to-albumin (S/A) ratio, 60 mg/g; and phosphatidylglycerol (PG) present. What conclusions will the nurse draw from this report? 1 The fetal lungs are well developed. 2 The gestational age is 36 weeks. 3 The fetus has a neural tube defect. 4 The fetus has an open neural tube defect.

1 Biochemical findings such as an L/S ratio of 2:1, an S/A ratio of 60 mg/g, and the presence of PG in amniotic fluid indicate that the fetal lungs are well developed. The gestational age can be predicted only with the help of creatinine and lipid levels in the amniotic fluid. Creatinine levels greater than 2 mg/dL in amniotic fluid indicate that the gestational age is more than 36 weeks. The presence of alpha-fetoprotein (AFP) in the amniotic fluid indicates a neural tube defect in the fetus. The nurse needs to assess AFP levels in the amniotic fluid to determine whether the fetus has an open neural tube defect. A high AFP level in amniotic fluid after 15 weeks' gestation indicates that the fetus has an open neural tube defect.

The nurse is teaching a pregnant woman to eat a nutritious diet and to attend regular antenatal health check-ups for the assessment of fetal well-being. The primary purpose of this nursing intervention is to reduce the neonatal mortality rate due to: 1 birth weight less than 2.5g. 2 gestational diabetes in mother. 3 birth weight of more than 3.5 g. 4 febrile convulsions in neonate

1 Birth weight of less than 2,500 grams or 5.5 pounds in considered low birth weight (LBW). LBW is associated with higher neonatal mortality rate in the United States when compared with other countries. The lower the birth weight, the higher the mortality rate. Birth weight of the neonates born to uncontrolled diabetic mother can be high. The mortality rate of neonates born with a birth weight of more than 3.5 g is lower than that of neonates born with LBW. Febrile convulsions seldom cause death in neonates. Test-Taking Tip: Key words or phrases in the stem of the question such as first, primary, early, or best are important. Similarly, words such as only, always, never, and all in the alternatives are frequently evidence of a wrong response. As in life, no real absolutes exist in nursing; however, every rule has its exceptions, SO answer with care.

During their school-age years, children best understand concepts that can be seen or illustrated. The nurse knows this type of thinking is termed as: 1 concrete operations. 2 preoperational. 3 school-age rhetoric. 4 formal operations.

1 Black-and-white reasoning involves a situation in which only two alternatives are considered, when in fact there are additional options. Preoperational thinking is concrete and tangible. During the school-age years, children deal with thoughts and learn through observation. They do not have the ability to do abstract reasoning and learn best with illustration. Thought at this time is dominated by what the school-age child can see, hear, or otherwise experience. School-age rhetoric simply refers to the type of ideas that arise out of the years children attend school. Formal operations are characterized by the adaptability and flexibility that occurs during the adolescent years. Test-Taking Tip: Answer every question because, on the NCLEX exam, you must answer a question before you can move on to the next question.

A child has recently been admitted to the hospital. The child's parents have not yet arrived at the hospital. What behavior is the child exhibiting that leads the nurse to believe the child is exhibiting the stage of protest? The child: 1 Screams and hits the nurse. 2 Is withdrawn from others. 3 Has the habit of bed-wetting. 4 Sits in a corner with a toy.

1 Children often undergo separation anxiety when they are separated from their parents. This separation anxiety manifests in different stages such as protest, despair, and detachment. Protest is the first stage of separation anxiety, during which the child screams, cries, or hits the other person for separating him or her from the parents. After this stage, the child enters the stage of despair, where the child begins to withdraw from others and stay depressed. During this stage, the child starts wetting the bed and sucking the thumb because of fear and anxiety. After the stage of despair, the child enters the stage of detachment. The child starts interacting with strangers and takes an increased interest in the surroundings or sits in a corner and plays with a toy.

The nurse sees a woman for the first time when she is 30 weeks pregnant. The woman has smoked throughout the pregnancy, and fundal height measurements now are suggestive of growth restriction in the fetus. In addition to ultrasound to measure fetal size, what tool is useful in confirming the diagnosis? 1 Doppler blood flow analysis 2 Contraction stress test (CST) 3 Amniocentesis 4 Daily fetal movement counts

1 Doppler blood flow analysis allows the examiner to study the blood flow noninvasively in the fetus and placenta. It is a helpful tool in the management of high risk pregnancies because of intrauterine growth restriction (IUGR), diabetes mellitus, multiple fetuses, or preterm labor. Because of the potential risk of inducing labor and causing fetal distress, a CST is not performed on a woman whose fetus is preterm. Indications for an amniocentesis include diagnosis of genetic disorders or congenital anomalies, assessment of pulmonary maturity, and the diagnosis of fetal hemolytic disease, not IUGR. Fetal kick count monitoring is performed to monitor the fetus in pregnancies complicated by conditions that may affect fetal oxygenation. Although this may be a useful tool at some point later in this woman's pregnancy, it is not used to diagnose IUGR.

The senior nurse instructed the student nurse to check the website, www.ndvh.org. Information about which topic will the student nurse find on this website? 1 Domestic violence 2 Malnourishment 3 Infant mortality rate 4 Hospital administration

1 In USA, www.ndvh.org is the official website of the National Domestic Violence Hotline. Pediatric nurses can find valuable information on this website related to the signs of exposure to violence in children. This website also provides nonviolent problem-solving strategies, counseling, and referrals for children who are the victims of domestic abuse. Information about malnourishment, infant mortality rates, and hospital administration is not available at this website.

A week-old newborn is assessed for body weight, birth marks, and height. The birth weight is lower than what it should be for height. Which physical feature of the newborn makes the nurse conclude that the newborn is affected by Down syndrome? 1 Short and broad neck 2 Long and thin fingers 3 Short and thin lips 4 Broad and long nose

1 One of the characteristics of Down syndrome is a short, broad neck. These children have an impaired immune system and are at risk for spinal cord compression. Physical features such as long and thin fingers, short and thin lips, and broad and long nose are all common in a normal child and do not indicate any abnormality.

A patient is put on a ventilator in the intensive care unit of a tertiary hospital for long-term care. While caring for the patient, the nurse continuously assesses the health status of the patient. How does this intervention affect the patient's outcome? It prevents: 1 pneumonia. 2 lung cancer. 3 cystic fibrosis. 4 pulmonary edema.

1 One of the major disadvantages of long-term ventilator care is ventilator-associated pneumonia. The nurse has to assess the patient for ventilator-associated pneumonia or for the early detection of respiratory complications due to ventilators. Lung cancer is not associated with ventilator support. Cystic fibrosis is due to defect in chromosome number 7. Pulmonary edema is not the main concern associated with ventilators in the ICU.

On assessing a large family, the nurse finds one of the adolescents is less peer-oriented, lacks autonomous inner controls, and does not tend to participate democratically in family interactions or depend on parents for advice. Which behavior of the adolescent will the nurse think to be uncharacteristic of an adolescent belonging to a large family? 1 Less peer orientation 2 Democratic participation 3 Autonomous inner controls 4 Depending on parents for advice

1 Peer orientation is typically seen in adolescents from large families. Democratic participation, developed autonomous inner controls, and depending on parents for advice are not normal behavioral patterns seen in adolescents from large families.

The nurse is preparing to administer a vaccine to a child. The child is refusing to take the vaccination because of fear of bleeding. What should the nurse do in this situation? 1 Tell the child he or she can pick the bandage color. 2 Tell the child bleeding will stop in a few seconds. 3 Request a staff member sit beside the child. 4 Give a favorite toy to the child for distraction.

1 The child is refusing to take vaccination because of fear of bleeding and pain. The nurse should ask the child to select the color of the bandage to be used. This reassures the child and will make him or her feel better. Even if the nurse tells the child that the bleeding will stop when the needle is removed, it does little to help relieve the child's fear. The nurse should not scold the child in a firm tone because the child may get frightened. Giving a favorite toy to the child for playing is not helpful for relieving the fear. A favorite toy may help the child sleep at night. Requesting a staff member sit beside the child may not be helpful for relieving the child's fear. It may be needed to help hold the child still during a procedure.

A child is hospitalized for treatment of the flu. Once the child's parents leave, the child starts crying, looks for parents, attempts to leave, refuses to take medicine, hits other children, and breaks toys. What should the nurse conclude from the child's behavior? The child is in the: 1 Protest stage. 2 Despair stage. 3 Denial stage. 4 Detachment stage.

1 The child's behavior indicates that the child is in the protest stage of separation anxiety. The child is less able to cope with separation because of stress from the illness and wants to stay with the parents. The child expresses anger indirectly by showing behavioral changes. These behavioral changes are observed in the protest stage of separation anxiety. In the despair stage, the child appears less active, depressed, and uninterested in play and refuses to eat food. The denial stage is also called the detachment stage. In this stage the child is interested in the surroundings, plays with others, and forms new but superficial relationships with others.

The nurse is obtaining the admission history of a recently admitted adolescent. The nurse notes the patient requires help inserting contact lenses. Under which functional health pattern should the nurse record this observation? 1 Activity-exercise pattern 2 Cognitive-perceptual pattern 3 Nutrition-metabolic pattern 4 Health perception-health management pattern

1 The nurse records the admission history of the patient in terms of different functional health patterns. This helps in documenting all the required information about the patient. The patient requires help inserting his or her contact lenses. This implies that the patient needs support to perform an activity. The nurse should record this information under the activity-exercise pattern. The cognitive-perceptual pattern recognizes the cognitive development in the child and includes information such as defects in vision, hearing, or grading in the school. The nutrition-metabolic pattern is used in the assessment of nutrition in the patient, food allergies, and food intake habits. The health perception-health management pattern reports the medication and the health history of the child.

A child who was terminally ill died. What intervention should the nurse provide for the child and the family? 1 Allow the family be with and hold or rock the child. 2 Do not remove any of the tubes or catheters. 3 Follow the hospital policy to administer last rites. 4 Instruct the family that the nurse bathes the body.

1 The nurse should be very sensitive to the emotions of the family members who are grieving their loss and try to support them as much as possible. The nurse should allow privacy to the family with the child and allow them to hold or rock the child if they want. The nurse should remove the tubes and catheters from the child's body and let the family hold the child. The nurse should allow the family to provide last rites according to their culture rather than following hospital policy. It helps the family cope. The family should be allowed to bathe and dress the child, as per their cultural practice.

A patient in the sixth month of pregnancy expresses her wish to see the fetus. What investigation does the nurse suggest for the patient to help her see the fetus? 1 Ultrasonography 2 Magnetic resonance imaging (MRI) 3 Computed tomography (CT) 4 Nuchal translucency (NT)

1 Three-dimensional (3D) or four-dimensional (4D) ultrasonography is advisable for women who want to see the fetus. MRI cannot be used in this case because it requires the fetus to be still for a long period of time for a clear image. CT uses ionizing radiation for imaging, which can be harmful to the fetus. Therefore CT is contraindicated for fetal imaging. NT is a specific ultrasonography screening procedure used to test for genetic abnormalities in the fetus.

Which pattern of growth and development would be seen when a parent is working with a toddler to read the alphabet? 1 Sensitive 2 Sequential 3 Directional 4 Developmental

1 When the child is learning or training and going through psychologic changes, the child is going through the sensitive phase of development and growth pattern. Sequential growth patterns are definite growth patterns where the child crawls before standing and stands before walking. Directional development refers to early development and symmetrical growth along with central and peripheral nervous system development. Developmental pace focuses on growth patterns of the child as toddler, preschooler, and adolescent.

Which physiologic parameters does the nurse check in the ultrasound report to assess fetal well-being? Select all that apply. 1 Amniotic fluid volume (AFV) 2 Fetal breathing movements (FBMs) 3 Fetal limb and head movements 4 Daily count of fetal movements 5 Fluid volume in the nape of the fetal neck

1, 2, 3 Assessment of physiologic parameters such as AFV, FBMs, and limb and head movements of the fetus by ultrasonography gives a reliable picture of fetal well-being. Abnormalities in the amniotic fluid volume are frequently associated with fetal disorders. Fetal breathing and limb and head movements reflect the status of the central nervous system. Daily fetal movement count is the most common method used to assess fetal activity. Ultrasound is not used to assess the daily fetal movement count. The fluid volume in the nape of the fetal neck is measured to assess structural abnormalities in the fetus.

Which child behaviors indicate maladaptive coping toward chronic illness? Select all that apply. 1 Is irritable both at home and during school 2 Demonstrates feelings of pessimism and inadequacy 3 Expresses feelings of worthlessness to others 4 Expresses anxiety during remissions 5 Demonstrates confidence during an exacerbation

1, 2, 3 Children with negative and declining attitudes toward their chronic illness use maladaptive coping patterns. These maladaptive coping patterns are characterized by irritability at home and school. The child has a pessimistic outlook toward the disease and feels inadequate in managing it. The child may feel different, withdrawn, and less worthy than others. The child who has healthy coping skills may expresses appropriate emotions, such as anxiety, sadness, and anger, during periods of illness exacerbations. Demonstrating confidence during remissions indicates healthy coping skills

What information should the nurse provide to a group of nursing students when teaching them about the analgesic stepladder for pain management in children with terminal illness? Select all that apply. 1 "It was implemented by the World Health Organization (WHO)." 2 It provides for the use of adjunctive drug therapy to ease pain." 3 It outlines the principles of analgesic selection and titration." 4 "It describes the use of combination therapy to decrease side effects." 5 It was created to prevent drug dependence and drug addiction."

1, 2, 3 Children with terminal illness experience some amount of pain in the last stage of their illness. Nurses should follow the analgesic stepladder framed by World Health Organization (WHO) for pain management in children. It outlines the use of adjunctive therapy depending on the severity of the pain as mild or severe/persistent. It outlines the principles of analgesic selection such as administration of nonnarcotic analgesics to ease mild pain and administration of narcotic analgesics (opioids) to ease severe or persistent pain. It outlines the use of combination therapy to counteract adverse effects, not side effects. A patient with a terminal illness will require increased levels of pain medication to manage pain. The patient will not become dependent or addicted.

Which approaches should the nurse adopt to provide support and guidance to a grieving family? Select all that apply. 1 Follow up regularly with telephone calls. 2 Provide a referral to a support group. 3 Recommend bereavement counseling. 4 Refrain from sending them cards. 5 Ask the family come for an office visit.

1, 2, 3 Nurses play an important role in helping the grieving family cope with their loss. The nurse should plan regular follow-ups through phone calls or meetings to assess whether they are coping well. Support groups and bereavement counseling may help the family vent their feelings and learn coping skills. The nurse should initiate contact by sending cards. The nurse should not expect the family to come in for a visit. The nurse should ask permission to make a home visit to make it easier on the family. This helps the family feel cared for.

The nurse is assessing a toddler's psychosocial development using Erikson's theory. What should the nurse include in the evaluation? Select all that apply. 1 Gross and fine motor skills 2 Mental acuity and capability 3 Level of doubt and shame 4 Competition with others 5 Inadequacy or inferior feelings

1, 2, 3 The stage in Erikson theory of psychosocial development that is used for toddlers (1-3 years) is autonomy versus shame and doubt. In this stage the toddler's motor skills, such as walking and climbing, are evaluated. The toddler's mental acuity and thought processes are also evaluated. The toddler develops negative feelings of doubt and shame when feeling low at this stage of growth. In the middle childhood growth pattern, the child tends to compete with others, aiming to accomplish tasks. This stage is referred to as industry versus inferiority in child. Inadequacy or inferiority complexes arise in this stage when parents impose huge expectations on the child. They tend to feel inferior in this stage of development.

The pediatric nurse is providing first aid to a child. The child sustained minor injuries while playing on the ground, and has severe pain in the knee joint. Which of the nursing interventions in the care of the child come under atraumatic care? Select all that apply. 1 Controlling pain 2 Allowing the child's privacy 3 Respecting cultural differences 4 Fostering the parent-child relationship 5 Giving vaccination for preventing tetanus

1, 2, 3, 4 Atraumatic care refers to the provision of the therapeutic interventions that would minimize or eliminate a patient's physical and psychologic distress. Atraumatic care includes interventions to reduce physical distress such as pain control measures. Interventions such as allowing the child privacy, respecting cultural differences, and fostering the parent-child relationship are examples of interventions those are helpful in minimizing psychologic distress in the child. Preventive measures such as vaccination for preventing tetanus are not an intervention included in a traumatic care.

The nurse is instructing the parents of a 6-month-old child about the dietary requirements and factors that may influence the eating habits of the child. Which statement made by the nurse is appropriate? Select all that apply. 1 "Culture will have some influence on children's eating habits." 2 "Cholesterol is required for the synthesis of neurons in child's brain." 3 "During adolescence, children tend to make food choices for sociability." 4 "First 3 years of life are crucial in establishing eating habits of children." 5 "Cholesterol content is high in nuts and vegetable oils so use them sparingly."

1, 2, 3, 4 The nurse should inform the parents that culture has some influence on children's eating habits, and the child is likely to follow it. Cholesterol is required for the synthesis of neurons in the child's brain and should be included in the diet. During adolescence, children tend to make food choices for sociability and the first 3 years of life are crucial in establishing eating habits of children. Cholesterol is present only in animal products such as meat, milk, and eggs but not present in plant products.

The nurse is assessing a school-age child. The child stays with a parent who is recently divorced and has a meager income. The child does not like to mingle with other students at school. The child's performance is poor in studies and is cruel toward pets at home. Which factors in the child could most likely lead to pediatric social illness? Select all that apply. 1 Poverty 2 Pet cruelty 3 Single parent 4 Going to school 5 Behavior with others

1, 2, 3, 5 Pediatric social illness is a new morbidity in children. It refers to "the behavior, social, and educational problems that the children face". Poor socioeconomic status is a social problem. Animal cruelty is a behavioral problem. Problem within the family is a social problem Failure at school is an educational problem, and behavior with other children is also a behavioral problem. Any of these could cause pediatric social illness. Going to school does not cause social illness. It helps the child to gain knowledge, learn moral values, and to lead a successful life.

The parents of a 12-year-old child are informed that the child is in the terminal stages of osteosarcoma. The nurse explains to the parents about the care that needs to be provided for the child. Which characteristics in the parents indicate that they are in denial? Select all that apply. 1 Want the child to be admitted for more treatment 2 Refuse to believe the results of the tests 3 Pleased to receive the medical diagnosis 4 Ask questions about treatment or prognosis 5 Insist that nobody is telling them the truth

1, 2, 3, 5 The initial diagnosis of a terminal illness in a child is met with shock or denial. Parents experience this reaction, and it can last for days, weeks, or months. If the parents ask questions about the child's treatment or prognosis, this indicates that they have accepted the diagnosis. Parents in denial may want the child to be admitted for more medical treatment. They may feel the terminal diagnosis is wrong and refuse to believe the test results. People are usually not pleased to receive a terminal diagnosis; they are upset and angry. Insisting that nobody is telling them the truth is also an indicator of denial.

What are the various guidelines that the nurse has to follow for the admission of a child into the hospital? Select all that apply. 1 Apply an identification band on the child's wrist. 2 Obtain the nursing admission history of the child. 3 Specimens for lab tests should not be taken. 4 Orient the parents about the inpatient facilities. 5 Hospital regulations should not be disclosed.

1, 2, 4 The nurse has to follow a few guidelines while admitting the child to the hospital. The nurse should apply an identification band on the child's wrist. This helps in providing appropriate care to the child. The nurse should take the nursing admission history in order to help to identify needs of the child. The nurse should explain to the parents as well as the child about the inpatient facilities. This would make them comfortable within the hospital. Specimens should be collected and may even be ordered for other specimens upon admission. The nurse should orient the parents and the child about the hospital regulations and schedules like visiting hours and food timings.

Which behaviors of the family members indicate avoidance coping behavior? Select all that apply. 1 Refuses to agree to the treatment 2 Demonstrates anger and hostility 3 Expresses deep feelings of sorrow 4 Makes realistic plans for the future 5 Seeks cures from the faith healers

1, 2, 5 Coping reactions to stressors include those that indicate avoidance. Avoidance includes behaviors that help the person deny the truth or escape from it. Refusing to agree to treatment, being angry toward the staff, and seeking nontraditional medical treatments such as faith healing are examples of avoidance behavior. Expressing feelings of sorrow and planning realistically for the future are approach behaviors that indicate acceptance of the condition.

The nurse plans to assess the role-relationship pattern in a child. Which questions should the nurse ask the parents? Select all that apply. 1 "Does the child have any security objects at home?" 2 "How do you handle discipline problems at home?" 3 "Have you ever noticed that your child sweats a lot?" 4 "How does your child usually handle disappointments?" 5 "Have any major changes in the family occurred lately?"

1, 2, 5 For assessing the role-relationship pattern in the child, the nurse should ask the parents about any security objects the child may have at home that provide comfort, discipline problems of the child, and family changes. From this information, the nurse can understand the relationship between the parents and the child. Information about sweating gives an idea about the elimination pattern in the child. Information about the disappointment handling potential of the child gives an idea about the child's coping-stress tolerance pattern.

The parents of a terminally ill child ask the nurse how hospice care is different from hospital care. What information should the nurse give them? Select all that apply. 1 No extraordinary efforts would be made to prolong the child's life. 2 Pain and control of symptoms are the primary concerns of treatment. 3 The child's needs are considered to be the priority over the family's needs. 4 Hospice professionals are solely responsible for providing care to the child. 5 Family members are the principal caregivers supported by the hospice team.

1, 2, 5 Nurses should offer information about hospice care to the parents of a terminally ill child. Hospice is a community health care organization which specializes in caring for a dying child. It combines principles of hospice care and palliative care. The client is taken care of at home in the final stages of life. The concepts that make it different from hospital care include: no extraordinary efforts are made to prolong the child's life, and pain and control of symptoms are the primary concerns of treatment. The family members are usually the principal caregivers supported by a team of professionals, and the family's needs are given the same importance as the child's needs.

What are the unique features of a primary social group as part of social and cultural roles? Select all that apply. 1 The members of the social group are intimate with each other. 2 The members of the social group are mutually supportive. 3 The members of the social group a part of professional association. 4 The member of the social group has less concern for other member's behavior. 5 The behaviorof the individual member of a social group is constrained

1, 2, 5 Primary and secondary are the two types of social groups. The primary group includes family and peer, and is characterized by intimate and face-to-face contacts. The features of primary social group are intimacy, mutual support, and constraint of individual members' behavior. Professional association and less concern for members' behavior are features of a secondary social group. STUDY TIP: Avoid planning other activities that will add stress to your life between now and the time you take the licensure examination. Enough will happen spontaneously; do not plan to add to it.

What are the ways the nurse may help a family meet its needs based on family strength and functioning styles? Select all that apply. 1 Providing individualized support 2 Strengthening the family resources 3 Helping to delineate the individualized work 4 Training the family members to avoid stressful events 5 Building on qualities that make family function in a better manner

1, 2, 5 Providing individualized support, strengthening the family resources, and building on qualities that makes the family work better are the ways a nurse may help the family to meet its demands. Efforts should be made to share work in groups rather than delineating work. Families actually cope and respond to stressful events, which has to be identified and appreciated.

The parents of a teenaged child tell the nurse they are worried that their child is hanging out with the wrong type of friends. What can the nurse advise the parents to do to help the adolescent make better choices? Select all that apply. 1 Provide support and love at home to make the child feel wanted. 2 Bring the adolescent to an inpatient psychiatric facility. 3 Establish clear and concise boundaries, rules, and expectations. 4 Talk to the child and allow the child to make her own decisions. 5 Ground the child until she shows appropriate behavior with friends.

1, 3, 4 In order to be an effective parent, the family should convey four external assets to the child so that the child can learn to make the correct choices. The parents should provide support, care, and love at home and in the community. It is also important to establish clear and concise boundaries and expectations both at home and in the community. It is also important for parents to discuss good decision-making strategies with their children to empower them to make correct choices. The adolescent is not displaying any behaviors that warrant an inpatient psychiatric admission, nor does she need to be grounded.

Children are taught the values of their culture through observation and feedback on their own behavior. A nurse teaching a class on cultural awareness-competence should be aware of which factor(s) that may be culturally determined? Select all that apply. 1 Social roles 2 Racial variation 3 Degree of competition 4 Determination of status 5 Geographic

1, 3, 4 Social roles are influenced by culture. Cultures that value individual resourcefulness/competition of status is acceptable. Determination of status is culturally determined and varies according to each culture. Racial variation refers to transmissible traits. Culture is composed of beliefs, values, practices, and social relationships that are learned. Cultural development may be limited by geography. The geographic boundaries are not culturally determined.

The nurse is assessing a patient with strabismus. Which finding would suggest the cause of strabismus? Select all that apply. 1 Poor vision 2 Short eyeball 3 Congenital defect 4 Muscle imbalance 5 Unequal curvature in the lens

1, 3, 4 Strabismus may result from poor vision and the resulting straining of eye muscles. Strabismus may result from a congenital defect as a developmental anomaly. Strabismus may also result from muscle imbalance caused by neuromuscular disorders. Short eyeball results in development of hyperopia, not strabismus. Unequal curvature of lens results in astigmatism, not strabismus.

What instructions should the nurse give to parents who have adopted a child? Select all that apply. 1 "You should reveal the adoption information as soon as possible." 2 "You should ask a third party to reveal the adoption information." 3 "You should always be honest and open toward the child." 4 "You should never reveal to the child that the child is adopted." 5 "Revealing adoption information causes concern and anxiety in children."

1, 3, 5 Discussing adoption with the adopted child requires utmost care. The parents should always be honest with their children. This strengthens the relationship between the parents and the adopted child. Revealing information early leads to less chance of misunderstanding and better continuance of relationship. Adoption information causes concern and anxiety in children but should not be avoided. Information about the adoption has to be revealed before any third party reveals it. Adoption information should not be withheld; the child has the right to know about the parents.

Parents inform the nurse that they had noticed some needle injuries on their child's left elbow and some syringes and needles in the child's school bag. What should the nurse suggest to the child's parent? Select all that apply. 1 Encourage the child to participate in scouts. 2 Discourage the child from participating in sports due to injury. 3 Encourage the child to participate in church activities. 4 Provide first aid to the child and apply bandage to elbow. 5 Educate the parent and children about the ill effects of drugs.

1, 3, 5 The nurse should understand that the child is taking to illicit drugs. Preventive measures to reduce the youth's illicit drug use include encouraging participation in organized sports, scouts, and other church activities. The children and parents should be educated on the ill effects of drugs. Sports are not contraindicated and first aid is not a preventive measure.

A hospitalized child is being released for home health care. What suggestions should the nurse provide to prepare the family for transporting the child home? Select all that apply. 1 Take a basin in case of vomiting 2 Avoid using the restraint system 3 Keep a blanket and pillow in the car 4 Discourage the use of a straw for drinking fluids 5 Administer prescribed pain medication before leaving

1, 3, 5 The parents should use a basin or plastic bag for managing vomiting in the child. A blanket and pillow should be kept in the car to provide comfort. Pain medication can be administered before leaving to provide a pain-free journey home. The use of a car safety restraint system should be encouraged for the child's safety. Also, the use of a straw for drinking fluids should be encouraged except for children with oral facial surgeries.

A baby is diagnosed with Turner syndrome shortly after birth. The parent is showing signs of depression and is finding it difficult to bond with the baby. Which characteristics indicate that the parent is in the adjustment stage? Select all that apply. 1 Assumes that the disability is genetically inherited 2 Acts happy and optimistic despite the diagnosis 3 Focuses on caregiving as the condition progresses 4 Believes the disability happened during pregnancy 5 Believes the disability is a punishment from God

1, 4, 5 The adjustment stage is the next phase after shock and is most often characterized by an open admission that the condition exists. This stage is accompanied by responses such as guilt. Many parents may feel responsible for the disability because it was genetically inherited. The mother may also believe she caused the disability by doing something wrong during pregnancy or may believe God is punishing her for previous misdeeds. Acting happy and optimistic despite the revealed diagnosis indicates the parent is still in denial and has not accepted the child's disability. Focusing on new caregiving practices as the condition progresses indicates total acceptance of the condition.

A female patient tells the nurse, "I am well educated and would like to start working full-time like my husband." How should the nurse respond? Select all that apply. 1 "You might potentially get really stressed with work and house chores." 2 "You may at times direct your stress toward children." 3 "Dual income would help you to lead a joyous life." 4 "You may not have adequate time for social activities." 5 "You may have problems fulfilling time demands."

1, 4, 5 Work overload is a common source of stress in a dual-earner family as both partners need to share the household chores. Social activities are significantly curtailed as most time is devoted in meeting the responsibilities of the household. Time demands and scheduling are major problems for all individuals who work. Dual income provides more economic stability rather than making them happy. It is usually an indirect stress to the child.

The nurse is reviewing mortality indicators in a city with a population of 30 million. In 2011, there were 3000 total live births, 60 stillbirths, 60 deaths younger than 28 weeks of age, 30 deaths younger than 4 weeks of age, and 90 deaths younger than 1 year (includes all subcategories of deaths younger than 1 year). What is the neonatal mortality rate of the city? Record your answer using a whole number. _____ per 1000 live births

10 Neonate refers to a baby who is younger than 28 days of age. The number of deaths younger than 28 days, or 4 weeks, was 30. Therefore, neonatal mortality rate = number of deaths younger than 28 days (4 weeks) / number of live births or 30 / 3000 = 10 / 1000.

The number of deaths of children in community are 24, 86, 100, and 200 respectively for the children under the age of 4 weeks, 4 months, 28 weeks, and 8 months per 2,000 live births. What is the neonatal mortality rate? Record your answer using whole number. _____/1,000 live births

12 Neonatal mortality rate is the number of deaths during the first 28 days of life per 1,000 live births. In this case, death rate of children under 4 weeks (28 days) in this community is 24/2,000 live births so this it is equal to 12/1,000 live births.

A new mother asks the nurse when the "soft spot" on her son's head will go away. The nurse's answer is based on the knowledge that the anterior fontanel closes after birth by _____ months.

18

Which behavior would most likely be manifested in a young child experiencing the protest phase of separation anxiety? 1 Inactivity 2 Clings to parent 3 Depressed, sad 4 Regression to earlier behavior

2 In the protest phase, the child aggressively responds to separation from parents (such as clinging to a parent). Inactivity is characteristic of despair. Depression and sadness are characteristics of despair. Regression to earlier behavior is characteristic of despair.

The nurse finds that a newborn infant weighs approximately 3 kg (7 lb). Approximately how much would the child weigh when he reaches 2.5 years of age? 1. 9 kg (20 lb) 2. 12 kg (27 lb) 3. 15 kg (33 lb) 4. 17 kg (38 lb)

2 A baby typically quadruples in weight by the time the child reaches 2 to 2.5 years of age. A child who weighed 3 kg (7 lb) at birth should weigh four times as much as that by 2.5 years, or 12 kg (27 lb). Birth weight triples by the age of 6-12 months, so the child's weight would be 9 kg (20 lb) at 6-12 months. The child would attain a body weight of 15 kg (33 lb) and 17 kg (38 lb) after 2.5 years of age.

The nurse is assessing a newborn who weighs 3 kg (7 lb). At what growth stage would the child weigh 12 kg (26 lb)? 1 Infancy 2 Toddlerhood 3 Preschool age 4 School age

2 A child's weight should quadruple by the toddler stage. The child's weight would be 12 kg if it was 3 kg at birth. Birth weight triples by the end of infancy, so it would be 9 kg if it is 3 kg at birth. After the toddler weight is achieved, an annual weight increase of 2-3 kg is seen in both preschool- and school-age children. STUDY TIP: You have a great resource in your classmates. We all have different learning styles, strengths, and perspectives on the material. Participating in a study group can be a valuable addition to your nursing school experience.

A nurse is examining a toddler and is discussing with the mother psychosocial development according to Erikson's theories. Based on the nurse's knowledge of Erikson, the most age-appropriate activity to suggest to the mother at this stage is to: 1 feed lunch. 2 allow the toddler to start making choices about what to wear. 3 allow the toddler to pull a talking-duck toy. 4 turn on a TV show with bright colors and loud songs

2 A toddler is developing autonomy and is able to start making some choices about what he or she can wear. A toddler is developing autonomy and focusing on doing things for himself or herself and therefore would not want the mother to feed him or her. The child is at the stage of autonomy versus shame and doubt, as defined by Erikson. At this age, the mother should provide opportunities for the child to be active and learn by experience and imitation. Providing toys the child can control will help achieve this stage. A toddler might easily become overstimulated by images from TV and loud sounds. Toddlers are more interested in manipulating and learning from objects in the environment.

The parent of a 2-year-old child tell the nurse that the child likes to play alone and asks people to repeat questions several times. The parent also says that the child uses gestures to communicate. What should the nurse infer from this? The child has: 1 Cognitive impairment. 2 Difficulty hearing. 3 Normal development. 4 Chronic mental illness.

2 Children 2 to 3 years old understand the common language used at home, and they try to communicate with family members in the same language. If the child has difficulty understanding and responding after the parent repeats a statement several times, this may indicate the child has a hearing problem. The child does not have lack of orientation, so the nurse should not infer that the child has cognitive impairment. Children stop using gestures and start communicating verbally around the age of 15 months. Therefore the child does not have normal development. The child is not bullying or being aggressive, so the nurse should not infer that the child has a chronic mental illness.

In what age group should the nurse expect a child to develop gross motor skills? 1 Birth through infancy 2 Early childhood 3 Later childhood 4 Middle childhood

2 Children between 1 and 6 years of age are considered to be in early childhood. This is when motor skills such as walking, talking, and climbing develop. The nurse should assess the child to determine whether the child is progressing appropriately. The period from birth to 12 months is called infancy. This is when the relationship between parent and child develops. This increases trust in the baby, and gross motor activity is not prominent at this growth stage. Later childhood is from 11 to 19 years of age. It is a period of rapid maturation and a point of entry into adulthood. Middle childhood encompasses the age group from 6 to 10 years old. This is when steady physical, mental, and social development is seen in children.

A camp nurse is assessing a group of children attending summer camp. Based on the nurse's knowledge of special parenting situations, which group of children is at risk for a sense of belonging? 1 Children adopted as infants 2 Children recently placed in foster care 3 Children whose parents recently divorced 4 Children who recently gained a stepparent

2 Children placed in foster care are at greater risk to have problems perceiving a sense of belonging. Children adopted at birth have fewer problems with acceptance when parents follow preadoption counseling about disclosure. Children of divorced parents often fear abandonment. Children who gain a stepparent are at risk for having trust problems with the new parent.

The nurse observes a child having difficulty getting a mobile phone to work and looking puzzled. What type of play is the child demonstrating? 1 Skill play 2 Dramatic or pretend play 3 Social-effective play 4 Sense-pleasure play

2 Children's play activities are categorized during each stage of development. Pretend play, which is also called dramatic play, is seen in 11- to 13-month-old children when they perform activities that might be puzzling or frustrating to them. Skill play is seen after infants have developed the ability to grasp objects with their hands and manipulate them. This is when they use their skills to do things they observe such as putting paper in and out of a toy. Social-effective play is seen in infancy, when infants take pleasure in relationships with people. Sense-pleasure play happens when infants become attracted to natural colors or things and focus intently on them. An example is playing with sand.

How should the nurse obtain cultural knowledge? 1 By interacting with people of different cultures 2 By seeking formal and informal education about various cultures and cultural practices 3 By learning to appreciate different cultural values 4 By seeking training in various nursing interventions

2 Cultural competence has five components. Cultural knowledge is formal and informal education about various cultures and cultural practices. It does not involve interacting with people from different cultures -- this is a cultural encounter. It also does not involve cultural appreciation by the nurse; this is cultural awareness. It does not consist of knowledge of nursing interventions.

Often parents are confused by the terms growth and development and use the terms interchangeably. Based on the nurse's knowledge of growth and development, the most appropriate explanation of development is: 1 a child grows taller all through early childhood. 2 a child learns to throw a ball overhand. 3 a child's weight triples during the first year. 4 a child's brain increases in size until school age.

2 Development is the mental and cognitive attainment of skills. Growth is the increase in physical size—both height and weight.

A patient who is undergoing stem cell therapy asks the nurse about undifferentiated cells. Which response given by the nurse is most appropriate? "These cells:" 1 are able to divide at a very rapid rate." 2 multiply to form any part of the body." 3 can perform specialized functions." 4 are similar to all other cells in the body."

2 Differentiation is the process by which immature cells transform into mature cells to form tissues. Thus undifferentiated cells are immature and not developed. These cells would be able to reproduce to form any part of the body. These cells are not similar to cancer cells and do not multiply rapidly. These cells are immature and are not able to perform specialized functions. These cells are not well developed and thus do not resemble other mature cells of the body.

The nurse works in a pediatric unit. Which child would have an increased vulnerability to the stresses of hospitalization? 1 A female child 2 A child with a difficult temperament 3 A child with an average intelligence 4 A child older than 6 years of age

2 Hospitalization is a stressor in children and so they may react differently to it. Certain children are more susceptible to the stressful effects of hospitalization than others. Children who have difficult temperament may not readily adjust with the unfamiliar environment of the hospital. These children may experience adverse effects of hospitalization. Female children are able to withhold stress more when compared to male children and thus are less likely to experience stressors. Children with average intelligence may be able to understand their condition and the importance of hospitalization and thus may be more adaptable. Children with lower IQ would not understand the purpose of hospital admission and thus would be extremely stressed due to hospitalization. Children who are older than 6 years of age have developed the maturity to understand their condition and the purpose of hospitalization. Thus, they would be more adaptable to their condition, and experience less stress related to hospitalization.

The nurse needs to start an intravenous (IV) line on an 8-year-old child to begin administering intravenous antibiotics. The child starts to cry and tells the nurse, "Do it later, O.K.?" The nurse should: 1 start the IV line because allowing the child to manipulate the nurse is bad. 2 start the IV line because unlimited procrastination results in heightened anxiety. 3 postpone starting the IV line until the child is ready so that the child experiences a sense of control. 4 postpone starting the IV line until the child is ready so that the child's anxiety is reduced.

2 Intravenous antibiotics are a priority action for the nurse. A short delay may be possible to allow the child some choice, but a prolonged delay only serves to increase the anxiety. The nurse should start the IV line, recognizing that the child is attempting to gain control. If the timing of the IV line start was not essential for the start of IV antibiotics, postponing might be acceptable. The child may never be ready. The anxiety is likely to increase with prolonged delay.

The nurse is caring for a dying boy whose religion is Islam (Muslim/Moslem). An important nursing consideration related to his impending death and religion is that: 1 there are no special rites. 2 there are specific practices to be followed. 3 the family is expected to "wait" away from the dying person. 4 baptism should be performed if it has not been done previously.

2 Islam has specific rituals for bathing and wrapping the body in cloth before it is to be moved. The nurse should contact someone from the person's mosque to assist. Family may be present. No baptism is performed at this time. Test-Taking Tip: Attempt to select the answer that is most complete and includes the other answers within it. For example, a stem might read, "A child's intelligence is influenced by:" and three options might be genetic inheritance, environmental factors, and past experiences. The fourth option might be multiple factors, which is a more inclusive choice and therefore the correct answer.

The nurse has been assigned to the pediatric respiratory unit. What is the preliminary requirement for the nurse to evaluate improvement in the respiratory function of the child with treatment? 1 The child's feedback 2 The baseline data 3 The parents' opinion 4 The primary health care provider's opinion

2 It is impossible to evaluate the improvement in the respiratory function of the child without having any baseline data. The child's feedback provides only subjective assessment. Evaluation of respiratory system function requires formal knowledge of the respiratory system assessment. Therefore, parent opinion is not reliable for improvement in the child's respiratory function. Evaluation of the improvement in the respiratory function requires objective assessment. Obtaining the primary health care provider's opinion is a type of subjective assessment.

While performing the fetal acoustic stimulation test (FAST) in a patient, the nurse observes that there is no fetal response even after 3 minutes of testing. Which test does the nurse suggest? 1 Amniocentesis 2 Biophysical profile (BPP) 3 Cordocentesis 4 Coombs' test

2 Lack of response after 3 minutes of FAST indicates that the fetus has low activity levels. In this situation, to accurately assess fetal activity, the nurse should recommend a BPP of the fetus. Amniocentesis helps detect genetic abnormalities in the fetus. Fetal activity cannot be determined using this technique. In cordocentesis, the umbilical blood is tested for Rh incompatibility and hemolytic anemia in the fetus. Coombs' test is used to determine the presence of antibody incompatibilities in the fetus and the mother.

What important information should the nurse include when teaching the parents of an adolescent about nutrition? 1 Adolescents are usually mature enough to make healthy food choices. 2 Resources are available to assist lower income families to obtain enough protein. 3 Behavior problems in this age group are not related to nutritional deficiencies. 4 Parental influence has the greatest impact on food choices at this age.

2 Lower income families may need resources and information about how to obtain assistance in getting expensive foods such as meats to get enough protein intake. During adolescence, parental influence diminishes and the adolescent makes food choices related to peer acceptability and sociability. Occasionally these choices are detrimental to adolescents with chronic illnesses, such as diabetes, obesity, chronic lung disease, hypertension, cardiovascular risk factors, and renal disease. Families that struggle with lower incomes, homelessness, and migrant status generally lack the resources to provide their children with adequate food intake; nutritious foods, such as fresh fruits and vegetables; and appropriate protein intake. The result is nutritional deficiencies with subsequent growth and developmental delays, depression, and behavior problems. Behavior problems can indeed be related to nutritional deficiencies.

The nurse is assessing a traditional Hindu woman. What should the nurse ask in order to learn about the patient's health traditions? 1 "Do you have an advance directive for medical decisions?" 2 "What are your beliefs about health and illness?" 3 "How often do you pray or visit a place of worship?" 4 "Can you tell me when you developed the symptoms?"

2 Many cultures have health practices and traditions different from those of the Western world. The nurse can ask questions about their health and illness beliefs. These include all the health practices, medicines, and food of the patient's culture. This can help the nurse and patient develop mutually acceptable goals. Asking a patient whether he or she had an advance directive is important but will not provide information about cultural health practices. How often the patient prays is not related to cultural health practices. The nurse should ask questions about when the symptoms developed, but this does not relate to cultural health practices.

The nurse is caring for a traditional Chinese patient. The patient avoids direct eye contact with the nurse. What should the nurse conclude from this behavior? 1 The patient has major depression. 2 The behavior is acceptable and normal. 3 The patient demonstrates low self-esteem. 4 The nurse should get a psychiatric referral.

2 Many traditional Chinese people avoid direct eye contact as a sign of respect. It may be normal for this patient. The nurse cannot conclude that the patient has major depression or low self-esteem just from this behavior. Other assessment data are needed. There is also no evidence that this patient needs a psychiatric referral because he or she does not make eye contact. Test-Taking Tip: Pace yourself during the testing period and work as accurately as possible. Do not be pressured into finishing early. Do not rush! Students who achieve higher scores on examinations are typically those who use their time judiciously.

The nurse is reviewing the contraction stress test (CST) reports of a pregnant patient. The nurse expects the fetus to have meconium-stained amniotic fluid. What would be the reason for that conclusion? 1 Negative CST results 2 Positive CST results 3 Suspicious CST results 4 Unsatisfactory CST results

2 Meconium is normally stored in the infant's intestines until after birth, but sometimes (in cases of fetal distress and hypoxia) it is expelled into the amniotic fluid before birth. The amniotic fluid is then said to be meconium stained. Fewer than three contractions in 10 minutes or late decelerations occurring with 50% or more of contractions constitute positive CST results. Positive CST results are associated with meconium-stained amniotic fluid. Negative CST results indicate that the fetus is normal. Suspicious or unsatisfactory CST results are not associated with any other fetal conditions.

What parameter does the nurse check in the amniocentesis report of a pregnant patient to assess fetal lung growth? 1 Alfa-fetoprotein (AFP) levels 2 Lecithin-to-sphingomyelin (L/S) ratio 3 Creatinine levels in the blood 4 Antibody titer in the blood

2 The L/S ratio indicates fetal lung maturity. AFP is assessed to check for the presence of neural defects. Presence of creatinine in the amniotic fluid indicates that the patient's gestational age is more than 36 weeks. The antibody titer is used to determine Rh incompatibility in the fetus.

The nurse who works in a pediatric ward wants to explore his/her own ability to develop a therapeutic relationship with children and their families. The nurse does a self-assessment to evaluate the caregiving style by using an assessment questionnaire. In the questionnaire the nurse answers "yes" to a question, which indicates a positive action. What was the question to which the nurse answered as "yes"? 1 "Do you control visitor access to children by using excuses?" 2 "Do you periodically interview children to determine their current issues?" 3 "Do you become critical when parents do not visit their children?" 4 "Does the senior nurse appreciate you for being close to the child?"

2 The nurse should maintain a therapeutic relationship with the children and their families to provide effective care. While performing the self assessment of the ability to develop therapeutic relationship with children, the nurse should evaluate her or his positive actions and negative actions. To evaluate the positive action the nurse should check whether she or he is taking periodical interviews of the children. It helps to evaluate their health, emotions, and feelings. The negative actions include restricting the parents not to see their children. Children feel comfortable with their parents, so the nurse should not restrict them. The nurse should not be critical and judgmental, as it is unprofessional. The nurse can suggest the parents spend time with children. The nurse should not be too close with any child.

The nurse is working with a family who has a child with a chronic illness. The nurse notices that the parents watch television and play on their electronic devices instead of spending time with the child. The parents are frequently reprimanding the child. What should the nurse interpret from these findings? The parents: 1 Are being overprotective. 2 Are displaying rejection. 3 Are in denial of the illness. 4 Have gradual acceptance.

2 The parents' reaction of detaching themselves from the child and constantly rebuking the child indicates rejection. However, even though the parents are rejecting the child, they may still meet the child's physical needs. When parents do not let their children learn new skills for fear of injury, this indicates overprotection. Pretending that the child does not have a problem indicates denial. Placing necessary restrictions on the child indicates gradual acceptance.

The nurse is assessing a child's level of self-care. The nurse documents a rating of II for dressing and grooming. What can be inferred from this rating? The child: 1 Is independent on all aspects of personal care. 2 Depends on the supervision of another person. 3 Needs to use equipment or another adaptive device. 4 Requires direction from a person and uses equipment.

2 The self-care scale can be used for rating the functional self-care abilities of the child. The score ranges from 0 to IV. If the child is scored a II, this implies that the child requires assistance or supervision from another person. A child who is independent with activities of daily living would receive a 0. A score of I implies that the child requires equipment or a device for self-care. A score of III implies that the child requires assistance or supervision from another person and equipment or a device. A score of IV implies that the child is totally dependent and does not participate.

Which statement made by a child's parent supports the nurse's conclusion that the child has a difficult temperament? 1 "My child has predictable habits." 2 "My child often cries and throws tantrums." 3 "My child responds with passive resistance to new routines." 4 "My child becomes inactive and moody with change in routine."

2 The temperament of a child is said to be difficult when the child exhibits frequent episodes of crying, frustration, and tantrums. Such children are irritable and show irregularity in habits. Almost 10 percent of children fall under the category of difficult temperament. A child with an easygoing temperament has predictable habits and a positive attitude towards new stimuli. A slow-to-warm-up child responds with passive resistance to new routines and becomes inactive and moody.

Which statement helps explain the growth and development of children? 1 Development proceeds at a predictable rate. 2 The sequence of developmental milestones is predictable. 3 Rates of growth are consistent among children. 4 At times of rapid growth, there is also acceleration of development

2 There is a fixed, precise order to development. There are periods of both accelerated and decelerated growth and development. Each child develops at his or her own rate. Physical growth and development proceed at differing rates. Test-Taking Tip: Do not worry if you select the same numbered answer repeatedly, because there usually is no pattern to the answers.

The parents of a terminally ill child are overwhelmed with the volume of telephone calls asking about the child's condition. What suggestion should the nurse give to the family to make communication easier? 1 "Allow open visiting hours for all family members and the child's friends." 2 "Designate one family member or friend to relay information to everyone else." 3 "Instruct the family to create a website for information and to post updates." 4 "Tell the parents that answering calls and e-mails will help keep them busy."

2 To minimize stress on parents, the family should designate one family member or friend to relay information to others. Open visiting hours would increase the workload of the staff and interfere with care. In addition, the parents would not get any rest because of the continual need to entertain friends and family. Suggesting that the family create a website to post updates or asking parents to answer calls to keep busy may not be helpful. The parents do not need more work to do while dealing with the child's condition.

The nurse is teaching a student nurse about a child who only has one X chromosome. What abnormality does the child have? 1 Down syndrome 2 Turner syndrome 3 Fragile X syndrome 4 Contiguous gene syndrome

2 Turner syndrome is the only viable condition that happens as a result of the child missing one X chromosome. Down syndrome occurs when the child has an extra autosome, chromosome 21. Fragile X syndrome is a condition in which the chromosomes are fragile or weak, and it is associated with other changes in the autosomes. Microdeletion or microduplication of chromosome segments is called contiguous gene syndrome.

A child is 50 cm (20 inches) long in the second month of infancy. The nurse checks the baby 2 months later and finds healthy growth in the child. Approximately how long would the baby be at 4 months? 1. 52 cm 2. 55 cm 3. 57 cm 4. 60 cm

2 Until 6 months after birth, infants should grow 2.5 cm every month, so this 50-cm baby would to grow by 5 cm in 2 months. Therefore, the baby should be 55 cm in length by 4 months of age. If the child is only 52 cm, then the nurse should assess the child's nutritional status to determine whether caloric needs are being met. If the child is 57 cm or 60 cm, the nurse should assess the parents' height first. The baby may be longer because of greater than average parental height. If this is not the case, then the nurse should assess the child's endocrine system for growth problems.

Arrange the steps the nurse takes while performing transvaginal ultrasonography for a pregnant patient, in the correct order. 1. Cover the transducer probe with a probe cover. 2. Position the pregnant patient in the lithotomy position. 3. Position the probe for proper view of pelvic structures. 4. Lubricate the transducer probe with water-soluble gel. 5. Insert the transducer probe into the patient's vagina.

2, 1, 4, 5, 3 While performing transvaginal ultrasonography, the nurse should first position the patient in the lithotomy position to ensure the optimal view of pelvic structures. The transducer probe is then covered with a suitable probe cover. Then the probe is lubricated with a water-soluble gel to increase penetration of ultrasonic waves. The probe is then inserted into the patient's vagina. Finally, the position of the probe is adjusted for a better view of the inner pelvic structures.

When admitting a child to the inpatient pediatric unit, the nurse should assess for which risk factors that can increase the child's stress level associated with hospitalization? Select all that apply. 1 Mild temperament 2 Lack of fit between parent and child 3 Below-average intelligence 4 Age 5 Gender

2, 3, 4, 5 Risk factors for increased stress level of a child to illness or hospitalization: "Difficult" temperament; Lack of fit between child and parent; Age (especially between 6 months and 5 years old); Male gender; Below-average intelligence; Multiple and continuing stresses (e.g., frequent hospitalizations).

The nurse is developing a teaching plan about preventing fetal exposure to teratogens. Which teratogenic agents or conditions should the nurse include? Select all that apply. 1 acetaminophen (Tylenol) 2 isotretinoin (Accutane) 3 cocaine 4 hyperthermia 5 ethyl alcohol 6 phenytoin (Dilantin)

2, 3, 4, 5, 6 Teratogens, agents that cause birth defects when present in the prenatal environment, account for the majority of adverse intrauterine effects not attributable to genetic factors. Types of teratogens include drugs (phenytoin [Dilantin], warfarin [Coumadin], isotretinoin [Accutane]); chemicals (ethyl alcohol, cocaine, lead); infectious agents (rubella, cytomegalovirus); physical agents (maternal ionizing radiation, hyperthermia); and metabolic agents (maternal PKU). Many of these teratogenic exposures and the resulting effects are completely preventable, such as ingestion of alcohol resulting in fetal alcohol syndrome or fetal alcohol effects, which causes severe birth defects, including cognitive impairment. The incidence of fetal alcohol syndrome is estimated at 5.2 per 10,000 live births (American Academy of Pediatrics, 2000).

The nurse is educating a group of parents and children in the pediatric ward about the benefits of ambulatory care. What benefits does the nurse discuss with the group? Select all that apply. 1 Improved care 2 Increased cost-saving 3 Reduced chances of infection 4 Ambulatory care is lesser challenging 5 Minimum stressors of hospitalization

2, 3, 5 Ambulatory care is associated with an increased cost-saving as compared to hospital admissions, since there are no admission-related costs. Ambulatory care is associated with lesser chances of acquiring infections due to limited exposure to health care facilities. Ambulatory care is devoid of the stressors of hospitalization. There is deficient care due to the absence of qualified medical person for supervision. Ambulatory care is more challenging when compared to hospitalization as the child and the parents need to rely mostly on themselves for providing care to the child.

The nursing student is caring for a child admitted to the hospital. The nursing student asks the nurse instructor, "How can we keep the child's routine habits while he is in the hospital?" What would be the best response by the nurse instructor? Select all that apply. "Ask the parents: 1 "About the use of any herbal therapies." 2 "When the child goes to sleep at night." 3 "What foods the child prefers to eat." 4 "How the child's grades are in school." 5 "Which toy the child plays with at home."

2, 3, 5 The nurse should assess the child's usual health habits at home to promote a more normal environment in the hospital. This includes the child's sleep-rest, nutritional-metabolic, and activity-exercise patterns. The nurse would assess the sleep-rest pattern by asking when the child goes to sleep at night. Assessing the nutritional-metabolic pattern would include asking about food preferences. The nurse should also ask what toy the child plays with at home as part of the activity-exercise pattern. These will help the nurse plan individualized care for the child. History about herbal and complementary therapy helps in preventing drug-drug interaction and severe adverse effects.

A nonstress test (NST) is ordered on a pregnant woman at 37 weeks of gestation. What are the most appropriate teaching points to include when explaining the procedure to the woman? Select all that apply. 1 After 20 minutes, a nonreactive reading indicates the test is complete. 2 Vibroacoustic stimulation may be used during the test. 3 Drinking orange juice before the test is appropriate. 4 A needle biopsy may be needed to stimulate contractions. 5 Two sensors are placed on the abdomen to measure contractions and fetal heart tones.

2, 3, 5 Vibroacoustic stimulation is often used to stimulate fetal activity if the initial NST result is nonreactive and thus hopefully shortens the time required to complete the test (Greenberg, Druzin, and Gabbe, 2012). A nonreactive test requires further evaluation. The testing period is often extended, usually for an additional 20 minutes, with the expectation that the fetal sleep state will change and the test will become reactive. Care providers sometimes suggest that the woman drink orange juice or be given glucose to increase her blood sugar level and thereby stimulate fetal movements. Although this practice is common, there is no evidence that it increases fetal activity (Greenberg, Druzin, and Gabbe, 2012). A needle biopsy is not part of a NST. The FHR is recorded with a Doppler transducer, and a tocodynamometer is applied to detect uterine contractions or fetal movements. The tracing is observed for signs of fetal activity and a concurrent acceleration of FHR.

A child is brought to the clinic for a routine check-up. The parent informs the nurse that the child wants to go to school, but the parent does not want the child to go. What other characteristics in the parent would indicate that the parent is overprotective? Select all that apply. 1 Sets same rules for the child and the siblings 2 Continually helps child even if the child is capable 3 Helps the child to learn new skills to be independent 4 Sets high goals without understanding child's abilities 5 Puts restrictions on child's play due to fear of injury

2, 4, 5 A nurse should be able to identify parents who tend to be overprotective about their child with disabilities. The parent may continually help the child even when the child is capable of doing the task. Setting very high or low goals without understanding the child's capabilities makes the child feel incapable and inadequate. Putting restrictions on their child's play due to fear of injury indicates that the parent is too concerned about the child. Setting same rules for all children is not suggestive of an overprotective attitude in the parent. A parent who is not overprotective of the child helps the child to learn skills to be independent.

After assessment, the nurse notices that a child is in the detachment stage of separation anxiety. Which behavioral changes would the nurse observe in the child? Select all that apply. 1 Refuses to eat, drink, or get out of the bed 2 Shows an increased interest in the surroundings 3 Tries to leave the hospital to find the parents 4 Begins to form new relationships with others 5 Interacts with strangers or familiar caregivers

2, 4, 5 Detachment is the third stage of separation anxiety. It is also referred to as the denial stage. In this stage the child begins to take an interest in the surroundings. The child also forms new but superficial relationships with others and becomes more interested in interacting with strangers or familiar caregivers. The child's behavior indicates that the child has finally adjusted to the loss of the parents. This is a serious stage because reversal of the potential adverse effects is less likely to occur after detachment. Refusing to eat, drink, and get out of bed are characteristics of the despair stage of separation anxiety. Attempting to leave the hospital to find the parents is observed in protest stage of separation anxiety.

In 2010, there were 2000 live births in an area with a population of 20 million. There were 10 deaths among infants younger than 27 days old, 20 deaths among children ages 0 to 6 months, and 40 deaths among children younger than 1 year of age (includes all subcategories of deaths under 1 year). What is the infant mortality rate (IMR)? Record your answer using a whole number. _____________ per 1000 live births.

20 The infant mortality rate is the number of deaths during the first year of life per 1000 live births. The number of deaths of infants younger than 1 year of age was 40. Thus, IMR = number of deaths under 1 year / number of live births or 40 / 2000 = 20 / 1000.

The nurse is caring for a patient who is on long-term catheterization. According to the National Quality Forum, what should the nurse assess in this patient? 1 Oxygen saturation using arterial or venous blood 2 Monitoring of respiratory rate while in a sitting position 3 Signs and symptoms of a urinary tract infection 4 Abnormal changes in the electrocardiogram (ECG)

3 According to National Quality Forum, the nurse has to measure the patient-centered outcome. The nurse has to assess the patients in the intensive care unit (ICU) regularly to identify the urinary tract infections associated with the urinary catheter. Signs and symptoms of a urinary tract infection are fever, burning urination, and yellow urine. The urinalysis is done to confirm the presence of urinary tract infection (UTI). Urinary catheterization has no direct association with changes in arterial or venous oxygen saturation, respiratory, or cardiac complication.

A 14-year-old patient with Ewing's sarcoma is scheduled for an amputation of the right leg. The patient has been grieving about the loss ever since the health care provider prescribed the surgery. What type of grief does the nurse identify in this patient? 1 Acute grief 2 Universal loss 3 Anticipatory grief 4 Complicated grief

3 Because the amputation has not occurred, the patient's reaction is caused by anticipation of a future loss. This is a healthy response to the anticipated loss of the leg. It will help foster healthy grieving. Acute grief is a definite syndrome characterized by psychological and somatic symptoms. It is characterized by emotional distancing in relationships with others, accompanied by erratic responses of irritability, hostility, and anger. Universal loss does not describe the emotions of grieving before an actual loss. Some people feel constantly upset and preoccupied with a person who has died, to the point where their relationships and work suffer for months on end. Such a reaction is known as complicated grief.

A child who has undergone orofacial surgery is getting discharged. The nurse teaches the parents about how to safely transport the child on the way home. Which statement made by the parents indicates a need for additional teaching? "We should: 1 Have a blanket and pillow for our child for the car ride home." 2 Have a plastic bag for our child in case of nausea and vomiting." 3 Use a cup with a lid and a straw for giving fluids to our child." 4 Make sure our child has pain medication before discharge."

3 Children who undergo orofacial surgery should not use a straw for drinking fluids because it can damage the surgical site. Therefore the parents should not use cup with a lid and a straw for giving fluids to the child. The parents should bring a blanket and pillow for the child in the car so that the child can sit or sleep properly. Parents should bring a plastic bag, which will be helpful if the child becomes nauseated or vomits. The parents should give prescribed pain medication to the child before leaving the facility for relieving pain.

During the assessment of a child, the nurse finds that the child is inactive, depressed, sad, and uncommunicative; refuses to eat; and generally lacks interest in everything around her. What should the nurse interpret from this assessment? The child is in the: 1 Denial stage. 2 Protest stage. 3 Despair stage. 4 Detachment stage

3 From this assessment, the nurse interprets that the child is in the despair stage. This is the second stage of separation anxiety. In the despair stage, the child appears less active, depressed, and uninterested in play or food. In this stage the child's physical condition may deteriorate from refusing to eat, drink, or get out of bed. The denial stage is the third stage of separation anxiety. In the denial stage, the child is more interested in the surroundings, plays with others, and forms new but superficial relationships with others. In the protest stage, the child reacts aggressively, cries, screams, and searches for the parents with the eyes. Detachment is the third stage of separation anxiety. It is also called denial.

The signs and symptoms in a nursing diagnosis describe: 1 projected changes in an individual's health status, clinical conditions, or behavior. 2 an individual's response to health pattern deficits in the child, family, or community. 3 a cluster of cues and/or defining characteristics that are derived from patient assessment and indicate actual health problems. 4 physiologic, situational, and maturational factors that cause the problem or influence its development.

3 Identifying characteristics derived from patient assessment is the third part of the nursing diagnosis , the signs and symptoms. Projected changes in health status are the outcomes or goals that are established. An individual's response to health pattern deficits is the definition of the problem statement, the first component of the nursing diagnosis. The factors that cause the problem or influence its development is the definition of etiology, the second component of the nursing diagnosis.

Why does the nurse ask the parents of a hospitalized child to bring the child's blanket from home? 1 To alleviate any fears in the child 2 To decrease any allergic reactions 3 To provide comfort for the child 4 To keep the child warm at night

3 If the parents cannot stay with the child in the hospital, the nurse may ask the parents to leave an article such as a blanket or toy from home. This is because young children associate such inanimate objects with significant people, and they gain comfort and reassurance from these possessions. When a child is frightened, the nurse should provide physical contact to ease the child. If the child is allergic to the linens at the hospital, it would be the hospital's responsibility to find alternative bedding. There are plenty of blankets available in the hospital, so the parents would not bring the blanket to keep the child warm.

A child is assessed and categorized in the industry versus inferiority stage according to Erikson's theory. The nurse compares the child with Freud's psychosexual development theory. At what stage would the child be categorized in Freud's theory? 1 Anal 2 Phallic 3 Latency 4 Genital

3 In Erikson's theory, the industry versus inferiority stage includes children 6-12 years old. The stage in Freud's theory that matches this age group is the latency stage. The anal stage of Freud's theory corresponds to the autonomy versus shame and doubt stage of Erikson's theory. The phallic stage of Freud's theory corresponds to initiative versus guilt, and the genital stage of Freud's theory corresponds to the identity versus role confusion stage of Erikson's theory.

After giving a bed bath and cleaning the feet of a 2-year-old boy, an intravenous line is inserted into a small area by shaving the scalp. His Jewish parents are upset with this procedure. What could be the most probable reason behind the parents' discontent? 1 Intravenous infusions are against Jewish culture. 2 Scalp vein insertions are against the Jewish culture. 3 Cutting the hair before 3 years of age is not permitted in Jewish culture. 4 Touching the head after touching the foot is disrespectful in Jewish culture.

3 In Jewish culture, first haircut is done for boys at age 3 years. This is called the "upsherenish ceremony." Therefore, the nurse should have gotten consent from the parents before shaving the scalp of the boy. Intravenous infusions and scalp vein insertion are allowed in the Jewish culture. Touching the head after touching the foot is considered disrespectful in the Vietnamese culture.

A family presents to the emergency room with a child with enuresis. They tell the nurse that they have tried their traditional cure of burning to alleviate the problem. What can the nurse expect as a physical sign of this practice? 1 Reddened blotches on the skin 2 Rash covering the child's body 3 Small blisters on the child's body 4 Small welt-like lesions on the skin

3 Many communities follow the practice of burning. As the name suggests, small burns are made on the child's body in an attempt to cure disorders such as enuresis and temper tantrums. The nurse is most likely to find blisters on the child's body. Reddened blotches, rashes, and welt-like lesions may be signs of an allergic reaction.

The nurse is caring for a Mexican patient with severe febrile seizures. As a part of the care, the nurse removes the child's clothing and provides a bath while maintaining privacy. The nurse also takes off an amulet and necklace worn by the child. Later in the day, the child's parents refuse to speak to the nurse. What is the likely cause for their behavior? 1 They oppose treatments in conventional medicine. 2 They did not like the nurse touching and bathing their child. 3 They did not want the amulet and necklace to be removed. 4 Their spiritual healer forbids them to talk to the nurse

3 Many religions believe in supernatural causes and cures of diseases. Many Mexican people wear amulets and necklaces, which they believe ward off evil and protect a person from evil eye and diseases. The nurse took off the amulet and necklace without speaking to the parents or asking their permission. This is the most likely cause of the parents' displeasure. The parents brought the child to the hospital, which suggests they have no objection to modern medicine or nursing care.

The nurse is assessing a pregnant patient and finds that her blood pressure is 150/90 mm Hg. What procedure does the nurse recommend for this patient? 1 Nuchal translucency (NT) test 2 Chorionic villus sampling (CVS) 3 Doppler blood flow analysis 4 Percutaneous umbilical blood sampling (PUBS)

3 Maternal hypertension can cause serious adverse effects on the fetus. A blood pressure reading of 150/90 mm Hg indicates that the mother is hypertensive. To assess the effect of maternal hypertension on the fetus, the nurse should refer the patient for a Doppler blood flow analysis. It is a noninvasive ultrasonic technique used to study fetal blood flow. NT is a technique used to assess genetic abnormalities in the fetus. CVS is a prenatal test used to diagnose structural defects in the fetus. PUBS is used to assess the fetal circulation.

The parents of a 10-year-old child diagnosed with terminal cancer ask the nurse not to inform their child about the condition. What would be the most appropriate response of the nurse? 1 Report the situation to the appropriate child welfare organization. 2 Ignore the parents' request and tell the child about the condition. 3 Inform the parents that being honest with the child is important. 4 Tell the parents that treatment cannot be initiated until the child is informed.

3 Often, parents ask health care providers not to reveal the prognosis to the terminally ill child. This is a difficult situation for health care providers. Although children do not understand that they are going to die, they are in a position to understand that something is seriously wrong with them. Therefore the nurse should make the parents understand that being honest with the child is important. Being honest creates an open environment where feelings may be shared by the entire family. This often encourages the parents to communicate openly with the child. The nurse is not required to report to the child welfare organization at this stage. The parents' decision should not be ignored; rather, the parents should be encouraged to talk openly with the child. The nurse cannot withhold treatment.

After reviewing the standard ultrasound scan reports of a pregnant patient, the nurse advises the patient to undergo a specialized ultrasound scan. What is the nurse's rationale for this suggestion? 1 To estimate the amniotic fluid volume 2 To identify the detailed fetal anatomy 3 To assess for physiologic abnormalities 4 To assess for fetal genetic abnormalities

3 Specialized or targeted ultrasound scans are performed only if a patient is suspected of carrying an anatomically or physiologically abnormal fetus. Limited ultrasound examination is used to estimate the amniotic fluid volume. Standard ultrasound scan is used to see the detailed anatomy of the fetus. Ultrasound scan is not used to find genetic abnormalities in the fetus.

A patient with two children is going through a divorce and does not understand what the term split custody means. What is the most appropriate answer? 1 Custody of both children is given to the grandparents. 2 Custody of both children is given to the parent who brings home the larger salary. 3 Custody of one child is given to the mother, and the father has custody of the other child. 4 Custody of the children is given to the father, and the mother is allowed to visit once a week.

3 Split custody means that custody of one child is given to the mother and custody of the other child is given to the father. This arrangement ensures that both parents have a child, but it separates the siblings. Custody of the children is not given to the grandparents, it is not determined by who has the larger salary, nor is it given solely to the father with visitation privileges given to the mother.

Which test does the nurse recommend for the patient to help assess fetal genetic abnormalities? 1 Amniotic fluid volume (AFV) 2 Fetal body movements 3 Nuchal translucency (NT) 4 Fetal heart activity

3 The NT ultrasound screening technique is used to measure fluid in the nape of the fetal neck between 10 and 14 weeks' gestation. Fluid volume greater than 3 mm is considered abnormal. NT is used mostly to identify possible fetal genetic abnormalities. AFV, fetal body movements, and fetal heart activity are measured to assess fetal well-being.

A parent tells a 13-year-old child that he is adopted. What could happen as a result of telling the child at this age? 1 Sympathy 2 Happiness 3 Depression 4 Excitement

3 The earlier the child knows of his adoption status, the better. Generally, older children display anger and sadness. This can often be manifested as depression. The child may feel abandoned, but a feeling of sympathy is rare. Similarly, happiness and excitement are not what a child feels after learning about being adopted.

The nurse finds that the nonstress test of a pregnant patient is nonreactive. Which factor in the report might have led the nurse to this finding? 1 No qualifying accelerations in a 20-minute period 2 Two qualifying accelerations in a 20-minute period 3 Less than two qualifying accelerations in a 20-minute period 4 More than two qualifying accelerations in a 20-minute period

3 The nonstress test is the most widely used technique for prenatal evaluation of the fetus. The results are either nonreactive or reactive. In a nonreactive test, there are less than two qualifying accelerations of the fetal heart rate in a 20-minute period. Absence of fetal heart rate accelerations during the nonstress test indicates that the fetus is sleeping. In a reactive test, there are at least two qualifying accelerations in a 20-minute time period. More than two fetal heart rate accelerations within a 20-minute time period also would be considered a reactive test.

The amniotic fluid index (AFI) of a pregnant patient is 3 cm. What clinical information related to the fetus does the nurse infer from this? 1 Neural tube defect 2 Fetal hydrops 3 Renal defects 4 Low activity level

3 The normal value of AFI is 10 cm or greater, with the upper limit of normal around 25 cm. An AFI less than 5 cm indicates oligohydramnios. This condition is associated with renal agenesis in the fetus. A high AFI indicates neural tube defects and fetal hydrops. The AFI is not directly related to fetal movement. Fetal activity can be assessed using ultrasonography.

The pediatric nurse works efficiently in providing nursing care to an acutely ill child. After discharge, parents of the child ask the nurse to visit their home for dinner. What should the nurse do? 1 Accept it; otherwise it may adversely affect the good relationship. 2 Tell them to schedule it later as it is a busy day in hospital. 3 Reject it courteously and thank them for the invitation. 4 Ask them to invite other staff who were involved in the care as well.

3 The nurse is not supposed to develop personal relationships with the children and families during the care and after the discharge. Therefore, the nurse has to courteously reject such invitations that may lead to personnel relationships. Even if the nurse is busy, the nurse should not accept invitations for lunch, dinners, or other parties, or ask the families to invite other medical staff. This shows unprofessional behavior.

To establish evidence-based practice, the nurse has to collect high-quality evidence from various sources. Where does the nurse find the best quality of evidence? 1 Observational studies 2 Hospital patient records 3 Randomized clinical trials 4 Direct interview with patients

3 The nurse knows the best source for consistence and unbiased evidence is from well-performed randomized clinical trials (RCT). Observational studies have biased information, so are not reliable, unless the study is done meticulously. RCTs are preferred to hospital records or patient interviews. As biases can affect the establishment of evidence-based practice, the nurse should take precautions to avoid biased or low-quality evidence.

The nurse is caring for a child with cancer. What should the nurse ask the child's parents about in order to obtain information about the child's coping-stress tolerance pattern? 1 "How do you both handle discipline problems at home?" 2 "Have you ever noticed if your child has many friends?" 3 "How does your child usually handle disappointment?" 4 "Who will be staying with your child at the hospital?"

3 The nurse should ask the child's parents about how the child usually handles disappointment. This can help the nurse understand the coping-stress tolerance pattern of the child. It is also helpful for identifying stressors in the child. It is important to know how discipline problems are managed in the child. This helps to know about the child and parent role-relationship pattern. When the nurse asks about the child's friends, it is to assess the child's role and relationship patterns outside the home. The nurse can understand the role and relationship pattern between the parents and child after knowing who will stay in the hospital with the child.

Before transporting a 16-year-old American Indian female for a magnetic resonance imaging (MRI) scan, the nurse notices the girl is wearing a decorated amulet necklace. The nurse's next best action is to: 1 remove the necklace and place it at the nurse's station. 2 explain the risks of wearing the necklace during the MRI. 3 ask the patient if there is a special reason for wearing the necklace. 4 place tape around the neck covering the necklace.

3 The nurse should first ask the patient the purpose of wearing the necklace. The amulet may be worn as a religious ritual or simply as an accessory. After assessing why the necklace is worn, the nurse can explain the reason for having to remove the necklace for the procedure. The first step is to assess. Placing tape around the neck is not an appropriate action and could be unsafe. The necklace should be left with family members if possible or in a locked cabinet, rather than at the nurse's station. Test-Taking Tip: Many times the correct answer is the longest alternative given, but do not count on it. NCLEX item writers (those who write the questions) are also aware of this and attempt to avoid offering you such "helpful hints."

The nurse is assessing the functional self-care level of a child and determines that the child requires the assistance of a caregiver for general hygiene and dressing. How would the nurse rate the child? 1. 0 2. I 3. II 4. IV

3 The nurse should rate the child as a II (two) because the child requires assistance of a caregiver for general hygiene and dressing. A grading of 0 (zero) is given to the child who is capable of taking full self-care. A grading of a I (one) is given to the child who requires the use of equipment or a device for self-care. A child who is totally dependent and does not participate in self-care would be rated a IV (four).

During assessment of a 7-month-old child, the nurse checks the child's height and weight and compares them with previous assessment records. The nurse finds that the child's height has increased by 1.25 cm, and the weight is 140 g more than in the previous month. What does the nurse infer from this observation? 1 The child is displaying symptoms of Down syndrome. 2 The child's weight is not ideal in relation to height. 3 The child's height and weight are ideal. 4 The child has a calcium deficiency due to malnutrition.

3 The nurse should regularly check the height and weight of the child and compare them with previous assessment records. These comparisons help the nurse identify genetic defects that can affect the child's growth and development. A child gains 140 g in weight, and height increases by 1.25 cm every month from ages 6 to 12 months. Therefore, this child has an ideal height and weight. Down syndrome is characterized by a slower growth rate. The child is having age-appropriate increases in height and weight and thus does not have Down syndrome. Calcium deficiency decreases bone density and causes fractures in children. The nurse cannot determine whether the child has calcium deficiency by assessing height and weight. Test-Taking Tip: Make certain that the answer you select is reasonable and obtainable under ordinary circumstances and that the action can be carried out in the given situation.

The nurse is caring for a 7-year-old child. The child wears an amulet because the family believes that it will protect the child from the evil eye. For a diagnostic procedure, the nurse has to take this amulet off. What should the nurse do? 1 Allow the child to continue to wear the amulet for comfort during all aspects of the procedure. 2 Do not perform the prescribed procedure because removing the amulet will hurt the family's feelings. 3 Get permission from the family to remove it for the procedure and replace it afterwards. 4 Ask the family to remove the amulet and not to let the child wear it because it obstructs medical care.

3 The nurse should try to avoid hurting the family's feelings. If the amulet is an obstacle to medical care, the nurse should explain that to the family and obtain their permission to remove the amulet for the procedure. The nurse should replace it as soon as the procedure is over. Continuing the procedure without taking off the amulet may interfere with the procedure or hurt the child. The nurse should not cancel the procedure because it is essential for the child's care. The nurse should not ask the family to remove the amulet permanently because it does not impede all medical care; affects only this procedure.

The nurse is caring for a child with an influenza viral infection. The child is anxious because the parents are unable to stay with the child. What should the nurse do to relieve the child's anxiety? The nurse should: 1 Not maintain any eye contact with the child. 2 Not speak with the child about missing the parents. 3 Use the phone to let the child talk with the parents. 4 Use a laptop to allow the child and parents to talk

3 The nurse should use a telephone to maintain contact between the child and parents so that the child can feel comfortable. It helps relieve the child's anxiety. The nurse should maintain eye contact and gently touch the child to establish rapport. The nurse should talk with the child about the parents and family to prevent detachment of the child from the parents. The nurse should not use a laptop to contact the child and parents. The laptop may not be compatible with medical equipment, and use may be restricted in certain areas.

The nurse gets out finger painting materials made from wallpaper paste for a child. After reviewing the child's medical record, the nurse decides to get out crayons and a coloring book instead. What information did the nurse find in the nursing history? The child: 1 Is on a salt-restricted diet. 2 Has lactose intolerance. 3 Has an allergy to wheat. 4 Is allergic to beetroot.

3 The painting materials made from wallpaper paste may contain wheat. If the paint comes in contact with the child's skin, it can cause an allergic reaction. Therefore the nurse changes the activity. The child who is on a salt-restricted diet need not worry about salt being in the paint. A patient is placed on salt restriction to prevent water retention, not because of an allergy. The child who is lactose intolerant cannot ingest dairy products. Beetroot is used as natural dyes in paintings; however, the wallpaper paste does not contain beetroot extract.

The parents of a terminally ill child are speaking with the nurse to learn about palliative care and how it differs from hospice care. What should the nurse tell them? "Palliative care: 1 Requires that a patient be a Medicare recipient." 2 Does not provide care for hospitalized patients." 3 Is provided when no effective cure is available." 4 Does not provide spiritual support to the family."

3 The terms palliative care and hospice care are associated with end-of-life care. Palliative care may be provided when no cure is available. Palliative care seeks to prevent, relieve, reduce, or soothe the symptoms of disease or disorder without effecting cure. Hospice care supports the patient and the family during the dying process (medical prognosis of 6 months or less) and the surviving family members through the process of bereavement. It provides total care for the patient in the hospital, home, and nursing home when the patient's disease is unresponsive to the curative treatment. To qualify for hospice care, the patient does not need to be a Medicare recipient. Most insurance programs cover hospice care. However, for palliative care, the patient needs to be a Medicare recipient. Palliative care can be provided to patients in a variety of settings, including hospitals. Both palliative care and hospice care include spirituality in order to provide holistic care. Nurses need to familiarize themselves with the attitudes and needs of various religious groups so that they can address the spiritual needs of their patients, families, and other members.

The nurse is caring for two children. The younger child creates complex imaginary stories using dolls and toys. The older child is engaged in building a model airplane. Which stages of development are the children likely in, according to Erikson? 1 The younger child is in the trust versus mistrust stage; the older child is in the initiative versus guilt stage. 2 The younger child is in the industry versus inferiority stage; the older child is in the identity versus role confusion stage. 3 The younger child is in the initiative versus guilt stage; the older child is in the industry versus inferiority stage. 4 The younger child is in the identity versus role confusion stage; the older child is in the trust versus mistrust stage.

3 The younger child exhibits a strong imagination and an urge to explore through her doll play, which indicates that she is in the initiative versus guilt stage of Erikson's psychosocial development theory. The older child is building a model, indicating a desire to produce something and complete a task, which is a primary characteristic of the industry versus inferiority stage according to Erikson. The trust versus mistrust stage of Erikson's theory consists of establishing trust and taking in the world using all the senses; neither child is exhibiting characteristics of the trust versus mistrust stage. The identity versus role confusion stage is characterized by rapid physical change in children and concern over how they are viewed by others; neither child is exhibiting behavior associated with this stage.

An infant's blood glucose levels are low, and the nurse instructs the mother to perform kangaroo care. Which condition would the nurse have assessed in the child? 1 Irregular sleep patterns 2 Reduced metabolism 3 Improper thermoregulation 4 Impaired maturation

3 Thermoregulation is one of the most important adaptations for an infant to develop. A hypothermic infant tends to develop conditions such as hypoglycemia and metabolic acidosis. Skin-to-skin contact, or kangaroo care, is beneficial in maintaining the infant's temperature. Sleep irregularities may develop when an infant is suffering from pain internally or externally, which would also affect the growth and development at early postnatal development. Metabolic rates are usually high in children. Neurological maturation, or a dramatic increase in the number of neurons, occurs when the infant is in the embryonic stage and the neonatal state.

A child with strabismus is undergoing treatment for impaired vision of the left eye. The nurse covers the child's right eye with an occlusion patch. Why does the nurse do so? 1 To protect the right eye from dust 2 To reduce intraocular pressure in the left eye 3 To increase vision in the left eye 4 To prevent the child from rubbing the right eye

3 While caring for a child with strabismus, the nurse should cover the unaffected eye with an occlusive patch because it helps stimulate vision and movement in the weaker eye. The main reason for applying an ocular patch is to improve vision in the left eye, not to protect the right eye from dust. Applying an ocular patch on the right eye does not reduce intraocular pressure in the left eye; antiglaucoma medications can be used to reduce intraocular pressure. Applying an occlusion patch will not prevent the child from rubbing his or her eyes. The nurse should explain to the child that rubbing the eyes may cause further damage.

The parent of a newborn child asks the nurse the importance of breastfeeding. What should the nurse tell the parent? Select all that apply. 1 Breast milk is not rich in micronutrients. 2 Breast milk is not recommended for infants with fever. 3 Enzymes in breast milk are helpful in the digestion of milk. 4 Immunoglobulins in milk can prevent infections and diseases. 5 Breastfeeding can decrease infant mortality and morbidity.

3, 4, 5 Breast milk contains enzymes that are helpful in the digestion of milk and improve the bioavailability of all nutrients in the milk. Immunoglobulins in milk give immunity against infections and allergies. Hence, breastfeeding can decrease infant mortality and morbidity. Breast milk is rich in micronutrients. Breast milk has immunologic properties, so it can be given to the infants with fever.

Which behaviors would characterize a child adequately coping with a chronic illness? Select all that apply. 1 Behavior problems at home and school 2 Negative attitudes about his or her condition 3 Acceptance of his or her limitations and coping accordingly 4 Taking responsibility for his or her own physical care 5 Identification with other similarly affected people

3, 4, 5 There are two ways of coping with the illness. Children with healthy coping mechanisms accept their limitations and assume responsibility for their care. These children may also identify with other similarly affected people. Maladaptive coping is evident in behavior problems at home and school. These children may also have a negative attitude toward the illness and begin acting out.

The nurse should expect to possibly incorporate which religious and cultural practices into the plan of care when caring for a 35-year-old Jewish mother who just gave birth to a healthy baby boy? Select all that apply. 1 Circumcision in hospital 2 Ordering house diet lunch tray of roasted pork with mashed potatoes 3 Allowing family, friends, and rabbi to visit patient often 4 Ask males to remove shawl and yarmulke while visiting 5 Ordering house diet with the exception of shellfish

3, 5 Family, friends, and rabbi should be allowed to visit. Individuals of the Jewish faith generally are prohibited from eating pork or shellfish. Ritual circumcision of male infants is custom on the eighth day and performed by a mohel. Asking males to remove shawls or yarmulkes is inconsistent with acceptance of religious values.

The nurse is reviewing the plan of care with the parent of a child with Crohn's disease. Which statement made by the parent indicates a need for further education? 1 "This illness is why my child has delayed puberty." 2 "This illness is caused by the intake of fiber-rich food." 3 "This illness has impaired my child's growth and development." 4 "This illness is God's way of testing our religious faith."

4 A parent who states that the child's illness is a trial sent by God to test religious faith is expressing a self-accusatory feeling that indicates a lack of understanding of the condition. Crohn's disease is an autoimmune disorder that affects the gastrointestinal tract, causing inflammation that can result in excessive diarrhea, abdominal pain, and other symptoms. Crohn's disease can stunt growth and weaken bones in children who are going through puberty. Parents are advised to refrain from serving fiber-rich food to such children because these foods are difficult to digest and may block the intestine.

A 14-year-old boy learns about his adoption from his relatives and wants to know his identity. He appears angry, embarrassed, and anxious. What should the nurse advise his parents to help resolve the conflict? 1 "Don't forget to obtain the boy's birth certificate." 2 "Don't reveal the information regarding the adoption." 3 "Don't encourage the boy to search for his identity in this situation." 4 "Don't forget to inform the child about the availability of a birth certificate."

4 Adoptive parents should inform the boy about the availability of a birth certificate. Open and honest communication between the boy and the parents is essential for the welfare of the adopted child. Legally, adoptive parents are not permitted to obtain the birth certificate. Parents should understand the need of the child to search for his identity and extend encouragement and support. Test-Taking Tip: Identify option components as correct or incorrect. This may help you identify a wrong answer

A 27-year-old patient reports having difficulty sleeping, having a poor appetite, and feeling sad since the death of her spouse. What should the nurse identify in this patient as a normal response to the death of the spouse? 1 Grief 2 Mourning 3 Acceptance 4 Bereavement

4 Bereavement is a process of grief experienced by most people. Examples include feelings of sadness, insomnia, poor appetite, deprivation, and desolation. The grieving person may seek professional help for the relief of symptoms if they interfere with daily activities and do not subside within a few months of the loss. Grief is a normal, appropriate emotional response to an external or consciously recognized loss. Mourning is a term used to describe a person's outward expression of grief. The person experiences emotional detachment from the loved object or person. Acceptance is the stage where the patient has achieved inner and outer peace through a personal victory over fear.

The community nurse has conducted a survey on the frequency of occurrences of various diseases and health problems such as acne, headache, diarrhea, and upper respiratory tract infections (URTI) in children. What trend would the nurse most likely notice from the survey? The frequency of: 1 Acne decreases with age 2 URTI increases with age 3 Diarrhea increases with age 4 Headaches increases with age

4 Children who have had a particular type of problem are more likely to have that problem again. Therefore, the frequency of headaches increases with age. The activity of sebaceous glands increases with age and therefore the frequency of acne also increase with age. The immune system becomes stronger with age in children. Therefore, the frequency of upper respiratory tract infections (URTI) and diarrhea decreases with age.

During the assessment of a 5-year-old child whose parents are from Southeast Asia, the nurse notices small burn injuries on the skin. The nurse learns that the child has a history of temper tantrums. Which is the immediate nursing action? 1 Call the police. 2 Report child abuse. 3 Inform the health care provider. 4 Ask the parents about the injury.

4 Communication is important to avoid wrong diagnosis and interpretations. The nurse should ask the parents about the burn injuries to understand their cultural and religious practices. If the child exhibits temper tantrums, some Southeast Asian cultural groups follow the practice of causing burn injuries on the skin for treatment. These burn injuries are not considered to be abusive by this cultural group even though the dominant culture and legal system may consider this to be child abuse. Therefore, there is no need to call the police or report child abuse. The nurse can inform the health care provider after discussing with the parents.

What is the characteristic of the type of play that is organized by children playing with other children with the purpose of accomplishing a goal? 1 The group members play independently. 2 The group members act according to their own wishes. 3 The group members do have assigned leadership roles. 4 The group members plan activities even if the group is formed loosely.

4 Cooperative play features one child supplementing another child's function with a common aim of goal completion. In cooperative play, the group may be loosely formed but the members plan activities with the aim of accomplishing a task. The group members play independently among others in parallel play. The members act according to individual wishes in associative play. There is no leadership assignment in associative play.

Evidence-based practice, a current health care trend, is best described as: 1 gathering evidence of mortality and morbidity in children. 2 meeting physical and psychosocial needs of the child and family in all areas of practice. 3 using a professional code of ethics as a means for professional self-regulation. 4 questioning why something is effective and whether there is a better approach.

4 Evidence-based practice helps to focus on measurable outcomes and the use of demonstrated, effective interventions and questions whether there is a better approach. Gathering evidence of mortality and morbidity in children will assist the nurse in determining areas of concern and potential involvement. It is not possible to meet all needs of the family and child in all areas of practice. The nurse is an advocate for the family. This is part of the professional role and licensure. Test-Taking Tip: Start by reading each of the answer options carefully. Usually at least one of them will be clearly wrong. Eliminate this one from consideration. Now you have reduced the number of response choices by one and improved the odds. Continue to analyze the options. If you can eliminate one more choice in a four-option question, you have reduced the odds to 50/50. While you are eliminating the wrong choices, recall often occurs. One of the options may serve as a trigger that causes you to remember what a few seconds ago had seemed completely forgotten.

A child is hospitalized for a chronic illness. Initially, the child showed symptoms of depression but later started interacting with others. What does the nurse infer from the patient's behavior? The child is: 1 Content with the care provided. 2 Showing improved social skills. 3 Getting used to the surroundings. 4 Detached from both parents.

4 Hospitalized children undergo depression when they are separated from their parents. As they go through the stages of separation anxiety, children eventually detach from their parents and develop new and shallow relationships. Children interact with others and develop new relationships as a result of resignation, not contentment. Children who are detached begin to show increased interest in their surroundings. They are also not developing their social interaction skills. Children try not to think about the separation; hence, they start developing new interactions.

After reviewing the biophysical profile (BPP) reports of a pregnant patient close to term, the nurse advises the patient to repeat the test on a weekly basis. What BPP score did the nurse find in the report? 1. 1 2. 4 3. 6 4. 9

4 If the BPP score is 8 to 10, then the test should be repeated weekly or twice weekly. If the BPP score is 0 to 2, then chronic asphyxia may be suspected. In this case the testing time should be extended to 120 minutes. If the BPP score is 4 after 36 weeks' gestation, then clinical conditions exist that may lead to an eminent delivery. If the BPP score is 4 before 32 weeks' gestation, the test should be repeated. If the BPP score is 6 at 36 to 37 weeks' gestation with positive fetal pulmonary testing, then delivery can be performed. If the BPP score is 6 before 36 weeks' gestation with negative pulmonary testing, then BPP can be repeated in 4 to 6 hours, and if oligohydramnios is present, then delivery can be done. The BPP provides an insight into fetal maturity and well-being and as such should be used as a diagnostic tool to plan and evaluate management of care. Findings are related to several factors involving both maternal and fetal characteristics.

The biophysical profile (BPP) testing report of a pregnant patient gives the following information: one episode of fetal breathing movement lasting for 30 seconds in a 30-minute observation; three limb movements of the fetus in 30 minutes; an amniotic fluid index greater than 5; a reactive nonstress test; and a BPP score of 1. The test is performed for 120 minutes. What does the nurse expect the primary health care provider to do? 1 Extend the test time to 120 minutes. 2 Repeat the test twice a week. 3 Repeat the test in 4 to 6 hours. 4 Consider delivery of the fetus.

4 If the BPP score is less than 2, regardless of gestational age, delivery can be performed. If the BPP score is 0 to 2 and chronic asphyxia is suspected, then testing time should be extended to 120 minutes. If the BPP score is 8 to 10 and a low risk for chronic asphyxia is suspected, then the test should be repeated at twice-weekly intervals. If the fetal pulmonary test result is negative and the BPP score is 6, then the BPP profile should be repeated in 4 to 6 hours.

A child who is terminally ill with bone cancer is in severe pain. Nursing interventions should be based on knowledge that: 1 children tend to be overmedicated for pain. 2 giving large doses of opioids causes euthanasia. 3 narcotic addiction is common in terminally ill children. 4 large doses of opioids are justified when there are no other treatment options.

4 Large doses may be needed because the child has become physiologically tolerant to the drug, requiring higher doses to achieve the same degree of pain control. Continuing studies report that children are consistently undermedicated for pain. The dose is titrated to relieve pain. Addiction refers to a psychological dependence on the medication, which does not happen in terminal care.

A pregnant patient is about to undergo magnetic resonance imaging (MRI). What information does the nurse give the patient before the procedure? The patient will: 1 Be positioned in a lithotomy position. 2 Need to take fluids to have a full bladder. 3 Be able to move freely during the procedure. 4 Not have pain during the process

4 MRI is a noninvasive technique that causes little pain. Therefore the patient undergoing MRI should be advised to not worry about pain. The patient undergoing MRI should be positioned in a supine position. The lithotomy position is not suitable for this procedure. The patient need not take fluids before the scan to ensure a full bladder because this procedure enables a full pelvic view without a full bladder. The patient should be instructed not to move during the scanning process because it may blur the images obtained.

Nurses should be aware of the strengths and limitations of various biochemical assessments during pregnancy, including that: 1 chorionic villus sampling (CVS) is becoming more popular because it provides early diagnosis. 2 screening for maternal serum alpha-fetoprotein (MSAFP) levels is recommended only for women at risk for neural tube defects. 3 percutaneous umbilical blood sampling (PUBS) is one of the quad-screen tests for Down syndrome. 4 MSAFP is a screening tool only; it identifies candidates for more definitive procedures.

4 MSAFP is a screening tool, not a diagnostic tool. Further diagnostic testing is indicated after an abnormal MSAFP. CVS does provide a rapid result, but it is declining in popularity because of advances in noninvasive screening techniques. MSAFP screening is recommended for all pregnant women. MSAFP, not PUBS, is part of the quad-screen tests for Down syndrome.

A single working parent says that caring for a child and managing work is becoming difficult. The nurse determines that the patient is exhausted by these responsibilities. Which is the most appropriate service that would benefit the patient? 1 Hospital 2 Change of job 3 Adoption service 4 Respite child care

4 Respite child care is a service available to help parents to relieve exhaustion and avoid burnout. Therefore, the patient should seek respite child care services. Hospitals are for patients who need medical attention. It would not be appropriate to change jobs because the stress of working would still be there. Adoption services are inappropriate in this situation.

A woman who is at 36 weeks of gestation is having a nonstress test. Which statement by the woman indicates a correct understanding of the test? 1 "I will need to have a full bladder for the test to be done accurately." 2 "I should have my husband drive me home after the test because I may be nauseous." 3 "This test will help to determine if the baby has Down syndrome or a neural tube defect." 4 "This test will observe for fetal activity and an acceleration of the fetal heart rate to determine the well-being of the baby."

4 The nonstress test is one of the most widely used techniques to determine fetal well-being and is accomplished by monitoring fetal heart rate in conjunction with fetal activity and movements. An ultrasound is the test that requires a full bladder. An amniocentesis is the test that a pregnant woman should be driven home afterward. A maternal alpha-fetoprotein test is used in conjunction with unconjugated estriol levels, and human chorionic gonadotropin helps to determine Down syndrome.

The parents of a child who is dying of leukemia are grieving their impending loss. What intervention should the nurse perform to provide effective care for the family? 1 Encourage the use of alcohol and drugs as a way to escape grief. 2 Refer the family to a self-help group to manage their grief. 3 Emphasize that acute grieving is a process that lasts for years. 4 Provide the child and family with time to share memories.

4 The nurse should be very sensitive to the feelings of a grieving family. The nurse should also provide the child and family with the opportunity to review special experiences or memories in their lives. This helps the family members share their emotions and bond with each other. Parents and family members should be advised to seek professional counseling or work with each other to handle grief. The nurse should never encourage people to use alcohol or drugs to escape grief. The nurse can refer the family to a self-help group, but this is not always an effective way to help grieving parents. They may need a professional counselor to help guide them through the grieving process. The nurse should emphasize to the family that acute grieving only lasts for a few weeks to months, but the entire grieving process is a painful process that often takes years to resolve.

Which intervention should the nurse incorporate to prevent hypothermia in an infant? 1 Give hot milk or hot water to the infant at regular intervals. 2 Place the unclothed, diapered infant in the sun for few hours. 3 Feed the infant formula, which is higher in calories. 4 Put the unclothed, diapered infant on the mother's bare chest

4 Thermoregulation is not well developed in infants. As a result, babies are at risk for hypothermia. Kangaroo care is an effective way to prevent hypothermia in the infant. In this method, an unclothed, diapered infant is placed on the mother's bare chest. This provides physiological warmth to the infant. It is not advisable to give hot milk or hot water to the infant because it can damage their tissues. An unclothed infant must not be kept in the sun for a long time because the sun's ultraviolet rays can cause skin damage. There is no difference in calories between breast milk and formula. Test-Taking Tip: Avoid selecting answers that state hospital rules or regulations as a reason or rationale for action.

The nurse is leading an educational program for parents of 5- to 9-year-old children. Which topic should the nurse include in the teaching plan to prevent childhood mortality in children of this age? 1 Suicide 2 Being overweight 3 Heart diseases 4 Unintentional injuries

4 Unintentional injuries and accidents are the leading cause of death in children ages 5 to 9 because this age group have the ability to run and climb and may experience falls, burns, and collisions. Therefore, the nurse should focus mainly on reducing the risk of accidents. Suicide is not a significant cause of mortality in children under the age of 10. Being overweight is a significant problem in childhood, but not the leading cause of deaths in the 5-9 age group. Heart diseases are the most significant causes of mortality in children ages 1-4 years and 15-19 years.

A primary health care provider is performing a transabdominal amniocentesis procedure in a pregnant patient. Arrange the steps of the amniocentesis procedure in the correct order. 1. Separate the supernatant fluid and cellular components. 2. Collect the cellular components for chromosomal studies. 3. Centrifuge the collected amniotic fluid. 4. Collect the amniotic fluid under ultrasonographic visualization. 5. Collect the supernatant for chemical analysis.

4, 3, 1, 5, 2 Amniocentesis is performed to obtain the amniotic fluid, which contains the fetal cells. Amniotic fluid can be collected transabdominally under ultrasonographic visualization. After the amniotic fluid is collected, the fluid should be centrifuged. Centrifugation is done to separate the supernatant fluid and cellular components. Because the supernatant fluid has a lighter weight, it collects in the upper part of the centrifuge tube. This supernatant fluid is collected first and is sent for chemical analysis. Cellular components, which lie in the bottom of the centrifuge tube, are then collected to perform chromosomal analysis.

The nurse has to follow a method of problem identification and problem solving. Arrange the steps of the nursing process model in an appropriate order. 1. Planning 2. Evaluation 3. Diagnosis 4. Assessment 5. Implementation

4, 3, 1, 5, 2 The five-step nursing process model is assessment, diagnosis, planning, implementation, and evaluation. The first step is assessment in which the nurse collects, and analyzes subjective and objective data of the patient. This step is very crucial in determining the Client Needs. The second step is the formulation of the nursing diagnosis. In this phase, the nurse interprets and makes decisions about the acquired data. The third step is planning wherein the nurse develops a care plan and establishes the desired outcomes in the patient. This care plan also includes the interventions that would help the patient to achieve the desired outcomes. The fourth step is implementation in which the interventions that have been listed in the care plan are carried out in a systematic manner. Evaluation is the last step of nursing process which establishes the efficacy of the interventions provided to the patient.

The nurse recognizes a delay in the child's developmental pattern and refers the child for further testing. Which characteristics of this child might the nurse have identified? Select all that apply: 1 Dermatological disorders 2 Impaired hearing ability 3 Immunocompromise 4 Improper communication 5 Impaired social interaction

4, 5 Inability to communicate properly or delays in speech may signify developmental delays. When a child is found to have difficulty making friends or has impaired social interactions, the child should be referred for testing. In this condition, the nurse should provide further interventions for the child to support his or her development. Dermatological disorders are assessed in a child to check for genetic abnormalities. Impaired hearing is assessed through testing of the child's auditory function. Impaired hearing does not directly affect a child's behavior. A child who is immunocompromised has a weakened immune system and is at higher risk for infections.

The student nurse is caring for a 10-year-old child in the terminal stages of acute lymphocytic leukemia. Which instructions should the nursing instructor provide to the student nurse when caring for the child? Select all that apply. "You should: 1 Challenge the child to learn ways to manage her own care." 2 Advise the child's parents to pray to God for recovery." 3 "Not involve yourself in the family's emotional distress." 4 "Urge the parents to accept the truth and spend time together." 5 "Encourage the child's parents to ask for help when they need it."

4, 5 The nurse should encourage child's parents to accept the truth and spend the remainder of the time left making memories with the sick child. The nurse should encourage the child's parents to ask for help when they need it and not try to handle everything alone. The child should not be asked to manage her own care. During terminal illness the child needs to depend on her parents and family more than ever. The nurse should not advise the child's parents to pray to God because it is unethical to force religion on a family. The nurse should not get involved in the family's emotional distress. Instead, the nurse should teach them about ways to manage the child's condition.

A pregnant patient reports abdominal pain in the right lower quadrant, along with nausea and vomiting. The patient's urinalysis report shows an absence of any urinary tract infection in the patient. A chest x-ray also rules out lower-lobe pneumonia. Which condition does the nurse suspect in the patient? A. Appendicitis B. Cholelithiasis C. Placenta previa D. Uterine rupture

A (Abdominal pain in the right lower quadrant, accompanied by nausea and vomiting, indicates appendicitis in a pregnant patient. Cholelithiasis is characterized by right upper quadrant pain. Placenta previa is a condition wherein the placenta is implanted in the lower uterine segment covering the cervix, which causes bleeding when the cervix dilates. Uterine rupture is seen in a pregnant patient as a result of trauma, which may cause fetal death.)

In caring for the woman with disseminated intravascular coagulation (DIC), what order should the nurse anticipate? A. Administration of blood B. Preparation of the woman for invasive hemodynamic monitoring C. Restriction of intravascular fluids D. Administration of steroids

A (Administration of blood Primary medical management in all cases of DIC involves correction of the underlying cause, volume replacement, blood component therapy, optimization of oxygenation and perfusion status, and continued reassessment of laboratory parameters. Central monitoring would not be ordered initially in a woman with DIC because this can contribute to more areas of bleeding. Management of DIC includes volume replacement, not volume restriction. Steroids are not indicated for the management of DIC.)

Which occurrence is associated with cervical dilation and effacement? a. Bloody show b. Lightening c. False labor d. Bladder distention

A (As the cervix begins to soften, dilate, and efface, expulsion of the mucous plug that sealed the cervix during pregnancy occurs. This causes rupture of small cervical capillaries. Cervical dilation and effacement do not occur with false labor. Lightening is the descent of the fetus toward the pelvic inlet before labor. Bladder distention occurs when the bladder is not emptied frequently. It may slow down the descent of the fetus during labor.)

Within the first hour after birth, the nurse would expect to find the woman's fundus: a) At the level of the umbilicus b) One fingerbreadth below the umbilicus c) Between the umbilicus and symphysis pubis d) 2 cm above the umbilicus

A (At the level of the umbilicus Rationale: After birth, the fundus is located midline between the umbilicus and symphysis pubis but then slowly rises to the level of the umbilicus during the first hour after birth. Then the uterus contracts, approximately 1 cm (or fingerbreadth) each day after birth.)

A woman at 37 weeks of gestation is admitted with a placental abruption after a motor vehicle accident. Which assessment data are most indicative of her condition worsening? A. Pulse (P) 112, respiration (R) 32, blood pressure (BP) 108/60; fetal heart rate (FHR) 166--178 B. P 98, R 22, BP 110/74; FHR 150--162 C. P 88, R 20, BP 114/70; FHR 140--158 D. P 80, R 18, BP 120/78; FHR 138--150

A (Bleeding is the most dangerous problem, which impacts the mother's well-being as well as that of her fetus. The decreasing blood volume would cause increases in pulse and respirations and a decrease in blood pressure. The fetus often responds to decreased oxygenation as a result of bleeding, causing a decrease in perfusion. This causes the fetus' heart rate to increase above the normal range of 120--160 beats per minute. The other options have measurements that are in the "normal" range and would not reflect a deterioration of the patient's physical status. Test-Taking Tip: Do not worry if you select the same numbered answer repeatedly, because there usually is no pattern to the answers.)

A fetus is in the LST position. The nurse interprets this as indicating which of the following as the presenting part? a) Buttocks b) Fetal head c) Shoulder d) Chin

A (Buttocks Rationale: The letter "S" indicates the sacrum or buttocks as the presenting part. The fetal head would be noted by the letter "O," indicating occiput. The fetal chin would be noted by the letter "M," indicating mentum. The fetal shoulder would be noted by the letter "A," indicating the acromion process.)

A petite, 5-foot tall, 95-pound woman who is 28 years old is about to deliver her first child and would like to have a vaginal delivery. She has two sisters, both of whom have given birth vaginally. She has gained 25 pounds during a normal, uneventful pregnancy. What type of pelvis would a nurse expect this woman to have upon assessment of the patient? a) Cannot be determined b) Android c) Platypelloid d) Gynecoid

A (Cannot be determined Rationale: Pelvis shape cannot be determined by the information included in the statement. Early in the pregnancy, particularly if a woman has never delivered a baby vaginally, the practitioner may take pelvic measurements to estimate the size of the true pelvis. This helps to determine if the size is adequate for vaginal delivery. However, these measurements do not consistently predict which women will have difficulty delivering vaginally, so most practitioners allow the woman to labor and attempt a vaginal birth.)

A client in labor is agitated and nervous about the birth of her child. The nurse explains to the client that fear and anxiety cause the release of certain compounds which can prolong labor. Which of the following is the nurse referring to in the explanation? a) Catecholamines b) Relaxin c) Prostaglandins d) Oxytocin

A (Catecholamines Rationale: Fear and anxiety cause the release of catecholamines, such as norepinephrine and epinephrine which stimulate the adrenergic receptors of the myometrium. This in turn interferes with effective uterine contractions and results in prolonged labor. Estrogen promotes the release of prostaglandins and oxytocin. Relaxin is a hormone that is involved in producing backache by acting on the pelvic joints. Prostaglandins, oxytocin and relaxin are not produced due to fear or anxiety in clients during labor.)

The nurse notes that the fetal head is at the vaginal opening and does not regress between contractions. The nurse interprets this finding as which of the following? a) Crowning b) Engagement c) Descent d) Restitution

A (Crowning Rationale: Crowning occurs when the top of the fetal head appears at the vaginal orifice and no longer regresses between contractions. Engagement occurs when the greatest transverse diameter of the head passes through the pelvic inlet. Descent is the downward movement of the fetal head until it is within the pelvic inlet. Restitution or external rotation occurs after the head is born and free of resistance. It untwists, causing the occiput to move about 45 degrees back to its original left or right position.)

The nurse would expect which maternal cardiovascular finding during labor? a. Increased cardiac output b. Decreased pulse rate c. Decreased white blood cell (WBC) count d. Decreased blood pressure

A (During each contraction, 400 mL of blood is emptied from the uterus into the maternal vascular system. This increases cardiac output by about 51% above baseline pregnancy values at term. The heart rate increases slightly during labor. The WBC count can increase during labor. During the first stage of labor, uterine contractions cause systolic readings to increase by about 10 mm Hg. During the second stage, contractions may cause systolic pressures to increase by 30 mm Hg and diastolic readings to increase by 25 mm Hg.)

The initial descent of the fetus into the pelvis to zero station is which one of the cardinal movements of labor? a) Engagement b) Felxion c) Extension d) Expulsion

A (Engagement Rationale: The movement of the fetus into the pelvis from the upper uterus is engagement. This is the first cardinal movement of the fetus in preparation for the spontaneous vaginal delivery. Flexion occurs as the fetus encounters resistance from the soft tissues and muscles of the pelvic floor. Extension is the state in which the fetal head is well flexed with the chin on the chest as the fetus travels through the birth canal. Expulsion occurs after delivery of the anterior and posterior shoulders.)

Fetal circulation can be affected by many factors during labor. Accurate assessment of the laboring woman and fetus requires knowledge of these expected adaptations. Which factor will not affect fetal circulation during labor? a) Fetal position b) Uterine contractions c) Blood pressure d) Umbilical cord blood flow

A (Fetal position)

Which description of the four stages of labor is correct for both definition and duration? a. First stage: onset of regular uterine contractions to full dilation; less than 1 hour to 20 hours b. Second stage: full effacement to 4 to 5 cm; visible presenting part; 1 to 2 hours c. Third state: active pushing to birth; 20 minutes (multiparous women), 50 minutes (first-timer) d. Fourth stage: delivery of the placenta to recovery; 30 minutes to 1 hour

A (Full dilation may occur in less than 1 hour, but in first-time pregnancies it can take up to 20 hours. The second stage extends from full dilation to birth and takes an average of 20 to 50 minutes, although 2 hours is still considered normal. The third stage extends from birth to expulsion of the placenta and usually takes a few minutes. The fourth stage begins after expulsion of the placenta and lasts until homeostasis is reestablished (about 2 hours).)

A woman diagnosed with marginal placenta previa gave birth vaginally 15 minutes ago. At the present time, she is at the greatest risk for: A. hemorrhage. B. infection. C. urinary retention. D. thrombophlebitis.

A (Hemorrhage is the most immediate risk because the lower uterine segment has limited ability to contract to reduce blood loss. Infection is a risk because of the location of the placental attachment site; however, it is not a priority concern at this time. Placenta previa poses no greater risk for urinary retention than with a normally implanted placenta. There is no greater risk for thrombophlebitis than with a normally implanted placenta.)

The quantitative human chorionic gonadotropin (β-hCG) levels are high in a patient who is on methotrexate therapy for dissolving abdominal pregnancy. Which instruction does the nurse give to this patient? A. "Avoid sexual activity." B. "Avoid next pregnancy." C. "Avoid feeling sad and low." D. "Take folic acid without fail."

A (High β-hCG levels indicate that the abdominal pregnancy is not yet dissolved. Therefore the nurse advises the patient to avoid sexual activity until the β-hCG levels drop and the pregnancy is dissolved completely. If the patient engages in vaginal intercourse, the pelvic pressure may rupture the mass and cause pain. Abdominal pregnancy increases the chances of infertility or recurrent ectopic pregnancy in patients. However, the nurse need not instruct the patient to avoid further pregnancy, because it may increase the feelings of sadness and guilt in the patient. The nurse encourages the patient to share feelings of guilt or sadness related to pregnancy loss. Folic acid is contraindicated with methotrexate therapy, because it may exacerbate ectopic rupture.)

A woman with severe preeclampsia has been receiving magnesium sulfate by intravenous infusion for 8 hours. The nurse assesses the woman and documents the following findings: temperature 37.1° C, pulse rate 96 beats/min, respiratory rate 24 breaths/min, blood pressure 155/112 mm Hg, 3+ deep tendon reflexes, and no ankle clonus. The nurse calls the physician, anticipating an order for: A. hydralazine. B. magnesium sulfate bolus. C. diazepam. D. morphine

A (Hydralazine is an antihypertensive commonly used to treat hypertension in severe preeclampsia. An additional bolus of magnesium sulfate may be ordered for increasing signs of central nervous system irritability related to severe preeclampsia (e.g., clonus) or if eclampsia develops. Diazepam sometimes is used to stop or shorten eclamptic seizures. Calcium gluconate is used as the antidote for magnesium sulfate toxicity. The patient is not currently displaying any signs or symptoms of magnesium toxicity.)

Which presentation is described accurately in terms of both presenting part and frequency of occurrence? a. Cephalic: occiput; at least 95% b. Shoulder: scapula; 10% to 15% c. Breech: sacrum; 10% to 15% d. Cephalic: cranial; 80% to 85%

A (In cephalic presentations (head first), the presenting part is the occiput; this occurs in 96% of births. In a breech birth, the sacrum emerges first; this occurs in about 3% of births. In shoulder presentations, the scapula emerges first; this occurs in only 1% of births.)

A patient with gestational hypertension is prescribed labetalol hydrochloride (Normodyne) therapy, which is continued after giving birth. What does the nurse instruct the patient about breastfeeding? A. "You may breastfeed the infant if you desire." B. "Breastfeeding may cause convulsions in the infant." C. "Breastfeed only once a day and use infant formulas." D. "There may be high levels of the drug in the breast milk."

A (Labetalol hydrochloride (Normodyne) has a low concentration in breast milk, so the patient can breastfeed the infant. Breastfeeding is safe and will not cause convulsions or any side effects in the infant. Infant formulas are used only if the mother is unable to breastfeed the infant or if the mother does not desire to breastfeed.)

After being rehydrated in the emergency department, a 24 year-old primipara in her 18th week of pregnancy is at home and is to rest at home for the next two days and take in small but frequent fluids and food as possible. Discharge teaching at the hospital by the nurse has been effective if the patient makes which statement? "A. I'm going to eat five to six small servings per day, which contain such foods and fluids as tea, crackers, or a few bites of baked potato." B. "A strip of bacon and a fried egg will really taste good as long as I eat them slowly." C. "As long as I eat small amounts and allow enough time for digestion, I can eat almost anything, like barbequed chicken or spaghetti." D. "I'm going to stay only on clear fluids for the next 24 hours and then add dairy products like eggs and milk."

A (Once the vomiting has stopped, feedings are started in small amounts at frequent intervals. In the beginning, limited amounts of oral fluids and bland foods such as crackers, toast, or baked chicken are offered. Clear fluids alone do not contain enough calories and contain no protein. Most women are able to take nourishment by mouth after several days of treatment. They should be encouraged to eat small, frequent meals and foods that sound appealing (e.g., nongreasy, dry, sweet, and salty foods). Test-Taking Tip: Many times the correct answer is the longest alternative given, but do not count on it. NCLEX item writers (those who write the questions) are also aware of this and attempt to avoid offering you such "helpful hints.")

Which condition is seen in a pregnant patient if uterine artery Doppler measurements in the second trimester of pregnancy are abnormal? A. Preeclampsia B. HELLP syndrome C. Molar pregnancy D. Gestational hypertension

A (Preeclampsia is a condition in which patients develop hypertension and proteinuria after 20 weeks' gestation. It can be diagnosed if uterine artery Doppler measurements in the second trimester of pregnancy are abnormal. HELLP syndrome is characterized by hemolysis (H), elevated liver enzymes (EL), and low platelet count (LP) in a patient with preeclampsia. Molar pregnancy refers to the growth of the placental trophoblast due to abnormal fertilization. Gestational hypertension is a condition in which hypertension develops in a patient after 20 weeks' gestation.)

With regard to primary and secondary powers, the maternity nurse should understand that: a) Primary powers are responsible for effacement and dilation of the cervix b) Effacement generally is well ahead of dilation in women giving birth for the first time; they are more together in subsequent pregnancies c) Scarring of the cervix caused by a previous infection or surgery may make the delivery a bit more painful, but it should not slow or inhibit dilation d) Pushing in the second stage of labor is more effective if the woman can breathe deeply and control some of her involuntary needs to push, as the nurse directs

A (Primary powers are responsible for effacement and dilation of the cervix)

When teaching a group of nursing students about uterine contractions, which of the following would the instructor include as a typical feature? a) Retracting of the lower segment b) Pulling down of the cervical portion c) Thinning out of the upper segment d) Contracting stronger in the lower segment

A (Retracting of the lower segment Rationale: The nurse should identify retraction of the lower segment of the uterus as a feature of typical uterine contractions. As labor progresses the uterine contractions become stronger. The upper segment of the uterus contracts more actively than the lower segment. The lower segment retracts, pulling up the cervix. The upper segment thickens with time and the lower segment thins.)

Which condition in a pregnant patient with severe preeclampsia is an indication for administering magnesium sulfate? A. Seizure activity B. Renal dysfunction C. Pulmonary edema D. Low blood pressure (BP)

A (Severe preeclampsia may cause seizure activity or eclampsia in the patient, which is treated with magnesium sulfate. Magnesium sulfate is not administered for renal dysfunction and can cause magnesium toxicity in the patient. Pulmonary enema can be prevented by restricting the patient's fluid intake to 125 mL/hr. Increasing magnesium toxicity can cause low BP in the patient.)

The nurse is caring for a pregnant patient who is receiving antibiotic therapy to treat a urinary tract infection (UTI). Which dietary changes does the nurse suggest for the pregnant patient who is receiving antibiotic therapy for UTI? A. "Include yogurt, cheese, and milk in your diet." B. "Avoid folic acid supplements until the end of therapy." C. "Include vitamins C and E supplementation in your diet." D. "Reduce your dietary fat intake by 40 to 50 g per day.

A (The antibiotic therapy kills normal flora in the genitourinary tract, as well as pathologic organisms. Therefore the nurse instructs the patient to include yogurt, cheese, and milk in daily diet because they contain active acidophilus cultures. Folic acid should not be avoided, because it may affect the fetal development. Vitamins C and E supplementation is usually included in the diet to treat preeclampsia in a patient. Dietary fat is reduced in patients with cholecystitis or cholelithiasis, because it may cause epigastric pain.)

To care for a laboring woman adequately, the nurse understands that the __________ stage of labor varies the most in length? a. First b. Third c. Second d. Fourth

A (The first stage of labor is considered to last from the onset of regular uterine contractions to full dilation of the cervix. The first stage is much longer than the second and third stages combined. In a first-time pregnancy the first stage of labor can take up to 20 hours. The second stage of labor lasts from the time the cervix is fully dilated to the birth of the fetus. The average length is 20 minutes for a multiparous woman and 50 minutes for a nulliparous woman. The third stage of labor lasts from the birth of the fetus until the placenta is delivered. This stage may be as short as 3 minutes or as long as 1 hour. The fourth stage of labor, recovery, lasts about 2 hours after delivery of the placenta.)

Which instructions does the nurse give to a patient who is prescribed methotrexate therapy for dissolving the tubal pregnancy? A. "Discontinue folic acid supplements." B. "Get adequate exposure to sunlight." C. "Take stronger analgesics for severe pain." D. "Vaginal intercourse is safe during the therapy."

A (The nurse advises the patient to discontinue folic acid supplements as they interact with methotrexate and may exacerbate ectopic rupture in the patient. Exposure to sunlight is avoided as the therapy makes the patient photosensitive. Analgesics stronger than acetaminophen are avoided, because they may mask symptoms of tubal rupture. Vaginal intercourse is avoided until the pregnancy is dissolved completely.)

In relation to primary and secondary powers, the maternity nurse comprehends that: a. Primary powers are responsible for effacement and dilation of the cervix. b. Effacement generally is well ahead of dilation in women giving birth for the first time; they are closer together in subsequent pregnancies. c. Scarring of the cervix caused by a previous infection or surgery may make the delivery a bit more painful, but it should not slow or inhibit dilation. d. Pushing in the second stage of labor is more effective if the woman can breathe deeply and control some of her involuntary needs to push, as the nurse directs.

A (The primary powers are responsible for dilation and effacement; secondary powers are concerned with expulsion of the fetus. Effacement generally is well ahead of dilation in first-timers; they are closer together in subsequent pregnancies. Scarring of the cervix may slow dilation. Pushing is more effective and less fatiguing when the woman begins to push only after she has the urge to do so.)

The woman has just delivered a healthy baby boy, but the placenta has not yet delivered. What stage of labor does this scenario represent? a) Third b) Second c) Fourth d) First

A (Third Rationale: Stage 3 begins with the birth of the baby and ends with delivery of the placenta.)

A nurse is assisting a client who is in the first stage of labor. Which of the following principles should the nurse keep in mind to help make this client's labor and birth as natural as possible? a) Women should be able to move about freely throughout labor b) The support person's access to the client should be limited to prevent the client from becoming overwhelmed c) Routine intravenous fluid should be implemented d) A woman should be allowed to assume a supine position

A (Women should be able to move about freely throughout labor Rationale: Six major concepts that make labor and birth as natural as possible are as follows: 1) labor should begin on its own, not be artificially induced; 2) women should be able to move about freely throughout labor, not be confined to bed; 3) women should receive continuous support from a caring other during labor; 4) no interventions such as intravenous fluid should be used routinely; 5) women should be allowed to assume a nonsupine position such as upright and side-lying for birth; and 6) mother and baby should be housed together after the birth, with unlimited opportunity for breast-feeding.)

A woman diagnosed with marginal placenta previa gave birth vaginally 15 minutes ago. At the present time she is at the greatest risk for: A. hemorrhage. B. infection. C. urinary retention. D. thrombophlebitis.

A (hemorrhage. Hemorrhage is the most immediate risk because the lower uterine segment has limited ability to contract to reduce blood loss. Infection is a risk because of the location of the placental attachment site; however, it is not a priority concern at this time. Placenta previa poses no greater risk for urinary retention than with a normally implanted placenta. There is no greater risk for thrombophlebitis than with a normally implanted placenta.)

A woman with severe preeclampsia has been receiving magnesium sulfate by IV infusion for 8 hours. The nurse assesses the woman and documents the following findings: temperature 37.1° C, pulse rate 96 beats/min, respiratory rate 24 breaths/min, blood pressure 155/112 mm Hg, 3+ deep tendon reflexes, and no ankle clonus. The nurse calls the physician, anticipating an order for: A. hydralazine. B. magnesium sulfate bolus . C. diazepam. D. calcium gluconate.

A (hydralazine. Hydralazine is an antihypertensive commonly used to treat hypertension in severe preeclampsia. An additional bolus of magnesium sulfate may be ordered for increasing signs of central nervous system irritability related to severe preeclampsia (e.g., clonus) or if eclampsia develops. Diazepam sometimes is used to stop or shorten eclamptic seizures. Calcium gluconate is used as the antidote for magnesium sulfate toxicity. The client is not currently displaying any signs or symptoms of magnesium toxicity.)

Which statement best describes a child who is abused by the parent(s)? a. Unintentionally contributes to the abusing situation b. Belongs to a low socioeconomic population c. Is healthier than the nonabused siblings d. Abuses siblings in the same way as child is abused by the parent(s)

A A child's temperament, position in the family, additional physical needs, activity level, or degree of sensitivity to parental needs unintentionally contributes to the abusing situation. Socioeconomic status is an environmental characteristic. This child is less likely to be abused than one who is premature, disabled, or very young. The abused child does not in turn abuse his or her siblings.

Preschoolers' fears can best be dealt with by which intervention? a. Actively involving them in finding practical methods to deal with the frightening experience b. Forcing them to confront the frightening object or experience in the presence of their parents c. Using logical persuasion to explain away their fears and help them recognize how unrealistic the fears are d. Ridiculing their fears so they understand that there is no need to be afraid

A Actively involving the child in finding practical methods to deal with the frightening experience is the best way to deal with fears. Forcing a child to confront fears may make the child more afraid. Preconceptual thought prevents logical understanding. Ridiculing fears does not make them go away.

Which aspect of cognition develops during adolescence? a. Capability to use a future time perspective b. Ability to place things in a sensible and logical order c. Ability to see things from the point of view of another d. Progress from making judgments based on what they see to making judgments based on what they reason

A Adolescents are no longer restricted to the real and actual. They also are concerned with the possible; they think beyond the present. During concrete operations (between ages 7 and 11 years), children exhibit the ability to place things in a sensible and logical order, the ability to see things from another's point of view, and the ability to make judgments based on what they reason rather than just what they see.

The nurse providing care for the laboring woman should understand that amnioinfusion is used to treat: a. Variable decelerations. c. Fetal bradycardia. b. Late decelerations. d. Fetal tachycardia.

A Amnioinfusion is used during labor either to dilute meconium-stained amniotic fluid or to supplement the amount of amniotic fluid to reduce the severity of variable decelerations caused by cord compression. Amnioinfusion has no bearing on late decelerations, fetal bradycardia, or fetal tachycardia alterations in fetal heart rate (FHR) tracings.

Which statement is true about smoking in adolescence? a. Smoking is related to other high-risk behaviors. b. Smoking is more common among athletes. c. Smoking is less common when the adolescent's parent(s) smokes. d. Smoking among adolescents is becoming more prevalent.

A Cigarettes are considered a gateway drug. Teenagers who smoke are 11.4 times more likely to use an illicit drug. Teens who refrain from smoking often have a desire to succeed in athletics. If a parent smokes, it is more likely that the teen will smoke. Cigarette smoking has declined among all groups since the 1990s.

Which deceleration of the fetal heart rate would not require the nurse to change the maternal position? a. Early decelerations b. Late decelerations c. Variable decelerations d. It is always a good idea to change the woman's position.

A Early decelerations (and accelerations) generally do not need any nursing intervention. Late decelerations suggest that the nurse should change the maternal position (lateral); variable decelerations also require a maternal position change (side to side). Although changing positions throughout labor is recommended, it is not required in response to early decelerations.

The nurse caring for the laboring woman should understand that early decelerations are caused by: a. Altered fetal cerebral blood flow. c. Uteroplacental insufficiency. b. Umbilical cord compression. d. Spontaneous rupture of membranes.

A Early decelerations are the fetus's response to fetal head compression. Variable decelerations are associated with umbilical cord compression. Late decelerations are associated with uteroplacental insufficiency. Spontaneous rupture of membranes has no bearing on the fetal heart rate unless the umbilical cord prolapses, which would result in variable or prolonged bradycardia.

The nurse providing care for the laboring woman comprehends that accelerations with fetal movement: a. Are reassuring. b. Are caused by umbilical cord compression. c. Warrant close observation. d. Are caused by uteroplacental insufficiency.

A Episodic accelerations in the fetal heart rate (FHR) occur during fetal movement and are indications of fetal well-being. Umbilical cord compression results in variable decelerations in the FHR. Accelerations in the FHR are an indication of fetal well-being and do not warrant close observation. Uteroplacental insufficiency would result in late decelerations in the FHR.

Fetal well-being during labor is assessed by: a. The response of the fetal heart rate (FHR) to uterine contractions (UCs). b. Maternal pain control. c. Accelerations in the FHR. d. An FHR above 110 beats/min.

A Fetal well-being during labor can be measured by the response of the FHR to UCs. In general, reassuring FHR patterns are characterized by an FHR baseline in the range of 110 to 160 beats/min with no periodic changes, a moderate baseline variability, and accelerations with fetal movement. Maternal pain control is not the measure used to determine fetal well-being in labor. Although FHR accelerations are a reassuring pattern, they are only one component of the criteria by which fetal well-being is assessed. Although an FHR above 110 beats/min may be reassuring, it is only one component of the criteria by which fetal well-being is assessed. More information would be needed to determine fetal well-being.

Which is the most commonly used method in completed suicides? a. Firearms c. Self-inflected laceration b. Drug overdose d. Carbon monoxide poisoning

A Firearms are the most commonly used instruments in completed suicides among both males and females. For adolescent boys, firearms are followed by hanging and overdose. For adolescent females, overdose and strangulation are the next most common means of completed suicide. The most common method of suicide attempt is overdose or ingestion of potentially toxic substances such as drugs. The second most common method of suicide attempt is self-inflicted laceration. Carbon monoxide poisoning is not one of the more frequent forms of suicide completion.

A young adolescent boy tells the nurse he "feels gawky." The nurse should explain that this occurs in adolescents because: a. Growth of the extremities and neck precedes growth in other areas. b. Growth is in the trunk and chest. c. The hip and chest breadth increases. d. The growth spurt occurs earlier in boys than it does in girls.

A Growth in length of the extremities and neck precedes growth in other areas, and, because these parts are the first to reach adult length, the hands and feet appear larger than normal during adolescence. Increases in hip and chest breadth take place in a few months, followed several months later by an increase in shoulder width. These changes are followed by increases in length of the trunk and depth of the chest. This sequence of changes is responsible for the characteristic long-legged, gawky appearance of early adolescent children. The growth spurt occurs earlier in girls than in boys.

Which statement is correct about childhood obesity? a. Heredity is an important factor in the development of obesity. b. Childhood obesity in the United States is decreasing. c. Childhood obesity is the result of inactivity. d. Childhood obesity can be attributed to an underlying disease in most cases.

A Heredity is an important fact that contributes to obesity. Identical twins reared apart tend to resemble their biologic parents to a greater extent than their adoptive parents. It is difficult to distinguish between hereditary and environmental factors. The rate of childhood obesity has increased so dramatically that it has now reached epidemic proportions. Inactivity is an important contributing factor; however, obesity is the result of a combination of a number of other factors. Fewer than 5% of all cases of obesity can be linked to underlying disease.

Matt, age 14 years, seems to be always eating, although his weight is appropriate for his height. The best explanation for this is: a. This is normal because of increase in body mass. b. This is abnormal and suggestive of future obesity. c. His caloric intake would have to be excessive. d. He is substituting food for unfilled needs.

A In adolescence, nutritional needs are closely related to the increase in body mass. The peak requirements occur in the years of maximal growth. The caloric and protein requirements are higher than at almost any other time of life. This describes the expected eating pattern for young adolescents as long as weight and height are appropriate; obesity and substitution of food for unfilled needs are not concerns.

Which maternal condition is considered a contraindication for the application of internal monitoring devices? a. Unruptured membranes b. External monitors in current use c. Cervix dilated to 4 cm d. Fetus with a known heart defect

A In order to apply internal monitoring devices, the membranes must be ruptured. Cervical dilation of 4 cm permits the insertion of fetal scalp electrodes and intrauterine catheter. The external monitor can be discontinued after the internal ones are applied. A compromised fetus should be monitored with the most accurate monitoring devices.

A (normal development This indicates normal development. Reflexive grasping occurs during the first 2 to 3 months and then gradually becomes voluntary. No evidence of developmental lag, delayed development, or neurologic dysfunction is present.)

A 3-month-old infant, born at 38 weeks of gestation, will hold a rattle if it is put in her hands; however, she will not voluntarily grasp it. The nurse should interpret this as: a. Normal development. b. Significant developmental lag. c. Slightly delayed development caused by prematurity. d. Suggestive of a neurologic disorder such as cerebral palsy.

While evaluating an external monitor tracing of a woman in active labor, the nurse notes that the fetal heart rate (FHR) for five sequential contractions begins to decelerate late in the contraction, with the nadir of the decelerations occurring after the peak of the contraction. The nurse's first priority is to: a. Change the woman's position. c. Assist with amnioinfusion. b. Notify the care provider. d. Insert a scalp electrode.

A Late decelerations may be caused by maternal supine hypotension syndrome. They usually are corrected when the woman turns on her side to displace the weight of the gravid uterus from the vena cava. If the fetus does not respond to primary nursing interventions for late decelerations, the nurse would continue with subsequent intrauterine resuscitation measures, including notifying the care provider. An amnioinfusion may be used to relieve pressure on an umbilical cord that has not prolapsed. The FHR pattern associated with this situation most likely reveals variable deceleration. A fetal scalp electrode would provide accurate data for evaluating the well-being of the fetus; however, this is not a nursing intervention that would alleviate late decelerations, nor is it the nurse's first priority.

The nurse caring for a laboring woman is aware that maternal cardiac output can be increased by: a. Change in position. c. Regional anesthesia. b. Oxytocin administration. d. Intravenous analgesic.

A Maternal supine hypotension syndrome is caused by the weight and pressure of the gravid uterus on the ascending vena cava when the woman is in a supine position. This reduces venous return to the woman's heart, as well as cardiac output, and subsequently reduces her blood pressure. The nurse can encourage the woman to change positions and avoid the supine position. Oxytocin administration, regional anesthesia, and intravenous analgesic may reduce maternal cardiac output.

Perinatal nurses are legally responsible for: a. Correctly interpreting fetal heart rate (FHR) patterns, initiating appropriate nursing interventions, and documenting the outcomes. b. Greeting the client on arrival, assessing her, and starting an intravenous line. c. Applying the external fetal monitor and notifying the care provider. d. Making sure that the woman is comfortable.

A Nurses who care for women during childbirth are legally responsible for correctly interpreting FHR patterns, initiating appropriate nursing interventions based on those patterns, and documenting the outcomes of those interventions. Greeting the client, assessing her, and starting an IV; applying the external fetal monitor and notifying the care provider; and making sure the woman is comfortable may be activities that a nurse performs, but they are not activities for which the nurse is legally responsible.

An adolescent girl tells the nurse that she has suicidal thoughts. The nurse asks her if she has a specific plan. Asking this should be considered: a. An appropriate part of the assessment. b. Not a critical part of the assessment. c. Suggesting that the adolescent needs a plan. d. Encouraging the adolescent to devise a plan.

A Routine health assessments of adolescents should include questions that assess the presence of suicidal ideation or intent. Questions such as "Have you ever developed a plan to hurt yourself or kill yourself?" should be part of that assessment. Threats of suicide should always be taken seriously and evaluated. Suggesting that the adolescent needs a plan and encouraging her to devise this plan would be inappropriate statements by the nurse.

A common characteristic of those who sexually abuse children is that they: a. Pressure the victim into secrecy. b. Are usually unemployed and unmarried. c. Are unknown to victims and victims' families. d. Have many victims that are each abused only once.

A Sex offenders may pressure the victim into secrecy, regarding the activity as a "secret between us" that other people may take away if they find out. Abusers are often employed upstanding members of the community. Most sexual abuse is committed by men and persons who are well known to the child. Abuse is often repeated with the same child over time. The relationship may start insidiously without the child realizing that sexual activity is part of the offer.

A nurse is teaching adolescent boys about pubertal changes. The first sign of pubertal change seen with boys is: a. Testicular enlargement. c. Scrotal enlargement. b. Facial hair. d. Voice deepens.

A The first sign of pubertal changes in boys is testicular enlargement in response to testosterone secretion, which usually occurs in Tanner stage 2. Slight pubic hair is present and the smooth skin texture of the scrotum is somewhat altered. As testosterone secretion increases, the penis, testes, and scrotum enlarge. During Tanner stages 4 and 5, rising levels of testosterone cause the voice to deepen and facial hair appears at the corners of the upper lip and chin.

The nurse knows that proper placement of the tocotransducer for electronic fetal monitoring is located: a. Over the uterine fundus. c. Inside the uterus. b. On the fetal scalp. d. Over the mother's lower abdomen.

A The tocotransducer monitors uterine activity and should be placed over the fundus, where the most intensive uterine contractions occur. The tocotransducer is for external use.

In assisting with the two factors that have an effect on fetal status (i.e., pushing and positioning), nurses should: a. Encourage the woman's cooperation in avoiding the supine position. b. Advise the woman to avoid the semi-Fowler position. c. Encourage the woman to hold her breath and tighten her abdominal muscles to produce a vaginal response. d. Instruct the woman to open her mouth and close her glottis, letting air escape after the push.

A The woman should maintain a side-lying position. The semi-Fowler position is the recommended side-lying position with a lateral tilt to the uterus. The Valsalva maneuver, which encourages the woman to hold her breath and tighten her abdominal muscles, should be avoided. Both the mouth and glottis should be open, letting air escape during the push.

A useful skill that the nurse should expect a 5-year-old child to be able to master is to: a. Tie shoelaces. c. Hammer a nail. b. Use a knife to cut meat. d. Make change from a quarter.

A Tying shoelaces is a fine motor task typical of 5-year-olds. Using a knife to cut meat is a fine motor task of a 7-year-old. Hammering a nail and making change from a quarter are fine motor tasks of an 8- to 9-year-old.

D (casein hydrolysate milk formula.)

A 3-month-old bottle-fed infant is allergic to cow's milk. The nurse's BEST option for a substitute is: A. goat's milk. B. soy-based formula. C. skim milk diluted with water. D. casein hydrolysate milk formula.

C ("When your baby learns to roll over, you must supervise him whenever he is on a surface from which he might fall.")

A 4-month-old infant is brought to the clinic by his parents for a well-baby checkup. What should the nurse include at this time concerning injury prevention? A. "Never shake baby powder directly on your infant because it can be aspirated into his lungs." B. "Do not permit your child to chew paint from window ledges because he might absorb too much lead." C. "When your baby learns to roll over, you must supervise him whenever he is on a surface from which he might fall." D. "Keep doors of appliances closed at all times."

D (Peanut butter)

A 9-month-old infant is seen in the emergency department after developing a urticaric rash with cough and wheezing. When collecting the history of events before the sudden onset of the rash with cough and wheezing, the mother states they were "feeding the baby new foods." Which food is the possible cause of this type of reaction in the infant? A. Potatoes B. Green beans C. Spinach D. Peanut butter

D (The child is exhibiting the ritualism that is characteristic at this age. Ritualism is the need to maintain sameness and reliability. It provides a sense of comfort to the toddler. It will dictate certain principles in feeding practices, including rejecting a favorite food because it is served in a different container. This does not indicate a child who has unreasonable expectations or a need to exert control, but rather normal development. Toddlers use ritualistic behaviors to maintain necessary structure in their lives. This is not regression, which is a retreat from a present pattern of functioning.)

A father tells the nurse that his daughter wants the same plate and cup used at every meal, even if they go to a restaurant. The nurse should explain that this is: a. A sign that the child is spoiled. b. A way to exert unhealthy control. c. Regression, common at this age. d. Ritualism, common at this age.

D (A topical anesthetic, eutectic mixture of local anesthetic (EMLA), can be applied before injections are given. Several topical anesthetic agents can be used to minimize the discomfort associated with immunization injections. These include EMLA and vapor coolant sprays. Pain associated with many procedures can be prevented or minimized by using the principles of atraumatic care. With preparation, the injection site can be properly anesthetized to decrease the amount of pain felt by the infant. Infants have the neural pathways to sense pain. Numerous research studies have indicated that infants perceive and react to pain in the same manner as do children and adults. The mother should be allowed to discuss her concerns and the alternatives available. This is part of the informed consent process.)

A mother tells the nurse that she doesn't want her infant immunized because of the discomfort associated with injections. The nurse should explain that: a. This cannot be prevented. b. Infants do not feel pain as adults do. c. This is not a good reason for refusing immunizations. d. A topical anesthetic, eutectic mixture of local anesthetic (EMLA), can be applied before injections are given.

C (commercial iron-fortified formula For children younger than 1 year, the American Academy of Pediatrics recommends the use of breast milk. If breastfeeding has been discontinued, iron-fortified commercial formula should be used. Cow's milk should not be used in children younger than 12 months. Maternal iron stores are almost depleted by this age; the iron-fortified formula will help prevent the development of iron deficiency anemia.)

A mother tells the nurse that she is discontinuing breastfeeding her 5-month-old infant. The nurse should recommend that the infant be given: a. Skim milk. b. Whole cow's milk. c. Commercial iron-fortified formula d. Commercial formula without iron.

A (The nurse is using the toddler's inability to conserve. This is when the toddler is unable to understand the idea that a mass can be changed in size, shape, volume, or length without losing or adding to the original mass. Instead, toddlers judge what they see by the immediate perceptual clues given to them. A small glass means less amount of contrast. Magical thinking is believing that thoughts are all-powerful and can cause events. Centration is focusing on one aspect rather than considering all possible alternatives. Irreversibility is the inability to undo or reverse the actions initiated, such as being unable to stop doing an action when told.)

A nurse places some x-ray contrast the toddler is to drink in a small cup instead of a large cup. Which concept of a toddler's preoperational thinking is the nurse using? a. Inability to conserve b. Magical thinking c. Centration d. Irreversibility

A, C , E (A. "We will put a plastic fillers in all electrical plugs." C. "we will place a gate at the top and bottom of stairways." E. "we will remove front knobs from the stove." By the time babies reach 6 months of age, they begin to become much more active, curious, and mobile. Putting plastic fillers on all electrical plugs can prevent an electrical shock. Putting gates at the top and bottom of stairways will prevent falls. Removing front knobs from the stove can prevent burns. Poisonous substances should be stored in a locked cabinet, not in a cabinet that children can reach when they begin to climb. The household hot water heater should be turned down to 120 degrees or less.)

A nurse has completed a teaching session for parents about "baby-proofing" the home. Which statements made by the parents indicate an understanding of the teaching (select all that apply)? a. "We will put plastic fillers in all electrical plugs." b. "We will place poisonous substances in a high cupboard." c. "We will place a gate at the top and bottom of stairways." d. "We will keep our household hot water heater at 130 degrees." e. "We will remove front knobs from the stove."

D (Semiformed, seedy, yellow)

A nurse is caring for a 2-month-old exclusively breastfed infant with an admitting diagnosis of colic. Based on the nurse's knowledge of breastfed infants, what type of stool is expected? A. Dark brown and small hard pebbles B. Loose with green mucus streaks C. Formed and with white mucus D. Semiformed, seedy, yellow

B, C, E (Certain groups of infants are at increased risk for SIDS: those with low birth weight, low Apgar scores, or recent viral illness, and those of male sex. Breastfed infants and infants of average or above-average weight are not at higher risk for SIDS.)

A nurse is conducting education classes for parents of infants. The nurse plans to discuss sudden infant death syndrome (SIDS). Which risk factors should the nurse include as increasing an infant's risk of a SIDS incident (select all that apply)? a. Breastfeeding b. Low Apgar scores c. Male sex d. Birth weight in the 50th or higher percentile e. Recent viral illness

B, C, E, F (B. Rotavirus (RV) C. Diptheria, tetanus, and acellular pertussis (DTaP) E. Haemophilus influenzae type b (HIB) F. Inactivated poliovirus (IPV) The recommended immunization schedule for a 4-month-old, up to date on immunizations, would be to administer the RV, DTaP, HIB, and IPV vaccinations. The MMR and varicella vaccinations would not be administered until the child is at least 1 year of age.)

A nurse is preparing to administer routine immunizations to a 4-month-old infant. The infant is currently up to date on all previously recommended immunizations. Which immunizations will the nurse prepare to administer (select all that apply)? a. Measles, mumps, and rubella (MMR) b. Rotavirus (RV) c. Diphtheria, tetanus, and acellular pertussis (DTaP) d. Varicella e. Haemophilus influenzae type b (HIB) f. Inactivated poliovirus (IPV)

C (kwashiorkor.)

A nurse is providing education to a community group in preparation for a mission trip to a third world country with limited access to protein-based food sources. The nurse is aware that children in this country are at increased risk for: A. rickets. B. marasmus. C. kwashiorkor. D. pellagra.

B (Eliminate all secondhand smoke contact. To prevent and treat colic, teach parents that if household members smoke, they should avoid smoking near the infant; smoking activity should preferably be confined to outside of the home. A pacifier can be introduced for added sucking. The infant should be swaddled tightly with a soft, stretchy blanket and placed in an upright seat after feedings.)

A nurse is teaching parents about prevention and treatment of colic. Which should the nurse include in the teaching plan? a. Avoid use of pacifiers. b. Eliminate all secondhand smoke contact. c. Lay infant flat after feeding. d. Avoid swaddling the infant.

C (6 months Between ages 6 and 8 months, fear of strangers and stranger anxiety become prominent and are related to the infant's ability to discriminate between familiar and nonfamiliar people. At age 2 months, the infant is just beginning to respond differentially to the mother. At age 4 months, the infant is beginning the process of separation individuation when the infant begins to recognize self and mother as separate beings. Twelve months is too late and requires referral for evaluation if the infant does not fear strangers at this age.)

A parent asks the nurse "At what age do most babies begin to fear strangers?" The nurse responds that most infants begin to fear strangers at age: a. 2 months. b. 4 months. c. 6 months d. 12 months.

D (The nurse should suggest to the parent that questions should be phrased with realistic choices rather than "yes" or "no" answers. This provides a sense of control for the toddler and reduces the opportunity for negativism. Negativism is not an indication of stubbornness or insolence and should not be punished. The negativism is not a function of attention; the child is testing limits to gain an understanding of the world. The toddler is too young to be asked to not always say "no.")

A parent asks the nurse about how to respond to negativism in toddlers. The most appropriate recommendation is to: a. Punish the child. b. Provide more attention. c. Ask child not always to say "no." d. Reduce the opportunities for a "no" answer.

D (The acellular pertussis vaccine is recommended by the American Academy of Pediatrics beginning at age 6 weeks. Infants are at greater risk for complications of pertussis. The vaccine is not given after age 7 years, when the risks of the vaccine become greater than those of pertussis. The infant is highly susceptible to pertussis, which can be a life-threatening illness in this age-group.)

A parent asks the nurse whether her infant is susceptible to pertussis. The nurse's response should be based on which statement concerning susceptibility to pertussis? a. Neonates will be immune the first few months. b. If the mother has had the disease, the infant will receive passive immunity. c. Children younger than 1 year seldom contract this disease. d. Most children are highly susceptible from birth.

D. ("We stopped using the car seat now that my child is older.")

A parent has a 2-year-old in the clinic for a well-child checkup. Which statement by the parent would indicate to the nurse that the parent needs more instruction regarding accident prevention? A. "We locked all the medicines in the bathroom cabinet." B. "We turned the thermostat down on our hot water heater." C. "We placed gates at the top and bottom of the basement steps." D. "We stopped using the car seat now that my child is older."

A (Toddlers use distinct behaviors in the quest for autonomy. They express their will with continued negativity and the use of the word "no." Children at this age also have rapid mood swings. The nurse should reassure the parents that their child is engaged in expected behavior for an 18-month-old.)

A parent of an 18-month-old boy tells the nurse that he says "no" to everything and has rapid mood swings. If he is scolded, he shows anger and then immediately wants to be held. The nurse's best interpretation of this behavior is that: a. This is normal behavior for his age. b. This is unusual behavior for his age. c. He is not effectively coping with stress. d. He is showing he needs more attention.

B (this is a normal reaction for this age.)

A parent of an 8-month-old infant tells the nurse that the baby cries and screams whenever he or she is left with the grandparents. The nurse's reply should be based on knowledge that: A. the infant is most likely spoiled. B. this is a normal reaction for this age. C. this is an abnormal reaction for this age. D. grandparents are not responsive to that infant.

A (Encourage parent to verbalize feelings. Colic is multifactorial, and no single treatment is effective for all infants. The parent is verbalizing concern and worry. The nurse should allow the parent to put these feelings into words. An empathic, gentle, and reassuring attitude, in addition to suggestions about remedies, will help alleviate the parent's anxieties. The nurse should reassure the parent that he or she is not doing anything wrong. The infant with colic is experiencing spasmodic pain that is manifested by loud crying, in some cases up to 3 hours each day. Telling the parent that it will eventually go away does not help him or her through the current situation.)

A parent of an infant with colic tells the nurse, "All this baby does is scream at me; it is a constant worry." The nurse's best action is: a. Encourage parent to verbalize feelings. b. Encourage parent not to worry so much. c. Assess parent for other signs of inadequate parenting. d. Reassure parent that colic rarely lasts past age 9 months.

17. Since the gene for cystic fibrosis was identified in 1989, data can be collected for the purposes of genetic counseling for couples regarding carrier status. According to statistics, how often does cystic fibrosis occur in Caucasian live births? A. 1 in 100 B. 1 in 1200 C. 1 in 2500 D. 1 in 3000

ANS: D Cystic fibrosis occurs in about 1 in 3000 Caucasian live births.

C (The parent should be told that the best way to deal with temper tantrums is to ignore the behaviors, provided that the actions are not dangerous to the child. Tantrums are common during this age-group as the child becomes more independent and increasingly complex tasks overwhelm him or her. The parents and caregivers need to have consistent and developmentally appropriate expectations. Punishment and explanations will not be beneficial.)

A toddler's parent asks the nurse for suggestions on dealing with temper tantrums. The most appropriate recommendation is to: a. Punish the child. b. Leave the child alone until the tantrum is over. c. Ignore the behavior, provided that it is not injurious. d. Explain to child that this is wrong.

Which clinical reports does the nurse evaluate to identify ectopic pregnancy in a patient? Select all that apply. A. Quantitative human chorionic gonadotropin (β-hCG) levels B. Transvaginal ultrasound C. Progesterone level D. Thyroid test reports E. Kleihauer-Betke (KB) test

A, B, C (An ectopic pregnancy is indicated when β-hCG levels are >1500 milli-international units/mL but no intrauterine pregnancy is seen on the transvaginal ultrasound. A transvaginal ultrasound is repeated to verify if the pregnancy is inside the uterus. A progesterone level <5 ng/mL indicates ectopic pregnancy. Thyroid test reports need to be evaluated in case the patient has hyperemesis gravidarum, as hyperthyroidism is associated with this disorder. The KB test is used to determine transplacental hemorrhage.)

A pregnant patient in the first trimester reports spotting of blood with the cervical os closed and mild uterine cramping. What does the nurse need to assess? Select all that apply. A. Progesterone levels B. Transvaginal ultrasounds C. Human chorionic gonadotropin (hCG) measurement D. Blood pressure E. Kleihauer-Betke (KB) test reports

A, B, C (The spotting of blood with the cervical os closed and mild uterine cramping in the first trimester indicates a threatened miscarriage. Therefore the nurse needs to assess progesterone levels, transvaginal ultrasounds, and measurement of hCG to determine whether the fetus is alive and within the uterus. Blood pressure measurements do not help determine the fetal status. KB assay is prescribed to identify fetal-to-maternal bleeding, usually after a trauma.)

39. Most women with uncomplicated pregnancies can use the nurse as their primary source for nutritional information. The nurse or midwife should refer a client to a registered dietitian for in-depth nutritional counseling in the following situations (Select all that apply). a. Preexisting or gestational illness such as diabetes b. Ethnic or cultural food patterns c. Obesity d. Vegetarian diet e. Allergy to tree nuts

A, B, C, D

Which conditions during pregnancy can result in preeclampsia in the patient? Select all that apply. A. Genetic abnormalities B. Dietary deficiencies C. Abnormal trophoblast invasion D. Cardiovascular changes E. Maternal hypotension

A, B, C, D (Current theories consider that genetic abnormalities and dietary deficiencies can result in preeclampsia. Abnormal trophoblast invasion causes fetal hypoxia and results in maternal hypertension. Cardiovascular changes stimulate the inflammatory system and result in preeclampsia in the pregnant patient. Maternal hypertension, and not hypotension, after 20 weeks' gestation is known as preeclampsia.)

Strict isolation is required for a child who is hospitalized with (select all that apply): a. Mumps. b. Chickenpox. c. Exanthema subitum (roseola). d. Erythema infectiosum (fifth disease). e. Parvovirus B19.

A, B, C, D Childhood communicable diseases requiring strict transmission-based precautions (Contact, Airborne, and Droplet Precautions) include diphtheria, chickenpox, measles, mumps, tuberculosis, adenovirus, Haemophilus influenzae type B, mumps, pertussis, plague, streptococcal pharyngitis, and scarlet fever. Strict isolation is not required for parvovirus B19.

Which toys should a nurse provide to promote imaginative play for a 3-year-old hospitalized child (select all that apply)? a. Plastic telephone b. Hand puppets c. Jigsaw puzzle (100 pieces) d. Farm animals and equipment e. Jump rope

A, B, D To promote imaginative play for a 3-year-old child, the nurse should provide: dress-up clothes, dolls and dollhouses, housekeeping toys, play-store toys, telephones, farm animals and equipment, village sets, trains, trucks, cars, planes, hand puppets, and medical kits. A 100-piece jigsaw puzzle and a jump rope would be appropriate for a young, school-age child but not a 3-year-old child.

While caring for a child, the nurse provides small toys and works overtime to take care of the child, and even calls the hospital during off-duty time to find out whether the child is improving. The nurse regularly meets the mother outside of the hospital. The nurse asks the mother if she is involved in care of the child. Which actions of the nurse indicate a nontherapeutic nurse-patient relationship? Select all that apply. A Giving a toy to the child B Working overtime to look after the child C Calling the hospital frequently to inquire about the child D Asking whether the mother is involved in care of the child E Meeting the mother outside of the hospital to discuss the child

A, B, C, E Giving toys, clothes, food, and other items to the patient; working overtime to take care of a particular patient; calling the hospital or patient's home frequently to inquire about the patient's health; and meeting the patient or family outside of the hospital indicate that the nurse is overinvolved with the child and the family. Thus, such actions should be avoided to develop a good nurse-patient relationship. Asking if the mother is involved in the child's care indicates that the nurse is concerned about the patient and the family. It indicates a therapeutic relationship between the nurse and the child and family.

Which play patterns does a 3-year-old child typically display (select all that apply)? a. Imaginary play b. Parallel play c. Cooperative play d. Structured play e. Associative play

A, B, C, E Children between ages 3 and 5 years enjoy parallel and associative play. Children learn to share and cooperate as they play in small groups. Play is often imitative, dramatic, and creative. Imaginary friends are common around age 3 years. Structured play is typical of school-age children.

Which hypertensive disorders can occur during pregnancy? Select all that apply. A. Chronic hypertension B. Preeclampsia-eclampsia C. Hyperemesis gravidarum D. Gestational hypertension E. Gestational trophoblastic disease

A, B, D (Chronic hypertension refers to hypertension that developed in the pregnant patient before 20 weeks' gestation. Preeclampsia refers to hypertension and proteinuria that develops after 20 weeks' gestation. Eclampsia is the onset of seizure activity in a pregnant patient with preeclampsia. Gestational hypertension is the onset of hypertension after 20 weeks' gestation. Gestational trophoblastic disease and hyperemesis gravidarum are not hypertensive disorders. Gestational trophoblastic disease refers to a disorder without a viable fetus that is caused by abnormal fertilization. Hyperemesis gravidarum is excessive vomiting during pregnancy that may result in weight loss and electrolyte imbalance.)

A nurse is caring for a client in her fourth stage of labor. Which of the following assessments would indicate normal physiologic changes occurring during the fourth stage of labor? Select all that apply. a) Well-contracted uterus in the midline b) Mild uterine cramping and shivering c) Increase in the blood pressure d) Decreased intra-abdominal pressure e) Decrease in the pulse rate

A, B, D (Rationale: The normal physiologic changes for which a nurse should assess during the fourth stage of labor are a well-contracted uterus in the midline of the abdomen, mild cramping pain and generalized shivering, and decreased intra-abdominal pressure. Hemodynamic changes are due to normal blood loss during delivery, causing moderate tachycardia and a slight fall in the blood pressure during the fourth stage of labor. A fall in the pulse rate and increased blood pressure are not normal findings occurring during the fourth stage of labor.)

Which factors influence cervical dilation (Select all that apply)? a. Strong uterine contractions b. The force of the presenting fetal part against the cervix c. The size of the female pelvis d. The pressure applied by the amniotic sac e. Scarring of the cervix

A, B, D, E (Dilation of the cervix occurs by the drawing upward of the musculofibrous components of the cervix, which is caused by strong uterine contractions. Pressure exerted by the amniotic fluid while the membranes are intact or by the force applied by the presenting part also can promote cervical dilation. Scarring of the cervix as a result of a previous infection or surgery may slow cervical dilation. Pelvic size does not affect cervical dilation.)

What does the nurse assess to detect the presence of a hypertensive disorder in a pregnant patient? Select all that apply. A. Proteinuria B. Epigastric pain C. Placenta previa D. Presence of edema E. Blood pressure (BP)

A, B, D, E (Proteinuria indicates hypertension in a pregnant patient. Proteinuria is concentration ≥300 mg/24 hours in a 24-hour urine collection. The nurse needs to assess the patient for epigastric pain because it indicates severe preeclampsia. Hypertension is likely to cause edema or swollen ankles as a result of greater hydrostatic pressure in the lower parts of the body. Therefore the nurse needs to assess the patient for the presence of edema. Accurate measurement of BP will help detect the presence of any hypertensive disorder. A systolic BP greater than 140 mm Hg or a diastolic BP greater than 90 mm Hg will indicate hypertension. Placenta previa is a condition wherein the placenta is implanted in the lower uterine segment covering the cervix, which causes bleeding when the cervix dilates.)

A 24-year-old primipara, who is 18 weeks pregnant, has been having increasing vomiting since she was 8 weeks pregnant. Upon arrival at the emergency department, her skin turgor is diminished, temperature is 99.2F (o), pulse is 102, respiration is 18, blood pressure is 102/68, and she has deep furrows on her tongue. What would the nurse expect to do to care for her? Select all that apply. A. Start an intravenous infusion. B. Check her urine for ketones C. Cross match blood for a transfusion. D. Obtain a complete history. E. Obtain blood for a complete blood count

A, B, D, E (Whenever a pregnant woman has nausea and vomiting, the first priority is a thorough assessment to determine the severity of the problem. In most cases the woman should be told to come immediately to the health care provider's office or the emergency department because the severity of the illness often is difficult to determine by telephone conversation. The assessment should include frequency, severity, and duration of episodes of nausea and vomiting. If the woman reports vomiting, the assessment also should include the approximate amount and color of the vomitus. The woman is asked to report any precipitating factors relating to the onset of her symptoms. Any pharmacologic or nonpharmacologic treatment measures used should be recorded. Prepregnancy weight and documented weight gain or loss during pregnancy are important to note. The woman's weight and vital signs are measured, and a complete physical examination is performed, with attention to signs of fluid and electrolyte imbalance and nutritional status. The most important initial laboratory test to be obtained is a determination of ketonuria. Other laboratory tests that may be prescribed are a urinalysis, a complete blood cell count, electrolytes, liver enzymes, and bilirubin levels. At this time, there is no supportive evidence that a blood transfusion is required. Based on provided objective data that the patient has deep furrows on her tongue, this may suggest a vitamin B deficiency which should be investigated further.)

The nurse is caring for a woman who is at 24 weeks of gestation with suspected severe preeclampsia. Which signs and symptoms would the nurse expect to observe? (Select all that apply.) A. Decreased urinary output and irritability B. Transient headache and +1 proteinuria C. Ankle clonus and epigastric pain D. Platelet count of less than 100,000/mm3 and visual problems E. Seizure activity and hypotension

A, C, D (Decreased urinary output and irritability are signs of severe eclampsia. Ankle clonus and epigastric pain are signs of severe eclampsia. Platelet count of less than 100,000/mm3 and visual problems are signs of severe preeclampsia. A transient headache and +1 proteinuria are signs of preeclampsia and should be monitored. Seizure activity and hyperreflexia are signs of eclampsia.)

The nurse is caring for a woman who is at 24 weeks of gestation with suspected severe preeclampsia. Which signs and symptoms should the nurse expect to observe? Select all that apply. A. Decreased urinary output and irritability B. Transient headache and +1 proteinuria C. Ankle clonus and epigastric pain D. Platelet count of less than 100,000/mm3 and visual problems E. Seizure activity and hypotension

A, C, D (Decreased urinary output and irritability are signs of severe eclampsia. Ankle clonus and epigastric pain are signs of severe eclampsia. Platelet count of less than 100,000/mm3 and visual problems are signs of severe preeclampsia. A transient headache and +1 proteinuria are signs of preeclampsia and should be monitored.Seizure activity and hyperreflexia are signs of eclampsia. Test-Taking Tip: Do not worry if you select the same numbered answer repeatedly, because there usually is no pattern to the answers.)

A 9-month-old child has been treated after a choking incident. Which advice does the nurse give to the parents to prevent further incidents? Select all that apply. A "Never leave your child unattended." B "Your child is too young to be allowed to eat solid food." C "Make sure all cabinets, drawers, and containers are childproof." D "Marbles and LEGOs are not appropriate toys for children at that age." E "Allowing your child to crawl on the floor increases the risk for injury."

A, C, D Crawling infants may explore their environments through taste and touch. They tend to put everything in their mouths. Therefore, the nurse should advise the parents to always keep a watch on their child and never leave their child unattended. Children at 9 months of age will begin to be able to pull themselves up to standing positions, potentially giving them access to cabinets, drawers, and containers that they were unable to access before. Therefore, the parents need to thoroughly childproof the home. The parents should be warned about providing small, colorful toys or leaving small objects on the floor because the child will be attracted to such objects and may attempt to swallow them, which may result in choking. The child is 8 months old. By this age, solid foods are permitted. The child should be allowed to play and crawl on the floor, as this is helpful for the development of the child's gross motor skills.

Research has shown that the most successful smoking cessation programs among teens include (select all that apply): a. Peer-led education and support. b. Information on the long-term effects of smoking. c. Programs including the media. d. School-based programs. e. Information on the immediate effects of smoking.

A, C, D, E Two areas of antismoking campaigns that have shown success are those that are peer-led and use media in education related to smoking prevention. School-based programs have also shown success and can be strengthened by expansion into the community and youth groups. Teens respond much better to education that focuses on the immediate effects of smoking. For the most part, smoking prevention programs that focus on the negative long-term effects of smoking have been ineffective.

Which actions of the nurse indicate that the nurse is providing atraumatic care to the patient who is admitted in the intensive care unit and his or her family? Select all that apply. A The nurse patiently listens to the parent's concerns. B The nurse spends off-duty time playing with the child. C The nurse allows a parent to stay with the child at all times. D The nurse gives the appropriate pain medications to the child. E The nurse explains the treatment given to the child to the parents

A, D, E One important goal of atraumatic care is to reduce psychological distress of patients and their families. It is important that the nurse patiently listens to the parents' concerns and provides emotional support. Preventing and minimizing bodily pain is another important aspect of atraumatic care. Therefore, the nurse should give appropriate pain medications to the child. Another goal of atraumatic care is to keep the parents informed about the child's disease, treatments given, and the disease progression. Thus, the nurse should explain the treatments given to the parents. Spending off-duty time with the patient indicates that the nurse is overinvolved with the patient. Allowing a parent to stay with the child at all times is not part of atraumatic care.

Injuries claim many lives during adolescence. Which factors contribute to early adolescents engaging in risk-taking behaviors (select all that apply)? a. Peer pressure b. A desire to master their environment c. Engagement in the process of separation from their parents d. A belief that they are invulnerable e. Impulsivity

A, D, E Peer pressure (including impressing peers) is a factor contributing to adolescent injuries. During early to middle adolescence, children feel that they are exempt from the consequences of risk-taking behaviors; they believe that negative consequences only happen to others. Feelings of invulnerability ("It can't happen to me") are evident in adolescence. Impulsivity places adolescents in unsafe situations. Mastering the environment is the task of young school-age children. Emancipation is a major issue for the older adolescent. The process is accomplished as the teenager gains an education or vocational training.

Which opiate causes euphoria, relaxation, drowsiness, and detachment from reality and has possible effects on the pregnancy, including preeclampsia, intrauterine growth restriction, and premature rupture of membranes? A. Heroin B. Alcohol C. Phencyclidine palmitate (PCP) D. Cocaine

A. Heroin The opiates include opium, heroin, meperidine, morphine, codeine, and methadone. The signs and symptoms of heroin use are euphoria, relaxation, relief from pain, detachment from reality, impaired judgment, drowsiness, constricted pupils, nausea, constipation, slurred speech, and respiratory depression. Possible effects on pregnancy include preeclampsia, intrauterine growth restriction, miscarriage, premature rupture of membranes, infections, breech presentation, and preterm labor. Alcohol is not an opiate. PCP is not an opiate. Cocaine is not an opiate.

The nurse is caring for a woman with mitral stenosis who is in the active stage. Which action should the nurse take to promote cardiac function? A. Maintain the woman in a side-lying position with the head and shoulders elevated to facilitate hemodynamics B. Prepare the woman for delivery by cesarean section since this is the recommended delivery method to sustain hemodynamics C. Encourage the woman to avoid the use of narcotics or epidural regional analgesia since this alters cardiac function D. Promote the use of the Valsalva maneuver during pushing in the second stage to improve diastolic ventricular filling

A. Maintain the woman in a side-lying position with the head and shoulders elevated to facilitate hemodynamics The side-lying position with the head and shoulders elevated helps to facilitate hemodynamics during labor. A vaginal delivery is the preferred method of delivery for a woman with cardiac disease as it sustains hemodynamics better than a cesarean section. The use of supportive care, medication, and narcotics or epidural regional analgesia is not contraindicated with a woman with heart disease. The use of the Valsalva maneuver during pushing in the second stage should be avoided because it reduces diastolic ventricular filling and obstructs left ventricular outflow.

A nurse is caring for a woman with mitral stenosis who is in the active stage. Which action should the nurse take to promote cardiac function? A. Maintain the woman in a side-lying position with the head and shoulders elevated to facilitate hemodynamics. B. Prepare the woman for delivery by cesarean section because this is the recommended delivery method to sustain hemodynamics. C. Encourage the woman to avoid the use of narcotics or epidural regional analgesia because this alters cardiac function. D. Promote the use of the Valsalva maneuver during pushing in the second stage to improve diastolic ventricular filling.

A. Maintain the woman in a side-lying position with the head and shoulders elevated to facilitate hemodynamics. The side-lying position with the head and shoulders elevated helps facilitate hemodynamics during labor. A vaginal delivery is the preferred method for a woman with cardiac disease because it sustains hemodynamics better than a cesarean section. The use of supportive care, medication, and narcotics or epidural regional analgesia is not contraindicated with a woman with heart disease. Epidural anesthesia for labor is preferred. (Easterling and Stout, 2012). Using the Valsalva maneuver during pushing in the second stage should be avoided because it reduces diastolic ventricular filling and obstructs left ventricular outflow.

A pregnant woman is being examined by the nurse in the outpatient obstetric clinic. The nurse suspects systemic lupus erythematosus (SLE) after revealing which symptoms? (Select all that apply.) A. Muscle aches B. Hyperactivity C. Weight changes D. Fever E. Hypotension

A. Muscle aches C. Weight changes D. Fever Fatigue, rather than hyperactivity is a common sign of SLE. Hypotension is not a characteristic sign of SLE. Common symptoms, including myalgias, fatigue, weight change, and fevers, occur in nearly all women with SLE at some time during the course of the disease. Although a diagnosis of SLE is suspected based on clinical signs and symptoms, it is confirmed by laboratory testing that demonstrates the presence of circulating autoantibodies. As is the case with other autoimmune diseases, SLE is characterized by a series of exacerbations (flares) and remissions (Chin and Branch, 2012).

Thalassemia is a relatively common anemia in which: A. an insufficient amount of hemoglobin is produced to fill the red blood cells (RBCs). B. RBCs have a normal life span but are sickled in shape. C. folate deficiency occurs. D. there are inadequate levels of vitamin B12.

A. an insufficient amount of hemoglobin is produced to fill the red blood cells (RBCs). Thalassemia is a hereditary disorder that involves the abnormal synthesis of the á or â chains of hemoglobin. An insufficient amount of hemoglobin is produced to fill the RBCs. This is the underlying description for sickle cell anemia. Folate deficiency is the most common cause of megaloblastic anemias during pregnancy. B12 deficiency must also be considered if the pregnant woman presents with anemia.

Hypothyroidism occurs in 2 to 3 pregnancies per 1000. Pregnant women with untreated hypothyroidism are at risk for: (Select all that apply.) A. miscarriage. B. macrosomia. C. gestational hypertension. D. placental abruption. E. stillbirth.

A. miscarriage. C. gestational hypertension. D. placental abruption. E. stillbirth. Hypothyroidism is often associated with both infertility and an increased risk of miscarriage. Infants born to mothers with hypothyroidism are more likely to be of low birth weight or preterm. These outcomes can be improved with early diagnosis and treatment. Pregnant women with hypothyroidism are more likely to experience both preeclampsia and gestational hypertension. Placental abruption and stillbirth are risks associated with hypothyroidism. Placental abruption and stillbirth are risks associated with hypothyroidism.

A 6-year-old child is having a generalized seizure in the classroom at school. Place in order the interventions the school nurse should implement starting with the highest-priority intervention sequencing to the lowest-priority intervention. Provide answer using lowercase letters separated by commas (e.g., a, b, c, d, e). a. Take vital signs. b. Ease child to the floor. c. Allow child to rest. d. Turn child to the side. e. Integrate child back into the school environment.

ANS: b, d, a, c, e The nurse should ease the child to the floor immediately during a generalized seizure. During (and sometimes after) the generalized seizure, the swallowing reflex is lost, salivation increases, and the tongue is hypotonic. Therefore, the child is at risk for aspiration and airway occlusion. Placing the child on the side facilitates drainage and helps maintain a patent airway. Vital signs should be taken next and the child should be allowed to rest. When feasible, the child is integrated into the environment as soon as possible.

16. Which of the following drugs is an angiotensin-converting enzyme (ACE) inhibitor? 1. Captopril (Capoten) 2. Furosemide (Lasix) 3. Spironolactone (Aldactone) 4. Chlorothiazide (Diuril)

ANS: 1 1. Capoten is a drug which is an ACE inhibitor.

15. A beneficial effect of administering digoxin (Lanoxin) is that it: 1. Decreases edema. 2. Decreases cardiac output. 3. Increases heart size. 4. Increases venous pressure.

ANS: 1 1. Digoxin has a rapid onset and is useful increasing cardiac output, decreasing venous pressure, and as a result, decreasing edema.

33. What painful, tender, pea-sized nodules may appear on the pads of the fingers or toes in bacterial endocarditis? 1. Osler nodes 2. Janeway lesions 3. Subcutaneous nodules 4. Aschoff nodes

ANS: 1 1. Osler nodes are red, painful, intradermal nodes found on pads of the phalanges in bacterial endocarditis.

36. Which of the following is a major clinical manifestation of rheumatic fever? 1. Polyarthritis 2. Osler nodes 3. Janeway spots 4. Splinter hemorrhages of distal third of nails

ANS: 1 1. Polyarthritis, which is swollen, hot, red, and painful joints. The affected joints will change every 1 to 2 days. Primarily the large joints are affected.

8. Which of the following should be included in the instructions to an active adolescent who is going home after a cardiac catheterization? 1. Avoid tub baths but may shower. 2. Maintain strict bed rest for 3 days. 3. Leave pressure dressing on for 7 days. 4. Stay home from school until Band-Aid is removed.

ANS: 1 1. The catheterization site should be kept relatively dry with a adhesive bandage. Showers are recommended.

21. The infant with congestive heart failure has a need for: 1. Increased calories. 2. Increased fluids. 3. Decreased protein. 4. Decreased fat.

ANS: 1 1. The metabolic rate of infants with congestive heart failure is greater because of poor cardiac function and increased heart and respiratory rates. Their caloric needs are greater than those of the average infants, yet their ability to take in the calories is diminished by their fatigue.

22. As part of the treatment for congestive heart failure, the child takes the diuretic furosemide. As part of teaching home care, the nurse encourages the family to give the child foods such as bananas, oranges, and leafy vegetables. These foods are recommended for this child because they are high in which of the following? 1. Chlorides 2. Potassium 3. Sodium 4. Vitamins

ANS: 2 2. Diuretics that work on the proximal and distal renal tubules contribute to increased losses of potassium. The child's diet should be supplemented with this electrolyte

17. An 8-year-old child is receiving digoxin (Lanoxin). The nurse should notify the practitioner and withhold the medication if the apical pulse is less than which of the following? 1. 60 2. 70 3. 90 4. 100

ANS: 2 2. If a 1-minute apical pulse is less than 70 for an older child, the digoxin is withheld. 1. This is the cut-off for holding the digoxin dose in an adult.

25. The nurse is caring for a child with persistent hypoxia secondary to a cardiac defect. The nurse recognizes that a risk exists of cerebrovascular accidents (strokes). Which of the following is an important objective to decrease this risk? 1. Minimize seizures 2. Prevent dehydration 3. Promote cardiac output 4. Reduce energy expenditure

ANS: 2 2. In children with persistent hypoxia, polycythemia develops. Dehydration must be prevented in hypoxemic children because it potentiates the risk of strokes

12. Which of the following defects results in decreased pulmonary blood flow? 1. Atrial septal defect 2. Tetralogy of Fallot 3. Ventricular septal defect 4. Patent ductus arteriosus

ANS: 2 2. Tetralogy of Fallot results in decreased blood flow to the lungs. The pulmonic stenosis increases the pressure in the right ventricle, causing the blood to go from right to left across the ventricular septal defect

27. Which of the following should the nurse consider when preparing a school-age child and the family for heart surgery? 1. Unfamiliar equipment should not be shown. 2. Let child hear the sounds of an ECG monitor. 3. Avoid mentioning postoperative discomfort and interventions. 4. Explain that an endotracheal tube will not be needed if the surgery goes well.

ANS: 2 2. The child and family should be exposed to the sights and sounds of the intensive care unit (ICU). All positive, nonfrightening aspects of the environment are emphasized

24. An 8-month-old infant has a hypercyanotic spell while blood is being drawn. The nurse's first action should be which of the following? 1. Assess for neurologic defects. 2. Place the child in the knee-chest position 3. Begin cardiopulmonary resuscitation. 4. Prepare family for imminent death.

ANS: 2 2. The first action is to place the infant in the knee-chest position. Blow-by oxygen may be indicated.

34. The primary nursing intervention to prevent bacterial endocarditis is which of the following? 1. Institute measures to prevent dental procedures. 2. Counsel parents of high-risk children about prophylactic antibiotics. 3. Observe children for complications, such as embolism and heart failure. 4. Encourage restricted mobility in susceptible children.

ANS: 2 2. The objective of nursing care is to counsel the parents of high-risk children about both the need for prophylactic antibiotics for dental procedures and the necessity of maintaining excellent oral health. The child's dentist should be aware of the child's cardiac condition.

40. Which of the following is the leading cause of death after heart transplantation? 1. Infection 2. Rejection 3. Cardiomyopathy 4. Congestive heart failure

ANS: 2 2. The posttransplant course is complex. The leading cause of death after cardiac transplant is rejection.

18. A 6-month-old infant is receiving digoxin (Lanoxin). The nurse should notify the practitioner and withhold the medication if the apical pulse is less than which of the following? 1. 60 2. 70 3. 90 to 110 4. 110 to 120

ANS: 3 3. If the 1-minute apical is below 90 to 110, the digoxin should not be given to a 6-month-old.

31. Which of the following is an important nursing consideration when chest tubes will be removed from a child? 1. Explain that it is not painful. 2. Explain that only a Band-Aid will be needed 3. Administer analgesics before procedure. 4. Expect bright red drainage for several hours after removal.

ANS: 3 3. It is appropriate to prepare the child for the removal of chest tubes with analgesics. Short-acting medications can be used that are administered through an existing IV line

38. Which of the following actions by the school nurse is important in the prevention of rheumatic fever? 1. Encourage routine cholesterol screenings. 2. Conduct routine blood pressure screenings. 3. Refer children with sore throats for throat cultures. 4. Recommend salicylates instead of acetaminophen for minor discomforts.

ANS: 3 3. Nurses have a role in prevention; primarily in screening school-age children for sore throats caused by group A streptococci. This can be by actively participating in throat culture screening or by referring children with possible streptococcal sore throats for testing.

14. Which of the following is a clinical manifestation of the systemic venous congestion that can occur with congestive heart failure? 1. Tachypnea 2. Tachycardia 3. Peripheral edema 4. Pale, cool extremities

ANS: 3 3. Peripheral edema, especially periorbital edema, is a clinical manifestation of systemic venous congestion

10. Which of the following defects results in increased pulmonary blood flow? 1. Pulmonic stenosis 2. Tricuspid atresia 3. Atrial septal defect 4. Transposition of the great arteries

ANS: 3 3. The atrial septal defect results in increased pulmonary blood flow. Blood flows from the left atrium (higher pressure) into the right atrium (lower pressure) and then to the lungs via the pulmonary artery.

13. Which of the following is best described as the inability of the heart to pump an adequate amount of blood to the systemic circulation at normal filling pressures? 1. Pulmonary congestion 2. Congenital heart defect 3. Congestive heart failure 4. Systemic venous congestion

ANS: 3 3. The definition of congestive heart failure is the inability of the heart to pump an adequate amount of blood to the systemic circulation at normal filling pressures to meet the metabolic demands of the body.

5. Nursing interventions for the child after a cardiac catheterization would include which of the following? 1. Allow ambulation as tolerated. 2. Monitor vital signs every 2 hours. 3. Assess the affected extremity for temperature and color. 4. Check pulses above the catheterization site for equality and symmetry.

ANS: 3 3. The extremity that was used for access for the cardiac catheterization must be checked for temperature and color. Coolness and blanching may indicate arterial occlusion.

6. After the child returns from cardiac catheterization, the nurse monitors the child's vital signs. The heart rate should be counted for how many seconds? 1. 10 2. 15 3. 30 4. 60

ANS: 4 4. After cardiac catheterization, the heart rate should be counted for a full minute to detect evidence of dysrhythmias or bradycardia.

35. Which of the following is a common, serious complication of rheumatic fever? 1. Seizures 2. Cardiac arrhythmias 3. Pulmonary hypertension 4. Cardiac valve damage

ANS: 4 4. Cardiac valve damage is the most significant complication of rheumatic fever.

20. The parents of a young child with congestive heart failure tell the nurse that they are "nervous" about giving digoxin. The nurse's response should be based on which of the following? 1. It is a very safe, frequently used drug. 2. It is difficult to either overmedicate or undermedicate with digoxin. 3. Parents lack the expertise necessary to administer digoxin. 4. Parents must learn specific, important guidelines for administration of digoxin.

ANS: 4 4. Digoxin has a narrow therapeutic range. The margin of safety between therapeutic, toxic, and lethal doses is very small. Specific guidelines are available for parents to learn how to administer the drug safely and to monitor for side effects

41. When caring for the child with Kawasaki disease, the nurse should know which of the following? 1. Child's fever is usually responsive to antibiotics within 48 hours. 2. Principal area of involvement is the joints. 3. Aspirin is contraindicated. 4. Therapeutic management includes administration of gamma globulin and aspirin.

ANS: 4 4. High-dose IV gamma globulin and aspirin therapy is indicated to reduce the incidence of coronary artery abnormalities when given within the first 10 days of the illness

7. The nurse is caring for a school-age girl who has had a cardiac catheterization. The child tells the nurse that her bandage is "too wet." The nurse finds the bandage and bed soaked with blood. The most appropriate initial nursing action is which of the following? 1. Notify physician. 2. Apply new bandage with more pressure. 3. Place in Trendelenburg position. 4. Apply direct pressure above catheterization site.

ANS: 4 4. If bleeding occurs, direct continuous pressure is applied 2.5 cm (1 inch) above the percutaneous skin site to localize pressure over the vessel puncture.

28. Seventy-two hours after cardiac surgery, a young child has a temperature of 101° F. The nurse should do which of the following? 1. Keep child warm with blankets. 2. Apply a hypothermia blanket. 3. Record temperature on nurses' notes. 4. Report findings to physician.

ANS: 4 4. In the first 24 to 48 hours after surgery, the body temperature may increase to 37.7° C or 100° F as part of the inflammatory response to tissue trauma. If the temperature is higher or continues after this period, it is most likely a sign of an infection and immediate investigation is indicated

23. Which of the following is a sign of hypokalemia? 1. Apnea 2. Oliguria 3. Twitching 4. Muscle weakness

ANS: 4 4. Muscle weakness is a characteristic clinical manifestation of hypokalemia.

32. What is the most common causative agent of bacterial endocarditis? 1. Staphylococcus albus 2. Streptococcus hemolyticus 3. Staphylococcus albicans 4. Streptococcus viridans

ANS: 4 4. Staphylococcus viridans is the most common causative agent in bacterial (infective) endocarditis

4. Which of the following explanations regarding cardiac catheterization is appropriate for a preschool child? 1. Postural drainage will be performed every 4 to 6 hours after the test. 2. It is necessary to be completely "asleep" during the test. 3. The test is very short, usually taking less than 1 hour. 4. When the procedure is done, you will have to keep your leg straight for at least 4 hours.

ANS: 4 4. The child's leg will have to be maintained in a straight position for approximately 4 hours. Younger children can be held in the parent's lap with the leg maintained in the correct position

At 1 minute after birth, the nurse assesses the infant and notes a heart rate of 80 beats/minute, some flexion of the extremities, a weak cry, grimacing, and a pink body with blue extremities. The nurse would calculate an Apgar score of: ________

ANS: 5 Each of the five signs the nurse noted would score an Apgar of 1 for a total of 5--Signs include heart rate, respiratory effort, muscle tone, reflex irritability, and color. The highest possible Apgar score is 10 PTS: 1 DIF: Cognitive Level: Application REF: 584 OBJ: Nursing Process: Assessment MSC: Client Needs: Physiologic integrity

18. As relates to the father's acceptance of the pregnancy and preparation for childbirth, the maternity nurse should know that: A. the father goes through three phases of acceptance of his own. B. the father's attachment to the fetus cannot be as strong as that of the mother because it does not start until after birth. C. in the last 2 months of pregnancy, most expectant fathers suddenly get very protective of their established lifestyle and resist making changes to the home. D. typically men remain ambivalent about fatherhood right up to the birth of their child.

ANS: A A father typically goes through three phases of development to reach acceptance of fatherhood: the announcement phase, the moratorium phase, and the focusing phase. The father-child attachment can be as strong as the mother-child relationship and can also begin during pregnancy. In the last 2 months of pregnancy, many expectant fathers work hard to improve the environment of the home for the child. Typically, the expectant father's ambivalence ends by the first trimester, and he progresses to adjusting to the reality of the situation and then to focusing on his role.

Meperidine (Demerol) is not recommended for children in sickle cell crisis because it: a. may induce seizures. b. is easily addictive. c. is not adequate for pain relief. d. is given by intramuscular injection.

ANS: A A metabolite of meperidine, normeperidine, is a central nervous system stimulant that produces anxiety, tremors, myoclonus, and generalized seizures when it accumulates with repetitive dosing. Patients with sickle cell disease are particularly at risk for normeperidine-induced seizures. Meperidine is no more addictive than other narcotic agents. Meperidine is adequate for pain relief. It is available for IV infusion.

A young child with leukemia has anorexia and severe stomatitis. The nurse should suggest that the parents try which intervention? a. Relax any eating pressures. b. Firmly insist that child eat normally. c. Begin gavage feedings to supplement diet. d. Serve foods that are either hot or cold.

ANS: A A multifaceted approach is necessary for children with severe stomatitis and anorexia. First, the parents should relax eating pressures. The nurse should suggest that the parents try soft, bland foods; normal saline or bicarbonate mouthwashes; and local anesthetics. The stomatitis is a temporary condition. The child can resume good food habits as soon as the condition resolves.

14. A woman who has completed one pregnancy with a fetus (or fetuses) reaching the stage of fetal viability is called a: a. primipara. b. primigravida. c. multipara. d. nulligravida.

ANS: A A primipara is a woman who has completed one pregnancy with a viable fetus. To remember terms, keep in mind: gravida is a pregnant woman; para comes from parity, meaning a viable fetus; primi means first; multi means many; and null means none. A primigravida is a woman pregnant for the first time. A multipara is a woman who has completed two or more pregnancies with a viable fetus. A nulligravida is a woman who has never been pregnant.

24. Before the physician performs an external version, the nurse should expect an order for a: a. tocolytic drug. b. contraction stress test (CST). c. local anesthetic. d. Foley catheter.

ANS: A A tocolytic drug will relax the uterus before and during version, thus making manipulation easier. CST is used to determine the fetal response to stress. A local anesthetic is not used with external version. The bladder should be emptied; however, catheterization is not necessary.

33. Identify the goal of a patient with the following nursing diagnosis: Imbalanced Nutrition: Less Than Body Requirements related to diet choices inadequate to meet nutrient requirements of pregnancy. A. Gain a total of 30 lbs. B. Take daily supplements consistently. C. Decrease intake of snack foods. D. Increase intake of complex carbohydrates.

ANS: A A weight gain of 30 lbs is one indication that the patient has gained a sufficient amount for the nutritional needs of pregnancy. A daily supplement is not the best goal for this patient. It does not meet the basic need of proper nutrition during pregnancy. Decreasing snack foods may be a problem and should be assessed; however, assessing weight gain is the best method of monitoring nutritional intake for this pregnant patient. Increasing the intake of complex carbohydrates is important for this patient, but monitoring the weight gain should be the end goal.

33. As the United States and Canada continue to become more culturally diverse, it is increasingly important for the nursing staff to recognize a wide range of varying cultural beliefs and practices. Nurses need to develop respect for these culturally diverse practices and learn to incorporate these into a mutually agreed on plan of care. Although it is common practice in the United States for the father of the baby to be present at the birth, in many societies this is not the case. When implementing care, the nurse would anticipate that a woman from which country would have the father of the baby in attendance? a. Mexico b. China c. Iran d. India

ANS: A A woman from Mexico may be stoic about discomfort until the second stage, at which time she will request pain relief. Fathers and female relatives are usually in attendance during the second stage of labor. The father of the baby is expected to provide encouragement, support, and reassurance that all will be well. Fathers are usually not present in China. The Iranian father will not be present. Female support persons and female care providers are preferred. For many, a male caregiver is unacceptable. The father is usually not present in India, but female relatives are usually present. Natural childbirth methods are preferred.

5. In vitro fertilization-embryo transfer (IVF-ET) is a common approach for women with blocked fallopian tubes or unexplained infertility and for men with very low sperm counts. A husband and wife have arrived for their preprocedural interview. The husband asks the nurse to explain what the procedure entails. The nurse's most appropriate response is: A. "IVF-ET is a type of assisted reproductive therapy that involves collecting eggs from your wife's ovaries, fertilizing them in the laboratory with your sperm, and transferring the embryo to her uterus." B. "A donor embryo will be transferred into your wife's uterus." C. "Donor sperm will be used to inseminate your wife." D. "Don't worry about the technical stuff; that's what we are here for."

ANS: A A woman's eggs are collected from her ovaries, fertilized in the laboratory with sperm, and transferred to her uterus after normal embryonic development has occurred. The statement, "A donor embryo will be transferred into your wife's uterus" describes therapeutic donor insemination. "Donor sperm will be used to inseminate your wife" describes the procedure for a donor embryo. "Don't worry about the technical stuff; that's what we are here for" discredits the patient's need for teaching and is an inappropriate response.

2. Nursing care measures are commonly offered to women in labor. Which nursing measure reflects application of the gate-control theory? a. Massaging the woman's back b. Changing the woman's position c. Giving the prescribed medication d. Encouraging the woman to rest between contractions

ANS: A According to the gate-control theory, pain sensations travel along sensory nerve pathways to the brain, but only a limited number of sensations, or messages, can travel through these nerve pathways at one time. Distraction techniques such as massage or stroking, music, focal points, and imagery reduce or completely block the capacity of nerve pathways to transmit pain. These distractions are thought to work by closing down a hypothetic gate in the spinal cord and thus preventing pain signals from reaching the brain. The perception of pain is thereby diminished. Changing the woman's position, giving prescribed medication, and encouraging rest do not reduce or block the capacity of nerve pathways to transmit pain using the gate-control theory.

7. A woman is undergoing a nipple-stimulated contraction stress test (CST). She is having contractions that occur every 3 minutes. The fetal heart rate (FHR) has a baseline of approximately 120 beats/min without any decelerations. The interpretation of this test is said to be: a. negative. b. positive. c. satisfactory. d. unsatisfactory.

ANS: A Adequate uterine activity necessary for a CST consists of the presence of three contractions in a 10-minute time frame. If no decelerations are observed in the FHR pattern with the contractions, the findings are considered to be negative. A positive CST indicates the presence of repetitive later FHR decelerations. Satisfactory and unsatisfactory are not applicable terms.

7. Individual irregularities in the ovarian (menstrual) cycle are most often caused by: A. variations in the follicular (preovulatory) phase. B. an intact hypothalamic-pituitary feedback mechanism. C. a functioning corpus luteum. D. a prolonged ischemic phase.

ANS: A Almost all variations in the length of the ovarian cycle are the result of variations in the length of the follicular phase. An intact hypothalamic-pituitary feedback mechanism is regular, not irregular. The luteal phase begins after ovulation. The corpus luteum depends on the ovulatory phase and fertilization. During the ischemic phase, the blood supply to the functional endometrium is blocked and necrosis develops. The functional layer separates from the basal layer, and menstrual bleeding begins.

2. A woman at 10 weeks of gestation who is seen in the prenatal clinic with presumptive signs and symptoms of pregnancy likely will have: A. amenorrhea. B. positive pregnancy test. C. Chadwick's sign. D. Hegar's sign.

ANS: A Amenorrhea is a presumptive sign of pregnancy. Presumptive signs of pregnancy are felt by the woman. A positive pregnancy test, the presence of Chadwick's sign, and the presence of Hegar's sign are all probable signs of pregnancy.

6. The nurse providing care for the laboring woman should understand that amnioinfusion is used to treat: a. variable decelerations. b. late decelerations. c. fetal bradycardia. d. fetal tachycardia.

ANS: A Amnioinfusion is used during labor either to dilute meconium-stained amniotic fluid or to supplement the amount of amniotic fluid to reduce the severity of variable decelerations caused by cord compression. Amnioinfusion has no bearing on late decelerations, fetal bradycardia, or fetal tachycardia alterations in fetal heart rate (FHR) tracings.

22. A maternity nurse should be aware of which fact about the amniotic fluid? A. It serves as a source of oral fluid and a repository for waste from the fetus. B. The volume remains about the same throughout the term of a healthy pregnancy. C. A volume of less than 300 mL is associated with gastrointestinal malformations. D. A volume of more than 2 L is associated with fetal renal abnormalities.

ANS: A Amniotic fluid serves as a source of oral fluid, serves as a repository for waste from the fetus, cushions the fetus, and helps maintain a constant body temperature. The volume of amniotic fluid changes constantly. Too little amniotic fluid (oligohydramnios) is associated with renal abnormalities. Too much amniotic fluid (hydramnios) is associated with gastrointestinal and other abnormalities.

11. Nurses should be aware that the biophysical profile (BPP): A. is an accurate indicator of impending fetal death. B. is a compilation of health risk factors of the mother during the later stages of pregnancy. C. consists of a Doppler blood flow analysis and an amniotic fluid index. D. involves an invasive form of ultrasound examination.

ANS: A An abnormal BPP score is an indication that labor should be induced. The BPP evaluates the health of the fetus, requires many different measures, and is a noninvasive procedure.

6. A patient asks her nurse, "My doctor told me that he is concerned with the grade of my placenta because I am overdue. What does that mean?" The best response by the nurse is: A. "Your placenta changes as your pregnancy progresses, and it is given a score that indicates the amount of calcium deposits it has. The more calcium deposits, the higher the grade, or number, that is assigned to the placenta. It also means that less blood and oxygen can be delivered to your baby." B. "Your placenta isn't working properly, and your baby is in danger." C. "This means that we will need to perform an amniocentesis to detect if you have any placental damage." D. "Don't worry about it. Everything is fine."

ANS: A An accurate and appropriate response is, "Your placenta changes as your pregnancy progresses, and it is given a score that indicates the amount of calcium deposits it has. The more calcium deposits, the higher the grade, or number, that is assigned to the placenta. It also means that less blood and oxygen can be delivered to your baby." Although "Your placenta isn't working properly, and your baby is in danger" may be valid, it does not reflect therapeutic communication techniques and is likely to alarm the patient. An ultrasound, not an amniocentesis, is the method of assessment used to determine placental maturation. The response "Don't worry about it. Everything is fine" is not appropriate and discredits the patient's concerns.

25. The nurse providing newborn stabilization must be aware that the primary side effect of maternal narcotic analgesia in the newborn is: a. respiratory depression. b. bradycardia. c. acrocyanosis. d. tachypnea.

ANS: A An infant delivered within 1 to 4 hours of maternal analgesic administration is at risk for respiratory depression from the sedative effects of the narcotic. Bradycardia is not the anticipated side effect of maternal analgesics. Acrocyanosis is an expected finding in a newborn and is not related to maternal analgesics. The infant who is having a side effect to maternal analgesics normally would have a decrease in respirations, not an increase.

2. A 39-year-old primigravida thinks that she is about 8 weeks pregnant, although she has had irregular menstrual periods all her life. She has a history of smoking approximately one pack of cigarettes a day, but she tells you that she is trying to cut down. Her laboratory data are within normal limits. What diagnostic technique could be used with this pregnant woman at this time? A. Ultrasound examination B. Maternal serum alpha-fetoprotein (MSAFP) screening C. Amniocentesis D. Nonstress test (NST)

ANS: A An ultrasound examination could be done to confirm the pregnancy and determine the gestational age of the fetus. It is too early in the pregnancy to perform MSAFP screening, amniocentesis, or NST. MSAFP screening is performed at 16 to 18 weeks of gestation, followed by amniocentesis if MSAFP levels are abnormal or if fetal/maternal anomalies are detected. NST is performed to assess fetal well-being in the third trimester.

14. As relates to dysfunctional uterine bleeding (DUB), the nurse should be aware that: A. it is most commonly caused by anovulation. B. it most often occurs in middle age. C. the diagnosis of DUB should be first considered for abnormal menstrual bleeding. D. the most effective medical treatment is steroids.

ANS: A Anovulation may occur because of hypothalamic dysfunction or polycystic ovary syndrome. DUB most often occurs when the menstrual cycle is being established or when it draws to a close at menopause. A diagnosis of DUB is made only after all other causes of abnormal menstrual bleeding have been ruled out. The most effective medical treatment is oral or intravenous estrogen.

4. A woman in preterm labor at 30 weeks of gestation receives two 12-mg doses of betamethasone intramuscularly. The purpose of this pharmacologic treatment is to: a. stimulate fetal surfactant production. b. reduce maternal and fetal tachycardia associated with ritodrine administration. c. suppress uterine contractions. d. maintain adequate maternal respiratory effort and ventilation during magnesium sulfate therapy.

ANS: A Antenatal glucocorticoids given as intramuscular injections to the mother accelerate fetal lung maturity. Inderal would be given to reduce the effects of ritodrine administration. Betamethasone has no effect on uterine contractions. Calcium gluconate would be given to reverse the respiratory depressive effects of magnesium sulfate therapy.

In which of the conditions are all the formed elements of the blood simultaneously depressed? a. Aplastic anemia b. Sickle cell anemia c. Thalassemia major d. Iron deficiency anemia

ANS: A Aplastic anemia refers to a bone marrowfailure condition in which the formed elements of the blood are simultaneously depressed. Sickle cell anemia is a hemoglobinopathy in which normal adult hemoglobin is partly or completely replaced by abnormal sickle hemoglobin. Thalassemia major is a group of blood disorders characterized by deficiency in the production rate of specific hemoglobin globin chains. Iron deficiency anemia results in a decreased amount of circulating red cells.

5. During a patient's physical examination the nurse notes that the lower uterine segment is soft on palpation. The nurse would document this finding as: A. Hegar's sign. B. McDonald's sign. C. Chadwick's sign. D. Goodell's sign.

ANS: A At approximately 6 weeks of gestation, softening and compressibility of the lower uterine segment occurs; this is called Hegar's sign. McDonald's sign indicates a fast food restaurant. Chadwick's sign is the blue-violet coloring of the cervix caused by increased vascularity; this occurs around the fourth week of gestation. Softening of the cervical tip is called Goodell's sign, which may be observed around the sixth week of pregnancy.

20. With regard to the process of augmentation of labor, the nurse should be aware that it: a. is part of the active management of labor that is instituted when the labor process is unsatisfactory. b. relies on more invasive methods when oxytocin and amniotomy have failed. c. is a modern management term to cover up the negative connotations of forceps-assisted birth. d. uses vacuum cups.

ANS: A Augmentation is part of the active management of labor that stimulates uterine contractions after labor has started but is not progressing satisfactorily. Augmentation uses amniotomy and oxytocin infusion, as well as some gentler, noninvasive methods. Forceps-assisted births and vacuum-assisted births are appropriately used at the end of labor and are not part of augmentation.

The nurse is planning care for an adolescent with AIDS. Which is the priority nursing goal? a. Preventing infection b. Preventing secondary cancers c. Restoring immunologic defenses d. Identifying source of infection

ANS: A Because the child is immunocompromised in association with HIV infection, the prevention of infection is paramount. Although certain precautions are justified in limiting exposure to infection, these must be balanced with the concern for the childs normal developmental needs. Preventing secondary cancers is not currently possible. Current drug therapy is affecting the disease progression; although not a cure, these drugs can suppress viral replication, preventing further deterioration. Case finding is not a priority nursing goal.

24. With regard to medications, herbs, shots, and other substances normally encountered by pregnant women, the maternity nurse should be aware that: A. both prescription and over-the-counter (OTC) drugs that otherwise are harmless can be made hazardous by metabolic deficiencies of the fetus. B. the greatest danger of drug-caused developmental deficits in the fetus is seen in the final trimester. C. killed-virus vaccines (e.g., tetanus) should not be given during pregnancy, but live-virus vaccines (e.g., measles) are permissible. D. no convincing evidence exists that secondhand smoke is potentially dangerous to the fetus.

ANS: A Both prescription and OTC drugs that otherwise are harmless can be made hazardous by metabolic deficiencies of the fetus. This is especially true for new medications and combinations of drugs. The greatest danger of drug-caused developmental defects exists in the interval from fertilization through the first trimester, when a woman may not realize that she is pregnant. Live-virus vaccines should be part of after birth care; killed-virus vaccines may be administered during pregnancy. Secondhand smoke is associated with fetal growth restriction and increases in infant mortality.

21. As related to the care of the patient with anemia, the nurse should be aware that: A. it is the most common medical disorder of pregnancy. B. it can trigger reflex brachycardia. C. the most common form of anemia is caused by folate deficiency. D. thalassemia is a European version of sickle cell anemia.

ANS: A Combined with any other complication, anemia can result in congestive heart failure. Reflex bradycardia is a slowing of the heart in response to the blood flow increases immediately after birth. The most common form of anemia is iron deficiency anemia. Both thalassemia and sickle cell hemoglobinopathy are hereditary but not directly related or confined to geographic areas.

20. With regard to nerve block analgesia and anesthesia, nurses should be aware that: a. most local agents are related chemically to cocaine and end in the suffix -caine. b. local perineal infiltration anesthesia is effective when epinephrine is added, but it can be injected only once. c. a pudendal nerve block is designed to relieve the pain from uterine contractions. d. a pudendal nerve block, if done correctly, does not significantly lessen the bearing-down reflex.

ANS: A Common agents include lidocaine and chloroprocaine. Injections can be repeated to prolong the anesthesia. A pudendal nerve block relieves pain in the vagina, vulva, and perineum but not the pain from uterine contractions, and it lessens or shuts down the bearing-down reflex.

28. When the pregnant diabetic woman experiences hypoglycemia while hospitalized, the nurse should intervene by having the patient: A. eat six saltine crackers. B. drink 8 ounces of orange juice with 2 tsp of sugar added. C. drink 4 ounces of orange juice followed by 8 ounces of milk. D. eat hard candy or commercial glucose wafers.

ANS: A Crackers provide carbohydrates in the form of polysaccharides. Orange juice and sugar will increase the blood sugar but not provide a slow-burning carbohydrate to sustain the blood sugar. Milk is a disaccharide and orange juice is a monosaccharide. They will provide an increase in blood sugar but will not sustain the level. Hard candy or commercial glucose wafers provide only monosaccharides.

15. Which statement regarding acronyms in nutrition is accurate? A. Dietary reference intakes (DRIs) consist of recommended dietary allowances (RDAs), adequate intakes (AIs), and upper limits (ULs). B. RDAs are the same as ULs except with better data. C. AIs offer guidelines for avoiding excessive amounts of nutrients. D. They all refer to green leafy vegetables, whole grains, and fruit.

ANS: A DRIs consist of RDAs, AIs, and ULs. AIs are similar to RDAs except that they deal with nutrients about which data are insufficient for certainty (RDA status). ULs are guidelines for avoiding excesses of nutrients for which excess is toxic. Green leafy vegetables, whole grains, and fruit are important, but they are not the whole nutritional story.

16. Which statement would indicate that the patient requires additional instruction about breast self-examination? A. "Yellow discharge from my nipple is normal if I am having my period." B. "I should check my breasts at the same time each month, like after my period." C. "I should also feel in my armpit area while performing my breast examination." D. "I should check each breast in a set way, such as in a circular motion."

ANS: A Discharge from the nipples requires further examination from a health care provider. "I should check my breasts at the same time each month, like after my period," "I should also feel in my armpit area while performing my breast examination," and "I should check each breast in a set way, such as in a circular motion" all indicate successful learning.

36. Fibrocystic changes in the breast most often appear in women in their 20s and 30s. The etiology is unknown but it may be an imbalance of estrogen and progesterone. The nurse who cares for this patient should be aware that treatment modalities are conservative. One proven modality that may provide relief is: A. diuretic administration. B. including caffeine daily in the diet. C. increased vitamin C supplementation. D. application of cold packs to the breast as necessary.

ANS: A Diuretic administration plus a decrease in sodium and fluid intake are recommended. Although not supported by research, some advocate eliminating dimethylxanthines (caffeine) from the diet. Smoking should also be avoided, and alcohol consumption should be reduced. Vitamin E supplements are recommended; however, the patient should avoid megadoses because this is a fat-soluble vitamin. Pain-relief measures include applying heat to the breast, wearing a supportive bra, and taking nonsteroidal anti-inflammatory drugs.

29. The placenta allows exchange of oxygen, nutrients, and waste products between the mother and fetus by: A. contact between maternal blood and fetal capillaries within the chorionic villi. B. interaction of maternal and fetal pH levels within the endometrial vessels. C. a mixture of maternal and fetal blood within the intervillous spaces. D. passive diffusion of maternal carbon dioxide and oxygen into the fetal capillaries.

ANS: A Fetal capillaries within the chorionic villi are bathed with oxygen-rich and nutrient-rich maternal blood within the intervillous spaces. The endometrial vessels are part of the uterus. There is no interaction with the fetal blood at this point. Maternal and fetal bloods do not normally mix. Maternal carbon dioxide does not enter into the fetal circulation.

3. The nurse sees a woman for the first time when she is 30 weeks pregnant. The woman has smoked throughout the pregnancy, and fundal height measurements now are suggestive of growth restriction in the fetus. In addition to ultrasound to measure fetal size, what other tool would be useful in confirming the diagnosis? A. Doppler blood flow analysis B. Contraction stress test (CST) C. Amniocentesis D. Daily fetal movement counts

ANS: A Doppler blood flow analysis allows the examiner to study the blood flow noninvasively in the fetus and the placenta. It is a helpful tool in the management of high risk pregnancies because of intrauterine growth restriction (IUGR), diabetes mellitus, multiple fetuses, or preterm labor. Because of the potential risk of inducing labor and causing fetal distress, CST is not performed on a woman whose fetus is preterm. Indications for amniocentesis include diagnosis of genetic disorders or congenital anomalies, assessment of pulmonary maturity, and diagnosis of fetal hemolytic disease, not IUGR. Fetal kick count monitoring is performed to monitor the fetus in pregnancies complicated by conditions that may affect fetal oxygenation. Although this may be a useful tool at some point later in this woman's pregnancy, it is not used to diagnose IUGR.

A possible cause of acquired aplastic anemia in children is: a. drugs. b. injury. c. deficient diet. d. congenital defect.

ANS: A Drugs, such as chemotherapeutic agents and several antibiotics (e.g., chloramphenicol), can cause aplastic anemia. Injury, deficient diet, and congenital defect are not causative agents in acquired aplastic anemia.

8. The nurse expects which maternal cardiovascular finding during labor? a. Increased cardiac output b. Decreased pulse rate c. Decreased white blood cell (WBC) count d. Decreased blood pressure

ANS: A During each contraction, 400 ml of blood is emptied from the uterus into the maternal vascular system, which increases cardiac output by approximately 10% to 15% during the first stage of labor and by approximately 30% to 50% in the second stage of labor. The heart rate increases slightly during labor. The WBC count can increase during labor. During the first stage of labor, uterine contractions cause systolic readings to increase by approximately 10 mm Hg. During the second stage, contractions may cause systolic pressures to increase by 30 mm Hg and diastolic readings to increase by 25 mm Hg.

26. Which deceleration of the fetal heart rate would not require the nurse to change the maternal position? a. Early decelerations b. Late decelerations c. Variable decelerations d. It is always a good idea to change the woman's position.

ANS: A Early decelerations (and accelerations) generally do not need any nursing intervention. Late decelerations suggest that the nurse should change the maternal position (lateral); variable decelerations also require a maternal position change (side to side). Although changing positions throughout labor is recommended, it is not required in response to early decelerations.

2. The nurse caring for the laboring woman should understand that early decelerations are caused by: a. altered fetal cerebral blood flow. b. umbilical cord compression. c. uteroplacental insufficiency. d. spontaneous rupture of membranes.

ANS: A Early decelerations are the fetus's response to fetal head compression. Variable decelerations are associated with umbilical cord compression. Late decelerations are associated with uteroplacental insufficiency. Spontaneous rupture of membranes has no bearing on the fetal heart rate unless the umbilical cord prolapses, which would result in variable or prolonged bradycardia.

17. Which nutritional recommendation about fluids is accurate? A. A woman's daily intake should be 8 to 10 glasses (2.3 L) of water, milk, or juice. B. Coffee should be limited to no more than two cups, but tea and cocoa can be consumed without worry. C. Of the artificial sweeteners, only aspartame has been not associated with any maternity health concerns. D. Water with fluoride is especially encouraged because it reduces the child's risk of tooth decay.

ANS: A Eight to ten glasses is the standard for fluids; however, they should be the right fluids. All beverages containing caffeine, including tea, cocoa, and some soft drinks should be avoided or drunk only in limited amounts. Artificial sweeteners, including aspartame, have no ill effects on the normal mother or fetus; however, mothers with phenylketonuria should avoid aspartame. No evidence indicates that prenatal fluoride consumption reduces childhood tooth decay.

10. A woman is in her seventh month of pregnancy. She has been reporting nasal congestion and occasional epistaxis. The nurse suspects that: A. this is a normal respiratory change in pregnancy caused by elevated levels of estrogen. B. this is an abnormal cardiovascular change, and the nosebleeds are an ominous sign. C. the woman is a victim of domestic violence and is being hit in the face by her partner. D. the woman has been using cocaine intranasally.

ANS: A Elevated levels of estrogen cause capillaries to become engorged in the respiratory tract. This may result in edema in the nose, larynx, trachea, and bronchi. This congestion may cause nasal stuffiness and epistaxis. Cardiovascular changes in pregnancy may cause edema in lower extremities. Determining that the woman is a victim of domestic violence and was hit in the face cannot be made on the basis of the sparse facts provided. If the woman had been hit in the face, she most likely would have additional physical findings. Determination of the use of cocaine by the woman cannot be made on the basis of the sparse facts provided.

27. Postcoital contraception with Ella: A. requires that the first dose be taken within 72 hours of unprotected intercourse. B. requires that the woman take second and third doses at 24 and 36 hours after the first dose. C. must be taken in conjunction with an IUD insertion. D. is commonly associated with the side effect of menorrhagia.

ANS: A Emergency contraception is most effective when used within 72 hours of intercourse; however, it may be used with lessened effectiveness 120 hours later. Insertion of the copper IUD within 5 days of intercourse may also be used and is up to 99% effective. The most common side effect of postcoital contraception is nausea.

3. The nurse providing care for the laboring woman comprehends that accelerations with fetal movement: a. are reassuring. b. are caused by umbilical cord compression. c. warrant close observation. d. are caused by uteroplacental insufficiency.

ANS: A Episodic accelerations in the fetal heart rate (FHR) occur during fetal movement and are indications of fetal well-being. Umbilical cord compression results in variable decelerations in the FHR. Accelerations in the FHR are an indication of fetal well-being and do not warrant close observation. Uteroplacental insufficiency would result in late decelerations in the FHR.

12. Fetal well-being during labor is assessed by: A. the response of the fetal heart rate (FHR) to uterine contractions (UCs). B. maternal pain control. C. accelerations in the FHR. D. an FHR above 110 beats/min.

ANS: A Fetal well-being during labor can be measured by the response of the FHR to UCs. In general, reassuring FHR patterns are characterized by an FHR baseline in the range of 110 to 160 beats/min with no periodic changes, a moderate baseline variability, and accelerations with fetal movement. Maternal pain control is not the measure used to determine fetal well-being in labor. Although FHR accelerations are a reassuring pattern, they are only one component of the criteria by which fetal well-being is assessed. Although an FHR above 110 beats/min may be reassuring, it is only one component of the criteria by which fetal well-being is assessed. More information would be needed to determine fetal well-being.

17. With regard to breathing techniques during labor, maternity nurses should understand that: a. breathing techniques in the first stage of labor are designed to increase the size of the abdominal cavity to reduce friction. b. by the time labor has begun, it is too late for instruction in breathing and relaxation. c. controlled breathing techniques are most difficult near the end of the second stage of labor. d. the patterned-paced breathing technique can help prevent hyperventilation.

ANS: A First-stage techniques promote relaxation of abdominal muscles, thereby increasing the size of the abdominal cavity. Instruction in simple breathing and relaxation techniques early in labor is possible and effective. Controlled breathing techniques are most difficult in the transition phase at the end of the first stage of labor when the cervix is dilated 8 to 10 cm. Patterned-paced breathing sometimes can lead to hyperventilation.

14. Which description of the four stages of labor is correct for both the definition and the duration? a. First stage: onset of regular uterine contractions to full dilation; less than 1 hour to 20 hours b. Second stage: full effacement to 4 to 5 cm; visible presenting part; 1 to 2 hours c. Third stage: active pushing to birth; 20 minutes (multiparous woman), 50 minutes (nulliparous woman) d. Fourth stage: delivery of the placenta to recovery; 30 minutes to 1 hour

ANS: A Full dilation may occur in less than 1 hour, but in first-time pregnancies full dilation can take up to 20 hours. The second stage of labor extends from full dilation to birth and takes an average of 20 to 50 minutes, although 2 hours is still considered normal. The third stage of labor extends from birth to the expulsion of the placenta and usually takes a few minutes. The fourth stage begins after the expulsion of the placenta and lasts until homeostasis is reestablished (approximately 2 hours).

16. With regard to protein in the diet of pregnant women, nurses should be aware that: A. many protein-rich foods are also good sources of calcium, iron, and B vitamins. B. many women need to increase their protein intake during pregnancy. C. as with carbohydrates and fat, no specific recommendations exist for the amount of protein in the diet. D. high-protein supplements can be used without risk by women on macrobiotic diets.

ANS: A Good protein sources such as meat, milk, eggs, and cheese have a lot of calcium and iron. Most women already eat a high-protein diet and do not need to increase their intake. Protein is sufficiently important that specific servings of meat and dairy are recommended. High-protein supplements are not recommended because they have been associated with an increased incidence of preterm births.

The nurse is conducting a staff in-service on common problems associated with myelomeningocele. Which common problem is associated with this defect? a. Hydrocephalus b. Craniosynostosis c. Biliary atresia d. Esophageal atresia

ANS: A Hydrocephalus is a frequently associated anomaly in 80% to 90% of children. Craniosynostosis is the premature closing of the cranial sutures and is not associated with myelomeningocele. Biliary and esophageal atresia is not associated with myelomeningocele.

When a preschool child is hospitalized without adequate preparation, the nurse should recognize that the child may likely see hospitalization as: a. Punishment. b. Threat to childs self-image. c. An opportunity for regression. d. Loss of companionship with friends

ANS: A If a toddler is not prepared for hospitalization, a typical preschooler fantasy is to attribute the hospitalization to punishment for real or imagined misdeeds. Threat to childs self-image and loss of companionship with friends are reactions typical of school-age children. Regression is a response characteristic of toddlers when threatened with loss of control.

The nurse is administering an IV chemotherapeutic agent to a child with leukemia. The child suddenly begins to wheeze and have severe urticaria. Which is the most appropriate nursing action? a. Stop drug infusion immediately. b. Recheck rate of drug infusion. c. Observe child closely for next 10 minutes. d. Explain to child that this is an expected side effect.

ANS: A If an allergic reaction is suspected, the drug should be immediately discontinued. Any drug in the line should be withdrawn, and a normal saline infusion begun to keep the line open. Rechecking the rate of drug infusion, observing the child closely for next 10 minutes, and explaining to the child that this is an expected side effect can all be done after the drug infusion is stopped and the child is evaluated.

5. A pregnant woman reports that she is still playing tennis at 32 weeks of gestation. The nurse would be most concerned that during and after tennis matches this woman consumes: A. several glasses of fluid. B. extra protein sources such as peanut butter. C. salty foods to replace lost sodium. D. easily digested sources of carbohydrate.

ANS: A If no medical or obstetric problems contraindicate physical activity, pregnant women should get 30 minutes of moderate physical exercise daily. Liberal amounts of fluid should be consumed before, during, and after exercise because dehydration can trigger premature labor. The woman's calorie intake should be sufficient to meet the increased needs of pregnancy and the demands of exercise.

25. Many parents-to-be have questions about multiple births. Maternity nurses should be able to tell them that: A. twinning and other multiple births are increasing because of the use of fertility drugs and delayed childbearing. B. dizygotic twins (two fertilized ova) have the potential to be conjoined twins. C. identical twins are more common in white families. D. fraternal twins are same gender, usually male.

ANS: A If the parents-to-be are older and have taken fertility drugs, they would be very interested to know about twinning and other multiple births. Conjoined twins are monozygotic; they are from a single fertilized ovum in which division occurred very late. Identical twins show no racial or ethnic preference; fraternal twins are more common among African-American women. Fraternal twins can be different genders or the same gender. Identical twins are the same gender.

10. Which presentation is accurately described in terms of both the resenting part and the frequency of occurrence? a. Cephalic: occiput, at least 96% b. Breech: sacrum, 10% to 15% c. Shoulder: scapula, 10% to 15% d. Cephalic: cranial, 80% to 85%

ANS: A In cephalic presentations (head first), the presenting part is the occiput; this presentation occurs in 96% of births. In a breech birth, the sacrum emerges first; this presentation occurs in approximately 3% of births. In shoulder presentations, the scapula emerges first; this presentation occurs in only 1% of births. In a cephalic presentation, the part of the head or cranium that emerges first is the occiput; cephalic presentations occur in 96% of births.

30. Which maternal condition is considered a contraindication for the application of internal monitoring devices? a. Unruptured membranes b. Cervix dilated to 4 cm c. External monitors in current use d. Fetus with a known heart defect

ANS: A In order to apply internal monitoring devices, the membranes must be ruptured. Cervical dilation of 4 cm permits the insertion of fetal scalp electrodes and intrauterine catheter. The external monitor can be discontinued after the internal ones are applied. A compromised fetus should be monitored with the most accurate monitoring devices.

A. 1. In assessing the knowledge of a pregestational woman with type 1 diabetes concerning changing insulin needs during pregnancy, the nurse recognizes that further teaching is warranted when the patient states: A. "I will need to increase my insulin dosage during the first 3 months of pregnancy." B. "Insulin dosage will likely need to be increased during the second and third trimesters." C. "Episodes of hypoglycemia are more likely to occur during the first 3 months." D. "Insulin needs should return to normal within 7 to 10 days after birth if I am bottle-feeding."

ANS: A Insulin needs are reduced in the first trimester because of increased insulin production by the pancreas and increased peripheral sensitivity to insulin. "Insulin dosage will likely need to be increased during the second and third trimesters," "Episodes of hypoglycemia are more likely to occur during the first 3 months," and "Insulin needs should return to normal within 7 to 10 days after birth if I am bottle-feeding" are accurate statements and signify that the woman has understood the teachings regarding control of her diabetes during pregnancy.

1. While evaluating an external monitor tracing of a woman in active labor, the nurse notes that the fetal heart rate (FHR) for five sequential contractions begins to decelerate late in the contraction, with the nadir of the decelerations occurring afte r the peak of the contraction. The nurse's first priority is to: a. change the woman's position. b. notify the care provider. c. assist with amnioinfusion. d. insert a scalp electrode.

ANS: A Late decelerations may be caused by maternal supine hypotension syndrome. They usually are corrected when the woman turns on her side to displace the weight of the gravid uterus from the vena cava. If the fetus does not respond to primary nursing interventions for late decelerations, the nurse would continue with subsequent intrauterine resuscitation measures, including notifying the care provider. An amnioinfusion may be used to relieve pressure on an umbilical cord that has not prolapsed. The FHR pattern associated with this situation most likely reveals variable deceleration. A fetal scalp electrode would provide accurate data for evaluating the well-being of the fetus; however, this is not a nursing intervention that would alleviate late decelerations, nor is it the nurse's first priority.

22. Leopold maneuvers would be an inappropriate method of assessment to determine: a. gender of the fetus. b. number of fetuses. c. fetal lie and attitude. d. degree of the presenting part's descent into the pelvis.

ANS: A Leopold maneuvers help identify the number of fetuses, the fetal lie and attitude, and the degree of descent of the presenting part into the pelvis. The gender of the fetus is not a goal of the examination at this time.

16. Which analysis of maternal serum may predict chromosomal abnormalities in the fetus? A. Multiple-marker screening B. Lecithin/sphingomyelin [L/S] ratio C. Biophysical profile D. Type and crossmatch of maternal and fetal serum

ANS: A Maternal serum can be analyzed for abnormal levels of alpha-fetoprotein, human chorionic gonadotropin, and estriol. The multiple-marker screening may predict chromosomal defects in the fetus. The L/S ratio is used to determine fetal lung maturity. A biophysical profile is used for evaluating fetal status during the antepartum period. Five variables are used, but none is concerned with chromosomal problems. The blood type and crossmatch would not predict chromosomal defects in the fetus.

8. With regard to prenatal genetic testing, nurses should be aware that: A. maternal serum screening can determine whether a pregnant woman is at risk of carrying a fetus with Down syndrome. B. carrier screening tests look for gene mutations of people already showing symptoms of a disease. C. predisposition testing predicts with near certainty that symptoms will appear. D. presymptomatic testing is used to predict the likelihood of breast cancer.

ANS: A Maternal serum screening identifies the risk for the neural tube defect and the specific chromosome abnormality involved in Down syndrome. Carriers of some diseases, such as sickle cell disease, do not display symptoms. Predisposition testing determines susceptibility, such as for breast cancer. presymptomatic testing indicates that symptoms are certain to appear if the gene is present.

8. The nurse caring for a laboring woman is aware that maternal cardiac output can be increased by: a. change in position. b. oxytocin administration. c. regional anesthesia. d. intravenous analgesic.

ANS: A Maternal supine hypotension syndrome is caused by the weight and pressure of the gravid uterus on the ascending vena cava when the woman is in a supine position. This reduces venous return to the woman's heart, as well as cardiac output, and subsequently reduces her blood pressure. The nurse can encourage the woman to change positions and avoid the supine position. Oxytocin administration, regional anesthesia, and intravenous analgesic may reduce maternal cardiac output.

23. With one exception, the safest pregnancy is one in which the woman is drug and alcohol free. For women addicted to opioids, ________________________ treatment is the current standard of care during pregnancy. A. methadone maintenance B. detoxification C. smoking cessation D. 4 Ps Plus

ANS: A Methadone maintenance treatment (MMT) is currently considered the standard of care for pregnant women who are dependent on heroin or other narcotics. Buprenorphine is another medication approved for opioid addiction treatment that is increasingly being used during pregnancy. Opioid replacement therapy has been shown to decrease opioid and other drug use, reduce criminal activity, improve individual functioning, and decrease rates of infections such as hepatitis B and C, HIV, and other sexually transmitted infections. Detoxification is the treatment used for alcohol addiction. Pregnant women requiring withdrawal from alcohol should be admitted for inpatient management. Women are more likely to stop smoking during pregnancy than at any other time in their lives. A smoking cessation program can assist in achieving this goal. The 4 Ps Plus is a screening tool designed specifically to identify pregnant women who need in-depth assessment related to substance abuse.

23. On vaginal examination of a 30-year-old woman, the nurse documents the following findings: profuse, thin, grayish white vaginal discharge with a "fishy" odor; complaint of pruritus. On the basis of these findings, the nurse suspects that this woman has: A. bacterial vaginosis (BV). B. candidiasis. C. trichomoniasis. D. gonorrhea.

ANS: A Most women with BV complain of a characteristic "fishy" odor. The discharge usually is profuse; thin; and white, gray, or milky in color. Some women also may have mild irritation or pruritus. The discharge associated with candidiasis is thick, white, and lumpy and resembles cottage cheese. Trichomoniasis may be asymptomatic, but women commonly have a characteristic yellowish-to-greenish, frothy, mucopurulent, copious, and malodorous discharge. Women with gonorrhea are often asymptomatic. They may have a purulent endocervical discharge, but discharge usually is minimal or absent.

The nurse is teaching a group of nursing students about newborns born with the congenital defect of myelomeningocele. Which common problem is associated with this defect? a. Neurogenic bladder b. Mental retardation c. Respiratory compromise d. Cranioschisis

ANS: A Myelomeningocele is one of the most common causes of neuropathic (neurogenic) bladder dysfunction among children. Risk of mental retardation is minimized through early intervention and management of hydrocephalus. Respiratory compromise is not a common problem in myelomeningocele. Cranioschisis is a skull defect through which various tissues protrude. It is not associated with myelomeningocele.

15. Perinatal nurses are legally responsible for: a. correctly interpreting fetal heart rate (FHR) patterns, initiating appropriate nursing interventions, and documenting the outcomes. b. greeting the patient on arrival, assessing her, and starting an intravenous line. c. applying the external fetal monitor and notifying the care provider. d. making sure that the woman is comfortable.

ANS: A Nurses who care for women during childbirth are legally responsible for correctly interpreting FHR patterns, initiating appropriate nursing interventions based on those patterns, and documenting the outcomes of those interventions. Greeting the patient, assessing her, and starting an IV; applying the external fetal monitor and notifying the care provider; and making sure the woman is comfortable may be activities that a nurse performs, but they are not activities for which the nurse is legally responsible.

26. The nurse caring for a pregnant patient knows that her health teaching regarding fetal circulation has been effective when the patient reports that she has been sleeping: A. in a side-lying position. B. on her back with a pillow under her knees. C. with the head of the bed elevated. D. on her abdomen.

ANS: A Optimal circulation is achieved when the woman is lying at rest on her side. Decreased uterine circulation may lead to intrauterine growth restriction. Previously it was believed that the left lateral position promoted maternal cardiac output, enhancing blood flow to the fetus. However, it is now known that the side-lying position enhances uteroplacental blood flow. If a woman lies on her back with the pressure of the uterus compressing the vena cava, blood return to the right atrium is diminished. Although having the head of the bed elevated is recommended and ideal for later in pregnancy, the woman still must maintain a lateral tilt to the pelvis to avoid compression of the vena cava. Many women find lying on her abdomen uncomfortable as pregnancy advances. Side-lying is the ideal position to promote blood flow to the fetus.

14. When assessing a multiparous woman who has just given birth to an 8-lb boy, the nurse notes that the woman's fundus is firm and has become globular in shape. A gush of dark red blood comes from her vagina. The nurse concludes that: a. the placenta has separated. b. a cervical tear occurred during the birth. c. the woman is beginning to hemorrhage. d. clots have formed in the upper uterine segment.

ANS: A Placental separation is indicated by a firmly contracting uterus, a change in the uterus from a discoid to a globular ovoid shape, a sudden gush of dark red blood from the introitus, an apparent lengthening of the umbilical cord, and a finding of vaginal fullness. Cervical tears that do not extend to the vagina result in minimal blood loss. Signs of hemorrhage are a boggy uterus, bright red vaginal bleeding, alterations in vital signs, pallor, lightheadedness, restlessness, decreased urinary output, and alter ation in the level of consciousness. If clots have formed in the upper uterine segment, the nurse would expect to find the uterus boggy and displaced to the side.

When both parents have sickle cell trait, which is the chance their children will have sickle cell anemia? a. 25% b. 50% c. 75% d. 100%

ANS: A Sickle cell anemia is inherited in an autosomal recessive pattern. If both parents have sickle cell trait (one copy of the sickle cell gene), then for each pregnancy, a 25% chance exists that their child will be affected with sickle cell disease. With each pregnancy, a 50% chance exists that the child will have sickle cell trait. Percentages of 75% and 100% are too high for the children of parents who have sickle cell trait.

28. A new mother asks the nurse about the "white substance" covering her infant. The nurse explains that the purpose of vernix caseosa is to: A. protect the fetal skin from amniotic fluid. B. promote normal peripheral nervous system development. C. allow transport of oxygen and nutrients across the amnion. D. regulate fetal temperature.

ANS: A Prolonged exposure to amniotic fluid during the fetal period could result in breakdown of the skin without the protection of the vernix caseosa. Normal development of the peripheral nervous system is dependent on nutritional intake of the mother. The amnion is the inner membrane that surrounds the fetus. It is not involved in the oxygen and nutrient exchange. The amniotic fluid aids in maintaining fetal temperature.

13. After you complete your nutritional counseling for a pregnant woman, you ask her to repeat your instructions so you can assess her understanding of the instructions given. Which statement indicates that she understands the role of protein in her pregnancy? A. "Protein will help my baby grow." B. "Eating protein will prevent me from becoming anemic." C. "Eating protein will make my baby have strong teeth after he/she is born." D. "Eating protein will prevent me from being diabetic."

ANS: A Protein is the nutritional element basic to growth. An adequate protein intake is essential to meeting the increasing demands of pregnancy. These demands arise from the rapid growth of the fetus; the enlargement of the uterus, mammary glands, and placenta; the increase in the maternal blood volume; and the formation of amniotic fluid. Iron intake prevents anemia. Calcium intake is needed for fetal bone and tooth development. Glycemic control is needed in diabetics; protein is one nutritional factor to consider, but this is not the primary role of protein intake.

A 4-year-old child has just been diagnosed with pseudohypertrophic (Duchenne) muscular dystrophy. The management plan should include which action? a. Recommend genetic counseling. b. Explain that the disease is easily treated. c. Suggest ways to limit use of muscles. d. Assist family in finding a nursing facility to provide his care.

ANS: A Pseudohypertrophic (Duchenne) muscular dystrophy is inherited as an X-linked recessive gene. Genetic counseling is recommended for parents, female siblings, maternal aunts, and their female offspring. No effective treatment exists at this time for childhood muscular dystrophy. Maintaining optimal function of all muscles for as long as possible is the primary goal. It has been found that children who remain as active as possible are able to avoid wheelchair confinement for a longer time. Assisting the family in finding a nursing facility to provide his care is inappropriate at the time of diagnosis. When the child becomes increasingly incapacitated, the family may consider home-based care, a skilled nursing facility, or respite care to provide the necessary care.

A previously potty-trained 30-month-old child has reverted to wearing diapers while hospitalized. The nurse should reassure the parents that this is normal because: a. Regression is seen during hospitalization. b. Developmental delays occur because of the hospitalization. c. The child is experiencing urinary urgency because of hospitalization. d. The child was too young to be potty-trained.

ANS: A Regression is expected and normal for all age-groups when hospitalized. Nurses should assure the parents this is temporary and the child will return to the previously mastered developmental milestone when back home. This does not indicate a developmental delay. The child should not be experiencing urinary urgency because of hospitalization and this would not be normal. Successful potty-training can be started at 2 years of age if the child is ready.

33. Despite warnings, prenatal exposure to alcohol continues to exceed by far exposure to illicit drugs. A diagnosis of fetal alcohol syndrome (FAS) is made when there are visible markers in each of three categories. Which category is not associated with the diagnosis of FAS? A. Respiratory conditions B. Impaired growth C. CNS abnormality D. Craniofacial dysmorphologies

ANS: A Respiratory difficulties are not a category of conditions that are related to FAS. Abnormalities related to FAS include organ deformities, genital malformations, and kidney and urinary defects. Impaired growth is a visible marker for FAS. CNS abnormalities with neurologic and intellectual impairments are categories used to assist in the diagnosis of FAS. An infant with FAS manifests at least two craniofacial abnormalities, such as microcephaly, short palpebral fissures, poorly developed philtrum, thin upper lip, or flattening of the maxilla.

Latasha, age 8 years, is being admitted to the hospital from the emergency department with an injury from falling off her bicycle. What will help her most in her adjustment to the hospital? a. Explain hospital schedules such as mealtimes. b. Use terms such as honey and dear to show a caring attitude. c. Explain when parents can visit and why siblings cannot come to see her. d. Orient her parents, because she is young, to her room and hospital facility.

ANS: A School-age children need to have control of their environment. The nurse should offer explanations or prepare the child for experiences that are unavoidable. The nurse should refer to the child by the preferred name. Telling the child about all of the limitations of visiting does not help her adjust to the hospital. At the age of 8 years, the child and parents should be oriented to the environment.

15. Nurses should be aware of the differences experience can make in labor pain such as: a. sensory pain for nulliparous women often is greater than for multiparous women during early labor. b. affective pain for nulliparous women usually is less than for multiparous women throughout the first stage of labor. c. women with a history of substance abuse experience more pain during labor. d. multiparous women have more fatigue from labor and therefore experience more pain.

ANS: A Sensory pain is greater for nulliparous women because their reproductive tract structures are less supple. Affective pain is higher for nulliparous women during the first stage but decreases for both nulliparous and multiparous during the second stage. Women with a history of substance abuse experience the same amount of pain as those without such a history. Nulliparous women have longer labors and therefore experience more fatigue.

The nurse is conducting a staff in-service on inherited childhood blood disorders. Which statement describes severe combined immunodeficiency syndrome (SCIDS)? a. There is a deficit in both the humoral and cellular immunity with this disease. b. Production of red blood cells is affected with this disease. c. Adult hemoglobin is replaced by abnormal hemoglobin in this disease. d. There is a deficiency of T and B lymphocyte production with this disease.

ANS: A Severe combined immunodeficiency syndrome (SCIDS) is a genetic disorder that results in deficits of both humoral and cellular immunity. Wiskott-Aldrich is an X-linked recessive disorder with selected deficiencies of T and B lymphocytes. Fanconi syndrome is a hereditary disorder of red cell production. Sickle cell disease is characterized by the replacement of adult hemoglobin with an abnormal hemoglobin S.

Parents of a child with sickle cell anemia ask the nurse, What happens to the hemoglobin in sickle cell anemia? Which statement by the nurse explains the disease process? a. Normal adult hemoglobin is replaced by abnormal hemoglobin. b. There is a lack of cellular hemoglobin being produced. c. There is a deficiency in the production of globulin chains. d. The size and depth of the hemoglobin are affected.

ANS: A Sickle cell anemia is one of a group of diseases collectively called hemoglobinopathies, in which normal adult hemoglobin is replaced by abnormal hemoglobin. Aplastic anemia is a lack of cellular elements being produced. Thalassemia major refers to a variety of inherited disorders characterized by deficiencies in production of certain globulin chains. Iron deficiency anemia affects the size and depth of the color.

23. To help a woman reduce the severity of nausea caused by morning sickness, the nurse might suggest that she: A. try a tart food or drink such as lemonade or salty foods such as potato chips. B. drink plenty of fluids early in the day. C. brush her teeth immediately after eating. D. never snack before bedtime.

ANS: A Some women can tolerate tart or salty foods when they are nauseous. The woman should avoid drinking too much when nausea is most likely, but she should make up the fluid levels later in the day when she feels better. The woman should avoid brushing her teeth immediately after eating. A small snack of cereal and milk or yogurt before bedtime may help the stomach in the morning.

4. A woman complains of severe abdominal and pelvic pain around the time of menstruation that has gotten worse over the last 5 years. She also complains of pain during intercourse and has tried unsuccessfully to get pregnant for the past 18 months. These symptoms are most likely related to: A. endometriosis. B. PMS. C. primary dysmenorrhea. D. secondary dysmenorrhea.

ANS: A Symptoms of endometriosis can change over time and may not reflect the extent of the disease. Major symptoms include dysmenorrhea and deep pelvic dyspareunia (painful intercourse). Impaired fertility may result from adhesions caused by endometriosis. Although endometriosis may be associated with secondary dysmenorrhea, it is not a cause of primary dysmenorrhea or PMS. In addition, this woman is complaining of dyspareunia and infertility, which are associated with endometriosis not with PMS or primary or secondary dysmenorrhea.

29. A means of controlling the birth of the fetal head with a vertex presentation is: a. the Ritgen maneuver. b. fundal pressure. c. the lithotomy position. d. the De Lee apparatus.

ANS: A The Ritgen maneuver extends the head during the actual birth and protects the perineum. Gentle, steady pressure against the fundus of the uterus facilitates vaginal birth. The lithotomy position has been commonly used in Western cultures, partly because it is convenient for the health care provider. The De Lee apparatus is used to suction fluid from the infant's mouth.

34. What important, immediate postoperative care practice should the nurse remember when caring for a woman who has had a mastectomy? A. The blood pressure (BP) cuff should not be applied to the affected arm. B. Venipuncture for blood work should be performed on the affected arm. C. The affected arm should be used for intravenous (IV) therapy. D. The affected arm should be held down close to the woman's side.

ANS: A The affected arm should not be used for BP readings, IV therapy, or venipuncture. The affected arm should be elevated with pillows above the level of the right atrium.

13. Appendicitis may be difficult to diagnose in pregnancy because the appendix is: A. displaced upward and laterally, high and to the right. B. displaced upward and laterally, high and to the left. C. deep at McBurney point. D. displaced downward and laterally, low and to the right.

ANS: A The appendix is displaced high and to the right, beyond McBurney point.

21. When using intermittent auscultation (IA) to assess uterine activity, the nurse should be cognizant that: a. the examiner's hand should be placed over the fundus before, during, and after contractions. b. the frequency and duration of contractions is measured in seconds for consistency. c. contraction intensity is given a judgment number of 1 to 7 by the nurse and patient together. d. the resting tone between contractions is described as either placid or turbulent.

ANS: A The assessment is done by palpation; duration, frequency, intensity, and resting tone must be assessed. The duration of contractions is measured in seconds; the frequency is measured in minutes. The intensity of contractions usually is described as mild, moderate, or strong. The resting tone usually is characterized as soft or relaxed.

9. A woman has chosen the calendar method of conception control. During the assessment process, it is most important that the nurse: A. obtain a history of menstrual cycle lengths for the past 6 months. B. determine the patient's weight gain and loss pattern for the previous year. C. examine skin pigmentation and hair texture for hormonal changes. D. explore the patient's previous experiences with conception control.

ANS: A The calendar method of conception control is based on the number of days in each cycle, counting from the first day of menses. The fertile period is determined after the lengths of menstrual cycles have been accurately recorded for 6 months. Weight gain or loss may be partly related to hormonal fluctuations, but it has no bearing on use of the calendar method. Integumentary changes may be related to hormonal changes, but they are not indicators for use of the calendar method. Exploring previous experiences with conception control may demonstrate patient understanding and compliancy, but it is not the most important aspect to assess for discussion of the calendar method.

13. The most critical nursing action in caring for the newborn immediately after birth is: a. keeping the newborn's airway clear. b. fostering parent-newborn attachment. c. drying the newborn and wrapping the infant in a blanket. d. administering eyedrops and vitamin K.

ANS: A The care given immediately after the birth focuses on assessing and stabilizing the newborn. Although fostering parent-infant attachment is an important task for the nurse, it is not the most critical nursing action in caring for the newborn immediately after birth. The nursing activities would be (in order of importance) to maintain a patent airway, support respiratory effort, and prevent cold stress by drying the newborn and covering the infant with a warmed blanket or placing the newborn under a radiant warmer. After the newborn has been stabilized, the nurse assesses the newborn's physical condition, weighs and measures the newborn, administers prophylactic eye ointment and a vitamin K injection, affixes an identification bracelet, wraps the newborn in warm blankets, and then gives the infant to the partner or mother when he or she is ready.

15. The transition phase during which ovarian function and hormone production decline is called: A. the climacteric. B. menarche. C. menopause. D. puberty.

ANS: A The climacteric is a transitional phase during which ovarian function and hormone production decline. Menarche is the term that denotes the first menstruation. Menopause refers only to the last menstrual period. Puberty is a broad term that denotes the entire transitional stage between childhood and sexual maturity.

7. Which stage of labor varies the most in length? a. First b. Second c. Third d. Fourth

ANS: A The first stage of labor is considered to last from the onset of regular uterine contractions to the full dilation of the cervix. The first stage is significantly longer than the second and third stages combined. In a first-time pregnancy, the first stage of labor can take up to 20 hours. The second stage of labor lasts from the time the cervix is fully dilated to the birth of the fetus. The average length is 20 minutes for a multiparous woman and 50 minutes for a nulliparous woman. The third stage of labor lasts from the birth of the fetus until the placenta is delivered. This stage may be as short as 3 minutes or as long as 1 hour. The fourth stage of labor, recovery, lasts approximately 2 hours after the delivery of the placenta.

10. The nurse must be cognizant that an individual's genetic makeup is known as his or her: a. genotype. b. phenotype. c. karyotype. d. chromotype.

ANS: A The genotype comprises all the genes the individual can pass on to a future generation. The phenotype is the observable expression of an individual's genotype. The karyotype is a pictorial analysis of the number, form, and size of an individual's chromosomes. Genotype refers to an individual's genetic makeup.

9. In evaluating the effectiveness of oxytocin induction, the nurse would expect: a. contractions lasting 80 to 90 seconds, 2 to 3 minutes apart. b. the intensity of contractions to be at least 110 to 130 mm Hg. c. labor to progress at least 2 cm/hr dilation. d. At least 30 mU/min of oxytocin will be needed to achieve cervical dilation.

ANS: A The goal of induction of labor would be to produce contractions that occur every 2 to 3 minutes and last 60 to 90 seconds. The intensity of the contractions should be 80 to 90 mm Hg by intrauterine pressure catheter. Cervical dilation of 1 cm/hr in the active phase of labor would be the goal in an oxytocin induction. The dose is increased by 1 to 2 mU/min at intervals of 30 to 60 minutes until the desired contraction pattern is achieved. Doses are increased up to a maximum of 20 to 40 mU/min.

12. Which behavior indicates that a woman is "seeking safe passage" for herself and her infant? A. She keeps all prenatal appointments. B. She "eats for two." C. She drives her car slowly. D. She wears only low-heeled shoes.

ANS: A The goal of prenatal care is to foster a safe birth for the infant and mother. Although eating properly, driving carefully, and using proper body mechanics all are healthy measures that a mother can take, obtaining prenatal care is the optimal method for providing safety for both herself and her baby.

29. A physician prescribes clomiphene citrate (Clomid, Serophene) for a woman experiencing infertility. She is very concerned about the risk of multiple births. The nurse's most appropriate response is: A. "This is a legitimate concern. Would you like to discuss this further before your treatment begins?" B. "No one has ever had more than triplets with Clomid." C. "Ovulation will be monitored with ultrasound so that this will not happen." D. "Ten percent is a very low risk, so you don't need to worry too much."

ANS: A The incidence of multiple pregnancies with the use of these medications is significantly increased. The patient's concern is legitimate and should be discussed so that she can make an informed decision. Stating that no one has ever had "more than triplets" is inaccurate and negates the patient's concerns. Ultrasound cannot ensure that a multiple pregnancy will not occur. The percentage quoted in this statement is inaccurate. The comment "don't worry" discredits the patient's concern.

28. Immediately after the forceps-assisted birth of an infant, the nurse should: a. assess the infant for signs of trauma. b. give the infant prophylactic antibiotics. c. apply a cold pack to the infant's scalp. d. measure the circumference of the infant's head.

ANS: A The infant should be assessed for bruising or abrasions at the site of application, facial palsy, and subdural hematoma. Prophylactic antibiotics are not necessary with a forceps delivery. A cold pack would put the infant at risk for cold stress and is contraindicated. Measuring the circumference of the head is part of the initial nursing assessment.

21. The exact cause of preterm labor is unknown and believed to be multifactorial. Infection is thought to be a major factor in many preterm labors. Select the type of infection that has not been linked to preterm births. a. Viral b. Periodontal c. Cervical d. Urinary tract

ANS: A The infections that increase the risk of preterm labor and birth are all bacterial. They include cervical, urinary tract, periodontal, and other bacterial infections. Therefore, it is important for the patient to participate in early, continual, and comprehensive prenatal care. Evidence has shown a link between periodontal infections and preterm labor. Researchers recommend regular dental care before and during pregnancy, oral assessment as a routine part of prenatal care, and scrupulous oral hygiene to prevent infection. Cervical infections of a bacterial nature have been linked to preterm labor and birth. The presence of urinary tract infections increases the risk of preterm labor and birth.

25. Which viral sexually transmitted infection is characterized by a primary infection followed by recurrent episodes? A. Herpes simplex virus (HSV)-2 B. Human papillomavirus (HPV) C. Human immunodeficiency virus (HIV) D. Cytomegalovirus (CMV)

ANS: A The initial HSV genital infection is characterized by multiple painful lesions, fever, chills, malaise, and severe dysuria; it may last 2 to 3 weeks. Recurrent episodes of HSV infection commonly have only local symptoms that usually are less severe than the symptoms of the initial infection. With HPV infection, lesions are a chronic problem. HIV is a retrovirus. Seroconversion to HIV positivity usually occurs within 6 to 12 weeks after the virus has entered the body. Severe depression of the cellular immune system associated with HIV infection characterizes acquired immunodeficiency syndrome (AIDS). AIDS has no cure. In most adults, the onset of CMV infection is uncertain and asymptomatic. However, the disease may become a chronic, persistent infection.

21. While teaching the expectant mother about personal hygiene during pregnancy, maternity nurses should be aware that: A. tub bathing is permitted even in late pregnancy unless membranes have ruptured. B. the perineum should be wiped from back to front. C. bubble bath and bath oils are permissible because they add an extra soothing and cleansing action to the bath. D. expectant mothers should use specially treated soap to cleanse the nipples.

ANS: A The main danger from taking baths is falling in the tub. The perineum should be wiped from front to back. Bubble baths and bath oils should be avoided because they may irritate the urethra. Soap, alcohol, ointments, and tinctures should not be used to cleanse the nipples because they remove protective oils. Warm water is sufficient.

5. The multiple marker test is used to assess the fetus for which condition? A. Down syndrome B. Diaphragmatic hernia C. Congenital cardiac abnormality D. Anencephaly

ANS: A The maternal serum level of alpha-fetoprotein is used to screen for Down syndrome, neural tube defects, and other chromosome anomalies. The multiple marker test would not detect diaphragmatic hernia, congenital cardiac abnormality, or anencephaly. Additional testing, such as ultrasonography and amniocentesis, would be required to diagnose these conditions.

18. A woman is 16 weeks pregnant and has elected to terminate her pregnancy. The nurse knows that the most common technique used for medical termination of a pregnancy in the second trimester is: A. dilation and evacuation (D&E). B. instillation of hypertonic saline into the uterine cavity. C. intravenous administration of Pitocin. D. vacuum aspiration.

ANS: A The most common technique for medical termination of a pregnancy in the second trimester is D&E. It is usually performed between 13 and 16 weeks. Hypertonic solutions injected directly into the uterus account for less than 1% of all abortions because other methods are safer and easier to use. Intravenous administration of Pitocin is used to induce labor in a woman with a third-trimester fetal demise. Vacuum aspiration is used for abortions in the first trimester.

24. Which major neonatal complication is carefully monitored after the birth of the infant of a diabetic mother? a. Hypoglycemia b. Hypercalcemiac. c. Hypobilirubinemia d. Hypoinsulinemia

ANS: A The neonate is at highest risk for hypoglycemia because fetal insulin production is accelerated during pregnancy to metabolize excessive glucose from the mother. At birth, the maternal glucose supply stops and the neonatal insulin exceeds the available glucose, thus leading to hypoglycemia. Hypocalcemia is associated with preterm birth, birth trauma, and asphyxia, all common problems of the infant of a diabetic mother. Excess erythrocytes are broken down after birth and release large amounts of bilirubin into the neonate's circulation, with resulting hyperbilirubinemia. Because fetal insulin production is accelerated during pregnancy, the neonate presents with hyperinsulinemia.

15. A pregnant woman's biophysical profile score is 8. She asks the nurse to explain the results. The nurse's best response is: A. "The test results are within normal limits." B. "Immediate delivery by cesarean birth is being considered." C. "Further testing will be performed to determine the meaning of this score." D. "An obstetric specialist will evaluate the results of this profile and, within the next week, will inform you of your options regarding delivery."

ANS: A The normal biophysical score ranges from 8 to 10 points if the amniotic fluid volume is adequate. A normal score allows conservative treatment of high risk patients. Delivery can be delayed if fetal well-being is indicated. Scores less than 4 should be investigated, and delivery could be initiated sooner than planned. This score is within normal range, and no further testing is required at this time. The results of the biophysical profile are usually available immediately after the procedure is performed.

19. The mucous plug that forms in the endocervical canal is called the: A. operculum. B. leukorrhea. C. funic souffle. D. ballottement.

ANS: A The operculum protects against bacterial invasion. Leukorrhea is the mucus that forms the endocervical plug (the operculum). The funic souffle is the sound of blood flowing through the umbilical vessels. Ballottement is a technique for palpating the fetus.

The school nurse is informed that a child with human immunodeficiency virus (HIV) will be attending school soon. Which is an important nursing intervention? a. Carefully follow universal precautions. b. Determine how the child became infected. c. Inform the parents of the other children. d. Reassure other children that they will not become infected.

ANS: A Universal precautions are necessary to prevent further transmission of the disease. It is not the role of the nurse to determine how the child became infected. Informing the parents of other children and reassuring children that they will not become infected is a violation of the childs right to privacy.

18. What is the nurse's understanding of the appropriate role of primary and secondary powers? a. Primary powers are responsible for the effacement and dilation of the cervix. b. Effacement is generally well ahead of dilation in women giving birth for the first time; they are closer together in subsequent pregnancies. c. Scarring of the cervix caused by a previous infection or surgery may make the delivery a bit more painful, but it should not slow or inhibit dilation. d. Pushing in the second stage of labor is more effective if the woman can breathe deeply and control some of her involuntary needs to push, as the nurse directs.

ANS: A The primary powers are responsible for dilation and effacement; secondary powers are concerned with expulsion of the fetus. Effacement is generally well ahead of dilation in first-time pregnancies; they are closer together in subsequent pregnancies. Scarring of the cervix may slow dilation. Pushing is more effective and less fatiguing when the woman begins to push only after she has the urge to do so.

2. The uterus is a muscular, pear-shaped organ that is responsible for: A. cyclic menstruation. B. sex hormone production. C. fertilization. D. sexual arousal.

ANS: A The uterus is an organ for reception, implantation, retention, and nutrition of the fertilized ovum; it also is responsible for cyclic menstruation. Hormone production and fertilization occur in the ovaries. Sexual arousal is a feedback mechanism involving the hypothalamus, the pituitary gland, and the ovaries.

17. The female reproductive organ(s) responsible for cyclic menstruation is/are the: A. uterus. B. ovaries. C. vaginal vestibule. D. urethra.

ANS: A The uterus is responsible for cyclic menstruation. It also houses and nourishes the fertilized ovum and the fetus. The ovaries are responsible for ovulation and production of estrogen; the uterus is responsible for cyclic menstruation. The vaginal vestibule is an external organ that has openings to the urethra and vagina; the uterus is responsible for cyclic menstruation. The urethra is not a reproductive organ, although it is found in the area.

6. When assessing a woman in the first stage of labor, the nurse recognizes that the most conclusive sign that uterine contractions are effective would be: a. dilation of the cervix. b. descent of the fetus. c. rupture of the amniotic membranes. d. increase in bloody show.

ANS: A The vaginal examination reveals whether the woman is in true labor. Cervical change, especially dilation, in the presence of adequate labor indicates that the woman is in true labor. Descent of the fetus, or engagement, may occur before labor. Rupture of membranes may occur with or without the presence of labor. Bloody show may indicate slow, progressive cervical change (e.g., effacement) in both true and false labor.

10. A pregnant woman's amniotic membranes rupture. Prolapsed umbilical cord is suspected. What intervention would be the top priority? a. Placing the woman in the knee-chest position. b. Covering the cord in sterile gauze soaked in saline. c. Preparing the woman for a cesarean birth. d. Starting oxygen by face mask.

ANS: A The woman is assisted into a position (e.g., modified Sims position, Trendelenburg position, or the knee-chest position) in which gravity keeps the pressure of the presenting part off the cord. Although covering the cord in sterile gauze soaked saline, preparing the woman for a cesarean, and starting oxygen by face mark are appropriate nursing interventions in the event of a prolapsed cord, the intervention of top priority would be positioning the mother to relieve cord compression.

29. In assisting with the two factors that have an effect on fetal status (i.e., pushing and positioning), nurses should: a. encourage the woman's cooperation in avoiding the supine position. b. advise the woman to avoid the semi-Fowler position. c. encourage the woman to hold her breath and tighten her abdominal muscles to produce a vaginal response. d. instruct the woman to open her mouth and close her glottis, letting air escape after the push.

ANS: A The woman should maintain a side-lying position. The semi-Fowler position is the recommended side-lying position with a lateral tilt to the uterus. The Valsalva maneuver, which encourages the woman to hold her breath and tighten her abdominal muscles, should be avoided. Both the mouth and glottis should be open, letting air escape during the push.

38. A laboring woman is lying in the supine position. The most appropriate nursing action at this time is to: a. ask her to turn to one side. b. elevate her feet and legs. c. take her blood pressure. d. determine whether fetal tachycardia is present.

ANS: A The woman's supine position may cause the heavy uterus to compress her inferior vena cava, thus reducing blood return to her heart and reducing placental blood flow. Elevating her legs will not relieve the pressure from the inferior vena cava. If the woman is allowed to stay in the supine position and blood flow to the placental is reduced significantly, fetal tachycardia may occur. The most appropriate nursing action is to prevent this from occurring by turning the woman to her side. Blood pressure readings may be obtained when the patient is in the appropriate and safest position.

11. To help patients manage discomfort and pain during labor, nurses should be aware that: a. the predominant pain of the first stage of labor is the visceral pain located in the lower portion of the abdomen. b. referred pain is the extreme discomfort between contractions. c. the somatic pain of the second stage of labor is more generalized and related to fatigue. d. pain during the third stage is a somewhat milder version of the second stage.

ANS: A This pain comes from cervical changes, distention of the lower uterine segment, and uterine ischemia. Referred pain occurs when the pain that originates in the uterus radiates to the abdominal wall, lumbosacral area of the back, iliac crests, and gluteal area. Second-stage labor pain is intense, sharp, burning, and localized. Third-stage labor pain is similar to that of the first stage.

17. A pregnant woman is in her third trimester. She asks the nurse to explain how she can tell true labor from false labor. The nurse would explain that "true" labor contractions: a. increase with activity such as ambulation. b. decrease with activity. c. are always accompanied by the rupture of the bag of waters. d. alternate between a regular and an irregular pattern.

ANS: A True labor contractions become more intense with walking. False labor contractions often stop with walking or position changes. Rupture of membranes may occur before or during labor. True labor contractions are regular.

Which of the following types of seizures may be difficult to detect? a. Absence b. Generalized c. Simple partial d. Complex partial

ANS: A Absence seizures may go unrecognized because little change occurs in the child's behavior during the seizure. Generalized, simple partial, and complex partial seizures all have clinical manifestations that are observable.

11. When managing the care of a woman in the second stage of labor, the nurse uses various measures to enhance the progress of fetal descent. These measures include: a. encouraging the woman to try various upright positions, including squatting and standing. b. telling the woman to start pushing as soon as her cervix is fully dilated. c. continuing an epidural anesthetic so pain is reduced and the woman can relax. d. coaching the woman to use sustained, 10- to 15-second, closed-glottis bearing-down efforts with each contraction.

ANS: A Upright positions and squatting both may enhance the progress of fetal descent. Many factors dictate when a woman will begin pushing. Complete cervical dilation is necessary, but it is only one factor. If the fetal head is still in a higher pelvic station, the physician or midwife may allow the woman to "labor down" (allowing more time for fetal descent, thereby reducing the amount of pushing needed) if the woman is able. The epidural may mask the sensations and muscle control needed for the woman to push effectively. Closed glottic breathing may trigger the Valsalva maneuver, which increases intrathoracic and cardiovascular pressures, reducing cardiac output and inhibiting perfusion of the uterus and placenta. In addition, holding the breath for longer than 5 to 7 seconds diminishes the perfusion of oxygen across the placenta and results in fetal hypoxia.

26. A first-time mother at 18 weeks of gestation comes for her regularly scheduled prenatal visit. The patient tells the nurse that she is afraid that she is going into premature labor because she is beginning to have regular contractions. The nurse explains that this is the Braxton Hicks sign and teaches the patient that this type of contraction: A. is painless. B. increases with walking. C. causes cervical dilation. D. impedes oxygen flow to the fetus.

ANS: A Uterine contractions can be felt through the abdominal wall soon after the fourth month of gestation. Braxton Hicks contractions are regular and painless and continue throughout the pregnancy. Although they are not painful, some women complain that they are annoying. Braxton Hicks contractions usually cease with walking or exercise. They can be mistaken for true labor; however, they do not increase in intensity or frequency or cause cervical dilation. In addition, they facilitate uterine blood flow through the intervillous spaces of the placenta and promote oxygen delivery to the fetus.

4. When planning care for a laboring woman whose membranes have ruptured, the nurse recognizes that the woman's risk for _________________________ has increased. a. intrauterine infection b. hemorrhage c. precipitous labor d. supine hypotension

ANS: A When the membranes rupture, microorganisms from the vagina can ascend into the amniotic sac and cause chorioamnionitis and placentitis. Rupture of membranes (ROM) is not associated with fetal or maternal bleeding. Although ROM may increase the intensity of contractions and facilitate active labor, it does not result in precipitous labor. ROM has no correlation with supine hypotension.

29. An essential component of counseling women regarding safe sex practices includes discussion regarding avoiding the exchange of body fluids. The physical barrier promoted for the prevention of sexually transmitted infections and human immunodeficiency virus is the condom. Nurses can help motivate patients to use condoms by initiating a discussion related to a number of aspects of condom use. The most important of these is: A. strategies to enhance condom use. B. choice of colors and special features. C. leaving the decision up to the male partner. D. places to carry condoms safely.

ANS: A When the nurse opens discussion on safe sex practices, it gives the woman permission to clear up any concerns or misapprehensions that she may have regarding condom use. The nurse can also suggest ways that the woman can enhance her condom negotiation and communications skills. These include role playing, rehearsal, cultural barriers, and situations that put the patient at risk. Although women can be taught the differences among condoms, such as size ranges, where to purchase, and price, this is not as important as negotiating the use of safe sex practices. Women must address the issue of condom use with every sexual contact. Some men need time to think about this. If they appear reluctant, the woman may want to reconsider the relationship. Although not ideal, women may safely choose to carry condoms in shoes, wallets, or inside their bra. They should be taught to keep the condom away from heat. This information is important; however, it is not germane if the woman cannot even discuss strategies on how to enhance condom use.

25. With regard to a woman's intake and output during labor, nurses should be aware that: a. the tradition of restricting the laboring woman to clear liquids and ice chips is being challenged because regional anesthesia is used more often than general anesthesia. b. intravenous (IV) fluids usually are necessary to ensure that the laboring woman stays hydrated. c. routine use of an enema empties the rectum and is very helpful for producing a clean, clear delivery. d. when a nulliparous woman experiences the urge to defecate, it often means birth will follow quickly.

ANS: A Women are awake with regional anesthesia and are able to protect their own airway, which reduces the worry over aspiration. Routine IV fluids during labor are unlikely to be beneficial and may be harmful. Routine use of an enema is at best ineffective and may be harmful. A multiparous woman may feel the urge to defecate and it may mean birth will follow quickly, but not for a first timer.

16. A woman was treated recently for toxic shock syndrome (TSS). She has intercourse occasionally and uses over-the-counter protection. On the basis of her history, what contraceptive method should she and her partner avoid? a. Cervical cap b. Condom c. Vaginal film d. Vaginal sheath

ANS: A Women with a history of TSS should not use a cervical cap. Condoms, vaginal films, and vaginal sheaths are not contraindicated for a woman with a history of TSS.

As related to the normal functioning of the renal system in newborns, nurses should be aware that: a. The pediatrician should be notified if the newborn has not voided in 24 hours. b. Breastfed infants likely will void more often during the first days after birth. c. Brick dust or blood on a diaper is always cause to notify the physician. d. Weight loss from fluid loss and other normal factors should be made up in 4 to 7 days.

ANS: A A newborn who has not voided in 24 hours may have any of a number of problems, some of which deserve the attention of the pediatrician. Formula-fed infants tend to void more frequently in the first 3 days; breastfed infants void less during this time because the mothers breast milk has not come in yet. Brick dust may be uric acid crystals; blood spotting could be caused by withdrawal of maternal hormones (pseudomenstruation) or a circumcision. The physician must be notified only if there is no apparent cause of bleeding. Weight loss from fluid loss may take 14 days to regain. PTS: 1 DIF: Cognitive Level: Comprehension REF: 562 OBJ: Nursing Process: Planning, Implementation MSC: Client Needs: Physiologic Integrity

During which phase of maternal adjustment will the mother relinquish the baby of her fantasies and accept the real baby? a. Letting go b. Taking hold c. Taking in d. Taking on

ANS: A Accepting the real infant and relinquishing the fantasy infant occurs during the letting-go phase of maternal adjustment. During the taking-hold phase the mother assumes responsibility for her own care and shifts her attention to the infant. In the taking-in phase the mother is primarily focused on her own needs. There is no taking-on phase of maternal adjustment. PTS: 1 DIF: Cognitive Level: Knowledge REF: 516 OBJ: Nursing Process: Assessment MSC: Client Needs: Psychosocial Integrity

When caring for the child with Reye syndrome, the priority nursing intervention should be to: a. monitor intake and output. b. prevent skin breakdown. c. observe for petechiae. d. do range-of-motion exercises.

ANS: A Accurate and frequent monitoring of intake and output is essential for adjusting fluid volumes to prevent both dehydration and cerebral edema. Preventing skin breakdown, observing for petechiae, and doing range-of-motion exercises are important interventions in the care of a critically ill or comatose child. Careful monitoring of intake and output is a priority.

A new mother states that her infant must be cold because the babys hands and feet are blue. The nurse explains that this is a common and temporary condition called: a. Acrocyanosis. b. Erythema neonatorum. c. Harlequin color. d. Vernix caseosa.

ANS: A Acrocyanosis, or the appearance of slightly cyanotic hands and feet, is caused by vasomotor instability, capillary stasis, and a high hemoglobin level. Acrocyanosis is normal and appears intermittently over the first 7 to 10 days. Erythema toxicum (also called erythema neonatorum) is a transient newborn rash that resembles flea bites. The harlequin sign is a benign, transient color change in newborns. Half of the body is pale, and the other half is ruddy or bluish red with a line of demarcation. Vernix caseosa is a cheeselike, whitish substance that serves as a protective covering. PTS: 1 DIF: Cognitive Level: Knowledge REF: 558 OBJ: Nursing Process: Diagnosis MSC: Client Needs: Health Promotion and Maintenance

18. Which activity is most appropriate for developing fine motor skills in the school-age child? a. Drawing b. Singing c. Soccer d. Swimming

ANS: A Activities such as drawing, building models, and playing a musical instrument increase the school-age child's fine motor skills. Singing is an appropriate activity for the school-age child, but it does not increase fine motor skills. The school-age child needs to participate in group activities to increase both gross motor skills and social skills, but group activities do not increase fine motor skills. Swimming is an activity that also increases gross motor skills.

The abuse of which of the following substances during pregnancy is the leading cause of cognitive impairment in the United States? a. Alcohol b. Tobacco c. Marijuana d. Heroin

ANS: A Alcohol abuse during pregnancy is recognized as one of the leading causes of cognitive impairment in the United States. PTS: 1 DIF: Cognitive Level: Knowledge REF: 670 OBJ: Nursing Process: Assessment MSC: Client Needs: Psychosocial Integrity

The nurse is caring for an adolescent with osteosarcoma being admitted to undergo chemotherapy. The adolescent had a right above-the-knee amputation 2 months ago and has been experiencing "phantom limb pain." Which prescribed medication is appropriate to administer to relieve phantom limb pain? a. Amitriptyline (Elavil) b. Hydrocodone (Vicodin) c. Oxycodone (OxyContin) d. Alprazolam (Xanax)

ANS: A Amitriptyline (Elavil) has been used successfully to decrease phantom limb pain. Opioids such as Vicodin or OxyContin would not be prescribed for this pain. A benzodiazepine, Xanax, would not be prescribed for this type of pain.

A woman gave birth to a 7-pound, 6-ounce infant girl 1 hour ago. The birth was vaginal, and the estimated blood loss (EBL) was approximately 1500 mL. When assessing the womans vital signs, the nurse would be concerned to see: a. Temperature 37.9 C, heart rate 120, respirations 20, blood pressure (BP) 90/50. b. Temperature 37.4 C, heart rate 88, respirations 36, BP 126/68. c. Temperature 38 C, heart rate 80, respirations 16, BP 110/80. d. Temperature 36.8 C, heart rate 60, respirations 18, BP 140/90.

ANS: A An EBL of 1500 mL with tachycardia and hypotension suggests hypovolemia caused by excessive blood loss. An increased respiratory rate of 36 may be secondary to pain from the birth. Temperature may increase to 38 C during the first 24 hours as a result of the dehydrating effects of labor. A BP of 140/90 is slightly elevated, which may be caused by the use of oxytocic medications. PTS: 1 DIF: Cognitive Level: Comprehension REF: 488 OBJ: Nursing Process: Assessment, Diagnosis MSC: Client Needs: Physiologic Integrity

Rho immune globulin will be ordered postpartum if which situation occurs? a. Mother Rh?2-, baby Rh+ b. Mother Rh?2-, baby Rh?2- c. Mother Rh+, baby Rh+ d. Mother Rh+, baby Rh?2-

ANS: A An Rh?2- mother delivering an Rh+ baby may develop antibodies to fetal cells that entered her bloodstream when the placenta separated. The Rho immune globulin works to destroy the fetal cells in the maternal circulation before sensitization occurs. If mother and baby are both Rh+ or Rh?2- the blood types are alike, so no antibody formation would be anticipated. If the Rh+ blood of the mother comes in contact with the Rh?2- blood of the infant, no antibodies would develop because the antigens are in the mothers blood, not the infants. PTS: 1 DIF: Cognitive Level: Comprehension REF: 503 OBJ: Nursing Process: Assessment MSC: Client Needs: Physiologic Integrity

23. A child has an avulsed (knocked-out) tooth. In which medium should the nurse instruct the parents to place the tooth for transport to the dentist? a. Cold milk b. Cold water c. Warm salt water d. A dry, clean jar

ANS: A An avulsed tooth should be placed in a suitable medium for transport, either cold milk or saliva (under the child's or parent's tongue). Cold milk is a more suitable medium for transport than cold water, warm salt water, or a dry, clean jar.

The nurse should immediately alert the physician when: a. The infant is dusky and turns cyanotic when crying. b. Acrocyanosis is present at age 1 hour. c. The infants blood glucose level is 45 mg/dL. d. The infant goes into a deep sleep at age 1 hour.

ANS: A An infant who is dusky and becomes cyanotic when crying is showing poor adaptation to extrauterine life. Acrocyanosis is an expected finding during the early neonatal life. This is within normal range for a newborn. Infants enter the period of deep sleep when they are about 1 hour old. PTS: 1 DIF: Cognitive Level: Application REF: 558 OBJ: Nursing Process: Assessment MSC: Client Needs: Physiologic Integrity

39. An 18-month-old child is seen in the clinic with AOM. Trimethoprim-sulfamethoxazole (Bactrim) is prescribed. Which statement made by the parent indicates a correct understanding of the instructions? a. I should administer all the prescribed medication. b. I should continue medication until the symptoms subside. c. I will immediately stop giving medication if I notice a change in hearing. d. I will stop giving medication if fever is still present in 24 hours

ANS: A Antibiotics should be given for their full course to prevent recurrence of infection with resistant bacteria. Symptoms may subside before the full course is given. Hearing loss is a complication of AOM. Antibiotics should continue to be given. Medication may take 24 to 48 hours to make symptoms subside. It should be continued

New parents express concern that, because of the mothers emergency cesarean birth under general anesthesia, they did not have the opportunity to hold and bond with their daughter immediately after her birth.The nurses response should convey to the parents that: a. Attachment, or bonding, is a process that occurs over time and does not require early contact. b. The time immediately after birth is a critical period for people. c. Early contact is essential for optimum parent-infant relationships. d. They should just be happy that the infant is healthy.

ANS: A Attachment, or bonding, is a process that occurs over time and does not require early contact. The formerly accepted definition of bonding held that the period immediately after birth was a critical time for bonding to occur. Research since has indicated that parent-infant attachment occurs over time. A delay does not inhibit the process. Parent-infant attachment involves activities such as touching, holding, and gazing; it is not exclusively eye contact. A response that conveys that the parents should just be happy that the infant is healthy is inappropriate because it is derogatory and belittling. PTS: 1 DIF: Cognitive Level: Application REF: 513 OBJ: Nursing Process: Implementation MSC: Client Needs: Health Promotion and Maintenance

The self-destruction of excess hypertrophied tissue in the uterus is called: a. Autolysis. b. Subinvolution. c. Afterpain. d. Diastasis.

ANS: A Autolysis is caused by a decrease in hormone levels. Subinvolution is failure of the uterus to return to a nonpregnant state. Afterpain is caused by uterine cramps 2 to 3 days after birth. Diastasis refers to the separation of muscles. PTS: 1 DIF: Cognitive Level: Knowledge REF: 483 OBJ: Nursing Process: Assessment MSC: Client Needs: Physiologic Integrity

Which can result from the bone demineralization associated with immobility? a. Osteoporosis b. Urinary retention c. Pooling of blood d. Susceptibility to infection

ANS: A Bone demineralization leads to a negative calcium balance, osteoporosis, pathologic fractures, extraosseous bone formation, and renal calculi. Urinary retention is secondary to the effect of immobilization on the urinary tract. Pooling of blood is a result of the cardiovascular effects of immobilization. Susceptibility to infection can result from the effects of immobilization on the respiratory and renal systems.

The best reason for recommending formula over breastfeeding is that: a. The mother has a medical condition or is taking drugs that could be passed along to the infant via breast milk. b. The mother lacks confidence in her ability to breastfeed. c. Other family members or care providers also need to feed the baby. d. The mother sees bottle-feeding as more convenient.

ANS: A Breastfeeding is contraindicated when mothers have certain viruses, are undergoing chemotherapy, or are using/abusing illicit drugs. A lack of confidence, the need for others to feed the baby, and the convenience of bottle-feeding are all honest reasons for not breastfeeding, although further education concerning the ease of breastfeeding and its convenience, benefits, and adaptability (expressing milk into bottles) could change some minds. In any case the nurse must provide information in a nonjudgmental manner and respect the mothers decision. Nonetheless, breastfeeding is definitely contraindicated when the mother has medical or drug issues of her own. PTS: 1 DIF: Cognitive Level: Comprehension REF: 656 OBJ: Nursing Process: Planning MSC: Client Needs: Physiologic Integrity

According to the recommendations of the American Academy of Pediatrics on infant nutrition: a. Infants should be given only human milk for the first 6 months of life. b. Infants fed on formula should be started on solid food sooner than breastfed infants. c. If infants are weaned from breast milk before 12 months, they should receive cows milk, not formula. d. After 6 months mothers should shift from breast milk to cows milk.

ANS: A Breastfeeding/human milk should also be the sole source of milk for the second 6 months. Infants start on solids when they are ready, usually at 6 months, whether they start on formula or breast milk. If infants are weaned from breast milk before 12 months, they should receive iron-fortified formula, not cows milk. PTS: 1 DIF: Cognitive Level: Knowledge REF: 633 OBJ: Nursing Process: Planning MSC: Client Needs: Physiologic Integrity

13. A school-age child has had an upper respiratory tract infection for several days and then began having a persistent dry, hacking cough that was worse at night. The cough has become productive in the past 24 hours. This is most suggestive of: a. Bronchitis. c. Viral-induced asthma. b. Bronchiolitis. d. Acute spasmodic laryngitis.

ANS: A Bronchitis is characterized by these symptoms and occurs in children older than 6 years. Bronchiolitis is rare in children older than 2 years. Asthma is a chronic inflammation of the airways that may be exacerbated by a virus. Acute spasmodic laryngitis occurs in children between 3 months and 3 years.

43. A child with cystic fibrosis (CF) receives aerosolized bronchodilator medication. When should this medication be administered? a. Before chest physiotherapy (CPT) c. Before receiving 100% oxygen b. After CPT d. After receiving 100% oxygen

ANS: A Bronchodilators should be given before CPT to open bronchi and make expectoration easier. Aerosolized bronchodilator medications are not helpful when used after CPT. Oxygen administration is necessary only in acute episodes with caution because of chronic carbon dioxide retention

Which clinical manifestations would suggest hydrocephalus in a neonate? a. Bulging fontanel and dilated scalp veins b. Closed fontanel and high-pitched cry c. Constant low-pitched cry and restlessness d. Depressed fontanel and decreased blood pressure

ANS: A Bulging fontanels, dilated scalp veins, and separated sutures are clinical manifestations of hydrocephalus in neonates. Closed fontanel and high-pitched cry, constant low-pitched cry and restlessness, and depressed fontanel and decreased blood pressure are not clinical manifestations of hydrocephalus, but all should be referred for evaluation.

17. The ability to mentally understand that 1 + 3 = 4 and 4 − 3 = 1 occurs in which stage of cognitive development? a. Concrete operations stage b. Formal operations stage c. Intuitive thought stage d. Preoperations stage

ANS: A By 7 to 8 years of age, the child is able to retrace a process (reversibility) and has the skills necessary for solving mathematical problems. This stage is called concrete operations. The formal operations stage deals with abstract reasoning and does not occur until adolescence. Thinking in the intuitive stage is based on immediate perceptions. A child in this stage often solves problems by random guessing. In preoperational thinking, the child is usually able to add 1 + 3 = 4 but is unable to retrace the process.

23. Which statement expresses accurately the genetic implications of cystic fibrosis (CF)? a. If it is present in a child, both parents are carriers of this defective gene. b. It is inherited as an autosomal dominant trait. c. It is a genetic defect found primarily in non-Caucasian population groups. d. There is a 50% chance that siblings of an affected child also will be affected.

ANS: A CF is an autosomal recessive gene inherited from both parents and is found primarily in Caucasian populations. An autosomal recessive inheritance pattern means that there is a 25% chance that a sibling will be infected but a 50% chance a sibling will be a carrier.

6. An 8-year-old girl tells the nurse that she has cancer because God is punishing her for "being bad." She shares her concern that, if she dies, she will go to hell. How should the nurse interpret this belief? a. It is a belief common at this age. b. It is a belief that forms the basis for most religions. c. The belief is suggestive of excessive family pressure. d. The statement suggests a failed attempt to develop a conscience.

ANS: A Children at this age may view illness or injury as a punishment for a real or imagined mystique. The belief in divine punishment is common at this age.

34. A nurse caring for a child receiving chemotherapy notes that the child's urine specific gravity is 1.010. Which action by the nurse is the most appropriate? A. Document the findings in the child's chart. B. Increase the rate of the IV fluids per protocol. C. Notify the provider about the laboratory results. D. Prepare to administer an alkalizing agent.

ANS: A Children on chemotherapy should remain well hydrated to ensure the medications and any toxic by-products are flushed out. The urine specific gravity should remain at 1.012 or below. The nurse needs to take no further action after documenting the findings. The IV rate should be increased if the specific gravity is above that level. The provider does not need to be notified specifically about this normal finding. An alkalizing agent is not needed.

28. In providing nourishment for a child with cystic fibrosis (CF), which factor should the nurse keep in mind? a. Diet should be high in carbohydrates and protein. b. Diet should be high in easily digested carbohydrates and fats. c. Most fruits and vegetables are not well tolerated. d. Fats and proteins must be greatly curtailed.

ANS: A Children with CF require a well-balanced, high-protein, high-calorie diet because of impaired intestinal absorption. Enzyme supplementation helps digest foods; other modifications are not necessary. A wellbalanced diet containing fruits and vegetables is important. Fats and proteins are a necessary part of a wellbalanced diet.

44. A nurse is interpreting the results of a tuberculin skin test (TST) on an adolescent who is human immunodeficiency virus (HIV) positive. Which induration size indicates a positive result for this child 48 to 72 hours after the test? a. 5 mm c. 15 mm Test Bank - Maternal Child Nursing Care by Perry (6th Edition, 2017) 663 b. 10 mm d. 20 mm

ANS: A Clinical evidence of a positive TST in children receiving immunosuppressive therapy, including immunosuppressive doses of steroids, or who have immunosuppressive conditions, including HIV infection, is an induration of 5 mm. Children younger than 4 years of age (a) with other medical risk conditions, including Hodgkins disease, lymphoma, diabetes mellitus, chronic renal failure, or malnutrition; (b) born or whose parents were born in high-prevalence tuberculosis (TB) regions of the world; (c) frequently exposed to adults who are HIV infected, homeless, users of illicit drugs, residents of nursing homes, incarcerated or institutionalized, or migrant farm workers; and (d) who travel to high-prevalence TB regions of the world are positive when the induration is 10 mm. Children 4 years of age or older without any risk factors are positive when the induration is 20 mm.

A new mother asks whether she should feed her newborn colostrum, because it is not real milk. The nurses most appropriate answer is: a. Colostrum is high in antibodies, protein, vitamins, and minerals. b. Colostrum is lower in calories than milk and should be supplemented by formula. c. Giving colostrum is important in helping the mother learn how to breastfeed before she goes home. d. Colostrum is unnecessary for newborns.

ANS: A Colostrum is important because it has high levels of the nutrients needed by the neonate and helps protect against infection. Supplementation is not necessary; it will decrease stimulation to the breast and decrease the production of milk. It is important for the mother to feel comfortable in this role before discharge; however, the importance of the colostrum to the infant is the top priority. Colostrum provides immunities and enzymes necessary to cleanse the gastrointestinal system, among other things. PTS: 1 DIF: Cognitive Level: Knowledge REF: 639 OBJ: Nursing Process: Implementation MSC: Client Needs: Physiologic Integrity

8.Therapeutic management of nephrosis includes: a. Corticosteroids. b. Antihypertensive agents. c. Long-term diuretics. d. Increased fluids to promote diuresis.

ANS: A Corticosteroids are the first line of therapy for nephrosis. Response is usually seen within 7 to 21 days. Antihypertensive agents and long-term diuretic therapy are usually not necessary. A diet that has fluid and salt restrictions may be indicated.

4.What should the nurse recommend to prevent urinary tract infections in young girls? a. Wearing cotton underpants b. Limiting bathing as much as possible c. Increasing fluids; decreasing salt intake d. Cleansing the perineum with water after voiding

ANS: A Cotton underpants are preferable to nylon underpants. No evidence exists that limiting bathing, increasing fluids, decreasing salt intake, or cleansing the perineum with water decreases urinary tract infections in young girls.

6. A woman who is 32 weeks' pregnant is informed by the nurse that a danger sign of pregnancy could be: A. constipation. B. alteration in the pattern of fetal movement. C. heart palpitations. D. edema in the ankles and feet at the end of the day.

ANS: B An alteration in the pattern or amount of fetal movement may indicate fetal jeopardy. Constipation, heart palpitations, and ankle and foot edema are normal discomforts of pregnancy that occur in the second and third trimesters.

Discharge instruction, or teaching the woman what she needs to know to care for herself and her newborn, officially begins: a. At the time of admission to the nurses unit. b. When the infant is presented to the mother at birth. c. During the first visit with the physician in the unit. d. When the take-home information packet is given to the couple.

ANS: A Discharge planning, the teaching of maternal and newborn care, begins on the womans admission to the unit, continues throughout her stay, and actually never ends as long as she has contact with medical personnel. PTS: 1 DIF: Cognitive Level: Comprehension REF: 506 OBJ: Nursing Process: Planning MSC: Client Needs: Health Promotion and Maintenance

37. A nurse is charting that a hospitalized child has labored breathing. Which describes labored breathing? a. Dyspnea c. Hypopnea b. Tachypnea d. Orthopnea

ANS: A Dyspnea is labored breathing. Tachypnea is rapid breathing. Hypopnea is breathing that is too shallow. Orthopnea is difficulty breathing except in upright position.

31. An appropriate nursing intervention when caring for a child with pneumonia is to: a. Encourage rest. b. Encourage the child to lie on the unaffected side. c. Administer analgesics. d. Place the child in the Trendelenburg position

ANS: A Encouraging rest by clustering care and promoting a quiet environment is the best intervention for a child with pneumonia. Lying on the affected side may promote comfort by splinting the chest and reducing pleural rubbing. Analgesics are not indicated. Children should be placed in a semi-erect position or position of comfort

A pregnant woman wants to breastfeed her infant; however, her husband is not convinced that there are any scientific reasons to do so. The nurse can give the couple printed information comparing breastfeeding and bottle-feeding. Which statement is most accurate? Bottle-feeding using commercially prepared infant formulas: a. Increases the risk that the infant will develop allergies. b. Helps the infant sleep through the night. c. Ensures that the infant is getting iron in a form that is easily absorbed. d. Requires that multivitamin supplements be given to the infant.

ANS: A Exposure to cows milk poses a risk of developing allergies, eczema, and asthma. Bottle-feeding using commercially prepared infant formulas helps the infant sleep through the night is a false statement. Iron is better absorbed from breast milk than from formula. Commercial formulas are designed to meet the nutritional needs of the infant and resemble breast milk. PTS: 1 DIF: Cognitive Level: Application REF: 657 OBJ: Nursing Process: Diagnosis MSC: Client Needs: Physiologic Integrity

An infant with severe meconium aspiration syndrome (MAS) is not responding to conventional treatment. Which highly technical method of treatment may be necessary for an infant who does not respond to conventional treatment? a. Extracorporeal membrane oxygenation b. Respiratory support with a ventilator c. Insertion of a laryngoscope and suctioning of the trachea d. Insertion of an endotracheal tube

ANS: A Extracorporeal membrane oxygenation is a highly technical method that oxygenates the blood while bypassing the lungs, thus allowing the infants lungs to rest and recover. The infant is likely to have been first connected to a ventilator. Laryngoscope insertion and tracheal suctioning are performed after birth before the infant takes the first breath. An endotracheal tube will be in place to facilitate deep tracheal suctioning and ventilation. PTS: 1 DIF: Cognitive Level: Knowledge REF: 693 OBJ: Nursing Process: Evaluation MSC: Client Needs: Physiologic Integrity

An adolescent with osteosarcoma is scheduled for a leg amputation in 2 days. The nurse's approach should include which action? a. Answering questions with straightforward honesty b. Avoiding discussing the seriousness of the condition c. Explaining that, although the amputation is difficult, it will cure the cancer d. Assisting the adolescent in accepting the amputation as better than a long course of chemotherapy

ANS: A Honesty is essential to gain the child's cooperation and trust. The diagnosis of cancer should not be disguised with falsehoods. The adolescent should be prepared for the surgery so he or she has time to reflect on the diagnosis and subsequent treatment. This allows questions to be answered. To accept the need for radical surgery, the child must be aware of the lack of alternatives for treatment. Amputation is necessary, but it will not guarantee a cure. Chemotherapy is an integral part of the therapy with surgery. The child should be informed of the need for chemotherapy and its side effects before surgery.

Parents have been asked by the neonatologist to provide breast milk for their newborn son, who was born prematurely at 32 weeks of gestation. The nurse who instructs them about pumping, storing, and transporting the milk needs to assess their knowledge of lactation. Which statement is valid? a. A premature infant more easily digests breast milk than formula. b. A glass of wine just before pumping will help reduce stress and anxiety. c. The mother should pump only as much as the infant can drink. d. The mother should pump every 2 to 3 hours, including during the night.

ANS: A Human milk is the ideal food for preterm infants, with benefits that are unique in addition to those received by term, healthy infants. Greater physiologic stability occurs with breastfeeding compared with formula feeding. Consumption of alcohol during lactation is approached with caution. Excessive amounts can have serious effects on the infant and can adversely affect the mothers milk ejection reflex. To establish an optimal milk supply, the mother should be instructed to pump 8 to 10 times a day for 10 to 15 minutes on each breast. PTS: 1 DIF: Cognitive Level: Analysis REF: 648 OBJ: Nursing Process: Evaluation MSC: Client Needs: Health Promotion and Maintenance

Which term is used to describe a type of fracture that does not produce a break in the skin? a. Simple b. Compound c. Complicated d. Comminuted

ANS: A If a fracture does not produce a break in the skin, it is called a simple, or closed, fracture. A compound, or open, fracture is one with an open wound through which the bone protrudes. A complicated fracture is one in which the bone fragments damage other organs or tissues. A comminuted fracture occurs when small fragments of bone are broken from the fractured shaft and lie in the surrounding tissue. These are rare in children.

What PPH conditions are considered medical emergencies that require immediate treatment? a. Inversion of the uterus and hypovolemic shock b. Hypotonic uterus and coagulopathies c. Subinvolution of the uterus and idiopathic thrombocytopenic purpura d. Uterine atony and disseminated intravascular coagulation

ANS: A Inversion of the uterus and hypovolemic shock are considered medical emergencies. Although hypotonic uterus and coagulopathies, subinvolution of the uterus and idiopathic thrombocytopenic purpura, and uterine atony and disseminated intravascular coagulation are serious conditions, they are not necessarily medical emergencies that require immediate treatment. PTS: 1 DIF: Cognitive Level: Comprehension REF: 534 OBJ: Nursing Process: Implementation MSC: Client Needs: Physiologic Integrity

Which infant would be more likely to have Rh incompatibility? a. Infant of an Rh-negative mother and a father who is Rh positive and homozygous for the Rh factor b. Infant who is Rh negative and whose mother is Rh negative c. Infant of an Rh-negative mother and a father who is Rh positive and heterozygous for the Rh factor d. Infant who is Rh positive and whose mother is Rh positive

ANS: A If the mother is Rh negative and the father is Rh positive and homozygous for the Rh factor, all the children will be Rh positive. Only Rh-positive children of an Rh-negative mother are at risk for Rh incompatibility. If the mother is Rh negative and the father is Rh positive and heterozygous for the factor, there is a 50% chance that each infant born of the union will be Rh positive and a 50% chance that each will be born Rh negative. PTS: 1 DIF: Cognitive Level: Comprehension REF: 679 OBJ: Nursing Process: Planning MSC: Client Needs: Health Promotion and Maintenance

The nurse is discussing storage of breast milk with a mother whose infant is preterm and in the special care unit. What statement would indicate that the mother needs additional teaching? a. I can store my breast milk in the refrigerator for 3 months. b. I can store my breast milk in the freezer for 3 months. c. I can store my breast milk at room temperature for 8 hours. d. I can store my breast milk in the refrigerator for 3 to 5 days.

ANS: A If the mother states that she can store her breast milk in the refrigerator for 3 months, she needs additional teaching about safe storage. Breast milk can be stored at room temperature for 8 hours, in the refrigerator for 3 to 5 days, in the freezer for 3 months, or in a deep freezer for 6 to 12 months. It is accurate and does not require additional teaching if the mother states that she can store her breast milk in the freezer for 3 months, at room temperature for 8 hours, and in the refrigerator for 3 to 5 days. PTS: 1 DIF: Cognitive Level: Analysis REF: 650 OBJ: Nursing Process: Evaluation MSC: Client Needs: Health Promotion and Maintenance

With regard to injuries to the infants plexus during labor and birth, nurses should be aware that: a. If the nerves are stretched with no avulsion, they should recover completely in 3 to 6 months. b. Erb palsy is damage to the lower plexus. c. Parents of children with brachial palsy are taught to pick up the child from under the axillae. d. Breastfeeding is not recommended for infants with facial nerve paralysis until the condition resolves.

ANS: A If the nerves are stretched with no avulsion, they should recover completely in 3 to 6 months. However, if the ganglia are disconnected completely from the spinal cord, the damage is permanent. Erb palsy is damage to the upper plexus and is less serious than brachial palsy. Parents of children with brachial palsy are taught to avoid picking up the child under the axillae or by pulling on the arms. Breastfeeding is not contraindicated, but both the mother and infant will need help from the nurse at the start. PTS: 1 DIF: Cognitive Level: Comprehension REF: 665 OBJ: Nursing Process: Planning MSC: Client Needs: Physiologic Integrity

In the recovery room, if a woman is asked either to raise her legs (knees extended) off the bed or to flex her knees, place her feet flat on the bed, and raise her buttocks well off the bed, most likely she is being tested to see whether she: a. Has recovered from epidural or spinal anesthesia. b. Has hidden bleeding underneath her. c. Has regained some flexibility. d. Is a candidate to go home after 6 hours.

ANS: A If the numb or prickly sensations are gone from her legs after these movements, she has likely recovered from the epidural or spinal anesthesia. PTS: 1 DIF: Cognitive Level: Comprehension REF: 500 OBJ: Nursing Process: Evaluation MSC: Client Needs: Physiologic Integrity

42. A nurse is conducting an in-service on asthma. Which statement is the most descriptive of bronchial asthma? a. There is heightened airway reactivity. b. There is decreased resistance in the airway. c. The single cause of asthma is an allergic hypersensitivity. d. It is inherited

ANS: A In bronchial asthma, spasm of the smooth muscle of the bronchi and bronchioles causes constriction, producing impaired respiratory function. In bronchial asthma, there is increased resistance in the airway. There are multiple causes of asthma, including allergens, irritants, exercise, cold air, infections, medications, medical conditions, and endocrine factors. Atopy or development of an immunoglobulin E (IgE)mediated response is inherited but is not the only cause of asthma.

Part of the health assessment of a newborn is observing the infants breathing pattern. A full-term newborns breathing pattern is predominantly: a. Abdominal with synchronous chest movements. b. Chest breathing with nasal flaring. c. Diaphragmatic with chest retraction. d. Deep with a regular rhythm.

ANS: A In normal infant respiration the chest and abdomen rise synchronously, and breaths are shallow and irregular. Breathing with nasal flaring is a sign of respiratory distress. Diaphragmatic breathing with chest retraction is a sign of respiratory distress. Infant breaths are not deep with a regular rhythm. PTS: 1 DIF: Cognitive Level: Comprehension REF: 557 OBJ: Nursing Process: Assessment MSC: Client Needs: Physiologic Integrity

The mother-baby nurse is able to recognize reciprocal attachment behavior. This refers to: a. The positive feedback an infant exhibits toward parents during the attachment process. b. Behavior during the sensitive period when the infant is in the quiet alert stage. c. Unidirectional behavior exhibited by the infant, initiated and enhanced by eye contact. d. Behavior by the infant during the sensitive period to elicit feelings of falling in love from the parents.

ANS: A In this definition, reciprocal refers to the feedback from the infant during the attachment process. This is a good time for bonding; however, it does not define reciprocal attachment. Reciprocal attachment applies to feedback behavior and is not unidirectional. PTS: 1 DIF: Cognitive Level: Application REF: 518 OBJ: Nursing Process: Assessment MSC: Client Needs: Health Promotion and Maintenance

The nurse can help a father in his transition to parenthood by: a. Pointing out that the infant turned at the sound of his voice. b. Encouraging him to go home to get some sleep. c. Telling him to tape the infants diaper a different way. d. Suggesting that he let the infant sleep in the bassinet.

ANS: A Infants respond to the sound of voices. Because attachment involves a reciprocal interchange, observing the interaction between parent and infant is very important. Separation of the parent and infant does not encourage parent-infant attachment. Educating the parent in infant care techniques is important; however, the manner in which a diaper is taped is not relevant and does not enhance parent-infant interactions. Parent-infant attachment involves touching, holding, and cuddling. It is appropriate for a father to want to hold the infant as the baby sleeps. PTS: 1 DIF: Cognitive Level: Application REF: 515 OBJ: Nursing Process: Implementation MSC: Client Needs: Health Promotion and Maintenance

The perinatal nurse caring for the postpartum woman understands that late postpartum hemorrhage (PPH) is most likely caused by: a. Subinvolution of the placental site. b. Defective vascularity of the decidua. c. Cervical lacerations. d. Coagulation disorders.

ANS: A Late PPH may be the result of subinvolution of the uterus, pelvic infection, or retained placental fragments. Late PPH is not typically a result of defective vascularity of the decidua, cervical lacerations, or coagulation disorders. PTS: 1 DIF: Cognitive Level: Comprehension REF: 531 OBJ: Nursing Process: Planning MSC: Client Needs: Physiologic Integrity

A primiparous woman is to be discharged from the hospital tomorrow with her infant girl. Which behavior indicates a need for further intervention by the nurse before the woman can be discharged? a. The woman leaves the infant on her bed while she takes a shower. b. The woman continues to hold and cuddle her infant after she has fed her. c. The woman reads a magazine while her infant sleeps. d. The woman changes her infants diaper and then shows the nurse the contents of the diaper.

ANS: A Leaving an infant on a bed unattended is never acceptable for various safety reasons. Holding and cuddling the infant after feeding and reading a magazine while the infant sleeps are appropriate parent-infant interactions. Changing the diaper and then showing the nurse the contents of the diaper is appropriate because the mother is seeking approval from the nurse and notifying the nurse of the infants elimination patterns. PTS: 1 DIF: Cognitive Level: Comprehension REF: 493 OBJ: Nursing Process: Evaluation MSC: Client Needs: Health Promotion and Maintenance

An infant at 26 weeks of gestation arrives intubated from the delivery room. The nurse weighs the infant, places him under the radiant warmer, and attaches him to the ventilator at the prescribed settings. A pulse oximeter and cardiorespiratory monitor are placed. The pulse oximeter is recording oxygen saturations of 80%. The prescribed saturations are 92%. The nurses most appropriate action would be to: a. Listen to breath sounds and ensure the patency of the endotracheal tube, increase oxygen, and notify a physician. b. Continue to observe and make no changes until the saturations are 75%. c. Continue with the admission process to ensure that a thorough assessment is completed. d. Notify the parents that their infant is not doing well.

ANS: A Listening to breath sounds and ensuring the patency of the endotracheal tube, increasing oxygen, and notifying a physician are appropriate nursing interventions to assist in optimal oxygen saturation of the infant. Oxygenation of the infant is crucial. O2 saturation should be maintained above 92%. Oxygenation status of the infant is crucial. The nurse should delay other tasks to stabilize the infant. Notifying the parents that the infant is not doing well is not an appropriate action. Further assessment and intervention are warranted before determination of fetal status. PTS: 1 DIF: Cognitive Level: Application REF: 688 OBJ: Nursing Process: Implementation MSC: Client Needs: Physiologic Integrity

The nurse is preparing an adolescent with scoliosis for a Luque-rod segmental spinal instrumentation procedure. Which consideration should the nurse include? a. Nasogastric intubation and urinary catheter may be required. b. Ambulation will not be allowed for up to 3 months. c. Surgery eliminates the need for casting and bracing. d. Discomfort can be controlled with nonpharmacologic methods.

ANS: A Luque-rod segmental spinal instrumentation is a surgical procedure. Nasogastric intubation and urinary catheterization may be required. Ambulation is allowed as soon as possible. Depending on the instrumentation used, most patients walk by the second or third postoperative day. Casting and bracing are required postoperatively. The child usually has considerable pain for the first few days after surgery. Intravenous opioids should be administered on a regular basis.

Nurses can assist parents who are trying to decide whether their son should be circumcised by explaining: a. The pros and cons of the procedure during the prenatal period. b. That the American Academy of Pediatrics (AAP) recommends that all newborn boys be routinely circumcised. c. That circumcision is rarely painful and any discomfort can be managed without medication. d. That the infant will likely be alert and hungry shortly after the procedure.

ANS: A Many parents find themselves making the decision during the pressure of labor. The AAP and other professional organizations note the benefits but stop short of recommendation for routine circumcision. Circumcision is painful and must be managed with environmental, nonpharmacologic, and pharmacologic measures. After the procedure the infant may be fussy for several hours, or he may be sleepy and difficult to awaken for feeding. PTS: 1 DIF: Cognitive Level: Comprehension REF: 615 OBJ: Nursing Process: Planning MSC: Client Needs: Health Promotion and Maintenance

A pregnant woman was admitted for induction of labor at 43 weeks of gestation with sure dates. A nonstress test (NST) in the obstetricians office revealed a nonreactive tracing. On artificial rupture of membranes, thick, meconium-stained fluid was noted. The nurse caring for the infant after birth should anticipate: a. Meconium aspiration, hypoglycemia, and dry, cracked skin. b. Excessive vernix caseosa covering the skin, lethargy, and respiratory distress syndrome. c. Golden yellow- to green stainedskin and nails, absence of scalp hair, and an increased amount of subcutaneous fat. d. Hyperglycemia, hyperthermia, and an alert, wide-eyed appearance.

ANS: A Meconium aspiration, hypoglycemia, and dry, cracked skin are consistent with a postmature infant. Excessive vernix caseosa covering the skin, lethargy, and respiratory distress syndrome would be consistent with a very premature infant. The skin may be meconium stained, but the infant would most likely have longer hair and decreased amounts of subcutaneous fat. Postmaturity with a nonreactive NST would indicate hypoxia. Signs and symptoms associated with fetal hypoxia are hypoglycemia, temperature instability, and lethargy. PTS: 1 DIF: Cognitive Level: Analysis REF: 685 OBJ: Nursing Process: Planning MSC: Client Needs: Physiologic Integrity

When the infants behaviors and characteristics call forth a corresponding set of maternal behaviors and characteristics, this is called: a. Mutuality. b. Bonding. c. Claiming. d. Acquaintance.

ANS: A Mutuality extends the concept of attachment to include this shared set of behaviors. Bonding is the process over time of parents forming an emotional attachment to their infant. Mutuality refers to a shared set of behaviors that is a part of the bonding process. Claiming is the process by which parents identify their new baby in terms of likeness to other family members and their differences and uniqueness. Like mutuality, acquaintance is part of attachment. It describes how parents get to know their baby during the immediate postpartum period through eye contact, touching, and talking. PTS: 1 DIF: Cognitive Level: Knowledge REF: 510 OBJ: Nursing Process: Evaluation MSC: Client Needs: Psychosocial Integrity

The nurse is teaching nursing students about childhood nervous system tumors. Which best describes a neuroblastoma? a. Diagnosis is usually made after metastasis occurs. b. Early diagnosis is usually possible because of the obvious clinical manifestations. c. It is the most common brain tumor in young children. d. It is the most common benign tumor in young children.

ANS: A Neuroblastoma is a silent tumor with few symptoms. In more than 70% of cases, diagnosis is made after metastasis occurs, with the first signs caused by involvement in the nonprimary site. In only 30% of cases is diagnosis made before metastasis. Neuroblastomas are the most common malignant extracranial solid tumors in children. The majority of tumors develop in the adrenal glands or the retroperitoneal sympathetic chain. They are not benign but metastasize.

The nurse providing couplet care should understand that nipple confusion results when: a. Breastfeeding babies receive supplementary bottle feedings. b. The baby is weaned too abruptly. c. Pacifiers are used before breastfeeding is established. d. Twins are breastfed together.

ANS: A Nipple confusion can result when babies go back and forth between bottles and breasts, especially before breastfeeding is established in 3 to 4 weeks, because the two require different skills. Abrupt weaning can be distressing to mother and/or baby but should not lead to nipple confusion. Pacifiers used before breastfeeding is established can be disruptive, but this does not lead to nipple confusion. Breastfeeding twins requires some logistical adaptations, but this should not lead to nipple confusion. PTS: 1 DIF: Cognitive Level: Comprehension REF: 646 OBJ: Nursing Process: Planning MSC: Client Needs: Health Promotion and Maintenance

5. Which statement characterizes moral development in older school-age children? a. They are able to judge an act by the intentions that prompted it rather than just by the consequences. b. Rules and judgments become more absolute and authoritarian. c. They view rule violations in an isolated context. d. They know the rules but cannot understand the reasons behind them.

ANS: A Older school-age children are able to judge an act by the intentions that prompted the behavior rather than just by the consequences. Rules and judgments become less absolute and authoritarian. Rule violation is likely to be viewed in relation to the total context in which it appears. Both the situation and the morality of the rule itself influence reactions.

A nurse is conducting a staff in-service on childhood cancers. Which is the primary site of osteosarcoma? a. Femur b. Humerus c. Pelvis d. Tibia

ANS: A Osteosarcoma is the most frequently encountered malignant bone cancer in children. The peak incidence is between ages 10 and 25 years. More than half occur in the femur. After the femur, most of the remaining sites are the humerus, tibia, pelvis, jaw, and phalanges.

A 3.8-kg infant was delivered vaginally at 39 weeks after a 30-minute second stage. There was a nuchal cord. After birth the infant is noted to have petechiae over the face and upper back. Information given to the infants parents should be based on the knowledge that petechiae: a. Are benign if they disappear within 48 hours of birth. b. Result from increased blood volume. c. Should always be further investigated. d. Usually occur with forceps delivery.

ANS: A Petechiae, or pinpoint hemorrhagic areas, acquired during birth may extend over the upper portion of the trunk and face. These lesions are benign if they disappear within 2 days of birth and no new lesions appear. Petechiae may result from decreased platelet formation. In this situation the presence of petechiae is most likely a soft-tissue injury resulting from the nuchal cord at birth. Unless they do not dissipate in 2 days, there is no reason to alarm the family. Petechiae usually occur with a breech presentation vaginal birth. PTS: 1 DIF: Cognitive Level: Application REF: 605 OBJ: Nursing Process: Assessment MSC: Client Needs: Health Promotion and Maintenance

According to Becks studies, what risk factor for postpartum depression is likely to have the greatest effect on the womans condition? a. Prenatal depression b. Single-mother status c. Low socioeconomic status d. Unplanned or unwanted pregnancy

ANS: A Prenatal depression has been found by Beck to have the greatest likely effect. Single-mother status and low socioeconomic status are small-relation predictors, as is an unwanted pregnancy. PTS: 1 DIF: Cognitive Level: Knowledge REF: 549 OBJ: Nursing Process: Assessment MSC: Client Needs: Psychosocial Integrity

9. With regard to the association of maternal diabetes and other risk situations affecting mother and fetus, nurses should be aware that: A. Diabetic ketoacidosis (DKA) can lead to fetal death at any time during pregnancy. B. Hydramnios occurs approximately twice as often in diabetic pregnancies. C. Infections occur about as often and are considered about as serious in diabetic and nondiabetic pregnancies. D. Even mild to moderate hypoglycemic episodes can have significant effects on fetal well-being.

ANS: A Prompt treatment of DKA is necessary to save the fetus and the mother. Hydramnios occurs 10 times more often in diabetic pregnancies. Infections are more common and more serious in pregnant women with diabetes. Mild-to-moderate hypoglycemic episodes do not appear to have significant effects on fetal well-being.

14.What should the nurse include in a teaching plan for the parents of a child with vesicoureteral reflux? a. The importance of taking prophylactic antibiotics b. Suggestions for how to maintain fluid restrictions c. The use of bubble baths as an incentive to increase bath time d. The need for the child to hold urine for 6 to 8 hours

ANS: A Prophylactic antibiotics are used to prevent urinary tract infections (UTIs) in a child with vesicoureteral reflux, although this treatment plan has become controversial. Fluids are not restricted when a child has vesicoureteral reflux. In fact, fluid intake should be increased as a measure to prevent UTIs. Bubble baths should be avoided to prevent urethral irritation and possible UTI. To prevent UTIs, the child should be taught to void frequently and never resist the urge to urinate.

Which is the priority nursing intervention for an unconscious child after a fall? a. Establish adequate airway. b. Perform neurologic assessment. c. Monitor intracranial pressure. d. Determine whether a neck injury is present.

ANS: A Respiratory effectiveness is the primary concern in the care of the unconscious child. Establishment of an adequate airway is always the first priority. A neurologic assessment and determination of whether a neck injury is present will be performed after breathing and circulation are stabilized. Intracranial, not intercranial, pressure is monitored if indicated after airway, breathing, and circulation are maintained.

A hospital has a number of different perineal pads available for use. A nurse is observed soaking several of them and writing down what she sees. This activity indicates that the nurse is trying to: a. Improve the accuracy of blood loss estimation, which usually is a subjective assessment. b. Determine which pad is best. c. Demonstrate that other nurses usually underestimate blood loss. d. Reveal to the nurse supervisor that one of them needs some time off.

ANS: A Saturation of perineal pads is a critical indicator of excessive blood loss, and anything done to aid in assessment is valuable. The nurse is noting the saturation volumes and soaking appearances. It is possible that the nurse is trying to determine which pad is best, but it is more likely that the nurse is noting saturation volumes and soaking appearances to improve the accuracy of blood loss estimation. Nurses usually overestimate blood loss, if anything. PTS: 1 DIF: Cognitive Level: Application REF: 497 OBJ: Nursing Process: Assessment MSC: Client Needs: Physiologic Integrity

A young girl has just injured her ankle at school. In addition to calling the child's parents, the most appropriate, immediate action by the school nurse is to: a. apply ice. b. observe for edema and discoloration. c. encourage child to assume a position of comfort. d. obtain parental permission for administration of acetaminophen or aspirin.

ANS: A Soft-tissue injuries should be iced immediately. In addition to ice, the extremity should be rested, be elevated, and have compression applied. Observing for edema and discoloration, encouraging the child to assume a position of comfort, and obtaining parental permission for administration of acetaminophen or aspirin are not immediate priorities. The application of ice can reduce the severity of the injury.

A premature infant with respiratory distress syndrome receives artificial surfactant. How would the nurse explain surfactant therapy to the parents? a. Surfactant improves the ability of your babys lungs to exchange oxygen and carbon dioxide. b. The drug keeps your baby from requiring too much sedation. c. Surfactant is used to reduce episodes of periodic apnea. d. Your baby needs this medication to fight a possible respiratory tract infection.

ANS: A Surfactant can be administered as an adjunct to oxygen and ventilation therapy. With administration of artificial surfactant, respiratory compliance is improved until the infant can generate enough surfactant on his or her own. Surfactant has no bearing on the sedation needs of the infant. Surfactant is used to improve respiratory compliance, including the exchange of oxygen and carbon dioxide. The goal of surfactant therapy in an infant with respiratory distress syndrome (RDS) is to stimulate production of surfactant in the type 2 cells of the alveoli. The clinical presentation of RDS and neonatal pneumonia may be similar. The infant may be started on broad-spectrum antibiotics to treat infection. PTS: 1 DIF: Cognitive Level: Application REF: 687 OBJ: Nursing Process: Planning MSC: Client Needs: Physiologic Integrity

30. The nurse is conducting an assessment on a school-age child with urosepsis. Which assessment finding should the nurse expect? a. Fever with a positive blood culture b. Proteinuria and edema c. Oliguria and hypertension d. Anemia and thrombocytopenia

ANS: A Symptoms of urosepsis include a febrile urinary tract infection coexisting with systemic signs of bacterial illness; blood culture reveals the presence of a urinary pathogen. Proteinuria and edema are symptoms of minimal change nephrotic syndrome. Oliguria and hypertension are symptoms of acute glomerulonephritis. Anemia and thrombocytopenia are symptoms of hemolytic uremic syndrome.

A first-time father is changing the diaper of his 1-day-old daughter. He asks the nurse, What is this black, sticky stuff in her diaper? The nurses best response is: a. Thats meconium, which is your babys first stool. Its normal. b. Thats transitional stool. c. That means your baby is bleeding internally. d. Oh, dont worry about that. Its okay.

ANS: A Thats meconium, which is your babys first stool. Its normal is an accurate statement and the most appropriate response. Transitional stool is greenish brown to yellowish brown and usually appears by the third day after initiation of feeding. That means your baby is bleeding internally is not accurate. Oh, dont worry about that. Its okay is not an appropriate statement. It is belittling to the father and does not educate him about the normal stool patterns of his daughter. PTS: 1 DIF: Cognitive Level: Application REF: 563 OBJ: Nursing Process: Implementation MSC: Client Needs: Health Promotion and Maintenance

A 10-year-old child, without a history of previous seizures, experiences a tonic-clonic seizure at school. Breathing is not impaired, but some postictal confusion occurs. The most appropriate initial action by the school nurse is to: a. stay with child and have someone call emergency medical service (EMS). b. notify parent and regular practitioner. c. notify parent that child should go home. d. stay with child, offering calm reassurance.

ANS: A The EMS should be called to transport the child because this is the child's first seizure. Because this is the first seizure, evaluation should be performed as soon as possible. The nurse should stay with the child while someone else notifies the EMS.

Four-year-old David is placed in Buck extension traction for Legg-Calvé-Perthes disease. He is crying with pain as the nurse assesses that the skin of his right foot is pale with an absence of pulse. Which action should the nurse take first? a. Notify the practitioner of the changes noted. b. Give the child medication to relieve the pain. c. Reposition the child and notify physician. d. Chart the observations and check the extremity again in 15 minutes.

ANS: A The absence of a pulse and change in color of the foot must be reported immediately for evaluation by the practitioner. Pain medication should be given after the practitioner is notified. Legg-Calvé-Perthes disease is an emergency condition; immediate reporting is indicated. The findings should be documented with ongoing assessment.

Early postpartum hemorrhage is defined as a blood loss greater than: a. 500 mL in the first 24 hours after vaginal delivery. b. 750 mL in the first 24 hours after vaginal delivery. c. 1000 mL in the first 48 hours after cesarean delivery. d. 1500 mL in the first 48 hours after cesarean delivery.

ANS: A The average amount of bleeding after a vaginal birth is 500 mL. Blood loss after a cesarean birth averages 1000 mL. Early postpartum hemorrhage occurs in the first 24 hours, not 48 hours. Late postpartum hemorrhage is 48 hours and later. PTS: 1 DIF: Cognitive Level: Knowledge REF: 530 OBJ: Nursing Process: Assessment MSC: Client Needs: Physiologic Integrity

5. A child will start treatment for precocious puberty. This involves injections of which synthetic medication? a. Thyrotropin b. Gonadotropins c. Somatotropic hormone d. Luteinizing hormone-releasing hormone

ANS: D Precocious puberty of central origin is treated with monthly subcutaneous injections of luteinizing hormone-releasing hormone. Thyrotropin, gonadotropin, and somatotropic hormone are not appropriate therapies for precocious puberty.

. A toddler fell out of a second-story window. She had a brief loss of consciousness and vomited four times. Since admission, she has been alert and oriented. Her mother asks why a computed tomography (CT) scan is required when she "seems fine." Which explanation should the nurse give? a. Your child may have a brain injury and the CT can rule one out. b. The CT needs to be done because of your child's age. c. Your child may start to have seizures and a baseline CT should be done. d. Your child probably has a skull fracture and the CT can confirm this diagnosis.

ANS: A The child's history of the fall, brief loss of consciousness, and vomiting four times necessitates evaluation of a potential brain injury. The severity of a head injury may not be apparent on clinical examination but will be detectable on a CT scan. The need for the CT scan is related to the injury and symptoms, not the child's age. The CT scan is necessary to determine whether a brain injury has occurred.

With regard to umbilical cord care, nurses should be aware that: a. The stump can easily become infected. b. A nurse noting bleeding from the vessels of the cord should immediately call for assistance. c. The cord clamp is removed at cord separation. d. The average cord separation time is 5 to 7 days.

ANS: A The cord stump is an excellent medium for bacterial growth. The nurse should first check the clamp (or tie) and apply a second one. If the bleeding does not stop, the nurse calls for assistance. The cord clamp is removed after 24 hours when it is dry. The average cord separation time is 10 to 14 days. PTS: 1 DIF: Cognitive Level: Comprehension REF: 627 OBJ: Nursing Process: Planning MSC: Client Needs: Safe and Effective Care Environment

A pregnant woman presents in labor at term, having had no prenatal care. After birth her infant is noted to be small for gestational age with small eyes and a thin upper lip. The infant also is microcephalic. On the basis of her infants physical findings, this woman should be questioned about her use of which substance during pregnancy? a. Alcohol b. Cocaine c. Heroin d. Marijuana

ANS: A The description of the infant suggests fetal alcohol syndrome, which is consistent with maternal alcohol consumption during pregnancy. Fetal brain, kidney, and urogenital system malformations have been associated with maternal cocaine ingestions. Heroin use in pregnancy frequently results in intrauterine growth restriction. The infant may have a shrill cry and sleep cycle disturbances and present with poor feeding, tachypnea, vomiting, diarrhea, hypothermia or hyperthermia, and sweating. Studies have found a higher incidence of meconium staining in infants born of mothers who used marijuana during pregnancy. PTS: 1 DIF: Cognitive Level: Comprehension REF: 670 OBJ: Nursing Process: Assessment MSC: Client Needs: Physiologic Integrity

Which breastfeeding patient is most likely to have severe afterbirth pains and request a narcotic analgesic? a. Gravida 5, para 5 b. Woman who is bottle-feeding her first child c. Primipara who delivered a 7-lb boy d. Woman who wishes to breastfeed as soon as her baby is out of the neonatal intensive care unit

ANS: A The discomfort of afterpains is more acute for multiparas because repeated stretching of muscle fibers leads to loss of uterine muscle tone. Afterpains are particularly severe during breastfeeding, not bottle-feeding. The uterus of a primipara tends to remain contracted. The nonnursing mother may have engorgement problems. The patient whose infant is in the NICU should pump regularly to stimulate milk production and ensure that she will have an adequate milk supply when the baby is strong enough to nurse. PTS: 1 DIF: Cognitive Level: Comprehension REF: 494 OBJ: Nursing Process: Assessment MSC: Client Needs: Physiologic Integrity

24. The earliest recognizable clinical manifestation of cystic fibrosis (CF) is: a. Meconium ileus. b. History of poor intestinal absorption. c. Foul-smelling, frothy, greasy stools. d. Recurrent pneumonia and lung infections.

ANS: A The earliest clinical manifestation of CF is a meconium ileus, which is found in about 10% of children with CF. Clinical manifestations include abdominal distention, vomiting, failure to pass stools, and rapid development of dehydration. History of malabsorption is a later sign that manifests as failure to thrive. Foulsmelling stools and recurrent respiratory infections are later manifestations of CF.

With regard to the respiratory development of the newborn, nurses should be aware that: a. The first gasping breath is an exaggerated respiratory reaction within 1 minute of birth. b. Newborns must expel the fluid from the respiratory system within a few minutes of birth. c. Newborns are instinctive mouth breathers. d. Seesaw respirations are no cause for concern in the first hour after birth.

ANS: A The first breath produces a cry. Newborns continue to expel fluid for the first hour of life. Newborns are natural nose breathers; they may not have the mouth-breathing response to nasal blockage for 3 weeks. Seesaw respirations instead of normal abdominal respirations are not normal and should be reported. PTS: 1 DIF: Cognitive Level: Comprehension REF: 558 OBJ: Nursing Process: Planning MSC: Client Needs: Physiologic Integrity

Which finding 12 hours after birth requires further assessment? a. The fundus is palpable two fingerbreadths above the umbilicus. b. The fundus is palpable at the level of the umbilicus. c. The fundus is palpable one fingerbreadth below the umbilicus. d. The fundus is palpable two fingerbreadths below the umbilicus.

ANS: A The fundus rises to the umbilicus after delivery and remains there for about 24 hours. A fundus that is above the umbilicus may indicate uterine atony or urinary retention. A fundus that is palpable at or below the level of the umbilicus is a normal finding for a patient who is 12 hours postpartum. Palpation of the fundus 2 fingerbreadths below the umbilicus is an unusual finding for 12 hours postpartum; however, it is still appropriate. PTS: 1 DIF: Cognitive Level: Application REF: 484 OBJ: Nursing Process: Assessment MSC: Client Needs: Physiologic Integrity

When preparing to administer a hepatitis B vaccine to a newborn, the nurse should: a. Obtain a syringe with a 25-gauge, 5/8-inch needle. b. Confirm that the newborns mother has been infected with the hepatitis B virus. c. Assess the dorsogluteal muscle as the preferred site for injection. d. Confirm that the newborn is at least 24 hours old.

ANS: A The hepatitis B vaccine should be administered with a 25-gauge, 5/8-inch needle. Hepatitis B vaccination is recommended for all infants. If the infant is born to an infected mother who is a chronic carrier, hepatitis vaccine and hepatitis B immune globulin should be administered within 12 hours of birth. Hepatitis B vaccine should be given in the vastus lateralis muscle. Hepatitis B vaccine can be given at birth. PTS: 1 DIF: Cognitive Level: Application REF: 613 OBJ: Nursing Process: Implementation MSC: Client Needs: Health Promotion and Maintenance

Nursing follow-up care often includes home visits for the new mother and her infant. Which information related to home visits is correct? a. Ideally, the visit is scheduled within 72 hours after discharge. b. Home visits are available in all areas. c. Visits are completed within a 30-minute time frame. d. Blood draws are not a part of the home visit.

ANS: A The home visit is ideally scheduled within 72 hours after discharge. This timing allows early assessment and intervention for problems with feedings, jaundice, newborn adaptation, and maternal-infant interaction. Because home visits are expensive, they are not available in all geographic areas. Visits are usually 60 to 90 minutes in length to allow enough time for assessment and teaching. When jaundice is found, the nurse can discuss the implications and check the transcutaneous bilirubin level or draw blood for testing. PTS: 1 DIF: Cognitive Level: Application REF: 627 OBJ: Nursing Process: Planning MSC: Client Needs: Health Promotion and Maintenance

Several changes in the integumentary system that appear during pregnancy disappear after birth, although not always completely. What change is almost certain to be completely reversed? a. Nail brittleness b. Darker pigmentation of the areolae and linea nigra c. Striae gravidarum on the breasts, abdomen, & thighs d. Spider nevi

ANS: A The nails return to their prepregnancy consistency and strength. Some women have permanent darker pigmentation of the areolae and linea nigra. Striae gravidarum (stretch marks) usually do not completely disappear. For some women spider nevi persist indefinitely. PTS: 1 DIF: Cognitive Level: Comprehension REF: 489 OBJ: Nursing Process: Planning MSC: Client Needs: Physiologic Integrity

The nurse notes that a Vietnamese woman does not cuddle or interact with her newborn other than to feed him, change his diapers or soiled clothes, and put him to bed. In evaluating the womans behavior with her infant, the nurse realizes that: a. What appears to be a lack of interest in the newborn is in fact the Vietnamese way of demonstrating intense love by attempting to ward off evil spirits. b. The woman is inexperienced in caring for newborns. c. The woman needs a referral to a social worker for further evaluation of her parenting behaviors once she goes home with the newborn. d. Extra time needs to be planned for assisting the woman in bonding with her newborn.

ANS: A The nurse may observe a Vietnamese woman who gives minimal care to her infant and refuses to cuddle or interact with her infant. The apparent lack of interest in the newborn is this cultural groups attempt to ward off evil spirits and actually reflects an intense love and concern for the infant. It is important to educate the woman in infant care, but it is equally important to acknowledge her cultural beliefs and practices. PTS: 1 DIF: Cognitive Level: Comprehension REF: 523 OBJ: Nursing Process: Evaluation MSC: Client Needs: Psychosocial Integrity

In the assessment of a preterm infant, the nurse notices continued respiratory distress even though oxygen and ventilation have been provided. The nurse should suspect: a. Hypovolemia and/or shock. b. A nonneutral thermal environment. c. Central nervous system injury. d. Pending renal failure.

ANS: A The nurse should suspect hypovolemia and/or shock. Other symptoms could include hypotension, prolonged capillary refill, and tachycardia followed by bradycardia. Intervention is necessary. PTS: 1 DIF: Cognitive Level: Application REF: 680 OBJ: Nursing Process: Diagnosis MSC: Client Needs: Physiologic Integrity

The nurse is taking care of a child who is alert but showing signs of increased intracranial pressure. Which test is contraindicated in this case? a. Oculovestibular response b. Doll's head maneuver c. Funduscopic examination for papilledema d. Assessment of pyramidal tract lesions

ANS: A The oculovestibular response (caloric test) involves the instillation of ice water into the ear of a comatose child. The caloric test is painful and is never performed on an awake child or one who has a ruptured tympanic membrane. Doll's head maneuver, funduscopic examination for papilledema, and assessment of pyramidal tract lesions can be performed on awake children.

The nurse is taking care of an adolescent with osteosarcoma. The parents ask the nurse about treatment. The nurse should make which accurate response about treatment for osteosarcoma? a. Treatment usually consists of surgery and chemotherapy. b. Amputation of affected extremity is rarely necessary. c. Intensive irradiation is the primary treatment. d. Bone marrow transplantation offers the best chance of long-term survival.

ANS: A The optimal therapy for osteosarcoma is a combination of surgery and chemotherapy. Intensive irradiation and bone marrow transplantation are usually not part of the therapeutic management.

The nurse should monitor for which effect on the cardiovascular system when a child is immobilized? a. Venous stasis b. Increased vasopressor mechanism c. Normal distribution of blood volume d. Increased efficiency of orthostatic neurovascular reflexes

ANS: A The physiologic effects of immobilization, as a result of decreased muscle contraction, include venous stasis. This can lead to pulmonary emboli or thrombi. A decreased vasopressor mechanism results in orthostatic hypotension, syncope, hypotension, decreased cerebral blood flow, and tachycardia. An altered distribution of blood volume is found with decreased cardiac workload and exercise tolerance. Immobilization causes a decreased efficiency of orthostatic neurovascular reflexes with an inability to adapt readily to the upright position and with pooling of blood in the extremities in the upright position.

16.The primary clinical manifestations of acute renal failure are: a. Oliguria and hypertension. b. Hematuria and pallor. c. Proteinuria and muscle cramps. d. Bacteriuria and facial edema.

ANS: A The principal feature of acute renal failure is oliguria. Hematuria and pallor, proteinuria and muscle cramps, and bacteriuria and facial edema are not principal features of acute renal failure

1.Which diagnostic test allows visualization of the renal parenchyma and renal pelvis without exposure to external beam radiation or radioactive isotopes? a. Renal ultrasound b. Computed tomography c. Intravenous pyelography d. Voiding cystourethrography

ANS: A The transmission of ultrasonic waves through the renal parenchyma allows visualization of the renal parenchyma and renal pelvis without exposure to external beam radiation or radioactive isotopes. Computed tomography uses external radiation, and sometimes contrast media are used. Intravenous pyelography uses contrast medium and external radiation for x-ray films. Contrast medium is injected into the bladder through the urethral opening for voiding cystourethrography. External radiation for x-ray films is used before, during, and after voiding.

Nurses providing nutritional instruction should be cognizant of the uniqueness of human milk. Which statement is correct? a. Frequent feedings during predictable growth spurts stimulate increased milk production. b. The milk of preterm mothers is the same as the milk of mothers who gave birth at term. c. The milk at the beginning of the feeding is the same as the milk at the end of the feeding. d. Colostrum is an early, less concentrated, less rich version of mature milk.

ANS: A These growth spurts (10 days, 3 weeks, 6 weeks, 3 months) usually last 24 to 48 hours, after which infants resume normal feeding. The milk of mothers of preterm infants is different from that of mothers of full-term infants to meet the needs of these newborns. Milk changes composition during feeding. The fat content of the milk increases as the infant feeds. Colostrum precedes mature milk and is more concentrated and richer in proteins and minerals (but not fat). PTS: 1 DIF: Cognitive Level: Comprehension REF: 640 OBJ: Nursing Process: Planning MSC: Client Needs: Health Promotion and Maintenance

14. A 4-year-old child is several days postoperative after a resection of a brain tumor. The nurse finds the child irritable and lethargic, and notes that she has vomited. Which medication does the nurse anticipate administering? A. Dexamethasone (Decadron) B. Fosphenytoin (Cerebyx) C. Odansetron (Zofran) D. Phenytoin (Dilantin)

ANS: A This child has manifestations of increased intracranial pressure, a possible outcome after brain surgery. The nurse prepares to administer a corticosteroid to decrease the edema. Fosphenytoin and phenytoin are for seizures. Odansetron is for nausea.

37. A child has the following laboratory values: WBC, 7.2 mm3; bands, 4%; and neutrophils, 60%. Based on these values, which action by the nurse is the most appropriate? A. Continue monitoring the child for infection. B. Place the child on protective isolation. C. Obtain two sets of blood cultures. D. Restrict visitors to the child.

ANS: A This child's absolute neutrophil count is 4,608; therefore, the child is not neutropenic. The nurse should continue to monitor. The other actions are not necessary.

The laboratory results for a postpartum woman are as follows: blood type, A; Rh status, positive; rubella titer, 1:8 (EIA 0.8); hematocrit, 30%. How would the nurse best interpret these data? a. Rubella vaccine should be given. b. A blood transfusion is necessary. c. Rh immune globulin is necessary within 72 hours of birth. d. A Kleihauer-Betke test should be performed.

ANS: A This clients rubella titer indicates that she is not immune and that she needs to receive a vaccine. These data do not indicate that the client needs a blood transfusion. Rh immune globulin is indicated only if the client has a negative Rh status and the infant has a positive Rh status. A Kleihauer-Betke test should be performed if a large fetomaternal transfusion is suspected, especially if the mother is Rh negative. The data do not provide any indication for performing this test. PTS: 1 DIF: Cognitive Level: Comprehension REF: 503 OBJ: Nursing Process: Planning MSC: Client Needs: Health Promotion and Maintenance

The cheeselike, whitish substance that fuses with the epidermis and serves as a protective coating is called: a. Vernix caseosa. b. Surfactant. c. Caput succedaneum. d. Acrocyanosis.

ANS: A This protection, vernix caseosa, is needed because the infants skin is so thin. Surfactant is a protein that lines the alveoli of the infants lungs. Caput succedaneum is the swelling of the tissue over the presenting part of the fetal head. Acrocyanosis is cyanosis of the hands and feet that results in a blue coloring. PTS: 1 DIF: Cognitive Level: Knowledge REF: 567 OBJ: Nursing Process: Assessment MSC: Client Needs: Health Promotion and Maintenance

In the United States the en face position is preferred immediately after birth. Nurses can facilitate this process by all of these actions except: a. Washing both the infants face and the mothers face. b. Placing the infant on the mothers abdomen or breast with their heads on the same plane. c. Dimming the lights. d. Delaying the instillation of prophylactic antibiotic ointment in the infants eyes.

ANS: A To facilitate the position in which the parents and infants faces are approximately 8 inches apart on the same plane, allowing them to make eye contact, the nurse can place the infant at the proper height on the mothers body, dim the light so that the infants eyes open, and delay putting ointment in the infants eyes. PTS: 1 DIF: Cognitive Level: Comprehension REF: 514 OBJ: Nursing Process: Implementation MSC: Client Needs: Health Promotion and Maintenance

35. Prior to administering IV chemotherapy, which action by the nurse is most important? A. Ensure the IV has a good blood return. B. Provide diversionary activities. C. Take and record a set of vital signs. D. Weigh the child.

ANS: A To prevent extravasation of IV chemotherapy it is important to make sure the line flushes easily and has a good blood return. This is a critical action to maintain patient safety. The other actions may also be utilized, but would not take priority over ensuring patient safety.

During a prenatal examination, the woman reports having two cats at home. The nurse informs her that she should not be cleaning the litter box while she is pregnant. When the woman asks why, the nurses best response would be: a. Your cats could be carrying toxoplasmosis. This is a zoonotic parasite that can infect you and have severe effects on your unborn child. b. You and your baby can be exposed to the human immunodeficiency virus (HIV) in your cats feces. c. Its just gross. You should make your husband clean the litter boxes. d. Cat feces are known to carry Escherichia coli, which can cause a severe infection in both you and your baby.

ANS: A Toxoplasmosis is a multisystem disease caused by the protozoal Toxoplasma gondii parasite, commonly found in cats, dogs, pigs, sheep, and cattle. About 30% of women who contract toxoplasmosis during gestation transmit the disease to their children. Clinical features ascribed to toxoplasmosis include hydrocephalus or microcephaly, chorioretinitis, seizures, or cerebral calcifications. HIV is not transmitted by cats. Although suggesting that the womans husband clean the litter boxes may be a valid statement, it is not appropriate, does not answer the clients question, and is not the nurses best response. E. coli is found in normal human fecal flora. It is not transmitted by cats. PTS: 1 DIF: Cognitive Level: Application REF: 670 OBJ: Nursing Process: Planning MSC: Client Needs: Safe and Effective Care Environment

18. When a child has chronic renal failure, the progressive deterioration produces a variety of clinical and biochemical disturbances that eventually are manifested in the clinical syndrome known as: a. Uremia. b. Oliguria. c. Proteinuria. d. Pyelonephritis.

ANS: A Uremia is the retention of nitrogenous products, producing toxic symptoms. Oliguria is diminished urine output. Proteinuria is the presence of protein, usually albumin, in the urine. Pyelonephritis is an inflammation of the kidney and renal pelvis

The perinatal nurse is caring for a woman in the immediate postbirth period. Assessment reveals that the woman is experiencing profuse bleeding. The most likely etiology for the bleeding is: a. Uterine atony. b. Uterine inversion. c. Vaginal hematoma. d. Vaginal laceration.

ANS: A Uterine atony is marked hypotonia of the uterus. It is the leading cause of postpartum hemorrhage. Uterine inversion may lead to hemorrhage, but it is not the most likely source of this clients bleeding. Furthermore, if the woman were experiencing a uterine inversion, it would be evidenced by the presence of a large, red, rounded mass protruding from the introitus. A vaginal hematoma may be associated with hemorrhage. However, the most likely clinical finding would be pain, not the presence of profuse bleeding. A vaginal laceration may cause hemorrhage, but it is more likely that profuse bleeding would result from uterine atony. A vaginal laceration should be suspected if vaginal bleeding continues in the presence of a firm, contracted uterine fundus. PTS: 1 DIF: Cognitive Level: Comprehension REF: 531 OBJ: Nursing Process: Diagnosis MSC: Client Needs: Physiologic Integrity

3. Decongestant nose drops are recommended for a 10-month-old infant with an upper respiratory tract infection. Instructions for nose drops should include: a. Avoiding use for more than 3 days. b. Keeping drops to use again for nasal congestion. c. Administering drops until nasal congestion subsides. d. Administering drops after feedings and at bedtime.

ANS: A Vasoconstrictive nose drops such as Neo-Synephrine should not be used for more than 3 days to avoid rebound congestion. Drops should be discarded after one illness because they may become contaminated with bacteria. Vasoconstrictive nose drops can have a rebound effect after 3 days of use. Drops administered before feedings are more helpful.

The most important nursing action in preventing neonatal infection is: a. Good handwashing. b. Isolation of infected infants. c. Separate gown technique. d. Standard Precautions.

ANS: A Virtually all controlled clinical trials have demonstrated that effective handwashing is responsible for the prevention of nosocomial infection in nursery units. Measures to be taken include Standard Precautions, careful and thorough cleaning, frequent replacement of used equipment, and disposal of excrement and linens in an appropriate manner. Overcrowding must be avoided in nurseries. However, the most important nursing action for preventing neonatal infection is effective handwashing. PTS: 1 DIF: Cognitive Level: Comprehension REF: 669 OBJ: Nursing Process: Implementation MSC: Client Needs: Safe and Effective Care Environment

An important nursing intervention when caring for a child who is experiencing a seizure would be to: a. describe and record the seizure activity observed. b. restrain the child when seizure occurs to prevent bodily harm. c. place a tongue blade between the teeth if they become clenched. d. suction the child during a seizure to prevent aspiration.

ANS: A When a child is having a seizure, the priority nursing care is observation of the child and seizure. The nurse then describes and records the seizure activity. The child should not be restrained, and nothing should be placed in the child's mouth. This may cause injury. To prevent aspiration, if possible, the child should be placed on the side, facilitating drainage.

A woman gave birth to an infant boy 10 hours ago. Where would the nurse expect to locate this womans fundus? a. One centimeter above the umbilicus b. Two centimeters below the umbilicus c. Midway between the umbilicus and the symphysis pubis d. Nonpalpable abdominally

ANS: A Within 12 hours after delivery the fundus may be approximately 1 cm above the umbilicus. The fundus descends about 1 to 2 cm every 24 hours. Within 12 hours after delivery the fundus may be approximately 1 cm above the umbilicus. By the sixth postpartum week the fundus normally is halfway between the symphysis pubis and the umbilicus. The fundus should be easily palpated using the maternal umbilicus as a reference point. PTS: 1 DIF: Cognitive Level: Comprehension REF: 483 OBJ: Nursing Process: Assessment MSC: Client Needs: Health Promotion and Maintenance

A client is warm and asks for a fan in her room for her comfort. The nurse enters the room to assess the mother and her infant and finds the infant unwrapped in his crib with the fan blowing over him on high. The nurse instructs the mother that the fan should not be directed toward the newborn and the newborn should be wrapped in a blanket. The mother asks why. The nurses best response is: a. Your baby may lose heat by convection, which means that he will lose heat from his body to the cooler ambient air. You should keep him wrapped and prevent cool air from blowing on him. b. Your baby may lose heat by conduction, which means that he will lose heat from his body to the cooler ambient air. You should keep him wrapped and prevent cool air from blowing on him. c. Your baby may lose heat by evaporation, which means that he will lose heat from his body to the cooler ambient air. You should keep him wrapped and prevent cool air from blowing on him. d. Your baby will get cold stressed easily and needs to be bundled up at all times.

ANS: A Your baby may lose heat by convection, which means that he will lose heat from his body to the cooler ambient air. You should keep him wrapped and prevent cool air from blowing on him is an accurate statement. Conduction is the loss of heat from the body surface to cooler surfaces, not air, in direct contact with the newborn. Evaporation is loss of heat that occurs when a liquid is converted into a vapor. In the newborn heat loss by evaporation occurs as a result of vaporization of moisture from the skin. Cold stress may occur from excessive heat loss, but this does not imply that the infant will become stressed if not bundled at all times. Furthermore, excessive bundling may result in a rise in the infants temperature. PTS: 1 DIF: Cognitive Level: Application REF: 560 OBJ: Nursing Process: Implementation MSC: Client Needs: Health Promotion and Maintenance

9. A young child diagnosed with leukemia is experiencing anorexia and severe stomatitis. The nurse should suggest that the parents try which intervention? a. Relax any eating pressures b. Firmly insist that child eat normally c. Begin gavage feedings to supplement diet d. Serve foods that are either hot or cold

ANS: A A multifaceted approach is necessary for children with severe stomatitis and anorexia. First, the parents should relax eating pressures. The nurse should suggest that the parents try soft, bland foods; normal saline or bicarbonate mouthwashes; and local anesthetics. The stomatitis is a temporary condition. The child can resume good food habits as soon as the condition resolves.

34. The nurse is implementing care for a school-age child admitted to the pediatric intensive care experiencing symptomology associated with diabetic ketoacidosis (DKA). Which prescribed intervention should the nurse implement first? a. Begin 0.9% saline solution intravenously as prescribed. b. Administer regular insulin intravenously as prescribed. c. Place child on a cardiac monitor. d. Place child on a pulse oximetry monitor.

ANS: A All patients with DKA experience dehydration (10% of total body weight in severe ketoacidosis) because of the osmotic diuresis, accompanied by depletion of electrolytes (sodium, potassium, chloride, phosphate, and magnesium). The initial hydrating solution is 0.9% saline solution. Insulin therapy should be started after the initial rehydration bolus because serum glucose levels fall rapidly after volume expansion. The child should be placed on the cardiac and pulse oximetry monitors after the rehydrating solution has been initiated.

31. A mother shares with the clinic nurse that she has been giving her 4 year old the antidiarrheal drug loperamide. What conclusion should the nurse arrive at based on knowledge of this classification of drugs? a. Not indicated b. Indicated because it slows intestinal motility c. Indicated because it decreases diarrhea d. Indicated because it decreases fluid and electrolyte losses

ANS: A Antimotility medications are not recommended for the treatment of acute infectious diarrhea. These medications have adverse effects and toxicity, such as worsening of the diarrhea because of slowing of motility and ileus, or a decrease in diarrhea with continuing fluid losses and dehydration. Antidiarrheal medications are not recommended in infants and small children.

18. An adolescent with osteosarcoma is scheduled for a leg amputation in 2 days. The nurse's approach should include which action? a. Answering questions with straightforward honesty b. Avoiding discussing the seriousness of the condition c. Explaining that, although the amputation is difficult, it will cure the cancer d. Assisting the adolescent in accepting the amputation as better than a long course of chemotherapy

ANS: A Honesty is essential to gain the child's cooperation and trust. The diagnosis of cancer should not be disguised with falsehoods. The adolescent should be prepared for the surgery so he or she has time to reflect on the diagnosis and subsequent treatment. This allows questions to be answered. To accept the need for radical surgery, the child must be aware of the lack of alternatives for treatment. Amputation is necessary, but it will not guarantee a cure. Chemotherapy is an integral part of the therapy with surgery. The child should be informed of the need for chemotherapy and its side effects before surgery.

2. A child with growth hormone (GH) deficiency is receiving GH therapy. What is the best time for the GH to be administered? a. At bedtime b. After meals c. Before meals d. On arising in the morning

ANS: A Injections are best given at bedtime to more closely approximate the physiologic release of GH. Before or after meals and on arising in the morning are times that do not mimic the physiologic release of the hormone.

8. A young child is brought to the emergency department with severe dehydration secondary to acute diarrhea and vomiting. Therapeutic management of this child will begin with which intervention? a. Intravenous fluids b. Oral rehydration solution (ORS) c. Clear liquids, 1 to 2 ounces at a time d. Administration of antidiarrheal medication

ANS: A Intravenous fluids are initiated in children with severe dehydration. ORS is acceptable therapy if the dehydration is not severe. Diarrhea is not managed by using clear liquids by mouth. These fluids have a high carbohydrate content, low electrolyte content, and high osmolality. Antidiarrheal medications are not recommended for the treatment of acute infectious diarrhea.

20. What is the earliest clinical manifestation of biliary atresia? a. Jaundice b. Vomiting c. Hepatomegaly d. Absence of stooling

ANS: A Jaundice is the earliest and most striking manifestation of biliary atresia. It is first observed in the sclera and may be present at birth, but is usually not apparent until ages 2 to 3 weeks. Vomiting is not associated with biliary atresia. Hepatomegaly and abdominal distention are common but occur later. Stools are large and lighter in color than expected because of the lack of bile.

15. Which statement appropriately describes a neuroblastoma? a. It is considered to be a "silent" tumor. b. Early diagnosis is usually possible because of the obvious clinical manifestations. c. It is the most common brain tumor in young children. d. It is the most common benign tumor in young children

ANS: A Neuroblastoma is a silent tumor with few symptoms. In more than 70% of cases, diagnosis is made after metastasis occurs, with the first signs caused by involvement in the nonprimary site. In only 30% of cases is diagnosis made before metastasis. Neuroblastomas are the most common malignant extracranial solid tumors in children. The majority of tumors develop in the adrenal glands or the retroperitoneal sympathetic chain. They are not benign; they metastasize.

16. A nurse is conducting a staff in-service on childhood cancers. Which is the primary site of osteosarcoma? a. Femur b. Humerus c. Pelvis d. Tibia

ANS: A Osteosarcoma is the most frequently encountered malignant bone cancer in children. The peak incidence is between ages 10 and 25 years. More than half occur in the femur. After the femur, most of the remaining sites are the humerus, tibia, pelvis, jaw, and phalanges

24. What is the most appropriate nursing action when a child with a probable intussusception has a normal, brown stool? a. Notify the practitioner b. Measure abdominal girth c. Auscultate for bowel sounds d. Take vital signs, including blood pressure

ANS: A Passage of a normal brown stool indicates that the intussusception has reduced itself. This is immediately reported to the practitioner, who may choose to alter the diagnostic/therapeutic plan of care.

12. The nurse is admitting a child with a Wilms' tumor. Which is the initial assessment finding associated with this tumor? a. Abdominal swelling b. Weight gain c. Hypotension d. Increased urinary output

ANS: A The initial assessment finding with a Wilms' (kidney) tumor is abdominal swelling. Weight loss, not weight gain, may be a finding. Hypertension occasionally occurs with a Wilms' tumor. Urinary output is not increased, but hematuria may be noted.

A young boy will receive a bone marrow transplant (BMT). This is possible because one of his older siblings is a histocompatible donor. Which is this type of BMT called? a. Syngeneic b. Allogeneic c. Monoclonal d. Autologous

ANS: B Allogeneic transplants are from another individual. Because he and his sibling are histocompatible, the BMT can be done. Syngeneic marrow is from an identical twin. There is no such thing as a monoclonal BMT. Autologous refers to the individuals own marrow.

25. An important nursing consideration in the care of a child with celiac disease is to facilitate which intervention? a. Refer to a nutritionist for detailed dietary instructions and education. b. Help the child and family understand that diet restrictions are usually only temporary. c. Teach proper hand washing and Standard Precautions to prevent disease transmission. d. Suggest ways to cope more effectively with stress to minimize symptoms.

ANS: A The main consideration is helping the child adhere to dietary management. Considerable time is spent in explaining to the child and parents the disease process, the specific role of gluten in aggravating the condition, and those foods that must be restricted. Referral to a nutritionist would help in this process. The most severe symptoms usually occur in early childhood and adult life. Dietary avoidance of gluten should be lifelong. Celiac disease is not transmissible or stress related.

19. The best chance of survival for a child with cirrhosis is: a. liver transplantation. b. treatment with corticosteroids. c. treatment with immune globulin. d. provision of nutritional support.

ANS: A The only successful treatment for end-stage liver disease and liver failure may be liver transplantation, which has improved the prognosis for many children with cirrhosis. Liver transplantation has revolutionized the approach to cirrhosis. Liver failure and cirrhosis are indications for transplantation. Liver transplantation reflects the failure of other medical and surgical measures, such as treatment with corticosteroids or immune globulin and nutritional support, to prevent or treat cirrhosis.

18. Bismuth subsalicylate may be prescribed for a child with a peptic ulcer to effect what result? a. Eradicate Helicobacter pylori b. Coat gastric mucosa c. Treat epigastric pain d. Reduce gastric acid production

ANS: A This combination of drug therapy is effective in the treatment and eradication of H. pylori. It does not bring about any of the results.

21. The nurse, caring for a neonate with a suspected tracheoesophageal fistula, should include what intervention into the plan of care? a. Elevating the head to facilitate secrete drainage. b. Elevating the head for feedings only. c. Feeding glucose water only. d. Avoiding suctioning unless the infant is cyanotic.

ANS: A When a newborn is suspected of having tracheoesophageal fistula, the most desirable position is supine with the head elevated on an inclined plane of at least 30 degrees to maintain an airway and facilitate drainage of secretions. It is imperative that any source of aspiration be removed at once; oral feedings are withheld. Feeding of fluids should not be given to infants suspected of having tracheoesophageal fistulas. The oral pharynx should be kept clear of secretion by oral suctioning. This is to avoid the cyanosis that is usually the result of laryngospasm caused by overflow of saliva into the larynx.

An adolescent with juvenile idiopathic arthritis (JIA) is prescribed abatacept (Orencia). Which should the nurse teach the adolescent regarding this medication? (Select all that apply.) a. Avoid receiving live immunizations while taking the medication. b. Before beginning this medication, a tuberculin screening test will be done. c. You will be getting a twice-a-day dose of this medication. d. This medication is taken orally.

ANS: A, B Abatacept reduces inflammation by inhibiting T cells and is given intravenously every 4 weeks. Possible side effects of biologics include an increased infection risk. Because of the infection risk, children should be evaluated for tuberculosis exposure before starting these medications. Live vaccines should be avoided while taking these agents.

A school-age child is diagnosed with systemic lupus erythematosus (SLE). The nurse should plan to implement which interventions for this child? (Select all that apply.) a. Instructions to avoid exposure to sunlight b. Teaching about body changes associated with SLE c. Preparation for home schooling d. Restricted activity

ANS: A, B Key issues for a child with SLE include therapy compliance; body-image problems associated with rash, hair loss, and steroid therapy; school attendance; vocational activities; social relationships; sexual activity; and pregnancy. Specific instructions for avoiding exposure to the sun and UVB light, such as using sunscreens, wearing sun-resistant clothing, and altering outdoor activities, must be provided with great sensitivity to ensure compliance while minimizing the associated feeling of being different from peers. The child should continue school attendance in order to gain interaction with peers and activity should not be restricted, but promoted.

1. Congenital anomalies can occur with the use of antiepileptic drugs (AEDs), including: (Select all that apply.) A. cleft lip. B. congenital heart disease. C. neural tube defects. D. gastroschisis. E. diaphragmatic hernia.

ANS: A, B, C Congenital anomalies that can occur with AEDs include cleft lip or palate, congenital heart disease, urogenital defects, and neural tube defects. Gastroschisis and diaphragmatic hernia are not associated with the use of AEDs.

2. Examples of sexual risk behaviors associated with exposure to a sexually transmitted infection (STI) include: (Select all that apply.) A. oral sex. B. unprotected anal intercourse. C. multiple sex partners. D. dry kissing. E. abstinence.

ANS: A, B, C Engaging in these sexual activities increases the exposure risk and the possibility of acquiring an STI. Dry kissing and abstinence are considered "safe" sexual practices.

The nurse in the neonatal intensive care unit is caring for an infant with myelomeningocele scheduled for surgical repair in the morning. Which early signs of infection should the nurse monitor on this infant? (Select all that apply.) a. Temperature instability b. Irritability c. Lethargy d. Bradycardia e. Hypertension

ANS: A, B, C The nurse should observe an infant with unrepaired myelomeningocele for early signs of infection, such as temperature instability (axillary), irritability, and lethargy. Bradycardia and hypertension are not early signs of infection in infants.

4. A student is learning about the process of hematopoiesis and how it is affected by leukemia. Which information does the student discover? (Select all that apply.) A. Blast cells multiply faster than mature cells. B. Leukemia disrupts normal hematopoiesis. C. Lymphoid cells differentiate into B and T cells. D. Myeloid cells crowd out normal cells in bone marrow. E. Pancytopenia occurs from proliferation of mast cells.

ANS: A, B, C Blast, or immature, cells have an increased rate of proliferation and multiply at the expense of normal cells. Leukemia does disrupt normal hematopoiesis (production and development of blood cells). Lymphoid cells differentiate into B and T cells. Myeloid cells differentiate into red blood cells, monocytes, granulocytes, and platelets; they do not reproduce and crowd out "normal" cells in the marrow. Pancytopenia occurs when large numbers of blast cells reproduce and crowd out normal marrow components.

2. Peer victimization is becoming a significant problem for school-age children and adolescents in the United States. Parents should be educated regarding signs that a child is being bullied. These might include: (Select all that apply.) a. The child spends an inordinate amount of time in the nurse's office. b. Belongings frequently go missing or are damaged. c. The child wants to be driven to school. d. School performance improves. e. The child freely talks about his or her day.

ANS: A, B, C Signs that may indicate a child is being bullied are similar to signs of other types of stress and include nonspecific illness or complaints, withdrawal, depression, school refusal, and decreased school performance. Children expressed fear of going to school or riding the school bus, and their belongings often are damaged or missing. Very often, children will not talk about what is happening to them.

1. Which demonstrates the school-age child's developing logic in the stage of concrete operations? (Select all that apply.) a. The school-age child is able to recognize that he can be a son, brother, or nephew at the same time. b. The school-age child understands the principles of adding, subtracting, and reversibility. c. The school-age child understands the principles of adding, subtracting, and reversibility. d. The school-age child has thinking that is characterized by egocentrism and animism

ANS: A, B, C The school-age child understands that the properties of objects do not change when their order, form, or appearance does. Conservation occurs in the concrete operations stage. Comprehension of class inclusion occurs as the school-age child's logic increases. The child begins to understand that a person can be in more than one class at the same time. This is characteristic of concrete thinking and logical reasoning. The school-age child is able to understand principles of adding, subtracting, and the process of reversibility, which occurs in the stage of concrete operations. Thinking that is characterized by egocentrism and animism occurs in the intuitive thought stage, not the concrete operations stage of development.

Examples of appropriate techniques to wake a sleepy infant for breastfeeding include (Select all that apply): a. Unwrapping the infant. b. Changing the diaper. c. Talking to the infant. d. Slapping the infants hands and feet. e. Applying a cold towel to the infants abdomen.

ANS: A, B, C Unwrapping the infant, changing the diaper, and talking to the infant are appropriate techniques to use when trying to wake a sleepy infant. Slapping the infants hand and feet and applying a cold towel to the infants abdomen are not appropriate. The parent can rub the infants hands or feet to wake the infant. Applying a cold towel to the infants abdomen may lead to cold stress in the infant. The parent may want to apply a cool cloth to the infants face to wake the infant. PTS: 1 DIF: Cognitive Level: Application REF: 647 OBJ: Nursing Process: Implementation MSC: Client Needs: Health Promotion and Maintenance

1. Most women with uncomplicated pregnancies can use the nurse as their primary source for nutritional information. The nurse or midwife should refer a patient to a registered dietitian for in-depth nutritional counseling in the following situations: (Select all that apply.) A. preexisting or gestational illness such as diabetes. B. ethnic or cultural food patterns. C. obesity. D. vegetarian diet. E. allergy to tree nuts.

ANS: A, B, C, D The nurse should be especially aware that conditions such as diabetes can require in-depth dietary planning and evaluation. To prevent issues with hypoglycemia and hyperglycemia and an increased risk for perinatal morbidity and mortality, this patient would benefit from a referral to a dietitian. Consultation with a dietitian may ensure that cultural food beliefs are congruent with modern knowledge of fetal development and that adjustments can be made to ensure that all nutritional needs are met. The obese pregnant patient may be under the misapprehension that because of her excess weight little or no weight gain is necessary. According to the Institute of Medicine, a patient with a body mass index in the obese range should gain at least 7 kg to ensure a healthy outcome. This patient may require in-depth counseling on optimal food choices. The vegetarian patient needs to have her dietary intake carefully assessed to ensure that the optimal combination of amino acids and protein intake is achieved. Very strict vegetarians (vegans) who consume only plant products may also require vitamin B and mineral supplementation. A patient with a food allergy would not alter that component of her diet during pregnancy; therefore, no additional consultation is necessary.

Many common drugs of abuse cause significant physiologic and behavioral problems in infants who are breastfed by mothers currently using (Select all that apply): a. Amphetamine. b. Heroin. c. Nicotine. d. PCP. e. Morphine.

ANS: A, B, C, D Amphetamine, heroin, nicotine, and PCP are contraindicated during breastfeeding because of the reported effects on the infant. Morphine is a medication that often is used to treat neonatal abstinence syndrome. PTS: 1 DIF: Cognitive Level: Comprehension REF: 676 OBJ: Nursing Process: Assessment MSC: Client Needs: Physiologic Integrity

Pain should be assessed regularly in all newborn infants. If the infant is displaying physiologic or behavioral cues indicating pain, measures should be taken to manage the pain. Examples of nonpharmacologic pain management techniques include (Select all that apply): a. Swaddling. b. Nonnutritive sucking. c. Skin-to-skin contact with the mother. d. Sucrose. e. Acetaminophen.

ANS: A, B, C, D Swaddling, nonnutritive sucking, skin-to-skin contact with the mother, and sucrose are all appropriate nonpharmacologic techniques used to manage pain in neonates. Acetaminophen is a pharmacologic method of treating pain. PTS: 1 DIF: Cognitive Level: Comprehension REF: 621 OBJ: Nursing Process: Implementation MSC: Client Needs: Physiologic Integrity

1. Congenital disorders refer to conditions that are present at birth. These disorders may be inherited and caused by environmental factors or maternal malnutrition. Toxic exposures have the greatest effect on development between 15 and 60 days of gestation. For the nurse to be able to conduct a complete assessment of the newly pregnant patient, she should understand the significance of exposure to known human teratogens. These include: (Select all that apply.) A. infections. B. radiation. C. maternal conditions. D. drugs. E. chemicals.

ANS: A, B, C, D, E Exposure to radiation and numerous infections may result in profound congenital deformities. These include but are not limited to varicella, rubella, syphilis, parvovirus, cytomegalovirus, and toxoplasmosis. Certain maternal conditions such as diabetes and phenylketonuria may also affect organs and other parts of the embryo during this developmental period. Drugs such as antiseizure medication and some antibiotics as well as chemicals, including lead, mercury, tobacco, and alcohol, also may result in structural and functional abnormalities.

2. Transvaginal ultrasonography is often performed during the first trimester. While preparing your 6-week gestation patient for this procedure, she expresses concerns over the necessity for this test. The nurse should explain that this diagnostic test may be indicated for a number of situations. (Select all that apply) A. Establish gestational age B. Obesity C. Fetal abnormalities D. Amniotic fluid volume E. Ectopic pregnancy

ANS: A, B, C, E Transvaginal ultrasound is useful in obese women whose thick abdominal layers cannot be penetrated with traditional abdominal ultrasound. This procedure is also used for identifying ectopic pregnancy, estimating gestational age, confirming fetal viability, and identifying fetal abnormalities. Amniotic fluid volume is assessed during the second and third trimester. Conventional ultrasound would be used.

A nurse is discussing the signs and symptoms of mastitis with a mother who is breastfeeding. What signs and symptoms should the nurse include in her discussion (Select all that apply)? a. Breast tenderness b. Warmth in the breast c. An area of redness on the breast often resembling the shape of a pie wedge d. A small white blister on the tip of the nipple e. Fever and flulike symptoms

ANS: A, B, C, E Breast tenderness, breast warmth, breast redness, and fever and flulike symptoms are commonly associated with mastitis and should be included in the nurses discussion of mastitis. A small white blister on the tip of the nipple generally is not associated with mastitis. It is commonly seen in women who have a plugged milk duct. PTS: 1 DIF: Cognitive Level: Analysis REF: 655 OBJ: Nursing Process: Planning MSC: Client Needs: Physiologic Integrity

3. The nurse is teaching a community group about early warning signs of cancer. Which signs does the nurse include? (Select all that apply.) A. A sore that does not heal B. Change in bowel or bladder habits C. Difficulty swallowing or indigestion D. Nagging feeling that something is wrong E. Unusual bleeding or discharge

ANS: A, B, C, E The American Cancer Society uses the acronym CAUTION to describe common warning signs of cancer: change in bowel or bladder habits; a sore that does not heal; unusual bleeding or discharge; thickening or lump in the breast, testicles, or elsewhere; indigestion or trouble swallowing; obvious change in the size, color shape, or thickness of a wart, mole, or mouth sore; and nagging cough or hoarseness.

The nurse is conducting discharge teaching to parents of a preschool child with myelomeningocele, repaired at birth, being discharged from the hospital after a urinary tract infection (UTI). Which should the nurse include in the discharge instructions related to management of the childs genitourinary function? (Select all that apply.) a. Continue to perform the clean intermittent catheterizations (CIC) at home. b. Administer the oxybutynin chloride (Ditropan) as prescribed. c. Reduce fluid intake in the afternoon and evening hours. d. Monitor for signs of a recurrent urinary tract infection. e. Administer furosemide (Lasix) as prescribed.

ANS: A, B, D Discharge teaching to prevent renal complications in a child with myelomeningocele include: (1) regular urologic care with prompt and vigorous treatment of infections; (2) a method of regular emptying of the bladder, such as clean intermittent catheterization (CIC) taught to and performed by parents and self-catheterization taught to children; (3) medications to improve bladder storage and continence, such as oxybutynin chloride (Ditropan) and tolterodine (Detrol). Fluids should not be limited and Lasix is not used to improve renal function for children with myelomeningocele.

Medications used to manage postpartum hemorrhage (PPH) include (Select all that apply): a. Pitocin. b. Methergine. . c. Terbutaline. d. Hemabate. e. Magnesium sulfate

ANS: A, B, D Pitocin, Methergine, and Hemabate are all used to manage PPH. Terbutaline and magnesium sulfate are tocolytics; relaxation of the uterus causes or worsens PPH. PTS: 1 DIF: Cognitive Level: Comprehension REF: 533 OBJ: Nursing Process: Implementation MSC: Client Needs: Physiologic Integrity

2. Which congenital malformations result from multifactorial inheritance? (Select all that apply.) A. Cleft lip B. Congenital heart disease C. Cri du chat syndrome D. Anencephaly E. Pyloric stenosis

ANS: A, B, D, E All these congenital malformations are associated with multifactorial inheritance. Cri du chat syndrome is related to a chromosome deletion.

2. Which factors influence cervical dilation? (Select all that apply.) a. Strong uterine contractions b. Force of the presenting fetal part against the cervix c. Size of the woman d. Pressure applied by the amniotic sac e. Scarring of the cervix

ANS: A, B, D, E Dilation of the cervix occurs by the drawing upward of the musculofibrous components of the cervix, which is caused by strong uterine contractions. Pressure exerted by the amniotic fluid while the membranes are intact or by the force applied by the presenting part can also promote cervical dilation. Scarring of the cervix as a result of a previous infection or surgery may slow cervical dilation. Pelvic size or the size of the woman does not affect cervical dilation.

1. There is little consensus on the management of premenstrual dysphoric disorder (PMDD). However, nurses can advise women on several self-help modalities that often improve symptoms. The nurse knows that health teaching has been effective when the patient reports that she has adopted a number of lifestyle changes, including: (Select all that apply.) A. regular exercise. B. improved nutrition. C. a daily glass of wine. D. smoking cessation. E. oil of evening primrose.

ANS: A, B, D, E These modalities may provide significant symptom relief in 1 to 2 months. If there is no improvement after these changes have been made, the patient may need to begin pharmacologic therapy. Women should decrease both their alcohol and caffeinated beverage consumption if they have PMDD.

2. Diabetes refers to a group of metabolic diseases characterized by hyperglycemia resulting from defects in insulin action, insulin secretion, or both. Over time, diabetes causes significant changes in the microvascular and macrovascular circulations. These complications include: (Select all that apply.) a. atherosclerosis. b. retinopathy. c. IUFD. d. nephropathy. e. neuropathy.

ANS: A, B, D, E These structural changes are most likely to affect a variety of systems, including the heart, eyes, kidneys, and nerves. Intrauterine fetal death (stillbirth) remains a major complication of diabetes in pregnancy; however, this is a fetal complication.

Which concerns about parenthood are often expressed by visually impaired mothers (Select all that apply)? a. Infant safety b. Transportation c. The ability to care for the infant d. Missing out visually e. Needing extra time for parenting activities to accommodate the visual limitations

ANS: A, B, D, E Concerns expressed by visually impaired mothers include infant safety, extra time needed for parenting activities, transportation, handling other peoples reactions, providing proper discipline, and missing out visually. Blind people sense reluctance on the part of others to acknowledge that they have a right to be parents; however, blind parents are fully capable of caring for their infants. PTS: 1 DIF: Cognitive Level: Comprehension REF: 524 OBJ: Nursing Process: Evaluation MSC: Client Needs: Health Promotion and Maintenance

12. The staff nurse is educating nursing students on the long-term effects of childhood chemotherapy. Which problems does the nurse include in the educational session? (Select all that apply.) A. Cardiac dysfunction B. Hearing loss C. Increased risk of multiple-gestation pregnancies D. Learning disabilities E. Peripheral neuropathy

ANS: A, B, D, E The list of long-term effects of chemotherapy is lengthy and includes cardiac dysfunction, hearing loss, learning disabilities, and peripheral neuropathy, among others. Sterility, not an increased risk for multiple-gestation pregnancies, is also an effect.

Which assessment findings should the nurse note in a school-age child with Duchenne muscular dystrophy (DMD)? (Select all that apply.) a. Lordosis b. Gower sign c. Kyphosis d. Scoliosis e. Waddling gait

ANS: A, B, E Difficulties in running, riding a bicycle, and climbing stairs are usually the first symptoms noted in Duchenne muscular dystrophy. Typically, affected boys have a waddling gait and lordosis, fall frequently, and develop a characteristic manner of rising from a squatting or sitting position on the floor (Gower sign). Lordosis occurs as a result of weakened pelvic muscles, and the waddling gait is a result of weakness in the gluteus medius and maximus muscles. Kyphosis and scoliosis are not assessment findings with DMD.

What is an age-appropriate nursing intervention to facilitate psychologic adjustment for an adolescent expected to have a prolonged hospitalization (select all that apply)? a. Encourage parents to bring in homework and schedule study times. b. Allow the adolescent to wear street clothes. c. Involve the parents in care. d. Follow home routines. e. Encourage parents to bring in favorite foods.

ANS: A, B, E Encouraging parents to bring in homework, street clothes, and favorite foods are all developmentally appropriate approaches to facilitate adjustment and coping for an adolescent who will be experiencing prolonged hospitalization. Involving parents in care and following home routines are important interventions for the preschool child who is in the hospital. Adolescents do not need parents to assist in their care. They are used to performing independent self-care. Adolescents may want their parents to be nearby, or they may enjoy the freedom and independence from parental control and routines.

23. What are the manifestations of hypoglycemia? a. Lethargy b. Thirst c. Nausea and vomiting d. Shaky feeling and dizziness

ANS: D Some of the clinical manifestations of hypoglycemia include shaky feelings; dizziness; difficulty concentrating, speaking, focusing, and coordinating; sweating; and pallor. Lethargy, thirst, and nausea and vomiting are manifestations of hyperglycemia.

A 14-year-old girl is in the intensive care unit after a spinal cord injury 2 days ago. Nursing care for this child includes which action(s)? (Select all that apply.) a. Monitoring and maintaining systemic blood pressure b. Administering corticosteroids c. Minimizing environmental stimuli d. Discussing long-term care issues with the family e. Monitoring for respiratory complications

ANS: A, B, E Spinal cord injury patients are physiologically labile, and close monitoring is required. They may be unstable for the first few weeks after the injury. Corticosteroids are administered to minimize the inflammation present with the injury. It is not necessary to minimize environmental stimuli for this type of injury. Discussing long-term care issues with the family is inappropriate. The family is focusing on the recovery of their child. It will not be known until the rehabilitation period how much function the child may recover.

1. The student nurse studying childhood cancers understands that neoplasms are caused by which factors? (Select all that apply.) A. Chromosomal/genetic abnormalities B. External stimuli or environment C. Maternal nutrition during gestation D. Substance abuse during pregnancy E. Viruses that alter the immune system

ANS: A, B, E Neoplasms are caused by one or a combination of the following: chromosomal or genetic abnormalities, external stimuli or the environment, and/or viruses that alter the immune system. Nutritional deficits and substance abuse by the mother can certainly lead to developmental and other health problems, but do not lead to childhood cancers.

Possible alternative and complementary therapies for postpartum depression (PPD) for breastfeeding mothers include (Select all that apply): a. Acupressure. b. Aromatherapy. c. St. Johns wort. d. Wine consumption. e. Yoga.

ANS: A, B, E Possible alternative/complementary therapies for postpartum depression include acupuncture, acupressure, aromatherapy, therapeutic touch, massage, relaxation techniques, reflexology, and yoga. St. Johns wort has not been proven to be safe for women who are breastfeeding. Women who are breastfeeding and/or have a history of PPD should not consume alcohol. PTS: 1 DIF: Cognitive Level: Comprehension REF: 547 OBJ: Nursing Process: Planning MSC: Client Needs: Physiologic Integrity

3. A nurse is planning care for a 7-year-old child hospitalized with osteomyelitis. Which activities should the nurse plan to bring from the playroom for the child? (Select all that apply.) a. Paper and some paints b. Board games c. Jack-in-the-box d. Stuffed animals e. Computer games

ANS: A, B, E School-age children become fascinated with complex board, card, or computer games that they can play alone, with a best friend, or with a group. They also enjoy sewing, cooking, carpentry, gardening, and creative activities such as painting. Jack-in-the-box and stuffed animals would be appropriate for a toddler or preschool child.

13. A child is getting induction therapy for Burkitt lymphoma. The nurse finds the child lethargic and complaining of side and back pain. The child's morning laboratory results indicate a serum calcium level of 7.2 mg/dL. What actions by the nurse are the most appropriate at this time? (Select all that apply.) A. Administer a dose of pain medication. B. Assess Chvostek and Trousseau signs. C. Call the rapid response team. D. Encourage an increased oral intake. E. Prepare to administer allopurinol (Aloprim).

ANS: A, B, E This child is manifesting signs of tumor lysis syndrome. The child is at risk due to the rapid destruction of cancer cells (induction therapy) and from the child's type of cancer (Burkitt lymphoma). Lethargy, flank pain, and hypocalcemia are common findings in this condition. The nurse should administer pain medication, assess for physical manifestations of hypocalcemia (Chvostek and Trousseau signs), and prepare to administer allopurinol. Adequate hydration is important as well, but because the child is lethargic, IV fluids should be given, not oral fluids. The rapid response team is not needed at this point.

The nurse is preparing to give oral care to a school-age child with mucositis secondary to chemotherapy administered to treat leukemia. Which preparations should the nurse use for oral care on this child? (Select all that apply.) a. Chlorhexidine gluconate (Peridex) b. Lemon glycerin swabs c. Antifungal troches (lozenges) d. Lip balm (Aquaphor) e. Hydrogen peroxide

ANS: A, C, D Preparations that may be used to prevent or treat mucositis include chlorhexidine gluconate (Peridex) because of its dual effectiveness against candidal and bacterial infections, antifungal troches (lozenges) or mouthwash, and lip balm (e.g., Aquaphor) to keep the lips moist. Agents that should not be used include lemon glycerin swabs (irritate eroded tissue and can decay teeth), hydrogen peroxide (delays healing by breaking down protein), and milk of magnesia (dries mucosa).

1. Which changes take place in the woman's reproductive system, days or even weeks before the commencement of labor? (Select all that apply.) a. Lightening b. Exhaustion c. Bloody show d. Rupture of membranes e. Decreased fetal movement

ANS: A, C, D Signs that precede labor may include lightening, urinary frequency, backache, weight loss, surge of energy, bloody show, and rupture of membranes. Many women experience a burst of energy before labor. A decrease in fetal movement is an ominous sign that does not always correlate with labor.

A young child with human immunodeficiency virus (HIV) is receiving several antiretroviral drugs. The purpose of these drugs is to: a. cure the disease. b. delay disease progression. c. prevent spread of disease. d. treat Pneumocystis carinii pneumonia.

ANS: B Although not a cure, these antiviral drugs can suppress viral replication, preventing further deterioration of the immune system and delaying disease progression. At this time, cure is not possible. These drugs do not prevent the spread of the disease. P. carinii prophylaxis is accomplished with antibiotics.

3. Along with gas exchange and nutrient transfer, the placenta produces many hormones necessary for normal pregnancy. These include: (Select all that apply.) A. human chorionic gonadotropin (hCG). B. insulin. C. estrogen. D. progesterone. E. testosterone.

ANS: A, C, D hCG causes the corpus luteum to persist and produce the necessary estrogens and progesterone for the first 6 to 8 weeks. Estrogens cause enlargement of the woman's uterus and breasts; cause growth of the ductal system in the breasts; and, as term approaches, play a role in the initiation of labor. Progesterone causes the endometrium to change, providing early nourishment. Progesterone also protects against spontaneous abortion by suppressing maternal reactions to fetal antigens and reduces unnecessary uterine contractions. Other hormones produced by the placenta include hCT, hCA, and numerous growth factors. Human placental lactogen promotes normal nutrition and growth of the fetus and maternal breast development for lactation. This hormone decreases maternal insulin sensitivity and utilization of glucose, making more glucose available for fetal growth. If a Y chromosome is present in the male fetus, hCG causes the fetal testes to secrete testosterone necessary for the normal development of male reproductive structures.

6. The nurse is explaining types of solid tumors to a group of students. Which information does the nurse include? (Select all that apply.) A. A sarcoma is found in bone or muscle. B. "Carcinoma" means any cancerous tumor. C. Epithelial cells give rise to carcinomas. D. Lymphoma might originate in the thymus. E. Pediatric and adult solid tumors are similar.

ANS: A, C, D Pediatric and adult solid tumors are very different. A sarcoma arises from connective or supporting tissues, such as bones or muscle. A carcinoma is cancer arising from glandular and/or epithelial cells. Lymphomas originate in lymphoid organs, such as the lymph nodes, spleen, and thymus.

Infants born between 34 0/7 and 36 6/7 weeks of gestation are called late-preterm infants because they have many needs similar to those of preterm infants. Because they are more stable than early-preterm infants, they may receive care that is much like that of a full-term baby. The mother-baby or nursery nurse knows that these babies are at increased risk for (Select all that apply): a. Problems with thermoregulation b. Cardiac distress c. Hyperbilirubinemia d. Sepsis e. Hyperglycemia

ANS: A, C, D Thermoregulation problems, hyperbilirubinemia, and sepsis are all conditions related to immaturity and warrant close observation. After discharge the infant is at risk for rehospitalization related to these problems. AWHONN launched the Near-Term Infant Initiative to study the problem and ways to ensure that these infants receive adequate care. The nurse should ensure that this infant is feeding adequately before discharge and that parents are taught the signs and symptoms of these complications. Late-preterm infants are also at increased risk for respiratory distress and hypoglycemia. PTS: 1 DIF: Cognitive Level: Comprehension REF: 685 OBJ: Nursing Process: Assessment MSC: Client Needs: Health Promotion and Maintenance

2. Induction of labor is considered an acceptable obstetric procedure if it is in the best interest to deliver the fetus. The charge nurse in the labor and delivery unit is often asked to schedule patients for this procedure and therefore must be cognizant of the specific conditions appropriate for labor induction. These include: (Select all that apply.) a. rupture of membranes at or near term. b. convenience of the woman or her physician. c. chorioamnionitis (inflammation of the amniotic sac). d. postterm pregnancy. e. fetal death.

ANS: A, C, D, E These are all acceptable indications for induction. Other conditions include intrauterine growth retardation (IUGR), maternal-fetal blood incompatibility, hypertension, and placental abruption. Elective inductions for the convenience of the woman or her provider are not recommended; however, they have become commonplace. Factors such as rapid labors and living a long distance from a health care facility may be valid reasons in such a circumstance. Elective delivery should not occur before 39 weeks' completed gestation.

Hearing loss is one of the genetic disorders included in the universal screening program. Auditory screening of all newborns within the first month of life is recommended by the American Academy of Pediatrics. Reasons for having this testing performed include (Select all that apply): a. Prevention or reduction of developmental delay. b. Reassurance for concerned new parents. c. Early identification and treatment. d. Helping the child communicate better. e. Recommendation by the Joint Committee on Infant Hearing.

ANS: A, C, D, E New parents are often anxious regarding this test and the impending results; however, it is not the reason for the screening to be performed. Auditory screening is usually done before hospital discharge. It is important for the nurse to ensure that the infant receives the appropriate testing and that the test is fully explained to the parents. For infants who are referred for further testing and follow-up, it is important for the nurse to provide further explanation and emotional support. All other responses are appropriate reasons for auditory screening of the newborn. Infants who do not pass the screening test should have it repeated. If they still do not pass the test, they should have a full audiologic and medical evaluation by 3 months of age. If necessary, the infant should be enrolled in early intervention by 6 months of age. PTS: 1 DIF: Cognitive Level: Application REF: 609 OBJ: Nursing Process: Diagnosis MSC: Client Needs: Physiologic Integrity COMPLETION

Ryan has just been unexpectedly admitted to the intensive care unit after abdominal surgery. The nursing staff has completed the admission process, and Ryans condition is beginning to stabilize. When speaking with the parents, the nurses should expect which stressors to be evident (select all that apply)? a. Unfamiliar environment b. Usual day-night routine c. Strange smells d. Provision of privacy e. Inadequate knowledge of condition and routine

ANS: A, C, E Intensive care units, especially when the family is unprepared for the admission, are a strange and unfamiliar place. There are many pieces of unfamiliar equipment, and the sights and sounds are much different from a general hospital unit. Also, with the childs condition being more precarious, it may be difficult to keep the parents updated and knowledgeable about what is happening. Lights are usually on around the clock, seriously disrupting the diurnal rhythm. There is usually little privacy available for families in intensive care units.

A child is being discharged from an ambulatory care center after an inguinal hernia repair. Which discharge interventions should the nurse implement (select all that apply)? a. Discuss dietary restrictions. b. Hold any analgesic medications until the child is home. c. Send a pain scale home with the family. d. Suggest the parents fill the prescriptions on the way home. e. Discuss complications that may occur.

ANS: A, C, E The discharge interventions a nurse should implement when a child is being discharged from an ambulatory care center should include dietary restrictions, being very specific and giving examples of clear fluids or what is meant by a full liquid diet. The nurse should give specific information on pain control and send a pain scale home with the family. All complications that may occur after an inguinal hernia repair should be discussed with the parents. The pain medication, as prescribed, should be given before the child leaves the building, and prescriptions should be filled and given to the family before discharge.

2. The class of drugs known as opioid analgesics (butorphanol, nalbuphine) is not suitable for administration to women with known opioid dependence. The antagonistic activity could precipitate withdrawal symptoms (abstinence syndrome) in both mothers and newborns. Signs of opioid/narcotic withdrawal in the mother would include: (Select all that apply.) a. yawning, runny nose. b. increase in appetite. c. chills and hot flashes. d. constipation. e. irritability, restlessness.

ANS: A, C, E The woman experiencing maternal opioid withdrawal syndrome will exhibit yawning, runny nose, sneezing, anorexia, chills or hot flashes, vomiting, diarrhea, abdominal pain, irritability, restlessness, muscle spasms, weakness, and drowsiness. It is important for the nurse to assess both mother and baby and to plan care accordingly.

10. A nurse is explaining radiation side effects to the parents of a child for whom it has been ordered. Which side effects does the nurse include in the explanation? (Select all that apply.) A. Hair loss B. Leukocytosis C. Nausea D. Polycythemia E. Skin desquamation

ANS: A, C, E Common side effects of radiation include nausea, alopecia (hair loss), fatigue and malaise, low WBC, skin desquamation, and mucous membrane inflammation and irritation. Leukocytosis (high WBC count) and polycythemia (increased RBC count) are not seen.

11. A student nurse is preparing to administer odansetron (Zofran) to a child receiving chemotherapy. The child weighs 44 lb (20 kg). Which actions by the student nurse require intervention by the faculty? (Select all that apply.) A. Assesses the child's pain with a pediatric scale B. Discusses side effects with the parents/child C. Draws up 300 mg for IV administration D. Prepares to administer 3 mg IV push E. Withdraws a 200-mg suppository for use

ANS: A, C, E Odansetron is an anti-emetic (not a pain medication) and can be given IV at 0.15 mg/kg, making the correct dose 3 mg. The faculty member should intervene if the student assesses pain, not nausea; draws up 300 mg; or tries to obtain a suppository for the child. Odansetron can also be given PO. Discussing side effects is a responsibility when giving medications.

3. Which interventions should a nurse implement when caring for a child with hepatitis? (Select all that apply.) a. Provide a well-balanced, low-fat diet. b. Schedule playtime in the playroom with other children. c. Teach parents not to administer any over-the-counter medications. d. Arrange for home schooling because the child will not be able to return to school. e. Instruct parents on the importance of good hand washing.

ANS: A, C, E The child with hepatitis should be placed on a well-balanced, low-fat diet. Parents should be taught to not give over-the-counter medications because of impaired liver function. Hand hygiene is the most important preventive measure for the spread of hepatitis. The child will be in contact isolation in the hospital, so playtime with other hospitalized children is not scheduled. The child will be on contact isolation for a minimum of 1 week after the onset of jaundice. After that period, the child will be allowed to return to school.

2. The treatment of brain tumors in children consists of which therapies? (Select all that apply.) a. Surgery b. Bone marrow transplantation c. Chemotherapy d. Stem cell transplantation e. Radiation f. Myelography

ANS: A, C, E Treatment for brain tumors in children may consist of surgery, chemotherapy, and radiotherapy alone or in combination. Bone marrow and stem cell transplantation therapies are used for leukemia, lymphoma, and other solid tumors where myeloablative therapies are used. Myelography is a radiographic examination after an intrathecal injection of contrast medium. It is not a treatment.

2. The nurse working with pediatric oncology patients educates the patients and families regarding best long-term follow-up practices. Which recommendations does this include? (Select all that apply.) A. Continued care by an interdisciplinary team B. Height measurements until puberty is reached C. Genetic testing prior to having children D. Risk-based follow-up appointments E. Thyroid screening for 5 years after remission

ANS: A, D Best-practice recommendations for follow-up include risk-based referrals and continued involvement of an interdisciplinary team of specialists. Height measurements are important for children until their adult height is achieved. Genetic testing is only recommended for certain types of cancer. Thyroid screening is important throughout the lifetime of survivors who were treated with radiotherapy to the neck, spine, or brain.

3. The exact cause of breast cancer remains undetermined. Researchers have found that there are many common risk factors that increase a woman's chance of developing a malignancy. It is essential for the nurse who provides care to women of any age to be aware of which of the following risk factors? (Select all that apply.) A. Family history B. Late menarche C. Early menopause D. Race E. Nulliparity or first pregnancy after age 30

ANS: A, D, E Family history, race, and nulliparity are known risk factors for the development of breast cancer. Other risk factors include age, personal history of cancer, high socioeconomic status, sedentary lifestyle, hormone replacement therapy, recent use of oral contraceptives, never having breastfed a child, and drinking more than one alcoholic beverage per day. Early menarche and late menopause are risk factors for breast malignancy, not late menarche and early menopause.

Nurses play a critical role in educating parents regarding measures to prevent infant abduction. Which instructions contribute to infant safety and security? a. The mother should check the photo ID of any person who comes to her room. b. The baby should be carried in the parents arms from the room to the nursery. c. Because of infant security systems, the baby can be left unattended in the patients room. d. Parents should use caution when posting photos of their infant on the Internet. e. The mom should request that a second staff member verify the identity of any questionable person.

ANS: A, D, E Nurses must discuss infant security precautions with the mother and her family because infant abduction continues to be a concern. The mother should be taught to check the identity of any person who comes to remove the baby from her room. Hospital personnel usually wear picture identification patches. On some units staff members also wear matching scrubs or special badges that are unique to the perinatal unit. As a rule the baby is never carried in arms between the mothers room and the nursery, but rather is always wheeled in a bassinet. The infant should never be left unattended, even if the facility has an infant security system. Parents should be instructed to use caution when posting photos of their new baby on the Internet and other public forums. PTS: 1 DIF: Cognitive Level: Application REF: 492 OBJ: Nursing Process: Intervention MSC: Client Needs: Safe and Effective Care Environment

A parent who has a hearing impairment is presented with a number of challenges in parenting. Which nursing approaches are appropriate for working with hearing-impaired new parents (Select all that apply)? a. Use devices that transform sound into light. b. Assume that the patient knows sign language. c. Speak quickly and loudly. d. Ascertain whether the patient can read lips before teaching. e. Written messages aid in communication.

ANS: A, D, E Section 504 of the Rehabilitation Act of 1973 requires that hospitals use various communication techniques and resources with the deaf and hard of hearing patient. This includes devices such as door alarms, cry alarms, and amplifiers. Before initiating communication, the nurse needs to be aware of the parents preferences for communication. Not all hearing-impaired patients know sign language. Do they wear a hearing aid? Do they read lips? Do they wish to have a sign language interpreter? If the parent relies on lip reading, the nurse should sit close enough so that the parent can visualize lip movements. The nurse should speak clearly in a regular voice volume, in short, simple sentences. Written messages such as on a black or white erasable board can be useful. Written materials should be reviewed with the parents before discharge. PTS: 1 DIF: Cognitive Level: Application REF: 524 OBJ: Nursing Process: Planning MSC: Client Needs: Safe and Effective Care Environment

An infant with hydrocephalus is hospitalized for surgical placement of a ventriculoperitoneal shunt. Which interventions should be included in the child's postoperative care? (Select all that apply.) a. Observe closely for signs of infection. b. Pump the shunt reservoir to maintain patency. c. Administer sedation to decrease irritability. d. Maintain Trendelenburg position to decrease pressure on the shunt. e. Maintain an accurate record of intake and output. f. Monitor for abdominal distention.

ANS: A, E, F Infection is a major complication of ventriculoperitoneal shunts. Observation for signs of infection is a priority nursing intervention. Intake and output should be measured carefully. Abdominal distention could be a sign of peritonitis or a postoperative ileus. Pumping of the shunt may cause obstruction or other problems and should not be performed unless indicated by the neurosurgeon. Pain management rather than sedation should be the goal of therapy. The child is kept flat to avoid too rapid a reduction of intracranial fluid.

1. You (the nurse) are reviewing the educational packet provided to a patient about tubal ligation. What is an important fact you should point out? (Select all that apply.) A. "It is highly unlikely that you will become pregnant after the procedure." B. "This is an effective form of 100% permanent sterilization. You won't be able to get pregnant." C. "Sterilization offers some form of protection against sexually transmitted infections (STIs)." D. "Sterilization offers no protection against STIs." E. "Your menstrual cycle will greatly increase after your sterilization."

ANS: A,D A woman is unlikely to become pregnant after tubal ligation, although it is not 100% effective. Sterilization offers no protection against STIs. The menstrual cycle typically remains the same after a tubal ligation.

18. A woman who is 39 weeks pregnant expresses fear about her impending labor and how she will manage. The nurse's best response is: a. "Don't worry about it. You'll do fine." b. "It's normal to be anxious about labor. Let's discuss what makes you afraid." c. "Labor is scary to think about, but the actual experience isn't." d. "You can have an epidural. You won't feel anything."

ANS: B "It's normal to be anxious about labor. Let's discuss what makes you afraid" allows the woman to share her concerns with the nurse and is a therapeutic communication tool. "Don't worry about it. You'll do fine" negates the woman's fears and is not therapeutic. "Labor is scary to think about, but the actual experience isn't" negates the woman's fears and offers a false sense of security. It is not true that every woman may have an epidural. A number of criteria must be met for use of an epidural. Furthermore, many women still experience the feeling of pressure with an epidural.

Chelation therapy is begun on a child with b-thalassemia major. The purpose of this therapy is to: a. treat the disease. b. eliminate excess iron. c. decrease risk of hypoxia. d. manage nausea and vomiting.

ANS: B A complication of the frequent blood transfusions in thalassemia is iron overload. Chelation therapy with deferoxamine (an iron-chelating agent) is given with oral supplements of vitamin C to increase iron excretion. Chelation therapy treats the side effect of the disease management. Decreasing the risk of hypoxia and managing nausea and vomiting are not the purposes of chelation therapy.

15. What type of cultural concern is the most likely deterrent to many women seeking prenatal care? A. Religion B. Modesty C. Ignorance D. Belief that physicians are evil

ANS: B A concern for modesty is a deterrent to many women seeking prenatal care. For some women, exposing body parts, especially to a man, is considered a major violation of their modesty. Many cultural variations are found in prenatal care. Even if the prenatal care described is familiar to a woman, some practices may conflict with the beliefs and practices of a subculture group to which she belongs.

20. With regard to nutritional needs during lactation, a maternity nurse should be aware that: A. the mother's intake of vitamin C, zinc, and protein now can be lower than during pregnancy. B. caffeine consumed by the mother accumulates in the infant, who may be unusually active and wakeful. C. critical iron and folic acid levels must be maintained. D. lactating women can go back to their prepregnant calorie intake.

ANS: B A lactating woman needs to avoid consuming too much caffeine. Vitamin C, zinc, and protein levels need to be moderately higher during lactation than during pregnancy. The recommendations for iron and folic acid are lower during lactation. Lactating women should consume about 500 kcal more than their prepregnancy intake, at least 1800 kcal daily overall.

25. A maternal indication for the use of forceps is: a. a wide pelvic outlet. b. maternal exhaustion. c. a history of rapid deliveries. d. failure to progress past 0 station.

ANS: B A mother who is exhausted may be unable to assist with the expulsion of the fetus.The patient with a wide pelvic outlet will likely not require vacuum extraction. With a rapid delivery, vacuum extraction is not necessary. A station of 0 is too high for a vacuum extraction.

An appropriate nursing intervention to minimize separation anxiety in a hospitalized toddler is to: a. Provide for privacy. b. Encourage parents to room in. c. Explain procedures and routines. d. Encourage contact with children the same age.

ANS: B A toddler experiences separation anxiety secondary to being separated from the parents. To avoid this, the parents should be encouraged to room in as much as possible. Maintaining routines and ensuring privacy are helpful interventions, but they would not substitute for the parents. Contact with same-aged children would not substitute for having the parents present.

22. Care management of a woman diagnosed with acute pelvic inflammatory disease (PID) most likely would include: A. oral antiviral therapy. B. bed rest in a semi-Fowler position. C. antibiotic regimen continued until symptoms subside. D. frequent pelvic examination to monitor the progress of healing.

ANS: B A woman with acute PID should be on bed rest in a semi-Fowler position. Broad-spectrum antibiotics are used. Antibiotics must be taken as prescribed, even if symptoms subside. Few pelvic examinations should be conducted during the acute phase of the disease.

6. Which statement made by a lactating woman would lead the nurse to believe that the woman might have lactose intolerance? A. "I always have heartburn after I drink milk." B. "If I drink more than a cup of milk, I usually have abdominal cramps and bloating." C. "Drinking milk usually makes me break out in hives." D. "Sometimes I notice that I have bad breath after I drink a cup of milk."

ANS: B Abdominal cramps and bloating are consistent with lactose intolerance. One problem that can interfere with milk consumption is lactose intolerance, which is the inability to digest milk sugar because of a lack of the enzyme lactase in the small intestine. Milk consumption may cause abdominal cramping, bloating, and diarrhea in people who are lactose intolerant, although many affected individuals can tolerate small amounts of milk without symptoms.

36. A woman who is gravida 3 para 2 enters the intrapartum unit. The most important nursing assessments are: a. contraction pattern, amount of discomfort, and pregnancy history. b. fetal heart rate, maternal vital signs, and the woman's nearness to birth. c. identification of ruptured membranes, the woman's gravida and para, and her support person. d. last food intake, when labor began, and cultural practices the couple desires.

ANS: B All options describe relevant intrapartum nursing assessments; however, this focused assessment has priority. If the maternal and fetal conditions are normal and birth is not imminent, other assessments can be performed in an unhurried manner. This includes: gravida, para, support person, pregnancy history, pain assessment, last food intake, and cultural practices.

8. The musculoskeletal system adapts to the changes that occur during pregnancy. A woman can expect to experience what change? A. Her center of gravity will shift backward. B. She will have increased lordosis. C. She will have increased abdominal muscle tone. D. She will notice decreased mobility of her pelvic joints.

ANS: B An increase in the normal lumbosacral curve (lordosis) develops, and a compensatory curvature in the cervicodorsal region develops to help the woman maintain her balance. The center of gravity shifts forward. She will have decreased muscle tone. She will notice increased mobility of her pelvic joints.

9. A 31-year-old woman believes that she may be pregnant. She took an OTC pregnancy test 1 week ago after missing her period; the test was positive. During her assessment interview, the nurse enquires about the woman's last menstrual period and asks whether she is taking any medications. The woman states that she takes medicine for epilepsy. She has been under considerable stress lately at work and has not been sleeping well. She also has a history of irregular periods. Her physical examination does not indicate that she is pregnant. She has an ultrasound scan that reveals she is not pregnant. What is the most likely cause of the false-positive pregnancy test result? A. She took the pregnancy test too early. B. She takes anticonvulsants. C. She has a fibroid tumor. D. She has been under considerable stress and has a hormone imbalance.

ANS: B Anticonvulsants may cause false-positive pregnancy test results. OTC pregnancy tests use enzyme-linked immunosorbent assay technology, which can yield positive results 4 days after implantation. Implantation occurs 6 to 10 days after conception. If the woman were pregnant, she would be into her third week at this point (having missed her period 1 week ago). Fibroid tumors do not produce hormones and have no bearing on hCG pregnancy tests. Although stress may interrupt normal hormone cycles (menstrual cycles), it does not affect human chorionic gonadotropin levels or produce positive pregnancy test results.

24. Three servings of milk, yogurt, or cheese plus two servings of meat, poultry, or fish adequately supply the recommended amount of protein for a pregnant woman. Many patients are concerned about the increased levels of mercury in fish and may be afraid to include this source of nutrients in their diet. Sound advice by the nurse to assist the patient in determining which fish is safe to consume would include: A. white tuna is a preferred choice. B. avoid shark, swordfish, and mackerel. C. fish caught in local waterways are the safest. D. salmon and shrimp contain high levels of mercury.

ANS: B As a precaution, the pregnant patient should avoid eating all of these and the less common tilefish. High levels of mercury can harm the developing nervous system of the fetus. It is essential for the nurse to assist the patient in understanding the differences between numerous sources of this product. A pregnant patient can take 12 ounces a week of canned light tuna; however, canned white, albacore, or tuna steaks contain higher levels of mercury and should be limited to no more than 6 ounces per week. It is a common misconception that fish caught in local waterways are the safest. Pregnant women and mothers of young children should check with local advisories about the safety of fish caught by families and friends in nearby bodies of water. If no information is available, these fish sources should be avoided, limited to less than 6 ounces, or the only fish consumed that week. Commercially caught fish that are low in mercury include salmon, shrimp, pollock, or catfish.

21. To reassure and educate pregnant patients about changes in their cardiovascular system, maternity nurses should be aware that: A. a pregnant woman experiencing disturbed cardiac rhythm, such as sinus arrhythmia requires close medical and obstetric observation, no matter how healthy she otherwise may appear. B. changes in heart size and position and increases in blood volume create auditory changes from 20 weeks to term. C. palpitations are twice as likely to occur in twin gestations. D. all of the above changes will likely occur.

ANS: B Auscultatory changes should be discernible after 20 weeks of gestation. A healthy woman with no underlying heart disease does not need any therapy. The maternal heart rate increases in the third trimester, but palpitations may not occur. Auditory changes are discernible at 20 weeks.

19. The U.S. Centers for Disease Control and Prevention (CDC) recommends that HPV be treated with patient-applied: A. miconazole ointment. B. topical podofilox 0.5% solution or gel. C. penicillin given intramuscularly for two doses. D. metronidazole by mouth.

ANS: B Available treatments are imiquimod, podophyllin, and podofilox. Miconazole ointment is used to treat athlete's foot. Intramuscular penicillin is used to treat syphilis. Metronidazole is used to treat bacterial vaginosis.

19. While caring for the patient who requires an induction of labor, the nurse should be cognizant that: a. ripening the cervix usually results in a decreased success rate for induction. b. labor sometimes can be induced with balloon catheters or laminaria tents. c. oxytocin is less expensive than prostaglandins and more effective but creates greater health risks. d. amniotomy can be used to make the cervix more favorable for labor.

ANS: B Balloon catheters or laminaria tents are mechanical means of ripening the cervix. Ripening the cervix, making it softer and thinner, increases the success rate of induced labor. Prostaglandin E1 is less expensive and more effective than oxytocin but carries a greater risk. Amniotomy is the artificial rupture of membranes, which is used to induce labor only when the cervix is already ripe.

23. Which contraceptive method best protects against sexually transmitted infections (STIs) and human immunodeficiency virus (HIV)? A. Periodic abstinence B. Barrier methods C. Hormonal methods D. They all offer about the same protection.

ANS: B Barrier methods such as condoms best protect against STIs and HIV. Periodic abstinence and hormonal methods (the pill) offer no protection against STIs or HIV.

24. It is important for the nurse to develop a realistic birth plan with the pregnant woman in her care. The nurse can explain that a major advantage of nonpharmacologic pain management is: a. greater and more complete pain relief is possible. b. no side effects or risks to the fetus are involved. c. the woman remains fully alert at all times. d. a more rapid labor is likely.

ANS: B Because nonpharmacologic pain management does not include analgesics, adjunct drugs, or anesthesia, it is harmless to the mother and the fetus. There is less pain relief with nonpharmacologic pain management during childbirth. The woman's alertness is not altered by medication; however, the increase in pain will decrease alertness. Pain management may or may not alter the length of

13. A woman who has a seizure disorder and takes barbiturates and phenytoin sodium daily asks the nurse about the pill as a contraceptive choice. The nurse's most appropriate response would be: A. "This is a highly effective method, but it has some side effects." B. "Your current medications will reduce the effectiveness of the pill." C. "The pill will reduce the effectiveness of your seizure medication." D. "This is a good choice for a woman of your age and personal history."

ANS: B Because the liver metabolizes oral contraceptives, their effectiveness is reduced when they are taken simultaneously with anticonvulsants. The statement "Your current medications will reduce the effectiveness of the pill" is true, but it is not the most appropriate response. The anticonvulsant will reduce the effectiveness of the pill, not the other way around. The statement "This is a good choice for a woman of your age and personal history" does not teach the patient that the effectiveness of the pill may be reduced because of her anticonvulsant therapy.

13. Nurses should know some basic definitions concerning preterm birth, preterm labor, and low birth weight. For instance: a. the terms preterm birth and low birth weight can be used interchangeably. b. preterm labor is defined as cervical changes and uterine contractions occurring between 20 and 37 weeks of pregnancy. c. low birth weight is anything below 3.7 lbs. d. in the United States early in this century, preterm birth accounted for 18% to 20% of all births.

ANS: B Before 20 weeks, it is not viable (miscarriage); after 37 weeks, it can be considered term. Although these terms are used interchangeably, they have different meanings: preterm birth describes the length of gestation (37 weeks) regardless of weight; low birth weight describes weight only (2500 g or less) at the time of birth, whenever it occurs. Low birth weight is anything less than 2500 g, or about 5.5 lbs. In 2003 the preterm birth rate in the United States was 12.3%, but it is increasing in frequency.

The nurse is caring for an infant with myelomeningocele scheduled for surgical closure in the morning. Which interventions should the nurse plan for the care of the myelomeningocele sac? a. Open to air b. Covered with a sterile moist nonadherent dressing c. Reinforcement of the original dressing if drainage noted d. A diaper secured over the dressing

ANS: B Before surgical closure, the myelomeningocele is prevented from drying by the application of a sterile, moist, nonadherent dressing over the defect. The moistening solution is usually sterile normal saline. Dressings are changed frequently (every 2 to 4 hours), and the sac is closely inspected for leaks, abrasions, irritation, and any signs of infection. The sac must be carefully cleansed if it becomes soiled or contaminated. The original dressing would not be reinforced but changed as needed. A diaper is not placed over the dressing because stool contamination can occur.

6. Cardiovascular system changes occur during pregnancy. Which finding would be considered normal for a woman in her second trimester? A. Less audible heart sounds (S1, S2) B. Increased pulse rate C. Increased blood pressure D. Decreased red blood cell (RBC) production

ANS: B Between 14 and 20 weeks of gestation, the pulse increases about 10 to 15 beats/min, which persists to term. Splitting of S1 and S2 is more audible. In the first trimester, blood pressure usually remains the same as at the prepregnancy level, but it gradually decreases up to about 20 weeks of gestation. During the second trimester, both the systolic and the diastolic pressures decrease by about 5 to 10 mm Hg. Production of RBCs accelerates during pregnancy.

6. Maternal phenylketonuria (PKU) is an important health concern during pregnancy because: A. it is a recognized cause of preterm labor. B. the fetus may develop neurologic problems. C. a pregnant woman is more likely to die without dietary control. D. women with PKU are usually retarded and should not reproduce.

ANS: B Children born to women with untreated PKU are more likely to be born with mental retardation, microcephaly, congenital heart disease, and low birth weight. Maternal PKU has no effect on labor. Women without dietary control of PKU are more likely to miscarry or bear a child with congenital anomalies. Screening for undiagnosed maternal PKU at the first prenatal visit may be warranted, especially in individuals with a family history of the disorder, with low intelligence of uncertain etiology, or who have given birth to microcephalic infants.

20. Nurses caring for antepartum women with cardiac conditions should be aware that: A. stress on the heart is greatest in the first trimester and the last 2 weeks before labor. B. women with Class II cardiac disease should avoid heavy exertion and any activity that causes even minor symptoms. C. women with Class III cardiac disease should have 8 to 10 hours of sleep every day and limit housework, shopping, and exercise. D. Women with Class I cardiac disease need bed rest through most of the pregnancy and face the possibility of hospitalization near term.

ANS: B Class II cardiac disease is symptomatic with ordinary activity. Women in this category need to avoid heavy exertion and limit regular activities as symptoms dictate. Stress is greatest between weeks 28 and 32, when homodynamic changes reach their maximum. Class III cardiac disease is symptomatic with less than ordinary activity. These women need bed rest most of the day and face the possibility of hospitalization near term. Class I cardiac disease is asymptomatic at normal levels of activity. These women can carry on limited normal activities with discretion, although they still need a good amount of sleep.

28. Which of the following statements about the various forms of hepatitis is accurate? A. A vaccine exists for hepatitis C but not for hepatitis B. B. Hepatitis A is acquired by eating contaminated food or drinking polluted water. C. Hepatitis B is less contagious than human immunodeficiency virus (HIV). D. The incidence of hepatitis C is decreasing.

ANS: B Contaminated milk and shellfish are common sources of infection with hepatitis A. A vaccine exists for hepatitis B but not for hepatitis C. Hepatitis B is more contagious than HIV. The incidence of hepatitis C is increasing.

17. An unmarried young woman describes her sex life as "active" and involving "many" partners. She wants a contraceptive method that is reliable and does not interfere with sex. She requests an intrauterine device (IUD). The nurse's most appropriate response is: A. "The IUD does not interfere with sex." B. "The risk of pelvic inflammatory disease (PID) will be higher for you." C. "The IUD will protect you from sexually transmitted infections (STIs)." D. "Pregnancy rates are high with IUDs."

ANS: B Disadvantages of IUDs include an increased risk of PID in the first 20 days after insertion and the risks of bacterial vaginosis and uterine perforation. The IUD offers no protection against STIs or human immunodeficiency virus. Because this woman has multiple sex partners, she is at higher risk of developing a STI. The IUD does not protect against infection, as does a barrier method. Although the statement "The IUD does not interfere with sex" may be correct, it is not the most appropriate response. The IUD offers no protection from STIs. The typical failure rate of the IUD in the first year of use is 0.8%.

10. In the first trimester, ultrasonography can be used to gain information on: A. amniotic fluid volume. B. location of gestational sacs. C. placental location and maturity. D. cervical length.

ANS: B During the first trimester, ultrasound examination is performed to obtain information regarding the number, size, and location of gestational sacs; the presence or absence of fetal cardiac and body movements; the presences or absence of uterine abnormalities (e.g., bicornuate uterus or fibroids) or adnexal masses (e.g., ovarian cysts or an ectopic pregnancy); and pregnancy dating.

24. During which phase of the cycle of violence does the batterer become contrite and remorseful? A. Battering phase B. Honeymoon phase C. Tension-building phase D. Increased drug-taking phase

ANS: B During the tension-building phase, the batterer becomes increasingly hostile, swears, threatens, and throws things. This is followed by the battering phase where violence actually occurs, and the victim feels powerless. During the honeymoon phase, the victim of IPV wants to believe that the battering will never happen again, and the batterer will promise anything to get back into the home. Often the batterer increases the use of drugs during the tension-building phase.

21. Which statement is the best rationale for assessing the maternal vital signs between uterine contractions? a. During a contraction, assessing the fetal heart rate is the priority. b. Maternal circulating blood volume temporarily increases during contractions. c. Maternal blood flow to the heart is reduced during contractions. d. Vital signs taken during contractions are not accurate.

ANS: B During uterine contractions, blood flow to the placenta temporarily stops, causing a relative increase in the mother's blood volume, which, in turn, temporarily increases blood pressure and slows the pulse. Monitoring fetal responses to the contractions is important; however, this question concerns the maternal vital signs. Maternal blood flow is increased during a contraction. Vital signs are altered by contractions but are considered accurate for that period.

9. During her gynecologic checkup, a 17-year-old girl states that recently she has been experiencing cramping and pain during her menstrual periods. The nurse would document this complaint as: A. amenorrhea. B. dysmenorrhea. C. dyspareunia. D. premenstrual syndrome (PMS).

ANS: B Dysmenorrhea is pain during or shortly before menstruation. Amenorrhea is the absence of menstrual flow. Dyspareunia is pain during intercourse. PMS is a cluster of physical, psychologic, and behavioral symptoms that begin in the luteal phase of the menstrual cycle and resolve within a couple of days of the onset of menses.

4. A pregnant woman experiencing nausea and vomiting should: A. drink a glass of water with a fat-free carbohydrate before getting out of bed in the morning. B. eat small, frequent meals (every 2 to 3 hours). C. increase her intake of high-fat foods to keep the stomach full and coated. D. limit fluid intake throughout the day.

ANS: B Eating small, frequent meals is the correct suggestion for a woman experiencing nausea and vomiting. A pregnant woman experiencing nausea and vomiting should avoid consuming fluids early in the day or when nauseated, but should compensate by drinking fluids at other times. A pregnant woman experiencing nausea and vomiting should reduce her intake of fried and other fatty foods.

19. With regard to systemic analgesics administered during labor, nurses should be aware that: a. systemic analgesics cross the maternal blood-brain barrier as easily as they do the fetal blood-brain barrier. b. effects on the fetus and newborn can include decreased alertness and delayed sucking. c. intramuscular (IM) administration is preferred over intravenous (IV) administration. d. IV patient-controlled analgesia (PCA) results in increased use of an analgesic.

ANS: B Effects depend on the specific drug given, the dosage, and the timing. Systemic analgesics cross the fetal blood-brain barrier more readily than the maternal blood-brain barrier. IV administration is preferred over IM administration because the drug acts faster and more predictably. PCA results in decreased use of an analgesic.

Which situation poses the greatest challenge to the nurse working with a child and family? a. Twenty-four-hour observation b. Emergency hospitalization c. Outpatient admission d. Rehabilitation admission

ANS: B Emergency hospitalization involves (1) limited time for preparation both for the child and family, (2) situations that cause fear for the family that the child may die or be permanently disabled, and (3) a high level of activity, which can foster further anxiety. Although preparation time may be limited with a 24-hour observation, this situation does not usually involve the acuteness of the situation and the high levels of anxiety associated with emergency admission. Outpatient admission generally involves preparation time for the family and child. Because of the lower level of acuteness in this setting, anxiety levels are not as high. Rehabilitation admission follows a serious illness or disease. This type of unit may resemble a home environment, which decreases the childs and familys anxiety.

11. A married couple is discussing alternatives for pregnancy prevention and has asked about fertility awareness methods (FAMs). The nurse's most appropriate reply is: A. "They're not very effective, and it's very likely you'll get pregnant." B. "They can be effective for many couples, but they require motivation." C. "These methods have a few advantages and several health risks." D. "You would be much safer going on the pill and not having to worry."

ANS: B FAMs are effective with proper vigilance about ovulatory changes in the body and adherence to coitus intervals. They are effective if used correctly by a woman with a regular menstrual cycle. The typical failure rate for all FAMs is 25% during the first year of use. FAMs have no associated health risks. The use of birth control has associated health risks. In addition, taking a pill daily requires compliance on the patient's part.

6. With regard to the assessment of female, male, and couple infertility, nurses should be aware that: A. the couple's religious, cultural, and ethnic backgrounds provide emotional clutter that does not affect the clinical scientific diagnosis. B. the investigation takes 3 to 4 months and a significant financial investment. C. the woman is assessed first; if she is not the problem, the male partner is analyzed. D. semen analysis is for men; the postcoital test is for women.

ANS: B Fertility assessment and diagnosis take time, money, and commitment from the couple. Religious, cultural, and ethnic-bred attitudes about fertility and related issues always have an impact on diagnosis and assessment. Both partners are assessed systematically and simultaneously, as individuals and as a couple. Semen analysis is for men, but the postcoital test is for the couple.

24. To reassure and educate pregnant patients about the functioning of their kidneys in eliminating waste products, maternity nurses should be aware that: A. increased urinary output makes pregnant women less susceptible to urinary infection. B. increased bladder sensitivity and then compression of the bladder by the enlarging uterus results in the urge to urinate even if the bladder is almost empty. C. renal (kidney) function is more efficient when the woman assumes a supine position. D. using diuretics during pregnancy can help keep kidney function regular.

ANS: B First bladder sensitivity and then compression of the bladder by the uterus result in the urge to urinate more often. Numerous anatomic changes make a pregnant woman more susceptible to urinary tract infection. Renal function is more efficient when the woman lies in the lateral recumbent position and less efficient when she is supine. Diuretic use during pregnancy can overstress the system and cause problems.

15. Which time-based description of a stage of development in pregnancy is accurate? A. Viability—22 to 37 weeks since the last menstrual period (LMP) (assuming a fetal weight >500 g). B. Full Term—Pregnancy from the beginning of week 39 of gestation to the end of week 40. C. Preterm—Pregnancy from 20 to 28 weeks. D. Postdate—Pregnancy that extends beyond 38 weeks.

ANS: B Full Term is 39 to 40 weeks of gestation. Viability is the ability of the fetus to live outside the uterus before coming to term, or 22 to 24 weeks since LMP. Preterm is 20 to 37 weeks of gestation. Postdate or postterm is a pregnancy that extends beyond 42 weeks or what is considered the limit of full term.

21. With regard to spinal and epidural (block) anesthesia, nurses should know that: a. this type of anesthesia is commonly used for cesarean births but is not suitable for vaginal births. b. a high incidence of after-birth headache is seen with spinal blocks. c. epidural blocks allow the woman to move freely. d. spinal and epidural blocks are never used together.

ANS: B Headaches may be prevented or mitigated to some degree by a number of methods. Spinal blocks may be used for vaginal births, but the woman must be assisted through labor. Epidural blocks limit the woman's ability to move freely. Combined use of spinal and epidural blocks is becoming increasingly popular.

4. A woman enquires about herbal alternative methods for improving fertility. Which statement by the nurse is the most appropriate when instructing the patient in which herbal preparations to avoid while trying to conceive? A. "You should avoid nettle leaf, dong quai, and vitamin E while you are trying to get pregnant." B. "You may want to avoid licorice root, lavender, fennel, sage, and thyme while you are trying to conceive." C. "You should not take anything with vitamin E, calcium, or magnesium. They will make you infertile." D. "Herbs have no bearing on fertility."

ANS: B Herbs that a woman should avoid while trying to conceive include licorice root, yarrow, wormwood, ephedra, fennel, golden seal, lavender, juniper, flaxseed, pennyroyal, passionflower, wild cherry, cascara, sage, thyme, and periwinkle. Nettle leaf, dong quai, and vitamin E all promote fertility. Vitamin E, calcium, and magnesium may promote fertility and conception. All supplements and herbs should be purchased from trusted sources.

22. What is the primary difference between the labor of a nullipara and that of a multipara? a. Amount of cervical dilation b. Total duration of labor c. Level of pain experienced d. Sequence of labor mechanisms

ANS: B In a first-time pregnancy, the descent is usually slow but steady; in subsequent pregnancies, the descent is more rapid, resulting in a shorter duration of labor. Cervical dilation is the same for all labors. The level of pain is individual to the woman, not to the number of labors she has experienced. The sequence of labor mechanisms is the same with all labors.

4. A woman is at 14 weeks of gestation. The nurse would expect to palpate the fundus at which level? A. Not palpable above the symphysis at this time B. Slightly above the symphysis pubis C. At the level of the umbilicus D. Slightly above the umbilicus

ANS: B In normal pregnancies, the uterus grows at a predictable rate. It may be palpated above the symphysis pubis sometime between the 12th and 14th weeks of pregnancy. As the uterus grows, it may be palpated above the symphysis pubis sometime between the 12th and 14th weeks of pregnancy. The uterus rises gradually to the level of the umbilicus at 22 to 24 weeks of gestation.

The nurse is caring for a neonate born with a myelomeningocele. Surgery to repair the defect is scheduled the next day. The most appropriate way to position and feed this neonate is which position? a. Prone and tube-fed b. Prone, head turned to side, and nipple-fed c. Supine in an infant carrier and nipple-fed d. Supine, with defect supported with rolled blankets, and nipple-fed

ANS: B In the prone position, feeding is a problem. The infants head is turned to one side for feeding. If the child is able to nipple-feed, tube feeding is not needed. Before surgery, the infant is kept in the prone position to minimize tension on the sac and risk of trauma.

How much folic acid is recommended for women of childbearing age? a. 1.0 mg b. 0.4 mg c. 1.5 mg d. 2.0 mg

ANS: B It has been estimated that a daily intake of 0.4 mg of folic acid in women of childbearing age will prevent 50% to 70% of cases of neural tube defects; 1.0 mg is too low a dose; 1.5 to 2.0 mg are not the recommended dosages of folic acid.

27. What correctly matches the type of deceleration with its likely cause? a. Early deceleration—umbilical cord compression b. Late deceleration—uteroplacental inefficiency c. Variable deceleration—head compression d. Prolonged deceleration—cause unknown

ANS: B Late deceleration is caused by uteroplacental inefficiency. Early deceleration is caused by head compression. Variable deceleration is caused by umbilical cord compression. Prolonged deceleration has a variety of either benign or critical causes.

30. The most important reason for evaluating the pattern of weight gain in pregnancy is to: A. prevent excessive adipose tissue deposits. B. identify potential nutritional problems or complications of pregnancy. C. assess the need to limit caloric intake in obese women. D. determine cultural influences on the woman's diet.

ANS: B Maternal and fetal risks in pregnancy are increased when the mother is significantly overweight. Excessive adipose tissue may occur with excess weight gain; however, this is not the reason for monitoring the weight gain pattern. It is important to monitor the pattern of weight gain to identify complications. The pattern of weight gain is not influenced by cultural influences.

27. A woman is 15 weeks pregnant with her first baby. She asks how long it will be before she feels the baby move. The best answer is: A. "You should have felt the baby move by now." B. "Within the next month, you should start to feel fluttering sensations." C. "The baby is moving; however, you can't feel it yet." D. "Some babies are quiet, and you don't feel them move."

ANS: B Maternal perception of fetal movement usually begins 16 to 20 weeks after conception. Because this is her first pregnancy, movement is felt toward the later part of the 16- to 20-week time period. Stating that "you should have felt the baby move by now" is incorrect and may be alarming to the patient. Fetal movement should be felt by 16 to 20 weeks. If movement is not felt by the end of that time, further assessment will be necessary.

11. During the first trimester, a woman can expect which of the following changes in her sexual desire? A. An increase, because of enlarging breasts B. A decrease, because of nausea and fatigue C. No change D. An increase, because of increased levels of female hormones

ANS: B Maternal physiologic changes such as breast enlargement, nausea, fatigue, abdominal changes, perineal enlargement, leukorrhea, pelvic vasocongestion, and orgasmic responses may affect sexuality and sexual expression. Libido may be depressed in the first trimester but often increases during the second and third trimesters. During pregnancy, the breasts may become enlarged and tender; this tends to interfere with coitus, decreasing the desire to engage in sexual activity.

6. While interviewing a 31-year-old woman before her routine gynecologic examination, the nurse collects data about the patient's recent menstrual cycles. The nurse should collect additional information associated with which patient statement? TA. he woman says her menstrual flow lasts 5 to 6 days. B. She describes her flow as very heavy. C. She reports that she has had a small amount of spotting midway between her periods for the past 2 months. D. She says the length of her menstrual cycle varies from 26 to 29 days.

ANS: B Menorrhagia is defined as excessive menstrual bleeding, in either duration or amount. Heavy bleeding can have many causes. The amount of bleeding and its effect on daily activities should be evaluated. A menstrual flow lasting 5 to 6 days is a normal finding. Mittlestaining, a small amount of bleeding or spotting that occurs at the time of ovulation (14 days before onset of the next menses), is considered normal. During her reproductive years, a woman may have physiologic variations in her menstrual cycle. Variations in the length of a menstrual cycle are considered normal.

16. The use of methamphetamine (meth) has been described as a significant drug problem in the United States. In order to provide adequate nursing care to this patient population the nurse must be cognizant that methamphetamine: A. is similar to opiates. B. is a stimulant with vasoconstrictive characteristics. C. should not be discontinued during pregnancy. D. is associated with a low rate of relapse.

ANS: B Methamphetamines are stimulants with vasoconstrictive characteristics similar to cocaine and are used similarly. As is the case with cocaine users, methamphetamine users are urged to immediately stop all use during pregnancy. Unfortunately, because methamphetamine users are extremely psychologically addicted, the rate of relapse is very high.

9. What is the correct term describing the slight overlapping of cranial bones or shaping of the fetal head during labor? a. Lightening b. Molding c. Ferguson reflex d. Valsalva maneuver

ANS: B Molding also permits adaptation to various diameters of the maternal pelvis. Lightening is the mother's sensation of decreased abdominal distention, which usually occurs the week before labor. The Ferguson reflex is the contraction urge of the uterus after the stimulation of the cervix. The Valsalva maneuver describes conscious pushing during the second stage of labor.

Which is often administered to prevent or control hemorrhage in a child with cancer? a. Nitrosoureas b. Platelets c. Whole blood d. Corticosteroids

ANS: B Most bleeding episodes can be prevented or controlled with the administration of platelet concentrate or platelet-rich plasma. Nitrosoureas, whole blood, and corticosteroids would not prevent or control hemorrhage.

35. The primary difference between the labor of a nullipara and that of a multipara is the: a. amount of cervical dilation. b. total duration of labor. c. level of pain experienced. d. sequence of labor mechanisms.

ANS: B Multiparas usually labor more quickly than nulliparas, thus making the total duration of their labor shorter. Cervical dilation is the same for all labors. The level of pain is individual to the woman, not to the number of labors she has experienced. The sequence of labor mechanisms remains the same with all labors.

30. A patient in her first trimester complains of nausea and vomiting. She asks, "Why does this happen?" The nurse's best response is: A. "It is due to an increase in gastric motility." B. "It may be due to changes in hormones." C. "It is related to an increase in glucose levels." D. "It is caused by a decrease in gastric secretions."

ANS: B Nausea and vomiting are believed to be caused by increased levels of hormones, decreased gastric motility, and hypoglycemia. Gastric motility decreases during pregnancy. Glucose levels decrease in the first trimester. Although gastric secretions decrease, this is not the main cause of nausea and vomiting.

14. Maternal nutritional status is an especially significant factor of the many factors that influence the outcome of pregnancy because: A. it is very difficult to adjust because of people's ingrained eating habits. B. it is an important preventive measure for a variety of problems. C. women love obsessing about their weight and diets. D. a woman's preconception weight becomes irrelevant.

ANS: B Nutritional status draws so much attention not only for its effect on a healthy pregnancy and birth but also because significant changes are within relatively easy reach.

15. The nurse expects to administer an oxytocic (e.g., Pitocin, Methergine) to a woman after expulsion of her placenta to: a. relieve pain. b. stimulate uterine contraction. c. prevent infection. d. facilitate rest and relaxation.

ANS: B Oxytocics stimulate uterine contractions, which reduce blood loss after the third stage of labor. Oxytocics are not used to treat pain or prevent infection. They cause the uterus to contract, which reduces blood loss. Oxytocics do not facilitate rest and relaxation.

21. Certain fatty acids classified as hormones that are found in many body tissues and that have roles in many reproductive functions are known as: A. gonadotropin-releasing hormone (GnRH). B. prostaglandins (PGs). C. follicle-stimulating hormone (FSH). D. luteinizing hormone (LH).

ANS: B PGs affect smooth muscle contraction and changes in the cervix. GnRH, FSH, and LH are part of the hypothalamic-pituitary cycle, which responds to the rise and fall of estrogen and progesterone.

12. A new mother with which of these thyroid disorders would be strongly discouraged from breastfeeding? A. Hyperthyroidism B. Phenylketonuria (PKU) C. Hypothyroidism D. Thyroid storm

ANS: B PKU is a cause of mental retardation in infants; mothers with PKU pass on phenylalanine. A woman with hyperthyroidism or hypothyroidism would have no particular reason not to breastfeed. A thyroid storm is a complication of hyperthyroidism.

8. A pregnant woman at 18 weeks of gestation calls the clinic to report that she has been experiencing occasional backaches of mild-to-moderate intensity. The nurse would recommend that she: A. do Kegel exercises. B. do pelvic rock exercises. C. use a softer mattress. D. stay in bed for 24 hours.

ANS: B Pelvic rock exercises may help stretch and strengthen the abdominal and lower back muscles and relieve low back pain. Kegel exercises increase the tone of the pelvic area, not the back. A softer mattress may not provide the support needed to maintain proper alignment of the spine and may contribute to back pain. Stretching and other exercises to relieve back pain should be performed several times a day.

9. Physiologically, sexual response can be characterized by: A. coitus, masturbation, and fantasy. B. myotonia and vasocongestion. C. erection and orgasm. D. excitement, plateau, and orgasm.

ANS: B Physiologically, according to Masters (1992), sexual response can be analyzed in terms of two processes: vasocongestion and myotonia. Coitus, masturbation, and fantasy are forms of stimulation for the physical manifestation of the sexual response. Erection and orgasm occur in two of the four phases of the sexual response cycle. Excitement, plateau, and orgasm are three of the four phases of the sexual response cycle.

14. Risk factors tend to be interrelated and cumulative in their effect. While planning the care for a laboring patient with diabetes mellitus, the nurse is aware that she is at a greater risk for: A. oligohydramnios. B. polyhydramnios. C. postterm pregnancy. D. Chromosomal abnormalities.

ANS: B Polyhydramnios (amniotic fluid >2000 mL) is 10 times more likely to occur in diabetic compared with nondiabetic pregnancies. Polyhydramnios puts the mother at risk for premature rupture of membranes, premature labor, and after birth hemorrhage. Prolonged rupture of membranes, intrauterine growth restriction, intrauterine fetal death, and renal agenesis (Potter syndrome) all put the patient at risk for developing oligohydramnios. Anencephaly, placental insufficiency, and perinatal hypoxia all contribute to the risk for postterm pregnancy. Maternal age older than 35 years and balanced translocation (maternal and paternal) are risk factors for chromosome abnormalities.

16. With regard to dysfunctional labor, nurses should be aware that: a. women who are underweight are more at risk. b. women experiencing precipitous labor are about the only "dysfunctionals" not to be exhausted. c. hypertonic uterine dysfunction is more common than hypotonic dysfunction. d. abnormal labor patterns are most common in older women.

ANS: B Precipitous labor lasts less than 3 hours. Short women more than 30 lbs overweight are more at risk for dysfunctional labor. Hypotonic uterine dysfunction, in which the contractions become weaker, is more common. Abnormal labor patterns are more common in women less than 20 years of age.

2. Preconception counseling is critical to the outcome of diabetic pregnancies because poor glycemic control before and during early pregnancy is associated with: A. frequent episodes of maternal hypoglycemia. B. congenital anomalies in the fetus. C. polyhydramnios. D. hyperemesis gravidarum.

ANS: B Preconception counseling is particularly important because strict metabolic control before conception and in the early weeks of gestation is instrumental in decreasing the risks of congenital anomalies. Frequent episodes of maternal hypoglycemia may occur during the first trimester (not before conception) as a result of hormone changes and the effects on insulin production and usage. Hydramnios occurs about 10 times more often in diabetic pregnancies than in nondiabetic pregnancies. Typically it is seen in the third trimester of pregnancy. Hyperemesis gravidarum may exacerbate hypoglycemic events because the decreased food intake by the mother and glucose transfer to the fetus contributes to hypoglycemia.

1. The nurse caring for a newly pregnant woman would advise her that ideally prenatal care should begin: A. before the first missed menstrual period. B. after the first missed menstrual period. C. after the second missed menstrual period. D. after the third missed menstrual period.

ANS: B Prenatal care ideally should begin soon after the first missed menstrual period. Regular prenatal visits offer opportunities to ensure the health of the expectant mother and her infant.

4. A 41-week pregnant multigravida presents in the labor and delivery unit after a nonstress test indicated that her fetus could be experiencing some difficulties in utero. Which diagnostic tool would yield more detailed information about the fetus? A. Ultrasound for fetal anomalies B. Biophysical profile (BPP) C. Maternal serum alpha-fetoprotein (MSAFP) screening D. Percutaneous umbilical blood sampling (PUBS)

ANS: B Real-time ultrasound permits detailed assessment of the physical and physiologic characteristics of the developing fetus and cataloging of normal and abnormal biophysical responses to stimuli. BPP is a noninvasive, dynamic assessment of a fetus that is based on acute and chronic markers of fetal disease. An ultrasound for fetal anomalies would most likely have been performed earlier in the pregnancy. It is too late in the pregnancy to perform MSAFP screening. Also, MSAFP screening does not provide information related to fetal well-being. Indications for PUBS include prenatal diagnosis or inherited blood disorders, karyotyping of malformed fetuses, detection of fetal infection, determination of the acid-base status of a fetus with IUGR, and assessment and treatment of isoimmunization and thrombocytopenia in the fetus.

14. What three measures should the nurse implement to provide intrauterine resuscitation? Select the response that best indicates the priority of actions that should be taken. a. Call the provider, reposition the mother, and perform a vaginal examination. b. Reposition the mother, increase intravenous (IV) fluid, and provide oxygen via face mask. c. Administer oxygen to the mother, increase IV fluid, and notify the care provider. d. Perform a vaginal examination, reposition the mother, and provide oxygen via face mask.

ANS: B Repositioning the mother, increasing intravenous (IV) fluid, and providing oxygen via face mask are correct nursing actions for intrauterine resuscitation. The nurse should initiate intrauterine resuscitation in an ABC manner, similar to basic life support. The first priority is to open the maternal and fetal vascular systems by repositioning the mother for improved perfusion. The second priority is to increase blood volume by increasing the IV fluid. The third priority is to optimize oxygenation of the circulatory volume by providing oxygen via face mask. If these interventions do not resolve the fetal heart rate issue quickly, the primary provider should be notified immediately.

10. A first-time mother is concerned about the type of medications she will receive during labor. She is in a fair amount of pain and is nauseous. In addition, she appears to be very anxious. You explain that opioid analgesics are often used with sedatives because: a. "The two together work the best for you and your baby." b. "Sedatives help the opioid work better, and they also will assist you to relax and relieve your nausea." c. "They work better together so you can sleep until you have the baby." d. "This is what the doctor has ordered for you."

ANS: B Sedatives can be used to reduce the nausea and vomiting that often accompany opioid use. In addition, some ataractics reduce anxiety and apprehension and potentiate the opioid analgesic affects. A potentiator may cause the two drugs to work together more effectively, but it does not ensure maternal or fetal complications will not occur. Sedation may be a related effect of some ataractics, but it is not the goal. Furthermore, a woman is unlikely to be able to sleep through transitional labor and birth. "This is what the doctor has ordered for you" may be true, but it is not an acceptable comment for the nurse to make.

27. Which finding in the urine analysis of a pregnant woman is considered a variation of normal? A. Proteinuria B. Glycosuria C. Bacteria in the urine D. Ketones in the urine

ANS: B Small amounts of glucose may indicate "physiologic spilling." The presence of protein could indicate kidney disease or preeclampsia. Urinary tract infections are associated with bacteria in the urine. An increase in ketones indicates that the patient is exercising too strenuously or has an inadequate fluid and food intake.

2. The nurse providing care for a woman with preterm labor who is receiving terbutaline would include which intervention to identify side effects of the drug? a. Assessing deep tendon reflexes (DTRs) b. Assessing for chest discomfort and palpitations c. Assessing for bradycardia d. Assessing for hypoglycemia

ANS: B Terbutaline is a β2-adrenergic agonist that affects the cardiopulmonary and metabolic systems of the mother. Signs of cardiopulmonary decompensation would include chest pain and palpitations. Assessing DTRs would not address these concerns. β2-Adrenergic agonist drugs cause tachycardia, not bradycardia. The metabolic effect leads to hyperglycemia, not hypoglycemia.

40. A nurse is preparing to administer chemotherapy to a child who has an Infuse-a-Port. Which action by the nurse is the most appropriate? A. Flush the catheter with normal saline and heparin. B. Obtain a Huber needle prior to administration. C. Unclamp the catheter prior to flushing the line. D. Wrap the catheter in gauze so it doesn't pull out.

ANS: B A centrally implanted port, such as an Infuse-a-Port, must be accessed with a Huber needle. Prior to administering medication is not the time to flush with heparin. The port is entirely indwelling, so there is no catheter to unclamp, nor will the device pull out.

24. Nurses can help their clients by keeping them informed about the distinctive stages of labor. Which description of the phases of the first stage of labor is accurate? a. Latent: Mild, regular contractions; no dilation; bloody show; duration of 2 to 4 hours b. Active: Moderate, regular contractions; 4- to 7-cm dilation; duration of 3 to 6 hours c. Lull: No contractions; dilation stable; duration of 20 to 60 minutes d. Transition: Very strong but irregular contractions; 8- to 10-cm dilation; duration of 1 to 2 hours

ANS: B The active phase is characterized by moderate, regular contractions; 4- to 7-cm dilation; and a duration of 3 to 6 hours. The latent phase is characterized by mild-to-moderate and irregular contractions; dilation up to 3 cm; brownish-to-pale pink mucus, and a duration of 6 to 8 hours. No official "lull" phase exists in the first stage. The transition phase is characterized by strong- to-very strong and regular contractions; 8- to 10-cm dilation; and a duration of 20 to 40 minutes.

20. Nurses can help their patients by keeping them informed about the distinctive stages of labor. Which description of the phases of the first stage of labor is accurate? .a Latent: Mild, regular contractions; no dilation; bloody show; duration of 2 to 4 hours b. Active: Moderate, regular contractions; 4- to 7-cm dilation; duration of 3 to 6 hours c. Lull: No contractions; dilation stable; duration of 20 to 60 minutes d. Transition: Very strong but irregular contractions; 8- to 10-cm dilation; duration of 1 to 2 hours

ANS: B The active phase is characterized by moderate, regular contractions; 4- to 7-cm dilation; and a duration of 3 to 6 hours. The latent phase is characterized by mild-to-moderate, irregular contractions; dilation up to 3 cm; brownish-to-pale pink mucus, and a duration of 6 to 8 hours. No official "lull" phase exists in the first stage. The transition phase is characterized by strong to very strong, regular contractions; 8- to 10-cm dilation; and a duration of 20 to 40 minutes.

5. The nurse should be aware that an effective plan to achieve adequate pain relief without maternal risk is most effective if: a. the mother gives birth without any analgesic or anesthetic. b. the mother and family's priorities and preferences are incorporated into the plan. c. the primary health care provider decides the best pain relief for the mother and family. d. the nurse informs the family of all alternative methods of pain relief available in the hospital setting.

ANS: B The assessment of the woman, her fetus, and her labor is a joint effort of the nurse and the primary health care providers, who consult with the woman about their findings and recommendations. The needs of each woman are different and many factors must be considered before a decision is made whether pharmacologic methods, nonpharmacologic methods, or a combination of the two will be used to manage labor pain.

24. During labor a fetus with an average heart rate of 135 beats/min over a 10-minute period would be considered to have: a. bradycardia. b. a normal baseline heart rate. c. tachycardia. d. hypoxia.

ANS: B The baseline heart rate is measured over 10 minutes; a normal range is 110 to 160 beats/min. Bradycardia is a fetal heart rate (FHR) below 110 beats/min for 10 minutes or longer. Tachycardia is an FHR over 160 beats/min for 10 minutes or longer. Hypoxia is an inadequate supply of oxygen; no indication of this condition exists with a baseline heart rate in the normal range.

Which immunization should be given with caution to children infected with human immunodeficiency virus (HIV)? a. Influenza b. Varicella c. Pneumococcal d. Inactivated poliovirus (IPV)

ANS: B The children should be carefully evaluated before being given live viral vaccines such as varicella, measles, mumps, and rubella. The child must be immunocompetent and not have contact with other severely immunocompromised individuals. Influenza, pneumococcal, and inactivated poliovirus (IPV) are not live vaccines.

1. A woman's obstetric history indicates that she is pregnant for the fourth time and all of her children from previous pregnancies are living. One was born at 39 weeks of gestation, twins were born at 34 weeks of gestation, and another child was born at 35 weeks of gestation. What is her gravidity and parity using the GTPAL system? a. 3-1-1-1-3 b. 4-1-2-0-4 c. 3-0-3-0-3 d. 4-2-1-0-3

ANS: B The correct calculation of this woman's gravidity and parity is 4-1-2-0-4. The numbers reflect the woman's gravidity and parity information. Using the GPTAL system, her information is calculated as:G: The first number reflects the total number of times the woman has been pregnant; she is pregnant for the fourth time.T: This number indicates the number of pregnancies carried to term, not the number of deliveries at term; only one of her pregnancies has resulted in a fetus at term.P: This is the number of pregnancies that resulted in a preterm birth; the woman has had two pregnancies in which she delivered preterm.A: This number signifies whether the woman has had any abortions or miscarriages before the period of viability; she has not.L: This number signifies the number of children born who are currently living; the woman has four children.

5. A man's wife is pregnant for the third time. One child was born with cystic fibrosis, and the other child is healthy. The man wonders what the chance is that this child will have cystic fibrosis. This type of testing is known as: A. occurrence risk. B. recurrence risk. C. predictive testing. D. predisposition testing.

ANS: B The couple already has a child with a genetic disease so they will be given a recurrence risk test. If a couple has not yet had children but are known to be at risk for having children with a genetic disease, they are given an occurrence risk test. Predictive testing is used to clarify the genetic status of an asymptomatic family member. Predisposition testing differs from presymptomatic testing in that a positive result does not indicate 100% risk of a condition developing.

12. Through vaginal examination the nurse determines that a woman is 4 cm dilated, and the external fetal monitor shows uterine contractions every 3.5 to 4 minutes. The nurse would report this as: a. first stage, latent phase. b. first stage, active phase. c. first stage, transition phase. d. second stage, latent phase.

ANS: B The first stage, active phase of maternal progress indicates that the woman is in the active phase of the first stage of labor. During the latent phase of the first stage of labor, the expected maternal progress would be 0 to 3 cm dilation with contractions every 5 to 30 minutes. During the transition phase of the first stage of labor, the expected maternal progress is 8 to 10 cm dilation with contractions every 2 to 3 minutes. During the latent phase of the second stage of labor, the woman is completely dilated and experiences a restful period of "laboring down."

A boy with leukemia screams whenever he needs to be turned or moved. Which is the most probable cause of this pain? a. Edema b. Bone involvement c. Petechial hemorrhages d. Changes within the muscles

ANS: B The invasion of the bone marrow with leukemic cells gradually causes a weakening of the bone and a tendency toward fractures. As leukemic cells invade the periosteum, increasing pressure causes severe pain. Edema, petechial hemorrhages, and changes within the muscles would not cause severe pain.

34. A patient whose cervix is dilated to 5 cm is considered to be in which phase of labor? a. Latent phase b. Active phase c. Second stage d. Third stage

ANS: B The latent phase is from the beginning of true labor until 3 cm of cervical dilation. The active phase of labor is characterized by cervical dilation of 4 to 7 cm. The second stage of labor begins when the cervix is completely dilated until the birth of the baby. The third stage of labor is from the birth of the baby until the expulsion of the placenta. This patient is in the active phase of labor.

A school-age child is admitted in vasoocclusive sickle cell crisis. The childs care should include: a. correction of acidosis. b. adequate hydration and pain management. c. pain management and administration of heparin. d. adequate oxygenation and replacement of factor VIII.

ANS: B The management of crises includes adequate hydration, minimization of energy expenditures, pain management, electrolyte replacement, and blood component therapy if indicated. Hydration and pain control are two of the major goals of therapy. The acidosis will be corrected as the crisis is treated. Heparin and factor VIII is not indicated in the treatment of vasoocclusive sickle cell crisis. Oxygen may prevent further sickling, but it is not effective in reversing sickling because it cannot reach the clogged blood vessels.

3. Unique muscle fibers make the uterine myometrium ideally suited for: A. menstruation. B. the birth process. C. ovulation. D. fertilization.

ANS: B The myometrium is made up of layers of smooth muscles that extend in three directions. These muscles assist in the birth process by expelling the fetus, ligating blood vessels after birth, and controlling the opening of the cervical os.

32. For women who have a history of sexual abuse, a number of traumatic memories may be triggered during labor. The woman may fight the labor process and react with pain or anger. Alternately, she may become a passive player and emotionally absent herself from the process. The nurse is in a unique position of being able to assist the patient to associate the sensations of labor with the process of childbirth and not the past abuse. The nurse can implement a number of care measures to help the patient view the childbirth experience in a positive manner. Which intervention would be key for the nurse to use while providing care? a. Telling the patient to relax and that it won't hurt much. b. Limiting the number of procedures that invade her body. c. Reassuring the patient that as the nurse you know what is best. d. Allowing unlimited care providers to be with the patient.

ANS: B The number of invasive procedures such as vaginal examinations, internal monitoring, and intravenous therapy should be limited as much as possible. The nurse should always avoid words and phrases that may result in the patient's recalling the phrases of her abuser (e.g., "Relax, this won't hurt" or "Just open your legs.") The woman's sense of control should be maintained at all times. The nurse should explain procedures at the patient's pace and wait for permission to proceed. Protecting the patient's environment by providing privacy and limiting the number of staff who observe the patient will help to make her feel safe.

Plantar creases should be evaluated within a few hours of birth because: a. The newborn has to be footprinted. b. As the skin dries, the creases will become more prominent. c. Heel sticks may be required. d. Creases will be less prominent after 24 hours.

ANS: B As the infants skin begins to dry, the creases will appear more prominent, and the infants gestation could be misinterpreted. Footprinting will not interfere with the creases. Heel sticks will not interfere with the creases. The creases will appear more prominent after 24 hours. PTS: 1 DIF: Cognitive Level: Knowledge REF: 567 OBJ: Nursing Process: Assessment MSC: Client Needs: Health Promotion and Maintenance

12. A 62-year-old woman has not been to the clinic for an annual examination for 5 years. The recent death of her husband reminded her that she should come for a visit. Her family doctor has retired, and she is going to see the women's health nurse practitioner for her visit. To facilitate a positive health care experience, the nurse should: A. remind the woman that she is long overdue for her examination and that she should come in annually. B. listen carefully and allow extra time for this woman's health history interview. C. reassure the woman that a nurse practitioner is just as good as her old doctor. D. encourage the woman to talk about the death of her husband and her fears about her own death.

ANS: B The nurse has an opportunity to use reflection and empathy while listening and to ensure open and caring communication. Scheduling a longer appointment time may be necessary because older women may have longer histories or may need to talk. A respectful and reassuring approach to caring for women older than age 50 can help ensure that they continue to seek health care. Reminding the woman about her overdue examination, reassuring the woman that she has a good practitioner, and encouraging conversation about the death of her husband and her own death are not the best approaches with women in this age-group.

A nurse in the emergency department is assessing a 5-year-old child with symptoms of pneumonia and a fever of 102 F. Which intervention can the nurse implement to promote a sense of control for the child? a. None, this is an emergency and the child should not participate in care. b. Allow the child to hold the digital thermometer while taking the childs blood pressure. c. Ask the child if it is OK to take a temperature in the ear. d. Have parents wait in the waiting room.

ANS: B The nurse should allow the child to hold the digital thermometer while taking the childs blood pressure. Unless an emergency is life threatening, children need to participate in their care to maintain a sense of control. Because emergency departments are frequently hectic, there is a tendency to rush through procedures to save time. However, the extra few minutes needed to allow children to participate may save many more minutes of useless resistance and uncooperativeness during subsequent procedures. The child may not give permission, if asked, for a procedure that is necessary to be performed. It is better to give choices such as, Which ear do you want me to do your temperature in? instead of, Can I take your temperature? Parents should remain with their child to help with decreasing the childs anxiety.

28. Nurses alert to signs of the onset of the second stage of labor can be certain that this stage has begun when: a. the woman has a sudden episode of vomiting. b. the nurse is unable to feel the cervix during a vaginal examination. c. bloody show increases. d. the woman involuntarily bears down.

ANS: B The only certain objective sign that the second stage has begun is the inability to feel the cervix because it is fully dilated and effaced. Vomiting, an increase in bloody show, and involuntary bearing down are only suggestions of second-stage labor.

The parents of a child hospitalized with sickle cell anemia tell the nurse that they are concerned about narcotic analgesics causing addiction. Which is appropriate for the nurse to explain about narcotic analgesics? a. Are often ordered but not usually needed b. Rarely cause addiction because they are medically indicated c. Are given as a last resort because of the threat of addiction d. Are used only if other measures, such as ice packs, are ineffective

ANS: B The pain of sickle cell anemia is best treated by a multidisciplinary approach. Mild to moderate pain can be controlled by ibuprofen and acetaminophen. When narcotics are indicated, they are titrated to effect and are given around the clock. Patient-controlled analgesia reinforces the patients role and responsibility in managing the pain and provides flexibility in dealing with pain. Few, if any, patients who receive opioids for severe pain become behaviorally addicted to the drug. Narcotics are often used because of the severe nature of the pain of vasoocclusive crisis. Ice is contraindicated because of its vasoconstrictive effects.

28. The maternity nurse understands that vascular volume increases 40% to 45% during pregnancy to: A. compensate for decreased renal plasma flow. B. provide adequate perfusion of the placenta. C. eliminate metabolic wastes of the mother. D. prevent maternal and fetal dehydration.

ANS: B The primary function of increased vascular volume is to transport oxygen and nutrients to the fetus via the placenta. Renal plasma flow increases during pregnancy. Assisting with pulling metabolic wastes from the fetus for maternal excretion is one purpose of the increased vascular volume.

21. To detect human immunodeficiency virus (HIV), most laboratory tests focus on the: A. virus. B. HIV antibodies. C. CD4 counts. D. CD8 counts.

ANS: B The screening tool used to detect HIV is the enzyme immunoassay, which tests for the presence of antibodies to the virus. CD4 counts are associated with the incidence of acquired immunodeficiency syndrome (AIDS) in HIV-infected individuals.

17. To reassure and educate pregnant patients about changes in the uterus, nurses should be aware that: A. lightening occurs near the end of the second trimester as the uterus rises into a different position. B. the woman's increased urinary frequency in the first trimester is the result of exaggerated uterine anteflexion caused by softening. C. Braxton Hicks contractions become more painful in the third trimester, particularly if the woman tries to exercise. D. the uterine souffle is the movement of the fetus.

ANS: B The softening of the lower uterine segment is called Hegar's sign. Lightening occurs in the last 2 weeks of pregnancy, when the fetus descends. Braxton Hicks contractions become more defined in the final trimester but are not painful. Walking or exercise usually causes them to stop. The uterine souffle is the sound made by blood in the uterine arteries; it can be heard with a fetal stethoscope.

12. A male patient asks the nurse why it is better to purchase condoms that are not lubricated with nonoxynol-9 (a common spermicide). The nurse's most appropriate response is: A. "The lubricant prevents vaginal irritation." B. "It has also been linked to an increase in the transmission of human immunodeficiency virus." C. "The additional lubrication improves sex." D. "Nonoxynol-9 improves penile sensitivity."

ANS: B The statement "Nonoxynol-9 does not provide protection against sexually transmitted infections, as originally thought; it has also been linked to an increase in the transmission of human immunodeficiency virus and can cause genital lesions" is true. Nonoxynol-9 may cause vaginal irritation, has no effect on the quality of sexual activity, and has no effect on penile sensitivity.

16. Which statement related to fetal positioning during labor is correct and important for the nurse to understand? a. Position is a measure of the degree of descent of the presenting part of the fetus through the birth canal. b. Birth is imminent when the presenting part is at +4 to +5 cm below the spine. c. The largest transverse diameter of the presenting part is the suboccipitobregmatic diameter. d. Engagement is the term used to describe the beginning of labor.

ANS: B The station of the presenting part should be noted at the beginning of labor to determine the rate of descent. Position is the relationship of the presenting part of the fetus to the four quadrants of the mother's pelvis; station is the measure of degree of descent. The largest diameter is usually the biparietal diameter. The suboccipitobregmatic diameter is the smallest, although one of the most critical. Engagement often occurs in the weeks just before labor in nulliparous women and before or during labor in multiparous women.

5. The nurse has received a report regarding a client in labor. The woman's last vaginal examination was recorded as 3 cm, 30%, and -2. What is the nurse's interpretation of this assessment? a. Cervix is effaced 3 cm and dilated 30%; the presenting part is 2 cm above the ischial spines. b. Cervix is dilated 3 cm and effaced 30%; the presenting part is 2 cm above the ischial spines. c. Cervix is effaced 3 cm and dilated 30%; the presenting part is 2 cm below the ischial spines. d. Cervix is dilated 3 cm and effaced 30%; the presenting part is 2 cm below the ischial spines.

ANS: B The sterile vaginal examination is recorded as centimeters of cervical dilation, percentage of cervical dilation, and the relationship of the presenting part to the ischial spines (either above or below). For this woman, the cervix is dilated 3 cm and effaced 30%, and the presenting part is 2 cm above the ischial spines. The first interpretation of this vaginal examination is incorrect; the cervix is dilated 3 cm and is 30% effaced. However, the presenting part is correct at 2 cm above the ischial spines. The remaining two interpretations of this vaginal examination are incorrect. Although the dilation and effacement are correct at 3 cm and 30%, the presenting part is actually 2 cm above the ischial spines.

30. To assist the woman after delivery of the infant, the nurse knows that the blood patch is used after spinal anesthesia to relieve: a. hypotension. b. headache. c. neonatal respiratory depression. d. loss of movement.

ANS: B The subarachnoid block may cause a postspinal headache resulting from loss of cerebrospinal fluid from the puncture in the dura. When blood is injected into the epidural space in the area of the dural puncture, it forms a seal over the hole to stop leaking of cerebrospinal fluid. Hypotension is prevented by increasing fluid volume before the procedure. Neonatal respiratory depression is not an expected outcome with spinal anesthesia. Loss of movement is an expected outcome of spinal anesthesia.

25. With regard to the use of intrauterine devices (IUDs), nurses should be aware that: A. return to fertility can take several weeks after the device is removed. B. IUDs containing copper can provide an emergency contraception option if inserted within a few days of unprotected intercourse. C. IUDs offer the same protection against sexually transmitted infections (STIs) as the diaphragm. D. consent forms are not needed for IUD insertion.

ANS: B The woman has up to 5 days to insert the IUD after unprotected sex. Return to fertility is immediate after removal of the IUD. IUDs offer no protection for STIs. A consent form is required for insertion, as is a negative pregnancy test.

8. If an opioid antagonist is administered to a laboring woman, she should be told that: a. her pain will decrease. b. her pain will return. c. she will feel less anxious. d. she will no longer feel the urge to push.

ANS: B The woman should be told that the pain that was relieved by the opioid analgesic will return with administration of the opioid antagonist. Opioid antagonists, such as Narcan, promptly reverse the central nervous system (CNS) depressant effects of opioids. In addition, the antagonist counters the effect of the stress-induced levels of endorphins. An opioid antagonist is especially valuable if labor is more rapid than expected and birth is anticipated when the opioid is at its peak effect.

29. Which statement by the patient indicates that she understands breast self-examination? A. "I will examine both breasts in two different positions." B. "I will perform breast self-examination 1 week after my menstrual period starts." C. "I will examine the outer upper area of the breast only." D. "I will use the palm of the hand to perform the examination."

ANS: B The woman should examine her breasts when hormonal influences are at their lowest level. The patient should be instructed to use four positions: standing with arms at her sides, standing with arms raised above her head, standing with hands pressed against hips, and lying down. The entire breast needs to be examined, including the outer upper area. The patient should use the sensitive pads of the middle three fingers.

32. The parent of a toddler calls the nurse, asking about croup. What is a distinguishing manifestation of spasmodic croup? a. Wheezing is heard audibly. c. It is bacterial in nature. b. It has a harsh, barky cough. d. The child has a high fever

ANS: B Spasmodic croup is viral in origin, is usually preceded by several days of symptoms of upper respiratory tract infection, and often begins at night. It is marked by a harsh, metallic, barky cough; sore throat; inspiratory stridor; and hoarseness. Wheezing is not a distinguishing manifestation of croup. It can accompany conditions such as asthma or bronchiolitis. A high fever is not usually present.

12. The nurse, caring for a patient whose labor is being augmented with oxytocin, recognizes that the oxytocin should be discontinued immediately if there is evidence of: a. uterine contractions occurring every 8 to 10 minutes. b. a fetal heart rate (FHR) of 180 with absence of variability. c. the patient's needing to void. d. rupture of the patient's amniotic membranes.

ANS: B This FHR is nonreassuring. The oxytocin should be discontinued immediately, and the physician should be notified. The oxytocin should be discontinued if uterine hyperstimulation occurs. Uterine contractions that are occurring every 8 to 10 minutes do not qualify as hyperstimulation. The patient's needing to void is not an indication to discontinue the oxytocin induction immediately or to call the physician. Unless a change occurs in the FHR pattern that is nonreassuring or the patient experiences uterine hyperstimulation, the oxytocin does not need to be discontinued. The physician should be notified that the patient's membranes have ruptured.

5. A 26-year-old primigravida has come to the clinic for her regular prenatal visit at 12 weeks. She appears thin and somewhat nervous. She reports that she eats a well-balanced diet, although her weight is 5 lbs less than it was at her last visit. The results of laboratory studies confirm that she has a hyperthyroid condition. Based on the available data, the nurse formulates a plan of care. What nursing diagnosis is most appropriate for the woman at this time? A. Deficient fluid volume B. Imbalanced nutrition: less than body requirements C. Imbalanced nutrition: more than body requirements D. Disturbed sleep pattern

ANS: B This patient's clinical cues include weight loss, which would support the nursing diagnosis of Imbalanced nutrition: less than body requirements. No clinical signs or symptoms support the nursing diagnosis of Deficient fluid volume. This patient reports weight loss, not weight gain. Imbalanced nutrition: more than body requirements is not an appropriate nursing diagnosis. Although the patient reports nervousness based on the patient's other clinical symptoms the most appropriate nursing diagnosis would be Imbalanced nutrition: less than body requirements.

The nurse is admitting a school-age child with suspected Guillain-Barr syndrome (GBS). Which is a priority in the care for this child? a. Monitoring intake and output b. Assessing respiratory efforts c. Placing on a telemetry monitor d. Obtaining laboratory studies

ANS: B Treatment of GBS is primarily supportive. In the acute phase, patients are hospitalized because respiratory and pharyngeal involvement may require assisted ventilation, sometimes with a temporary tracheotomy. Treatment modalities include aggressive ventilatory support in the event of respiratory compromise, intravenous (IV) administration of immunoglobulin (IVIG), and sometimes steroids; plasmapheresis and immunosuppressive drugs may also be used. Intake and output, telemetry monitoring and obtaining laboratory studies may be part of the plan of care but are not the priority.

25. Which statement about multifetal pregnancy is inaccurate? A. The expectant mother often develops anemia because the fetuses have a greater demand for iron. B. Twin pregnancies come to term with the same frequency as single pregnancies. C. The mother should be counseled to increase her nutritional intake and gain more weight. D. Backache and varicose veins often are more pronounced.

ANS: B Twin pregnancies often end in prematurity. Serious efforts should be made to bring the pregnancy to term. A woman with a multifetal pregnancy often develops anemia, suffers more or worse backache, and needs to gain more weight. Counseling is needed to help her adjust to these conditions.

7. In terms of the incidence and classification of diabetes, maternity nurses should know that: A. type 1 diabetes is most common. B. type 2 diabetes often goes undiagnosed. C. gestational diabetes mellitus (GDM) means that the woman will be receiving insulin treatment until 6 weeks after birth. D. type 1 diabetes may become type 2 during pregnancy.

ANS: B Type 2 diabetes often goes undiagnosed because hyperglycemia develops gradually and often is not severe. Type 2 diabetes, sometimes called adult onset diabetes, is the most common. GDM refers to any degree of glucose intolerance first recognized during pregnancy. Insulin may or may not be needed. People do not go back and forth between type 1 and 2 diabetes.

4. The nurse providing care for the laboring woman realizes that variable fetal heart rate (FHR) decelerations are caused by: a. altered fetal cerebral blood flow. b. umbilical cord compression. c. uteroplacental insufficiency. d. fetal hypoxemia.

ANS: B Variable decelerations can occur any time during the uterine contracting phase and are caused by compression of the umbilical cord. Altered fetal cerebral blood flow would result in early decelerations in the FHR. Uteroplacental insufficiency would result in late decelerations in the FHR. Fetal hypoxemia would result in tachycardia initially and then bradycardia if hypoxia continues.

29. Physiologic anemia often occurs during pregnancy as a result of: A. inadequate intake of iron. B. dilution of hemoglobin concentration. C. the fetus establishing iron stores. D. decreased production of erythrocytes.

ANS: B When blood volume expansion is more pronounced and occurs earlier than the increase in red blood cells, the woman has physiologic anemia, which is the result of dilution of hemoglobin concentration rather than inadequate hemoglobin. Inadequate intake of iron may lead to true anemia. There is an increased production of erythrocytes during pregnancy.

18. Which nursing intervention is necessary before a second-trimester transabdominal ultrasound? A. Place the woman NPO for 12 hours. B. Instruct the woman not void until after the test. C. Administer an enema. D. Perform an abdominal preparation.

ANS: B When the uterus is still in the pelvis, visualization may be difficult. It is necessary to perform the test when the woman has a full bladder, which provides a "window" through which the uterus and its contents can be viewed. The woman needs a full bladder to elevate the uterus; therefore being NPO is not appropriate. Neither an enema nor an abdominal preparation is necessary for this procedure.

33. The nurse is completing an admission assessment on a 3-year-old child. The child's Humpty Dumpty score is 15. Which action by the nurse is the most appropriate? A. Allow the child access to the play room. B. Classify the child as at high risk for falls. C. Place the child on seizure precautions. D. Put the child in isolation precautions.

ANS: B A Humpty Dumpty score of 12 or above indicates a high risk for falls. The child has been classified as at high risk for falls, and nursing care should be implemented to prevent them. Access to the play room can be accomplished with almost any child. Seizures and isolation actions are not related.

Necrotizing enterocolitis (NEC) is an acute inflammatory disease of the gastrointestinal mucosa that can progress to perforation of the bowel. Approximately 2% to 5% of premature infants succumb to this fatal disease. Care is supportive; however, known interventions may decrease the risk of NEC. To develop an optimal plan of care for this infant, the nurse must understand which intervention has the greatest effect on lowering the risk of NEC: a. Early enteral feedings b. Breastfeeding c. Exchange transfusion d. Prophylactic probiotics

ANS: B A decrease in the incidence of NEC is directly correlated with exclusive breastfeeding. Breast milk enhances maturation of the gastrointestinal tract and contains immune factors that contribute to a lower incidence or severity of NEC, Crohns disease, and celiac illness. The neonatal intensive care unit nurse can be very supportive of the mother in terms of providing her with equipment to pump breast milk, ensuring privacy, and encouraging skin-to-skin contact with the infant. Early enteral feedings of formula or hyperosmolar feedings are a risk factor known to contribute to the development of NEC. The mother should be encouraged to pump or feed breast milk exclusively. Exchange transfusion may be necessary; however, it is a known risk factor for the development of NEC. Although still early, a study in 2005 found that the introduction of prophylactic probiotics appeared to enhance the normal flora of the bowel and therefore decrease the severity of NEC when it did occur. This treatment modality is not as widespread as encouraging breastfeeding; however, it is another strategy that the care providers of these extremely fragile infants may have at their disposal. PTS: 1 DIF: Cognitive Level: Application REF: 693 OBJ: Nursing Process: Planning MSC: Client Needs: Physiologic Integrity

The nurse should be aware that a pessary would be most effective in the treatment of what disorder? a. Cystocele b. Uterine prolapse c. Rectocele d. Stress urinary incontinence

ANS: B A fitted pessary may be inserted into the vagina to support the uterus and hold it in the correct position. A pessary is not used for a cystocele, a rectocele, or stress urinary incontinence. PTS: 1 DIF: Cognitive Level: Knowledge REF: 544 OBJ: Nursing Process: Diagnosis MSC: Client Needs: Health Promotion and Maintenance

A school-age child has sustained a head injury and multiple fractures after being thrown from a horse. The child's level of consciousness is variable. The parents tell the nurse that they think their child is in pain because of periodic crying and restlessness. The most appropriate nursing action is to: a. discuss with parents the child's previous experiences with pain. b. discuss with practitioner what analgesia can be safely administered. c. explain that analgesia is contraindicated with a head injury. d. explain that analgesia is unnecessary when child is not fully awake and alert.

ANS: B A key nursing role is to provide sedation and analgesia for the child. Consultation with the appropriate practitioner is necessary to avoid conflict between the necessity to monitor the child's neurologic status and the promotion of comfort and relief of anxiety. Information on the child's previous experiences with pain should be obtained as part of the assessment, but because of the severity of injury, analgesia should be provided as soon as possible. Analgesia can be safely used in individuals who have sustained head injuries and can decrease anxiety and resultant increased ICP.

To prevent the abduction of newborns from the hospital, the nurse should: a. Instruct the mother not to give her infant to anyone except the one nurse assigned to her that day. b. Apply an electronic and identification bracelet to mother and infant. c. Carry the infant when transporting him or her in the halls. d. Restrict the amount of time infants are out of the nursery.

ANS: B A measure taken by many facilities is to band both the mother and the baby with matching identification bracelets and band the infant with an electronic device that will alarm if the infant is removed from the maternity unit. It is impossible for one nurse to be on call for one mother and baby for the entire shift, so parents need to be able to identify the nurses who are working on the unit. Infants should always be transported in their bassinette, for both safety and security reasons. All maternity unit nursing staff should have unique identification bracelets in comparison with the rest of the hospital. Infants should remain with their parents and spend as little time in the nursery as possible. PTS: 1 DIF: Cognitive Level: Comprehension REF: 613 OBJ: Nursing Process: Implementation MSC: Client Needs: Safe and Effective Care Environment

Which statement best describes a subdural hematoma? a. Bleeding occurs between the dura and the skull. b. Bleeding occurs between the dura and the cerebrum. c. Bleeding is generally arterial, and brain compression occurs rapidly. d. The hematoma commonly occurs in the parietotemporal region.

ANS: B A subdural hematoma is bleeding that occurs between the dura and the cerebrum as a result of a rupture of cortical veins that bridge the subdural space. An epidural hemorrhage occurs between the dura and the skull, is usually arterial with rapid brain concussion, and occurs most often in the parietotemporal region.

The nurse is conducting a staff in-service on casts. Which is an advantage to using a fiberglass cast instead of a plaster of Paris cast? a. Cheaper b. Dries rapidly c. Molds closely to body parts d. Smooth exterior

ANS: B A synthetic casting material dries in 5 to 30 minutes as compared with a plaster cast, which takes 10 to 72 hours to dry. Synthetic casts are more expensive and have a rough exterior, which may scratch surfaces. Plaster casts mold closer to body parts.

When caring for a newly delivered woman, the nurse is aware that the best measure to prevent abdominal distention after a cesarean birth is: a. Rectal suppositories. b. Early and frequent ambulation. c. Tightening and relaxing abdominal muscles. d. Carbonated beverages.

ANS: B Activity will aid the movement of accumulated gas in the gastrointestinal tract. Rectal suppositories can be helpful after distention occurs; however, they do not prevent it. Ambulation is the best prevention. Carbonated beverages may increase distention. PTS: 1 DIF: Cognitive Level: Knowledge REF: 500 OBJ: Nursing Process: Planning MSC: Client Needs: Physiologic Integrity

A new father is ready to take his wife and newborn son home. He proudly tells the nurse who is discharging them that within the next week he plans to start feeding the infant cereal between breastfeeding sessions. The nurse can explain to him that beginning solid foods before 4 to 6 months may: a. Decrease the infants intake of sufficient calories. b. Lead to early cessation of breastfeeding. c. Help the infant sleep through the night. d. Limit the infants growth.

ANS: B Introduction of solid foods before the infant is 4 to 6 months of age may result in overfeeding and decreased intake of breast milk. It is not true that feeding of solids helps infants sleep through the night. The proper balance of carbohydrate, protein, and fat for an infant to grow properly is in the breast milk or formula. PTS: 1 DIF: Cognitive Level: Application REF: 653 OBJ: Nursing Process: Evaluation MSC: Client Needs: Health Promotion and Maintenance

With regard to the postpartum uterus, nurses should be aware that: a. At the end of the third stage of labor it weighs approximately 500 g. b. After 2 weeks postpartum it should not be palpable abdominally. c. After 2 weeks postpartum it weighs 100 g. d. It returns to its original (prepregnancy) size by 6 weeks postpartum.

ANS: B After 2 weeks postpartum, the uterus should not be palpable abdominally; however, it has not yet returned to its original size. At the end of the third stage of labor, the uterus weighs approximately 1000 g. It takes 6 full weeks for the uterus to return to its original size. After 2 weeks postpartum the uterus weighs about 350 g, not its original size. The normal self-destruction of excess hypertrophied tissue accounts for the slight increase in uterine size after each pregnancy. PTS: 1 DIF: Cognitive Level: Comprehension REF: 483 OBJ: Nursing Process: Assessment MSC: Client Needs: Health Promotion and Maintenance

20. A child is being discharged after surgical resection of a retinoblastoma with enucleation. Which discharge instruction is most important based on the diagnosis? A. Encouraging healthy eating B. Irrigation of the surgical site C. Monitoring the child's temperature D. Pain assessment and control

ANS: B After enucleation (removal of the eye), the eye socket must be irrigated and a thin layer of antibiotic ointment applied. The other options are valid for all postoperative pediatric patients.

Which woman is most likely to experience strong afterpains? a. A woman who experienced oligohydramnios b. A woman who is a gravida 4, para 4-0-0-4 c. A woman who is bottle-feeding her infant d. A woman whose infant weighed 5 pounds, 3 ounces

ANS: B Afterpains are more common in multiparous women. Afterpains are more noticeable with births in which the uterus was greatly distended, as in a woman who experienced polyhydramnios or a woman who delivered a large infant. Breastfeeding may cause afterpains to intensify. PTS: 1 DIF: Cognitive Level: Comprehension REF: 484 OBJ: Nursing Process: Assessment MSC: Client Needs: Health Promotion and Maintenance

The perinatal nurse assisting with establishing lactation is aware that acute mastitis can be minimized by: a. Washing the nipples and breasts with mild soap and water once a day. b. Using proper breastfeeding techniques. c. Wearing a nipple shield for the first few days of breastfeeding. d. Wearing a supportive bra 24 hours a day.

ANS: B Almost all instances of acute mastitis can be avoided by proper breastfeeding technique to prevent cracked nipples. Washing the nipples and breasts daily is no longer indicated. In fact, this can cause tissue dryness and irritation, which can lead to tissue breakdown and infection. Wearing a nipple shield does not prevent mastitis. Wearing a supportive bra 24 hours a day may contribute to mastitis, especially if an underwire bra is worn, because it may put pressure on the upper, outer area of the breast, thus contributing to blocked ducts and mastitis. PTS: 1 DIF: Cognitive Level: Comprehension REF: 551 OBJ: Nursing Process: Planning MSC: Client Needs: Health Promotion and Maintenance

Which instructions should be included in the discharge teaching plan to assist the patient in recognizing early signs of complications? a. Palpate the fundus daily to ensure that it is soft. b. Notify the physician of any increase in the amount of lochia or a return to bright red bleeding. c. Report any decrease in the amount of brownish red lochia. d. The passage of clots as large as an orange can be expected.

ANS: B An increase in lochia or a return to bright red bleeding after the lochia has become pink indicates a complication. The fundus should stay firm. The lochia should decrease in amount over time. Large clots after discharge are a sign of complications and should be reported. PTS: 1 DIF: Cognitive Level: Application REF: 532 OBJ: Nursing Process: Implementation MSC: Client Needs: Health Promotion and Maintenance

A woman gave birth 48 hours ago to a healthy infant girl. She has decided to bottle-feed. During your assessment you notice that both of her breasts are swollen, warm, and tender on palpation. The woman should be advised that this condition can best be treated by: a. Running warm water on her breasts during a shower. b. Applying ice to the breasts for comfort. c. Expressing small amounts of milk from the breasts to relieve pressure. d. Wearing a loose-fitting bra to prevent nipple irritation.

ANS: B Applying ice to the breasts for comfort is appropriate for treating engorgement in a mother who is bottlefeeding. This woman is experiencing engorgement, which can be treated by using ice packs (because she is not breastfeeding) and cabbage leaves. A bottle-feeding mother should avoid any breast stimulation, including pumping or expressing milk. A bottle-feeding mother should wear a well-fitted support bra or breast binder continuously for at least the first 72 hours after giving birth. A loose-fitting bra will not aid lactation suppression. Furthermore, the shifting of the bra against the breasts may stimulate the nipples and thereby stimulate lactation. PTS: 1 DIF: Cognitive Level: Application REF: 499 OBJ: Nursing Process: Implementation MSC: Client Needs: Health Promotion and Maintenance

17. The nurse is caring for a child with carbon monoxide (CO) poisoning associated with smoke inhalation. What is essential in this childs care? a. Monitor pulse oximetry. b. Monitor arterial blood gases. c. Administer oxygen if respiratory distress develops. d. Administer oxygen if childs lips become bright, cherry red

ANS: B Arterial blood gases and COHb levels are the best way to monitor CO poisoning. PaO2 monitored with pulse oximetry may be normal in the case of CO poisoning. Oxygen at 100% should be given as quickly as possible, not only if respiratory distress or other symptoms develop.

To promote bonding and attachment immediately after delivery, the most important nursing intervention is to: a. Allow the mother quiet time with her infant. b. Assist the mother in assuming an en face position with her newborn. c. Teach the mother about the concepts of bonding and attachment. d. Assist the mother in feeding her baby.

ANS: B Assisting the mother in assuming an en face position with her newborn will support the bonding process. The mother should be given as much privacy as possible; however, nursing assessments must still be continued during this critical time. The mother has just delivered and is more focused on the infant; she will not be receptive to teaching at this time. This is a good time to initiate breastfeeding; however, the mother first needs time to explore the new infant and begin the bonding process. PTS: 1 DIF: Cognitive Level: Application REF: 519 OBJ: Nursing Process: Implementation MSC: Client Needs: Health Promotion and Maintenance

A man calls the nurses station and states that his wife, who delivered 2 days ago, is happy one minute and crying the next. The man says, She was never like this before the baby was born. The nurses initial response could be to: a. Tell him to ignore the mood swings, as they will go away. b. Reassure him that this behavior is normal. c. Advise him to get immediate psychological help for her. d. Instruct him in the signs, symptoms, and duration of postpartum blues.

ANS: B Before providing further instructions, inform family members of the fact that postpartum blues are a normal process. Telling her partner to ignore the mood swings does not encourage further communication and may belittle the husbands concerns. Postpartum blues are usually short-lived; no medical intervention is needed. Client teaching is important; however, the new fathers anxieties need to be allayed before he will be receptive to teaching. PTS: 1 DIF: Cognitive Level: Application REF: 517 OBJ: Nursing Process: Implementation MSC: Client Needs: Psychosocial Integrity

The goal of treatment of the infant with phenylketonuria (PKU) is to: a. Cure mental retardation. b. Prevent central nervous system (CNS) damage, which leads to mental retardation. c. Prevent gastrointestinal symptoms. d. Cure the urinary tract infection.

ANS: B CNS damage can occur as a result of toxic levels of phenylalanine. No known cure exists for mental retardation. Digestive problems are a clinical manifestation of PKU. PKU does not involve any urinary problems. PTS: 1 DIF: Cognitive Level: Comprehension REF: 711 OBJ: Nursing Process: Planning MSC: Client Needs: Physiologic Integrity

Infants in whom cephalhematomas develop are at increased risk for: a. Infection. b. Jaundice. c. Caput succedaneum. d. Erythema toxicum.

ANS: B Cephalhematomas are characterized by bleeding between the bone and its covering, the periosteum. Because of the breakdown of the red blood cells within a hematoma, the infants are at greater risk for jaundice. Cephalhematomas do not increase the risk for infections. Caput is an edematous area on the head from pressure against the cervix. Erythema toxicum is a benign rash of unknown cause that consists of blotchy red areas. PTS: 1 DIF: Cognitive Level: Application REF: 564 OBJ: Nursing Process: Assessment MSC: Client Needs: Physiologic Integrity

The transition period between intrauterine and extrauterine existence for the newborn: a. Consists of four phases, two reactive and two of decreased responses. b. Lasts from birth to day 28 of life. c. Applies to full-term births only. d. Varies by socioeconomic status and the mothers age.

ANS: B Changes begin right after birth; the cutoff time when the transition is considered over (although the baby keeps changing) is 28 days. The transition period has three phases: first reactivity, decreased response, and second reactivity. All newborns experience this transition regardless of age or type of birth. Although stress can cause variation in the phases, the mothers age and wealth do not disturb the pattern. PTS: 1 DIF: Cognitive Level: Comprehension REF: 556 OBJ: Nursing Process: Evaluation MSC: Client Needs: Health Promotion and Maintenance

1. Which statement best describes why children have fewer respiratory tract infections as they grow older? a. The amount of lymphoid tissue decreases. b. Repeated exposure to organisms causes increased immunity. c. Viral organisms are less prevalent in the population. d. Secondary infections rarely occur after viral illnesses.

ANS: B Children have increased immunity after exposure to a virus. The amount of lymphoid tissue increases as children grow older. Viral organisms are not less prevalent, but older children have the ability to resist invading organisms. Secondary infections after viral illnesses include Mycoplasma pneumoniae and groups A and B streptococcal infections

The process in which bilirubin is changed from a fat-soluble product to a water-soluble product is known as: a. Enterohepatic circuit. b. Conjugation of bilirubin. c. Unconjugation of bilirubin. d. Albumin binding.

ANS: B Conjugation of bilirubin is the process of changing the bilirubin from a fat-soluble to a water-soluble product. This is the route by which part of the bile produced by the liver enters the intestine, is reabsorbed by the liver, and then is recycled into the intestine. Unconjugated bilirubin is fat soluble. Albumin binding is to attach something to a protein molecule. PTS: 1 DIF: Cognitive Level: Knowledge REF: 564 OBJ: Nursing Process: Assessment MSC: Client Needs: Physiologic Integrity

A mother in late middle age who is certain she is not pregnant tells the nurse during an office visit that she has urinary problems and sensations of bearing down and of something in her vagina. The nurse would realize that the client most likely is suffering from: a. Pelvic relaxation. b. Cystoceles and/or rectoceles. c. Uterine displacement. d. Genital fistulas.

ANS: B Cystoceles are protrusions of the bladder downward into the vagina; rectoceles are herniations of the anterior rectal wall through a relaxed or ruptured vaginal fascia. Both can manifest as a bearing down sensation with urinary dysfunction. They occur more often in older women who have borne children. PTS: 1 DIF: Cognitive Level: Analysis REF: 543 OBJ: Nursing Process: Diagnosis MSC: Client Needs: Health Promotion and Maintenance

A woman gave birth to a 7-pound, 3-ounce infant boy 2 hours ago. The nurse determines that the womans bladder is distended because her fundus is now 3 cm above the umbilicus and to the right of the midline. In the immediate postpartum period, the most serious consequence likely to occur from bladder distention is: a. Urinary tract infection. b. Excessive uterine bleeding. c. A ruptured bladder. d. Bladder wall atony.

ANS: B Excessive bleeding can occur immediately after birth if the bladder becomes distended because it pushes the uterus up and to the side and prevents it from contracting firmly. A urinary tract infection may result from overdistention of the bladder, but it is not the most serious consequence. A ruptured bladder may result from a severely overdistended bladder. However, vaginal bleeding most likely would occur before the bladder reaches this level of overdistention. Bladder distention may result from bladder wall atony. The most serious concern associated with bladder distention is excessive uterine bleeding. PTS: 1 DIF: Cognitive Level: Comprehension REF: 486 OBJ: Nursing Process: Diagnosis, Planning MSC: Client Needs: Health Promotion and Maintenance

A woman delivered a 9-lb, 10-oz baby 1 hour ago. When you arrive to perform her 15-minute assessment, she tells you that she feels all wet underneath. You discover that both pads are completely saturated and that she is lying in a 6-inch-diameter puddle of blood. What is your first action? a. Call for help. b. Assess the fundus for firmness. c. Take her blood pressure. d. Check the perineum for lacerations.

ANS: B Firmness of the uterus is necessary to control bleeding from the placental site. The nurse should first assess for firmness and massage the fundus as indicated. Assessing blood pressure is an important assessment with a bleeding patient; however, the top priority is to control the bleeding. If bleeding continues in the presence of a firm fundus, lacerations may be the cause. PTS: 1 DIF: Cognitive Level: Application REF: 530 OBJ: Nursing Process: Implementation MSC: Client Needs: Physiologic Integrity

The nurse is preparing a school-age child for computed tomography (CT scan) to assess cerebral function. The nurse should include which statement in preparing the child? a. "Pain medication will be given." b. "The scan will not hurt." c. "You will be able to move once the equipment is in place." d. "Unfortunately, no one can remain in the room with you during the test."

ANS: B For CT scans, the child must be immobilized. It is important to emphasize to the child that at no time is the procedure painful. Pain medication is not required; however, sedation is sometimes necessary. Someone is able to remain with the child during the procedure.

Many first-time parents do not plan on their parents help immediately after the newborn arrives. What statement by the nurse is the most appropriate when counseling new parents about the involvement of grandparents? a. You should tell your parents to leave you alone. b. Grandparents can help you with parenting skills and also help preserve family traditions. c. Grandparent involvement can be very disruptive to the family. d. They are getting old. You should let them be involved while they can.

ANS: B Grandparents can help you with parenting skills and also help preserve family traditions is the most appropriate response. Intergenerational help may be perceived as interference; however, a statement of this sort is not therapeutic to the adaptation of the family. Not only is Grandparent involvement can be very disruptive to the family invalid, it also is not an appropriate nursing response. Regardless of age, grandparents can help with parenting skills and preserve family traditions. Talking about the age of the grandparents is not the most appropriate statement, and it does not demonstrate sensitivity on the part of the nurse. PTS: 1 DIF: Cognitive Level: Analysis REF: 526 OBJ: Nursing Process: Planning MSC: Client Needs: Psychosocial Integrity

To care adequately for infants at risk for neonatal bacterial infection, nurses should be aware that: a. Congenital infection progresses more slowly than does nosocomial infection. b. Nosocomial infection can be prevented by effective handwashing; early-onset infections cannot. c. Infections occur with about the same frequency in boy and girl infants, although female mortality is higher. d. The clinical sign of a rapid, high fever makes infection easier to diagnose.

ANS: B Handwashing is an effective preventive measure for late-onset (nosocomial) infections because these infections come from the environment around the infant. Early-onset, or congenital, infections are caused by the normal flora at the maternal vaginal tract and progress more rapidly than do nosocomial (late-onset) infections. Infection occurs about twice as often in boys and results in higher mortality. Clinical signs of neonatal infection are nonspecific and are similar to those of noninfectious problems, thus making diagnosis difficult. PTS: 1 DIF: Cognitive Level: Comprehension REF: 672 OBJ: Nursing Process: Implementation MSC: Client Needs: Safe and Effective Care Environment

The most common cause of pathologic hyperbilirubinemia is: a. Hepatic disease. b. Hemolytic disorders in the newborn. c. Postmaturity. d. Congenital heart defect.

ANS: B Hemolytic disorders in the newborn are the most common cause of pathologic jaundice. Hepatic damage may be a cause of pathologic hyperbilirubinemia, but it is not the most common cause. Prematurity would be a potential cause of pathologic hyperbilirubinemia in neonates, but it is not the most common cause. Congenital heart defect is not a common cause of pathologic hyperbilirubinemia in neonates. PTS: 1 DIF: Cognitive Level: Knowledge REF: 679 OBJ: Nursing Process: Diagnosis MSC: Client Needs: Physiologic Integrity

40. The nurse is assessing a child with acute epiglottitis. Examining the childs throat by using a tongue depressor might precipitate which symptom or condition? a. Inspiratory stridor c. Sore throat b. Complete obstruction d. Respiratory tract infection

ANS: B If a child has acute epiglottitis, examination of the throat may cause complete obstruction and should be performed only when immediate intubation can take place. Stridor is aggravated when a child with epiglottitis is supine. Sore throat and pain on swallowing are early signs of epiglottitis. Epiglottitis is caused by Haemophilus influenzae in the respiratory tract.

21. A child has been cured of a retinoblastoma. When the parents ask how long monitoring for bone-related complications of radiation therapy should continue, which is the most appropriate response by the nurse? A. "After 5 years, you can stop worrying about this." B. "Cancers of the bone can occur up to 15 years later." C. "Probably all complications will occur within 3 years." D. "Radiation complications do not occur in bones."

ANS: B Osteosarcoma can occur as a consequence of radiation therapy up to 15 years later.

Which position should the nurse place a 10-year-old child after a large tumor was removed through a supratentorial craniotomy? a. On the inoperative side with the bed flat b. On the inoperative side with the head of bed elevated 20 to 30 degrees c. On the operative side with the bed flat and pillows behind the head d. On the operative side with the head of bed elevated 45 degrees

ANS: B If a large tumor was removed, the child is not placed on the operative side because the brain may suddenly shift to that cavity, causing trauma to the blood vessels, linings, and the brain itself. The child with an infratentorial procedure is usually positioned on either side with the bed flat. When a supratentorial craniotomy is performed, the head of bed is elevated 20 to 30 degrees with the child on either side or on the back. In a supratentorial craniotomy, the head elevation facilitates CSF drainage and decreases excessive blood flow to the brain to prevent hemorrhage. Pillows should be placed against the child's back, not head, to maintain the desired position.

To prevent nipple trauma, the nurse should instruct the new mother to: a. Limit the feeding time to less than 5 minutes. b. Position the infant so the nipple is far back in the mouth. c. Assess the nipples before each feeding. d. Wash the nipples daily with mild soap and water.

ANS: B If the infants mouth does not cover as much of the areola as possible, the pressure during sucking will be applied to the nipple, thus causing trauma to the area. Stimulating the breast for less than 5 minutes will not produce the extra milk the infant may need. This will also limit access to the higher-fat hindmilk. Assessing the nipples for trauma is important; however, this action alone will not prevent sore nipples. Soap can be drying to the nipples and should be avoided during breastfeeding. PTS: 1 DIF: Cognitive Level: Application REF: 643 OBJ: Nursing Process: Implementation MSC: Client Needs: Physiologic Integrity

The nurse is caring for a preschool child immobilized by a spica cast. Which effect on metabolism should the nurse monitor on this child related to the immobilized status? a. Hypocalcemia b. Decreased metabolic rate c. Positive nitrogen balance d. Increased production of stress hormones

ANS: B Immobilization causes a decreased metabolic rate with slowing of all systems and a decreased food intake. Immobilization leads to hypercalcemia and causes a negative nitrogen balance secondary to muscle atrophy. A decreased production of stress hormones occurs with decreased physical and emotional coping capacity.

29. Cardiopulmonary resuscitation is begun on a toddler. Which pulse is usually palpated because it is the most central and accessible? a. Radial c. Femoral b. Carotid d. Brachia

ANS: B In a toddler, the carotid pulse is palpated. The radial pulse is not considered a central pulse. The femoral pulse is not the most central and accessible. The brachial pulse is felt in infants younger than 1 year.

1. Which statement accurately describes physical development of a child during the school-age years? a. The child's weight almost triples. b. A child grows an average of 2 inches/year. c. Few physical differences are apparent among children at the end of middle childhood. d. Fat gradually increases, which contributes to the child's heavier appearance.

ANS: B In middle childhood, growth in height and weight occur at a slower pace. Between the ages of 6 and 12 years, children grow 2 inches/year. In middle childhood, children's weight will almost double; they gain 3 kg/year. At the end of middle childhood, girls grow taller and gain more weight than boys. Children take on a slimmer look with longer legs in middle childhood.

The parents of a newborn ask the nurse how much the newborn can see. The parents specifically want to know what type of visual stimuli they should provide for their newborn. The nurse responds to the parents by telling them: a. Infants can see very little until about 3 months of age. b. Infants can track their parents eyes and distinguish patterns; they prefer complex patterns. c. The infants eyes must be protected. Infants enjoy looking at brightly colored stripes. d. Its important to shield the newborns eyes. Overhead lights help them see better.

ANS: B Infants can track their parents eyes and distinguish patterns; they prefer complex patterns is an accurate statement. Development of the visual system continues for the first 6 months of life. Visual acuity is difficult to determine, but the clearest visual distance for the newborn appears to be 19 cm. Infants prefer to look at complex patterns, regardless of the color. Infants prefer low illumination and withdraw from bright light. PTS: 1 DIF: Cognitive Level: Application REF: 577 OBJ: Nursing Process: Planning MSC: Client Needs: Health Promotion and Maintenance

Other early sensual contacts between infant and mother involve sound and smell. Nurses should be aware that, despite what folk wisdom may say: a. High-pitched voices irritate newborns. b. Infants can learn to distinguish their mothers voice from others soon after birth. c. All babies in the hospital smell alike. d. A mothers breast milk has no distinctive odor.

ANS: B Infants know the sound of their mothers voice early. Infants respond positively to high-pitched voices. Each infant has a unique odor. Infants quickly learn to distinguish the odor of their mothers breast milk. PTS: 1 DIF: Cognitive Level: Knowledge REF: 515 OBJ: Nursing Process: Planning MSC: Client Needs: Health Promotion and Maintenance

The breastfeeding mother should be taught a safe method to remove the breast from the babys mouth. Which suggestion by the nurse is most appropriate? a. Slowly remove the breast from the babys mouth when the infant has fallen asleep and the jaws are relaxed. b. Break the suction by inserting your finger into the corner of the infants mouth. c. A popping sound occurs when the breast is correctly removed from the infants mouth. d. Elicit the Moro reflex to wake the baby and remove the breast when the baby cries.

ANS: B Inserting a finger into the corner of the babys mouth between the gums to break the suction avoids trauma to the breast. The infant who is sleeping may lose grasp on the nipple and areola, resulting in chewing on the nipple that makes it sore. A popping sound indicates improper removal of the breast from the babys mouth and may cause cracks or fissures in the breast. Most mothers prefer the infant to continue to sleep after the feeding. Gentle wake-up techniques are recommended. PTS: 1 DIF: Cognitive Level: Application REF: 654 OBJ: Nursing Process: Implementation MSC: Client Needs: Health Promotion and Maintenance

1. A nursing faculty member explains to the class that which item is the most important for tumor cell growth? A. Age of transforming cells B. Programmed cell death C. Proximity to a capillary D. Rapidity of cell growth

ANS: C All cells, including tumor cells, need a consistent supply of oxygen and nutrients, delivered via the capillaries. Neoplastic cells must be in close enough proximity to a capillary to provide these required elements. The other factors do not have such an important role, if any, in neoplastic growth.

The nurses initial action when caring for an infant with a slightly decreased temperature is to: a. Notify the physician immediately. b. Place a cap on the infants head and have the mother perform kangaroo care. c. Tell the mother that the infant must be kept in the nursery and observed for the next 4 hours. d. Change the formula because this is a sign of formula intolerance.

ANS: B Keeping the head well covered with a cap will prevent further heat loss from the head, and having the mother place the infant skin to skin should increase the infants temperature. Nursing actions are needed first to correct the problem. If the problem persists after interventions, notification may then be necessary. A slightly decreased temperature can be treated in the mothers room. This would be an excellent time for parent teaching on prevention of cold stress. Mild temperature instability is an expected deviation from normal during the first days as the infant adapts to external life. PTS: 1 DIF: Cognitive Level: Application REF: 620 OBJ: Nursing Process: Implementation MSC: Client Needs: Health Promotion and Maintenance

A newborn is placed under a radiant heat warmer, and the nurse evaluates the infants body temperature every hour. Maintaining the newborns body temperature is important for preventing: a. Respiratory depression. b. Cold stress. c. Tachycardia. d. Vasoconstriction.

ANS: B Loss of heat must be controlled to protect the infant from the metabolic and physiologic effects of cold stress, and that is the primary reason for placing a newborn under a radiant heat warmer. Cold stress results in an increased respiratory rate and vasoconstriction. PTS: 1 DIF: Cognitive Level: Comprehension REF: 561 OBJ: Nursing Process: Assessment MSC: Client Needs: Physiologic Integrity

Which woman is at greatest risk for early postpartum hemorrhage (PPH)? a. A primiparous woman (G 2 P 1 0 0 1) being prepared for an emergency cesarean birth for fetal distress b. A woman with severe preeclampsia who is receiving magnesium sulfate and whose labor is being induced c. A multiparous woman (G 3 P 2 0 0 2) with an 8-hour labor d. A primigravida in spontaneous labor with preterm twins

ANS: B Magnesium sulfate administration during labor poses a risk for PPH. Magnesium acts as a smooth muscle relaxant, thereby contributing to uterine relaxation and atony. Although many causes and risk factors are associated with PPH, the primiparous woman being prepared for an emergency C-section, the multiparous woman with 8-hour labor, and the primigravida in spontaneous labor do not pose risk factors or causes of early PPH. PTS: 1 DIF: Cognitive Level: Analysis REF: 531 OBJ: Nursing Process: Planning MSC: Client Needs: Physiologic Integrity

16. The nurse is caring for a child with acute respiratory distress syndrome (ARDS) associated with sepsis. Nursing actions should include: a. Force fluids. c. Institute seizure precautions. b. Monitor pulse oximetry. d. Encourage a high-protein diet

ANS: B Monitoring cardiopulmonary status is an important evaluation tool in the care of the child with ARDS. Maintenance of vascular volume and hydration is important and should be done parenterally. Seizures are not a side effect of ARDS. Adequate nutrition is necessary, but a high-protein diet is not helpful

To provide optimal care of infants born to mothers who are substance abusers, nurses should be aware that: a. Infants born to addicted mothers are also addicted. b. Mothers who abuse one substance likely will use or abuse another, thus compounding the infants difficulties. c. The NICU Network Neurobehavioral Scale (NNNS) is designed to assess the damage the mother has done to herself. d. No laboratory procedures are available that can identify the intrauterine drug exposure of the infant.

ANS: B Multiple substance use (even just alcohol and tobacco) makes it difficult to assess the problems of the exposed infant, particularly with regard to withdrawal manifestations. Infants of substance-abusing mothers may have some of the physiologic signs but are not addicted in the behavioral sense. Drug-exposed newborn is a more accurate description than addict. The NNNS is designed to assess the neurologic, behavioral, and stress/abstinence function of the neonate. Newborn urine, hair, or meconium sampling may be used to identify an infants intrauterine drug exposure. PTS: 1 DIF: Cognitive Level: Comprehension REF: 676 OBJ: Nursing Process: Evaluation MSC: Client Needs: Psychosocial Integrity

Which statement by a newly delivered woman indicates that she knows what to expect about her menstrual activity after childbirth? a. My first menstrual cycle will be lighter than normal and then will get heavier every month thereafter. b. My first menstrual cycle will be heavier than normal and will return to my prepregnant volume within three or four cycles. c. I will not have a menstrual cycle for 6 months after childbirth. d. My first menstrual cycle will be heavier than normal and then will be light for several months after.

ANS: B My first menstrual cycle will be heavier than normal and will return to my prepregnant volume within three or four cycles is an accurate statement and indicates her understanding of her expected menstrual activity. She can expect her first menstrual cycle to be heavier than normal (which occurs by 3 months after childbirth), and the volume of her subsequent cycles will return to prepregnant levels within three or four cycles. PTS: 1 DIF: Cognitive Level: Application REF: 486 OBJ: Nursing Process: Evaluation MSC: Client Needs: Health Promotion and Maintenance

36. A school-age child with chronic renal failure is admitted to the hospital with a serum potassium level of 5.2 mEq/L. Which prescribed medication should the nurse plan to administer? a. Spironolactone (Aldactone) b. Sodium polystyrene sulfonate (Kayexalate) c. Lactulose (Cephulac) d. Calcium carbonate (Calcitab)

ANS: B Normal serum potassium levels in a school-age child are 3.5 to 5 mEq/L. Sodium polystyrene sulfonate is administered to reduce serum potassium levels. Spironolactone is a potassium-sparing diuretic and should not be used if the serum potassium is elevated. Lactulose is administered to reduce ammonia levels in patients with liver disease. Calcium carbonate may be prescribed as a calcium supplement, but it will not reduce serum potassium levels.

22.Calcium carbonate is given with meals to a child with chronic renal disease. The purpose of this is to: a. Prevent vomiting. b. Bind phosphorus. c. Stimulate appetite. d. Increase absorption of fat-soluble vitamins.

ANS: B Oral calcium carbonate preparations combine with phosphorus to decrease gastrointestinal absorption and the serum levels of phosphate; serum calcium levels are increased by the calcium carbonate, and vitamin D administration is necessary to increase calcium absorption. Calcium carbonate does not prevent vomiting, stimulate appetite, or increase the absorption of fat-soluble vitamins.

When working with parents who have some form of sensory impairment, nurses should understand that ________ is an inaccurate statement. a. One of the major difficulties visually impaired parents experience is the skepticism of health care professionals. b. Visually impaired mothers cannot overcome the infants need for eye-to-eye contact. c. The best approach for the nurse is to assess the parents capabilities rather than focusing on their disabilities. d. Technologic advances, including the Internet, can provide deaf parents with a full range of parenting activities and information.

ANS: B Other sensory output can be provided by the parent, other people can participate, and other coping devices can be used. The skepticism, open or hidden, of health care professionals places an additional and unneeded hurdle for the parents. After the parents capabilities have been assessed (including some the nurse may not have expected), the nurse can help find ways to assist the parents that play to their strengths. The Internet affords an extra teaching tool for the deaf, as do videos with subtitles or nurses signing. A number of electronic devices can turn sound into light flashes to help pick up a childs cry. Sign language is readily acquired by young children. PTS: 1 DIF: Cognitive Level: Comprehension REF: 523 OBJ: Nursing Process: Planning MSC: Client Needs: Health Promotion and Maintenance

39. A nurse works on a pediatric oncology unit. After receiving report, which child should the nurse assess first? A. Having infusion of D5 NS and sodium bicarbonate B. On high-dose methotrexate (Rheumatrex), urine pH of 7.8 C. Receiving cyclophosphamide (Cytoxan), urine specific gravity of 1.008 D. 2 days post-tumor resection, complaining of pain

ANS: B Patients on high-dose methotrexate need their urine pH to be higher than 7.0. This child needs the nurse's attention first. An IV with NaHCO3 is common prior to receiving methotrexate. A urine specific gravity of 1.010 is required for children on chemotherapy. Pain would be an expected finding 2 days postoperatively, and should be treated, but not before the nurse assesses the other child.

The prevalence of urinary incontinence (UI) increases as women age, with more than one third of women in the United States suffering from some form of this disorder. The symptoms of mild to moderate UI can be successfully decreased by a number of strategies. Which of these should the nurse instruct the client to use first? a. Pelvic floor support devices b. Bladder training and pelvic muscle exercises c. Surgery d. Medications

ANS: B Pelvic muscle exercises, known as Kegel exercises, along with bladder training can significantly decrease or entirely relieve stress incontinence in many women. Pelvic floor support devices, also known as pessaries, come in a variety of shapes and sizes. Pessaries may not be effective for all women and require scrupulous cleaning to prevent infection. Anterior and posterior repairs and even a hysterectomy may be performed. If surgical repair is performed, the nurse must focus her care on preventing infection and helping the woman avoid putting stress on the surgical site. Pharmacologic therapy includes serotonin-norepinephrine uptake inhibitors or vaginal estrogen therapy. These are not the first action a nurse should recommend. PTS: 1 DIF: Cognitive Level: Application REF: 545 OBJ: Nursing Process: Implementation MSC: Client Needs: Physiologic Integrity

28. The narrowing of preputial opening of foreskin is called: a. Chordee b. Phimosis c. Epispadias d. Hypospadias

ANS: B Phimosis is the narrowing or stenosis of the preputial opening of the foreskin. Chordee is the ventral curvature of the penis. Epispadias is the meatal opening on the dorsal surface of the penis. Hypospadias is a congenital condition in which the urethral opening is located anywhere along the ventral surface of the penis.

6.The narrowing of the preputial opening of the foreskin is called: a. Chordee. b. Phimosis. c.Epispadias d.Hypospadias.

ANS: B Phimosis is the narrowing or stenosis of the preputial opening of the foreskin. Chordee is the ventral curvature of the penis. Epispadias is the meatal opening on the dorsal surface of the penis. Hypospadias is a congenital condition in which the urethral opening is located anywhere along the ventral surface of the penis.

9. Which statement is descriptive of the play of school-age children? a. Individuality in play is better tolerated than at earlier ages. b. Knowing the rules of a game gives an important sense of belonging. c. They like to invent games, making up the rules as they go. d. Team play helps children learn the universal importance of competition and winning.

ANS: B Play involves increased physical skill, intellectual ability, and fantasy. Children form groups and cliques and develop a sense of belonging to a team or club. At this age, children begin to see the need for rules. Conformity and ritual permeate their play. Their games have fixed and unvarying rules, which may be bizarre and extraordinarily rigid. With team play, children learn about competition and the importance of winning, an attribute highly valued in the United States.

The nurse observes that a 15-year-old mother seems to ignore her newborn. A strategy that the nurse can use to facilitate mother-infant attachment in this mother is to: a. Tell the mother she must pay attention to her infant. b. Show the mother how the infant initiates interaction and pays attention to her. c. Demonstrate for the mother different positions for holding her infant while feeding. d. Arrange for the mother to watch a video on parent-infant interaction.

ANS: B Pointing out the responsiveness of the infant is a positive strategy for facilitating parent-infant attachment. Telling the mother that she must pay attention to her infant may be perceived as derogatory and is not appropriate. Educating the young mother in infant care is important; however, pointing out the responsiveness of her baby is a better tool for facilitating mother-infant attachment. Videos are an educational tool that can demonstrate parent-infant attachment, but encouraging the mother to recognize the infants responsiveness is more appropriate. PTS: 1 DIF: Cognitive Level: Application REF: 516 OBJ: Nursing Process: Implementation MSC: Client Needs: Health Promotion and Maintenance

Necrotizing enterocolitis (NEC) is an inflammatory disease of the gastrointestinal mucosa. The signs of NEC are nonspecific. Some generalized signs include: a. Hypertonia, tachycardia, and metabolic alkalosis. b. Abdominal distention, temperature instability, and grossly bloody stools. c. Hypertension, absence of apnea, and ruddy skin color. d. Scaphoid abdomen, no residual with feedings, and increased urinary output.

ANS: B Some generalized signs of NEC include decreased activity, hypotonia, pallor, recurrent apnea and bradycardia, decreased oxygen saturation values, respiratory distress, metabolic acidosis, oliguria, hypotension, decreased perfusion, temperature instability, cyanosis, abdominal distention, residual gastric aspirates, vomiting, grossly bloody stools, abdominal tenderness, and erythema of the abdominal wall. The infant may display hypotonia, bradycardia, and metabolic acidosis. PTS: 1 DIF: Cognitive Level: Comprehension REF: 693 OBJ: Nursing Process: Assessment MSC: Client Needs: Physiologic Integrity

With regard to the adaptation of other family members, mainly siblings and grandparents, to the newborn, nurses should be aware that: a. Sibling rivalry cannot be dismissed as overblown psychobabble; negative feelings and behaviors can take a long time to blow over. b. Participation in preparation classes helps both siblings and grandparents. c. In the United States paternal and maternal grandparents consider themselves of equal importance and status. d. In the past few decades the number of grandparents providing permanent care to their grandchildren has been declining.

ANS: B Preparing older siblings and grandparents helps everyone to adapt. Sibling rivalry should be expected initially, but the negative behaviors associated with it have been overemphasized and stop in a comparatively short time. In the United States, in contrast to other cultures, paternal grandparents frequently consider themselves secondary to maternal grandparents. The number of grandparents providing permanent child care has been on the increase. PTS: 1 DIF: Cognitive Level: Comprehension REF: 525 OBJ: Nursing Process: Planning MSC: Client Needs: Psychosocial Integrity

The hormone necessary for milk production is: a. Estrogen. b. Prolactin. c. Progesterone. d. Lactogen.

ANS: B Prolactin, secreted by the anterior pituitary, is a hormone that causes the breasts to produce milk. Estrogen decreases the effectiveness of prolactin and prevents mature breast milk from being produced. Progesterone decreases the effectiveness of prolactin and prevents mature breast milk from being produced. Human placental lactogen decreases the effectiveness of prolactin and prevents mature breast milk from being produced. PTS: 1 DIF: Cognitive Level: Knowledge REF: 638 OBJ: Nursing Process: Assessment MSC: Client Needs: Physiologic Integrity

In caring for the preterm infant, what complication is thought to be a result of high arterial blood oxygen level? a. Necrotizing enterocolitis (NEC) b. Retinopathy of prematurity (ROP) c. Bronchopulmonary dysplasia (BPD) d. Intraventricular hemorrhage (IVH)

ANS: B ROP is thought to occur as a result of high levels of oxygen in the blood. NEC is caused by the interference of blood supply to the intestinal mucosa. Necrotic lesions occur at that site. BPD is caused by the use of positive pressure ventilation against the immature lung tissue. IVH results from rupture of the fragile blood vessels in the ventricles of the brain. It is most often associated with hypoxic injury, increased blood pressure, and fluctuating cerebral blood flow. PTS: 1 DIF: Cognitive Level: Comprehension REF: 690 OBJ: Nursing Process: Assessment MSC: Client Needs: Physiologic Integrity

The postpartum woman who continually repeats the story of her labor, delivery, and recovery experience is: a. Providing others with her knowledge of events. b. Making the birth experience real. c. Taking hold of the events leading to her labor and delivery. d. Accepting her response to labor and delivery.

ANS: B Reliving the birth experience makes the event real and helps the mother realize that the pregnancy is over and that the infant is born and is now a separate individual. The retelling of the story is to satisfy her needs, not the needs of others. This new mother is in the taking-in phase, trying to make the birth experience seem real and separate the infant from herself. PTS: 1 DIF: Cognitive Level: Knowledge REF: 516 OBJ: Nursing Process: Assessment MSC: Client Needs: Psychosocial Integrity

26.Which statement is descriptive of renal transplantation in children? a. It is an acceptable means of treatment after age 10 years. b. It is preferred means of renal replacement therapy in children. c. Children can receive kidneys only from other children. d. The decision for transplantation is difficult since a relatively normal lifestyle is not possible.

ANS: B Renal transplantation offers the opportunity for a relatively normal lifestyle versus dependence on dialysis and is the preferred means of renal replacement therapy in end-stage renal disease. It can be done in children as young as age 6 months. Both children and adults can serve as donors for renal transplant purposes.

With regard to the postpartum changes and developments in a womans cardiovascular system, nurses should be aware that: a. Cardiac output, the pulse rate, and stroke volume all return to prepregnancy normal values within a few hours of childbirth. b. Respiratory function returns to nonpregnant levels by 6 to 8 weeks after birth. c. The lowered white blood cell count after pregnancy can lead to false-positive results on tests for infections. d. A hypercoagulable state protects the new mother from thromboembolism, especially after a cesarean birth.

ANS: B Respirations should decrease to within the womans normal prepregnancy range by 6 to 8 weeks after birth. Stroke volume increases, and cardiac output remains high for a couple of days. However, the heart rate and blood pressure return to normal quickly. Leukocytosis increases 10 to 12 days after childbirth and can obscure the diagnosis of acute infections (false-negative results). The hypercoagulable state increases the risk of thromboembolism, especially after a cesarean birth. PTS: 1 DIF: Cognitive Level: Comprehension REF: 487 OBJ: Nursing Process: Planning MSC: Client Needs: Physiologic Integrity

25. Parents of a 12-year-old child ask the clinic nurse, "How many hours of sleep should our child get each night?" The nurse should respond that 12-year-old children need ____ hours of sleep at night. a. 8 b. 9 c. 10 d. 11

ANS: B School-age children usually do not require naps, but they do need to sleep approximately 11 hours at age 5 years and 9 hours at age 12 years each night.

A new mother recalls from prenatal class that she should try to feed her newborn daughter when she exhibits feeding readiness cues rather than waiting until her infant is crying frantically. On the basis of this information, this woman should feed her infant about every 2.5 to 3 hours when she: a. Waves her arms in the air. b. Makes sucking motions. c. Has hiccups. d. Stretches her legs out straight.

ANS: B Sucking motions, rooting, mouthing, and hand-to-mouth motions are examples of feeding-readiness cues. Waving the arms in the air, hiccupping, and stretching the legs out straight are not typical feeding-readiness cues. PTS: 1 DIF: Cognitive Level: Comprehension REF: 641 OBJ: Nursing Process: Planning MSC: Client Needs: Health Promotion and Maintenance

By knowing about variations in infants blood count, nurses can explain to their clients that: a. A somewhat lower than expected red blood cell count could be the result of delay in clamping the umbilical cord. b. The early high white blood cell (WBC) count is normal at birth and should decrease rapidly. c. Platelet counts are higher than in adults for a few months. d. Even a modest vitamin K deficiency means a problem with the ability of the blood to clot properly.

ANS: B The WBC count is high the first day of birth and then declines rapidly. Delayed clamping of the cord results in an increase in hemoglobin and the red blood cell count. The platelet count essentially is the same for newborns and adults. Clotting is sufficient to prevent hemorrhage unless the vitamin K deficiency is significant. PTS: 1 DIF: Cognitive Level: Comprehension REF: 560 OBJ: Nursing Process: Planning MSC: Client Needs: Health Promotion and Maintenance

22. A nurse is assessing an older school-age child recently admitted to the hospital. Which assessment indicates that the child is in an appropriate stage of cognitive development? a. The child's addition and subtraction ability b. The child's ability to classify c. The child's vocabulary d. The child's play activity

ANS: B The ability to classify things from simple to complex and the ability to identify differences and similarities are cognitive skills of the older school-age child; this demonstrates use of classification and logical thought processes. Subtraction and addition are appropriate cognitive activities for the young school-age child. Vocabulary is not as valid an assessment of cognitive ability as is the child's ability to classify. Play activity is not as valid an assessment of cognitive function as is the ability to classify.

A 5-year-old boy is being prepared for surgery to remove a brain tumor. Nursing actions should be based on which statement? a. Removal of tumor will stop the various symptoms. b. Usually the postoperative dressing covers the entire scalp. c. He is not old enough to be concerned about his head being shaved. d. He is not old enough to understand the significance of the brain.

ANS: B The child should be told what he will look and feel like after surgery. This includes the size of the dressing. The nurse can demonstrate on a doll the expected size and shape of the dressing. Some of the symptoms may be alleviated by the removal of the tumor, but postsurgical headaches and cerebellar symptoms such as ataxia may be aggravated. Children should be prepared for the loss of their hair, and it should be removed in a sensitive, positive manner if the child is awake. Children at this age have poorly defined body boundaries and little knowledge of internal organs. Intrusive experiences are frightening, especially those that disrupt the integrity of the skin.

12. The nurse encourages the mother of a toddler with acute laryngotracheobronchitis to stay at the bedside as much as possible. The nurses rationale for this action is primarily that: a. Mothers of hospitalized toddlers often experience guilt. b. The mothers presence will reduce anxiety and ease the childs respiratory efforts. c. Separation from the mother is a major developmental threat at this age. d. The mother can provide constant observations of the childs respiratory efforts.

ANS: B The familys presence will decrease the childs distress. The mother may experience guilt, but this is not the best answer. Although separation from the mother is a developmental threat for toddlers, the main reason to keep parents at the childs bedside is to ease anxiety and therefore respiratory effort. The child should have constant cardiorespiratory monitoring and noninvasive oxygen saturation monitoring, but the parent should not play this role in the hospital.

A woman gave birth to a healthy 7-pound, 13-ounce infant girl. The nurse suggests that the woman place the infant to her breast within 15 minutes after birth. The nurse knows that breastfeeding is effective during the first 30 minutes after birth because this is the: a. Transition period. b. First period of reactivity. c. Organizational stage. d. Second period of reactivity.

ANS: B The first period of reactivity is the first phase of transition and lasts up to 30 minutes after birth. The infant is highly alert during this phase. The transition period is the phase between intrauterine and extrauterine existence. There is no such phase as the organizational stage. The second period of reactivity occurs roughly between 4 and 8 hours after birth, after a period of prolonged sleep. PTS: 1 DIF: Cognitive Level: Comprehension REF: 556 OBJ: Nursing Process: Planning MSC: Client Needs: Health Promotion and Maintenance

The nurse has received report on four children. Which child should the nurse assess first? a. A school-age child in a coma with stable vital signs b. A preschool child with a head injury and decreasing level of consciousness c. An adolescent admitted after a motor vehicle accident is oriented to person and place d. A toddler in a persistent vegetative state with a low-grade fever

ANS: B The nurse should assess the child with a head injury and decreasing level of consciousness first (LOC). Assessment of LOC remains the earliest indicator of improvement or deterioration in neurologic status. The next child the nurse should assess is a toddler in a persistent vegetative state with a low-grade fever. The school-age child in a coma with stable vital signs and the adolescent adm

A newborn in the neonatal intensive care unit (NICU) is dying as a result of a massive infection. The parents speak to the neonatologist, who informs them of their sons prognosis. When the father sees his son, he says, He looks just fine to me. I cant understand what all this is about. The most appropriate response by the nurse would be: a. Didnt the doctor tell you about your sons problems? b. This must be a difficult time for you. Tell me how youre doing. c. To stand beside him quietly. d. Youll have to face up to the fact that he is going to die sooner or later.

ANS: B The grief phase can be very difficult, especially for fathers. Parents should be encouraged to share their feelings as the initial steps in the grieving process. This father is in a phase of acute distress and is reaching out to the nurse as a source of direction in his grieving process. Shifting the focus is not in the best interest of the parent. Nursing actions may help the parents actualize the loss of their infant through sharing and verbalization of feelings of grief. Youll have to face up to the fact that he is going to die sooner or later is a dispassionate and inappropriate statement. PTS: 1 DIF: Cognitive Level: Application REF: 534 OBJ: Nursing Process: Planning MSC: Client Needs: Psychosocial Integrity

A woman is diagnosed with having a stillborn. At first, she appears stunned by the news, cries a little, and then asks you to call her mother. The phase of bereavement the woman is experiencing is called: a. Anticipatory grief. b. Acute distress. c. Intense grief. d. Reorganization.

ANS: B The immediate reaction to news of a perinatal loss or infant death encompasses a period of acute distress. Disbelief and denial can occur. However, parents also feel very sad and depressed. Intense outbursts of emotion and crying are normal. However, lack of affect, euphoria, and calmness may occur and may reflect numbness, denial, or personal ways of coping with stress. Anticipatory grief applies to the grief related to a potential loss of an infant. The parent grieves in preparation of the infants possible death, although he or she clings to the hope that the child will survive. Intense grief occurs in the first few months after the death of the infant. This phase encompasses many different emotions, including loneliness, emptiness, yearning, guilt, anger, and fear. Reorganization occurs after a long and intense search for meaning. Parents are better able to function at work and home, experience a return of self-esteem and confidence, can cope with new challenges, and have placed the loss in perspective. PTS: 1 DIF: Cognitive Level: Comprehension REF: 546 OBJ: Nursing Process: Diagnosis MSC: Client Needs: Psychosocial Integrity

34. The nurse is conducting teaching for an adolescent being discharged to home after a renal transplantation. The adolescent needs further teaching if which statement is made? a. "I will report any fever to my primary health care provider." b. "I am glad I only have to take the immunosuppressant medication for two weeks." c. "I will observe my incision for any redness or swelling." d. "I won't miss doing kidney dialysis every week."

ANS: B The immunosuppressant medications are taken indefinitely after a renal transplantation, so they should not be discontinued after 2 weeks. Reporting a fever and observing an incision for redness and swelling are accurate statements. The adolescent is correct in indicating dialysis will not need to be done after the transplantation.

A primary nursing responsibility when caring for a woman experiencing an obstetric hemorrhage associated with uterine atony is to: a. Establish venous access. b. Perform fundal massage. c. Prepare the woman for surgical intervention. d. Catheterize the bladder.

ANS: B The initial management of excessive postpartum bleeding is firm massage of the uterine fundus. Although establishing venous access may be a necessary intervention, the initial intervention would be fundal massage. The woman may need surgical intervention to treat her postpartum hemorrhage, but the initial nursing intervention would be to assess the uterus. After uterine massage the nurse may want to catheterize the client to eliminate any bladder distention that may be preventing the uterus from contracting properly. PTS: 1 DIF: Cognitive Level: Application REF: 532 OBJ: Nursing Process: Implementation MSC: Client Needs: Physiologic Integrity

15. What is an important consideration for the school nurse who is planning a class on bicycle safety to consider? a. Most bicycle injuries involve collision with an automobile. b. Head injuries are the major causes of bicycle-related fatalities. c. Children should wear bicycle helmets if they ride on paved streets. d. Children should not ride double unless the bicycle has an extra-large seat.

ANS: B The most important aspect of bicycle safety is to encourage the rider to use a protective helmet. Head injuries are the major cause of bicycle-related fatalities. Although motor vehicle collisions do cause injuries to bicyclists, most injuries result from falls. The child should always wear a properly fitted helmet approved by the U.S. Consumer Product Safety Commission. Children should not ride double.

A newborn was admitted to the neonatal intensive care unit after being delivered at 29 weeks of gestation to a 28-year-old multiparous, married, Caucasian woman whose pregnancy was uncomplicated until premature rupture of membranes and preterm birth. The newborns parents arrive for their first visit after the birth. The parents walk toward the bedside but remain approximately 5 feet away from the bed. The nurses most appropriate action would be to: a. Wait quietly at the newborns bedside until the parents come closer. b. Go to the parents, introduce himself or herself, and gently encourage the parents to come meet their infant; explain the equipment first, and then focus on the newborn. c. Leave the parents at the bedside while they are visiting so they can have some privacy. d. Tell the parents only about the newborns physical condition, and caution them to avoid touching their baby.

ANS: B The nurse is instrumental in the initial interactions with the infant. The nurse can help the parents see the infant, rather than focus on the equipment. The importance and purpose of the apparatus that surrounds their infant also should be explained to them. Parents often need encouragement and recognition from the nurse to acknowledge the reality of the infants condition. Parents need to see and touch their infant as soon as possible to acknowledge the reality of the birth and the infants appearance and condition. Encouragement from the nurse is instrumental in this process. Telling the parents only about the newborns physical condition and cautioning them to avoid touching their baby is an inappropriate action. PTS: 1 DIF: Cognitive Level: Application REF: 705 OBJ: Nursing Process: Implementation MSC: Client Needs: Psychosocial Integrity

3. A child is diagnosed with juvenile hypothyroidism. The nurse should expect to assess which symptoms associated with hypothyroidism? (Select all that apply.) a. Weight loss b. Sleepiness c. Diarrhea d. Puffiness around the eyes e. Spare hair

ANS: B, D, E A child diagnosed with juvenile hypothyroidism will display sleepiness; dry, periorbital puffiness; and spare hair growth. Weight loss and diarrhea are signs of hyperthyroidism.

During the complete physical examination 24 hours after birth: a. The parents are excused to reduce their normal anxiety. b. The nurse can gauge the neonates maturity level by assessing the infants general appearance. c. Once often neglected, blood pressure is now routinely checked. d. When the nurse listens to the heart, the S1 and S2 sounds can be heard; the first sound is somewhat higher in pitch and sharper than the second.

ANS: B The nurse will be looking at skin color, alertness, cry, head size, and other features. The parents presence actively involves them in child care and gives the nurse a chance to observe interactions. Blood pressure is not usually taken unless cardiac problems are suspected. The second sound is higher and sharper than the first. PTS: 1 DIF: Cognitive Level: Comprehension REF: 583 OBJ: Nursing Process: Assessment MSC: Client Needs: Health Promotion and Maintenance

7.An objective of care for the child with nephrosis is to: a. Reduce blood pressure. b. Reduce excretion of urinary protein. c. Increase excretion of urinary protein. d. Increase ability of tissues to retain fluid.

ANS: B The objectives of therapy for the child with nephrosis include reduction of the excretion of urinary protein, reduction of fluid retention, prevention of infection, and minimizing of complications associated with therapy. Blood pressure is usually not elevated in nephrosis. Increased excretion of urinary protein and increased ability of tissues to retain fluid are part of the disease process and must be reversed.

The process whereby parents awaken the infant to feed every 3 hours during the day and at least every 4 hours at night is: a. Known as demand feeding. b. Necessary during the first 24 to 48 hours after birth. c. Used to set up the supply-meets-demand system. d. A way to control cluster feeding.

ANS: B The parents do this to make sure that the infant has at least eight feedings in 24 hours. Demand feeding is when the infant determines the frequency of feedings; this is appropriate once the infant is feeding well and gaining weight. The supply-meets-demand system is a milk production system that occurs naturally. Cluster feeding is not a problem if the baby has eight feedings in 24 hours. PTS: 1 DIF: Cognitive Level: Comprehension REF: 643 OBJ: Nursing Process: Diagnosis MSC: Client Needs: Health Promotion and Maintenance

A new father wants to know what medication was put into his infants eyes and why it is needed. The nurse explains to the father that the purpose of the Ilotycin ophthalmic ointment is to: a. Destroy an infectious exudate caused by Staphylococcus that could make the infant blind. b. Prevent gonorrheal and chlamydial infection of the infants eyes potentially acquired from the birth canal. c. Prevent potentially harmful exudate from invading the tear ducts of the infants eyes, leading to dry eyes. d. Prevent the infants eyelids from sticking together and help the infant see.

ANS: B The purpose of the Ilotycin ophthalmic ointment is to prevent gonorrheal and chlamydial infection of the infants eyes potentially acquired from the birth canal. Prophylactic ophthalmic ointment is instilled in the eyes of all neonates to prevent gonorrheal or chlamydial infection. Prophylactic ophthalmic ointment is not instilled to prevent dry eyes. Prophylactic ophthalmic ointment has no bearing on vision other than to protect against infection that may lead to vision problems. PTS: 1 DIF: Cognitive Level: Comprehension REF: 603 OBJ: Nursing Process: Planning MSC: Client Needs: Health Promotion and Maintenance

The nurse is closely monitoring a child who is unconscious after a fall and notices that the child suddenly has a fixed and dilated pupil. The nurse should interpret this as: a. eye trauma. b. neurosurgical emergency. c. severe brainstem damage. d. indication of brain death.

ANS: B The sudden appearance of a fixed and dilated pupil(s) is a neurosurgical emergency. The nurse should immediately report this finding. Although a dilated pupil may be associated with eye trauma, this child has experienced a neurologic insult. Pinpoint pupils or bilateral fixed pupils for more than 5 minutes are indicative of brainstem damage. The unilateral fixed and dilated pupil is suggestive of damage on the same side of the brain. One fixed and dilated pupil is not suggestive of brain death.

21. Which behavior by parents or teachers will best assist the child in negotiating the developmental task of industry? a. Identifying failures immediately and asking the child's peers for feedback b. Structuring the environment so the child can master tasks c. Completing homework for children who are having difficulty in completing assignments d. Decreasing expectations to eliminate potential failures

ANS: B The task of the caring teacher or parent is to identify areas in which a child is competent and to build on successful experiences to foster feelings of mastery and success. Structuring the environment to enhance self-confidence and to provide the opportunity to solve increasingly more complex problems will promote a sense of mastery. Asking peers for feedback reinforces the child's feelings of failure. When teachers or parents complete children's homework for them, it sends the message that they do not trust the children to do a good job. Providing assistance and suggestions and praising their best efforts are more appropriate. Decreasing expectations to eliminate failures will not promote a sense of achievement or mastery.

The nurse observes several interactions between a postpartum woman and her new son. What behavior, if exhibited by this woman, would the nurse identify as a possible maladaptive behavior regarding parent-infant attachment? a. Talks and coos to her son b. Seldom makes eye contact with her son c. Cuddles her son close to her d. Tells visitors how well her son is feeding

ANS: B The woman should be encouraged to hold her infant in the en face position and make eye contact with the infant. Normal infant-parent interactions include talking and cooing to her son, cuddling her son close to her, and telling visitors how well her son is feeding. PTS: 1 DIF: Cognitive Level: Application REF: 524 OBJ: Nursing Process: Assessment, Diagnosis MSC: Client Needs: Psychosocial Integrity

32. A nurse assesses a toddler using the FLACC score. The child is kicking and crying steadily. The mother is upset, as she is unable to console the child. Which action by the nurse is most appropriate? A. Administer acetaminophen (Tylenol). B. Give a dose of morphine (Duramorph). C. Play soothing, quiet music. D. Prepare a dose of propofol (Diprovan).

ANS: B This child exhibits several behaviors seen in the severe pain category according to the FLACC score. The best medication for this level of pain is an opioid analgesic, such as morphine. Tylenol is used for mild to moderate pain. Nonpharmacological measures can be used as an adjunct, but it will not relieve this degree of pain alone. Propofol is usually used for procedures.

5. A parent brings a child to the clinic and reports that the child is reluctant to walk and has a new limp. The parent also reports that the child seems lethargic and tired all the time. The nurse notes that the child appears pale. Which other finding would warrant immediate notification of the health-care provider? A. Difficulty staying asleep at night B. Left-sided abdominal enlargement C. Polyphagia and polydipsia D. Swelling of the legs and feet

ANS: B This child has some manifestations of acute lymphocytic leukemia (ALL). Left-sided abdominal enlargement could be indicative of splenomegaly, which is another manifestation of this disease. The nurse should report these findings immediately. Difficulty staying asleep at night is vague and could be related to a number of causes, both physical and behavioral. Polydipsia and polyphagia are two of the three classic signs of diabetes. Swelling of the legs and feet is not a manifestation of ALL.

Which is an appropriate nursing intervention when caring for a child in traction? a. Remove adhesive traction straps daily to prevent skin breakdown. b. Assess for tightness, weakness, or contractures in uninvolved joints and muscles. c. Provide active range-of-motion exercises to affected extremity three times a day. d. Keep the child in one position to maintain good alignment.

ANS: B Traction places stress on the affected bone, joint, and muscles. The nurse must assess for tightness, weakness, or contractures developing in the uninvolved joints and muscles. The adhesive straps should be released or replaced only when absolutely necessary. Active, passive, or active with resistance exercises should be carried out for the unaffected extremity only. Movement is expected with children. Each time the child moves, the nurse should check to ensure that proper alignment is maintained.

26. A child with cystic fibrosis is receiving recombinant human deoxyribonuclease (rhDNase). This drug: a. May cause mucus to thicken. b. May cause voice alterations. c. Is given subcutaneously. d. Is not indicated for children younger than 12 years.

ANS: B Two of the only adverse effects of rhDNase are voice alterations and laryngitis. rhDNase decreases viscosity of mucus, is given in an aerosolized form, and is safe for children younger than 12 years of age.

A steady trickle of bright red blood from the vagina in the presence of a firm fundus suggests: a. Uterine atony. b. Lacerations of the genital tract. c. Perineal hematoma. d. Infection of the uterus.

ANS: B Undetected lacerations will bleed slowly and continuously. Bleeding from lacerations is uncontrolled by uterine contraction. The fundus is not firm in the presence of uterine atony. A hematoma would develop internally. Swelling and discoloration would be noticeable; however, bright bleeding would not be. With an infection of the uterus there would be an odor to the lochia and systemic symptoms such as fever and malaise. PTS: 1 DIF: Cognitive Level: Comprehension REF: 531 OBJ: Nursing Process: Assessment MSC: Client Needs: Physiologic Integrity

The vector reservoir for agents causing viral encephalitis in the United States is: a. tarantula spiders. b. mosquitoes. c. carnivorous wild animals. d. domestic and wild animals.

ANS: B Viral encephalitis, not attributable to a childhood viral disease, is usually transmitted by mosquitoes. The vector reservoir for most agents pathogenic for humans and detected in the United States are mosquitoes and ticks; therefore, most cases of encephalitis appear during the hot summer months. Tarantula spiders, carnivorous wild animals, and domestic and wild animals are not reservoirs for the agents that cause viral encephalitis.

18. Asthma in infants is usually triggered by: a. Medications. c. Exposure to cold air. b. A viral infection. d. Allergy to dust or dust mites.

ANS: B Viral illnesses cause inflammation that causes increased airway reactivity in asthma. Medications such as aspirin, nonsteroidal antiinflammatory drugs, and antibiotics may aggravate asthma, but not frequently in infants. Exposure to cold air may exacerbate already existing asthma. Allergy is associated with asthma, but 20% to 40% of children with asthma have no evidence of allergic disease.

16. When teaching injury prevention during the school-age years, the nurse should include what topic? a. Teaching the need to fear strangers. b. Teaching basic rules of water safety. c. Avoiding letting children cook in microwave ovens. d. Cautioning children against engaging in competitive sports.

ANS: B Water safety instruction is an important source of injury prevention at this age. The child should be taught to swim, select safe and supervised places to swim, swim with a companion, check sufficient water depth for diving, and use an approved flotation device. Teach stranger safety, not fearing strangers. This includes not going with strangers, not having personalized clothing in public places, having children tell parents if anyone makes them uncomfortable, and teaching children to say "no" in uncomfortable situations. Teach children safe cooking methods. Caution against engaging in hazardous sports, such as those involving trampolines.

An adolescent with a fractured femur is in Russell's traction. Surgical intervention to correct the fracture is scheduled for the morning. Nursing actions should include which action? a. Maintaining continuous traction until 1 hour before the scheduled surgery b. Maintaining continuous traction and checking position of traction frequently c. Releasing traction every hour to perform skin care d. Releasing traction once every 8 hours to check circulation

ANS: B When the muscles are stretched, muscle spasm ceases and permits realignment of the bone ends. The continued maintenance of traction is important during this phase because releasing the traction allows the muscle's normal contracting ability to again cause malpositioning of the bone ends. Continuous traction must be maintained to keep the bone ends in satisfactory realignment. Releasing at any time, either 1 hour before surgery, once every hour for skin care, or once every 8 hours would not keep the fracture in satisfactory alignment.

As relates to rubella and Rh issues, nurses should be aware that: a. Breastfeeding mothers cannot be vaccinated with the live attenuated rubella virus. b. Women should be warned that the rubella vaccination is teratogenic, and that they must avoid pregnancy for 1 month after vaccination. c. Rh immune globulin is safely administered intravenously because it cannot harm a nursing infant. d. Rh immune globulin boosts the immune system and thereby enhances the effectiveness of vaccinations.

ANS: B Women should understand they must practice contraception for 1 month after being vaccinated. Because the live attenuated rubella virus is not communicable in breast milk, breastfeeding mothers can be vaccinated. Rh immune globulin is administered intramuscularly; it should never be given to an infant. Rh immune globulin suppresses the immune system and therefore could thwart the rubella vaccination. PTS: 1 DIF: Cognitive Level: Comprehension REF: 503 OBJ: Nursing Process: Planning MSC: Client Needs: Health Promotion and Maintenance

The nurse is explaining the benefits associated with breastfeeding to a new mother. Which statement by the nurse would be inaccurate and provide conflicting information to the patient? a. Women who breastfeed have a decreased risk of breast cancer. b. Breastfeeding is an effective method of birth control. c. Breastfeeding increases bone density. d. Breastfeeding may enhance postpartum weight loss.

ANS: B Women who breastfeed have a decreased risk of breast cancer, an increase in bone density, and a possibility of quicker postpartum weight loss. Breastfeeding delays the return of fertility; however, it is not an effective birth control method. PTS: 1 DIF: Cognitive Level: Comprehension REF: 635 OBJ: Nursing Process: Planning MSC: Client Needs: Health Promotion and Maintenance

26. What should a nurse advise the parents of a child with type 1 diabetes mellitus who is not eating as a result of a minor illness? a. Give the child half his regular morning dose of insulin. b. Substitute simple carbohydrates or calorie-containing liquids for solid foods. c. Give the child plenty of unsweetened, clear liquids to prevent dehydration. d. Take the child directly to the emergency department.

ANS: B A sick-day diet of simple carbohydrates or calorie-containing liquids will maintain normal serum glucose levels and decrease the risk of hypoglycemia. The child should receive his regular dose of insulin even if he does not have an appetite. If the child is not eating as usual, he needs calories to prevent hypoglycemia. During periods of minor illness, the child with type 1 diabetes mellitus can be managed safely at home.

4. What is a common cause of acute diarrhea? a. Hirschsprung's disease b. Antibiotic therapy c. Hypothyroidism d. Meconium ileus

ANS: B Acute diarrhea is a sudden increase in frequency and change in consistency of stools and may be associated with antibiotic therapy. Hirschsprung's disease, hypothyroidism, and meconium ileus are usually manifested with constipation rather than diarrhea.

4. A youngster will receive a bone marrow transplant (BMT) made possible because an older siblings is a histocompatible donor. Which is this type of BMT called? a. Syngeneic b. Allogeneic c. Monoclonal d. Autologous

ANS: B Allogeneic transplants are from another individual. Because he and his sibling are histocompatible, the BMT can be done. Syngeneic marrow is from an identical twin. There is no such thing as a monoclonal BMT. Autologous refers to the individual's own marrow.

18. What is the most appropriate intervention for the parents of a 6-year-old girl with precocious puberty? a. Advise the parents to consider birth control for their daughter. b. Explain the importance of having the child foster relationships with same-age peers. c. Assure the child's parents that there is no increased risk for sexual abuse because of her appearance. d. Counsel parents that there is no treatment currently available for this disorder.

ANS: B Despite the child's appearance, the child needs to be treated according to her chronologic age and to interact with children in the same age-group. An expected outcome is that the child will adjust socially by exhibiting age-appropriate behaviors and social interactions. Advising the parents of a 6 year old to put their daughter on birth control is not appropriate and will not reverse the effects of precocious puberty. Parents need to be aware that there is an increased risk of sexual abuse for a child with precocious puberty. Treatment for precocious puberty is the administration of gonadotropin-releasing hormone blocker, which slows or reverses the development of secondary sexual characteristics and slows rapid growth and bone aging

29. An adolescent is being seen in the clinic for evaluation of acromegaly. The nurse understands that which event occurs with acromegaly? a. There is a lack of growth hormone (GH) being produced. b. There is excess GH after closure of the epiphyseal plates. c. There is an excess of GH before the closure of the epiphyseal plates. d. There is a lack of thyroid hormone being produced.

ANS: B Excess GH after closure of the epiphyseal plates results in acromegaly. A lack of growth hormone results in delayed growth or even dwarfism. Gigantism occurs when there is hypersecretion of GH before the closure of the epiphyseal plates. Cretinism is associated with hypothyroidism.

10. Exophthalmos may occur in children with what diagnosis? a. Hypothyroidism b. Hyperthyroidism c. Hypoparathyroidism d. Hyperparathyroidism

ANS: B Exophthalmos (protruding eyeballs) is a clinical manifestation of hyperthyroidism. Hypothyroidism, hypoparathyroidism, and hyperparathyroidism are not associated with exophthalmos.

11. A 4-month-old infant diagnosed with gastroesophageal reflux disease (GERD) is thriving without other complications. What should the nurse suggest to minimize reflux? a. Place in Trendelenburg position after eating. b. Thicken formula with rice cereal. c. Give continuous nasogastric tube feedings. d. Give larger, less frequent feedings.

ANS: B Giving small frequent feedings of formula combined with 1 teaspoon to 1 tablespoon of rice cereal per ounce of formula has been recommended. Milk thickening agents have been shown to decrease the number of episodes of vomiting and increase the caloric density of the formula. This may benefit infants who are underweight as a result of GERD. Placing the child in Trendelenburg position would increase the reflux. Continuous nasogastric feedings are reserved for infants with severe reflux and failure to thrive. Smaller, more frequent feedings are recommended in reflux.

6. Which is often administered to prevent or control hemorrhage in a child diagnosed with cancer? a. Nitrosoureas b. Platelets c. Whole blood d. Corticosteroids

ANS: B Most bleeding episodes can be prevented or controlled with the administration of platelet concentrate or platelet-rich plasma. Nitrosoureas, whole blood, and corticosteroids would not prevent or control hemorrhage.

24. The nurse is caring for an 11-year-old boy who has recently been diagnosed with diabetes. What should be included in the teaching plan for daily injections? a. The parents do not need to learn the procedure. b. He is old enough to give most of his own injections. c. Self-injections will be possible when he is closer to adolescence. d. He can learn about self-injections when he is able to reach all injection sites.

ANS: B School-age children are able to give their own injections. Parents should participate in learning and giving the insulin injections. He is already old enough to administer his own insulin. The child is able to use thighs, abdomen, part of the hip, and arm. Assistance can be obtained if other sites are used.

14. A 5 year old is being prepared for surgery to remove a brain tumor. Nursing actions should be based on which statement? a. Removal of tumor will stop the various symptoms. b. Usually the postoperative dressing covers the entire scalp. c. He is not old enough to be concerned about his head being shaved. d. He is not old enough to Comprehension the significance of the brain.

ANS: B The child should be told what he will look and feel like after surgery. This includes the size of the dressing. The nurse can demonstrate on a doll the expected size and shape of the dressing. Some of the symptoms may be alleviated by the removal of the tumor, but postsurgical headaches and cerebellar symptoms such as ataxia may be aggravated. Children should be prepared for the loss of their hair, and it should be removed in a sensitive, positive manner if the child is awake. Children at this age have poorly defined body boundaries and little knowledge of internal organs. Intrusive experiences are frightening, especially those that disrupt the integrity of the skin.

35. A nurse is reviewing the laboratory results on a school-age child diagnosed with hypoparathyroidism. Which results are consistent with this condition? a. Decreased serum phosphorus b. Decreased serum calcium c. Increased serum glucose d. Decreased serum cortisol

ANS: B The diagnosis of hypoparathyroidism is made on the basis of clinical manifestations associated with decreased serum calcium and increased serum phosphorus. Decreased serum phosphorus would be seen in hyperparathyroidism, elevated glucose in diabetes, and decreased serum cortisol in adrenocortical insufficiency (Addison's disease).

2. A boy with leukemia screams whenever he needs to be turned or moved. Which is the most probable cause of this pain? a. Edema b. Bone involvement c. Petechial hemorrhages d. Changes within the muscles

ANS: B The invasion of the bone marrow with leukemic cells gradually causes weakening of the bone and a tendency toward fractures. As leukemic cells invade the periosteum, increasing pressure causes severe pain. Edema, petechial hemorrhages, and changes within the muscles would not cause severe pain.

15. A neonate born with ambiguous genitalia is diagnosed with congenital adrenogenital hyperplasia. Therapeutic management includes administration of which medication? a. Vitamin D b. Cortisone c. Stool softeners d. Calcium carbonate

ANS: B The most common biochemical defect with congenital adrenal hyperplasia is partial or complete 21-hydroxylase deficiency. With complete deficiency, insufficient amounts of aldosterone and cortisol are produced, so circulatory collapse occurs without immediate replacement. Vitamin D, stool softeners, and calcium carbonate have no role in the therapy of adrenogenital hyperplasia.

6. Diabetes insipidus is a disorder of which organ? a. Anterior pituitary b. Posterior pituitary c. Adrenal cortex d. Adrenal medulla

ANS: B The principal disorder of posterior pituitary hypofunction is diabetes insipidus. The anterior pituitary produces hormones such as growth hormone, thyroid-stimulating hormone, adrenocorticotropic hormone, gonadotropin, prolactin, and melanocyte-stimulating hormone. The adrenal cortex produces aldosterone, sex hormones, and glucocorticoids. The adrenal medulla produces catecholamines.

3. An infant is brought to the emergency department with poor skin turgor, sunken fontanel, lethargy, and tachycardia. This is suggestive of which condition? a. Overhydration b. Dehydration c. Sodium excess d. Calcium excess

ANS: B These clinical manifestations indicate dehydration. Symptoms of overhydration are edema and weight gain. Regardless of extracellular sodium levels, total body sodium is usually depleted in dehydration. Symptoms of hypocalcemia are a result of neuromuscular irritability and manifest as jitteriness, tetany, tremors, and muscle twitching.

Which should the nurse expect to find in the cerebral spinal fluid (CSF) results of a child with Guillain-Barr syndrome (GBS)? (Select all that apply.) a. Decreased protein concentration b. Normal glucose c. Fewer than 10 white blood cells (WBCs/mm3) d. Elevated red blood cell (RBC) count

ANS: B, C Diagnosis of GBS is based on clinical manifestations, CSF analysis, and EMG findings. CSF analysis reveals an abnormally elevated protein concentration, normal glucose, and fewer than 10 WBCs/mm3. CSF fluid should not contain RBCs.

1. The nurse, caring for an infant whose cleft lip was repaired, should include which interventions into the infant's postoperative plan of care? (Select all that apply.) a. Postural drainage b. Petroleum jelly to the suture line c. Elbow restraints d. Supine and side-lying positions e. Mouth irrigations

ANS: B, C Apply petroleum jelly to the operative site for several days after surgery. Elbows are restrained to prevent the child from accessing the operative site for up to 7 to 10 days. The child should be positioned on back or side or in an infant seat. Postural drainage is not indicated. This would increase the pressure on the operative site when the child is placed in different positions. Mouth irrigations would not be indicated.

The nurse is planning care for a school-age child admitted to the hospital with hemophilia. Which interventions should the nurse plan to implement for this child? (Select all that apply.) a. Finger sticks for blood work instead of venipunctures b. Avoidance of IM injections c. Acetaminophen (Tylenol) for mild pain control d. Soft tooth brush for dental hygiene e. Administration of packed red blood cells

ANS: B, C, D Nurses should take special precautions when caring for a child with hemophilia to prevent the use of procedures that may cause bleeding, such as IM injections. The subcutaneous route is substituted for IM injections whenever possible. Venipunctures for blood samples are usually preferred for these children. There is usually less bleeding after the venipuncture than after finger or heel punctures. Neither aspirin nor any aspirin-containing compound should be used. Acetaminophen is a suitable aspirin substitute, especially for controlling mild pain. A soft toothbrush is recommended for dental hygiene to prevent bleeding from the gums. Packed red blood cells are not administered. The primary therapy for hemophilia is replacement of the missing clotting factor. The products available are factor VIII concentrates.

17. While working with the pregnant woman in her first trimester, the nurse is aware that chorionic villus sampling (CVS) can be performed during pregnancy at: A. 4 weeks. B. 8 weeks. C. 10 weeks. D. 14 weeks.

ANS: C CVS can be performed in the first or second trimester, ideally between 10 and 13 weeks of gestation. During this procedure, a small piece of tissue is removed from the fetal portion of the placenta. If performed after 9 completed weeks of gestation, the risk of limb reduction is no greater than in the general population.

1. Maternal hypotension is a potential side effect of regional anesthesia and analgesia. What nursing interventions could you use to raise the patient's blood pressure? (Select all that apply.) a. Place the woman in a supine position. b. Place the woman in a lateral position. c. Increase intravenous (IV) fluids. d. Administer oxygen. e. Perform a vaginal examination.

ANS: B, C, D Nursing interventions for maternal hypotension arising from analgesia or anesthesia include turning the woman to a lateral position, increasing IV fluids, administering oxygen via face mask, elevating the woman's legs, notifying the physician, administering an IV vasopressor, and monitoring the maternal and fetal status at least every 5 minutes until these are stable. Placing the patient in a supine position would cause venous compression, thereby limiting blood flow to and oxygenation of the placenta and fetus. A sterile vaginal examination has no bearing on maternal blood pressure.

A nurse plans therapeutic play time for a hospitalized child. Which are the benefits of therapeutic play (select all that apply)? a. Serves as method to assist disturbed children b. Allows the child to express feelings c. The nurse can gain insight into the childs feelings d. The child can deal with concerns and feelings e. Gives the child a structured play environment

ANS: B, C, D Therapeutic play is an effective, nondirective modality for helping children deal with their concerns and fears, and at the same time, it often helps the nurse gain insights into childrens needs and feelings. Play and other expressive activities provide one of the best opportunities for encouraging emotional expression, including the safe release of anger and hostility. Nondirective play that allows children freedom for expression can be tremendously therapeutic. Play therapy is a structured therapy that helps disturbed children. It should not be confused with therapeutic play.

A nurse is interviewing the parents of a toddler about use of complementary or alternative medical practices. The parents share several practices they use in their household. Which should the nurse document as complementary or alternative medical practices (select all that apply)? a. Use of acetaminophen (Tylenol) for fever b. Administration of chamomile tea at bedtime c. Hypnotherapy for relief of pain d. Acupressure to relieve headaches e. Cool mist vaporizer at the bedside for stuffiness

ANS: B, C, D When conducting an assessment, the nurse should inquire about the use of complementary or alternative medical practices. Administration of chamomile tea at bedtime, hypnotherapy for relief of pain, and acupressure to relieve headaches are complementary or alternative medical practices. Using Tylenol for fever relief and a cool mist vaporizer at the bedside to reduce stuffiness are not considered complementary or alternative medical practices.

Late in pregnancy, the womans breasts should be assessed by the nurse to identify any potential concerns related to breastfeeding. Some nipple conditions make it necessary to provide intervention before birth. These include(Select all that apply): a. Everted nipples b. Flat nipples c. Inverted nipples d. Nipples that contract when compressed e. Cracked nipples

ANS: B, C, D Flat nipples appear soft, like the areola, and do not stand erect unless stimulated by rolling them between the fingers. Inverted nipples are retracted into the breast tissue. These nipples appear normal; however, they will draw inward when the areola is compressed by the infants mouth. Dome-shaped devices known as breast shells can be worn during the last weeks of pregnancy and between feedings after birth. The shells are placed inside the bra with the opening over the nipple. The shells exert slight pressure against the areola to help the nipples protrude. The helpfulness of breast shells is debated. A breast pump can be used to draw the nipples out before feedings after delivery. Everted nipples protrude and are normal. No intervention will be required. Cracked, blistered, and bleeding nipples occur after breastfeeding has been initiated and are the result of improper latch. The infant should be repositioned during feeding. Application of colostrum and breast milk after feedings will aid in healing. PTS: 1 DIF: Cognitive Level: Application REF: 639 OBJ: Nursing Process: Assessment MSC: Client Needs: Health Promotion and Maintenance

4. A nurse teaches parents that team play is important for school-age children. Which abilities can children develop by experiencing team play? (Select all that apply.) a. Achieve personal goals over group goals. b. Learn complex rules. c. Experience competition. d. Learn about division of labor.

ANS: B, C, D Team play helps stimulate cognitive growth because children are called on to learn many complex rules, make judgments about those rules, plan strategies, and assess the strengths and weaknesses of members of their own team and members of the opposing team. Team play can also contribute to children's social, intellectual, and skill growth. Children work hard to develop the skills needed to become team members, to improve their contribution to the group, and to anticipate the consequences of their behavior for the group. Team play teaches children to modify or exchange personal goals for goals of the group; it also teaches them that division of labor is an effective strategy for attaining a goal.

3. Autoimmune disorders often occur during pregnancy because a large percentage of women with an autoimmune disorder are of childbearing age. Identify all disorders that fall into the category of collagen vascular disease. A. Multiple sclerosis B. Systemic lupus erythematosus C. Antiphospholipid syndrome D. Rheumatoid arthritis E. Myasthenia gravis

ANS: B, C, D, E Multiple sclerosis is not an autoimmune disorder. This patchy demyelinization of the spinal cord may be a viral disorder. Autoimmune disorders (collagen vascular disease) make up a large group of conditions that disrupt the function of the immune system of the body. They include those listed, as well as systemic sclerosis.

2. Which statements regarding hepatitis B are correct? (Select all that apply.) a. Hepatitis B cannot exist in a carrier state. b. Hepatitis B can be prevented by hepatitis B virus vaccine. c. Hepatitis B can be transferred to an infant of a breastfeeding mother. d. The onset of hepatitis B is insidious. e. Immunity to hepatitis B occurs after one attack.

ANS: B, C, D, E The vaccine elicits the formation of an antibody to the hepatitis B surface antigen, which is protective against hepatitis B. Hepatitis B can be transferred to an infant of a breastfeeding mother, especially if the mother's nipples are cracked. The onset of hepatitis B is insidious. Immunity develops after one exposure to hepatitis B. Hepatitis B can exist in a carrier state.

39. The drug of choice for treatment of gonorrhea is: A. penicillin G. B. tetracycline. C. ceftriaxone. D. acyclovir.

ANS: C Ceftriaxone is effective for treatment of all gonococcal infections. Penicillin is used to treat syphilis. Tetracycline is prescribed for chlamydial infections. Acyclovir is used to treat herpes genitalis.

Which should the nurse teach about prevention of sickle cell crises to parents of a preschool child with sickle cell disease? (Select all that apply.) a. Limit fluids at bedtime. b. Notify the health care provider if a fever of 38.5 C (101.3 F) or greater occurs. c. Give penicillin as prescribed. d. Use ice packs to decrease the discomfort of vasoocclusive pain in the legs. e. Notify the health care provider if your child begins to develop symptoms of a cold.

ANS: B, C, E The most important issues to teach the family of a child with sickle cell anemia are to (1) seek early intervention for problems, such as a fever of 38.5 C (101.3 F) or greater; (2) give penicillin as ordered; (3) recognize signs and symptoms of splenic sequestration, as well as respiratory problems that can lead to hypoxia; and (4) treat the child normally. The nurse emphasizes the importance of adequate hydration to prevent sickling and to delay the adhesionstasisthrombosisischemia cycle. It is not sufficient to advise parents to force fluids or encourage drinking. They need specific instructions on how many daily glasses or bottles of fluid are required. Many foods are also a source of fluid, particularly soups, flavored ice pops, ice cream, sherbet, gelatin, and puddings. Increased fluids combined with impaired kidney function result in the problem of enuresis. Parents who are unaware of this fact frequently use the usual measures to discourage bedwetting, such as limiting fluids at night. Enuresis is treated as a complication of the disease, such as joint pain or some other symptom, to alleviate parental pressure on the child. Ice should not be used during a vasoocclusive pain crisis because it vasoconstricts and impairs circulation even more.

A clinic nurse is conducting a staff in-service for other clinic nurses about signs and symptoms of a rhabdomyosarcoma tumor. Which should be included in the teaching session? (Select all that apply.) a. Bone fractures b. Abdominal mass c. Sore throat and ear pain d. Headache e. Ecchymosis of conjunctiva

ANS: B, C, E The initial signs and symptoms of rhabdomyosarcoma tumors are related to the site of the tumor and compression of adjacent organs. Some tumor locations, such as the orbit, manifest early in the course of the illness. Other tumors, such as those of the retroperitoneal area, only produce symptoms when they are relatively large and compress adjacent organs. Unfortunately, many of the signs and symptoms attributable to rhabdomyosarcoma are vague and frequently suggest a common childhood illness, such as "earache" or "runny nose." An abdominal mass, sore throat and ear pain, and ecchymosis of conjunctiva are signs of a rhabdomyosarcoma tumor. Bone fractures would be seen in osteosarcoma and a headache is a sign of a brain tumor.

2. Which children admitted to the pediatric unit would the nurse monitor closely for development of syndrome of inappropriate antidiuretic hormone (SIADH)? (Select all that apply.) a. A newly diagnosed preschooler with type 1 diabetes. b. A school-age child returning from surgery for removal of a brain tumor. c. An infant with suspected meningitis. d. An adolescent with blunt abdominal trauma following a car accident. e. A school-age child with head trauma.

ANS: B, C, E The disorder that results from hypersecretion of ADH from the posterior pituitary hormone. Childhood SIADH usually is caused by disorders affecting the central nervous system, such as infections (meningitis), head trauma, and brain tumors. Type 1 diabetes and blunt abdominal trauma are not likely to cause SIADH since do not affect secretion of this hormone.

3. A clinic nurse is conducting a staff in-service for other clinic nurses about signs and symptoms of a rhabdomyosarcoma tumor. Which should be included in the teaching session? (Select all that apply.) a. Bone fractures b. Abdominal mass c. Sore throat and ear pain d. Headache e. Ecchymosis of conjunctiva

ANS: B, C, E The initial signs and symptoms of rhabdomyosarcoma tumors are related to the site of the tumor and compression of adjacent organs. Some tumor locations, such as the orbit, manifest early in the course of the illness. Other tumors, such as those of the retroperitoneal area, only produce symptoms when they are relatively large and compress adjacent organs. Unfortunately, many of the signs and symptoms attributable to rhabdomyosarcoma are vague and frequently suggest a common childhood illness, such as "earache" or "runny nose." An abdominal mass, sore throat and ear pain, and ecchymosis of conjunctiva are signs of a rhabdomyosarcoma tumor. Bone fractures would be seen in osteosarcoma, and a headache is a sign of a brain tumor.

6. A mother who intended to breastfeed has given birth to an infant with a cleft palate. Which nursing interventions should be included in the plan of care? (Select all that apply.) a. Giving medication to suppress lactation. b. Encouraging and helping mother to breastfeed. c. Teaching mother to feed breast milk by gavage. d. Recommending use of a breast pump to maintain lactation until infant can suck.

ANS: B, D The mother who wishes to breastfeed may need encouragement and support because the defect does present some logistical issues. The nipple must be positioned and stabilized well back in the infant's oral cavity so that the tongue action facilitates milk expression. The suction required to stimulate milk, absent initially, may be useful before nursing to stimulate the let-down reflex. Because breastfeeding is an option, if the mother wishes to breastfeed, medications should not be given to suppress lactation. Because breastfeeding can usually be accomplished, gavage feedings are not indicated.

Parents of a school-age child with hemophilia ask the nurse, Which sports are recommended for children with hemophilia? Which sports should the nurse recommend? (Select all that apply.) a. Soccer b. Swimming c. Basketball d. Golf e. Bowling

ANS: B, D, E Because almost all persons with hemophilia are boys, the physical limitations in regard to active sports may be a difficult adjustment, and activity restrictions must be tempered with sensitivity to the childs emotional and physical needs. Use of protective equipment, such as padding and helmets, is particularly important, and noncontact sports, especially swimming, walking, jogging, tennis, golf, fishing, and bowling, are encouraged. Contact sport such as soccer and basketball are not recommended.

40.A school-age child has been admitted to the hospital with an exacerbation of nephrotic syndrome. Which clinical manifestations should the nurse expect to assess (Select all that apply)? a. Weight loss b. Facial edema c. Cloudy, smoky brown-colored urine d. Fatigue e. Frothy-appearing urine

ANS: B, D, E A child with nephrotic syndrome will present with facial edema, fatigue, and frothy-appearing urine (proteinuria). Weight gain, not loss, is expected because of the fluid retention. Cloudy, smoky brown-colored urine is seen with acute glomerulonephritis but not with nephrotic syndrome because there is no gross hematuria associated with nephrotic syndrome.

1. Which expected appearance will the nurse explain to parents of an infant returning from surgery after an enucleation was performed to treat retinoblastoma? (Select all that apply.) a. A lot of drainage will come from the affected socket. b. The face may be edematous or ecchymotic. c. The eyelids will be sutured shut for the first week. d. There will be an eye pad dressing taped over the surgical site. e. The implanted sphere is covered with conjunctiva and resembles the lining of the mouth.

ANS: B, D, E After enucleation surgery, the parents are prepared for the child's facial appearance. An eye patch is in place, and the child's face may be edematous or ecchymotic. Parents often fear seeing the surgical site because they imagine a cavity in the skull. A surgically implanted sphere maintains the shape of the eyeball, and the implant is covered with conjunctiva. When the eyelids are open, the exposed area resembles the mucosal lining of the mouth. The dressing, consisting of an eye pad taped over the surgical site, is changed daily. The wound itself is clean and has little or no drainage. So expecting a lot of drainage is not accurate to tell parents. The eyelids are not sutured shut after enucleation surgery.

4. The nurse is preparing to care for an infant returning from pyloromyotomy surgery. Which prescribed orders should the nurse anticipate implementing? (Select all that apply.) a. Nothing by mouth for 24 hours b. Administration of analgesics for pain c. Ice bag to the incisional area d. Intravenous (IV) fluids continued until tolerating fluids by mouth e. Clear liquids as the first feeding

ANS: B, D, E Feedings are usually instituted soon after a pyloromyotomy surgery, beginning with clear liquids and advancing to formula or breast milk as tolerated. IV fluids are administered until the infant is taking and retaining adequate amounts by mouth. Appropriate analgesics should be given round the clock because pain is continuous. Ice should not be applied to the incisional area as it vasoconstricts and would reduce circulation to the incisional area and impair healing.

11. To prevent gastrointestinal upset, patients should be instructed to take iron supplements: A. on a full stomach. B. at bedtime. C. after eating a meal. D. with milk.

ANS: BPatients should be instructed to take iron supplements at bedtime. Iron supplements are best absorbed if they are taken when the stomach is empty. Bran, tea, coffee, milk, and eggs may reduce absorption. Iron can be taken at bedtime if abdominal discomfort occurs when it is taken between meals.

13. A pregnant woman at 25 weeks' gestation tells the nurse that she dropped a pan last week and her baby jumped at the noise. Which response by the nurse is most accurate? A. "That must have been a coincidence; babies can't respond like that." B. "The fetus is demonstrating the aural reflex." C. "Babies respond to sound starting at about 24 weeks of gestation." D. "Let me know if it happens again; we need to report that to your midwife."

ANS: C "Babies respond to sound starting at about 24 weeks of gestation" is an accurate statement. "That must have been a coincidence; babies can't respond like that" is inaccurate. Acoustic stimulations can evoke a fetal heart rate response. There is no such thing as an aural reflex. The statement, "Let me know if it happens again; we need to report that to your midwife" is not appropriate; it gives the impression that something is wrong.

39. Which nursing assessment indicates that a woman who is in second-stage labor is almost ready to give birth? a. The fetal head is felt at 0 station during vaginal examination. b. Bloody mucus discharge increases. c. The vulva bulges and encircles the fetal head. d. The membranes rupture during a contraction.

ANS: C A bulging vulva that encircles the fetal head describes crowning, which occurs shortly before birth. Birth of the head occurs when the station is +4. A 0 station indicates engagement. Bloody show occurs throughout the labor process and is not an indication of an imminent birth. Rupture of membranes can occur at any time during the labor process and does not indicate an imminent birth.

24. With regard to the development of the respiratory system, maternity nurses should understand that: A. the respiratory system does not begin developing until after the embryonic stage. B. the infant's lungs are considered mature when the lecithin/sphingomyelin [L/S] ratio is 1:1, at about 32 weeks. C. maternal hypertension can reduce maternal-placental blood flow, accelerating lung maturity. D. fetal respiratory movements are not visible on ultrasound scans until at least 16 weeks.

ANS: C A reduction in placental blood flow stresses the fetus, increases blood levels of corticosteroids, and accelerates lung maturity. Development of the respiratory system begins during the embryonic phase and continues into childhood. The infant's lungs are mature when the L/S ratio is 2:1, at about 35 weeks. Lung movements have been seen on ultrasound scans at 11 weeks.

Which is caused by a virus that primarily infects a specific subset of T lymphocytes, the CD4+ T cells? a. Wiskott-Aldrich syndrome b. Idiopathic thrombocytopenic purpura c. Acquired immunodeficiency syndrome (AIDS) d. Severe combined immunodeficiency disease

ANS: C AIDS is caused by the human immunodeficiency virus (HIV), which primarily attacks the CD4+ T cells. Wiskott-Aldrich syndrome, idiopathic thrombocytopenic purpura, and severe combined immunodeficiency disease are not viral illnesses.

14. Injectable progestins (DMPA, Depo-Provera) are a good contraceptive choice for women who: A. want menstrual regularity and predictability. B. have a history of thrombotic problems or breast cancer. C. have difficulty remembering to take oral contraceptives daily. D. are homeless or mobile and rarely receive health care.

ANS: C Advantages of DMPA include a contraceptive effectiveness comparable to that of combined oral contraceptives with the requirement of only four injections a year. Disadvantages of injectable progestins are prolonged amenorrhea and uterine bleeding. Use of injectable progestin carries an increased risk of venous thrombosis and thromboembolism. To be effective, DMPA injections must be administered every 11 to 13 weeks. Access to health care is necessary to prevent pregnancy or potential complications.

38. Which patient is most at risk for fibroadenoma of the breast? A. A 38-year-old woman B. A 50-year-old woman C. A 16-year-old girl D. A 27-year-old woman

ANS: C Although it may occur at any age, fibroadenoma is most common in the teenage years. Ductal ectasia and intraductal papilloma become more common as a woman approaches menopause. Fibrocystic breast changes are more common during the reproductive years.

7. When evaluating a patient whose primary complaint is amenorrhea, the nurse must be aware that lack of menstruation is most often the result of: A. stress. B. excessive exercise. C. pregnancy. D. eating disorders.

ANS: C Amenorrhea, or the absence of menstrual flow, is most often a result of pregnancy. Although stress, excessive exercise, and eating disorders all may be contributing factors, none is the most common factor associated with amenorrhea.

14. A woman arrives at the clinic for her annual examination. She tells the nurse that she thinks she has a vaginal infection and has been using an over-the-counter cream for the past 2 days to treat it. The nurse's initial response should be to: A. inform the woman that vaginal creams may interfere with the Papanicolaou (Pap) test for which she is scheduled. B. reassure the woman that using vaginal cream is not a problem for the examination. C. ask the woman to describe the symptoms that indicate to her that she has a vaginal infection. D. ask the woman to reschedule the appointment for the examination.

ANS: C An important element of the history and physical examination is the patient's description of any symptoms she may be experiencing. Although vaginal creams may interfere with the Pap test, the best response is for the nurse to inquire about the symptoms the patient is experiencing. Women should not douche, use vaginal medications, or have sexual intercourse for 24 to 48 hours before obtaining a Pap test. Although the woman may need to reschedule a visit for her Pap test, her current symptoms should still be addressed.

3. A laboring woman received an opioid agonist (meperidine) intravenously 90 minutes before she gave birth. Which medication should be available to reduce the postnatal effects of Demerol on the neonate? a. Fentanyl (Sublimaze) b. Promethazine (Phenergan) c. Naloxone (Narcan) d. Nalbuphine (Nubain)

ANS: C An opioid antagonist can be given to the newborn as one part of the treatment for neonatal narcosis, which is a state of central nervous system (CNS) depression in the newborn produced by an opioid. Opioid antagonists such as naloxone (Narcan) can promptly reverse the CNS depressant effects, especially respiratory depression. Fentanyl, promethazine, and nalbuphine do not act as opioid antagonists to reduce the postnatal effects of Demerol on the neonate. Although meperidine (Demerol) is a low-cost medication and readily available, the use of Demerol in labor has been controversial because of its effects on the neonate.

Which child should the nurse document as being anemic? a. 7-year-old child with a hemoglobin of 11.5 g/dl b. 3-year-old child with a hemoglobin of 12 g/dl c. 14-year-old child with a hemoglobin of 10 g/dl d. 1-year-old child with a hemoglobin of 13 g/dl

ANS: C Anemia is a condition in which the number of red blood cells, or hemoglobin concentration, is reduced below the normal values for age. Anemia is defined as a hemoglobin level below 10 or 11 g/dl. The child with a hemoglobin of 10 g/dl would be considered anemic. The normal hemoglobin for a child after 2 years of age is 11.5 to 15.5 g/dl.

2. The nurse is performing an initial assessment of a client in labor. What is the appropriate terminology for the relationship of the fetal body parts to one another? a. Lie b. Presentation c. Attitude d. Position

ANS: C Attitude is the relationship of the fetal body parts to one another. Lie is the relationship of the long axis (spine) of the fetus to the long axis (spine) of the mother. Presentation refers to the part of the fetus that enters the pelvic inlet first and leads through the birth canal during labor at term. Position is the relationship of the presenting part of the fetus to the four quadrants of the mother's pelvis.

27. The priority nursing care associated with an oxytocin (Pitocin) infusion is: a. measuring urinary output. b. increasing infusion rate every 30 minutes. c. monitoring uterine response. d. evaluating cervical dilation.

ANS: C Because of the risk of hyperstimulation, which could result in decreased placental perfusion and uterine rupture, the nurse's priority intervention is monitoring uterine response. Monitoring urinary output is also important; however, it is not the top priority during the administration of Pitocin. The infusion rate may be increased after proper assessment that it is an appropriate interval to do so. Monitoring labor progression is the standard of care for all labor patients.

The nurse is planning activity for a 4-year-old child with anemia. Which activity should the nurse plan for this child? a. Game of hide and seek in the childrens outdoor play area b. Participation in dance activities in the playroom c. Puppet play in the childs room d. A walk down to the hospital lobby

ANS: C Because the basic pathologic process in anemia is a decrease in oxygen-carrying capacity, an important nursing responsibility is to assess the childs energy level and minimize excess demands. The childs level of tolerance for activities of daily living and play is assessed, and adjustments are made to allow as much self-care as possible without undue exertion. Puppet play in the childs room would not be overly tiring. Hide and seek, dancing, and walking to the lobby would not conserve the anemic childs energy.

An adolescent will receive a bone marrow transplant (BMT). The nurse should explain that the bone marrow will be administered by which route? a. Bone grafting b. Bone marrow injection c. IV infusion d. Intra-abdominal infusion

ANS: C Bone marrow from a donor is infused intravenously, and the transfused stem cells will repopulate the marrow. Because the stem cells migrate to the recipients marrow when given intravenously, this is the method of administration.

15. As relates to the use of tocolytic therapy to suppress uterine activity, nurses should be aware that: a. the drugs can be given efficaciously up to the designated beginning of term at 37 weeks. b. there are no important maternal (as opposed to fetal) contraindications. c. its most important function is to afford the opportunity to administer antenatal glucocorticoids. d. if the patient develops pulmonary edema while receiving tocolytics, intravenous (IV) fluids should be given.

ANS: C Buying time for antenatal glucocorticoids to accelerate fetal lung development may be the best reason to use tocolytics. Once the pregnancy has reached 34 weeks, the risks of tocolytic therapy outweigh the benefits. There are important maternal contraindications to tocolytic therapy. Tocolytic-induced edema can be caused by IV fluids.

During the first 4 days of hospitalization, Eric, age 18 months, cried inconsolably when his parents left him, and he refused the staffs attention. Now the nurse observes that Eric appears to be settled in and unconcerned about seeing his parents. The nurse should interpret this as which of the following? a. He has successfully adjusted to the hospital environment. b. He has transferred his trust to the nursing staff. c. He may be experiencing detachment, which is the third stage of separation anxiety. d. Because he is at home in the hospital now, seeing his mother frequently will only start the cycle again.

ANS: C Detachment is a behavioral manifestation of separation anxiety. Superficially it appears that the child has adjusted to the loss and transferred his trust to the nursing staff. Detachment is a sign of resignation, not contentment. Parents should be encouraged to be with their child. If parents restrict visits, they may begin a pattern of misunderstanding the childs cues and not meeting his needs.

28. As a girl progresses through development, she may be at risk for a number of age-related conditions. While preparing a 21-year-old patient for her first adult physical examination and Papanicolaou (Pap) test, the nurse is aware of excessiveness shyness. The young woman states that she will not remove her bra because, "There is something wrong with my breasts; one is way bigger." What is the best response by the nurse in this situation? A. "Please reschedule your appointment until you are more prepared." B. "It is okay; the provider will not do a breast examination." C. "I will explain normal growth and breast development to you." D. "That is unfortunate; this must be very stressful for you."

ANS: C During adolescence, one breast may grow faster than the other. Discussion regarding this aspect of growth and development with the patient will reassure her that there may be nothing wrong with her breasts. Young women usually enter the health system for screening (Pap tests begin at age 21 or 3 years after first sexual activity). Situations such as these can produce great stress for the young woman, and the nurse and health care provider should treat her carefully. Asking her to reschedule would likely result in the patient's not returning for her appointment at all. A breast examination at her age is part of the complete physical examination. Young women should be taught about normal breast development and begin doing breast self-examinations. Although the last response shows empathy on the part of the nurse and acknowledges the patient's stress, it does not correct the patient's deficient knowledge related to normal growth and development.

23. Which nursing assessment indicates that a woman who is in second-stage labor is almost ready to give birth? a. Fetal head is felt at 0 station during the vaginal examination. b. Bloody mucous discharge increases. c. Vulva bulges and encircles the fetal head. d. Membranes rupture during a contraction.

ANS: C During the active pushing (descent) phase, the woman has strong urges to bear down as the presenting part of the fetus descends and presses on the stretch receptors of the pelvic floor. The vulva stretches and begins to bulge, encircling the fetal head. Birth of the head occurs when the station is +4. A 0 station indicates engagement. Bloody show occurs throughout the labor process and is not an indication of an imminent birth. Rupture of membranes can occur at any time during the labor process and does not indicate an imminent birth.

10. The nurse knows that the second stage of labor, the descent phase, has begun when: a. the amniotic membranes rupture. b. the cervix cannot be felt during a vaginal examination. c. the woman experiences a strong urge to bear down. d. the presenting part is below the ischial spines.

ANS: C During the descent phase of the second stage of labor, the woman may experience an increase in the urge to bear down. Rupture of membranes has no significance in determining the stage of labor. The second stage of labor begins with full cervical dilation. Many women may have an urge to bear down when the presenting part is below the level of the ischial spines. This can occur during the first stage of labor, as early as 5-cm dilation.

23. Some pregnant patients may complain of changes in their voice and impaired hearing. The nurse can tell these patients that these are common reactions to: A. a decreased estrogen level. B. displacement of the diaphragm, resulting in thoracic breathing. C. congestion and swelling, which occur because the upper respiratory tract has become more vascular. D. increased blood volume.

ANS: C Estrogen levels increase, causing the upper respiratory tract to become more vascular producing swelling and congestion in the nose and ears leading to voice changes and impaired hearing. The diaphragm is displaced, and the volume of blood is increased. However, the main concern is increased estrogen levels.

3. When assessing the fetus using Leopold's maneuvers, the nurse feels a round, firm, and movable fetal part in the fundal portion of the uterus and a long, smooth surface in the mother's right side close to midline. What is the position of the fetus? a. ROA b. LSP c. RSA d. LOA

ANS: C Fetal position is denoted with a three-letter abbreviation. The first letter indicates the presenting part in either the right or the left side of the maternal pelvis. The second letter indicates the anatomic presenting part of the fetus. The third letter stands for the location of the presenting part in relationship to the anterior, posterior, or transverse portion of the maternal pelvis. Palpation of a round, firm fetal part in the fundal portion of the uterus would be the fetal head, indicating that the fetus is in a breech position with the sacrum as the presenting part in the maternal pelvis. Palpation of the fetal spine along the mother's right side denotes the location of the presenting part in the mother's pelvis. The ability to palpate the fetal spine indicates that the fetus is anteriorly positioned in the maternal pelvis. This fetus is anteriorly positioned in the right side of the maternal pelvis with the sacrum as the presenting part. RSA is the correct three-letter abbreviation to indicate this fetal position. ROA denotes a fetus that is anteriorly positioned in the right side of the maternal pelvis with the occiput as the presenting part. LSP describes a fetus that is posteriorly positioned in the left side of the pelvis with the sacrum as the presenting part. A fetus that is LOA would be anteriorly positioned in the left side of the pelvis with the occiput as the presenting part.

Which is most descriptive of the pathophysiology of leukemia? a. Increased blood viscosity occurs. b. Thrombocytopenia (excessive destruction of platelets) occurs. c. Unrestricted proliferation of immature white blood cells (WBCs) occurs. d. First stage of coagulation process is abnormally stimulated.

ANS: C Leukemia is a group of malignant disorders of the bone marrow and lymphatic system. It is defined as an unrestricted proliferation of immature WBCs in the blood-forming tissues of the body. Increased blood viscosity may occur secondary to the increased number of WBCs. Thrombocytopenia may occur secondary to the overproduction of WBCs in the bone marrow. The coagulation process is unaffected by leukemia.

40. The nurse providing education regarding breast care should explain to the woman that fibrocystic changes in breasts are: A. a disease of the milk ducts and glands in the breasts. B. a premalignant disorder characterized by lumps found in the breast tissue. C. lumpiness with or without tenderness found in varying degrees in the breast tissue of healthy women during menstrual cycles. D. lumpiness accompanied by tenderness after menses.

ANS: C Fibrocystic changes are palpable thickenings in the breast usually associated with or without tenderness. The pain and tenderness fluctuate with the menstrual cycle. Fibrocystic changes are not premalignant changes.

2. Which meal would provide the most absorbable iron? A. Toasted cheese sandwich, celery sticks, tomato slices, and a grape drink B. Oatmeal, whole wheat toast, jelly, and low-fat milk C. Black bean soup, wheat crackers, orange sections, and prunes D. Red beans and rice, cornbread, mixed greens, and decaffeinated tea

ANS: C Food sources that are rich in iron include liver, meats, whole grain or enriched breads and cereals, deep green leafy vegetables, legumes, and dried fruits. In addition, the vitamin C in orange sections aids absorption. Dairy products and tea are not sources of iron.

12. A woman's position is an important component of the labor progress. Which guidance is important for the nurse to provide to the laboring client? a. The supine position, which is commonly used in the United States, increases blood flow. b. The laboring client positioned on her hands and knees ("all fours" position) is hard on the woman's back. c. Frequent changes in position help relieve fatigue and increase the comfort of the laboring client. d. In a sitting or squatting position, abdominal muscles of the laboring client will have to work harder.

ANS: C Frequent position changes relieve fatigue, increase comfort, and improve circulation. Blood flow can be compromised in the supine position; any upright position benefits cardiac output. The "all fours" position is used to relieve backache in certain situations. In a sitting or squatting position, the abdominal muscles work in greater harmony with uterine contractions.

27. In response to requests by the U.S. Public Health Service for new models of prenatal care, an innovative new approach to prenatal care known as centering pregnancy was developed. Which statement would accurately apply to the centering model of care? A. Group sessions begin with the first prenatal visit. B. At each visit, blood pressure, weight, and urine dipsticks are obtained by the nurse. C. Eight to twelve women are placed in gestational-age cohort groups. D. Outcomes are similar to those of traditional prenatal care.

ANS: C Gestational-age cohorts comprise the groups with approximately 8 to 12 women in each group. This group remains intact throughout the pregnancy. Individual follow-up visits are scheduled as needed. Group sessions begin at 12 to 16 weeks of gestation and end with an early after birth visit. Before group sessions the patient has an individual assessment, physical examination, and history. At the beginning of each group meeting, patients measure their own blood pressure, weight, and urine dips and enter these in their record. Fetal heart rate assessment and fundal height are obtained by the nurse. Results evaluating this approach have been very promising. In a study of adolescent patients, there was a decrease in low-birth-weight infants and an increase in breastfeeding rates.

27. To manage her diabetes appropriately and ensure a good fetal outcome, the pregnant woman with diabetes will need to alter her diet by: A. eating six small equal meals per day. B. reducing carbohydrates in her diet. C. eating her meals and snacks on a fixed schedule. D. increasing her consumption of protein.

ANS: C Having a fixed meal schedule will provide the woman and the fetus with a steadier blood sugar level, provide better balance with insulin administration, and help prevent complications. It is more important to have a fixed meal schedule than equal division of food intake. Approximately 45% of the food eaten should be in the form of carbohydrates.

13. Nurses with an understanding of cultural differences regarding likely reactions to pain may be better able to help patients. Nurses should know that _____ women may be stoic until late in labor, when they may become vocal and request pain relief. a. Chinese b. Arab or Middle Eastern c. Hispanic d. African-American

ANS: C Hispanic women may be stoic early and more vocal and ready for medications later. Chinese women may not show reactions to pain. Medical interventions must be offered more than once. Arab or Middle Eastern women may be vocal in response to labor pain from the start. They may prefer pain medications. African-American women may express pain openly; use of medications for pain is more likely to vary with the individual.

13. The nurse providing care for the antepartum woman should understand that contraction stress test (CST): A. sometimes uses vibroacoustic stimulation. B. is an invasive test; however, contractions are stimulated. C. is considered negative if no late decelerations are observed with the contractions. D. is more effective than nonstress test (NST) if the membranes have already been ruptured.

ANS: C No late decelerations are good news. Vibroacoustic stimulation is sometimes used with NST. CST is invasive if stimulation is by intravenous oxytocin but not if by nipple stimulation and is contraindicated if the membranes have ruptured.

28. The laboring woman who imagines her body opening to let the baby out is using a mental technique called: a. dissociation. b. effleurage. c. imagery. d. distraction.

ANS: C Imagery is a technique of visualizing images that will assist the woman in coping with labor. Dissociation helps the woman learn to relax all muscles except those that are working. Effleurage is self-massage. Distraction can be used in the early latent phase by having the woman engage in another activity.

The nurse is teaching parents of an infant about the causes of iron deficiency anemia. Which statement best describes iron deficiency anemia in infants? a. It is caused by depression of the hematopoietic system. b. It is easily diagnosed because of an infants emaciated appearance. c. Clinical manifestations are similar regardless of the cause of the anemia. d. Clinical manifestations result from a decreased intake of milk and the premature addition of solid foods.

ANS: C In iron deficiency anemia, the childs clinical appearance is a result of the anemia, not the underlying cause. Usually the hematopoietic system is not depressed in iron deficiency anemia. The bone marrow produces red cells that are smaller and contain less hemoglobin than normal red cells. Children who are iron deficient from drinking excessive quantities of milk are usually pale and overweight. They are receiving sufficient calories, but are deficient in essential nutrients. The clinical manifestations result from decreased intake of iron-fortified solid foods and an excessive intake of milk.

Which statement best describes b-thalassemia major (Cooley anemia)? a. All formed elements of the blood are depressed. b. Inadequate numbers of red blood cells are present. c. Increased incidence occurs in families of Mediterranean extraction. d. Increased incidence occurs in persons of West African descent.

ANS: C Individuals who live near the Mediterranean Sea and their descendants have the highest incidence of thalassemia. An overproduction of red cells occurs. Although numerous, the red cells are relatively unstable. Sickle cell disease is common in persons of West African descent.

41. The nurse thoroughly dries the infant immediately after birth primarily to: a. stimulate crying and lung expansion. b. remove maternal blood from the skin surface. c. reduce heat loss from evaporation. d. increase blood supply to the hands and feet.

ANS: C Infants are wet with amniotic fluid and blood at birth, and this accelerates evaporative heat loss. The primary purpose of drying the infant is to prevent heat loss. Rubbing the infant does stimulate crying; however, it is not the main reason for drying the infant. This process does not remove all the maternal blood.

35. A woman has a breast mass that is not well delineated and is nonpalpable, immobile, and nontender. This is most likely: A. fibroadenoma. B. lipoma. C. intraductal papilloma. D. mammary duct ectasia.

ANS: C Intraductal papilloma is the only benign breast mass that is nonpalpable. Fibroadenoma is well delineated, palpable, and movable. Lipoma is palpable and movable. Mammary duct ectasia is not well delineated and is immobile, but it is palpable and painful.

18. Which minerals and vitamins usually are recommended to supplement a pregnant woman's diet? A. Fat-soluble vitamins A and D B. Water-soluble vitamins C and B6 C. Iron and folate D. Calcium and zinc

ANS: C Iron generally should be supplemented, and folic acid supplements often are needed because folate is so important. Fat-soluble vitamins should be supplemented as a medical prescription, as vitamin D might be for lactose-intolerant women. Water-soluble vitamin C sometimes is consumed in excess naturally; vitamin B6 is prescribed only if the woman has a very poor diet. Zinc sometimes is supplemented. Most women obtain enough calcium through their regular diet.

11. Prepidil (prostaglandin gel) has been ordered for a pregnant woman at 43 weeks of gestation. The nurse recognizes that this medication will be administered to: a. enhance uteroplacental perfusion in an aging placenta. b. increase amniotic fluid volume. c. ripen the cervix in preparation for labor induction. d. stimulate the amniotic membranes to rupture.

ANS: C It is accurate to state that Prepidil will be administered to ripen the cervix in preparation for labor induction. It is not administered to enhance uteroplacental perfusion in an aging placenta, increase amniotic fluid volume, or stimulate the amniotic membranes to rupture.

20. To reassure and educate pregnant patients about changes in their breasts, nurses should be aware that: A. the visibility of blood vessels that form an intertwining blue network indicates full function of Montgomery's tubercles and possibly infection of the tubercles. B. the mammary glands do not develop until 2 weeks before labor. C. lactation is inhibited until the estrogen level declines after birth. D. colostrum is the yellowish oily substance used to lubricate the nipples for breastfeeding.

ANS: C Lactation is inhibited until after birth. The visible blue network of blood vessels is a normal outgrowth of a richer blood supply. The mammary glands are functionally complete by midpregnancy. Colostrum is a creamy, white-to-yellow premilk fluid that can be expressed from the nipples before birth.

The nurse is caring for an adolescent who had an external fixator placed after suffering a fracture of the wrist during a bicycle accident. Which statement by the adolescent would be expected about separation anxiety? a. I wish my parents could spend the night with me while I am in the hospital. b. I think I would like for my siblings to visit me but not my friends. c. I hope my friends dont forget about visiting me. d. I will be embarrassed if my friends come to the hospital to visit.

ANS: C Loss of peer-group contact may pose a severe emotional threat to an adolescent because of loss of group status; friends visiting is an important aspect of hospitalization for an adolescent and would be very reassuring. Adolescents may welcome the opportunity to be away from their parents. The separation from siblings may produce reactions from difficulty coping to a welcome relief.

16. With regard to a woman's reordering of personal relationships during pregnancy, the maternity nurse should understand that: A. because of the special motherhood bond, a woman's relationship with her mother is even more important than with the father of the child. B. nurses need not get involved in any sexual issues the couple has during pregnancy, particularly if they have trouble communicating them to each other. C. women usually express two major relationship needs during pregnancy: feeling loved and valued and having the child accepted by the father. D. the woman's sexual desire is likely to be highest in the first trimester because of the excitement and because intercourse is physically easier.

ANS: C Love and support help a woman feel better about her pregnancy. The most important person to the pregnant woman is usually the father. Nurses can facilitate communication between partners about sexual matters if, as is common, they are nervous about expressing their worries and feelings. The second trimester is the time when a woman's sense of well-being, along with certain physical changes, increases her desire for sex. Desire is decreased in the first and third trimesters.

A school-age child, admitted for intravenous antibiotic therapy for osteomyelitis, reports difficulty in going to sleep at night. Which intervention should the nurse implement to assist the child in going to sleep at bedtime? a. Request a prescription for a sleeping pill. b. Allow the child to stay up late and sleep late in the morning. c. Create a schedule similar to the one the child follows at home. d. Plan passive activities in the morning and interactive activities right before bedtime.

ANS: C Many children obtain significantly less sleep in the hospital than at home; the primary causes are a delay in sleep onset and early termination of sleep because of hospital routines. One technique that can minimize the disruption in the childs routine is establishing a daily schedule. This approach is most suitable for noncritically ill school-age and adolescent children who have mastered the concept of time. It involves scheduling the childs day to include all those activities that are important to the child and nurse, such as treatment procedures, schoolwork, exercise, television, playroom, and hobbies. The school-age child with osteomyelitis would benefit from a schedule similar to the one followed at home. Requesting a prescription for a sleeping pill would be inappropriate, and allowing the child to stay up late and sleep late would not be keeping the child in a routine followed at home. Passive activities in the morning and interactive activities at bedtime should be reversed; it would be better to keep the child active in the morning hours and plan quiet activities at bedtime.

8. Prostaglandins are produced in most organs of the body, including the uterus. Other source(s) of prostaglandins is/are: A. ovaries. B. breast milk. C. menstrual blood. D. the vagina.

ANS: C Menstrual blood is a potent source of prostaglandins. Prostaglandins are produced in most organs of the body and in menstrual blood. The ovaries, breast milk, and vagina are neither organs nor a source of prostaglandins.

6. Menstruation is periodic uterine bleeding: A. that occurs every 28 days. B. in which the entire uterine lining is shed. C. that is regulated by ovarian hormones. D. that leads to fertilization.

ANS: C Menstruation is periodic uterine bleeding that is controlled by a feedback system involving three cycles: endometrial, hypothalamic-pituitary, and ovarian. The average length of a menstrual cycle is 28 days, but variations are normal. During the endometrial cycle, the functional two thirds of the endometrium are shed. Lack of fertilization leads to menstruation.

15. Management of primary dysmenorrhea often requires a multifaceted approach. The nurse who provides care for a patient with this condition should be aware that the optimal pharmacologic therapy for pain relief is: A. acetaminophen. B. oral contraceptives (OCPs). C. nonsteroidal anti-inflammatory drugs (NSAIDs). D. aspirin.

ANS: C NSAIDs are prostaglandin inhibitors and show the strongest research results for pain relief. Often if one NSAID is not effective, another one can provide relief. Approximately 80% of women find relief from NSAIDs. Preparations containing acetaminophen are less effective for dysmenorrhea because they lack the antiprostaglandin properties of NSAIDs. OCPs are a reasonable choice for women who also want birth control. The benefit of OCPs is the reduction of menstrual flow and irregularities. OCPs may be contraindicated for some women and have numerous potential side effects. NSAIDs are the drug of choice. If a woman is taking a NSAID, she should avoid taking aspirin.

25. The nurse caring for the woman in labor should understand that increased variability of the fetal heart rate may be caused by: a. narcotics. b. barbiturates. c. methamphetamines. d. tranquilizers.

ANS: C Narcotics, barbiturates, and tranquilizers may be causes of decreased variability; methamphetamines may cause increased variability.

22. While instructing a couple regarding birth control, the nurse should be aware that the method called natural family planning: A. is the same as coitus interruptus, or "pulling out." B. uses the calendar method to align the woman's cycle with the natural phases of the moon. C. is the only contraceptive practice acceptable to the Roman Catholic Church. D. relies on barrier methods during fertility phases.

ANS: C Natural family planning is another name for periodic abstinence, which is the accepted way to pass safely through the fertility phases without relying on chemical or physical barriers. Natural family planning is the only contraceptive practice acceptable to the Roman Catholic Church. "Pulling out" is not the same as periodic abstinence, another name for natural family planning. The phases of the moon are not part of the calendar method or any method.

8. Chronic otitis media with effusion (OME) is differentiated from acute otitis media (AOM) because it is usually characterized by: a. Fever as high as 40 C (104 F). c. Nausea and vomiting. b. Severe pain in the ear. d. A feeling of fullness in the ear.

ANS: D OME is characterized by an immobile or orange-discolored tympanic membrane and nonspecific complaints and does not cause severe pain. Fever and severe pain may be signs of AOM. Nausea and vomiting are associated with otitis media

24. Because the risk for childbirth complications may be revealed, nurses should know that the point of maximal intensity (PMI) of the fetal heart tone (FHT) is: a. usually directly over the fetal abdomen. b. in a vertex position heard above the mother's umbilicus. c. heard lower and closer to the midline of the mother's abdomen as the fetus descends and rotates internally. d. in a breech position heard below the mother's umbilicus.

ANS: C Nurses should be prepared for the shift. The PMI of the FHT usually is directly over the fetal back. In a vertex position it is heard below the mother's umbilicus. In a breech position it is heard above the mother's umbilicus.

7. In practical terms regarding genetic health care, nurses should be aware that: A. genetic disorders affect people of all socioeconomic backgrounds, races, and ethnic groups equally. B. genetic health care is more concerned with populations than individuals. C. the most important of all nursing functions is providing emotional support to the family during counseling. D. taking genetic histories is the province of large universities and medical centers.

ANS: C Nurses should be prepared to help with various stress reactions from a couple facing the possibility of a genetic disorder. Although anyone may have a genetic disorder, certain disorders appear more often in certain ethnic and racial groups. Genetic health care is highly individualized because treatments are based on the phenotypic responses of the individual. Individual nurses at any facility can take a genetic history, although larger facilities may have better support services.

What is the primary disadvantage associated with outpatient and day facility care? a. Increased cost b. Increased risk of infection c. Lack of physical connection to the hospital d. Longer separation of the child from family

ANS: C Outpatient and day facility care do not provide extended care; therefore, a child requiring extended care must be transferred to the hospital, causing increased stress to the child and parents. Outpatient care decreases cost and reduces the risk of infection. Outpatient care also minimizes separation of the child from family.

11. Which statement concerning cyclic perimenstrual pain and discomfort (CPPD) is accurate? A. Premenstrual dysphoric disorder (PMDD) is a milder form of premenstrual syndrome (PMS) and more common in younger women. B. Secondary dysmenorrhea is more intense and medically significant than primary dysmenorrhea. C. Premenstrual syndrome is a complex, poorly understood condition that may include any of a hundred symptoms. D. The causes of PMS have been well established.

ANS: C PMS may manifest with one or more of a hundred or so physical and psychologic symptoms. PMDD is a more severe variant of PMS. Secondary dysmenorrhea is characterized by more muted pain than that seen in primary dysmenorrhea; the medical treatment is much the same. The cause of PMS is unknown. It may be a collection of different problems.

Emma, age 3 years, is being admitted for about 1 week of hospitalization. Her parents tell the nurse that they are going to buy her a lot of new toys because she will be in the hospital. The nurses reply should be based on an understanding that: a. New toys make hospitalization easier. b. New toys are usually better than older ones for children of this age. c. At this age children often need the comfort and reassurance of familiar toys from home. d. Buying new toys for a hospitalized child is a maladaptive way to cope with parental guilt.

ANS: C Parents should bring favorite items from home to be with the child. Young children associate inanimate objects with significant people; they gain comfort and reassurance from these items. New toys will not serve the purpose of familiar toys and objects from home. The parents may experience some guilt as a response to the hospitalization, but there is no evidence that it is maladaptive.

12. A patient at 24 weeks of gestation contacts the nurse at her obstetric provider's office to complain that she has cravings for dirt and gravel. The nurse is aware that this condition is known as ________ and may indicate anemia. A. ptyalism B. pyrosis C. pica D. decreased peristalsis

ANS: C Pica (a desire to eat nonfood substances) is an indication of iron deficiency and should be evaluated. Ptyalism (excessive salivation), pyrosis (heartburn), and decreased peristalsis are normal findings of gastrointestinal change during pregnancy. Food cravings during pregnancy are normal.

29. Surgical, medical, or mechanical methods may be used for labor induction. Which technique is considered a mechanical method of induction? a. Amniotomy b. Intravenous Pitocin c. Transcervical catheter d. Vaginal insertion of prostaglandins

ANS: C Placement of a balloon-tipped Foley catheter into the cervix is a mechanical method of induction. Other methods to expand and gradually dilate the cervix include hydroscopic dilators such as laminaria tents (made from desiccated seaweed), or Lamicel (contains magnesium sulfate). Amniotomy is a surgical method of augmentation and induction.Intravenous Pitocin and insertion of prostaglandins are medical methods of induction.

8. Metabolic changes throughout pregnancy that affect glucose and insulin in the mother and the fetus are complicated but important to understand. Nurses should understand that: A. insulin crosses the placenta to the fetus only in the first trimester, after which the fetus secretes its own. B. women with insulin-dependent diabetes are prone to hyperglycemia during the first trimester because they are consuming more sugar. C. during the second and third trimesters, pregnancy exerts a diabetogenic effect that ensures an abundant supply of glucose for the fetus. D. maternal insulin requirements steadily decline during pregnancy.

ANS: C Pregnant women develop increased insulin resistance during the second and third trimesters. Insulin never crosses the placenta; the fetus starts making its own insulin around the 10th week. As a result of normal metabolic changes during pregnancy, insulin-dependent women are prone to hypoglycemia (low levels). Maternal insulin requirements may double or quadruple by the end of pregnancy.

Several blood tests are ordered for a preschool child with severe anemia. The child is crying and upset because of memories of the venipuncture done at the clinic 2 days ago. The nurse should explain: a. venipuncture discomfort is very brief. b. only one venipuncture will be needed. c. topical application of local anesthetic can eliminate venipuncture pain. d. most blood tests on children require only a finger puncture because a small amount of blood is needed.

ANS: C Preschool children are concerned with both pain and the loss of blood. When preparing the child for venipuncture, the nurse will use a topical anesthetic to eliminate any pain. This is a traumatic experience for preschool children. They are concerned about their bodily integrity. A local anesthetic should be used, and a bandage should be applied to maintain bodily integrity. The nurse should not promise one attempt in case multiple attempts are required. Both finger punctures and venipunctures are traumatic for children. Both require preparation.

25. Which factor is known to increase the risk of gestational diabetes mellitus? A. Underweight before pregnancy B. Maternal age younger than 25 years C. Previous birth of large infant D. Previous diagnosis of type 2 diabetes mellitus

ANS: C Previous birth of a large infant suggests gestational diabetes mellitus. Obesity (BMI of 30 or greater) creates a higher risk for gestational diabetes. A woman younger than 25 years generally is not at risk for gestational diabetes mellitus. The person with type 2 diabetes mellitus already has diabetes and will continue to have it after pregnancy. Insulin may be required during pregnancy because oral hypoglycemia drugs are contraindicated during pregnancy.

1. In planning for home care of a woman with preterm labor, which concern must the nurse address? a. Nursing assessments will be different from those done in the hospital setting. b. Restricted activity and medications will be necessary to prevent recurrence of preterm labor. c. Prolonged bed rest may cause negative physiologic effects. d. Home health care providers will be necessary.

ANS: C Prolonged bed rest may cause adverse effects such as weight loss, loss of appetite, muscle wasting, weakness, bone demineralization, decreased cardiac output, risk for thrombophlebitis, alteration in bowel functions, sleep disturbance, and prolonged after birth recovery. Nursing assessments will differ somewhat from those performed in the acute care setting, but this is not the concern that needs to be addressed. Restricted activity and medication may prevent preterm labor, but not in all women. In addition, the plan of care is individualized to meet the needs of each woman. Many women will receive home health nurse visits, but care is individualized for each woman.

15. A woman with asthma is experiencing a after birth hemorrhage. Which drug would not be used to treat her bleeding because it may exacerbate her asthma? A. Pitocin B. Nonsteroidal anti-inflammatory drugs (NSAIDs) C. Hemabate D. Fentanyl

ANS: C Prostaglandin derivatives should not be used to treat women with asthma because they may exacerbate symptoms. Pitocin would be the drug of choice to treat this woman's bleeding because it would not exacerbate her asthma. NSAIDs are not used to treat bleeding. Fentanyl is used to treat pain, not bleeding.

4. Which statement by the client would lead the nurse to believe that labor has been established? a. "I passed some thick, pink mucus when I urinated this morning." b. "My bag of waters just broke." c. "The contractions in my uterus are getting stronger and closer together." d. "My baby dropped, and I have to urinate more frequently now."

ANS: C Regular, strong contractions with the presence of cervical change indicate that the woman is experiencing true labor. Although the loss of the mucous plug (operculum) often occurs during the first stage of labor or before the onset of labor, it is not the indicator of true labor. Spontaneous rupture of membranes often occurs during the first stage of labor; however, it is not an indicator of true labor. The presenting part of the fetus typically becomes engaged in the pelvis at the onset of labor but is not the indicator of true labor.

1. The nurse recognizes that a woman is in true labor when she states: a. "I passed some thick, pink mucus when I urinated this morning." b. "My bag of waters just broke." c. "The contractions in my uterus are getting stronger and closer together." d. "My baby dropped, and I have to urinate more frequently now."

ANS: C Regular, strong contractions with the presence of cervical change indicate that the woman is experiencing true labor. Loss of the mucous plug (operculum) often occurs during the first stage of labor or before the onset of labor, but it is not the indicator of true labor. Spontaneous rupture of membranes often occurs during the first stage of labor, but it is not the indicator of true labor. The presenting part of the fetus typically becomes engaged in the pelvis at the onset of labor, but this is not the indicator of true labor.

30. A pregnant woman who abuses cocaine admits to exchanging sex for her drug habit. This behavior places her at a greater risk for: A. depression of the central nervous system. B. hypotension and vasodilation. C. sexually transmitted diseases. D. postmature birth.

ANS: C Sex acts exchanged for drugs places the woman at increased risk for sexually transmitted diseases because of multiple partners and lack of protection. Cocaine is a central nervous system stimulant that causes hypertension and vasoconstriction. Premature delivery of the infant is one of the most common problems associated with cocaine use during pregnancy.

12. With regard to endometriosis, nurses should be aware that: A. it is characterized by the presence and growth of endometrial tissue inside the uterus. B. it is found more often in African-American women than in white or Asian women. C. it may worsen with repeated cycles or remain asymptomatic and disappear after menopause. D. it is unlikely to affect sexual intercourse or fertility.

ANS: C Symptoms vary among women, ranging from nonexistent to incapacitating. With endometriosis, the endometrial tissue is outside the uterus. Symptoms vary among women, ranging from nonexistent to incapacitating. Endometriosis is found equally in white and African-American women and is slightly more prevalent in Asian women. Women can experience painful intercourse and impaired fertility.

19. According to standard professional thinking, nurses should auscultate the fetal heart rate (FHR): a. every 15 minutes in the active phase of the first stage of labor in the absence of risk factors. b. every 20 minutes in the second stage, regardless of whether risk factors are present. c. before and after ambulation and rupture of membranes. d. more often in a woman's first pregnancy.

ANS: C The FHR should be auscultated before and after administration of medications and induction of anesthesia. In the active phase of the first stage of labor, the FHR should be auscultated every 30 minutes if no risk factors are involved; with risk factors it should be auscultated every 15 minutes. In the second stage of labor the FHR should be auscultated every 15 minutes if no risk factors are involved; with risk factors it should be auscultated every 5 minutes. The fetus of a first-time mother is automatically at greater risk.

19. A fully matured endometrium that has reached the thickness of heavy, soft velvet describes the _____ phase of the endometrial cycle. a. menstrual b. proliferative c. secretory d. ischemic

ANS: C The secretory phase extends from the day of ovulation to approximately 3 days before the next menstrual cycle. During this phase, the endometrium becomes fully mature. During the menstrual phase, the endometrium is being shed; the endometrium is fully mature again during the secretory phase. The proliferative phase is a period of rapid growth, but the endometrium becomes fully mature again during the secretory phase. During the ischemic phase, the blood supply is blocked, and necrosis develops. The endometrium is fully mature during the secretory phase.

A home care nurse is caring for an adolescent with a T1 spinal cord injury. The adolescent suddenly becomes flushed, hypertensive, and diaphoretic. Which intervention should the nurse perform first? a. Place the adolescent in a flat right side-lying position. b. Place a cool washcloth on the adolescents forehead and continue to monitor the blood pressure. c. Implement a standing prescription to empty the bladder with a sterile in and out Foley catheter. d. Take a full set of vital signs and notify the health care provider.

ANS: C The adolescent is experiencing an autonomic dysreflexia episode. The paralytic nature of autonomic function is replaced by autonomic dysreflexia, especially when the lesions are above the mid-thoracic level. This autonomic phenomenon is caused by visceral distention or irritation, particularly of the bowel or bladder. Sensory impulses are triggered and travel to the cord lesion, where they are blocked, which causes activation of sympathetic reflex action with disturbed central inhibitory control. Excessive sympathetic activity is manifested by a flushing face, sweating forehead, pupillary constriction, marked hypertension, headache, and bradycardia. The precipitating stimulus may be merely a full bladder or rectum or other internal or external sensory input. It can be a catastrophic event unless the irritation is relieved. Placing a cool washcloth on the adolescents forehead, continuing to monitor blood pressure and vital signs, and notifying the healthcare provider would not reverse the sympathetic reflex situation.

A. 1. A father and mother are carriers of phenylketonuria (PKU). Their 2-year-old daughter has PKU. The couple tells the nurse that they are planning to have a second baby. Because their daughter has PKU, they are sure that their next baby won't be affected. What response by the nurse is most accurate? A. "Good planning; you need to take advantage of the odds in your favor." B. "I think you'd better check with your doctor first." C. "You are both carriers, so each baby has a 25% chance of being affected." D. "The ultrasound indicates a boy, and boys are not affected by PKU."

ANS: C The chance is one in four that each child produced by this couple will be affected by PKU disorder. This couple still has an increased likelihood of having a child with PKU. Having one child already with PKU does not guarantee that they will not have another. These parents need to discuss their options with their physician. However, an opportune time has presented itself for the couple to receive correct teaching about inherited genetic risks. No correlation exists between gender and inheritance of the disorder because PKU is an autosomal recessive disorder.

The nurse is conducting a staff in-service on sickle cell anemia. Which describes the pathologic changes of sickle cell anemia? a. Sickle-shaped cells carry excess oxygen. b. Sickle-shaped cells decrease blood viscosity. c. Increased red blood cell destruction occurs. d. Decreased adhesion of sickle-shaped cells occurs.

ANS: C The clinical features of sickle cell anemia are primarily the result of increased red blood cell destruction and obstruction caused by the sickle-shaped red blood cells. Sickled red cells have decreased oxygen-carrying capacity and transform into the sickle shape in conditions of low oxygen tension. When the sickle cells change shape, they increase the viscosity in the area where they are involved in the microcirculation. Increased adhesion and entanglement of cells occurs.

27. Which description of the phases of the second stage of labor is accurate? a. Latent phase: Feeling sleepy, fetal station 2+ to 4+, duration 30 to 45 minutes b. Active phase: Overwhelmingly strong contractions, Ferguson reflux activated, duration 5 to 15 minutes c. Descent phase: Significant increase in contractions, Ferguson reflux activated, average duration varied d. Transitional phase: Woman "laboring down," fetal station 0, duration 15 minutes

ANS: C The descent phase begins with a significant increase in contractions; the Ferguson reflex is activated, and the duration varies, depending on a number of factors. The latent phase is the lull, or "laboring down," period at the beginning of the second stage. It lasts 10 to 30 minutes on average. The second stage of labor has no active phase. The transition phase is the final phase in the second stage of labor; contractions are strong and painful.

22. Which statement regarding female sexual response is inaccurate? A. Women and men are more alike than different in their physiologic response to sexual arousal and orgasm. B. Vasocongestion is the congestion of blood vessels. C. The orgasmic phase is the final state of the sexual response cycle. D. Facial grimaces and spasms of hands and feet are often part of arousal.

ANS: C The final state of the sexual response cycle is the resolution phase after orgasm. Men and women are surprisingly alike. Vasocongestion causes vaginal lubrication and engorgement of the genitals. Arousal is characterized by increased muscular tension (myotonia).

Which clinical manifestations in an infant would be suggestive of spinal muscular atrophy (Werdnig-Hoffmann disease)? a. Hyperactive deep tendon reflexes b. Hypertonicity c. Lying in the frog position d. Motor deficits on one side of body

ANS: C The infant lies in the frog position with the legs externally rotated, abducted, and flexed at knees. The deep tendon reflexes are absent. The child has hypotonia and inactivity as the most prominent features. The motor deficits are bilateral.

Parents of a hemophiliac child ask the nurse, Can you describe hemophilia to us? Which response by the nurse is descriptive of most cases of hemophilia? a. Autosomal dominant disorder causing deficiency in a factor involved in the blood-clotting reaction b. X-linked recessive inherited disorder causing deficiency of platelets and prolonged bleeding c. X-linked recessive inherited disorder in which a blood-clotting factor is deficient d. Y-linked recessive inherited disorder in which the red blood cells become moon-shaped

ANS: C The inheritance pattern in 80% of all of the cases of hemophilia is X-linked recessive. The two most common forms of the disorder are factor VIII deficiency, hemophilia A or classic hemophilia; and factor IX deficiency, hemophilia B or Christmas disease. The inheritance pattern is X-linked recessive. The disorder involves coagulation factors, not platelets, and does not involve red cells or the Y chromosomes.

8. A 36-year-old woman has been given a diagnosis of uterine fibroids. When planning care for this patient, the nurse should know that: A. fibroids are malignant tumors of the uterus that require radiation or chemotherapy. B. fibroids increase in size during the perimenopausal period. C. menorrhagia is a common finding. D. the woman is unlikely to become pregnant as long as the fibroids are in her uterus.

ANS: C The major symptoms associated with fibroids are menorrhagia and the physical effects produced by large myomas. Fibroids are benign tumors of the smooth muscle of the uterus, and their etiology is unknown. Fibroids are estrogen sensitive and shrink as levels of estrogen decline. Fibroids occur in 25% of women of reproductive age and are seen in 2% of pregnant women.

A school-age child with leukemia experienced severe nausea and vomiting when receiving chemotherapy for the first time. Which is the most appropriate nursing action to prevent or minimize these reactions with subsequent treatments? a. Encourage drinking large amounts of favorite fluids. b. Encourage child to take nothing by mouth (remain NPO) until nausea and vomiting subside. c. Administer an antiemetic before chemotherapy begins. d. Administer an antiemetic as soon as child has nausea.

ANS: C The most beneficial regimen to minimize nausea and vomiting associated with chemotherapy is to administer the antiemetic before the chemotherapy is begun. The goal is to prevent anticipatory symptoms. Drinking fluids will add to the discomfort of the nausea and vomiting. Remaining until nausea and vomiting subside will help with this episode, but the child will have the discomfort and be at risk for dehydration. Administering an antiemetic as soon as the child has nausea does not prevent anticipatory nausea.

4. A woman in labor has just received an epidural block. The most important nursing intervention is to: a. limit parenteral fluids. b. monitor the fetus for possible tachycardia. c. monitor the maternal blood pressure for possible hypotension. d. monitor the maternal pulse for possible bradycardia.

ANS: C The most important nursing intervention for a woman who has received an epidural block is to monitor the maternal blood pressure frequently for signs of hypotension. Intravenous fluids are increased for a woman receiving an epidural, to prevent hypotension. The nurse observes for signs of fetal bradycardia. The nurse monitors for signs of maternal tachycardia secondary to hypotension.

11. A labor and delivery nurse should be cognizant of which information regarding how the fetus moves through the birth canal? a. Fetal attitude describes the angle at which the fetus exits the uterus. b. Of the two primary fetal lies, the horizontal lie is that in which the long axis of the fetus is parallel to the long axis of the mother. c. Normal attitude of the fetus is called general flexion. d. Transverse lie is preferred for vaginal birth.

ANS: C The normal attitude of the fetus is called general flexion. The fetal attitude is the relationship of the fetal body parts to each one another. The horizontal lie is perpendicular to the mother; in the longitudinal (or vertical) lie, the long axes of the fetus and the mother are parallel. Vaginal birth cannot occur if the fetus stays in a transverse lie.

5. Which action is correct when palpation is used to assess the characteristics and pattern of uterine contractions? a. Place the hand on the abdomen below the umbilicus and palpate uterine tone with the fingertips. b. Determine the frequency by timing from the end of one contraction to the end of the next contraction. c. Evaluate the intensity by pressing the fingertips into the uterine fundus. d. Assess uterine contractions every 30 minutes throughout the first stage of labor.

ANS: C The nurse or primary care provider may assess uterine activity by palpating the fundal section of the uterus using the fingertips. Many women may experience labor pain in the lower segment of the uterus that may be unrelated to the firmness of the contraction detectable in the uterine fundus. The frequency of uterine contractions is determined by palpating from the beginning of one contraction to the beginning of the next contraction. Assessment of uterine activity is performed in intervals based on the stage of labor. As labor progresses this assessment is performed more frequently.

A nurse is preparing to complete an admission assessment on a 2-year-old child. The child is sitting on the parents lap. Which technique should the nurse implement to complete the physical exam? a. Ask the parent to place the child in the hospital crib. b. Take the child and parent to the exam room. c. Perform the exam while the child is on the parents lap. d. Ask the child to stand by the parent while completing the exam.

ANS: C The nurse should complete the exam while the child is on the parents lap. For young children, particularly infants and toddlers, preserving parent-child contact is the best means of decreasing the need for or stress of restraint. The entire physical examination can be done in a parents lap with the parent hugging the child for procedures such as an otoscopic examination. Placing the child in the crib, taking the child to the exam room, or asking the child to stand by the parent would separate the child from the parent and cause anxiety.

Which should the nurse include when teaching the mother of a 9-month-old infant about administering liquid iron preparations? a. They should be given with meals. b. They should be stopped immediately if nausea and vomiting occur. c. Adequate dosage will turn the stools a tarry green color. d. Allow preparation to mix with saliva and bathe the teeth before swallowing.

ANS: C The nurse should prepare the mother for the anticipated change in the childs stools. If the iron dose is adequate, the stools will become a tarry green color. The lack of the color change may indicate insufficient iron. The iron should be given in two divided doses between meals when the presence of free hydrochloric acid is greatest. Iron is absorbed best in an acidic environment. Vomiting and diarrhea may occur with iron administration. If these occur, the iron should be given with meals, and the dosage reduced, then gradually increased as the child develops tolerance. Liquid preparations of iron stain the teeth. They should be administered through a straw and the mouth rinsed after administration.

10. The nurse should be aware that the partner's main role in pregnancy is to: A. provide financial support. B. protect the pregnant woman from "old wives' tales." C. support and nurture the pregnant woman. D. make sure the pregnant woman keeps prenatal appointments.

ANS: C The partner's main role in pregnancy is to nurture the pregnant woman and to respond her feelings of vulnerability. In older societies, the man enacted the ritual couvade. Changing cultural and professional attitudes have encouraged fathers' participation in the birth experience over the past 30 years.

20. Sally comes in for her first prenatal examination. This is her first child. She asks you (the nurse), "How does my baby get air inside my uterus?" The correct response is: A. "The baby's lungs work in utero to exchange oxygen and carbon dioxide." B. "The baby absorbs oxygen from your blood system." C. "The placenta provides oxygen to the baby and excretes carbon dioxide into your bloodstream." D. "The placenta delivers oxygen-rich blood through the umbilical artery to the baby's abdomen."

ANS: C The placenta functions by supplying oxygen and excreting carbon dioxide to the maternal bloodstream. The fetal lungs do not function for respiratory gas exchange in utero. The baby does not simply absorb oxygen from a woman's blood system. Blood and gas transport occur through the placenta. The placenta delivers oxygen-rich blood through the umbilical vein and not the artery.

A child is playing in the playroom. The nurse needs to take a blood pressure on the child. Which is the appropriate procedure for obtaining the blood pressure? a. Take the blood pressure in the playroom. b. Ask the child to come to the exam room to obtain the blood pressure. c. Ask the child to return to his or her room for the blood pressure, then escort the child back to the playroom. d. Document that the blood pressure was not obtained because the child was in the playroom.

ANS: C The playroom is a safe haven for children, free from medical or nursing procedures. The child can be returned to his or her room for the blood pressure and then escorted back to the playroom. The exam room is reserved for painful procedures that should not be performed in the childs hospital bed. Documenting that the blood pressure was not obtained because the child was in the playroom is inappropriate.

29. When providing care to the prenatal patient, the nurse understands that pica is defined as: A. intolerance of milk products. B. iron deficiency anemia. C. ingestion of nonfood substances. D. episodes of anorexia and vomiting.

ANS: C The practice of eating substances not normally thought of as food is called pica. Clay or dirt and solid laundry starch are the substances most commonly ingested. Intolerance of milk products is referred to as lactose intolerance. Pica may produce iron deficiency anemia if proper nutrition is decreased. Pica is not related to anorexia and vomiting.

22. To provide the patient with accurate information about dental care during pregnancy, maternity nurses should be aware that: A. dental care can be dropped from the priority list because the woman has enough to worry about and is getting a lot of calcium anyway. B. dental surgery, in particular, is contraindicated because of the psychologic stress it engenders. C. if dental treatment is necessary, the woman will be most comfortable with it in the second trimester. D. dental care interferes with the expectant mother's need to practice conscious relaxation.

ANS: C The second trimester is best for dental treatment because that is when the woman will be able to sit most comfortably in the dental chair. Dental care such as brushing with fluoride toothpaste is especially important during pregnancy because nausea during pregnancy may lead to poor oral hygiene. Emergency dental surgery is permissible, but the mother must clearly understand the risks and benefits. Conscious relaxation is useful, and it may even help the woman get through any dental appointments; it is not a reason to avoid them.

15. Nurses should be cognizant of what regarding the mechanism of labor? a. Seven critical movements must progress in a more or less orderly sequence. b. Asynclitism is sometimes achieved by means of the Leopold's maneuver. c. Effects of the forces determining descent are modified by the shape of the woman's pelvis and the size of the fetal head. d. At birth, the baby is said to achieve "restitution"; that is, a return to the C-shape of the womb.

ANS: C The size of the maternal pelvis and the ability of the fetal head to mold also affect the process. The seven identifiable movements of the mechanism of labor simultaneously occur in combinations, not in precise sequences. Asynclitism is the deflection of the baby's head; the Leopold's maneuver is a means of judging descent by palpating the mother's abdomen. Restitution is the rotation of the baby's head after the infant is born.

22. What is an advantage of external electronic fetal monitoring? a. The ultrasound transducer can accurately measure short-term variability and beat-to-beat changes in the fetal heart rate. b. The tocotransducer can measure and record the frequency, regularity, intensity, and approximate duration of uterine contractions (UCs). c. The tocotransducer is especially valuable for measuring uterine activity during the first stage of labor. d. Once correctly applied by the nurse, the transducer need not be repositioned even when the woman changes positions.

ANS: C The tocotransducer is especially valuable for measuring uterine activity during the first stage of labor, particularly when the membranes are intact. Short-term changes cannot be measured with this technology. The tocotransducer cannot measure and record the intensity of UCs. The transducer must be repositioned when the woman or fetus changes position.

The nurse is reviewing prenatal vitamin supplements with an expectant client. Which supplement should be included in the teaching? a. Vitamin A throughout pregnancy b. Multivitamin preparations as soon as pregnancy is suspected c. Folic acid for all women of childbearing age d. Folic acid during the first and second trimesters of pregnancy

ANS: C The widespread use of folic acid among women of childbearing age has decreased the incidence of spina bifida significantly. Vitamin A is not related to the prevention of spina bifida. Folic acid supplementation is recommended for the preconception period and during the pregnancy. Only 42% of women actually follow these guidelines.

2. A couple comes in for an infertility workup, having attempted to get pregnant for 2 years. The woman, 37 years, has always had irregular menstrual cycles but is otherwise healthy. The man has fathered two children from a previous marriage and had a vasectomy reversal 2 years ago. The man has had two normal semen analyses, but the sperm seem to be clumped together. What additional test is needed? A. Testicular biopsy B. Antisperm antibodies C. Follicle-stimulating hormone (FSH) level D. Examination for testicular infection

ANS: C The woman has irregular menstrual cycles. The scenario does not indicate that she has had any testing related to this irregularity. Hormone analysis is performed to assess endocrine function of the hypothalamic-pituitary-ovarian axis when menstrual cycles are absent or irregular. Determination of blood levels of prolactin, FSH, luteinizing hormone (LH), estradiol, progesterone, and thyroid hormones may be necessary to diagnose the cause of irregular menstrual cycles. A testicular biopsy would be indicated only in cases of azoospermia (no sperm cells) or severe oligospermia (low number of sperm cells). Antisperm antibodies are produced by a man against his own sperms. This is unlikely to be the case here because the man has already produced children. Examination for testicular infection would be done before semen analysis. Infection would affect spermatogenesis.

20. With regard to follow-up visits for women receiving prenatal care, nurses should be aware that: A. the interview portions become more intensive as the visits become more frequent over the course of the pregnancy. B. monthly visits are scheduled for the first trimester, every 2 weeks for the second trimester, and weekly for the third trimester. C. during the abdominal examination, the nurse should be alert for supine hypotension. D. for pregnant women, a systolic blood pressure (BP) of 130 and a diastolic BP of 80 is sufficient to be considered hypertensive.

ANS: C The woman lies on her back during the abdominal examination, possibly compressing the vena cava and aorta, which can cause a decrease in blood pressure and a feeling of faintness. The interview portion of follow-up examinations is less extensive than in the initial prenatal visits, during which so much new information must be gathered. Monthly visits are routinely scheduled for the first and second trimesters; visits increase to every 2 weeks at week 28 and to once a week at week 36. For pregnant women hypertension is defined as a systolic BP of 140 or greater and a diastolic BP of 90 or greater.

32. After a mastectomy, a woman should be instructed to perform all of the following except: A. emptying surgical drains twice a day and as needed. B. avoiding lifting more than 4.5 kg (10 lbs) or reaching above her head until given permission by her surgeon. C. wearing clothing with snug sleeves to support the tissue of the arm on the operative side. D. reporting immediately if inflammation develops at the incision site or in the affected arm.

ANS: C The woman should not be advised to wear snug clothing. Rather, she should be advised to avoid tight clothing, tight jewelry, and other causes of decreased circulation in the affected arm. As part of the teaching plan, the woman should be instructed to empty surgical drains, to avoid lifting more than 10 lbs or reaching above her head, and to report the development of incision site inflammation.

25. A patient at 24 weeks of gestation says she has a glass of wine with dinner every evening. The nurse will counsel her to eliminate all alcohol intake because: A. a daily consumption of alcohol indicates a risk for alcoholism. B. she will be at risk for abusing other substances as well. C. the fetus is placed at risk for altered brain growth. D. the fetus is at risk for multiple organ anomalies.

ANS: C There is no period during pregnancy when it is safe to consume alcohol. The documented effects of alcohol consumption during pregnancy include intellectual disability, learning disabilities, high activity level, and short attention span. The bra in grows most rapidly in the third trimester and is vulnerable to alcohol exposure during this time. Abuse of other substances has not been linked to alcohol use.

14. At approximately _____ weeks of gestation, lecithin is forming on the alveolar surfaces, the eyelids open, and the fetus measures approximately 27 cm crown to rump and weighs approximately 1110 g. a. 20 b. 24 c. 28 d. 30

ANS: C These milestones human development occur at approximately 28 weeks.

22. A woman in labor is breathing into a mouthpiece just before the start of her regular contractions. As she inhales, a valve opens, and gas is released. She continues to inhale the gas slowly and deeply until the contraction starts to subside. When the inhalation stops, the valve closes. This procedure is: a. not used much anymore. b. likely to be used in the second stage of labor but not in the first stage. c. an application of nitrous oxide. d. a prelude to cesarean birth.

ANS: C This is an application of nitrous oxide, which could be used in either the first or second stage of labor (or both) as part of the preparation for a vaginal birth. Nitrous oxide is self-administered and found to be very helpful.

11. The nurse guides a woman to the examination room and asks her to remove her clothes and put on an examination gown with the front open. The woman states, "I have special undergarments that I do not remove for religious reasons." The most appropriate response from the nurse would be: A. "You can't have an examination without removing all your clothes." B. "I'll ask the doctor to modify the examination." C. "Tell me about your undergarments. I'll explain the examination procedure, and then we can discuss how you can have your examination comfortably." D. "What? I've never heard of such a thing! That sounds different and strange."

ANS: C This statement reflects cultural competence by the nurse and shows respect for the woman's religious practices. The nurse must respect the rich and unique qualities that cultural diversity brings to individuals. In recognizing the value of these differences, the nurse can modify the plan of care to meet the needs of each woman.

19. A woman will be taking oral contraceptives using a 28-day pack. The nurse should advise this woman to protect against pregnancy by: A. limiting sexual contact for one cycle after starting the pill. B. using condoms and foam instead of the pill for as long as she takes an antibiotic. C. taking one pill at the same time every day. D. throwing away the pack and using a backup method if she misses two pills during Week 1 of her cycle.

ANS: C To maintain adequate hormone levels for contraception and to enhance compliance, patients should take oral contraceptives at the same time each day. If contraceptives are to be started at any time other than during normal menses or within 3 weeks after birth or abortion, another method of contraception should be used through the first week to prevent the risk of pregnancy. Taken exactly as directed, oral contraceptives prevent ovulation, and pregnancy cannot occur. No strong pharmacokinetic evidence indicates a link between the use of broad-spectrum antibiotics and altered hormone levels in oral contraceptive users. If the patient misses two pills during Week 1, she should take two pills a day for 2 days, finish the package, and use a backup method the next 7 consecutive days.

20. A woman had unprotected intercourse 36 hours ago and is concerned that she may become pregnant because it is her "fertile" time. She asks the nurse about emergency contraception. The nurse tells her that: A. it is too late; she needed to begin treatment within 24 hours after intercourse. B. Preven, an emergency contraceptive method, is 98% effective at preventing pregnancy. C. an over-the-counter antiemetic can be taken 1 hour before each contraceptive dose to prevent nausea and vomiting. D. the most effective approach is to use a progestin-only preparation.

ANS: C To minimize the side effect of nausea that occurs with high doses of estrogen and progestin, the woman can take an over-the-counter antiemetic 1 hour before each dose. Emergency contraception is used within 72 hours of unprotected intercourse to prevent pregnancy. Postcoital contraceptive use is 74% to 90% effective at preventing pregnancy. Oral emergency contraceptive regimens may include progestin-only and estrogen-progestin pills. Women with contraindications to estrogen use should use progestin-only pills.

2. The nurse teaches a pregnant woman about the characteristics of true labor contractions. The nurse evaluates the woman's understanding of the instructions when she states, "True labor contractions will: a. subside when I walk around." b. cause discomfort over the top of my uterus." c. continue and get stronger even if I relax and take a shower." d. remain irregular but become stronger."

ANS: C True labor contractions occur regularly, becoming stronger, lasting longer, and occurring closer together. They may become intense during walking and continue despite comfort measures. Typically true labor contractions are felt in the lower back, radiating to the lower portion of the abdomen. During false labor, contractions tend to be irregular and felt in the abdomen above the navel. Typically the contractions often stop with walking or a change of position.

4. A pregnant woman at 10 weeks of gestation jogs three or four times per week. She is concerned about the effect of exercise on the fetus. The nurse should inform her: A. "You don't need to modify your exercising any time during your pregnancy." B. "Stop exercising because it will harm the fetus." C. "You may find that you need to modify your exercise to walking later in your pregnancy, around the seventh month." D. "Jogging is too hard on your joints; switch to walking now."

ANS: C Typically running should be replaced with walking around the seventh month of pregnancy. The nurse should inform the woman that she may need to reduce her exercise level as the pregnancy progresses. Physical activity promotes a feeling of well-being in pregnant women. It improves circulation, promotes relaxation and rest, and counteracts boredom. Simple measures should be initiated to prevent injuries, such as warm-up and stretching exercises to prepare the joints for more strenuous exercise.

20. Which adaptation of the maternal-fetal exchange of oxygen occurs in response to uterine contraction? a. The maternal-fetal exchange of oxygen and waste products continues except when placental functions are reduced. b. This maternal-fetal exchange increases as the blood pressure decreases. c. It diminishes as the spiral arteries are compressed. d. This exchange of oxygen and waste products is not significantly affected by contractions.

ANS: C Uterine contractions during labor tend to decrease circulation through the spiral electrodes and subsequent perfusion through the intervillous space. The maternal blood supply to the placenta gradually stops with contractions. The exchange of oxygen and waste products decreases. The exchange of oxygen and waste products is affected by contractions.

5. The nurse providing care for the laboring woman should understand that late fetal heart rate (FHR) decelerations are the result of: a. altered cerebral blood flow. b. umbilical cord compression. c. uteroplacental insufficiency. d. meconium fluid.

ANS: C Uteroplacental insufficiency would result in late decelerations in the FHR. Altered fetal cerebral blood flow would result in early decelerations in the FHR. Umbilical cord compression would result in variable decelerations in the FHR. Meconium-stained fluid may or may not produce changes in the fetal heart rate, depending on the gestational age of the fetus and whether other causative factors associated with fetal distress are present.

3. A couple is trying to cope with an infertility problem. They want to know what they can do to preserve their emotional equilibrium. The nurse's most appropriate response is: A. "Tell your friends and family so they can help you." B. "Talk only to other friends who are infertile because only they can help." C. "Get involved with a support group. I'll give you some names." D. "Start adoption proceedings immediately because it is very difficult to obtain an infant."

ANS: C Venting negative feelings may unburden the couple. A support group may provide a safe haven for the couple to share their experiences and gain insight from others' experiences. Although talking about their feelings may unburden them of negative feelings, infertility can be a major stressor that affects the couple's relationships with family and friends. Limiting their interactions to other infertile couples may be a beginning point for addressing psychosocial needs, but depending on where the other couple is in their own recovery process, this may or may not help them. The statement about adoption proceedings is not supportive of the psychosocial needs of this couple and may be detrimental to their well-being.

The nurse is admitting a child with Werdnig-Hoffmann disease (spinal muscular atrophy type 1). Which signs and symptoms are associated with this disease? a. Spinal muscular atrophy b. Neural atrophy of muscles c. Progressive weakness and wasting of skeletal muscle d. Pseudohypertrophy of certain muscle groups

ANS: C Werdnig-Hoffmann disease (spinal muscular atrophy type 1) is the most common paralytic form of floppy infant syndrome (congenital hypotonia). It is characterized by progressive weakness and wasting of skeletal muscle caused by degeneration of anterior horn cells. Kugelberg-Welander disease is a juvenile spinal muscular atrophy with a later onset. Charcot-Marie-Tooth disease is a form of progressive neural atrophy of muscles supplied by the peroneal nerves. Progressive weakness is found of the distal muscles of the arms and feet. Duchenne muscular dystrophy is characterized by muscles, especially in the calves, thighs, and upper arms, which become enlarged from fatty infiltration and feel unusually firm or woody on palpation. The term pseudohypertrophy is derived from this muscular enlargement.

A 14-year-old boy is being admitted to the hospital for an appendectomy. Which roommate should the nurse assign with this patient? a. A 4-year-old boy who is first day post-appendectomy surgery b. A 6-year-old boy with pneumonia c. A 15-year-old boy admitted with a vaso-occlusive sickle cell crisis d. A 12-year-old boy with cellulitis

ANS: C When a child is admitted, nurses follow several fairly universal admission procedures. The minimum considerations for room assignment are age, sex, and nature of the illness. Age-grouping is especially important for adolescents. The 14-year-old boy being admitted to the unit after appendectomy surgery should be placed with a noninfectious child of the same sex and age. The 15-year-old child with sickle cell is the best choice. The 4-year-old boy who is post-appendectomy is too young, and the child with pneumonia is too young and possibly has an infectious process. The 12-year-old boy with cellulitis is the right age, but he has an infection (cellulitis).

8. A woman is having her first child. She has been in labor for 15 hours. Two hours ago her vaginal examination revealed the cervix to be dilated to 5 cm and 100% effaced, and the presenting part was at station 0. Five minutes ago her vaginal examination indicated that there had been no change. What abnormal labor pattern is associated with this description? a. Prolonged latent phase b. Protracted active phase c. Arrest of active phase d. Protracted descent

ANS: C With an arrest of the active phase, the progress of labor has stopped. This patient has not had any anticipated cervical change, thus indicating an arrest of labor. In the nulliparous woman a prolonged latent phase typically would last more than 20 hours. A protracted active phase, the first or second stage of labor, would be prolonged (slow dilation). With protracted descent, the fetus would fail to descend at an anticipated rate during the deceleration phase and second stage of labor.

6. A primigravida at 40 weeks of gestation is having uterine contractions every 1.5 to 2 minutes and says that they are very painful. Her cervix is dilated 2 cm and has not changed in 3 hours. The woman is crying and wants an epidural. What is the likely status of this woman's labor? a. She is exhibiting hypotonic uterine dysfunction. b. She is experiencing a normal latent stage. c. She is exhibiting hypertonic uterine dysfunction. d. She is experiencing pelvic dystocia.

ANS: C Women who experience hypertonic uterine dysfunction, or primary dysfunctional labor, often are anxious first-time mothers who are having painful and frequent contractions that are ineffective at causing cervical dilation or effacement to progress. With hypotonic uterine dysfunction, the woman initially makes normal progress into the active stage of labor; then the contractions become weak and inefficient or stop altogether. The contraction pattern seen in this woman signifies hypertonic uterine activity. Typically uterine activity in this phase occurs at 4- to 5-minute intervals lasting 30 to 45 seconds. Pelvic dystocia can occur whenever contractures of the pelvic diameters reduce the capacity of the bony pelvis, including the inlet, midpelvis, outlet, or any combination of these planes.

30. In caring for a pregnant woman with sickle cell anemia, the nurse is aware that signs and symptoms of sickle cell crisis include: A. anemia. B. endometritis. C. fever and pain. D. urinary tract infection.

ANS: C Women with sickle cell anemia have recurrent attacks (crisis) of fever and pain, most often in the abdomen, joints, and extremities. These attacks are attributed to vascular occlusion when RBCs assume the characteristic sickled shape. Crises are usually triggered by dehydration, hypoxia, or acidosis. Women with sickle cell anemia are not iron deficient. Therefore, routine iron supplementation, even that found in prenatal vitamins, should be avoided in order to prevent iron overload. Women with sickle cell trait usually are at greater risk for after birth endometritis (uterine wall infection); however, this is not likely to occur in pregnancy and is not a sign of crisis. These women are at an increased risk for UTIs; however, this is not an indication of sickle cell crisis.

Which neurologic diagnostic test gives a visualized horizontal and vertical cross-section of the brain at any axis? a. Nuclear brain scan b. Echoencephalography c. CT scan d. Magnetic resonance imaging (MRI)

ANS: C A CT scan provides a visualization of the horizontal and vertical cross-sections of the brain at any axis. A nuclear brain scan uses a radioisotope that accumulates where the blood-brain barrier is defective. Echoencephalography identifies shifts in midline structures of the brain as a result of intracranial lesions. MRI permits visualization of morphologic features of target structures and permits tissue discrimination that is unavailable with any other techniques.

The nurse is planning care for an 8-year-old child with a concussion. Which is descriptive of a concussion? a. Petechial hemorrhages cause amnesia. b. Visible bruising and tearing of cerebral tissue occur. c. It is a transient and reversible neuronal dysfunction. d. A slight lesion develops remotely from the site of trauma.

ANS: C A concussion is a transient, reversible neuronal dysfunction with instantaneous loss of awareness and responsiveness resulting from trauma to the head. Petechial hemorrhages along the superficial aspects of the brain along the point of impact are a type of contusion, but are not necessarily associated with amnesia. A contusion is visible bruising and tearing of cerebral tissue. Contrecoup is a lesion that develops remote from the site of trauma as a result of an acceleration-deceleration injury.

With regard to afterbirth pains, nurses should be aware that these pains are: a. Caused by mild, continuous contractions for the duration of the postpartum period. b. More common in first-time mothers. c. More noticeable in births in which the uterus was overdistended. d. Alleviated somewhat when the mother breastfeeds.

ANS: C A large baby or multiple babies overdistend the uterus. The cramping that causes afterbirth pains arises from periodic, vigorous contractions and relaxations, which persist through the first part of the postpartum period. Afterbirth pains are more common in multiparous women because first-time mothers have better uterine tone. Breastfeeding intensifies afterbirth pain because it stimulates contractions. PTS: 1 DIF: Cognitive Level: Comprehension REF: 484 OBJ: Nursing Process: Assessment MSC: Client Needs: Health Promotion and Maintenance

A child has been seizure-free for 2 years. A father asks the nurse how much longer the child will need to take the antiseizure medications. The nurse includes which intervention in the response? a. Medications can be discontinued at this time. b. The child will need to take the drugs for 5 years after the last seizure. c. A step-wise approach will be used to reduce the dosage gradually. d. Seizure disorders are a lifelong problem. Medications cannot be discontinued.

ANS: C A predesigned protocol is used to wean a child gradually off antiseizure medications, usually when the child is seizure-free for 2 years and has a normal electroencephalogram (EEG). Medications must be gradually reduced to minimize the recurrence of seizures. Seizure medications can be safely discontinued. The risk of recurrence is greatest within the first year.

As related to central nervous system injuries that could occur to the infant during labor and birth, nurses should be aware that: a. Intracranial hemorrhage (ICH) as a result of birth trauma is more likely to occur in the preterm, low-birth-weight infant. b. Subarachnoid hemorrhage (the most common form of ICH) occurs in term infants as a result of hypoxia. c. In many infants signs of hemorrhage in a full-term infant are absent and are diagnosed only through laboratory tests. d. Spinal cord injuries almost always result from forceps-assisted deliveries.

ANS: C Abnormalities in lumbar punctures or red blood cell counts, for instance, or in visuals on computed tomography scan may reveal a hemorrhage. ICH as a result of birth trauma is more likely to occur in the fullterm, large infant. Subarachnoid hemorrhage in term infants is a result of trauma; in preterm infants it is a result of hypoxia. Spinal cord injuries are almost always from breech births; they are rare today because cesarean birth often is used for breech presentation. PTS: 1 DIF: Cognitive Level: Comprehension REF: 666 OBJ: Nursing Process: Planning MSC: Client Needs: Physiologic Integrity

While discussing the societal impacts of breastfeeding, the nurse should be cognizant of the benefits and educate the patient accordingly. Which statement as part of this discussion would be incorrect? a. Breastfeeding requires fewer supplies and less cumbersome equipment. b. Breastfeeding saves families money. c. Breastfeeding costs employers in terms of time lost from work. d. Breastfeeding benefits the environment.

ANS: C Actually less time is lost to work by breastfeeding mothers, in part because infants are healthier. Breastfeeding is convenient because it does not require cleaning or transporting bottles and other equipment. It saves families money because the cost of formula far exceeds the cost of extra food for the lactating mother. Breastfeeding uses a renewable resource; it does not need fossil fuels, advertising, shipping, or disposal. PTS: 1 DIF: Cognitive Level: Comprehension REF: 650 OBJ: Nursing Process: Evaluation MSC: Client Needs: Health Promotion and Maintenance

Of the many factors that influence parental responses, nurses should be conscious of negative stereotypes that apply to specific patient populations. Which response could be an inappropriate stereotype of adolescent mothers? a. An adolescent mothers egocentricity and unmet developmental needs interfere with her ability to parent effectively. b. An adolescent mother is likely to use less verbal instruction, be less responsive, and interact less positively than other mothers. c. Adolescent mothers have a higher documented incidence of child abuse. d. Mothers older than 35 often deal with more stress related to work and career issues and decreasing libido.

ANS: C Adolescent mothers are more inclined to have a number of parenting difficulties that benefit from counseling; however, a higher incidence of child abuse is not one of them. Midlife mothers have many competencies, but they are more likely to have to deal with career issues and the accompanying stress. PTS: 1 DIF: Cognitive Level: Comprehension REF: 521 OBJ: Nursing Process: Evaluation MSC: Client Needs: Psychosocial Integrity

6. It is generally recommended that a child with acute streptococcal pharyngitis can return to school: a. When the sore throat is better. c. After taking antibiotics for 24 hours. b. If no complications develop. d. After taking antibiotics for 3 days.

ANS: C After children have taken antibiotics for 24 hours, even if the sore throat persists, they are no longer contagious to other children. Complications may take days to weeks to develop

A 10-year-old boy on a bicycle has been hit by a car in front of the school. The school nurse immediately assesses airway, breathing, and circulation. The next nursing action: should be to a. place on side. b. take blood pressure. c. stabilize neck and spine. d. check scalp and back for bleeding.

ANS: C After determining that the child is breathing and has adequate circulation, the next action is to stabilize the neck and spine to prevent any additional trauma. The child's position should not be changed until the neck and spine are stabilized. Blood pressure is a later assessment. Less urgent, but an important assessment, is inspection of the scalp for bleeding.

13. A child is 2 hours postoperative after a resection of a brain tumor. Which assessment by the nurse takes priority? A. Blood pressure B. Intake and output C. Neurological exam D. Temperature

ANS: C All actions are appropriate for a child postoperatively. However, the answer that is most specific to this child's procedure is the neurological exam.

A 3-year-old child is hospitalized after a submersion injury. The child's mother complains to the nurse, "Being at the hospital seems unnecessary when he is perfectly fine." The nurse's best reply should be: a. "He still needs a little extra oxygen." b. "I'm sure he is fine, but the doctor wants to make sure." c. "The reason for this is that complications could still occur." d. "It is important to observe for possible central nervous system problems."

ANS: C All children who have a submersion injury should be admitted to the hospital for observation. Although many children do not appear to have suffered adverse effects from the event, complications such as respiratory compromise and cerebral edema may occur 24 hours after the incident. The mother would not think the child is fine if oxygen were still required. The nurse should clarify that different complications can occur up to 24 hours later and that observations are necessary.

25. A nurse is caring for four patients who have Hodgkin's lymphoma. Which child should the nurse see first? A. Anorexia for a week B. Enlarged cervical lymph nodes C. Fever of 102.1°F (38.9°C) D. Mediastinal mass

ANS: C All options are possible manifestations of Hodgkin's lymphoma. However, the child with a fever may have another cause for the temperature, including infection, that needs to be ruled out. This is especially true of a child receiving chemotherapy, a standard treatment for this disorder.

7. A child is diagnosed with influenza, probably type A disease. Management includes: a. Clear liquid diet for hydration. b. Aspirin to control fever. c. Amantadine hydrochloride to reduce symptoms. d. Antibiotics to prevent bacterial infection

ANS: C Amantadine hydrochloride may reduce symptoms related to influenza type A if administered within 24 to 48 hours of onset. It is ineffective against type B or C. A clear liquid diet is not necessary for influenza, but maintaining hydration is important. Aspirin is not recommended in children because of increased risk of Reyes syndrome. Acetaminophen or ibuprofen is a better choice. Preventive antibiotics are not indicated for influenza unless there is evidence of a secondary bacterial infection

2. A nursing student asks the faculty member to explain an oncogene. Which response by the faculty member is the most appropriate? A. A cell that changes into a malignancy after environmental stress B. Any gene found inside a solid tumor that can be removed for biopsy C. A gene in a virus that encourages malignant transformation in cells D. An inherited gene that is programmed to become a malignant cell

ANS: C An oncogene is a gene found inside a virus that has the ability to encourage a normal cell to become malignant.

35. The nurse is teaching parents of a child with chronic renal failure (CRF) about the use of recombinant human erythropoietin (rHuEPO) subcutaneous injections. Which statement indicates the parents have understood the teaching? a. "These injections will help with the hypertension." b. "We're glad the injections only need to be given once a month." c. "The red blood cell count should begin to improve with these injections." d. "Urine output should begin to improve with these injections."

ANS: C Anemia in children with CRF is related to decreased production of erythropoietin. Recombinant human erythropoietin (rHuEPO) is being offered to these children as thrice-weekly or weekly subcutaneous injections and is replacing the need for frequent blood transfusions. The parents understand the teaching if they say that the red blood cell count will begin to improve with these injections.

An infant boy was born just a few minutes ago. The nurse is conducting the initial assessment. Part of the assessment includes the Apgar score. The Apgar assessment is performed: a. Only if the newborn is in obvious distress. b. Once by the obstetrician, just after the birth. c. At least twice, 1 minute and 5 minutes after birth. d. Every 15 minutes during the newborns first hour after birth.

ANS: C Apgar scoring is performed at 1 minute and 5 minutes after birth. Scoring may continue at 5-minute intervals if the infant is in distress and requires resuscitation efforts. PTS: 1 DIF: Cognitive Level: Comprehension REF: 583 OBJ: Nursing Process: Assessment MSC: Client Needs: Health Promotion and Maintenance

With regard to the gastrointestinal (GI) system of the newborn, nurses should be aware that: a. The newborns cheeks are full because of normal fluid retention. b. The nipple of the bottle or breast must be placed well inside the babys mouth because teeth have been developing in utero, and one or more may even be through. c. Regurgitation during the first day or two can be reduced by burping the infant and slightly elevating the babys head. d. Bacteria are already present in the infants GI tract at birth because they traveled through the placenta.

ANS: C Avoiding overfeeding can also reduce regurgitation. The newborns cheeks are full because of well-developed sucking pads. Teeth do develop in utero, but the nipple is placed deep because the baby cannot move food from the lips to the pharynx. Bacteria are not present at birth, but they soon enter through various orifices. PTS: 1 DIF: Cognitive Level: Comprehension REF: 563 OBJ: Nursing Process: Planning MSC: Client Needs: Health Promotion and Maintenance

The nurse administers vitamin K to the newborn for which reason? a. Most mothers have a diet deficient in vitamin K, which results in the infants being deficient. b. Vitamin K prevents the synthesis of prothrombin in the liver and must be given by injection. c. Bacteria that synthesize vitamin K are not present in the newborns intestinal tract. d. The supply of vitamin K is inadequate for at least 3 to 4 months, and the newborn must be supplemented.

ANS: C Bacteria that synthesize vitamin K are not present in the newborns intestinal tract. Vitamin K is provided because the newborn does not have the intestinal flora to produce this vitamin for the first week. The maternal diet has no bearing on the amount of vitamin K found in the newborn. Vitamin K promotes the formation of clotting factors in the liver and is used for the prevention and treatment of hemorrhagic disease in the newborn. Vitamin K is not produced in the intestinal tract of the newborn until after microorganisms are introduced. By day 8, normal newborns are able to produce their own vitamin K. PTS: 1 DIF: Cognitive Level: Comprehension REF: 613 OBJ: Nursing Process: Implementation MSC: Client Needs: Health Promotion and Maintenance

12. An 8-year-old child has been diagnosed with a brain tumor. Based on knowledge of childhood cancers, which intervention does the nurse plan to implement when the child is admitted to the hospital? A. Aspiration precautions B. Protective isolation C. Safety precautions D. Seizure precautions

ANS: C Brain tumors in children 1 to 10 years of age are usually infratentorial and involve the brainstem and cerebellum. Manifestations of brainstem tumors result from involvement of the cranial nerves and include hemiparesis, spastic gait, and frequent stumbling and falling. The nurse implements safety precautions for this child. The other precautions may or may not be needed depending on the child's specific condition, treatment, and side effects of treatment.

As the nurse assists a new mother with breastfeeding, the client asks, If formula is prepared to meet the nutritional needs of the newborn, what is in breast milk that makes it better? The nurses best response is that it contains: a. More calories. b. Essential amino acids. c. Important immunoglobulins. d. More calcium.

ANS: C Breast milk contains immunoglobulins that protect the newborn against infection. The calorie count of formula and breast milk is about the same. All the essential amino acids are in both formula and breast milk; however, the concentrations may differ. Calcium levels are higher in formula than in breast milk. This higher level can cause an excessively high renal solute load if the formula is not diluted properly. PTS: 1 DIF: Cognitive Level: Knowledge REF: 639 OBJ: Nursing Process: Implementation MSC: Client Needs: Physiologic Integrity

23. A child is in the hospital receiving chemotherapy for Hodgkin's lymphoma. What action by a new nurse causes the precepting nurse to intervene? A. Assesses the need for anti-emetics prior to starting chemotherapy B. Checks the IV for blood return before giving the chemotherapy C. Double wraps the chemotherapy bags and places in the trash can D. Performs hand hygiene prior to and after caring for the patient

ANS: C Chemotherapeutic agents are considered hazardous waste and must be disposed of in specific containers, not the trash can. The other actions are appropriate.

4. What describes moral development in younger school-age children? a. The standards of behavior now come from within themselves. b. They do not yet experience a sense of guilt when they misbehave. c. They know the rules and behaviors expected of them but do not understand the reasons behind them. d. They no longer interpret accidents and misfortunes as punishment for misdeeds.

ANS: C Children who are ages 6 and 7 years know the rules and behaviors expected of them but do not understand the reasons for them. Young children do not believe that standards of behavior come from within themselves but that rules are established and set down by others. Younger school-age children learn standards for acceptable behavior, act according to these standards, and feel guilty when they violate them. Misfortunes and accidents are viewed as punishment for bad acts.

19. A major complication in a child with chronic renal failure is: a. Hypokalemia. b. Metabolic alkalosis. c. Water and sodium retention. d. Excessive excretion of blood urea nitrogen.

ANS: C Chronic renal failure leads to water and sodium retention, which contributes to edema and vascular congestion. Hyperkalemia, metabolic acidosis, and retention of blood urea nitrogen are complications of chronic renal failure.

The nurse hears a primiparous woman talking to her son and telling him that his chin is just like his dads chin. This womans statement reflects: a. Mutuality. b. Synchrony. c. Claiming. d. Reciprocity.

ANS: C Claiming refers to the process by which the child is identified in terms of likeness to other family members. Mutuality occurs when the infants behaviors and characteristics call forth a corresponding set of maternal behaviors and characteristics. Synchrony refers to the fit between the infants cues and the parents responses. Reciprocity is a type of body movement or behavior that provides the observer with cues. PTS: 1 DIF: Cognitive Level: Comprehension REF: 511 OBJ: Nursing Process: Evaluation MSC: Client Needs: Psychosocial Integrity

A mother expresses fear about changing her infants diaper after he is circumcised. What does the woman need to be taught to take care of the infant when she gets home? a. Cleanse the penis with prepackaged diaper wipes every 3 to 4 hours. b. Apply constant, firm pressure by squeezing the penis with the fingers for at least 5 minutes if bleeding occurs. c. Cleanse the penis gently with water and put petroleum jelly around the glans after each diaper change. d. Wash off the yellow exudate that forms on the glans at least once every day to prevent infection.

ANS: C Cleansing the penis gently with water and putting petroleum jelly around the glans after each diaper change are appropriate when caring for an infant who has had a circumcision. With each diaper change, the penis should be washed off with warm water to remove any urine or feces. If bleeding occurs, the nurse should apply gentle pressure to the site of the bleeding with a sterile gauze square. Yellow exudates cover the glans penis in 24 hours after the circumcision. This is part of normal healing and not an infective process. The exudates should not be removed. PTS: 1 DIF: Cognitive Level: Application REF: 617 OBJ: Nursing Process: Planning MSC: Client Needs: Health Promotion and Maintenance

Which type of seizure involves both hemispheres of the brain? a. Focal b. Partial c. Generalized d. Acquired

ANS: C Clinical observations of generalized seizures indicate that the initial involvement is from both hemispheres. Focal seizures may arise from any area of the cerebral cortex, but the frontal, temporal, and parietal lobes are most commonly affected. Partial seizures are caused by abnormal electric discharges from epileptogenic foci limited to a circumscribed region of the cerebral cortex. A seizure disorder that is acquired is a result of a brain injury from a variety of factors; it does not specify the type of seizure.

The best way for the nurse to promote and support the maternal-infant bonding process is to: a. Help the mother identify her positive feelings toward the newborn. b. Encourage the mother to provide all newborn care. c. Assist the family with rooming-in. d. Return the newborn to the nursery during sleep periods.

ANS: C Close and frequent interaction between mother and infant, which is facilitated by rooming-in, is important in the bonding process. This is often referred to as the mother-baby care or couplet care. Having the mother express her feelings is important; however, it is not the best way to promote bonding. The mother needs time to rest and recuperate; she should not be expected to do all of the care. The patient needs to observe the infant during all stages so she will be aware of what to anticipate when they go home. PTS: 1 DIF: Cognitive Level: Comprehension REF: 516 OBJ: Nursing Process: Planning MSC: Client Needs: Psychosocial Integrity

A young child's parents call the nurse after their child was bitten by a raccoon in the woods. The nurse's recommendation should be based on which statement? a. Child should be hospitalized for close observation. b. No treatment is necessary if thorough wound cleaning is done. c. Antirabies prophylaxis must be initiated. d. Antirabies prophylaxis must be initiated if clinical manifestations appear.

ANS: C Current therapy for a rabid animal bite consists of a thorough cleansing of the wound and passive immunization with human rabies immune globulin (HRIG) as soon as possible. Hospitalization is not necessary. The wound cleansing, passive immunization, and immune globulin administration can be done as an outpatient. The child needs to receive both HRIG and rabies vaccine.

The nurse is monitoring a 7-year-old child post-surgical resection of an infratentorial brain tumor. Which vital sign findings indicate Cushing's triad? a. Increased temperature, tachycardia, tachypnea b. Decreased temperature, bradycardia, bradypnea c. Bradycardia, hypertension, irregular respirations d. Bradycardia, hypotension, tachypnea

ANS: C Cushing's triad is a hallmark sign of increased intracranial pressure (ICP). The triad includes bradycardia, hypertension, and irregular respirations. Increased or decreased temperature is not a sign of Cushing's triad.

Despite popular belief, there is a rare type of hemophilia that affects women of childbearing age. von Willebrand disease is the most common of the hereditary bleeding disorders and can affect males and females alike. It results from a factor VIII deficiency and platelet dysfunction. Although factor VIII levels increase naturally during pregnancy, there is an increased risk for postpartum hemorrhage from birth until 4 weeks after delivery as levels of von Willebrand factor (vWf) and factor VIII decrease. The treatment that should be considered first for the client with von Willebrand disease who experiences a postpartum hemorrhage is: a. Cryoprecipitate. b. Factor VIII and vWf. c. Desmopressin. d. Hemabate.

ANS: C Desmopressin is the primary treatment of choice. This hormone can be administered orally, nasally, and intravenously. This medication promotes the release of factor VIII and vWf from storage. Cryoprecipitate may be used; however, because of the risk of possible donor viruses, other modalities are considered safer. Treatment with plasma products such as factor VIII and vWf is an acceptable option for this client. Because of the repeated exposure to donor blood products and possible viruses, this is not the initial treatment of choice. Although the administration of this prostaglandin is known to promote contraction of the uterus during postpartum hemorrhage, it is not effective for the client who presents with a bleeding disorder. PTS: 1 DIF: Cognitive Level: Application REF: 538 OBJ: Nursing Process: Implementation MSC: Client Needs: Physiologic Integrity

11. A nurse is caring for a child who has acute lymphocytic leukemia and has been treated with doxorubicin (Adriamycin). Which assessment finding would the nurse report immediately? A. Loss of appetite B. Low WBC count C. Peripheral edema D. Temperature of 100.6°F (38.1°C), once

ANS: C Doxorubicin and other anthracycline drugs are known to cause heart damage. Peripheral edema may signal heart failure and should be reported right away. Loss of appetite and low WBC count are common findings for a child on chemotherapy. A single temperature of 100.6°F does not need to be reported.

During a phone follow-up conversation with a woman who is 4 days postpartum, the woman tells the nurse, I dont know whats wrong. I love my son, but I feel so let down. I seem to cry for no reason! The nurse would recognize that the woman is experiencing: a. Taking-in. b. Postpartum depression (PPD). c. Postpartum (PP) blues. d. Attachment difficulty.

ANS: C During the PP blues women are emotionally labile, often crying easily and for no apparent reason. This lability seems to peak around the fifth PP day. The taking-in phase is the period after birth when the mother focuses on her own psychologic needs. Typically this period lasts 24 hours. PPD is an intense, pervasive sadness marked by severe, labile mood swings; it is more serious and persistent than the PP blues. Crying is not a maladaptive attachment response; it indicates PP blues. PTS: 1 DIF: Cognitive Level: Comprehension REF: 517 OBJ: Nursing Process: Assessment, Diagnosis MSC: Client Needs: Psychosocial Integrity

If the patients white blood cell (WBC) count is 25,000/mm on her second postpartum day, the nurse should: a. Tell the physician immediately. b. Have the laboratory draw blood for reanalysis. c. Recognize that this is an acceptable range at this point postpartum. d. Begin antibiotic therapy immediately.

ANS: C During the first 10 to 12 days after childbirth, values between 20,000 and 25,000/mm are common. Because this is a normal finding there is no reason to alert the physician. There is no need for reassessment or antibiotics because it is expected for the WBCs to be elevated. PTS: 1 DIF: Cognitive Level: Knowledge REF: 488 OBJ: Nursing Process: Implementation MSC: Client Needs: Health Promotion and Maintenance

On observing a woman on her first postpartum day sitting in bed while her newborn lies awake in the bassinet, the nurse should: a. Realize that this situation is perfectly acceptable. b. Offer to hand the baby to the woman. c. Hand the baby to the woman. d. Explain taking in to the woman.

ANS: C During the taking-in phase of maternal adaptation (the mother may be passive and dependent), the nurse should encourage bonding when the infant is in the quiet alert stage. This is done best by simply giving the baby to the mother. The patient is exhibiting expected behavior during the taking-in phase; however, interventions by the nurse can facilitate infant bonding. The patient will learn best during the taking-hold phase. PTS: 1 DIF: Cognitive Level: Application REF: 516 OBJ: Nursing Process: Implementation MSC: Client Needs: Psychosocial Integrity

As relates to the condition and reconditioning of the urinary system after childbirth, nurses should be aware that: a. Kidney function returns to normal a few days after birth. b. Diastasis recti abdominis is a common condition that alters the voiding reflex. c. Fluid loss through perspiration and increased urinary output accounts for a weight loss of more than 2 kg during the puerperium. d. With adequate emptying of the bladder, bladder tone usually is restored 2 to 3 weeks after childbirth.

ANS: C Excess fluid loss through other means occurs as well. Kidney function usually returns to normal in about a month. Diastasis recti abdominis is the separation of muscles in the abdominal wall; it has no effect on the voiding reflex. Bladder tone usually is restored 5 to 7 days after childbirth. PTS: 1 DIF: Cognitive Level: Comprehension REF: 486 OBJ: Nursing Process: Planning MSC: Client Needs: Physiologic Integrity

An infant is to receive gastrostomy feedings. What intervention should the nurse institute to prevent bloating, gastrointestinal reflux into the esophagus, vomiting, and respiratory compromise? a. Rapid bolusing of the entire amount in 15 minutes b. Warm cloths to the abdomen for the first 10 minutes c. Slow, small, warm bolus feedings over 30 minutes d. Cold, medium bolus feedings over 20 minutes

ANS: C Feedings by gravity are done slowly over 20- to 30-minute periods to prevent adverse reactions. Rapid bolusing of the entire amount in 15 minutes would most likely lead to the adverse reactions listed. Temperature stability in the newborn is critical. Warm cloths to the abdomen for the first 10 minutes would not be appropriate because it is not a thermoregulated environment. Additionally, abdominal warming is not indicated with feedings of any kind. Small feedings at room temperature are recommended to prevent adverse reactions. PTS: 1 DIF: Cognitive Level: Application REF: 700 OBJ: Nursing Process: Implementation MSC: Client Needs: Physiologic Integrity

32. The nurse is teaching the parent about the diet of a child experiencing severe edema associated with acute glomerulonephritis. Which information should the nurse include in the teaching? a. "You will need to decrease the number of calories in your child's diet." b. "Your child's diet will need an increased amount of protein." c. "You will need to avoid adding salt to your child's food." d. "Your child's diet will consist of low-fat, low-carbohydrate foods."

ANS: C For most children, a regular diet is allowed, but it should contain no added salt. The child should be offered a regular diet with favorite foods. Severe sodium restrictions are not indicated.

How many kilocalories per kilogram (kcal/kg) of body weight does a breastfed term infant require each day? a. 50 to 65 b. 75 to 90 c. 95 to 110 d. 150 to 200

ANS: C For the first 3 months the infant needs 110 kcal/kg/day. At ages 3 to 6 months the requirement is 100 kcal/kg/day. This level decreases slightly to 95 kcal/kg/day from 6 to 9 months and increases again to 100 kcal/kg/day until the baby reaches 12 months. PTS: 1 DIF: Cognitive Level: Knowledge REF: 636 OBJ: Nursing Process: Assessment MSC: Client Needs: Physiologic Integrity

24. A nurse is teaching parents of first-grade children general guidelines to assist their children in adapting to school. Which statement by the parents indicates they understand the teaching? a. "We will only meet with the teacher if problems occur." b. "We will discourage hobbies so our child focuses on schoolwork." c. "We will plan a trip to the library as often as possible." d. "We will expect our child to make all As in school."

ANS: C General guidelines for parents to help their child in school include sharing an interest in reading. The library should be used frequently and books the child is reading should be discussed. Hobbies should be encouraged. The parents should not expect all As. They should focus on growth more than grades.

13.The most appropriate nursing diagnosis for the child with acute glomerulonephritis is: a. Risk for Injury related to malignant process and treatment. b. Deficient Fluid Volume related to excessive losses. c. Excess Fluid Volume related to decreased plasma filtration. d. Excess Fluid Volume related to fluid accumulation in tissues and third spaces.

ANS: C Glomerulonephritis has a decreased filtration of plasma. The decrease in plasma filtration results in an excessive accumulation of water and sodium that expands plasma and interstitial fluid volumes, leading to circulatory congestion and edema. No malignant process is involved in acute glomerulonephritis. A fluid volume excess is found. The fluid accumulation is secondary to the decreased plasma filtration, not fluid accumulation.

As part of Standard Precautions, nurses wear gloves when handling the newborn. The chief reason is: a. To protect the baby from infection. b. That it is part of the Apgar protocol. c. To protect the nurse from contamination by the newborn. d. the nurse has primary responsibility for the baby during the first 2 hours.

ANS: C Gloves are worn to protect the nurse from infection until the blood and amniotic fluid are cleaned off the newborn. PTS: 1 DIF: Cognitive Level: Comprehension REF: 613 OBJ: Nursing Process: Implementation MSC: Client Needs: Safe and Effective Care Environment

The most common clinical manifestation(s) of brain tumors in children is/are: a. irritability. b. seizures. c. headaches and vomiting. d. fever and poor fine motor control.

ANS: C Headaches, especially on awakening, and vomiting that is not related to feeding are the most common clinical manifestation(s) of brain tumors in children. Irritability, seizures, and fever and poor fine motor control are clinical manifestations of brain tumors, but headaches and vomiting are the most common.

As related to the eventual discharge of the high risk newborn or transfer to a different facility, nurses and families should be aware that: a. Infants will stay in the neonatal intensive care unit (NICU) until they are ready to go home. b. Once discharged to home, the high risk infant should be treated like any healthy term newborn. c. Parents of high risk infants need special support and detailed contact information. d. If a high risk infant and mother need transfer to a specialized regional center, it is better to wait until after birth and the infant is stabilized.

ANS: C High risk infants can cause profound parental stress and emotional turmoil. Parents need support, special teaching, and quick access to various resources available to help them care for their baby. Parents and their high risk infant should spend a night or two in a predischarge room, where care for the infant is provided away from the NICU. Just because high risk infants are discharged does not mean that they are normal, healthy babies. Follow-up by specialized practitioners is essential. Ideally, the mother and baby are transported with the fetus in utero; this reduces neonatal morbidity and mortality. PTS: 1 DIF: Cognitive Level: Comprehension REF: 664 OBJ: Nursing Process: Planning MSC: Client Needs: Psychosocial Integrity

With regard to the nutrient needs of breastfed and formula-fed infants, nurses should be understand that: a. Breastfed infants need extra water in hot climates. b. During the first 3 months breastfed infants consume more energy than do formula-fed infants. c. Breastfeeding infants should receive oral vitamin D drops daily at least during the first 2 months. d. Vitamin K injections at birth are not needed for infants fed on specially enriched formula.

ANS: C Human milk contains only small amounts of vitamin D. Neither breastfed nor formula-fed infants need to be given water, even in very hot climates. During the first 3 months formula-fed infants consume more energy than do breastfed infants and therefore tend to grow more rapidly. Vitamin K shots are required for all infants because the bacteria that produce it are absent from the babys stomach at birth. PTS: 1 DIF: Cognitive Level: Comprehension REF: 637 OBJ: Nursing Process: Planning MSC: Client Needs: Physiologic Integrity

An infant was born 2 hours ago at 37 weeks of gestation and weighing 4.1 kg. The infant appears chubby with a flushed complexion and is very tremulous. The tremors are most likely the result of: a. Birth injury. b. Hypocalcemia. c. Hypoglycemia. d. Seizures.

ANS: C Hypoglycemia is common in the macrosomic infant. Signs of hypoglycemia include jitteriness, apnea, tachypnea, and cyanosis. PTS: 1 DIF: Cognitive Level: Comprehension REF: 682 OBJ: Nursing Process: Assessment MSC: Client Needs: Physiologic Integrity

A school nurse is conducting a staff in-service to other school nurses on idiopathic scoliosis. During which period of child development does idiopathic scoliosis become most noticeable? a. Newborn period b. When child starts to walk c. Preadolescent growth spurt d. Adolescence

ANS: C Idiopathic scoliosis is most noticeable during the preadolescent growth spurt. Idiopathic scoliosis is seldom apparent before age 10 years. Diagnosis usually occurs during the preadolescent growth spurt.

A recently delivered mother and her baby are at the clinic for a 6-week postpartum checkup. The nurse should be concerned that psychosocial outcomes are not being met if the woman: a. Discusses her labor and birth experience excessively. b. Believes that her baby is more attractive and clever than any others. c. Has not given the baby a name. d. Has a partner or family members who react very positively about the baby.

ANS: C If the mother is having difficulty naming her new infant, it may be a signal that she is not adapting well to parenthood. Other red flags include refusal to hold or feed the baby, lack of interaction with the infant, and becoming upset when the baby vomits or needs a diaper change. A new mother who is having difficulty would be unwilling to discuss her labor and birth experience. An appropriate nursing diagnosis could be Impaired parenting related to a long, difficult labor or unmet expectations of birth. A mother who is willing to discuss her birth experience is making a healthy personal adjustment. The mother who is not coping well would find her baby unattractive and messy. She may also be overly disappointed in the babys sex. The client may voice concern that the baby reminds her of a family member whom she does not like. Having a partner and/or other family members react positively is an indication that this new mother has a good support system in place. This support system will help reduce anxiety related to her new role as a mother. PTS: 1 DIF: Cognitive Level: Synthesis REF: 504 OBJ: Nursing Process: Evaluation MSC: Client Needs: Psychosocial Integrity

Postpartal overdistention of the bladder and urinary retention can lead to which complications? a. Postpartum hemorrhage and eclampsia b. Fever and increased blood pressure c. Postpartum hemorrhage and urinary tract infection d. Urinary tract infection and uterine rupture

ANS: C Incomplete emptying and overdistention of the bladder can lead to urinary tract infection. Overdistention of the bladder displaces the uterus and prevents contraction of the uterine muscle, thus leading to postpartum hemorrhage. There is no correlation between bladder distention and high blood pressure or eclampsia. The risk of uterine rupture decreases after the birth of the infant. PTS: 1 DIF: Cognitive Level: Comprehension REF: 496 OBJ: Nursing Process: Assessment MSC: Client Needs: Physiologic Integrity

19. A nurse is looking at photographs of a friend's infant. The nurse notes a whitish glow in the child's eyes, and the friend asks why the baby's eyes look so odd. Which response by the nurse is the most appropriate? A. "If his eyes look like this by 6 months, he needs to see a doctor." B. "Take him to the doctor to see what's wrong with his eyes." C. "This is called leukocoria and may signify retinoblastoma." D. "Your baby may have a brain tumor; take him to the hospital."

ANS: C Leukocoria (also known as the cat's-eye reflex) is a whitish glow in the pupil, often noticed on photographs, and is seen in children with retinoblastoma. The child needs to be seen by his health-care provider. The mother should not wait 6 months. Advising the mother to find out what's wrong with his eyes is not as accurate as explaining the manifestation. This sign is not seen in brain tumors.

What infection is contracted mostly by first-time mothers who are breastfeeding? a. Endometritis b. Wound infections c. Mastitis d. Urinary tract infections

ANS: C Mastitis is infection in a breast, usually confined to a milk duct. Most women who suffer this are primiparas who are breastfeeding. PTS: 1 DIF: Cognitive Level: Knowledge REF: 540 OBJ: Nursing Process: Diagnosis MSC: Client Needs: Health Promotion and Maintenance

36. A child is receiving chemotherapy. The nurse assesses the child's oral cavity and notes the following: raspy voice, thick saliva, and debris on the teeth. Which action by the nurse is the most appropriate? A. Have the child use commercial mouthwash. B. Hold the next dose of chemotherapy. C. Increase the frequency of oral care. D. Place the child on NPO status.

ANS: C Mucositis is a diffuse inflammation of the mouth and oral mucous membranes, and is common during chemotherapy. The nurse should increase the frequency of oral care in the child who is manifesting signs of this problem. Commercial mouthwash contains alcohol, which would burn the tissues. The chemotherapy would not be interrupted. The child should be encouraged to eat and drink as tolerated.

Providing care for the neonate born to a mother who abuses substances can present a challenge for the health care team. Nursing care for this infant requires a multisystem approach. The first step in the provision of this care is: a. Pharmacologic treatment. b. Reduction of environmental stimuli. c. Neonatal abstinence syndrome scoring. d. Adequate nutrition and maintenance of fluid and electrolyte balance.

ANS: C Neonatal abstinence syndrome (NAS) is the term used to describe the cohort of symptoms associated with drug withdrawal in the neonate. The Neonatal Abstinence Scoring System evaluates central nervous system (CNS), metabolic, vasomotor, respiratory, and gastrointestinal disturbances. This evaluation tool enables the care team to develop an appropriate plan of care. The infant is scored throughout the length of stay, and the treatment plan is adjusted accordingly. Pharmacologic treatment is based on the severity of withdrawal symptoms. Symptoms are determined by using a standard assessment tool. Medications of choice are morphine, phenobarbital, diazepam, or diluted tincture of opium. Swaddling, holding, and reducing environmental stimuli are essential in providing care to the infant who is experiencing withdrawal. These nursing interventions are appropriate for the infant who displays CNS disturbances. Poor feeding is one of the gastrointestinal symptoms common to this client population. Fluid and electrolyte balance must be maintained and adequate nutrition provided. These infants often have a poor suck reflex and may need to be fed via gavage. PTS: 1 DIF: Cognitive Level: Application REF: 670 OBJ: Nursing Process: Assessment MSC: Client Needs: Physiologic Integrity

30. A child needs surgery to resect a tumor, but is scheduled for several weeks of radiation therapy first. The parents are frustrated and want to know why the surgery that can cure the cancer is being delayed. Which response by the nurse is the most appropriate? A. "Children who have radiation first generally do better than others." B. "If the radiation destroys the tumor, surgery will not be needed." C. "Radiation will shrink the tumor, making it easier to get all of it out." D. "The surgeons must be worried that they cannot get the whole tumor."

ANS: C Often radiation or chemotherapy is used prior to surgical resection to shrink the size of the tumor, maximizing the chances of complete removal. The other responses are not accurate.

26. A nurse planning care for a school-age child should take into account that which thought process is seen at this age? a. Animism b. Magical thinking c. Ability to conserve d. Thoughts are all-powerful

ANS: C One cognitive task of school-age children is mastering the concept of conservation. At an early age (5 to 7 years), children grasp the concept of reversibility of numbers as a basis for simple mathematics problems (e.g., 2 + 4 = 6 and 6 − 4 = 2). They learn that simply altering their arrangement in space does not change certain properties of the environment, and they are able to resist perceptual cues that suggest alterations in the physical state of an object. Animism, magical thinking, and believing that thoughts are all-powerful are thought processes seen in preschool children.

8. A group of boys ages 9 and 10 years have formed a "boys-only" club that is open to neighborhood and school friends who have skateboards. How should this behavior be interpreted? a. Behavior that encourages bullying and sexism. b. Behavior that reinforces poor peer relationships. c. Characteristic of social development of this age. d. Characteristic of children who later are at risk for membership in gangs.

ANS: C One of the outstanding characteristics of middle childhood is the creation of formalized groups or clubs. Peer-group identification and association are essential to a child's socialization. Poor relationships with peers and a lack of group identification can contribute to bullying. A boys-only club does not have a direct correlation with later gang activity.

The nurse is taking care of a 10-year-old child who has osteomyelitis. Which treatment plan is considered the primary method of treating osteomyelitis? a. Joint replacement b. Bracing and casting c. Intravenous antibiotic therapy d. Long-term corticosteroid therapy

ANS: C Osteomyelitis is an infection of the bone, most commonly caused by Staphylococcus aureus. The treatment of choice is antibiotics. Joint replacement, bracing and casting, and long-term corticosteroid therapy are not indicated for infectious processes.

The early postpartum period is a time of emotional and physical vulnerability. Many mothers can easily become psychologically overwhelmed by the reality of their new parental responsibilities. Fatigue compounds these issues. Although the baby blues are a common occurrence in the postpartum period, about one-half million women in America experience a more severe syndrome known as postpartum depression (PPD). Which statement regarding PPD is essential for the nurse to be aware of when attempting to formulate a nursing diagnosis? a. PPD symptoms are consistently severe. b. This syndrome affects only new mothers. c. PPD can easily go undetected. d. Only mental health professionals should teach new parents about this condition.

ANS: C PPD can go undetected because parents do not voluntarily admit to this type of emotional distress out of embarrassment, fear, or guilt. PPD symptoms range from mild to severe, with women having both good and bad days. Both mothers and fathers should be screened. PPD may also affect new fathers. The nurse should include information on PPD and how to differentiate this from the baby blues for all clients on discharge. Nurses also can urge new parents to report symptoms and seek follow-up care promptly if symptoms occur. PTS: 1 DIF: Cognitive Level: Synthesis REF: 517 OBJ: Nursing Process: Diagnosis MSC: Client Needs: Psychosocial Integrity

22. A parent brings a 10-year-old child to the clinic, reporting that the child fell while playing and now has a limp several days later. In completing a history, which other finding would the nurse correlate more with bone cancer than a minor trauma? A. Decreased appetite for the last month B. Fatigues easily when playing outdoors C. Limping several weeks prior to the fall D. Often has unexplained extremity bruises

ANS: C Pain and swelling are the most common manifestations of osteosarcoma. Often the child has a limp. The child also may have a dull pain at the tumor site, and if it is on a leg (weight-bearing), it could easily cause a limp that has lasted for several weeks before really being noticed. The other manifestations are vague and could be related to other problems.

Complicated bereavement: a. Occurs when, in multiple births, one child dies, and the other or others live. b. Is a state in which the parents are ambivalent, as with an abortion. c. Is an extremely intense grief reaction that persists for a long time. d. Is felt by the family of adolescent mothers who lose their babies.

ANS: C Parents showing signs of complicated grief should be referred for counseling. Multiple births in which not all the babies survive creates a complicated parenting situation, but this is not complicated bereavement. Abortion can generate complicated emotional responses, but they do not constitute complicated bereavement. Families of lost adolescent pregnancies may have to deal with complicated issues, but this is not complicated bereavement. PTS: 1 DIF: Cognitive Level: Comprehension REF: 553 OBJ: Nursing Process: Diagnosis MSC: Client Needs: Psychosocial Integrity

What marks on a babys skin may indicate an underlying problem that requires notification of a physician? a. Mongolian spots on the back b. Telangiectatic nevi on the nose or nape of the neck c. Petechiae scattered over the infants body d. Erythema toxicum anywhere on the body

ANS: C Petechiae (bruises) scattered over the infants body should be reported to the pediatrician because they may indicate underlying problems. Mongolian spots are bluish-black spots that resemble bruises but fade gradually over months and have no clinical significance. Telangiectatic nevi (stork bites, angel kisses) fade by the second year and have no clinical significance. Erythema toxicum is an appalling-looking rash, but it has no clinical significance and requires no treatment. PTS: 1 DIF: Cognitive Level: Comprehension REF: 568 OBJ: Nursing Process: Assessment MSC: Client Needs: Physiologic Integrity

28. Which description of a stool is characteristic of intussusception? a. Ribbon-like stools b. Hard stools positive for guaiac c. "Currant jelly" stools d. Loose, foul-smelling stools

ANS: C With intussusception, passage of bloody mucus-coated stools occurs. Pressure on the bowel from obstruction leads to passage of "currant jelly" stools. Ribbon-like stools are characteristic of Hirschsprung's disease. Stools will not be hard. Loose, foul-smelling stools may indicate infectious gastroenteritis.

For clinical purposes, preterm and post-term infants are defined as: a. Preterm before 34 weeks if appropriate for gestational age (AGA) and before 37 weeks if small for gestational age (SGA). b. Post-term after 40 weeks if large for gestational age (LGA) and beyond 42 weeks if AGA. c. Preterm before 37 weeks, and post-term beyond 42 weeks, no matter the size for gestational age at birth. d. Preterm, SGA before 38 to 40 weeks, and post-term, LGA beyond 40 to 42 weeks.

ANS: C Preterm and post-term are strictly measures of timebefore 37 weeks and beyond 42 weeks, respectivelyregardless of size for gestational age. PTS: 1 DIF: Cognitive Level: Comprehension REF: 664 OBJ: Nursing Process: Diagnosis MSC: Client Needs: Health Promotion and Maintenance

18. A nurse sees the term "proptosis" in a child's medical record. Which physical assessment does the nurse plan to incorporate into the child's exam based on this finding? A. Balance testing B. Hearing screen C. Visual acuity D. Strength testing

ANS: C Proptosis is a downward displacement of the eyeball that can affect visual acuity and is frequently seen in children with rhabdomyosarcoma. The other assessments are not related.

2. Generally what is the earliest age at which puberty begins? a. 13 years in girls, 13 years in boys b. 11 years in girls, 11 years in boys c. 10 years in girls, 12 years in boys d. 12 years in girls, 10 years in boys

ANS: C Puberty signals the beginning of the development of secondary sex characteristics. This begins in girls earlier than in boys. Usually a 2-year difference occurs in the age at onset. Girls and boys do not usually begin puberty at the same age; girls usually begin earlier than boys do.

Which type of formula is not diluted before being administered to an infant? a. Powdered b. Concentrated c. Ready-to-use d. Modified cows milk

ANS: C Ready-to-use formula can be poured directly from the can into babys bottle and is good (but expensive) when a proper water supply is not available. Formula should be well mixed to dissolve the powder and make it uniform in consistency. Improper dilution of concentrated formula may cause malnutrition or sodium imbalances. Cows milk is more difficult for the infant to digest and is not recommended, even if it is diluted. PTS: 1 DIF: Cognitive Level: Comprehension REF: 657 OBJ: Nursing Process: Assessment MSC: Client Needs: Physiologic Integrity

A neonate is born with bilateral mild talipes equinovarus (clubfoot). When the parents ask the nurse how this will be corrected, the nurse should give which explanation? a. Traction is tried first. b. Surgical intervention is needed. c. Frequent, serial casting is tried first. d. Children outgrow this condition when they learn to walk.

ANS: C Serial casting is begun shortly after birth before discharge from nursery. Successive casts allow for gradual stretching of skin and tight structures on the medial side of the foot. Manipulation and casting of the leg are repeated frequently (every week) to accommodate the rapid growth of early infancy. Serial casting is the preferred treatment. Surgical intervention is done only if serial casting is not successful. Children do not improve without intervention.

In caring for the mother who has abused (or is abusing) alcohol and for her infant, nurses should be aware that: a. The pattern of growth restriction of the fetus begun in prenatal life is halted after birth, and normal growth takes over. b. Two thirds of newborns with fetal alcohol syndrome (FAS) are boys. c. Alcohol-related neurodevelopmental disorders not sufficient to meet FAS criteria (learning disabilities, speech and language problems) are often not detected until the child goes to school. d. Both the distinctive facial features of the FAS infant and the diminished mental capacities tend toward normal over time.

ANS: C Some learning problems do not become evident until the child is at school. The pattern of growth restriction persists after birth. Two thirds of newborns with FAS are girls. Although the distinctive facial features of the FAS infant tend to become less evident, the mental capacities never become normal. PTS: 1 DIF: Cognitive Level: Comprehension REF: 676 OBJ: Nursing Process: Planning MSC: Client Needs: Health Promotion and Maintenance

33. Which intervention for treating croup at home should be taught to parents? a. Have a decongestant available to give the child when an attack occurs. b. Have the child sleep in a dry room. c. Take the child outside. d. Give the child an antibiotic at bedtime.

ANS: C Taking the child into the cool, humid, night air may relieve mucosal swelling and improve symptoms. Decongestants are inappropriate for croup, which affects the middle airway level. A dry environment may contribute to symptoms. Croup is caused by a virus. Antibiotic treatment is not indicated

20. Identify the statement that is the most accurate about moral development in the 9-year-old school-age child. a. Right and wrong are based on physical consequences of behavior. b. The child obeys parents because of fear of punishment. c. The school-age child conforms to rules to please others. d. Parents are the determiners of right and wrong for the school-age child.

ANS: C The 7- to 12-year-old child bases right and wrong on a good-boy or good-girl orientation in which the child conforms to rules to please others and avoid disapproval. Children 4 to 7 years of age base right and wrong on consequences, the most important consideration for this age-group. Parents determine right and wrong for the child younger than 4 years of age.

Near the end of the first week of life, an infant who has not been treated for any infection develops a copper-colored, maculopapular rash on the palms and around the mouth and anus. The newborn is showing signs of: a. Gonorrhea. b. Herpes simplex virus infection. c. Congenital syphilis. d. Human immunodeficiency virus.

ANS: C The rash is indicative of congenital syphilis. The lesions may extend over the trunk and extremities. PTS: 1 DIF: Cognitive Level: Analysis REF: 673 OBJ: Nursing Process: Diagnosis MSC: Client Needs: Physiologic Integrity

An infant is being discharged from the neonatal intensive care unit after 70 days of hospitalization. The infant was born at 30 weeks of gestation with several conditions associated with prematurity, including respiratory distress syndrome, mild bronchopulmonary dysplasia, and retinopathy of prematurity requiring surgical treatment. During discharge teaching the infants mother asks the nurse whether her baby will meet developmental milestones on time, as did her son who was born at term. The nurses most appropriate response is: a. Your baby will develop exactly like your first child did. b. Your baby does not appear to have any problems at the present time. c. Your baby will need to be corrected for prematurity. Your baby is currently 40 weeks of postconceptional age and can be expected to be doing what a 40-week-old infant would be doing. d. Your baby will need to be followed very closely.

ANS: C The age of a preterm newborn is corrected by adding the gestational age and the postnatal age. The infants responses are evaluated accordingly against the norm expected for the corrected age of the infant. Although it is impossible to predict with complete accuracy the growth and development potential of each preterm infant, certain measurable factors predict normal growth and development. The preterm infant experiences catch-up body growth during the first 2 to 3 years of life. The growth and developmental milestones are corrected for gestational age until the child is approximately 2.5 years old. Stating that the baby does not appear to have any problems at the present time is inaccurate. Development will need to be evaluated over time. PTS: 1 DIF: Cognitive Level: Application REF: 676 OBJ: Nursing Process: Planning MSC: Client Needs: Health Promotion and Maintenance

Which documentation on a womans chart on postpartum day 14 indicates a normal involution process? a. Moderate bright red lochial flow b. Breasts firm and tender c. Fundus below the symphysis and not palpable d. Episiotomy slightly red and puffy

ANS: C The fundus descends 1 cm/day, so by postpartum day 14 it is no longer palpable. The lochia should be changed by this day to serosa. Breasts are not part of the involution process. The episiotomy should not be red or puffy at this stage. PTS: 1 DIF: Cognitive Level: Knowledge REF: 483 OBJ: Nursing Process: Assessment MSC: Client Needs: Physiologic Integrity

20. It is now recommended that children with asthma who are taking long-term inhaled steroids should be assessed frequently because they may develop: a. Cough. c. Slowed growth. b. Osteoporosis. d. Cushings syndrome.

ANS: C The growth of children on long-term inhaled steroids should be assessed frequently to assess for systemic effects of these drugs. Cough is prevented by inhaled steroids. No evidence e

As a result of large body surface in relation to weight, the preterm infant is at high risk for heat loss and cold stress. By understanding the four mechanisms of heat transfer (convection, conduction, radiation, and evaporation), the nurse can create an environment for the infant that prevents temperature instability. While evaluating the plan that has been implemented, the nurse knows that the infant is experiencing cold stress when he or she exhibits: a. Decreased respiratory rate. b. Bradycardia followed by an increased heart rate. c. Mottled skin with acrocyanosis. d. Increased physical activity.

ANS: C The infant has minimal to no fat stores. During times of cold stress the skin will become mottled, and acrocyanosis will develop, progressing to cyanosis. Even if the infant is being cared for on a radiant warmer or in an isolette, the nurses role is to observe the infant frequently to prevent heat loss and respond quickly if signs and symptoms occur. The respiratory rate increases followed by periods of apnea. The infant initially tries to conserve heat and burns more calories, after which the metabolic system goes into overdrive. In the preterm infant experiencing heat loss, the heart rate initially increases, followed by periods of bradycardia. In the term infant, the natural response to heat loss is increased physical activity. However, in a term infant experiencing respiratory distress or in a preterm infant, physical activity is decreased. PTS: 1 DIF: Cognitive Level: Analysis REF: 696 OBJ: Nursing Process: Evaluation MSC: Client Needs: Physiologic Integrity

In assisting the breastfeeding mother position the baby, nurses should keep in mind that: a. The cradle position usually is preferred by mothers who had a cesarean birth. b. Women with perineal pain and swelling prefer the modified cradle position. c. Whatever the position used, the infant is belly to belly with the mother. d. While supporting the head, the mother should push gently on the occiput.

ANS: C The infant inevitably faces the mother, belly to belly. The football position usually is preferred after cesarean nbirth. Women with perineal pain and swelling prefer the side-lying position because they can rest while breastfeeding. The mother should never push on the back of the head. It may cause the baby to bite, hyperextend the neck, or develop an aversion to being brought near the breast. PTS: 1 DIF: Cognitive Level: Application REF: 641 OBJ: Nursing Process: Implementation MSC: Client Needs: Physiologic Integrity

A plan of care for an infant experiencing symptoms of drug withdrawal should include: a. Administering chloral hydrate for sedation. b. Feeding every 4 to 6 hours to allow extra rest. c. Swaddling the infant snugly and holding the baby tightly. d. Playing soft music during feeding.

ANS: C The infant should be wrapped snugly to reduce self-stimulation behaviors and protect the skin from abrasions. Phenobarbital or diazepam may be administered to decrease central nervous system (CNS) irritability. The infant should be fed in small, frequent amounts and burped well to diminish aspiration and maintain hydration. The infant should not be stimulated (such as with music) because this will increase activity and potentially increase CNS irritability. PTS: 1 DIF: Cognitive Level: Application REF: 675 OBJ: Nursing Process: Implementation MSC: Client Needs: Physiologic Integrity

A newborn is jaundiced and receiving phototherapy via ultraviolet bank lights. An appropriate nursing intervention when caring for an infant with hyperbilirubinemia and receiving phototherapy by this method would be to: a. Apply an oil-based lotion to the newborns skin to prevent dying and cracking. b. Limit the newborns intake of milk to prevent nausea, vomiting, and diarrhea. c. Place eye shields over the newborns closed eyes. d. Change the newborns position every 4 hours.

ANS: C The infants eyes must be protected by an opaque mask to prevent overexposure to the light. Eye shields should cover the eyes completely but not occlude the nares. Lotions and ointments should not be applied to the infant because they absorb heat, and this can cause burns. The lights increase insensible water loss, placing the infant at risk for fluid loss and dehydration. Therefore, it is important that the infant be adequately hydrated. The infant should be turned every 2 hours to expose all body surfaces to the light. PTS: 1 DIF: Cognitive Level: Application REF: 605 OBJ: Nursing Process: Planning MSC: Client Needs: Safe and Effective Care Environment

The first and most important nursing intervention when a nurse observes profuse postpartum bleeding is to: a. Call the womans primary health care provider. b. Administer the standing order for an oxytocic. c. Palpate the uterus and massage it if it is boggy. d. Assess maternal blood pressure and pulse for signs of hypovolemic shock.

ANS: C The initial management of excessive postpartum bleeding is firm massage of the uterine fundus. Although calling the health care provider, administering an oxytocic, and assessing maternal BP are appropriate interventions, the primary intervention should be to assess the uterus. Uterine atony is the leading cause of postpartum hemorrhage (PPH). PTS: 1 DIF: Cognitive Level: Application REF: 532 OBJ: Nursing Process: Implementation MSC: Client Needs: Physiologic Integrity

The nurse is caring for a school-age child diagnosed with juvenile idiopathic arthritis (JIA). Which intervention should be a priority? a. Apply ice packs to relieve stiffness and pain. b. Administer acetaminophen to reduce inflammation. c. Teach the child and family correct administration of medications. d. Encourage range-of-motion exercises during periods of inflammation.

ANS: C The management of JIA is primarily pharmacologic. The family should be instructed regarding administration of medications and the value of regular schedule of administration to maintain a satisfactory blood level in the body. They need to know that NSAIDs should not be given on an empty stomach and to be alert for signs of toxicity. Warm moist heat is best for relieving stiffness and pain. Acetaminophen does not have antiinflammatory effects. Range-of-motion exercises should not be done during periods of inflammation.

4. A nurse hears that a new admission to the hospital was recently diagnosed with the most common kind of childhood cancer. Which collaborative care does the nurse prepare to provide to this patient? A. Antibiotic administration B. Bone marrow transplant C. Chemotherapy D. Liver transplant

ANS: C The most common type of childhood cancer is acute lymphocytic leukemia (ALL). First-line treatment for ALL is inducing remission with chemotherapy. Antibiotics are not used unless the child has an infection. Bone marrow transplant may be considered later in the child's course of care. A liver transplant would not be a treatment for ALL.

29. The nurse is admitting a school-age child in acute renal failure with reduced glomerular filtration rate. Which urine test is the most useful clinical indication of glomerular filtration rate? a. pH b. Osmolality c. Creatinine clearance d. Protein level

ANS: C The most useful clinical indication of glomerular filtration is the clearance of creatinine. It is a substance that is freely filtered by the glomerulus and secreted by the renal tubule cells. The pH and osmolality are not estimates of glomerular filtration. Although protein in the urine demonstrates abnormal glomerular permeability, it is not a measure of filtration rate.

The normal term infant has little difficulty clearing the airway after birth. Most secretions are brought up to the oropharynx by the cough reflex. However, if the infant has excess secretions, the mouth and nasal passages can be cleared easily with a bulb syringe. When instructing parents on the correct use of this piece of equipment, it is important that the nurse teach them to: a. Avoid suctioning the nares. b. Insert the compressed bulb into the center of the mouth. c. Suction the mouth first. d. Remove the bulb syringe from the crib when finished.

ANS: C The mouth should be suctioned first to prevent the infant from inhaling pharyngeal secretions by gasping as the nares are suctioned. The nasal passages should be suctioned one nostril at a time. After compression of the bulb it should be inserted into one side of the mouth. If the bulb is inserted into the center of the mouth, the gag reflex is likely to be initiated. When the infants cry no longer sounds as though it is through mucus or a bubble, suctioning can be stopped. The bulb syringe should remain in the crib so that it is easily accessible if needed again. PTS: 1 DIF: Cognitive Level: Application REF: 602 OBJ: Nursing Process: Implementation MSC: Client Needs: Health Promotion and Maintenance

During the initial acute distress phase of grieving, parents still must make unexpected and unwanted decisions about funeral arrangements and even naming the baby. The nurses role should be to: a. Take over as much as possible to relieve the pressure. b. Encourage grandparents to take over. c. Make sure the parents themselves approve the final decisions. d. Let them alone to work things out.

ANS: C The nurse is always the clients advocate. Nurses can offer support and guidance and leave room for the same from grandparents. However, in the end nurses should strive to let the parents make the final decisions. PTS: 1 DIF: Cognitive Level: Comprehension REF: 553 OBJ: Nursing Process: Implementation MSC: Client Needs: Psychosocial Integrity

The nurse is performing a gestational age and physical assessment on the newborn. The infant appears to have an excessive amount of saliva. The nurse recognizes that this finding: a. Is normal. b. Indicates that the infant is hungry. c. May indicate that the infant has a tracheoesophageal fistula or esophageal atresia. d. May indicate that the infant has a diaphragmatic hernia.

ANS: C The presence of excessive saliva in a neonate should alert the nurse to the possibility of tracheoesophageal fistula or esophageal atresia. PTS: 1 DIF: Cognitive Level: Analysis REF: 595 OBJ: Nursing Process: Assessment MSC: Client Needs: Physiologic Integrity

13. What is the most common clinical manifestation(s) of brain tumors in children? a. Irritability b. Seizures c. Headaches and vomiting d. Fever and poor fine motor control

ANS: C Headaches, especially on awakening, and vomiting that is not related to feeding are the most common clinical manifestation(s) of brain tumors in children. Irritability, seizures, and fever and poor fine motor control are clinical manifestations of brain tumors, but headaches and vomiting are the most common.

Under the Newborns and Mothers Health Protection Act, all health plans are required to allow new mothers and newborns to remain in the hospital for a minimum of _____ hours after a normal vaginal birth and for _____ hours after a cesarean birth. a. 24, 73 b. 24, 96 c. 48, 96 d. 48, 120

ANS: C The specified stays are 48 hours (2 days) for a vaginal birth and 96 hours (4 days) for a cesarean birth. The attending provider and the mother together can decide on an earlier discharge. PTS: 1 DIF: Cognitive Level: Knowledge REF: 492 OBJ: Nursing Process: Planning MSC: Client Needs: Safe and Effective Care Environment

A nurse is conducting discharge teaching for parents of an infant with osteogenesis imperfecta (OI). Further teaching is indicated if the parents make which statement? a. "We will be very careful handling the baby." b. "We will lift the baby by the buttocks when diapering." c. "We're glad there is a cure for this disorder." d. "We will schedule follow-up appointments as instructed."

ANS: C The treatment for OI is primarily supportive. Although patients and families are optimistic about new research advances, there is no cure. The use of bisphosphonate therapy with IV pamidronate to promote increased bone density and prevent fractures has become standard therapy for many children with OI; however, long bones are weakened by prolonged treatment. Infants and children with this disorder require careful handling to prevent fractures. They must be supported when they are being turned, positioned, moved, and held. Even changing a diaper may cause a fracture in severely affected infants. These children should never be held by the ankles when being diapered but should be gently lifted by the buttocks or supported with pillows. Follow-up appointments for treatment with bisphosphonate can be expected.

A 25-year-old gravida 2, para 2-0-0-2 gave birth 4 hours ago to a 9-pound, 7-ounce boy after augmentation of labor with Pitocin. She puts on her call light and asks for her nurse right away, stating, Im bleeding a lot. The most likely cause of postpartum hemorrhage in this woman is: a. Retained placental fragments. b. Unrepaired vaginal lacerations. c. Uterine atony. d. Puerperal infection.

ANS: C This woman gave birth to a macrosomic boy after Pitocin augmentation. The most likely cause of bleeding 4 hours after delivery, combined with these risk factors, is uterine atony. Although retained placental fragments may cause postpartum hemorrhage, this typically would be detected in the first hour after delivery of the placenta and is not the most likely cause of hemorrhage in this woman. Although unrepaired vaginal lacerations may cause bleeding, they typically would occur in the period immediately after birth. Puerperal infection can cause subinvolution and subsequent bleeding; however, this typically would be detected 24 hours after delivery. PTS: 1 DIF: Cognitive Level: Analysis REF: 496 OBJ: Nursing Process: Assessment MSC: Client Needs: Health Promotion and Maintenance

When a woman is diagnosed with postpartum depression (PPD) with psychotic features, one of the main concerns is that she may: a. Have outbursts of anger. b. Neglect her hygiene. c. Harm her infant. d. Lose interest in her husband.

ANS: C Thoughts of harm to oneself or the infant are among the most serious symptoms of PPD and require immediate assessment and intervention. Although outbursts of anger, hygiene neglect, and loss of interest in her husband are attributable to PPD, the major concern would be the potential to harm herself or her infant. PTS: 1 DIF: Cognitive Level: Comprehension REF: 547 OBJ: Nursing Process: Diagnosis MSC: Client Needs: Psychosocial Integrity

The nurse is caring for a 12-year-old child with a left leg below the knee amputation (BKA). The child had the surgery 1 week ago. Which intervention should the nurse plan to implement for this child? a. Elevate the left stump on a pillow. b. Place ice pack on the stump. c. Encourage the child to use an overhead bed trapeze when repositioning. d. Replace the ace wrap covering the stump with a gauze dressing.

ANS: C Use of the overhead bed trapeze should be encouraged to begin to build up the arm muscles necessary for walking with crutches. Stump elevation may be used during the first 24 hours, but after this time, the extremity should not be left in this position because contractures in the proximal joint will develop and seriously hamper ambulation. Ice would not be an appropriate intervention and would decrease circulation to the stump. Stump shaping is done postoperatively with special elastic bandaging using a figure-eight bandage, which applies pressure in a cone-shaped fashion. This technique decreases stump edema, controls hemorrhage, and aids in developing desired contours so the child will bear weight on the posterior aspect of the skin flap rather than on the end of the stump. This wrap should not be replaced with a gauze dressing.

At a 2-month well-baby examination, it was discovered that a breastfed infant had only gained 10 ounces in the past 4 weeks. The mother and the nurse agree that, to gain weight faster, the infant needs to: a. Begin solid foods. b. Have a bottle of formula after every feeding. c. Add at least one extra breastfeeding session every 24 hours. d. Start iron supplements.

ANS: C Usually the solution to slow weight gain is to improve the feeding technique. Position and latch-on are evaluated, and adjustments are made. It may help to add a feeding or two in a 24-hour period. Solid foods should not be introduced to an infant for at least 4 to 6 months. Bottle-feeding may cause nipple confusion and limit the supply of milk. Iron supplements have no bearing on weight gain. PTS: 1 DIF: Cognitive Level: Application REF: 647 OBJ: Nursing Process: Planning, Implementation MSC: Client Needs: Physiologic Integrity

1. Which statement best describes idiopathic hypopituitarism? a. Growth is normal during the first 3 years of life. b. Weight is usually more retarded than height. c. Skeletal proportions are normal for age. d. Most of these children have subnormal intelligence.

ANS: C In children with idiopathic hypopituitarism, the skeletal proportions are normal. Growth is within normal limits for the first year of life. Height is usually more delayed than weight. Intelligence is not affected by hypopituitarism.

Excessive blood loss after childbirth can have several causes; the most common is: a. Vaginal or vulvar hematomas. b. Unrepaired lacerations of the vagina or cervix. c. Failure of the uterine muscle to contract firmly. d. Retained placental fragments.

ANS: C Uterine atony can best be thwarted by maintaining good uterine tone and preventing bladder distention. Although vaginal or vulvar hematomas, unpaired lacerations of the vagina or cervix, and retained placental fragments are possible causes of excessive blood loss, uterine muscle failure (uterine atony) is the most common cause. PTS: 1 DIF: Cognitive Level: Knowledge REF: 496 OBJ: Nursing Process: Evaluation MSC: Client Needs: Health Promotion and Maintenance

16. A nurse notes in a patient's medical record high levels of vanillymandelic acid (VMA). Based on this information, which condition does the nurse prepare to educate the patient and family about? A. Ewing's sarcoma B. Hodgkin's lymphoma C. Neuroblastoma D. Wilms' tumor

ANS: C VMA and homovanillic acid (HVA) are tests used to measure the level of catecholamine metabolites in the urine. Neuroblastomas typically secrete catecholamines, so high levels of either substance are indicative of neuroblastoma.

17. A nursing student is caring for a child diagnosed with Wilms' tumor. Which action by the student causes the faculty member to intervene? A. Assesses urinary output per protocol B. Involves the parents in the child's care C. Palpates the abdomen in all four quadrants D. Provides frequent nutritious snacks

ANS: C Wilms' tumor is a solid, encapsulated mass that can rupture with palpation. Once the child is diagnosed with this cancer, palpation of the child's abdomen is prohibited. The other actions are appropriate.

A primigravida has just delivered a healthy infant girl. The nurse is about to administer erythromycin ointment in the infants eyes when the mother asks, What is that medicine for? The nurse responds: a. It is an eye ointment to help your baby see you better. b. It is to protect your baby from contracting herpes from your vaginal tract. c. Erythromycin is given prophylactically to prevent a gonorrheal infection. d. This medicine will protect your babys eyes from drying out over the next few days.

ANS: C With the prophylactic use of erythromycin, the incidence of gonococcal conjunctivitis has declined to less than 0.5%. Eye prophylaxis is administered at or shortly after birth to prevent ophthalmia neonatorum. Erythromycin has no bearing on enhancing vision, is used to prevent an infection caused by gonorrhea, not herpes, and is not used for eye lubrication. PTS: 1 DIF: Cognitive Level: Application REF: 670 OBJ: Nursing Process: Planning MSC: Client Needs: Health Promotion and Maintenance

Two days ago a woman gave birth to a full-term infant. Last night she awakened several times to urinate and noted that her gown and bedding were wet from profuse diaphoresis. One mechanism for the diaphoresis and diuresis that this woman is experiencing during the early postpartum period is: a. Elevated temperature caused by postpartum infection. b. Increased basal metabolic rate after giving birth. c. Loss of increased blood volume associated with pregnancy. d. Increased venous pressure in the lower extremities.

ANS: C Within 12 hours of birth women begin to lose the excess tissue fluid that has accumulated during pregnancy. One mechanism for reducing these retained fluids is the profuse diaphoresis that often occurs, especially at night, for the first 2 or 3 days after childbirth. Postpartal diuresis is another mechanism by which the body rids itself of excess fluid. An elevated temperature would cause chills and may cause dehydration, not diaphoresis and diuresis. Diaphoresis and diuresis sometimes are referred to as reversal of the water metabolism of pregnancy, not as the basal metabolic rate. Postpartal diuresis may be caused by the removal of increased venous pressure in the lower extremities. PTS: 1 DIF: Cognitive Level: Comprehension REF: 486 OBJ: Nursing Process: Diagnosis MSC: Client Needs: Physiologic Integrity

According to demographic research, the woman least likely to breastfeed and therefore most likely to need education regarding the benefits and proper techniques of breastfeeding would be: a. A woman who is 30 to 35 years of age, Caucasian, and employed part time outside the home. b. A woman who is younger than 25 years of age, Hispanic, and unemployed. c. A woman who is younger than 25 years of age, African-American, and employed full time outside the home. d. A woman who is 35 years of age or older, Caucasian, and employed full time at home.

ANS: C Women least likely to breastfeed typically are younger than 25 years of age, have a lower income, are less educated, are employed full time outside the home, and are African-American. PTS: 1 DIF: Cognitive Level: Comprehension REF: 635 OBJ: Nursing Process: Assessment MSC: Client Needs: Health Promotion and Maintenance

28. A child diagnosed with hypopituitarism is being started on growth hormone (GH) therapy. Nursing considerations should be based on which information? a. Treatment is most successful if it is started during adolescence. b. Treatment is considered successful if children attain full stature by adulthood. c. Replacement therapy requires daily subcutaneous injections. d. Replacement therapy will be required throughout the child's lifetime.

ANS: C Additional support is required for children who require hormone replacement therapy, such as preparation for daily subcutaneous injections and education for self-management during the school-age years. Young children, obese children, and those who are severely GH deficient have the best response to therapy. When therapy is successful, children can attain their actual or near-final adult height at a slower rate than their peers. Replacement therapy is not needed after attaining final height. They are no longer GH deficient.

33. To help the adolescent deal with diabetes, the nurse must consider which characteristic of adolescence? a. Desire to be unique b. Preoccupation with the future c. Need to be perfect and similar to peers d. Need to make peers aware of the seriousness of hypoglycemic reactions

ANS: C Adolescence is a time when the individual wants to be perfect and similar to peers. Having diabetes makes adolescents different from their peers. Adolescents do not wish to be unique; they desire to fit in with the peer group and are usually not future oriented. Forcing peer awareness of the seriousness of hypoglycemic reactions would further alienate the adolescent with diabetes since the peer group would likely focus on the differences.

20. A parent asks the nurse why self-monitoring of blood glucose is being recommended for her child with diabetes. The nurse should base the explanation on what information? a. It is a less expensive method of testing. b. It is not as accurate as laboratory testing. c. Children need to learn to manage their diabetes. d. The parents are better able to manage the disease.

ANS: C Blood glucose self-management has improved diabetes management and can be used successfully by children from the time of diagnosis. Insulin dosages can be adjusted based on blood sugar results. Blood glucose monitoring is more expensive but provides improved management. It is as accurate as equivalent testing done in laboratories. The ability to self-test allows the child to balance diet, exercise, and insulin. The parents are partners in the process, but the child should be taught how to manage the disease.

15. Which statement is most descriptive of Meckel's diverticulum? a. It is more common in females than in males. b. It is acquired during childhood. c. Intestinal bleeding may be mild or profuse. d. Medical interventions are usually sufficient to treat the problem.

ANS: C Blood stools are often a presenting sign of Meckel's diverticulum. It is associated with mild-to-profuse intestinal bleeding. It is twice as common in males as in females, and complications are more frequent in males. Meckel's diverticulum is the most common congenital malformation of the gastrointestinal tract and is present in 2% of the general population. The standard therapy is surgical removal of the diverticulum.

11. The nurse is teaching the parents of a child who is receiving propylthiouracil for the treatment of hyperthyroidism (Graves' disease). Which statement made by the parent indicates a correct understanding of the teaching? a. "I would expect my child to gain weight while taking this medication." b. "I would expect my child to experience episodes of ear pain while taking this medication." c. "If my child develops a sore throat and fever, I should contact the physician immediately." d. "If my child develops the stomach flu, my child will need to be hospitalized."

ANS: C Children being treated with propylthiouracil must be carefully monitored for the side effects of the drug. Parents must be alerted that sore throat and fever accompany the grave complication of leukopenia. These symptoms should be immediately reported. Weight gain, episodes of ear pain, and stomach flu are not usually associated with leukopenia.

3. What is the priority nursing goal for a 14 year old diagnosed with Graves' disease? a. Relieving constipation b. Allowing the adolescent to make decisions about whether or not to take medication c. Verbalizing the importance of monitoring for medication side effects d. Developing alternative educational goals

ANS: C Children being treated with propylthiouracil or methimazole must be carefully monitored for side effects of the drug. Because sore throat and fever accompany the grave complication of leukopenia, these children should be seen by a health care practitioner if such symptoms occur. Parents and children should be taught to recognize and report symptoms immediately. The adolescent with Graves' disease is not likely to be constipated. Adherence to the medication schedule is important to ensure optimal health and wellness. Medications should not be skipped and dose regimens should not be tapered by the child without consultation with the child's medical provider. The management of Graves' disease does not interfere with school attendance and does not require alternative educational plans.

9. Constipation has recently become a problem for a school-age child who is being treated for seasonal allergies. The nurse should focus the assessment on what possibly related factor? a. Diet b. Allergies c. Antihistamines d. Emotional factors

ANS: C Constipation may be associated with drugs such as antihistamines, antacids, diuretics, opioids, antiepileptics, and iron. Because this is the only known recent change in her habits, the addition of antihistamines is most likely the etiology of the diarrhea, rather than diet, allergies, or emotional factors. With a change in bowel habits, the presence and role of any recently prescribed medications should be assessed.

17. What is used to treat moderate-to-severe inflammatory bowel disease? a. Antacids b. Antibiotics c. Corticosteroids d. Antidiarrheal medications

ANS: C Corticosteroids such as prednisone and prednisolone are used in short bursts to suppress the inflammatory response in inflammatory bowel disease. Antacids and antidiarrheals are not drugs of choice to treat the inflammatory process of inflammatory bowel disease. Antibiotics may be used as adjunctive therapy to treat complications.

30. The nurse is admitting a toddler with the diagnosis of juvenile hypothyroidism. Which is a common clinical manifestation of this disorder? a. Insomnia b. Diarrhea c. Dry skin d. Accelerated growth

ANS: C Dry skin, mental decline, and myxedematous skin changes are associated with juvenile hypothyroidism. Children with hypothyroidism are usually sleepy. Constipation is associated with hypothyroidism. Decelerated growth is common in juvenile hypothyroidism.

8. What is a common clinical manifestation of juvenile hypothyroidism? a. Insomnia b. Diarrhea c. Dry skin d. Accelerated growth

ANS: C Dry skin, mental decline, and myxedematous skin changes are associated with juvenile hypothyroidism. Children with hypothyroidism are usually sleepy. Constipation is associated with hypothyroidism. Decelerated growth is common in juvenile hypothyroidism.

21. The parents of a child who has just been diagnosed with type 1 diabetes ask about exercise. The nurse should provide the parents with what information to address the child's safety needs? a. Exercise will increase blood glucose. b. Exercise should be restricted. c. Extra snacks are needed before exercise. d. Extra insulin is required during exercise.

ANS: C Exercise lowers blood glucose levels, which can be compensated for by extra snacks. Exercise is encouraged and not restricted unless indicated by other health conditions. Extra insulin is contraindicated because exercise decreases blood glucose levels.

1. Which is most descriptive of the pathophysiology of leukemia? a. Increased blood viscosity occurs. b. Thrombocytopenia (excessive destruction of platelets) occurs. c. Unrestricted proliferation of immature white blood cells (WBCs) occurs. d. First stage of coagulation process is abnormally stimulated.

ANS: C Leukemia is a group of malignant disorders of the bone marrow and lymphatic system. It is defined as an unrestricted proliferation of immature WBCs in the blood-forming tissues of the body. Increased blood viscosity may occur secondary to the increased number of WBCs. Thrombocytopenia may occur secondary to the overproduction of WBCs in the bone marrow. The coagulation process is unaffected by leukemia.

4. At what age is sexual development in boys and girls considered to be precocious? a. Boys, 11 years; girls, 9 years b. Boys, 12 years; girls, 10 years c. Boys, 9 years; girls, 8 years d. Boys, 10 years; girls, 9.5 years

ANS: C Manifestations of sexual development before age 9 in boys and age 8 in girls are considered precocious and should be investigated. Boys older than 9 years of age and girls older than 8 years of age fall within the expected range of pubertal onset.

7. Therapeutic management of the child with acute diarrhea and dehydration usually begins with what intervention? a. Clear liquids b. Adsorbents such as kaolin and pectin c. Oral rehydration solution (ORS) d. Antidiarrheal medications such as paregoric

ANS: C ORS is the first treatment for acute diarrhea. Clear liquids are not recommended because they contain too much sugar, which may contribute to diarrhea. Adsorbents are not recommended and neither are antidiarrheal because they do not get rid of pathogens.

5. The viral pathogen that frequently causes acute diarrhea in young children is: a. Giardia organisms. b. Shigella organisms. c. Rotavirus. d. Salmonella organisms.

ANS: C Rotavirus is the most frequent viral pathogen that causes diarrhea in young children. Giardia and Salmonella are bacterial pathogens that cause diarrhea. Shigella is a bacterial pathogen that is uncommon in the United States.

14. When caring for a child with probable appendicitis, the nurse should be alert to recognize what sign of perforation? a. Bradycardia b. Anorexia c. Sudden relief from pain d. Decreased abdominal distention

ANS: C Signs of peritonitis, in addition to fever, include sudden relief from pain after perforation. Tachycardia, not bradycardia, is a manifestation of peritonitis. Anorexia is already a clinical manifestation of appendicitis. Abdominal distention usually increases in addition to an increase in pain (usually diffuse and accompanied by rigid guarding of the abdomen).

26. What is the major focus of the therapeutic management for a child with lactose intolerance? a. Compliance with the medication regimen b. Providing emotional support to family members c. Teaching dietary modifications d. Administration of daily normal saline enemas

ANS: C Simple dietary modifications are effective in the management of lactose intolerance. Symptoms of lactose intolerance are usually relieved after instituting a lactose-free diet. Medications are not typically ordered in the management of lactose intolerance. Providing emotional support to family members is not specific to this medical condition. Diarrhea is a manifestation of lactose intolerance. Enemas are contraindicated for this alteration in bowel elimination.

16. What is the characteristic of the immune-mediated type 1 diabetes mellitus? a. Ketoacidosis is infrequent b. Onset is gradual c. Age at onset is usually younger than 18 years d. Oral agents are often effective for treatment

ANS: C The immune-mediated type 1 diabetes mellitus typically has its onset in children or young adults. Peak incidence is between the ages of 10 and 15 years. Infrequent ketoacidosis, gradual onset, and treatment with oral agents are more consistent with type 2 diabetes

12. What is the primary purpose of prescribing a histamine receptor antagonist for an infant diagnosed with gastroesophageal reflux? a. Prevent reflux b. Prevent hematemesis. c. Reduce gastric acid production. d. Increase gastric acid production.

ANS: C The mechanism of action of histamine receptor antagonists is to reduce the amount of acid present in gastric contents and may prevent esophagitis. None of the remaining options are modes of action of histamine receptor antagonists but rather desired effects of medication therapy.

8. A school-age child diagnosed with leukemia experienced severe nausea and vomiting when receiving chemotherapy for the first time. Which is the most appropriate nursing action to prevent or minimize these reactions with subsequent treatments? a. Encourage drinking large amounts of favorite fluids. b. Encourage child to take nothing by mouth (remain NPO) until nausea and vomiting subside. c. Administer an antiemetic at least 30 minutes before chemotherapy begins. d. Administer an antiemetic as soon as child has nausea.

ANS: C The most beneficial regimen to minimize nausea and vomiting associated with chemotherapy is to administer the antiemetic 30 minutes to an hour before the chemotherapy is begun. The goal is to prevent anticipatory symptoms. Drinking fluids will add to the discomfort of the nausea and vomiting. Waiting until nausea and vomiting subside will help with this episode, but the child will have the discomfort and be at risk for dehydration. Administering an antiemetic as soon as the child has nausea does not prevent anticipatory nausea.

13. Glucocorticoids, mineralocorticoids, and sex steroids are secreted by which organ? a. Thyroid gland b. Parathyroid glands c. Adrenal cortex d. Anterior pituitary

ANS: C These hormones are secreted by the adrenal cortex. The thyroid gland produces thyroid hormone and thyrocalcitonin. The parathyroid glands produce parathyroid hormone. The anterior pituitary produces hormones such as growth hormone, thyroid-stimulating hormone, adrenocorticotropic hormone, gonadotropin, prolactin, and melanocyte-stimulating hormone.

23. The nurse is caring for an infant with suspected pyloric stenosis. Which clinical manifestation would indicate pyloric stenosis? a. Abdominal rigidity and pain on palpation b. Rounded abdomen and hypoactive bowel sounds c. Visible peristalsis and weight loss d. Distention of lower abdomen and constipation

ANS: C Visible gastric peristaltic waves that move from left to right across the epigastrium are observed in pyloric stenosis, as is weight loss. Abdominal rigidity and pain on palpation, and rounded abdomen and hypoactive bowel sounds, are usually not present. The upper abdomen is distended, not the lower abdomen.

Changes in blood volume after childbirth depend on several factors such as blood loss during childbirth and the amount of extravascular water (physiologic edema) mobilized and excreted. A postpartum nurse anticipates blood loss of (Select all that apply): a. 100 mL b. 250 mL or less c. 300 to 500 mL d. 500 to 1000 mL e. 1500 mL or greater

ANS: C, D The average blood loss for a vaginal birth of a single fetus ranges from 300 to 500 mL (10% of blood volume). The typical blood loss for women who gave birth by cesarean is 500 to 1000 mL (15% to 30% of blood volume). During the first few days after birth the plasma volume decreases further as a result diuresis. Pregnancy-induced hypervolemia (an increase in blood volume of at least 35%) allows most women to tolerate considerable blood loss during childbirth. PTS: 1 DIF: Cognitive Level: Comprehension REF: 487 OBJ: Nursing Process: Assessment, Planning MSC: Client Needs: Physiologic Integrity

5. A nurse is planning care for a school-age child diagnosed with type 1 diabetes. Which insulin preparations are either rapid or short acting? (Select all that apply.) a. Novolin N b. Lantus c. NovoLog d. Novolin R

ANS: C, D Rapid-acting insulin (e.g., NovoLog) reaches the blood within 15 minutes after injection. The insulin peaks 30 to 90 minutes later and may last as long as 5 hours. Short-acting (regular) insulin (e.g., Novolin R) usually reaches the blood within 30 minutes after injection. The insulin peaks 2 to 4 hours later and stays in the blood for about 4 to 8 hours. Intermediate-acting insulins (e.g., Novolin N) reach the blood 2 to 6 hours after injection. The insulins peak 4 to 14 hours later and stay in the blood for about 14 to 20 hours. Long-acting insulin (e.g., Lantus) takes 6 to 14 hours to start working. It has no peak or a very small peak 10 to 16 hours after injection. The insulin stays in the blood between 20 and 24 hours.

6. The nurse is caring for a school-age child with hyperthyroidism (Graves' disease). Which clinical manifestations should the nurse monitor that may indicate a thyroid storm? (Select all that apply.) a. Constipation b. Hypotension c. Hyperthermia d. Tachycardia e. Vomiting

ANS: C, D, E A child with a thyroid storm will have severe irritability and restlessness, vomiting, diarrhea, hyperthermia, hypertension, severe tachycardia, and prostration.

5. A nurse is conducting dietary teaching on high-fiber foods for parents of a child with constipation. Which foods should the nurse include as being high in fiber? (Select all that apply.) a. White rice b. Avocados c. Whole grain breads d. Bran pancakes e. Raw carrots

ANS: C, D, E High-fiber foods include whole grain breads, bran pancakes, and raw carrots. Unrefined (brown) rice is high in fiber but white rice is not. Raw fruits, especially those with skins or seeds, other than ripe banana or avocados are high in fiber

2. The baseline fetal heart rate (FHR) is the average rate during a 10-minute segment. Changes in FHR are categorized as periodic or episodic. These patterns include both accelerations and decelerations. The labor nurse is evaluating the patient's most recent 10-minute segment on the monitor strip and notes a late deceleration. This is likely to be caused by which physiologic alteration? (Select all that apply.) a. Spontaneous fetal movement b. Compression of the fetal head c. Placental abruption d. Cord around the baby's neck e. Maternal supine hypotension

ANS: C,E Late decelerations are almost always caused by uteroplacental insufficiency. Insufficiency is caused by uterine tachysystole, maternal hypotension, epidural or spinal anesthesia, IUGR, intraamniotic infection, or placental abruption. Spontaneous fetal movement, vaginal examination, fetal scalp stimulation, fetal reaction to external sounds, uterine contractions, fundal pressure and abdominal palpation are all likely to cause accelerations of the FHR. Early decelerations are most often the result of fetal head compression and may be caused by uterine contractions, fundal pressure, vaginal examination, and placement of an internal electrode. A variable deceleration is likely caused by umbilical cord compression. This may happen when the umbilical cord is around the baby's neck, arm, leg, or other body part or when there is a short cord, a knot in the cord, or a prolapsed cord.

11. A woman with gestational diabetes has had little or no experience reading and interpreting glucose levels. She shows the nurse her readings for the past few days. Which one should the nurse tell her indicates a need for adjustment (insulin or sugar)? A. 75 mg/dL before lunch. This is low; better eat now. B. 115 mg/dL 1 hour after lunch. This is a little high; maybe eat a little less next time. C. 115 mg/dL 2 hours after lunch; This is too high; it is time for insulin. D. 60 mg/dL just after waking up from a nap. This is too low; maybe eat a snack before going to sleep.

ANS: D 60 mg/dL after waking from a nap is too low. During hours of sleep glucose levels should not be less than 70 mg/dL. Snacks before sleeping can be helpful. The premeal acceptable range is 65 to 95 mg/dL. The readings 1 hour after a meal should be less than 140 mg/dL. Two hours after eating, the readings should be less than 120 mg/dL.

10. A woman has come to the clinic for preconception counseling because she wants to start trying to get pregnant in 3 months. She can expect the following advice: A. "Discontinue all contraception now." B. "Lose weight so that you can gain more during pregnancy." C. "You may take any medications you have been taking regularly." D. "Make sure that you include adequate folic acid in your diet."

ANS: D A healthy diet before conception is the best way to ensure that adequate nutrients are available for the developing fetus. A woman's folate or folic acid intake is of particular concern in the periconception period. Neural tube defects are more common in infants of women with a poor folic acid intake. Depending on the type of contraception used, discontinuing all contraception may not be appropriate advice. Losing weight is not appropriate advice. Depending on the type of medication the woman is taking, continuing its use may not be appropriate.

21. Which statement is true about the term contraceptive failure rate? A. It refers to the percentage of users expected to have an accidental pregnancy over a 5-year span. B. It refers to the minimum level that must be achieved to receive a government license. C. It increases over time as couples become more careless. D. It varies from couple to couple, depending on the method and the users.

ANS: D Contraceptive effectiveness varies from couple to couple, depending on how well a contraceptive method is used and how well it suits the couple. The contraceptive failure rate measures the likelihood of accidental pregnancy in the first year only. Failure rates decline over time because users gain experience.

The nurse is preparing to admit a newborn with myelomeningocele to the neonatal intensive care nursery. Which describes this newborns defect? a. Fissure in the spinal column that leaves the meninges and the spinal cord exposed b. Herniation of the brain and meninges through a defect in the skull c. Hernial protrusion of a saclike cyst of meninges with spinal fluid but no neural elements d. Visible defect with an external saclike protrusion containing meninges, spinal fluid, and nerves

ANS: D A myelomeningocele is a visible defect with an external saclike protrusion, containing meninges, spinal fluid, and nerves. Rachischisis is a fissure in the spinal column that leaves the meninges and the spinal cord exposed. Encephalocele is a herniation of brain and meninges through a defect in the skull, producing a fluid-filled sac. Meningocele is a hernial protrusion of a saclike cyst of meninges with spinal fluid, but no neural elements.

17. Which basic type of pelvis includes the correct description and percentage of occurrence in women? a. Gynecoid: classic female pelvis; heart shaped; 75% b. Android: resembling the male pelvis; wide oval; 15% c. Anthropoid: resembling the pelvis of the ape; narrow; 10% d. Platypelloid: flattened, wide, and shallow pelvis; 3%

ANS: D A platypelloid pelvis is flattened, wide, and shallow; approximately 3% of women have this shape. The gynecoid pelvis is the classic female shape, slightly ovoid and rounded; approximately 50% of women have this shape. An android or malelike pelvis is heart shaped; approximately 23% of women have this shape. An anthropoid or apelike pelvis is oval and wide; approximately 24% of women have this shape.

11. The most common cause of decreased variability in the fetal heart rate (FHR) that lasts 30 minutes or less is: a. altered cerebral blood flow. b. fetal hypoxemia. c. umbilical cord compression. d. fetal sleep cycles.

ANS: D A temporary decrease in variability can occur when the fetus is in a sleep state. These sleep states do not usually last longer than 30 minutes. Altered fetal cerebral blood flow would result in early decelerations in the FHR. Fetal hypoxemia would be evidenced by tachycardia initially and then bradycardia. A persistent decrease or loss of FHR variability may be seen. Umbilical cord compression would result in variable decelerations in the FHR.

Which clinical manifestation should the nurse expect when a child with sickle cell anemia experiences an acute vasoocclusive crisis? a. Circulatory collapse b. Cardiomegaly, systolic murmurs c. Hepatomegaly, intrahepatic cholestasis d. Painful swelling of hands and feet; painful joints

ANS: D A vasoocclusive crisis is characterized by severe pain in the area of involvement. If in the extremities, painful swelling of the hands and feet is seen; if in the abdomen, severe pain resembles that of acute surgical abdomen; and if in the head, stroke and visual disturbances occur. Circulatory collapse results from sequestration crises. Cardiomegaly, systolic murmurs, hepatomegaly, and intrahepatic cholestasis result from chronic vasoocclusive phenomena.

26. Which pregnant woman should restrict her weight gain during pregnancy? A. Woman pregnant with twins B. Woman in early adolescence C. Woman shorter than 62 inches or 157 cm D. Woman who was 20 lbs overweight before pregnancy

ANS: D A weight gain of 5 to 9 kg will provide sufficient nutrients for the fetus. Overweight and obese women should be advised to lose weight before conception to achieve the best pregnancy outcomes. A higher weight gain in twin gestations may help prevent low birth weights. Adolescents need to gain weight toward the higher acceptable range, which provides for their own growth as well as for fetal growth. In the past, women of short stature were advised to restrict their weight gain; however, evidence to support these guidelines has not been found.

21. The most basic information a maternity nurse should have concerning conception is that: A. ova are considered fertile 48 to 72 hours after ovulation. B. sperm remain viable in the woman's reproductive system for an average of 12 to 24 hours. C. conception is achieved when a sperm successfully penetrates the membrane surrounding the ovum. D. implantation in the endometrium occurs 6 to 10 days after conception.

ANS: D After implantation, the endometrium is called the decidua. Ova are considered fertile for about 24 hours after ovulation. Sperm remain viable in the woman's reproductive system for an average of 2 to 3 days. Penetration of the ovum by the sperm is called fertilization. Conception occurs when the zygote, the first cell of the new individual, is formed.

26. The phenomenon of someone other than the mother-to-be experiencing pregnancy-like symptoms such as nausea and weight gain applies to the: A. mother of the pregnant woman. B. couple's teenage daughter. C. sister of the pregnant woman. D. expectant father.

ANS: D An expectant father's experiencing pregnancy-like symptoms is called the couvade syndrome.

29. The obstetric nurse is preparing the patient for an emergency cesarean birth, with no time to administer spinal anesthesia. The nurse is aware and prepared for the greatest risk of administering general anesthesia to the patient. This risk is: a. respiratory depression. b. uterine relaxation. c. inadequate muscle relaxation. d. aspiration of stomach contents.

ANS: D Aspiration of acidic gastric contents with possible airway obstruction is a potentially fatal complication of general anesthesia. Respirations can be altered during general anesthesia, and the anesthesiologist will take precautions to maintain proper oxygenation. Uterine relaxation can occur with some anesthesia; however, this can be monitored and prevented. Inadequate muscle relaxation can be improved with medication.

Which is a common clinical manifestation of Hodgkin disease? a. Petechiae b. Bone and joint pain c. Painful, enlarged lymph nodes d. Enlarged, firm, nontender lymph nodes

ANS: D Asymptomatic, enlarged, cervical or supraclavicular lymphadenopathy is the most common presentation of Hodgkin disease. Petechiae are usually associated with leukemia. Bone and joint pain are not likely in Hodgkin disease. The enlarged nodes are rarely painful.

The parents of a child with cerebral palsy ask the nurse whether any drugs can decrease their childs spasticity. The nurses response should be based on which statement? a. Anticonvulsant medications are sometimes useful for controlling spasticity. b. Medications that would be useful in reducing spasticity are too toxic for use with children. c. Many different medications can be highly effective in controlling spasticity. d. Implantation of a pump to deliver medication into the intrathecal space to decrease spasticity has recently become available.

ANS: D Baclofen, given intrathecally, is best suited for children with severe spasticity that interferes with activities of daily living and ambulation. Anticonvulsant medications are used when seizures occur in children with cerebral palsy. The intrathecal route decreases the side effects of the drugs that reduce spasticity. Few medications are currently available for the control of spasticity.

28. While you are assessing the vital signs of a pregnant woman in her third trimester, the patient complains of feeling faint, dizzy, and agitated. Which nursing intervention is appropriate? A. Have the patient stand up and retake her blood pressure. B. Have the patient sit down and hold her arm in a dependent position. C. Have the patient lie supine for 5 minutes and recheck her blood pressure on both arms. D. Have the patient turn to her left side and recheck her blood pressure in 5 minutes.

ANS: D Blood pressure is affected by maternal position during pregnancy. The supine position may cause occlusion of the vena cava and descending aorta. Turning the pregnant woman to a lateral recumbent position alleviates pressure on the blood vessels and quickly corrects supine hypotension. Pressures are significantly higher when the patient is standing. This option causes an increase in systolic and diastolic pressures. The arm should be supported at the same level of the heart. The supine position may cause occlusion of the vena cava and descending aorta, creating hypotension.

The nurse is recommending how to prevent iron deficiency anemia in a healthy, term, breast-fed infant. Which should be suggested? a. Iron (ferrous sulfate) drops after age 1 month b. Iron-fortified commercial formula by age 4 to 6 months c. Iron-fortified infant cereal by age 2 months d. Iron-fortified infant cereal by age 4 to 6 months

ANS: D Breast milk supplies inadequate iron for growth and development after age 5 months. Supplementation is necessary at this time. The mother can supplement the breastfeeding with iron-fortified infant cereal. Iron supplementation or the introduction of solid foods in a breast-fed baby is not indicated. Providing iron-fortified commercial formula by age 4 to 6 months should be done only if the mother is choosing to discontinue breastfeeding.

5. A woman at 26 weeks of gestation is being assessed to determine whether she is experiencing preterm labor. What finding indicates that preterm labor is occurring? a. Estriol is not found in maternal saliva. b. Irregular, mild uterine contractions are occurring every 12 to 15 minutes. c. Fetal fibronectin is present in vaginal secretions. d. The cervix is effacing and dilated to 2 cm.

ANS: D Cervical changes such as shortened endocervical length, effacement, and dilation are predictors of imminent preterm labor. Changes in the cervix accompanied by regular contractions indicate labor at any gestation. Estriol is a form of estrogen produced by the fetus that is present in plasma at 9 weeks of gestation. Levels of salivary estriol have been shown to increase before preterm birth. Irregular, mild contractions that do not cause cervical change are not considered a threat. The presence of fetal fibronectin in vaginal secretions between 24 and 36 weeks of gestation could predict preterm labor, but it has only a 20% to 40% positive predictive value. Of more importance are other physiologic clues of preterm labor such as cervical changes.

17. The _____ is/are responsible for oxygen and carbon dioxide transport to and from the maternal bloodstream. A. decidua basalis B. blastocyst C. germ layer D. chorionic villi

ANS: D Chorionic villi are finger-like projections that develop out of the trophoblast and extend into the blood-filled spaces of the endometrium. The villi obtain oxygen and nutrients from the maternal bloodstream and dispose of carbon dioxide and waste products into the maternal blood. The decidua basalis is the portion of the decidua (endometrium) under the blastocyst where the villi attach. The blastocyst is the embryonic development stage after the morula. Implantation occurs at this stage. The germ layer is a layer of the blastocyst.

26. As a powerful central nervous system stimulant, which of these substances can lead to miscarriage, preterm labor, placental separation (abruption), and stillbirth? a. Heroin b. Alcohol c. PCP d. Cocaine

ANS: D Cocaine is a powerful CNS stimulant. Effects on pregnancy associated with cocaine use include abruptio placentae, preterm labor, precipitous birth, and stillbirth. Heroin is an opiate. Its use in pregnancy is associated with preeclampsia, intrauterine growth restriction, miscarriage, premature rupture of membranes, infections, breech presentation, and preterm labor. The most serious effect of alcohol use in pregnancy is fetal alcohol syndrome. The major concerns regarding PCP use in pregnant women are its association with polydrug abuse and the neurobehavioral effects on the neonate.

27. The major source of nutrients in the diet of a pregnant woman should be composed of: A. simple sugars. B. fats. C. fiber. D. complex carbohydrates.

ANS: D Complex carbohydrates supply the pregnant woman with vitamins, minerals, and fiber. The most common simple carbohydrate is table sugar, which is a source of energy but does not provide any nutrients. Fats provide 9 kcal in each gram, in contrast to carbohydrates and proteins, which provide only 4 kcal in each gram. Fiber is supplied primarily by complex carbohydrates.

22. To reassure and educate their pregnant patients about changes in their blood pressure, maternity nurses should be aware that: A. a blood pressure cuff that is too small produces a reading that is too low; a cuff that is too large produces a reading that is too high. B. shifting the patient's position and changing from arm to arm for different measurements produces the most accurate composite blood pressure reading at each visit. C. the systolic blood pressure increases slightly as pregnancy advances; the diastolic pressure remains constant. D. compression of the iliac veins and inferior vena cava by the uterus contributes to hemorrhoids in the later stage of term pregnancy.

ANS: D Compression of the iliac veins and inferior vena cava also leads to varicose veins in the legs and vulva. The tightness of a cuff that is too small produces a reading that is too high; similarly the looseness of a cuff that is too large results in a reading that is too low. Because maternal positioning affects readings, blood pressure measurements should be obtained in the same arm and with the woman in the same position. The systolic blood pressure generally remains constant but may decline slightly as pregnancy advances. The diastolic blood pressure first decreases and then gradually increases.

22. When counseling a patient about getting enough iron in her diet, the maternity nurse should tell her that: A. milk, coffee, and tea aid iron absorption if consumed at the same time as iron. B. iron absorption is inhibited by a diet rich in vitamin C. C. iron supplements are permissible for children in small doses. D. constipation is common with iron supplements.

ANS: D Constipation can be a problem. Milk, coffee, and tea inhibit iron absorption when consumed at the same time as iron. Vitamin C promotes iron absorption. Children who ingest iron can get very sick and even die.

18. A woman who is 8 months pregnant asks the nurse, "Does my baby have any antibodies to fight infection?" The most appropriate response by the nurse is: A. "Your baby has all the immune globulins necessary: IgG, IgM, and IgA." B. "Your baby won't receive any antibodies until he/she is born and you breastfeed him." C. "Your baby does not have any antibodies to fight infection." D. "Your baby has IgG and IgM."

ANS: D During the third trimester, the only immune globulin that crosses the placenta, IgG, provides passive acquired immunity to specific bacterial toxins. The fetus produces IgM by the end of the first trimester. IgA is not produced by the baby. By the third trimester, the fetus has IgG and IgM. Breastfeeding supplies the baby with IgA. "Your baby does not have any antibodies to fight infection" is an inaccurate statement.

9. With regard to the estimation and interpretation of the recurrence of risks for genetic disorders, nurses should be aware that: A. with a dominant disorder, the likelihood of the second child also having the condition is 100%. B. an autosomal recessive disease carries a one in eight risk of the second child also having the disorder. C. disorders involving maternal ingestion of drugs carry a one in four chance of being repeated in the second child. D. the risk factor remains the same no matter how many affected children are already in the family.

ANS: D Each pregnancy is an independent event. The risk factor (e.g., one in two, one in four) remains the same for each child, no matter how many children are born to the family.In a dominant disorder, the likelihood of recurrence in subsequent children is 50% (one in two). An autosomal recessive disease carries a one in four chance of recurrence. In disorders involving maternal ingestion of drugs, subsequent children would be at risk only if the mother continued to take drugs; the rate of risk would be difficult to calculate.

1. An 18-year-old pregnant woman, gravida 1, is admitted to the labor and birth unit with moderate contractions every 5 minutes that last 40 seconds. The woman states, "My contractions are so strong that I don't know what to do with myself." The nurse should: a. assess for fetal well-being. b. encourage the woman to lie on her side. c. disturb the woman as little as possible. d. recognize that pain is personalized for each individual.

ANS: D Each woman's pain during childbirth is unique and is influenced by a variety of physiologic, psychosocial, and environmental factors. A critical issue for the nurse is how support can make a difference in the pain of the woman during labor and birth. Assessing for fetal well-being includes no information that would indicate fetal distress or a logical reason to be overly concerned about the well-being of the fetus. The left lateral position is used to alleviate fetal distress, not maternal stress. The nurse has an obligation to provide physical, emotional, and psychosocial care and support to the laboring woman. This patient clearly needs support.

9. A woman in week 34 of pregnancy reports that she is very uncomfortable because of heartburn. The nurse would suggest that the woman: A. substitute other calcium sources for milk in her diet. B. lie down after each meal. C. reduce the amount of fiber she consumes. D. eat five small meals daily.

ANS: D Eating small, frequent meals may help with heartburn, nausea, and vomiting. Substituting other calcium sources for milk, lying down after eating, and reducing fiber intake are inappropriate dietary suggestions for all pregnant women and do not alleviate heartburn.

Which is the most effective pain-management approach for a child who is having a bone marrow aspiration? a. Relaxation techniques b. Administration of an opioid c. EMLA cream applied over site d. Conscious or unconscious sedation

ANS: D Effective pharmacologic and nonpharmacologic measures should be used to minimize pain associated with procedures. For bone marrow aspiration, conscious or unconscious sedation should be used. Relaxation, opioids, and EMLA can be used to augment the conscious or unconscious sedation.

16. In the current practice of childbirth preparation, emphasis is placed on: a. the Dick-Read (natural) childbirth method. b. the Lamaze (psychoprophylactic) method. c. the Bradley (husband-coached) method. d. having expectant parents attend childbirth preparation in any or no specific method.

ANS: D Encouraging expectant parents to attend childbirth preparation class is most important because preparation increases a woman's confidence and thus her ability to cope with labor and birth. Although still popular, the "method" format of classes is being replaced with other offerings such as Hypnobirthing and Birthing from Within.

The nurse is conducting reflex testing on infants at a well-child clinic. Which reflex finding should be reported as abnormal and considered as a possible sign of cerebral palsy? a. Tonic neck reflex at 5 months of age b. Absent Moro reflex at 8 months of age c. Moro reflex at 3 months of age d. Extensor reflex at 7 months of age

ANS: D Establishing a diagnosis of cerebral palsy (CP) may be confirmed with the persistence of primitive reflexes: (1) either the asymmetric tonic neck reflex or persistent Moro reflex (beyond 4 months of age) and (2) the crossed extensor reflex. The tonic neck reflex normally disappears between 4 and 6 months of age. The crossed extensor reflex, which normally disappears by 4 months, is elicited by applying a noxious stimulus to the sole of one foot with the knee extended. Normally, the contralateral foot responds with extensor, abduction, and then adduction movements. The possibility of CP is suggested if these reflexes occur after 4 months.

23. When assessing the relative advantages and disadvantages of internal and external electronic fetal monitoring, nurses comprehend that both: a. can be used when membranes are intact. b. measure the frequency, duration, and intensity of uterine contractions. c. may need to rely on the woman to indicate when uterine activity (UA) is occurring. d. can be used during the antepartum and intrapartum periods.

ANS: D External monitoring can be used in both periods; internal monitoring can be used only in the intrapartum period. For internal monitoring the membranes must have ruptured, and the cervix must be sufficiently dilated. Internal monitoring measures the intensity of contractions; external monitoring cannot do this. With external monitoring, the woman may need to alert the nurse that UA is occurring; internal monitoring does not require this.

13. Certain changes stimulate chemoreceptors in the aorta and carotid bodies to prepare the fetus for initiating respirations immediately after birth. Which change in fetal physiologic activity is not part of this process? a. Fetal lung fluid is cleared from the air passages during labor and vaginal birth. b. Fetal partial pressure of oxygen (PO2) decreases. c. Fetal partial pressure of carbon dioxide in arterial blood (PaCO2) increases. d. Fetal respiratory movements increase during labor.

ANS: D Fetal respiratory movements actually decrease during labor. Fetal lung fluid is cleared from the air passages during labor and vaginal birth. Fetal PO2 decreases, and fetal PaCO2 increases.

33. A nurse practitioner performs a clinical breast examination on a woman diagnosed with fibroadenoma. The nurse knows that fibroadenoma is characterized by: A. inflammation of the milk ducts and glands behind the nipples. B. thick, sticky discharge from the nipple of the affected breast. C. lumpiness in both breasts that develops 1 week before menstruation. D. a single lump in one breast that can be expected to shrink as the woman ages.

ANS: D Fibroadenomas are characterized by discrete, usually solitary lumps smaller than 3 cm in diameter. Fibroadenomas increase in size during pregnancy and shrink as the woman ages. Inflammation of the milk ducts is associated with mammary duct ectasia, not fibroadenoma. A thick, sticky discharge is associated with galactorrhea, not fibroadenoma. Lumpiness before menstruation is associated with fibrocystic changes of the breast.

A child with leukemia is receiving triple intrathecal chemotherapy consisting of methotrexate, cytarabine, and hydrocortisone. The purpose of this is to prevent: a. infection. b. brain tumor. c. drug side effects. d. central nervous system (CNS) disease.

ANS: D For certain children, CNS prophylactic therapy is indicated. This drug regimen is used to prevent CNS leukemia and will not prevent infection or drug side effects. If the child has a brain tumor in addition to leukemia, additional therapy would be indicated.

14. With regard to the care management of preterm labor, nurses should be aware that: a. all women must be considered at risk for preterm labor and prediction is so hit-and-miss, teaching pregnant women the symptoms probably causes more harm through false alarms. b. Braxton Hicks contractions often signal the onset of preterm labor. c. preterm labor is likely to be the start of an extended labor, a woman with symptoms can wait several hours before contacting the primary caregiver. d. the diagnosis of preterm labor is based on gestational age, uterine activity, and progressive cervical change.

ANS: D Gestational age of 20 to 37 weeks, uterine contractions, and a cervix that is 80% effaced or dilated 2 cm indicates preterm labor. It is essential that nurses teach women how to detect the early symptoms of preterm labor. Braxton Hicks contractions resemble preterm labor contractions, but they are not true labor. Waiting too long to see a health care provider could result in not administering essential medications. Preterm labor is not necessarily long-term labor.

1. A woman arrives at the clinic seeking confirmation that she is pregnant. The following information is obtained: She is 24 years old with a body mass index (BMI) of 17.5. She admits to having used cocaine "several times" during the past year and drinks alcohol occasionally. Her blood pressure (BP) is 108/70 mm Hg, her pulse rate is 72 beats/min, and her respiratory rate is 16 breaths/min. The family history is positive for diabetes mellitus and cancer. Her sister recently gave birth to an infant with a neural tube defect (NTD). Which characteristics place the woman in a high risk category? A. Blood pressure, age, and BMI B. Drug/alcohol use, age, and family history C. Family history, blood pressure, and BMI D. Family history, BMI, and drug/alcohol abuse

ANS: D Her family history of NTD, low BMI, and substance abuse all are high risk factors of pregnancy. The woman's BP is normal, and her age does not put her at risk. Her BMI is low and may indicate poor nutritional status, which would be a high risk. The woman's drug/alcohol use and family history put her in a high risk category, but her age does not. The woman's family history puts her in a high risk category. Her BMI is low and may indicate poor nutritional status, which would be high risk. Her BP is normal.

12. Which statement correctly describes the effects of various pain factors? a. Higher prostaglandin levels arising from dysmenorrhea can blunt the pain of childbirth. b. Upright positions in labor increase the pain factor because they cause greater fatigue. c. Women who move around trying different positions are experiencing more pain. d. Levels of pain-mitigating β-endorphins are higher during a spontaneous, natural childbirth.

ANS: D Higher endorphin levels help women tolerate pain and reduce anxiety and irritability. Higher prostaglandin levels correspond to more severe labor pains. Upright positions in labor usually result in improved comfort and less pain. Moving freely to find more comfortable positions is important for reducing pain and muscle tension.

16. Human chorionic gonadotropin (hCG) is an important biochemical marker for pregnancy and the basis for many tests. A maternity nurse should be aware that: A. hCG can be detected 2.5 weeks after conception. B. the hCG level increases gradually and uniformly throughout pregnancy. C. much lower than normal increases in the level of hCG may indicate a postdate pregnancy. D. a higher than normal level of hCG may indicate an ectopic pregnancy or Down syndrome.

ANS: D Higher levels also could be a sign of multiple gestation. hCG can be detected 7 to 8 days after conception. The hCG level fluctuates during pregnancy: peaking, declining, stabilizing, and increasing again. Abnormally slow increases may indicate impending miscarriage.

The nurse is reviewing first aid with a group of school nurses. Which statement made by a participant indicates a correct understanding of the information? a. If a child loses a tooth due to injury, I should place the tooth in warm milk. b. If a child has recurrent abdominal pain, I should send him or her back to class until the end of the day. c. If a child has a chemical burn to the eye, I should irrigate the eye with normal saline. d. If a child has a nosebleed, I should have the child sit up and lean forward.

ANS: D If a child has a nosebleed, the child should lean forward, not lie down. A tooth should be placed in cold milk or saliva for transporting to a dentist. Recurrent abdominal pain is a physiologic problem and requires further evaluation. If a chemical burn occurs in the eye, the eye should be irrigated with water for 20 minutes.

18. To reassure and educate pregnant patients about changes in the cervix, vagina, and position of the fetus, nurses should be aware that: A. because of a number of changes in the cervix, abnormal Papanicolaou (Pap) tests are much easier to evaluate. B. Quickening is a technique of palpating the fetus to engage it in passive movement. C. the deepening color of the vaginal mucosa and cervix (Chadwick's sign) usually appears in the second trimester or later as the vagina prepares to stretch during labor. D. increased vascularity of the vagina increases sensitivity and may lead to a high degree of arousal, especially in the second trimester.

ANS: D Increased sensitivity and an increased interest in sex sometimes go together. This frequently occurs during the second trimester. Cervical changes make evaluation of abnormal Pap tests more difficult. Quickening is the first recognition of fetal movements by the mother. Ballottement is a technique used to palpate the fetus. Chadwick's sign appears from the sixth to eighth weeks.

26. Which of the following statements is the most complete and accurate description of medical abortions? A. They are performed only for maternal health. B. They can be achieved through surgical procedures or with drugs. C. They are mostly performed in the second trimester. D. They can be either elective or therapeutic.

ANS: D Medical abortions are performed through the use of medications (rather than surgical procedures). They are mostly done in the first trimester, and they can be either elective (the woman's choice) or therapeutic (for reasons of maternal or fetal health).

13. One of the alterations in cyclic bleeding that occurs between periods is called: a. oligomenorrhea. b. menorrhagia. c. leiomyoma. d. metrorrhagia.

ANS: D Metrorrhagia is bleeding between periods. It can be caused by progestin injections and implants. Oligomenorrhea is infrequent or scanty menstruation. Menorrhagia is excessive menstruation. Leiomyoma is a common cause of excessive bleeding.

5. Nafarelin is currently used as a treatment for mild-to-severe endometriosis. The nurse should tell a woman taking this medication that the drug: A. stimulates the secretion of gonadotropin-releasing hormone (GnRH), thereby stimulating ovarian activity. B. should be sprayed into one nostril every other day. C. should be injected into subcutaneous tissue bid. D. can cause her to experience some hot flashes and bone loss.

ANS: D Nafarelin is a GnRH agonist, and its side effects are similar to effects of menopause. The hypoestrogenism effect results in hot flashes and bone loss. Nafarelin is a GnRH agonist that suppresses the secretion of GnRH and is administered twice daily by nasal spray.

27. Which method of pain management is safest for a gravida 3 para 2 admitted at 8 cm cervical dilation? a. Epidural anesthesia b. Narcotics c. Spinal block d. Breathing and relaxation techniques

ANS: D Nonpharmacologic methods of pain management may be the best option for a woman in advanced labor. It is unlikely that enough time remains to administer epidural or spinal anesthesia. A narcotic given at this time may reach its peak about the time of birth and result in respiratory depression in the newborn.

16. As a perinatal nurse you realize that a fetal heart rate that is tachycardic, is bradycardic, or has late decelerations or loss of variability is nonreassuring and is associated with: a. hypotension. b. cord compression. c. maternal drug use. d. hypoxemia.

ANS: D Nonreassuring heart rate patterns are associated with fetal hypoxemia. Fetal bradycardia may be associated with maternal hypotension. Fetal variable decelerations are associated with cord compression. Maternal drug use is associated with fetal tachycardia.

25. Which statement about a condition of pregnancy is accurate? A. Insufficient salivation (ptyalism) is caused by increases in estrogen. B. Acid indigestion (pyrosis) begins early but declines throughout pregnancy. C. Hyperthyroidism often develops (temporarily) because hormone production increases. D. Nausea and vomiting rarely have harmful effects on the fetus and may be beneficial.

ANS: D Normal nausea and vomiting rarely produce harmful effects, and nausea and vomiting periods may be less likely to result in miscarriage or preterm labor. Ptyalism is excessive salivation, which may be caused by a decrease in unconscious swallowing or stimulation of the salivary glands. Pyrosis begins in the first trimester and intensifies through the third trimester. Increased hormone production does not lead to hyperthyroidism in pregnant women.

10. With regard to dysmenorrhea, nurses should be aware that: A. it is more common in older women. B. it is more common in leaner women who exercise strenuously. C. symptoms can begin at any point in the ovulatory cycle. D. pain usually occurs in the suprapubic area or lower abdomen.

ANS: D Pain is described as sharp and cramping or sometimes as a dull ache. It may radiate to the lower back or upper thighs. Dysmenorrhea is more common in women 17 to 24 years old, women who smoke, and women who are obese. Symptoms begin with menstruation or sometimes a few hours before the onset of flow.

24. The recommended treatment for the prevention of human immunodeficiency virus (HIV) transmission to the fetus during pregnancy is: a. acyclovir. b. ofloxacin. c. podophyllin. d. zidovudine.

ANS: D Perinatal transmission of HIV has decreased significantly in the past decade as a result of prophylactic administration of the antiretroviral drug zidovudine to pregnant women in the prenatal and perinatal periods. Acyclovir is an antiviral treatment for HSV. Ofloxacin is an antibacterial treatment for gonorrhea. Podophyllin is a solution used in the treatment of human papillomavirus.

23. When discussing work and travel during pregnancy with a pregnant patient, nurses should instruct them that: A. women should sit for as long as possible and cross their legs at the knees from time to time for exercise. B. women should avoid seat belts and shoulder restraints in the car because they press on the fetus. C. metal detectors at airport security checkpoints can harm the fetus if the woman passes through them a number of times. D. while working or traveling in a car or on a plane, women should arrange to walk around at least every 2 hours or so.

ANS: D Periodic walking helps prevent thrombophlebitis. Pregnant women should avoid sitting or standing for long periods and crossing the legs at the knees. Pregnant women must wear lap belts and shoulder restraints. The most common injury to the fetus comes from injury to the mother. Metal detectors at airport security checkpoints do not harm fetuses.

32. The most dangerous effect on the fetus of a mother who smokes cigarettes while pregnant is: A. genetic changes and anomalies. B. extensive central nervous system damage. C. fetal addiction to the substance inhaled. D. intrauterine growth restriction.

ANS: D The major consequences of smoking tobacco during pregnancy are low-birth-weight infants, prematurity, and increased perinatal loss. Cigarettes normally will not cause genetic changes or extensive central nervous system damage. Addiction to tobacco is not a usual concern related to the neonate.

26. Glucose metabolism is profoundly affected during pregnancy because: A. pancreatic function in the islets of Langerhans is affected by pregnancy. B. the pregnant woman uses glucose at a more rapid rate than the nonpregnant woman. C. the pregnant woman increases her dietary intake significantly. D. placental hormones are antagonistic to insulin, thus resulting in insulin resistance.

ANS: D Placental hormones, estrogen, progesterone, and human placental lactogen (HPL) create insulin resistance. Insulin is also broken down more quickly by the enzyme placental insulinase. Pancreatic functioning is not affected by pregnancy. The glucose requirements differ because of the growing fetus. The pregnant woman should increase her intake by 200 calories a day.

7. Which assessment is least likely to be associated with a breech presentation? a. Meconium-stained amniotic fluid b. Fetal heart tones heard at or above the maternal umbilicus c. Preterm labor and birth d. Postterm gestation

ANS: D Postterm gestation is not likely to be seen with a breech presentation. The presence of meconium in a breech presentation may result from pressure on the fetal wall as it traverses the birth canal. Fetal heart tones heard at the level of the umbilical level of the mother are a typical finding in a breech presentation because the fetal back would be located in the upper abdominal area. Breech presentations often occur in preterm births.

11. The nurse caring for the pregnant patient must understand that the hormone essential for maintaining pregnancy is: A. estrogen. B. human chorionic gonadotropin (hCG). C. oxytocin. D. progesterone.

ANS: D Progesterone is essential for maintaining pregnancy; it does so by relaxing smooth muscles. This reduces uterine activity and prevents miscarriage. Estrogen plays a vital role in pregnancy, but it is not the primary hormone for maintaining pregnancy. hCG levels increase at implantation but decline after 60 to 70 days. Oxytocin stimulates uterine contractions.

30. Which collection of risk factors most likely would result in damaging lacerations (including episiotomies)? a. A dark-skinned woman who has had more than one pregnancy, who is going through prolonged second-stage labor, and who is attended by a midwife. b. A reddish-haired mother of two who is going through a breech birth. c. A dark-skinned, first-time mother who is going through a long labor. d. A first-time mother with reddish hair whose rapid labor was overseen by an obstetrician.

ANS: D Reddish-haired women have tissue that is less distensible than that of darker-skinned women and therefore may have less efficient healing. First time mothers are also more at risk, especially with breech births, long second-stage labors, or rapid labors in which there is insufficient time for the perineum to stretch. The rate of episiotomies is higher when obstetricians rather than midwives attend births.

16. The two primary areas of risk for sexually transmitted infections (STIs) are: A. sexual orientation and socioeconomic status. B. age and educational level. C. large number of sexual partners and race. D. risky sexual behaviors and inadequate preventive health behaviors.

ANS: D Risky sexual behaviors and inadequate preventive health behaviors put a person at risk for acquiring or transmitting an STI. Although low socioeconomic status may be a factor in avoiding purchasing barrier protection, sexual orientation does not put one at higher risk. Younger individuals and individuals with less education may be unaware of proper prevention techniques; however, these are not the primary areas of risk for STIs. Having a large number of sexual partners is a risk-taking behavior, but race does not increase the risk for STIs.

7. A pregnant woman's diet history indicates that she likes the following list of foods. The nurse would encourage this woman to consume more of which food to increase her calcium intake? A. Fresh apricots B. Canned clams C. Spaghetti with meat sauce D. Canned sardines

ANS: D Sardines are rich in calcium. Fresh apricots, canned clams, and spaghetti with meat sauce are not high in calcium.

Several complications can occur when a child receives a blood transfusion. Which is an immediate sign or symptom of an air embolus? a. Chills and shaking b. Nausea and vomiting c. Irregular heart rate d. Sudden difficulty in breathing

ANS: D Signs of air embolism are sudden difficulty breathing, sharp pain in the chest, and apprehension. Air emboli should be avoided by carefully flushing all tubing of air before connecting to patient. Chills, shaking, nausea, and vomiting are associated with hemolytic reactions. Irregular heart rate is associated with electrolyte disturbances and hypothermia.

13. When caring for a pregnant woman with cardiac problems, the nurse must be alert for signs and symptoms of cardiac decompensation, which include: A. a regular heart rate and hypertension. B. an increased urinary output, tachycardia, and dry cough. C. shortness of breath, bradycardia, and hypertension. D. dyspnea; crackles; and an irregular, weak pulse.

ANS: D Signs of cardiac decompensation include dyspnea; crackles; an irregular, weak, rapid pulse; rapid respirations; a moist, frequent cough; generalized edema; increasing fatigue; and cyanosis of the lips and nail beds. A regular heart rate and hypertension are not generally associated with cardiac decompensation. Tachycardia would indicate cardiac decompensation, but increased urinary output and a dry cough would not. Shortness of breath would indicate cardiac decompensation, but bradycardia and hypertension would not.

7. Numerous changes in the integumentary system occur during pregnancy. Which change persists after birth? A. Epulis B. Chloasma C. Telangiectasia D. Striae gravidarum

ANS: D Striae gravidarum, or stretch marks, reflect separation within the underlying connective tissue of the skin. They usually fade after birth, although they never disappear completely. An epulis is a red, raised nodule on the gums that bleeds easily. Chloasma, or mask of pregnancy, is a blotchy, brown hyperpigmentation of the skin over the cheeks, nose, and forehead, especially in dark-complexioned pregnant women. Chloasma usually fades after the birth. Telangiectasia, or vascular spiders, are tiny, star-shaped or branch-like, slightly raised, pulsating end-arterioles usually found on the neck, thorax, face, and arms. They occur as a result of elevated levels of circulating estrogen. These usually disappear after birth.

19. During a physical assessment of an at-risk patient, the nurse notes generalized edema, crackles at the base of the lungs, and some pulse irregularity. These are most likely signs of: A. euglycemia. B. rheumatic fever. C. pneumonia. D. cardiac decompensation.

ANS: D Symptoms of cardiac decompensation may appear abruptly or gradually. Euglycemia is a condition of normal glucose levels. These symptoms indicate cardiac decompensation. Rheumatic fever can cause heart problems, but it does not manifest with these symptoms, which indicate cardiac decompensation. Pneumonia is an inflammation of the lungs and would not likely generate these symptoms, which indicate cardiac decompensation.

Therapeutic management of a child with tetanus includes the administration of: a. nonsteroidal anti-inflammatory drugs (NSAIDs) to reduce inflammation. b. muscle stimulants to counteract muscle weakness. c. bronchodilators to prevent respiratory complications. d. tetanus immunoglobulin therapy.

ANS: D Tetanus immunoglobulin therapy, to neutralize toxins, is the most specific therapy for tetanus. Tetanus toxin acts at the myoneural junction to produce muscular stiffness and lowers the threshold for reflex excitability. NSAIDs are not routinely used. Sedatives or muscle relaxants are used to help reduce titanic spasm and prevent seizures. Respiratory status is carefully evaluated for any signs of distress because muscle relaxants, opioids, and sedatives that may be prescribed may cause respiratory depression. Bronchodilators would not be used unless specifically indicated.

9. While evaluating an external monitor tracing of a woman in active labor whose labor is being induced, the nurse notes that the fetal heart rate (FHR) begins to decelerate at the onset of several contractions and returns to baseline before each contraction ends. The nurse should: a. change the woman's position. b. discontinue the oxytocin infusion. c. insert an internal monitor. d. document the finding in the patient's record.

ANS: D The FHR indicates early decelerations, which are not an ominous sign and do not require any intervention. The nurse should simply document these findings.

19. The nurse recognizes that a nonstress test (NST) in which two or more fetal heart rate (FHR) accelerations of 15 beats/min or more occur with fetal movement in a 20-minute period is: a. nonreactive. b. positive. c. negative. d. reactive.

ANS: D The NST is reactive (normal) when two or more FHR accelerations of at least 15 beats/min (each with a duration of at least 15 seconds) occur in a 20-minute period. A nonreactive result means that the heart rate did not accelerate during fetal movement. A positive result is not used with NST. Contraction stress test (CST) uses positive as a result term. A negative result is not used with NST. CST uses negative as a result term.

10. A woman is using the basal body temperature (BBT) method of contraception. She calls the clinic and tells the nurse, "My period is due in a few days, and my temperature has not gone up." The nurse's most appropriate response is: A. "This probably means that you're pregnant." B. "Don't worry; it's probably nothing." C. "Have you been sick this month?" D. "You probably didn't ovulate during this cycle."

ANS: D The absence of a temperature decrease most likely is the result of lack of ovulation. Pregnancy cannot occur without ovulation (which is being measured using the BBT method). A comment such as "Don't worry; it's probably nothing" discredits the patient's concerns. Illness would most likely cause an increase in BBT.

15. A woman currently uses a diaphragm and spermicide for contraception. She asks the nurse what the major differences are between the cervical cap and the diaphragm. The nurse's most appropriate response is: A. "No spermicide is used with the cervical cap, so it's less messy." B. "The diaphragm can be left in place longer after intercourse." C. "Repeated intercourse with the diaphragm is more convenient." D. "The cervical cap can safely be used for repeated acts of intercourse without adding more spermicide later."

ANS: D The cervical cap can be inserted hours before sexual intercourse without the need for additional spermicide later. No additional spermicide is required for repeated acts of intercourse. Spermicide should be used inside the cap as an additional chemical barrier. The cervical cap should remain in place for 6 hours after the last act of intercourse. Repeated intercourse with the cervical cap is more convenient because no additional spermicide is needed.

Myelosuppression, associated with chemotherapeutic agents or some malignancies such as leukemia, can cause bleeding tendencies because of a(n): a. decrease in leukocytes. b. increase in lymphocytes. c. vitamin C deficiency. d. decrease in blood platelets.

ANS: D The decrease in blood platelets secondary to the myelosuppression of chemotherapy can cause an increase in bleeding. The child and family should be alerted to avoid risk of injury. Decrease in leukocytes, increase in lymphocytes, and vitamin C deficiency would not affect bleeding tendencies.

19. Which statement regarding the care of a client in labor is correct and important to the nurse as he or she formulates the plan of care? a. The woman's blood pressure will increase during contractions and fall back to prelabor normal levels between contractions. b. The use of the Valsalva maneuver is encouraged during the second stage of labor to relieve fetal hypoxia. c. Having the woman point her toes will reduce leg cramps. d. Endogenous endorphins released during labor will raise the woman's pain threshold and produce sedation.

ANS: D The endogenous endorphins released during labor will raise the woman's pain threshold and produce sedation. In addition, physiologic anesthesia of the perineal tissues, caused by the pressure of the presenting part, decreases the mother's perception of pain. Blood pressure levels increase during contractions but remain somewhat elevated between them. The use of the Valsalva maneuver is discouraged during the second stage labor because of a number of unhealthy outcomes, including fetal hypoxia. Pointing the toes can cause leg cramps, as can the process of labor itself.

30. The nurse who is teaching a group of women about breast cancer would tell the women that: A. risk factors identify more than 50% of women who will develop breast cancer. B. nearly 90% of lumps found by women are malignant. C. 1 in 10 women in the United States will develop breast cancer in her lifetime. D. the exact cause of breast cancer is unknown.

ANS: D The exact cause of breast cancer is unknown. Risk factors help to identify less than 30% of women in whom breast cancer eventually will develop. Women detect about 90% of all breast lumps. Of this 90%, only 20% to 25% are malignant. One in eight women in the United States will develop breast cancer in her lifetime.

26. The priority nursing intervention after an amniotomy should be to: a. assess the color of the amniotic fluid. b. change the patient's gown. c. estimate the amount of amniotic fluid. d. assess the fetal heart rate.

ANS: D The fetal heart rate must be assessed immediately after the rupture of the membranes to determine whether cord prolapse or compression has occurred. Secondary to FHR assessment, amniotic fluid amount, color, odor, and consistency is assessed. Dry clothing is important for patient comfort; however, it is not the top priority.

26. If a woman complains of back labor pain, the nurse could best suggest that she: a. lie on her back for a while with her knees bent. b. do less walking around. c. take some deep, cleansing breaths. d. lean over a birth ball with her knees on the floor.

ANS: D The hands-and-knees position, with or without the aid of a birth ball, should help with the back pain. The supine position should be discouraged. Walking generally is encouraged.

The nurse is caring for an intubated infant with botulism in the pediatric intensive care unit. Which health care provider prescriptions should the nurse clarify with the health care provider before implementing? a. Administer 250 mg botulism immune globulin intravenously (BIG-IV) one time. b. Provide total parenteral nutrition (TPN) at 25 ml/hr intravenously. c. Titrate oxygen to keep pulse oximetry saturations greater than 92. d. Administer gentamicin sulfate (Garamycin) 10 mg per intravenous piggyback every 12 hours.

ANS: D The nurse should clarify the administration of an aminoglycoside antibiotic. Antibiotic therapy is not part of the management of infant botulism because the botulinum toxin is an intracellular molecule, and antibiotics would not be effective; aminoglycosides in particular should not be administered because they may potentiate the blocking effects of the neurotoxin. Treatment consists of immediate administration of botulism immune globulin intravenously (BIG-IV) without delaying for laboratory diagnosis. Early administration of BIG-IV neutralizes the toxin and stops the progression of the disease. The human-derived botulism antitoxin (BIG-IV) has been evaluated and is now available nationwide for use only in infant botulism. Approximately 50% of affected infants require intubation and mechanical ventilation; therefore, respiratory support is crucial, as is nutritional support because these infants are unable to feed.

7. The nurse who performs vaginal examinations to assess a woman's progress in labor should: a. perform an examination at least once every hour during the active phase of labor. b. perform the examination with the woman in the supine position. c. wear two clean gloves for each examination. d. discuss the findings with the woman and her partner.

ANS: D The nurse should discuss the findings of the vaginal examination with the woman and her partner and report them to the primary care provider. A vaginal examination should be performed only when indicated by the status of the woman and her fetus. The woman should be positioned to avoid supine hypotension. The examiner should wear a sterile glove while performing a vaginal examination for a laboring woman.

25. Nutrition is one of the most significant factors influencing the outcome of a pregnancy. It is an alterable and important preventive measure for various potential problems, such as low birth weight and prematurity. While completing the physical assessment of the pregnant patient, the nurse can evaluate the patient's nutritional status by observing a number of physical signs. Which sign would indicate that the patient has unmet nutritional needs? A. Normal heart rate, rhythm, and blood pressure B. Bright, clear, shiny eyes C. Alert, responsive, and good endurance D. Edema, tender calves, and tingling

ANS: D The physiologic changes of pregnancy may complicate the interpretation of physical findings. Lower-extremity edema often occurs when caloric and protein deficiencies are present; however, it may also be a common physical finding during the third trimester. It is essential that the nurse complete a thorough health history and physical assessment and request further laboratory testing if indicated. A malnourished pregnant patient may display rapid heart rate, abnormal rhythm, enlarged heart, and elevated blood pressure. A patient receiving adequate nutrition has bright, shiny eyes with no sores and moist, pink membranes. Pale or red membranes, dryness, infection, dull appearance of the cornea, or blue sclerae all are signs of poor nutrition. This patient is well nourished. Cachexia, listlessness, and tiring easily would be indications of poor nutritional status.

14. In her work with pregnant women of various cultures, a nurse practitioner has observed various practices that seemed strange or unusual. She has learned that cultural rituals and practices during pregnancy seem to have one purpose in common. Which statement best describes that purpose? A. To promote family unity B. To ward off the "evil eye" C. To appease the gods of fertility D. To protect the mother and fetus during pregnancy

ANS: D The purpose of all cultural practices is to protect the mother and fetus during pregnancy. Although many cultures consider pregnancy normal, certain practices are expected of women of all cultures to ensure a good outcome. Cultural prescriptions tell women what to do, and cultural proscriptions establish taboos. The purposes of these practices are to prevent maternal illness resulting from a pregnancy-induced imbalanced state and to protect the vulnerable fetus.

23. The nurse practicing in a labor setting knows that the woman most at risk for uterine rupture is: a. a gravida 3 who has had two low-segment transverse cesarean births. b. a gravida 2 who had a low-segment vertical incision for delivery of a 10-lb infant. c. a gravida 5 who had two vaginal births and two cesarean births. d. a gravida 4 who has had all cesarean births.

ANS: D The risk of uterine rupture increases for the patient who has had multiple prior births with no vaginal births. As the number of prior uterine incisions increases, so does the risk for uterine rupture. Low-segment transverse cesarean scars do not predispose the patient to uterine rupture.

28. A nurse may be called on to stimulate the fetal scalp: a. as part of fetal scalp blood sampling. b. in response to tocolysis. c. in preparation for fetal oxygen saturation monitoring. d. to elicit an acceleration in the fetal heart rate (FHR).

ANS: D The scalp can be stimulated using digital pressure during a vaginal examination. Fetal scalp blood sampling involves swabbing the scalp with disinfectant before a sample is collected. The nurse would stimulate the fetal scalp to elicit an acceleration of the FHR. Tocolysis is relaxation of the uterus. Fetal oxygen saturation monitoring involves the insertion of a sensor.

An 8-year-old girl is receiving a blood transfusion when the nurse notes that she has developed precordial pain, dyspnea, distended neck veins, slight cyanosis, and a dry cough. These manifestations are most suggestive of: a. air emboli. b. allergic reaction. c. hemolytic reaction. d. circulatory overload.

ANS: D The signs of circulatory overload include distended neck veins, hypertension, crackles, dry cough, cyanosis, and precordial pain. Signs of air embolism are sudden difficulty breathing, sharp pain in the chest, and apprehension. Allergic reactions are manifested by urticaria, pruritus, flushing, asthmatic wheezing, and laryngeal edema. Hemolytic reactions are characterized by chills, shaking, fever, pain at infusion site, nausea, vomiting, tightness in chest, flank pain, red or black urine, and progressive signs of shock and renal failure.

7. A woman who is 14 weeks pregnant tells the nurse that she always had a glass of wine with dinner before she became pregnant. She has abstained during her first trimester and would like to know if it is safe for her to have a drink with dinner now. The nurse would tell her: A. "Since you're in your second trimester, there's no problem with having one drink with dinner." B. "One drink every night is too much. One drink three times a week should be fine." C. "Since you're in your second trimester, you can drink as much as you like." D. "Because no one knows how much or how little alcohol it takes to cause fetal problems, the best course is to abstain throughout your pregnancy."

ANS: D The statement "Because no one knows how much or how little alcohol it takes to cause fetal problems, the best course is to abstain throughout your pregnancy" is accurate. A safe level of alcohol consumption during pregnancy has not yet been established. Although the consumption of occasional alcoholic beverages may not be harmful to the mother or her developing fetus, complete abstinence is strongly advised.

9. A woman is 3 months pregnant. At her prenatal visit, she tells the nurse that she does not know what is happening; one minute she's happy that she is pregnant, and the next minute she cries for no reason. Which response by the nurse is most appropriate? A. "Don't worry about it; you'll feel better in a month or so." B. "Have you talked to your husband about how you feel?" C. "Perhaps you really don't want to be pregnant." D. "Hormonal changes during pregnancy commonly result in mood swings."

ANS: D The statement "Hormonal changes during pregnancy commonly result in mood swings" is accurate and the most appropriate response by the nurse. The statement "Don't worry about it; you'll feel better in a month or so" dismisses the patient's concerns and is not the most appropriate response. Although women should be encouraged to share their feelings, "Have you talked to your husband about how you feel" is not the most appropriate response and does not provide the patient with a rationale for the psychosocial dynamics of her pregnancy. "Perhaps you really don't want to be pregnant" is completely inappropriate and deleterious to the psychologic well-being of the woman. Hormonal and metabolic adaptations often cause mood swings in pregnancy. The woman's responses are normal. She should be reassured about her feelings.

1. A 22-year-old woman pregnant with a single fetus has a preconception body mass index (BMI) of 24. When she was seen in the clinic at 14 weeks of gestation, she had gained 1.8 kg (4 lbs) since conception. How would the nurse interpret this? A. This weight gain indicates possible gestational hypertension. B. This weight gain indicates that the woman's infant is at risk for intrauterine growth restriction (IUGR). C. This weight gain cannot be evaluated until the woman has been observed for several more weeks. D. The woman's weight gain is appropriate for this stage of pregnancy.

ANS: D The statement "The woman's weight gain is appropriate for this stage of pregnancy" is accurate. This woman's BMI is within the normal range. During the first trimester, the average total weight gain is only 1 to 2 kg. Although weight gain does indicate possible gestational hypertension, it does not apply to this patient. The desirable weight gain during pregnancy varies among women. The primary factor to consider in making a weight gain recommendation is the appropriateness of the prepregnancy weight for the woman's height. A commonly used method of evaluating the appropriateness of weight for height is the BMI. Although weight gain does indicate risk for IUGR, this does not apply to this patient. Weight gain should occur at a steady rate throughout the pregnancy. The optimal rate of weight gain also depends on the stage of the pregnancy.

3. In evaluating the effectiveness of magnesium sulfate for the treatment of preterm labor, what finding would alert the nurse to possible side effects? a. Urine output of 160 mL in 4 hours b. Deep tendon reflexes 2+ and no clonus c. Respiratory rate of 16 breaths/min d. Serum magnesium level of 10 mg/dL

ANS: D The therapeutic range for magnesium sulfate management is 5 to 8 mg/dL. A serum magnesium level of 10 mg/dL could lead to signs and symptoms of magnesium toxicity, including oliguria and respiratory distress. Urine output of 160 mL in 4 hours, deep tendon reflexes 2+ with no clonus, and respiratory rate of 16 breaths/min are normal findings.

9. In comparing the abdominal and transvaginal methods of ultrasound examination, nurses should explain to their patients that: A. both require the woman to have a full bladder. B. the abdominal examination is more useful in the first trimester. C. initially the transvaginal examination can be painful. D. the transvaginal examination allows pelvic anatomy to be evaluated in greater detail.

ANS: D The transvaginal examination allows pelvic anatomy to be evaluated in greater detail and allows intrauterine pregnancies to be diagnosed earlier. The abdominal examination requires a full bladder; the transvaginal examination requires an empty bladder. The transvaginal examination is more useful in the first trimester; the abdominal examination works better after the first trimester. Neither method should be painful, although with the transvaginal examination the woman feels pressure as the probe is moved.

10. The long-term treatment plan for an adolescent with an eating disorder focuses on: A. managing the effects of malnutrition. B. establishing sufficient caloric intake. C. improving family dynamics. D. restructuring perception of body image.

ANS: D The treatment of eating disorders is initially focused on reestablishing physiologic homeostasis. Once body systems are stabilized, the next goal of treatment for eating disorders is maintaining adequate caloric intake. Although family therapy is indicated when dysfunctional family relationships exist, the primary focus of therapy for eating disorders is to help the adolescent cope with complex issues. The focus of treatment in individual therapy for an eating disorder involves restructuring cognitive perceptions about the individual's body image.

1. The two primary functions of the ovaries are: A. normal female development and sex hormone release. B. ovulation and internal pelvic support. C. sexual response and ovulation. D. ovulation and hormone production.

ANS: D The two functions of the ovaries are ovulation and hormone production. The presence of ovaries does not guarantee normal female development. The ovaries produce estrogen, progesterone, and androgen. Ovulation is the release of a mature ovum from the ovary; the ovaries are not responsible for internal pelvic support. Sexual response is a feedback mechanism involving the hypothalamus, anterior pituitary gland, and the ovaries. Ovulation does occur in the ovaries.

8. A multiparous woman has been in labor for 8 hours. Her membranes have just ruptured. The nurse's initial response would be to: a. prepare the woman for imminent birth. b. notify the woman's primary health care provider. c. document the characteristics of the fluid. d. assess the fetal heart rate and pattern.

ANS: D The umbilical cord may prolapse when the membranes rupture. The fetal heart rate and pattern should be monitored closely for several minutes immediately after ROM to ascertain fetal well-being, and the findings should be documented. Rupture of membranes (ROM) may increase the intensity and frequency of the uterine contractions, but it does not indicate that birth is imminent. The nurse may notify the primary care provider after ROM occurs and fetal well-being and the response to ROM have been assessed. The nurse's priority is to assess fetal well-being. The nurse should document the characteristics of the amniotic fluid, but the initial response is to assess fetal well-being and the response to ROM.

37. The nurse providing care in a women's health care setting must be aware regarding which sexually transmitted infection that can be successfully treated and cured? A. Herpes B. Acquired immunodeficiency syndrome (AIDS) C. Venereal warts D. Chlamydia

ANS: D The usual treatment for infection by the bacterium Chlamydia is doxycycline or azithromycin. Concurrent treatment of all sexual partners is needed to prevent recurrence. There is no known cure for herpes, and treatment focuses on pain relief and preventing secondary infections. Because there is no known cure for AIDS, prevention and early detection are the primary focus of care management. Condylomata acuminata are caused by human papillomavirus. No treatment eradicates the virus.

Which immunization should not be given to a child receiving chemotherapy for cancer? a. Tetanus vaccine b. Inactivated poliovirus vaccine c. Diphtheria, pertussis, tetanus (DPT) d. Measles, rubella, mumps

ANS: D The vaccine used for measles, mumps, and rubella is a live virus and can result in an overwhelming infection. Tetanus vaccine, inactivated poliovirus vaccine, and diphtheria, pertussis, tetanus (DPT) are not live virus vaccines.

17. What represents a typical progression through the phases of a woman's establishing a relationship with the fetus? A. Accepts the fetus as distinct from herself—accepts the biologic fact of pregnancy—has a feeling of caring and responsibility. B. Fantasizes about the child's gender and personality—views the child as part of herself—becomes introspective. C. Views the child as part of herself—has feelings of well-being—accepts the biologic fact of pregnancy. D. "I am pregnant."—"I am going to have a baby."—"I am going to be a mother."

ANS: D The woman first centers on herself as pregnant, then on the baby as an entity separate from herself, and then on her responsibilities as a mother. The expressions, "I am pregnant," "I am going to have a baby," and "I am going to be a mother" sum up the progression through the three phases.

The nurse is talking to a parent with a child who has a latex allergy. Which statement by the parent would indicate a correct understanding of the teaching? a. My child will have an allergic reaction if he comes in contact with yeast products. b. My child may have an upset stomach if he eats a food made with wheat or barley. c. My child will probably develop an allergy to peanuts. d. My child should not eat bananas or kiwis.

ANS: D There are cross-reactions between latex allergies and a number of foods such as bananas, avocados, kiwi, and chestnuts. Children with a latex allergy will not develop allergies to other food products such as yeast, wheat, barley, or peanuts.

The nurse is doing a prehospitalization orientation for Kayla, age 7, who is scheduled for cardiac surgery. As part of the preparation, the nurse explains that Kayla will not be able to talk because of an endotracheal tube but that she will be able to talk when it is removed. This explanation is: a. Unnecessary. b. The surgeons responsibility. c. Too stressful for a young child. d. An appropriate part of the childs preparation.

ANS: D This is a necessary part of preoperative preparation that will help reduce the anxiety associated with surgery. If the child wakes and is not prepared for the inability to speak, she will be even more anxious. It is a joint responsibility of nursing, medical staff, and child life personnel. This is a necessary component of preparation that will help reduce the anxiety associated with surgery.

37. A primigravida at 39 weeks of gestation is observed for 2 hours in the intrapartum unit. The fetal heart rate has been normal. Contractions are 5 to 9 minutes apart, 20 to 30 seconds in duration, and of mild intensity. Cervical dilation is 1 to 2 cm and uneffaced (unchanged from admission). Membranes are intact. The nurse should expect the woman to be: a. admitted and prepared for a cesarean birth. b. admitted for extended observation. c. discharged home with a sedative. d. discharged home to await the onset of true labor.

ANS: D This situation describes a woman with normal assessments who is probably in false labor and will likely not deliver rapidly once true labor begins. There is no indication that further assessments or observations are indicated; therefore, the patient will be discharged along with instructions to return when contractions increase in intensity and frequency. Neither a cesarean birth nor a sedative is required at this time.

28. Informed consent concerning contraceptive use is important because some of the methods: A. are invasive procedures that require hospitalization. B. require a surgical procedure to insert. C. may not be reliable. D. have potentially dangerous side effects.

ANS: D To make an informed decision about the use of contraceptives, it is important for couples to be aware of potential side effects. The only contraceptive method that is a surgical procedure and requires hospitalization is sterilization. Some methods have greater efficacy than others, and this should be included in the teaching.

13. You are evaluating the fetal monitor tracing of your patient, who is in active labor. Suddenly you see the fetal heart rate (FHR) drop from its baseline of 125 beats/min down to 80 beats/min. You reposition the mother, provide oxygen, increase intravenous (IV) fluid, and perform a vaginal examination. The cervix has not changed. Five minutes have passed, and the fetal heart rate remains in the 80s. What additional nursing measures should you take? a. Call for staff assistance. b. Insert a Foley catheter. c. Start Pitocin. d. Notify the care provider immediately.

ANS: D To relieve an FHR deceleration, the nurse can reposition the mother, increase IV fluid, and provide oxygen. If oxytocin is infusing, it should be discontinued. If the FHR does not resolve, the primary care provider should be notified immediately. Inserting a Foley catheter is an inappropriate nursing action. If the FHR were to continue in a nonreassuring pattern, a cesarean section could be warranted, which would require a Foley catheter. However, the physician must make that determination. Pitocin may place additional stress on the fetus.

18. Maternity nurses often have to answer questions about the many, sometimes unusual ways people have tried to make the birthing experience more comfortable. For instance, nurses should be aware that: a. music supplied by the support person has to be discouraged because it could disturb others or upset the hospital routine. b. women in labor can benefit from sitting in a bathtub, but they must limit immersion to no longer than 15 minutes at a time. c. effleurage is permissible, but counterpressure is almost always counterproductive. d. electrodes attached to either side of the spine to provide high-intensity electrical impulses facilitate the release of endorphins.

ANS: D Transcutaneous electrical nerve stimulation does help. Music may be very helpful for reducing tension and certainly can be accommodated by the hospital. Women can stay in a bath as long as they want, although repeated baths with breaks may be more effective than a long soak. Counterpressure can help the woman cope with lower back pain.

5. A 40-year-old woman is 10 weeks pregnant. Which diagnostic tool would be appropriate to suggest to her at this time? A. Biophysical profile (BPP) B. Amniocentesis C. Maternal serum alpha-fetoprotein (MSAFP) screening D. Transvaginal ultrasound

ANS: D Ultrasound would be performed at this gestational age for biophysical assessment of the infant. BPP would be a method of biophysical assessment of fetal well-being in the third trimester. Amniocentesis is performed after the 14th week of pregnancy. MSAFP screening is performed from week 15 to week 22 of gestation (weeks 16 to 18 are ideal).

32. To determine the cultural influence on a patient's diet, the nurse should first: A. evaluate the patient's weight gain during pregnancy. B. assess the socioeconomic status of the patient. C. discuss the four food groups with the patient. D. identify the food preferences and methods of food preparation common to that culture.

ANS: D Understanding the patient's food preferences and how she prepares food will assist the nurse in determining whether the patient's culture is adversely affecting her nutritional intake. Evaluation of a patient's weight gain during pregnancy should be included for all patients, not just for patients who are culturally different. The socioeconomic status of the patient may alter the nutritional intake but not the cultural influence. Teaching the food groups to the patient should come after assessing food preferences.

1. A man smokes two packs of cigarettes a day. He wants to know if smoking is contributing to the difficulty he and his wife are having getting pregnant. The nurse's most appropriate response is: A. "Your sperm count seems to be okay in the first semen analysis." B. "Only marijuana cigarettes affect sperm count." C. "Smoking can give you lung cancer, even though it has no effect on sperm." D. "Smoking can reduce the quality of your sperm."

ANS: D Use of tobacco, alcohol, and marijuana may affect sperm counts. "Your sperm count seems to be okay in the first semen analysis" is inaccurate. Sperm counts vary from day to day and depend on emotional and physical status and sexual activity. A single analysis may be inconclusive. A minimum of two analyses must be performed several weeks apart to assess male fertility.

23. In documenting labor experiences, nurses should know that a uterine contraction is described according to all these characteristics except: a. frequency (how often contractions occur). b. intensity (the strength of the contraction at its peak). c. resting tone (the tension in the uterine muscle). d. appearance (shape and height).

ANS: D Uterine contractions are described in terms of frequency, intensity, duration, and resting tone.

6. A pregnant woman is at 38 weeks of gestation. She wants to know whether there are any signs that "labor is getting close to starting." Which finding is an indication that labor may begin soon? a. Weight gain of 1.5 to 2 kg (3 to 4 lb) b. Increase in fundal height c. Urinary retention d. Surge of energy

ANS: D Women speak of having a burst of energy before labor. The woman may lose 0.5 to 1.5 kg, as a result of water loss caused by electrolyte shifts that, in turn, are caused by changes in the estrogen and progesterone levels. When the fetus descends into the true pelvis (called lightening), the fundal height may decrease. Urinary frequency may return before labor.

3. In planning for the care of a 30-year-old woman with pregestational diabetes, the nurse recognizes that the most important factor affecting pregnancy outcome is the: A. mother's age. B. number of years since diabetes was diagnosed. C. amount of insulin required prenatally. D. degree of glycemic control during pregnancy.

ANS: D Women with excellent glucose control and no blood vessel disease should have good pregnancy outcomes.

19. Which vitamins or minerals can lead to congenital malformations of the fetus if taken in excess by the mother? A. Zinc B. Vitamin D C. Folic acid D. Vitamin A

ANS: D Zinc, vitamin D, and folic acid are vital to good maternal and fetal health and are highly unlikely to be consumed in excess. Vitamin A taken in excess causes a number of problems. An analog of vitamin A appears in prescribed acne medications, which must not be taken during pregnancy.

An African-American woman noticed some bruises on her newborn girls buttocks. She asks the nurse who spanked her daughter. The nurse explains that these marks are called: a. Lanugo. b. Vascular nevi. c. Nevus flammeus. d. Mongolian spots.

ANS: D A Mongolian spot is a bluish black area of pigmentation that may appear over any part of the exterior surface of the body. It is more commonly noted on the back and buttocks and most frequently is seen on infants whose ethnic origins are Mediterranean, Latin American, Asian, or African. Lanugo is the fine, downy hair seen on a term newborn. A vascular nevus, commonly called a strawberry mark, is a type of capillary hemangioma. A nevus flammeus, commonly called a port-wine stain, is most frequently found on the face. PTS: 1 DIF: Cognitive Level: Comprehension REF: 568 OBJ: Nursing Process: Diagnosis MSC: Client Needs: Health Promotion and Maintenance

During life in utero, oxygenation of the fetus occurs through transplacental gas exchange. When birth occurs, four factors combine to stimulate the respiratory center in the medulla. The initiation of respiration then follows. Which is not one of these essential factors? a. Chemical b. Mechanical c. Thermal d. Psychologic

ANS: D A psychologic factor is not one of the essential factors in the initiation of breathing; the fourth factor is sensory. The sensory factors include handling by the provider, drying by the nurse, lights, smells, and sounds. Chemical factors are essential for the initiation of breathing. During labor, decreased levels of oxygen and increased levels of carbon dioxide seem to have a cumulative effect that is involved in the initiation of breathing. Clamping of the cord may also contribute to the start of respirations. Prostaglandins are known to inhibit breathing, and clamping of the cord results in a drop in the level of prostaglandins. Mechanical factors also are necessary to initiate respirations. As the infant passes through the birth canal, the chest is compressed. With birth the chest is relaxed, which allows for negative intrathoracic pressure that encourages air to flow into the lungs. The profound change in temperature between intrauterine and extrauterine life stimulates receptors in the skin to communicate with the receptors in the medulla. This also contributes to the initiation of breathing. PTS: 1 DIF: Cognitive Level: Comprehension REF: 557 OBJ: Nursing Process: Implementation MSC: Client Needs: Physiologic Integrity

3. A nurse is reviewing a patient's chart and notes that the patient has a cancerous tumor that has invaded other organs. Based on this information, at which stage is this patient's cancer classified? A. Stage O B. Stage I C. Stage III D. Stage IV

ANS: D A stage IV cancer is one that has invaded other organs. Stage 0 is early cancer, present only in the cells in which it began. Stages I-III are more extensive, with larger tumors and spread to nearby lymph nodes or adjacent organs.

25. Cystic fibrosis (CF) is suspected in a toddler. Which test is essential in establishing this diagnosis? a. Bronchoscopy c. Urine creatinine b. Serum calcium d. Sweat chloride test

ANS: D A sweat chloride test result greater than 60 mEq/L is diagnostic of CF. Although bronchoscopy is helpful for identifying bacterial infection in children with CF, it is not diagnostic. Serum calcium is normal in children with CF. Urine creatinine is not diagnostic of CF.

31. A student nurse wants to provide nonpharmacological pain management interventions to a hospitalized child with cancer. Which action by the student causes the faculty member to intervene? A. Applying a moist heat pack B. Giving the child a massage C. Reading the child a story D. Using candles for aromatherapy

ANS: D Actions that have been reported by children to be effective pain control strategies are moist heat, massage, adequate rest and sleep, distraction (reading a story), and providing opportunities for social support. Open flames are prohibited in hospitals due to the risk of fire and explosion.

A postpartum woman telephones about her 4-day-old infant. She is not scheduled for a weight check until the infant is 10 days old, and she is worried about whether breastfeeding is going well. Effective breastfeeding is indicated by the newborn who: a. Sleeps for 6 hours at a time between feedings. b. Has at least one breast milk stool every 24 hours. c. Gains 1 to 2 ounces per week. d. Has at least six to eight wet diapers per day.

ANS: D After day 4, when the mothers milk comes in, the infant should have six to eight wet diapers every 24 hours. Sleeping for 6 hours between feedings is not an indication of whether the infant is breastfeeding well. Typically infants sleep 2 to 4 hours between feedings, depending on whether they are being fed on a 2- to 3- hour schedule or cluster fed. The infant should have a minimum of three bowel movements in a 24-hour period. Breastfed infants typically gain 15 to 30 g/day. PTS: 1 DIF: Cognitive Level: Comprehension REF: 646 OBJ: Nursing Process: Evaluation MSC: Client Needs: Health Promotion and Maintenance

A 25-year-old gravida 1 para 1 who had an emergency cesarean birth 3 days ago is scheduled for discharge. As you prepare her for discharge, she begins to cry. Your initial action should be to a. Assess her for pain. b. Point out how lucky she is to have a healthy baby. c. Explain that she is experiencing postpartum blues. d. Allow her time to express her feelings.

ANS: D Although many women experience transient postpartum blues, they need assistance in expressing their feelings. This condition affects 50% to 80% of new mothers. There should be no assumption that the patient is in pain, when in fact she may have no pain whatsoever. This is blocking communication and inappropriate in this situation. The patient needs the opportunity to express her feelings first; patient teaching can occur later. PTS: 1 DIF: Cognitive Level: Application REF: 517 OBJ: Nursing Process: Implementation MSC: Client Needs: Psychosocial Integrity

31. A mother asks the nurse what would be the first indication that acute glomerulonephritis is improving. The nurse's best response should be that the: a. Blood pressure will stabilize. . b. Child will have more energy. c. Urine will be free of protein d. Urinary output will increase.

ANS: D An increase in urinary output may signal resolution of the acute glomerulonephritis. If blood pressure is elevated, stabilization usually occurs with the improvement in renal function. The child having more energy and the urine being free of protein are related to the improvement in urinary output.

With regard to hemolytic diseases of the newborn, nurses should be aware that: a. Rh incompatibility matters only when an Rh-negative child is born to an Rh-positive mother. b. ABO incompatibility is more likely than Rh incompatibility to precipitate significant anemia. c. Exchange transfusions frequently are required in the treatment of hemolytic disorders. d. The indirect Coombs test is performed on the mother before birth; the direct Coombs test is performed on the cord blood after birth.

ANS: D An indirect Coombs test may be performed on the mother a few times during pregnancy. Only the Rh-positive child of an Rh-negative mother is at risk. ABO incompatibility is more common than Rh incompatibility but causes less severe problems; significant anemia, for instance, is rare with ABO. Exchange transfers are needed infrequently because of the decrease in the incidence of severe hemolytic disease in newborns from Rh incompatibility. PTS: 1 DIF: Cognitive Level: Comprehension REF: 680 OBJ: Nursing Process: Planning MSC: Client Needs: Health Promotion and Maintenance

An Apgar score of 10 at 1 minute after birth would indicate a(n): a. Infant having no difficulty adjusting to extrauterine life and needing no further testing. b. Infant in severe distress who needs resuscitation. c. Prediction of a future free of neurologic problems. d. Infant having no difficulty adjusting to extrauterine life but who should be assessed again at 5 minutes after birth.

ANS: D An initial Apgar score of 10 is a good sign of healthy adaptation; however, it must be repeated at the 5-minute mark. PTS: 1 DIF: Cognitive Level: Comprehension REF: 583 OBJ: Nursing Process: Planning MSC: Client Needs: Physiologic Integrity

Postbirth uterine/vaginal discharge, called lochia: a. Is similar to a light menstrual period for the first 6 to 12 hours. b. Is usually greater after cesarean births. c. Will usually decrease with ambulation and breastfeeding. d. Should smell like normal menstrual flow unless an infection is present.

ANS: D An offensive odor usually indicates an infection. Lochia flow should approximate a heavy menstrual period for the first 2 hours and then steadily decrease. Less lochia usually is seen after cesarean births and usually increases with ambulation and breastfeeding. PTS: 1 DIF: Cognitive Level: Comprehension REF: 484 OBJ: Nursing Process: Assessment MSC: Client Needs: Health Promotion and Maintenance

The nurse caring for the postpartum woman understands that breast engorgement is caused by: a. Overproduction of colostrum. b. Accumulation of milk in the lactiferous ducts. c. Hyperplasia of mammary tissue. d. Congestion of veins and lymphatics.

ANS: D Breast engorgement is caused by the temporary congestion of veins and lymphatics, not by overproduction of colostrum, overproduction of milk, or hyperplasia of mammary tissue. PTS: 1 DIF: Cognitive Level: Knowledge REF: 487 OBJ: Nursing Process: Assessment MSC: Client Needs: Health Promotion and Maintenance

Which statement concerning the benefits or limitations of breastfeeding is inaccurate? a. Breast milk changes over time to meet changing needs as infants grow. b. Long-term studies have shown that the benefits of breast milk continue after the infant is weaned. c. Breast milk/breastfeeding may enhance cognitive development. d. Breastfeeding increases the risk of childhood obesity.

ANS: D Breastfeeding actually decreases the risk of childhood obesity. There are multiple benefits of breastfeeding. Breast milk changes over time to meet changing needs as infants grow. Long-term studies have shown that the benefits of breast milk continue after the infant is weaned. Breast milk/breastfeeding may enhance cognitive development. PTS: 1 DIF: Cognitive Level: Comprehension REF: 635 OBJ: Nursing Process: Planning MSC: Client Needs: Health Promotion and Maintenance

Which statement describing physiologic jaundice is incorrect? a. Neonatal jaundice is common, but kernicterus is rare. b. The appearance of jaundice during the first 24 hours or beyond day 7 indicates a pathologic process. c. Because jaundice may not appear before discharge, parents need instruction on how to assess it and when to call for medical help. d. Breastfed babies have a lower incidence of jaundice.

ANS: D Breastfeeding is associated with an increased incidence of jaundice. Neonatal jaundice occurs in 60% of newborns; the complication called kernicterus is rare. Jaundice in the first 24 hours or that persists past day 7 is cause for medical concern. Parents need to know how to assess for jaundice in their newborn. PTS: 1 DIF: Cognitive Level: Comprehension REF: 566 OBJ: Nursing Process: Diagnosis MSC: Client Needs: Health Promotion and Maintenance

Nursing activities that promote parent-infant attachment are many and varied. One activity that should not be overlooked is management of the environment. While providing routine mother-baby care, the nurse should ensure that: a. The baby is able to return to the nursery at night so that the new mother can sleep. b. Routine times for care are established to reassure the parents. c. The father should be encouraged to go home at night to prepare for mother-baby discharge. d. An environment that fosters as much privacy as possible should be created.

ANS: D Care providers need to knock before gaining entry. Nursing care activities should be grouped. Once the baby has demonstrated adjustment to extrauterine life (either in the mothers room or the transitional nursery), all care should be provided in one location. This important principle of family-centered maternity care fosters attachment by giving parents the opportunity to learn about their infant 24 hours a day. One nurse should provide care to both mother and baby in this couplet care or rooming-in model. It is not necessary for the baby to return to the nursery at night. In fact, the mother will sleep better with the infant close by. Care should be individualized to meet the parents needs, not the routines of the staff. Teaching goals should be developed in collaboration with the parents. The father or other significant other should be permitted to sleep in the room with the mother. The maternity unit should develop policies that allow for the presence of significant others as much as the new mother desires. PTS: 1 DIF: Cognitive Level: Application REF: 528 OBJ: Nursing Process: Implementation MSC: Client Needs: Health Promotion and Maintenance

14. Skin testing for tuberculosis (the Mantoux test) is recommended: a. Every year for all children older than 2 years. b. Every year for all children older than 10 years. c. Every 2 years for all children starting at age 1 year. d. Periodically for children who reside in high-prevalence regions

ANS: D Children who reside in high prevalence regions for tuberculosis should be tested every 2 to 3 years. Annual testing is not necessary. Testing is not necessary unless exposure is likely or an underlying medical risk factor is present.

20. Which clinical manifestation would be seen in a child with chronic renal failure? a. Hypotension b. Massive hematuria c. Hypokalemia d. Unpleasant "uremic" breath odor

ANS: D Children with chronic renal failure have a characteristic breath odor resulting from the retention of waste products. Hypertension may be a complication of chronic renal failure. With chronic renal failure, little or no urine output occurs. Hyperkalemia is a concern in chronic renal failure.

If nonsurgical treatment for late postpartum hemorrhage is ineffective, which surgical procedure is appropriate to correct the cause of this condition? a. Hysterectomy b. Laparoscopy c. Laparotomy d. D&C

ANS: D D&C allows examination of the uterine contents and removal of any retained placental fragments or blood clots. Hysterectomy is the removal of the uterus and is not indicated for this condition. A laparoscopy is the insertion of an endoscope through the abdominal wall to examine the peritoneal cavity. It is not the appropriate treatment for this condition. A laparotomy is also not indicated for this condition. A laparotomy is a surgical incision into the peritoneal cavity to explore it. PTS: 1 DIF: Cognitive Level: Comprehension REF: 532 OBJ: Nursing Process: Assessment MSC: Client Needs: Physiologic Integrity

Children taking phenobarbital (phenobarbital sodium) and/or phenytoin (Dilantin) may experience a deficiency of: a. calcium. b. vitamin C. c. fat-soluble vitamins. d. vitamin D and folic acid.

ANS: D Deficiencies of vitamin D and folic acid have been reported in children taking phenobarbital and phenytoin. Calcium, vitamin C, and fat-soluble vitamin deficiencies are not associated with phenobarbital or phenytoin.

Nurses need to know the basic definitions and incidence data about postpartum hemorrhage (PPH). For instance: a. PPH is easy to recognize early; after all, the woman is bleeding. b. Traditionally it takes more than 1000 mL of blood after vaginal birth and 2500 mL after cesarean birth to define the condition as PPH. c. If anything, nurses and doctors tend to overestimate the amount of blood loss. d. Traditionally PPH has been classified as early or late with respect to birth.

ANS: D Early PPH is also known as primary, or acute, PPH; late PPH is known as secondary PPH. Unfortunately PPH can occur with little warning and often is recognized only after the mother has profound symptoms. Traditionally a 500-mL blood loss after a vaginal birth and a 1000-mL blood loss after a cesarean birth constitute PPH. Medical personnel tend to underestimate blood loss by as much as 50% in their subjective observations. PTS: 1 DIF: Cognitive Level: Knowledge REF: 530 OBJ: Nursing Process: Diagnosis MSC: Client Needs: Physiologic Integrity

When teaching parents about mandatory newborn screening, it is important for the nurse to explain that the main purpose is to: a. Keep the state records updated. b. Allow accurate statistical information. c. Document the number of births. d. Recognize and treat newborn disorders early.

ANS: D Early treatment of disorders will prevent morbidity associated with inborn errors of metabolism or other genetic conditions. Keeping records and reporting for statistical purposes are not the primary reason for the screening test. The number of births recorded is not reported from the newborn screening test. PTS: 1 DIF: Cognitive Level: Application REF: 609 OBJ: Nursing Process: Assessment MSC: Client Needs: Physiologic Integrity

27. Pancreatic enzymes are administered to the child with cystic fibrosis. Nursing considerations should include: a. Do not administer pancreatic enzymes if the child is receiving antibiotics. b. Decrease dose of pancreatic enzymes if the child is having frequent, bulky stools. c. Administer pancreatic enzymes between meals if at all possible. d. Pancreatic enzymes can be swallowed whole or sprinkled on a small amount of food taken at the beginning of a meal.

ANS: D Enzymes may be administered in a small amount of cereal or fruit or swallowed whole at the beginning of a meal, not between meals. Pancreatic enzymes are not contraindicated with antibiotics. The dose of enzymes should be increased if the child is having frequent, bulky stools.

35. Which vitamin supplements are necessary for children with cystic fibrosis? a. Vitamin C and calcium c. Magnesium b. Vitamins B6 and B12 d. Vitamins A, D, E, and K

ANS: D Fat-soluble vitamins are poorly absorbed because of deficient pancreatic enzymes in children with cystic fibrosis; therefore, supplements are necessary. Vitamin C and calcium are not fat soluble. Vitamins B6 and B12 are not fat-soluble vitamins. Magnesium is a mineral, not a vitamin.

Which maternal event is abnormal in the early postpartum period? a. Diuresis and diaphoresis b. Flatulence and constipation c. Extreme hunger and thirst d. Lochial color changes from rubra to alba

ANS: D For the first 3 days after childbirth, lochia is termed rubra. Lochia serosa follows, and then at about 11 days, the discharge becomes clear, colorless, or white. Diuresis and diaphoresis are the methods by which the body rids itself of increased plasma volume. Urine output of 3000 mL/day is common for the first few days after delivery and is facilitated by hormonal changes in the mother. Bowel tone remains sluggish for days. Many women anticipate pain during defecation and are unwilling to exert pressure on the perineum. The new mother is hungry because of energy used in labor and thirsty because of fluid restrictions during labor. PTS: 1 DIF: Cognitive Level: Knowledge REF: 485 OBJ: Nursing Process: Assessment MSC: Client Needs: Health Promotion and Maintenance

The mother of a 1-month-old infant tells the nurse she worries that her baby will get meningitis like her oldest son did when he was an infant. The nurse should base her response on which statement? a. Meningitis rarely occurs during infancy. b. Often a genetic predisposition to meningitis is found. c. Vaccination to prevent all types of meningitis is now available. d. Vaccination to prevent Haemophilus influenzae type B meningitis has decreased the frequency of this disease in children.

ANS: D H. influenzae type B meningitis has been virtually eradicated in areas of the world where the vaccine is administered routinely. Bacterial meningitis remains a serious illness in children. It is significant because of the residual damage caused by undiagnosed and untreated or inadequately treated cases. The leading causes of neonatal meningitis are the group B streptococci and Escherichia coli organisms. Meningitis is an extension of a variety of bacterial infections. No genetic predisposition exists. Vaccinations are not available for all of the potential causative organisms.

After giving birth to a healthy infant boy, a primiparous woman, 16 years old, is admitted to the postpartum unit. An appropriate nursing diagnosis for her at this time is Risk for impaired parenting related to deficient knowledge of newborn care. In planning for the womans discharge, what should the nurse be certain to include in the plan of care? a. Instruct the patient how to feed and bathe her infant. b. Give the patient written information on bathing her infant. c. Advise the patient that all mothers instinctively know how to care for their infants. d. Provide time for the patient to bathe her infant after she views an infant bath demonstration.

ANS: D Having the mother demonstrate infant care is a valuable method of assessing the clients understanding of her newly acquired knowledge, especially in this age group, because she may inadvertently neglect her child. Although verbalizing how to care for the infant is a form of patient education, it is not the most developmentally appropriate teaching for a teenage mother. Advising the patient that all mothers instinctively know how to care for their infants is an inappropriate statement; it is belittling and false. PTS: 1 DIF: Cognitive Level: Application REF: 523 OBJ: Nursing Process: Diagnosis, Planning MSC: Client Needs: Health Promotion and Maintenance

Which condition, not uncommon in pregnancy, is likely to require careful medical assessment during the puerperium? a. Varicosities of the legs b. Carpal tunnel syndrome c. Periodic numbness and tingling of the fingers d. Headaches

ANS: D Headaches in the postpartum period can have a number of causes, some of which deserve medical attention. Total or nearly total regression of varicosities is expected after childbirth. Carpal tunnel syndrome is relieved in childbirth when the compression on the median nerve is lessened. Periodic numbness of the fingers usually disappears after birth unless carrying the baby aggravates the condition. PTS: 1 DIF: Cognitive Level: Comprehension REF: 489 OBJ: Nursing Process: Evaluation MSC: Client Needs: Physiologic Integrity

When caring for a postpartum woman experiencing hemorrhagic shock, the nurse recognizes that the most objective and least invasive assessment of adequate organ perfusion and oxygenation is: a. Absence of cyanosis in the buccal mucosa. b. Cool, dry skin. c. Diminished restlessness. d. Urinary output of at least 30 mL/hr.

ANS: D Hemorrhage may result in hemorrhagic shock. Shock is an emergency situation in which the perfusion of body organs may become severely compromised and death may occur. The presence of adequate urinary output indicates adequate tissue perfusion. The assessment of the buccal mucosa for cyanosis can be subjective. The presence of cool, pale, clammy skin would be an indicative finding associated with hemorrhagic shock. Hemorrhagic shock is associated with lethargy, not restlessness. PTS: 1 DIF: Cognitive Level: Analysis REF: 536 OBJ: Nursing Process: Assessment MSC: Client Needs: Physiologic Integrity

9. Which statement is characteristic of acute otitis media (AOM)? a. The etiology is unknown. b. Permanent hearing loss often results. c. It can be treated by intramuscular antibiotics. d. It is treated with a broad range of antibiotics.

ANS: D Historically AOM has been treated with a range of antibiotics, and it is the most common disorder treated with antibiotics in the ambulatory setting. The etiology of AOM may be bacterial, such as Streptococcus pneumoniae, Haemophilus influenzae, and Moraxella catarrhalis, or a viral agent. Recent concerns about drugresistant organisms have caused authorities to recommend judicious use of antibiotics and that antibiotics are not required for initial treatment. Permanent hearing loss is not a frequent cause of properly treated AOM. Intramuscular antibiotics are not necessary. Oral amoxicillin is the treatment of choice.

5.The nurse is assisting the pediatric provider with a newborn examination. The provider notes that the infant has hypospadias. The nurse understands that hypospadias refers to: a. Absence of a urethral opening. b. Penis shorter than usual for age. c. Urethral opening along dorsal surface of penis. d. Urethral opening along ventral surface of penis.

ANS: D Hypospadias is a congenital condition in which the urethral opening is located anywhere along the ventral surface of the penis. The urethral opening is present, but not at the glans. Hypospadias does not refer to the size of the penis. When the urethral opening is along the dorsal surface of the penis, it is known as epispadias.

Which should cause a nurse to suspect that an infection has developed under a cast? a. Complaint of paresthesia b. Cold toes c. Increased respirations d. "Hot spots" felt on cast surface

ANS: D If hot spots are felt on the cast surface, they usually indicate infection beneath the area. This should be reported so that a window can be made in the cast to observe the site. The five Ps of ischemia from a vascular injury are pain, pallor, pulselessness, paresthesia, and paralysis. Paresthesia is an indication of vascular injury, not infection. Cold toes may be indicative of too tight a cast and need further evaluation. Increased respirations may be indicative of a respiratory tract infection or pulmonary emboli. This should be reported, and child should be evaluated.

A 25-year-old multiparous woman gave birth to an infant boy 1 day ago. Today her husband brings a large container of brown seaweed soup to the hospital. When the nurse enters the room, the husband asks for help with warming the soup so that his wife can eat it. The nurses most appropriate response is to ask the woman: a. Didnt you like your lunch? b. Does your doctor know that you are planning to eat that? c. What is that anyway? d. Ill warm the soup in the microwave for you.

ANS: D Ill warm the soup in the microwave for you shows cultural sensitivity to the dietary preferences of the woman and is the most appropriate response. Cultural dietary preferences must be respected. Women may request that family members bring favorite or culturally appropriate foods to the hospital. What is that anyway? does not show cultural sensitivity. PTS: 1 DIF: Cognitive Level: Application REF: 502 OBJ: Nursing Process: Implementation MSC: Client Needs: Psychosocial Integrity

3. Which statement describes the cognitive abilities of school-age children? a. Have developed the ability to reason abstractly b. Become capable of scientific reasoning and formal logic c. Progress from making judgments based on what they reason to making judgments based on what they see d. Have the ability to classify, group and sort, and hold a concept in their minds while making decisions based on that concept

ANS: D In Piaget's stage of concrete operations, children have the ability to group and sort and make conceptual decisions. Children cannot reason abstractly until late adolescence. Scientific reasoning and formal logic are skills of adolescents. Making judgments on what the child sees versus what he or she reasons is not a developmental skill.

Because a full bladder prevents the uterus from contracting normally, nurses intervene to help the woman empty her bladder spontaneously as soon as possible. If all else fails, the last thing the nurse could try is: a. Pouring water from a squeeze bottle over the womans perineum. b. Placing oil of peppermint in a bedpan under the woman. c. Asking the physician to prescribe analgesics. d. Inserting a sterile catheter.

ANS: D Invasive procedures usually are the last to be tried, especially with so many other simple and easy methods available (e.g., water, peppermint vapors, pain medication). Pouring water over the perineum may stimulate voiding. It is easy, noninvasive, and should be tried early. The oil of peppermint releases vapors that may relax the necessary muscles. If the woman is anticipating pain from voiding, pain medications may be helpful. Other nonmedical means and pain medication should be tried before insertion of a catheter. PTS: 1 DIF: Cognitive Level: Comprehension REF: 498 OBJ: Nursing Process: Implementation MSC: Client Needs: Health Promotion and Maintenance

The nurse is taking care of an adolescent diagnosed with kyphosis. Which describes this condition? a. Lateral curvature of the spine b. Immobility of the shoulder joint c. Exaggerated concave lumbar curvature of the spine d. Increased convex angulation in the curve of the thoracic spine

ANS: D Kyphosis is an abnormally increased convex angulation in the curve of the thoracic spine. Scoliosis is a complex spinal deformity usually involving lateral curvature, spinal rotation causing rib asymmetry, and thoracic hypokyphosis. Ankylosis is the immobility of a joint. Lordosis is an exaggerated concave lumbar curvature of the spine.

2.Which diagnostic finding is present when a child has primary nephrotic syndrome? a. Hyperalbuminemia b. Positive ASO titer c. Leukocytosis d. Proteinuria

ANS: D Large amounts of protein are lost through the urine as a result of an increased permeability of the glomerular basement membrane. Hypoalbuminemia is present because of loss of albumin through the defective glomerulus and the liver's inability to synthesize proteins to balance the loss. ASO titer is negative in a child with primary nephrotic syndrome. Leukocytosis is not a diagnostic finding in primary nephrotic syndrome.

A woman gave birth to a healthy infant boy 5 days ago. What type of lochia would the nurse expect to find when assessing this woman? a. Lochia rubra b. Lochia sangra c. Lochia alba d. Lochia serosa

ANS: D Lochia serosa, which consists of blood, serum, leukocytes, and tissue debris, generally occurs around day 3 or 4 after childbirth. Lochia rubra consists of blood and decidual and trophoblastic debris. The flow generally lasts 3 to 4 days and pales, becoming pink or brown. There is no such term as lochia sangra. Lochia alba occurs in most women after day 10 and can continue up to 6 weeks after childbirth. PTS: 1 DIF: Cognitive Level: Comprehension REF: 484 OBJ: Nursing Process: Assessment MSC: Client Needs: Health Promotion and Maintenance

To provide adequate postpartum care, the nurse should be aware that postpartum depression (PPD) with psychotic features: a. Is more likely to occur in women with more than two children. b. Is rarely delusional and then is usually about someone trying to harm her (the mother). c. Although serious, is not likely to need psychiatric hospitalization. d. May include bipolar disorder (formerly called manic depression).

ANS: D Manic mood swings are possible. PPD is more likely to occur in first-time mothers. Delusions may be present in 50% of women with PPD, usually about something being wrong with the infant. PPD with psychosis is a psychiatric emergency that requires hospitalization. PTS: 1 DIF: Cognitive Level: Comprehension REF: 551 OBJ: Nursing Process: Assessment MSC: Client Needs: Psychosocial Integrity

Nursing care in the fourth trimester includes an important intervention sometimes referred to as taking the time to mother the mother. Specifically this expression refers to: a. Formally initializing individualized care by confirming the womans and infants identification (ID) numbers on their respective wrist bands. (This is your baby.) b. Teaching the mother to check the identity of any person who comes to remove the baby from the room. (Its a dangerous world out there.) c. Including other family members in the teaching of self-care and child care. (Were all in this together.) d. Nurturing the woman by providing encouragement and support as she takes on the many tasks of motherhood.

ANS: D Many professionals believe that the nurses nurturing and support function is more important than providing physical care and teaching. Matching ID wrist bands is more of a formality, but it is also a get-acquainted procedure. Mothering the mother is more a process of encouraging and supporting the woman in her new role. Having the mother check IDs is a security measure for protecting the baby from abduction. Teaching the whole family is just good nursing practice. PTS: 1 DIF: Cognitive Level: Comprehension REF: 492 OBJ: Nursing Process: Implementation MSC: Client Needs: Psychosocial Integrity

Which medication is usually tried first when a child is diagnosed with juvenile idiopathic arthritis (JIA)? a. Aspirin b. Corticosteroids c. Cytotoxic drugs such as methotrexate d. Nonsteroidal anti-inflammatory drugs (NSAIDs)

ANS: D NSAIDs are the first drugs used in JIA. Naproxen, ibuprofen, and tolmetin are approved for use in children. Aspirin, once the drug of choice, has been replaced by the NSAIDs because they have fewer side effects and easier administration schedules. Corticosteroids are used for life-threatening complications, incapacitating arthritis, and uveitis. Methotrexate is a second-line therapy for JIA.

All parents are entitled to a birthing environment in which breastfeeding is promoted and supported. The Baby Friendly Hospital Initiative endorsed by WHO and UNICEF was founded to encourage institutions to offer optimal levels of care for lactating mothers. Which instruction is not included in the Ten Steps to Successful Breastfeeding for Hospitals? a. Give newborns no food or drink other than breast milk. b. Have a written breastfeeding policy that is communicated to all staff. c. Help mothers initiate breastfeeding within one half hour of birth. d. Give artificial teats or pacifiers as necessary.

ANS: D No artificial teats or pacifiers (also called dummies or soothers) should be given to breastfeeding infants. No other food or drink should be given to the newborn unless medically indicated. The breastfeeding policy should be routinely communicated to all health care staff. All staff should be trained in the skills necessary to maintain this policy. Breastfeeding should be initiated within one half hour of birth, and all mothers need to be shown how to maintain lactation even if they are separated from their babies. PTS: 1 DIF: Cognitive Level: Application REF: 641 OBJ: Nursing Process: Implementation MSC: Client Needs: Safe and Effective Care Environment

24.The nurse is caring for an adolescent who has just started dialysis. The child seems always angry, hostile, or depressed. The nurse should recognize that this is most likely related to: a. Neurologic manifestations that occur with dialysis. b. Physiologic manifestations of renal disease. c. Adolescents having few coping mechanisms. d. Adolescents often resenting the control and enforced dependence imposed by dialysis.

ANS: D Older children and adolescents need control. The necessity of dialysis forces the adolescent into a dependent relationship, which results in these behaviors. Neurologic manifestations that occur with dialysis and physiologic manifestations of renal disease are a function of the age of the child, not neurologic or physiologic manifestations of the dialysis. Adolescents do have coping mechanisms, but they need to have some control over their disease management.

10. What is the characteristic of dishonest behavior in children ages 8 to 10 years? a. Cheating during games is now more common. b. Lying results from the inability to distinguish between fact and fantasy. c. They may steal because their sense of property rights is limited. d. They may lie to meet expectations set by others that they have been unable to attain.

ANS: D Older school-age children may lie to meet expectations set by others to which they have been unable to measure up. Cheating usually becomes less frequent as the child matures. In this age-group, children are able to distinguish between fact and fantasy. Young children may lack a sense of property rights; older children may steal to supplement an inadequate allowance, or it may be an indication of serious problems.

A woman experienced a miscarriage at 10 weeks of gestation and had a dilation and curettage (D&C). She states that she is just fine and wants to go home as soon as possible. While you are assessing her responses to her loss, she tells you that she had purchased some baby things and had picked out a name. On the basis of your assessment of her responses, what nursing intervention would you use first? a. Ready her for discharge. b. Notify pastoral care to offer her a blessing. c. Ask her whether she would like to see what was obtained from her D&C. d. Ask her what name she had picked out for her baby.

ANS: D One way of actualizing the loss is to allow parents to name the infant. The nurse should follow this clients cues and inquire about naming the infant. The client is looking for an opportunity to express her feelings of loss. The nurse should take this opportunity to offer support by allowing the woman to talk about her feelings. Furthermore, one way of actualizing the loss is to allow parents to name the infant. The nurse should follow this clients cues and inquire about naming the infant. Although it may be therapeutic to offer religious support, the nurse should take this opportunity to offer support by allowing the woman to talk about her feelings. Furthermore, one way of actualizing the loss is to allow parents to name the infant. Asking the woman whether she would like to see what was obtained from her D&C is completely inappropriate. PTS: 1 DIF: Cognitive Level: Application REF: 532 OBJ: Nursing Process: Implementation MSC: Client Needs: Psychosocial Integrity

38. Parents have understood teaching about prevention of childhood otitis media if they make which statement? a. We will only prop the bottle during the daytime feedings. b. Breastfeeding will be discontinued after 4 months of age. c. We will place the child flat right after feedings. d. We will be sure to keep immunizations up to date.

ANS: D Parents have understood the teaching about preventing childhood otitis media if they respond they will keep childhood immunizations up to date. The child should be maintained upright during feedings and after. Otitis media can be prevented by exclusively breastfeeding until at least 6 months of age. Propping bottles is discouraged to avoid pooling of milk while the child is in the supine position.

7. What is the role of the peer group in the life of school-age children? a. Gives them an opportunity to learn dominance and hostility. b. Allows them to remain dependent on their parents for a longer time. c. Decreases their need to learn appropriate sex roles. d. Provides them with security as they gain independence from their parents.

ANS: D Peer-group identification is an important factor in gaining independence from parents. Through peer relationships, children learn ways to deal with dominance and hostility. They also learn how to relate to people in positions of leadership and authority and explore ideas and the physical environment. Peer-group identification helps in gaining independence rather than remaining dependent. A child's concept of appropriate sex roles is influenced by relationships with peers.

What bacterial infection is definitely decreasing because of effective drug treatment? a. Escherichia coli infection b. Tuberculosis c. Candidiasis d. Group B streptococcal infection

ANS: D Penicillin has significantly decreased the incidence of group B streptococcal infection. E. coli may be increasing, perhaps because of the increasing use of ampicillin (resulting in a more virulent E. coli resistant to the drug). Tuberculosis is increasing in the United States and Canada. Candidiasis is a fairly benign fungal infection. PTS: 1 DIF: Cognitive Level: Comprehension REF: 674 OBJ: Nursing Process: Evaluation MSC: Client Needs: Health Promotion and Maintenance

The nurse is caring for a child with severe head trauma after a car accident. Which is an ominous sign that often precedes death? a. Papilledema b. Delirium c. Doll's head maneuver d. Periodic and irregular breathing

ANS: D Periodic or irregular breathing is an ominous sign of brainstem (especially medullary) dysfunction that often precedes complete apnea. Papilledema is edema and inflammation of optic nerve. It is commonly a sign of increased ICP. Delirium is a state of mental confusion and excitement marked by disorientation for time and place. The doll's head maneuver is a test for brainstem or oculomotor nerve dysfunction.

25.An advantage of peritoneal dialysis is that: a. Treatments are done in hospitals. b. Protein loss is less extensive. c. Dietary limitations are not necessary. d. Parents and older children can perform treatments.

ANS: D Peritoneal dialysis is the preferred form of dialysis for parents, infants, and children who wish to remain independent. Parents and older children can perform the treatments themselves. Treatments can be done at home. Protein loss is not significantly different. The dietary limitations are necessary, but they are not as stringent as those for hemodialysis.

13. Which clinical manifestation would most suggest acute appendicitis? a. Rebound tenderness b. Bright red or dark red rectal bleeding c. Abdominal pain that is relieved by eating d. Abdominal pain that is most intense at McBurney point

ANS: D Pain is the cardinal feature. It is initially generalized and usually periumbilical. The pain localizes to the right lower quadrant at McBurney point. Rebound tenderness is not a reliable sign and is extremely painful to the child. Abdominal pain that is relieved by eating and bright or dark red rectal bleeding are not signs of acute appendicitis.

HIV may be perinatally transmitted: a. Only in the third trimester from the maternal circulation. b. From the use of unsterile instruments. c. Only through the ingestion of amniotic fluid. d. Through the ingestion of breast milk from an infected mother.

ANS: D Postnatal transmission of HIV through breastfeeding may occur. Transmission of HIV from the mother to the infant may occur transplacentally at various gestational ages. This is highly unlikely because most health care facilities must meet sterility standards for all instrumentation. Transmission of HIV may occur during birth from blood or secretions. PTS: 1 DIF: Cognitive Level: Comprehension REF: 676 OBJ: Nursing Process: Planning MSC: Client Needs: Physiologic Integrity: Reduction of Risk Potential

Human immunodeficiency virus (HIV) may be perinatally transmitted: a. Only in the third trimester from the maternal circulation. b. By a needlestick injury at birth from unsterile instruments. c. Only through the ingestion of amniotic fluid. d. Through the ingestion of breast milk from an infected mother.

ANS: D Postnatal transmission of HIV through breastfeeding may occur. Transmission of HIV from the mother to the infant may occur transplacentally at various gestational ages. Transmission close to or at the time of birth is thought to account for 50% to 80% of cases. PTS: 1 DIF: Cognitive Level: Comprehension REF: 676 OBJ: Nursing Process: Planning MSC: Client Needs: Physiologic Integrity

Which condition is a transient, self-limiting mood disorder that affects new mothers after childbirth? a. Postpartum depression b. Postpartum psychosis c. Postpartum bipolar disorder d. Postpartum blues

ANS: D Postpartum blues or baby blues is a transient self-limiting disease that is believed to be related to hormonal fluctuations after childbirth. Postpartum depression is not the normal worries (blues) that many new mothers experience. Many caregivers believe that postpartum depression is underdiagnosed and underreported. Postpartum psychosis is a rare condition that usually surfaces within 3 weeks of delivery. Hospitalization of the woman is usually necessary for treatment of this disorder. Bipolar disorder is one of the two categories of postpartum psychosis, characterized by both manic and depressive episodes. PTS: 1 DIF: Cognitive Level: Knowledge REF: 546 OBJ: Nursing Process: Assessment MSC: Client Needs: Psychosocial Integrity

One of the first symptoms of puerperal infection to assess for in the postpartum woman is: a. Fatigue continuing for longer than 1 week. b. Pain with voiding. c. Profuse vaginal bleeding with ambulation. d. Temperature of 38 C (100.4 F) or higher on 2 successive days starting 24 hours after birth

ANS: D Postpartum or puerperal infection is any clinical infection of the genital canal that occurs within 28 days after miscarriage, induced abortion, or childbirth. The definition used in the United States continues to be the presence of a fever of 38 C (100.4 F) or higher on 2 successive days of the first 10 postpartum days, starting 24 hours after birth. Fatigue would be a late finding associated with infection. Pain with voiding may indicate a urinary tract infection, but it is not typically one of the earlier symptoms of infection. Profuse lochia may be associated with endometritis, but it is not the first symptom associated with infection. PTS: 1 DIF: Cognitive Level: Comprehension REF: 540 OBJ: Nursing Process: Assessment MSC: Client Needs: Health Promotion and Maintenance

When assessing the preterm infant the nurse understands that compared with the term infant, the preterm infant has: a. Few blood vessels visible through the skin. b. More subcutaneous fat. c. Well-developed flexor muscles. d. Greater surface area in proportion to weight.

ANS: D Preterm infants have greater surface area in proportion to their weight. More subcutaneous fat and welldeveloped muscles are indications of a more mature infant. PTS: 1 DIF: Cognitive Level: Analysis REF: 685 OBJ: Nursing Process: Assessment MSC: Client Needs: Physiologic Integrity

4. When caring for an infant with an upper respiratory tract infection and elevated temperature, an appropriate nursing intervention is to: a. Give tepid water baths to reduce fever. b. Encourage food intake to maintain caloric needs. c. Have child wear heavy clothing to prevent chilling. d. Give small amounts of favorite fluids frequently to prevent dehydration.

ANS: D Preventing dehydration by small frequent feedings is an important intervention in the febrile child. Tepid water baths may induce shivering, which raises temperature. Food should not be forced; it may result in the child vomiting. The febrile child should be dressed in light, loose clothing.

38. A child has nausea after chemotherapy despite anti-emetics. However, the child complains that "my tummy is growling." Which other action should the nurse take to promote comfort for this child? A. Avoid hard, difficult-to-chew foods. B. Encourage a high fluid intake with meals. C. Offer the child hard candy to suck on. D. Provide bland items, such as plain mashed potatoes.

ANS: D Several actions can help the child with nausea: offering plain, bland foods; avoiding spicy, heavy, or fatty foods; decreasing the odor associated with foods if that bothers the child; and having the child take food separately from liquids. Liquid together with food can make the child feel full, inducing nausea. The other options are good choices for other nutritional problems.

With shortened hospital stays, new mothers are often discharged before they begin to experience symptoms of the baby blues or postpartum depression. As part of the discharge teaching, the nurse can prepare the mother for this adjustment to her new role by instructing her regarding self-care activities to help prevent postpartum depression. The most accurate statement as related to these activities is to: a. Stay home and avoid outside activities to ensure adequate rest. b. Be certain that you are the only caregiver for your baby, to facilitate infant attachment. c. Keep feelings of sadness and adjustment to your new role to yourself. d. Realize that this is a common occurrence that affects many women.

ANS: D Should the new mother experience symptoms of the baby blues, it is important that she be aware that this is nothing to be ashamed of. Up to 80% of women experience this type of mild depression after the birth of their infant. Although it is important for the mother to obtain enough rest, she should not distance herself from family and friends. Her spouse or partner can communicate the best visiting times so the new mother can obtain adequate rest. It is also important that she not isolate herself at home during this time of role adjustment. Even if breastfeeding, other family members can participate in the infants care. If depression occurs, the symptoms can often interfere with mothering functions, and this support will be essential. The new mother should share her feelings with someone else. It is also important that she not overcommit herself or think she has to be superwoman. A telephone call to the hospital warm line may provide reassurance with lactation issues and other infant care questions. Should symptoms continue, a referral to a professional therapist may be necessary. PTS: 1 DIF: Cognitive Level: Application REF: 546 OBJ: Nursing Process: Implementation MSC: Client Needs: Psychosocial Integrity

2. A child has had cold symptoms for more than 2 weeks, a headache, nasal congestion with purulent nasal drainage, facial tenderness, and a cough that increases during sleep. The nurse recognizes that these symptoms are characteristic of which respiratory condition? a. Allergic rhinitis c. Asthma b. Bronchitis d. Sinusitis

ANS: D Sinusitis is characterized by signs and symptoms of a cold that do not improve after 14 days, a low-grade fever, nasal congestion and purulent nasal discharge, headache, tenderness, a feeling of fullness over the affected sinuses, halitosis, and a cough that increases when the child is lying down. The classic symptoms of allergic rhinitis are watery rhinorrhea; itchy nose, eyes, ears, and palate; and sneezing. Symptoms occur as long as the child is exposed to the allergen. Bronchitis is characterized by a gradual onset of rhinitis and a cough that is initially nonproductive but may change to a loose cough. The manifestations of asthma may vary, with wheezing being a classic sign. The symptoms presented in the question do not suggest asthma.

A postpartum patient asks, Will these stretch marks go away? The nurses best response is: a. They will continue to fade and should be gone by your 6-week checkup. b. No, never. c. Yes, eventually. d. They will fade to silvery lines but won`t disappear completely.

ANS: D Stretch marks never disappear altogether; however, they gradually fade to silvery lines. PTS: 1 DIF: Cognitive Level: Knowledge REF: 489 OBJ: Nursing Process: Implementation MSC: Client Needs: Health Promotion and Maintenance

The nurse caring for a family during a loss may notice that survival guilt sometimes is felt at the death of an infant by the childs: a. Siblings. b. Mother. c. Father. d. Grandparents.

ANS: D Survival guilt sometimes is felt by grandparents because they believe that the death upsets the natural order of things. They are angry that they are alive and their grandchild is not. PTS: 1 DIF: Cognitive Level: Comprehension REF: 553 OBJ: Nursing Process: Evaluation MSC: Client Needs: Psychosocial Integrity

36. Why do infants and young children quickly have respiratory distress in acute and chronic alterations of the respiratory system? a. They have a widened, shorter airway. b. There is a defect in their sucking ability. c. The gag reflex increases mucus production. d. Mucus and edema obstruct small airways.

ANS: D The airway in infants and young children is narrower, not wider, and respiratory distress can occur quickly because mucus and edema can cause obstruction to their small airways. Sucking is not necessarily related to problems with the airway. The gag reflex is necessary to prevent aspiration. It does not produce mucus.

When responding to the question Will I produce enough milk for my baby as she grows and needs more milk at each feeding? the nurse should explain that: a. The breast milk will gradually become richer to supply additional calories. b. As the infant requires more milk, feedings can be supplemented with cows milk. c. Early addition of baby food will meet the infants needs. d. The mothers milk supply will increase as the infant demands more at each feeding.

ANS: D The amount of milk produced depends on the amount of stimulation of the breast. Increased demand with more frequent and longer breastfeeding sessions results in more milk available for the infant. Mature breast milk will stay the same. The amounts will increase as the infant feeds for longer times. Supplementation will decrease the amount of stimulation of the breast and decrease the milk production. Solids should not be added until about 4 to 6 months, when the infants immune system is more mature. This will decrease the chance of allergy formations. PTS: 1 DIF: Cognitive Level: Knowledge REF: 640 OBJ: Nursing Process: Implementation MSC: Client Needs: Physiologic Integrity

While evaluating the reflexes of a newborn, the nurse notes that with a loud noise the newborn symmetrically abducts and extends his arms, his fingers fan out and form a C with the thumb and forefinger, and he has a slight tremor. The nurse would document this finding as a positive: a. Tonic neck reflex. b. Glabellar (Myerson) reflex. c. Babinski reflex. d. Moro reflex.

ANS: D The characteristics displayed by the infant are associated with a positive Moro reflex. The tonic neck reflex occurs when the infant extends the leg on the side to which the infants head simultaneously turns. The glabellar reflex is elicited by tapping on the infants head while the eyes are open. A characteristic response is blinking for the first few taps. The Babinski reflex occurs when the sole of the foot is stroked upward along the lateral aspect of the sole and then across the ball of the foot. A positive response occurs when all the toes hyperextend, with dorsiflexion of the big toe. PTS: 1 DIF: Cognitive Level: Comprehension REF: 576 OBJ: Nursing Process: Assessment MSC: Client Needs: Health Promotion and Maintenance

Nurses should be able to teach breastfeeding mothers the signs that the infant has latched on correctly. Which statement indicates a poor latch? a. She feels a firm tugging sensation on her nipples but not pinching or pain. b. The baby sucks with cheeks rounded, not dimpled. c. The babys jaw glides smoothly with sucking. d. She hears a clicking or smacking sound.

ANS: D The clicking or smacking sound may indicate that the baby is having difficulty keeping the tongue out over the lower gum ridge. The mother should hope to hear the sound of swallowing. The tugging sensation without pinching is a good sign. Rounded cheeks are a positive indicator of a good latch. A smoothly gliding jaw is a good sign. PTS: 1 DIF: Cognitive Level: Comprehension REF: 641 OBJ: Nursing Process: Planning MSC: Client Needs: Health Promotion and Maintenance

19. Type 1 diabetes mellitus is suspected in an adolescent. Which clinical manifestation may be present? a. Moist skin b. Weight gain c. Fluid overload d. Poor wound healing

ANS: D Poor wound healing is often an early sign of type 1 diabetes mellitus. Dry skin, weight loss, and dehydration are clinical manifestations of type 1 diabetes mellitus.

A nursing student is helping the nursery nurses with morning vital signs. A baby born 10 hours ago by cesarean section is found to have moist lung sounds. What is the best interpretation of these data? a. The nurse should notify the pediatrician stat for this emergency situation. b. The neonate must have aspirated surfactant. c. If this baby was born vaginally, it could indicate a pneumothorax. d. The lungs of a baby delivered by cesarean section may sound moist for 24 hours after birth.

ANS: D The condition will resolve itself within a few hours. For this common condition of newborns, surfactant acts to keep the expanded alveoli partially open between respirations. In vaginal births, absorption of remaining lung fluid is accelerated by the process of labor and delivery. Remaining lung fluid will move into interstitial spaces and be absorbed by the circulatory and lymphatic systems. This is a particularly common condition for infants delivered by cesarean section. Surfactant is produced by the lungs, so aspiration is not a concern. PTS: 1 DIF: Cognitive Level: Comprehension REF: 557 OBJ: Nursing Process: Assessment MSC: Client Needs: Health Promotion and Maintenance

27. The parents of a child with cancer ask the nurse why the child is losing weight even though he is eating what he normally does. Which response by the nurse is the most appropriate? A. "Cancer consumes body tissues, causing weight loss." B. "He may be going through a growth spurt right now." C. "How do you know he is eating like he normally does?" D. "When you are sick, you need more nutrition than usual."

ANS: D The demands of illness lead to increased nutritional needs. A child with cancer needs increased nutrition. Cancer does not consume body tissues. The child may be going through a growth spurt, but this is not always the case and is not the best answer. Asking the parents how they know the child's eating habits have changed may put them on the defensive.

A 4-year-old child is newly diagnosed with Legg-Calvé-Perthes disease. Nursing considerations should include which action? a. Encouraging normal activity for as long as is possible b. Explaining the cause of the disease to the child and family c. Preparing the child and family for long-term, permanent disabilities d. Teaching the family the care and management of the corrective appliance

ANS: D The family needs to learn the purpose, function, application, and care of the corrective device and the importance of compliance to achieve the desired outcome. The initial therapy is rest and non-weight bearing, which helps reduce inflammation and restore motion. Legg-Calvé-Perthes is a disease with an unknown etiology. A disturbance of circulation to the femoral capital epiphysis produces an ischemic aseptic necrosis of the femoral head. The disease is self-limiting, but the ultimate outcome of therapy depends on early and efficient therapy and the child's age at onset.

With regard to postpartum ovarian function, nurses should be aware that: a. Almost 75% of women who do not breastfeed resume menstruating within a month after birth. b. Ovulation occurs slightly earlier for breastfeeding women. c. Because of menstruation/ovulation schedules, contraception considerations can be postponed until after the puerperium. d. The first menstrual flow after childbirth usually is heavier than normal.

ANS: D The first flow is heavier, but within three or four cycles, it is back to normal. Ovulation can occur within the first month, but for 70% of nonlactating women, it returns within 12 weeks after birth. Breastfeeding women take longer to resume ovulation. Because many women ovulate before their first postpartum menstrual period, contraceptive options need to be discussed early in the puerperium. PTS: 1 DIF: Cognitive Level: Comprehension REF: 486 OBJ: Nursing Process: Planning MSC: Client Needs: Physiologic Integrity

Which statement describing the first phase of the transition period is inaccurate? a. It lasts no longer than 30 minutes. b. It is marked by spontaneous tremors, crying, and head movements. c. It includes the passage of meconium. d. It may involve the infants suddenly sleeping briefly.

ANS: D The first phase is an active phase in which the baby is alert. Decreased activity and sleep mark the second phase. The first phase is the shortest, lasting less than 30 minutes. Such exploratory behaviors include spontaneous startle reactions. In the first phase the newborn also produces saliva. PTS: 1 DIF: Cognitive Level: Comprehension REF: 556 OBJ: Nursing Process: Assessment MSC: Client Needs: Health Promotion and Maintenance

15. The mother of a toddler yells to the nurse, Help! He is choking to death on his food. The nurse determines that lifesaving measures are necessary based on: a. Gagging. c. Pulse over 100 beats/min. b. Coughing. d. Inability to speak.

ANS: D The inability to speak indicates a foreign-body airway obstruction of the larynx. Abdominal thrusts are needed for treatment of the choking child. Gagging indicates irritation at the back of the throat, not obstruction. Coughing does not indicate a complete airway obstruction. Tachycardia may be present for many reasons.

15.The most common cause of acute renal failure in children is: a. Pyelonephritis. b. Tubular destruction. c. Urinary tract obstruction. d. Severe dehydration.

ANS: D The most common cause of acute renal failure in children is dehydration or other causes of poor perfusion that may respond to restoration of fluid volume. Pyelonephritis and tubular destruction are not common causes of acute renal failure in children. Obstructive uropathy may cause acute renal failure, but it is not the most common cause.

The nurse assessing a newborn knows that the most critical physiologic change required of the newborn is: a. Closure of fetal shunts in the circulatory system. b. Full function of the immune defense system at birth. c. Maintenance of a stable temperature. d. Initiation and maintenance of respirations.

ANS: D The most critical adjustment of a newborn at birth is the establishment of respirations. The cardiovascular system changes markedly after birth as a result of fetal respiration, which reduces pulmonary vascular resistance to the pulmonary blood flow and initiates a chain of cardiac changes that support the cardiovascular system. The infant relies on passive immunity received from the mother for the first 3 months of life. After the establishment of respirations, heat regulation is critical to newborn survival. PTS: 1 DIF: Cognitive Level: Comprehension REF: 557 OBJ: Nursing Process: Assessment MSC: Client Needs: Physiologic Integrity

The nurse is assessing a child who was just admitted to the hospital for observation after a head injury. Which is the most essential part of the nursing assessment to detect early signs of a worsening condition? a. Posturing b. Vital signs c. Focal neurologic signs d. Level of consciousness

ANS: D The most important nursing observation is assessment of the child's level of consciousness. Alterations in consciousness appear earlier in the progression of an injury than do alterations of vital signs or focal neurologic signs. Neurologic posturing is indicative of neurologic damage. Vital signs and focal neurologic signs are later signs of progression when compared with level-of-consciousness changes.

A newly delivered mother who intends to breastfeed tells her nurse, I am so relieved that this pregnancy is over so I can start smoking again. The nurse encourages the client to refrain from smoking. However, this new mother insists that she will resume smoking. The nurse will need to adapt her health teaching to ensure that the client is aware that: a. Smoking has little or no effect on milk production. b. There is no relation between smoking and the time of feedings. c. The effects of secondhand smoke on infants are less significant than for adults. d. The mother should always smoke in another room.

ANS: D The new mother should be encouraged not to smoke. If she continues to smoke, she should be encouraged to always smoke in another room removed from the baby. Smoking may impair milk production. When the products of tobacco are broken down, they cross over into the breast milk. Tobacco also results in a reduction of the immunologic properties of breast milk. Research supports that mothers should not smoke within 2 hours before a feeding. The effects of secondhand smoke on infants include sudden infant death syndrome. PTS: 1 DIF: Cognitive Level: Application REF: 653 OBJ: Nursing Process: Planning MSC: Client Needs: Physiologic Integrity

After they are born, a crying infant may be soothed by being held in a position in which the newborn can hear the mothers heartbeat. This phenomenon is known as: a. Entrainment. b. Reciprocity. c. Synchrony. d. Biorhythmicity.

ANS: D The newborn is in rhythm with the mother. The infant develops a personal biorhythm with the parents help over time. Entrainment is the movement of newborns in time to the structure of adult speech. Reciprocity is body movement or behavior that gives cues to the persons desires. These take several weeks to develop with a new baby. Synchrony is the fit between the infants behavioral cues and the parents responses. PTS: 1 DIF: Cognitive Level: Knowledge REF: 515 OBJ: Nursing Process: Implementation MSC: Client Needs: Health Promotion and Maintenance

What infant response to cool environmental conditions is either not effective or not available to them? a. Constriction of peripheral blood vessels b. Metabolism of brown fat c. Increased respiratory rates d. Unflexing from the normal position

ANS: D The newborns flexed position guards against heat loss because it reduces the amount of body surface exposed to the environment. The newborns body is able to constrict the peripheral blood vessels to reduce heat loss. Burning brown fat generates heat. The respiratory rate may rise to stimulate muscular activity, which generates heat. PTS: 1 DIF: Cognitive Level: Comprehension REF: 561 OBJ: Nursing Process: Planning MSC: Client Needs: Physiologic Integrity

A new father states, I know nothing about babies, but he seems to be interested in learning. This is an ideal opportunity for the nurse to: a. Continue to observe his interaction with the newborn. b. Tell him when he does something wrong. c. Show no concern, as he will learn on his own. d. Include him in teaching sessions.

ANS: D The nurse must be sensitive to the fathers needs and include him whenever possible. As fathers take on their new role, the nurse should praise every attempt, even if his early care is awkward. It is important to note the bonding process of the mother and the father; however, that does not satisfy the expressed needs of the father. The new father should be encouraged in caring for his baby by pointing out the things that he does right. Criticizing him will discourage him. PTS: 1 DIF: Cognitive Level: Application REF: 521 OBJ: Nursing Process: Planning MSC: Client Needs: Health Promotion and Maintenance

On examining a woman who gave birth 5 hours ago, the nurse finds that the woman has completely saturated a perineal pad within 15 minutes. The nurses first action is to: a. Begin an intravenous (IV) infusion of Ringers lactate solution. b. Assess the womans vital signs. c. Call the womans primary health care provider. d. Massage the womans fundus.

ANS: D The nurse should assess the uterus for atony. Uterine tone must be established to prevent excessive blood loss. The nurse may begin an IV infusion to restore circulatory volume, but this would not be the first action. Blood pressure is not a reliable indicator of impending shock from impending hemorrhage; assessing vital signs should not be the nurses first action. The physician would be notified after the nurse completes the assessment of the woman. PTS: 1 DIF: Cognitive Level: Application REF: 496 OBJ: Nursing Process: Implementation MSC: Client Needs: Health Promotion and Maintenance

A nurse is observing a family. The mother is holding the baby she delivered less than 24 hours ago. Her husband is watching his wife and asking questions about newborn care. The 4-year-old brother is punching his mother on the back. The nurse should: a. Report the incident to the social services department. b. Advise the parents that the toddler needs to be reprimanded. c. Report to oncoming staff that the mother is probably not a good disciplinarian. d. Realize that this is a normal family adjusting to family change.

ANS: D The observed behaviors are normal variations of families adjusting to change. There is no need to report this one incident. Giving advice at this point would make the parents feel inadequate. PTS: 1 DIF: Cognitive Level: Analysis REF: 518 OBJ: Nursing Process: Assessment MSC: Client Needs: Psychosocial Integrity

The nurse is caring for a 4-year-old child immobilized by a fractured hip. Which complication should the nurse monitor related to the child's immobilization status? a. Metabolic rate increases b. Increased joint mobility leading to contractures c. Bone calcium increases, releasing excess calcium into the body (hypercalcemia) d. Venous stasis leading to thrombi or emboli formation

ANS: D The physiologic effects of immobilization, as a result of decreased muscle contraction, include venous stasis. This can lead to pulmonary emboli or thrombi. The metabolic rate decreases with immobilization. Loss of joint mobility leads to contractures. Bone demineralization with osteoporosis and hypercalcemia occur with immobilization.

The interval between the birth of the newborn and the return of the reproductive organs to their normal nonpregnant state is called the: a. Involutionary period because of what happens to the uterus. b. Lochia period because of the nature of the vaginal discharge. c. Mini-tri period because it lasts only 3 to 6 weeks. d. Puerperium, or fourth trimester of pregnancy.

ANS: D The puerperium, also called the fourth trimester or the postpartum period of pregnancy, lasts about 3 to 6 weeks. Involution marks the end of the puerperium, or the fourth trimester of pregnancy. Lochia refers to the various vaginal discharges during the puerperium, or fourth trimester of pregnancy. PTS: 1 DIF: Cognitive Level: Knowledge REF: 486 OBJ: Nursing Process: Assessment MSC: Client Needs: Health Promotion and Maintenance

After giving birth to a stillborn infant, the woman turns to the nurse and says, I just finished painting the babys room. Do you think that caused my baby to die? The nurses best response to this woman is: a. Thats an old wives tale; lots of women are around paint during pregnancy, and this doesnt happen to them. b. Thats not likely. Paint is associated with elevated pediatric lead levels. c. Silence. d. I can understand your need to find an answer to what caused this. What else are you thinking about?

ANS: D The statement, I can understand your need to find an answer to what caused this. What else are you thinking about? is very appropriate for the nurse. It demonstrates caring and compassion and allows the mother to vent her thoughts and feelings, which is therapeutic in the process of grieving. The nurse should resist the temptation to give advice or to use clichs in offering support to the bereaved. In addition, trying to give bereaved parents answers when no clear answers exist or trying to squelch their guilt feeling does not help the process of grief. Silence probably would increase the mothers feelings of guilt. One of the most important goals of the nurse is to validate the experience and feelings of the parents by encouraging them to tell their stories and listening with care. The nurse should encourage the mother to express her ideas. PTS: 1 DIF: Cognitive Level: Application REF: 550 OBJ: Nursing Process: Implementation MSC: Client Needs: Psychosocial Integrity

One reason the brain is vulnerable to nutritional deficiencies and trauma in early infancy is the: a. Incompletely developed neuromuscular system. b. Primitive reflex system. c. Presence of various sleep-wake states. d. Cerebellum growth spurt.

ANS: D The vulnerability of the brain likely is to the result of the cerebellum growth spurt. The neuromuscular system is almost completely developed at birth. The reflex system is not relevant. The various sleep-wake states are not relevant. PTS: 1 DIF: Cognitive Level: Analysis REF: 572 OBJ: Nursing Process: Diagnosis MSC: Client Needs: Health Promotion and Maintenance

33. A preschool child is being admitted to the hospital with dehydration and a urinary tract infection (UTI). Which urinalysis result should the nurse expect with these conditions? a. WBC <1; specific gravity 1.008 b. WBC <2; specific gravity 1.025 c. WBC >2; specific gravity 1.016 d. WBC >2; specific gravity 1.030

ANS: D The white blood cell count (WBC) in a routine urinalysis should be <1 or 2. Over that amount indicates a urinary tract inflammatory process. The urinalysis specific gravity for children with normal fluid intake is 1.016 to 1.022. When the specific gravity is high, dehydration is indicated. A low specific gravity is seen with excessive fluid intake, distal tubular dysfunction, or insufficient antidiuretic hormone secretion.

Childbirth may result in injuries to the vagina and uterus. Pelvic floor exercises also known as Kegel exercises will help to strengthen the perineal muscles and encourage healing. The nurse knows that the client understands the correct process for completing these conditioning exercises when she reports: a. I contract my thighs, buttocks, and abdomen. b. I do 10 of these exercises every day. c. I stand while practicing this new exercise routine. d. I pretend that I am trying to stop the flow of urine midstream.

ANS: D The woman can pretend that she is attempting to stop the passing of gas or the flow of urine midstream. This will replicate the sensation of the muscles drawing upward and inward. Each contraction should be as intense as possible without contracting the abdomen, buttocks, or thighs. Guidelines suggest that these exercises should be done 24 to 100 times per day. Positive results are shown with a minimum of 24 to 45 repetitions per day. The best position to learn Kegel exercises is to lie supine with knees bent. A secondary position is on the hands and knees. PTS: 1 DIF: Cognitive Level: Analysis REF: 485 OBJ: Nursing Process: Evaluation MSC: Client Needs: Health Promotion and Maintenance

A woman gave birth vaginally to a 9-pound, 12-ounce girl yesterday. Her primary health care provider has written orders for perineal ice packs, use of a sitz bath tid, and a stool softener. What information is most closely correlated with these orders? a. The woman is a gravida 2, para b. The woman had a vacuum-assisted birth. c. The woman received epidural anesthesia. d. The woman has an episiotomy.

ANS: D These orders are typical interventions for a woman who has had an episiotomy, lacerations, and hemorrhoids. A multiparous classification is not an indication for these orders. A vacuum-assisted birth may be used in conjunction with an episiotomy, which would indicate these interventions. Use of epidural anesthesia has no correlation with these orders. PTS: 1 DIF: Cognitive Level: Comprehension REF: 497 OBJ: Nursing Process: Planning MSC: Client Needs: Health Promotion and Maintenance

13. The father of 12 year old tells the nurse that he is concerned about his child getting "fat." The child's body mass index for age is at the 60th percentile. What is the most appropriate nursing action to address the father's concern? a. Reassure the father that his child is not "fat." b. Reassure the father that the weight is just a growing child. c. Suggest a low-calorie, low-fat diet, and provide food suggestions. d. Explain that this is typical of the growth pattern of children at this age.

ANS: D This is a characteristic pattern of growth in preadolescent boys, in which the growth in height has slowed in preparation for the pubertal growth spurt but weight is still gained. This should be reviewed with both the father and child, and a plan should be developed to maintain physical exercise and a balanced diet. Saying that the child is not "fat" is false reassurance. The child's weight is high for his/her height. The child needs to maintain his/her physical activity. The father is concerned; an explanation is required. A nutritional diet with physical activity should be sufficient to maintain his balance.

The nurse uses the palms of the hands when handling a wet cast for which reason? a. To assess dryness of the cast b. To facilitate easy turning c. To keep the patient's limb balanced d. To avoid indenting the cast

ANS: D Wet casts should be handled by the palms of the hands, not the fingers, to avoid creating pressure points. Assessing dryness, facilitating easy turning, and keeping the patient's limb balanced are not reasons for using the palms of the hand rather than the fingers when handling a wet cast.

On day 3 of life, a newborn continues to require 100% oxygen by nasal cannula. The parents ask whether they can hold their infant during his next gavage feeding. Given that this newborn is physiologically stable, what response would the nurse give? a. Parents are not allowed to hold infants who depend on oxygen. b. You may hold only your babys hand during the feeding. c. Feedings cause more physiologic stress, so the baby must be closely monitored. Therefore, I dont think you should hold the baby. d. You may hold your baby during the feeding.

ANS: D You may hold your baby during the feeding is an accurate statement. Parental interaction via holding is encouraged during gavage feedings so that the infant will associate the feeding with positive interactions. Nasal cannula oxygen therapy allows for easier feedings and psychosocial interactions. The parent can swaddle the infant during gavage feedings to help the infant associate the feeding with positive interactions. Some parents like to do kangaroo care while gavage feeding their infant. Swaddling or kangaroo care during feedings provides positive interactions for the infant. PTS: 1 DIF: Cognitive Level: Application REF: 700 OBJ: Nursing Process: Planning MSC: Client Needs: Health Promotion and Maintenance

10. The nurse is preparing a child for possible alopecia from chemotherapy. Which intervention should be included? a. Explain to child that hair usually regrows in 1 year. b. Advise child to expose head to sunlight to minimize alopecia. c. Explain to child that wearing a hat or scarf is preferable to wearing a wig. d. Explain to child that when hair regrows, it may have a slightly different color or texture.

ANS: D Alopecia is a side effect of certain chemotherapeutic agents. When the hair regrows, it may be a different color or texture. The hair usually grows back within 3 to 6 months after cessation of treatment. The head should be protected from sunlight to avoid sunburn. Children should choose the head covering they prefer.

11. Which is a common clinical manifestation of Hodgkin disease? a. Petechiae b. Bone and joint pain c. Painful, enlarged lymph nodes d. Enlarged, firm, nontender lymph nodes

ANS: D Asymptomatic, enlarged, cervical or supraclavicular lymphadenopathy is the most common presentation of Hodgkin disease. Petechiae are usually associated with leukemia. Bone and joint pain are not likely in Hodgkin disease. The enlarged nodes are rarely painful.

31. Which clinical manifestation may occur in the child who is prescribed methimazole for the treatment of hyperthyroidism (Graves' disease)? a. Seizures b. Enlargement of all lymph glands c. Pancreatitis or cholecystitis d. Sore throat or fever

ANS: D Children being treated with propylthiouracil or methimazole must be carefully monitored for side effects of the drug. Because sore throat and fever accompany the grave complication of leukopenia, these children should be seen by a health care practitioner if such symptoms occur. Neither seizures, cholecystitis nor pancreatitis are associated with the administration of methimazole. Enlargement of the salivary and cervical lymph glands may occur.

29. What should the nurse stress in a teaching plan for the mother of an 11-year-old diagnosed with ulcerative colitis? a. Preventing the spread of illness to others b. Nutritional guidance and preventing constipation c. Teaching daily use of enemas d. Coping with stress and avoiding triggers

ANS: D Coping with the stress of chronic illness and the clinical manifestations associated with ulcerative colitis (diarrhea, pain) are important teaching foci. Avoidance of triggers can help minimize the impact of the disease and its effect on the child. Ulcerative colitis is not infectious. Although nutritional guidance is a priority teaching focus, diarrhea is a problem with ulcerative colitis, not constipation. Daily enemas are not part of the therapeutic plan of care.

7. The nurse is caring for a child with suspected diabetes insipidus. Which clinical manifestation would be observable? a. Oliguria b. Glycosuria c. Nausea and vomiting d. Polydipsia

ANS: D Excessive urination accompanied by insatiable thirst is the primary clinical manifestation of diabetes. These symptoms may be so severe that the child does little other than drink and urinate. Oliguria is decreased urine production and is not associated with diabetes insipidus. Glycosuria is associated with diabetes mellitus. Nausea and vomiting are associated with inappropriate antidiuretic hormone secretion.

17. Which symptom is considered a cardinal sign of diabetes mellitus? a. Nausea b. Seizures c. Impaired vision d. Frequent urination

ANS: D Hallmarks of diabetes mellitus are glycosuria, polyuria, and polydipsia. Nausea and seizures are not clinical manifestations of diabetes mellitus. Impaired vision is a long-term complication of the disease.

30. Careful hand washing before and after contact can prevent the spread of which condition in day care and school settings? a. Irritable bowel syndrome b. Ulcerative colitis c. Hepatic cirrhosis d. Hepatitis A

ANS: D Hepatitis A is spread person to person, by the fecal-oral route, and through contaminated food or water. Good hand washing is critical in preventing its spread. The virus can survive on contaminated objects for weeks. Irritable bowel syndrome is the result of increased intestinal motility and is not contagious. Ulcerative colitis and cirrhosis are not infectious.

2. Which type of dehydration results from water loss in excess of electrolyte loss? a. Isotonic dehydration b. Isosmotic dehydration c. Hypotonic dehydration d. Hypertonic dehydration

ANS: D Hypertonic dehydration results from water loss in excess of electrolyte loss. This is the most dangerous type of dehydration. It is caused by feeding children fluids with high amounts of solute. Isotonic dehydration occurs in conditions in which electrolyte and water deficits are present in balanced proportion. Isosmotic dehydration is another term for isotonic dehydration. Hypotonic dehydration occurs when the electrolyte deficit exceeds the water deficit, leaving the serum hypotonic.

33. An infant diagnosed with pyloric stenosis experiences excessive vomiting that can result in which condition? a. Hyperchloremia b. Hypernatremia c. Metabolic acidosis d. Metabolic alkalosis

ANS: D Infants with excessive vomiting are prone to metabolic alkalosis from the loss of hydrogen ions. Chloride ions and sodium are lost with vomiting. Metabolic alkalosis, not acidosis, is likely.

6. A stool specimen from a child with diarrhea shows the presence of neutrophils and red blood cells. This is most suggestive of which condition? a. Protein intolerance b. Parasitic infection c. Fat malabsorption d. Bacterial gastroenteritis

ANS: D Neutrophils and red blood cells in stool indicate bacterial gastroenteritis. Protein intolerance is suspected in the presence of eosinophils. Parasitic infection is indicated by eosinophils. Fat malabsorption is indicated by foul-smelling, greasy, bulky stools

22. A child eats some sugar cubes after experiencing symptoms of hypoglycemia. This rapid-releasing sugar should be followed by: a. saturated and unsaturated fat. b. fruit juice. c. several glasses of water. d. complex carbohydrate and protein.

ANS: D Symptoms of hypoglycemia are treated with a rapid-releasing sugar source followed by a complex carbohydrate and protein. Saturated and unsaturated fat, fruit juice, and several glasses of water do not provide the child with complex carbohydrate and protein necessary to stabilize the blood sugar.

25. The nurse is discussing various sites used for insulin injections with a child and her family. Which site usually has the fastest rate of absorption? a. Arm b. Leg c. Buttock d. Abdomen

ANS: D The abdomen has the fastest rate of absorption but the shortest duration. The arm has a fast rate of absorption but short duration. The leg has a slow rate of absorption but a long duration. The buttock has the slowest rate of absorption and the longest duration.

3. Myelosuppression, associated with chemotherapeutic agents or some malignancies such as leukemia can cause bleeding tendencies because of what resulting outcome? a. Decrease in leukocytes b. Increase in lymphocytes c. Vitamin C deficiency d. Decrease in blood platelets

ANS: D The decrease in blood platelets secondary to the myelosuppression of chemotherapy can cause an increase in bleeding. The child and family should be alerted to avoid risk of injury. Decrease in leukocytes, increase in lymphocytes, and vitamin C deficiency would not affect bleeding tendencies.

5. Which immunization should be considered carefully before being given to a child receiving chemotherapy for cancer? a. Tetanus vaccine b. Inactivated poliovirus vaccine c. Diphtheria, pertussis, tetanus (DPT) d. Measles, rubella, mumps

ANS: D The vaccine used for measles, mumps, and rubella is a live virus and can result in an overwhelming infection. Tetanus vaccine, inactivated poliovirus vaccine, and diphtheria, pertussis, tetanus (DPT) are not live virus vaccines.

38.A child with secondary enuresis who complains of dysuria or urgency should be evaluated for what condition (Select all that apply)? a. Hypocalciuria b. Nephrotic syndrome c. Glomerulonephritis d. Urinary tract infection (UTI) e. Diabetes mellitus

ANS: D, E Complaints of dysuria or urgency from a child with secondary enuresis suggest the possibility of a UTI. If accompanied by excessive thirst and weight loss, these symptoms may indicate the onset of diabetes mellitus. An excessive loss of calcium in the urine (hypercalciuria) can be associated with complaints of painful urination, urgency, frequency, and wetting. Nephrotic syndrome is not usually associated with complaints of dysuria or urgency. Glomerulonephritis is not a likely cause of dysuria or urgency.

Which are clinical manifestations of increased intracranial pressure (ICP) in infants? (Select all that apply.) a. Low-pitched cry b. Sunken fontanel c. Diplopia and blurred vision d. Irritability e. Distended scalp veins f. Increased blood pressure

ANS: D, E Diplopia and blurred vision, irritability, and distended scalp veins are signs of increased ICP in infants. Diplopia and blurred vision is indicative of ICP in children. A high-pitched cry and a tense or bulging fontanel are characteristics of increased ICP. Increased blood pressure, common in adults, is rarely seen in children.

2. When a nurse is counseling a woman for primary dysmenorrhea, which nonpharmacologic intervention might be recommended? A. Increasing the intake of red meat and simple carbohydrates B. Reducing the intake of diuretic foods such as peaches and asparagus C. Temporarily substituting physical activity for a sedentary lifestyle D. Using a heating pad on the abdomen to relieve cramping

ANS: DHeat minimizes cramping by increasing vasodilation and muscle relaxation and minimizing uterine ischemia. Dietary changes such as eating less red meat may be recommended for women experiencing dysmenorrhea. Increasing the intake of diuretics, including natural diuretics such as asparagus, cranberry juice, peaches, parsley, and watermelon, may help ease the symptoms associated with dysmenorrhea. Exercise has been found to help relieve menstrual discomfort through increased vasodilation and subsequent decreased ischemia.

C (secondary circular reactions Infants are usually in the secondary circular reaction stage from age 4 months to 8 months. This stage is characterized by a continuation of the primary circular reaction for the response that results. For example, shaking of a rattle is performed to hear the noise of the rattle, not just for shaking. The use of reflexes is primarily during the first month of life. The primary circular reaction stage marks the replacement of reflexes with voluntary acts. The infant is in this stage from age 1 month to 4 months. The fourth sensorimotor stage is coordination of secondary schemata. This is a transitional stage in which increasing motor skills enable greater exploration of the environment.)

According to Piaget, the 6-month-old infant would be in what stage of the sensorimotor phase? a. Use of reflexes b. Primary circular reactions c. Secondary circular reactions d. Coordination of secondary schemata

C (During the tertiary circular reactions stage, children have only a rudimentary sense of the classification of objects. The appearance of an object denotes its function for these children. The slot of an outlet is for putting things into. Her cognitive development is appropriate for her age and represents typical behavior for a toddler. Only some awareness exists of a causal relation between events.)

Although a 14-month-old girl received a shock from an electrical outlet recently, her parents find her about to place a paper clip in another outlet. The best interpretation of this behavior is that: a. Her cognitive development is delayed. b. This is typical behavior because toddlers are not very developed. c. This is typical behavior because of inability to transfer knowledge to new situations. d. This is not typical behavior because toddlers should know better than to repeat an act that caused pain.

C (10 months)

At what age would the nurse advise parents to expect their infant to be able to say "mama" and "dada" with meaning? A. 4 months B. 6 months C. 10 months D. 14 months

C (8 months Sitting erect without support is a developmental milestone usually achieved by 8 months. At age 4 months, an infant can sit with support. At age 6 months, the infant will maintain a sitting position if propped. By 10 months, the infant can maneuver from a prone to a sitting position.)

At which age can most infants sit steadily unsupported? a. 4 months b. 6 months c. 8 months d. 10 months

Assessment reveals that a woman's cervix is approximately 1 cm in length. The nurse would document this as: a) 100% effaced b) 50% effaced c) 0% effaced d) 75% effaced

B (50% effaced Rationale: A cervix 1 cm in length is described as 50% effaced. A cervix that measures approximately 2 cm in length is described as 0% effaced. A cervix 1/2 cm in length would be described as 75% effaced. A cervix 0 cm in length would be described as 100% effaced.)

Nurses can help their clients by keeping them informed about the distinctive stages of labor. What description of the phases of the first stage of labor is accurate? a) Latent: milk, regular contractions; no dilation; blood show; duration of 2 to 4 hours b) Active: moderate, regular contractions; 4 to 7cm dilation; duration of 3 to 6 hours c) Lull: no contractions; dilation stable; duration of 20 to 60 minutes d) Transition: very strong but irregular contractions; 8 to 10cm dilation; duration of 1 to 2 hours

B (Active: moderate, regular contractions; 4 to 7cm dilation; duration of 3 to 6 hours.)

What action does the nurse take to relieve choking in a pregnant patient who is in the third trimester? A. Administering anesthesia B. Administering chest thrusts C. Placing a towel under the hips D. Positioning the patient onto one side

B (Choking is often relieved in patients by administering abdominal thrusts. However if the patient is in the third trimester of pregnancy, chest thrusts are administered to prevent injury to the uterus. Administering anesthesia or positioning the patient onto one side will not help dislodge the object and relieve choking. The nurse needs to place a towel under the hips to displace the uterus while administering cardiopulmonary resuscitation (CPR).)

The nurse observes that a pregnant patient with gestational hypertension who is on magnesium sulfate therapy is prescribed nifedipine (Adalat). What action does the nurse take? A. Evaluates the patient's renal function test B. Obtains a prescription for a change of drug C. Reduces the nifedipine (Adalat) dose by 50% D. Administers both medications simultaneously

B (Concurrent use of nifedipine (Adalat) and magnesium sulfate can result in skeletal muscle blockade in the patient. Therefore the nurse needs to report immediately to the primary health care provider (PHP) and obtain a prescription for a change of drug. The nurse assesses the patient's renal function to determine the risk for toxicity after administering any drug. However, it is not a priority in this case. Reducing the nifedipine (Adalat) dose is not likely to prevent the drug interaction in the patient. The nurse does not administer both drugs simultaneously because it may be harmful for the patient.)

The nurse is measuring a contraction from the beginning of the increment to the end of the decrement for the same contraction, documenting this as which of the following? a) Peak b) Duration c) Frequency d) Intensity

B (Duration Rationale: Duration refers to how long a contraction lasts and is measured from the beginning of the increment to the end of the decrement for the same contraction. Intensity refers to the strength of the contraction determined by manual palpation or measured by an internal intrauterine catheter. Frequency refers to how often contractions occur and is measured from the increment of one contraction to the increment of the next contraction. The peak or acme of a contraction is the highest intensity of a contraction.)

Which statement is the best rationale for assessing maternal vital signs between contractions? a. During a contraction, assessing fetal heart rates is the priority. b. Maternal circulating blood volume increases temporarily during contractions. c. Maternal blood flow to the heart is reduced during contractions. d. Vital signs taken during contractions are not accurate.

B (During uterine contractions, blood flow to the placenta temporarily stops, causing a relative increase in the mother's blood volume, which in turn temporarily increases blood pressure and slows pulse. It is important to monitor fetal response to contractions; however, this question is concerned with the maternal vital signs. Maternal blood flow is increased during a contraction. Vital signs are altered by contractions but are considered accurate for that period of time.)

The slight overlapping of cranial bones or shaping of the fetal head during labor is called: a. Lightening. b. Molding. c. Ferguson reflex. d. Valsalva maneuver.

B (Fetal head formation is called molding. Molding also permits adaptation to various diameters of the maternal pelvis. Lightening is the mother's sensation of decreased abdominal distention, which usually occurs the week before labor. The Ferguson reflex is the contraction urge of the uterus after stimulation of the cervix. The Valsalva maneuver describes conscious pushing during the second stage of labor.)

Which is an important nursing intervention when a patient has an incomplete miscarriage with heavy bleeding? A. Initiate expectant management at once. B. Prepare the patient for dilation and curettage. C. Administer the prescribed oxytocin (Pitocin). D. Obtain a prescription for ergonovine (Methergine).

B (In the case of an incomplete miscarriage, sometimes there is heavy bleeding and excessive cramping and some part of fetal tissue remains in the uterus. Therefore the nurse needs to prepare the patient for dilation and curettage for the removal of the fetal tissue. Expectant management is initiated if the pregnancy continues after a threatened miscarriage. Oxytocin (Pitocin) is administered to prevent hemorrhage after evacuation of the uterus. Ergonovine (Methergine) is administered to contract the uterus.)

The nurse assesses a client in labor and finds that the fetal long axis is longitudinal to the maternal long axis. The nurse documents this finding as which of the following? a) Attitude b) Lie c) Position d) Presentation

B (Lie Rationale: The nurse is assessing fetal lie, the relationship of the fetal long axis to the maternal long axis. When the fetal long axis is longitudinal to the maternal long axis, the lie is said to longitudinal. Presentation is the portion of the fetus that overlies the maternal pelvic inlet. Attitude is the relationship of the different fetal parts to one another. Position is the relationship of the fetal denominator to the different sides of the maternal pelvis.)

A nurse is performing a vaginal examination of a woman in the early stages of labor. The woman has been at 2 cm dilated for the past 2 hours, but effacement has progressed steadily. Which of the following should the nurse do to best encourage the client regarding her progress? a) Don't mention anything to the client yet; wait for further dilatation to occur. b) Say, "you are still 2 cm dilated, but the cervix is thinning out nicely." c) Say, "you haven't dilated any further, but hang in there; it will happen eventually." d) Say, "there has been no further dilatation; effacement is progressing."

B (Say, "you are still 2 cm dilated, but the cervix is thinning out nicely." Rationale: Women are anxious to have frequent reports during labor, to reassure them everything is progressing well. If giving a progress report, remember most women are aware of the word dilatation but not effacement. Just saying, "no further dilatation", therefore, is a depressing report. "You're not dilated a lot more, but a lot of thinning is happening and that's just as important" is the same report given in a positive manner.)

The nurse has received report regarding her patient in labor. The woman's last vaginal examination was recorded as 3 cm, 30%, and ?2-2. The nurse's interpretation of this assessment is that: a. The cervix is effaced 3 cm, it is dilated 30%, and the presenting part is 2 cm above the ischial spines. b. The cervix is 3 cm dilated, it is effaced 30%, and the presenting part is 2 cm above the ischial spines. c. The cervix is effaced 3 cm, it is dilated 30%, and the presenting part is 2 cm below the ischial spines. d. The cervix is dilated 3 cm, it is effaced 30%, and the presenting part is 2 cm below the ischial spines.

B (The correct description of the vaginal examination for this woman in labor is the cervix is 3 cm dilated, it is effaced 30%, and the presenting part is 2 cm above the ischial spines. The sterile vaginal examination is recorded as centimeters of cervical dilation, percentage of cervical dilation, and the relationship of the presenting part to the ischial spines (either above or below).)

Concerning the third stage of labor, nurses should be aware that: a) The placenta eventually detaches itself from a flaccid uterus b) The duration of the third stage may be as short as 3 to 5 minutes c) It is important that the dark, roughened maternal surface of the placenta appear before the shiny fetal surface d) The major risk for women during the third stage is a rapid heart rate

B (The duration of the third stage may be as short as 3 to 5 minutes)

A pregnant patient is at risk for cardiac arrest as a result of profound hypovolemia after a trauma. Which action does the nurse take? The nurse: A. Assesses airway, breathing, and pulse rate. B. Administers warmed crystalloid solutions. C. Administers calcium gluconate intravenously. D. Obtains a prescription for magnesium sulfate.

B (The nurse administers warmed crystalloid solutions for massive fluid resuscitation in the patient who has profound hypovolemia after a trauma. The nurse needs to assess the airway, breathing, and pulse in a patient after a convulsion so that prompt actions can be taken to stabilize the patient. The nurse administers calcium gluconate as an antidote to a patient who has magnesium toxicity. The nurse may administer magnesium sulfate for the treatment of eclamptic seizures in a patient with preeclampsia.)

As relates to fetal positioning during labor, nurses should be aware that: a. Position is a measure of the degree of descent of the presenting part of the fetus through the birth canal. b. Birth is imminent when the presenting part is at +4 to +5 cm below the spine. c. The largest transverse diameter of the presenting part is the suboccipitobregmatic diameter. d. Engagement is the term used to describe the beginning of labor.

B (The station of the presenting part should be noted at the beginning of labor so that the rate of descent can be determined. Position is the relation of the presenting part of the fetus to the four quadrants of the mother's pelvis; station is the measure of degree of descent. The largest diameter usually is the biparietal diameter. The suboccipitobregmatic diameter is the smallest, although one of the most critical. Engagement often occurs in the weeks just before labor in nulliparas and before or during labor in multiparas.)

Which statement is the most appropriate advice to give parents of a 16-year-old girl who is rebellious? a. "You need to be stricter so that your teen stops trying to test the limits." b. "You need to collaborate with your daughter and set limits that are perceived as being reasonable." c. "Increasing your teen's involvement with her peers will improve her self-esteem." d. "Allow your teenager to choose the type of discipline that is used in your home."

B Allowing teenagers to choose between realistic options and offering consistent and structured discipline typically enhances cooperation and decreases rebelliousness. Structure helps adolescents to feel more secure and assists them in the decision-making process. Setting stricter limits typically does not decrease rebelliousness or decrease testing of parental limits. Increasing peer involvement does not typically increase self-esteem.

A 24-year-old primipara, 10 weeks pregnant, who has been experiencing vomiting every morning for the past few weeks, asks the nurse at her check-up how long this "morning sickness" will continue. Which statement by the nurse is most accurate? A. "It will end by the 15th week of pregnancy." B. "It usually subsides by the 20th week of pregnancy." C. "It's a very common but not serious problem." D. "In some women, it can last throughout the pregnancy and become serious."

B (This discomfort of pregnancy usually subsides by the 20th week of pregnancy. An absolute definite end of vomiting during pregnancy can never be stated. Test-Taking Tip: Identifying content and what is being asked about that content is critical to your choosing the correct response. Be alert for words in the stem of the item that are the same or similar in nature to those in one or two of the options. Example: If the item relates to and identifies stroke rehabilitation as its focus and only one of the options contains the word stroke in relation to rehabilitation, you are safe in identifying this choice as the correct response.)

A woman with severe preeclampsia is receiving a magnesium sulfate infusion. The nurse becomes concerned after assessment when the woman exhibits: A. a sleepy, sedated affect. B. a respiratory rate of 10 breaths/min. C. deep tendon reflexes of 2+. D. absent ankle clonus.

B (a respiratory rate of 10 breaths/min. Because magnesium sulfate is a central nervous system (CNS) depressant, the client will most likely become sedated when the infusion is initiated. A respiratory rate of 10 breaths/min indicates that the client is experiencing respiratory depression (bradypnea) from magnesium toxicity. Deep tendon reflexes of 2+ are a normal finding. Absent ankle clonus is a normal finding.)

The most prevalent clinical manifestation of abruptio placentae (as opposed to placenta previa) is: A. bleeding. B. intense abdominal pain. C. uterine activity. D. cramping.

B (intense abdominal pain. Bleeding may be present in varying degrees for both placental conditions. Pain is absent with placenta previa and may be agonizing with abruptio placentae. Uterine activity may be present with both placental conditions. Cramping is a form of uterine activity that may be present in both placental conditions.)

A woman with severe preeclampsia is being treated with an IV infusion of magnesium sulfate. This treatment is considered successful if: A. blood pressure is reduced to prepregnant baseline. B. seizures do not occur. C. deep tendon reflexes become hypotonic. D. diuresis reduces fluid retention.

B (seizures do not occur. A temporary decrease in blood pressure can occur; however, this is not the purpose of administering this medication. Magnesium sulfate is a central nervous system (CNS) depressant given primarily to prevent seizures. Hypotonia is a sign of an excessive serum level of magnesium. It is critical that calcium gluconate be on hand to counteract the depressant effects of magnesium toxicity. Diuresis is not an expected outcome of magnesium sulfate administration.)

A new client and her partner arrive on the labor, delivery, recovery, and postpartum unit for the birth of their first child. You apply the electronic fetal monitor (EFM) to the woman. Her partner asks you to explain what is printing on the graph, referring to the EFM strip. He wants to know what the baby's heart rate should be. Your best response is: a. "Don't worry about that machine; that's my job." b. "The top line graphs the baby's heart rate. Generally the heart rate is between 110 and 160. The heart rate will fluctuate in response to what is happening during labor." c. "The top line graphs the baby's heart rate, and the bottom line lets me know how strong the contractions are." d. "Your doctor will explain all of that later."

B "The top line graphs the baby's heart rate. Generally the heart rate is between 110 and 160. The heart rate will fluctuate in response to what is happening during labor" educates the partner about fetal monitoring and provides support and information to alleviate his fears. "Don't worry about that machine; that's my job" discredits the partner's feelings and does not provide the teaching he is requesting. "The top line graphs the baby's heart rate, and the bottom line lets me know how strong the contractions are" provides inaccurate information and does not address the partner's concerns about the fetal heart rate. The EFM graphs the frequency and duration of the contractions, not the intensity. Nurses should take every opportunity to provide client and family teaching, especially when information is requested.

The parent of a 4-year-old son tells the nurse that the child believes "monsters and the boogeyman" are in his bedroom at night. The nurse's best suggestion for coping with this problem is to: a. Insist that the child sleep with his parents until the fearful phase passes. b. Suggest involving the child to find a practical solution such as a night-light. c. Help the child understand that these fears are illogical. d. Tell the child frequently that monsters and the boogeyman do not exist.

B A night-light shows a child that imaginary creatures do not lurk in the darkness. Letting the child sleep with parents or telling the child that these creatures do not exist will not get rid of the fears. A 4-year-old is in the preconceptual age and cannot understand logical thought.

A school nurse is conducting a class with adolescents on suicide. Which true statement about suicide should the nurse include in the teaching session? a. A sense of hopelessness and despair are a normal part of adolescence. b. Gay and lesbian adolescents are at a particularly high risk for suicide. c. Problem-solving skills are of limited value to the suicidal adolescent. d. Previous suicide attempts are not an indication of risk for completed suicides.

B A significant number of teenage suicides occur among homosexual youths. Gay and lesbian adolescents who live in families or communities that do not accept homosexuality are likely to suffer low self-esteem, self-loathing, depression, and hopelessness as a result of a lack of acceptance from their family or community. At-risk teenagers include those who are depressed, have poor problem-solving skills, or use drugs and alcohol. History of previous suicide attempt is a serious indicator for possible suicide completion in the future.

Acyclovir (Zovirax) is given to children with chickenpox to: a. Minimize scarring. c. Prevent aplastic anemia. b. Decrease the number of lesions. d. Prevent spread of the disease.

B Acyclovir decreases the number of lesions, shortens duration of fever, and decreases itching, lethargy, and anorexia; however, it does not prevent scarring. Preventing aplastic anemia is not a function of acyclovir. Only quarantine of the infected child can prevent the spread of disease.

Which statement is most descriptive of central nervous system stimulants? a. They produce strong physical dependence. b. They can result in strong psychologic dependence. c. Withdrawal symptoms are life threatening. d. Acute intoxication can lead to coma.

B Central nervous system stimulants such as amphetamines and cocaine produce a strong psychologic dependence. This class of drugs does not produce strong physical dependence and can be withdrawn without much danger. Acute intoxication leads to violent, aggressive behavior or psychotic episodes characterized by paranoia, uncontrollable agitation, and restlessness.

In terms of language and cognitive development, a 4-year-old child would be expected to: a. Think in abstract terms. b. Follow simple commands. c. Understand conservation of matter. d. Comprehend another person's perspective.

B Children ages 3 to 4 years can give and follow simple commands. Children cannot think abstractly at age 4 years. Conservation of matter is a developmental task of the school-age child. A 4-year-old child cannot comprehend another's perspective.

What correctly matches the type of deceleration with its likely cause? a. Early deceleration—umbilical cord compression b. Late deceleration—uteroplacental inefficiency c. Variable deceleration—head compression d. Prolonged deceleration—cause unknown

B Late deceleration is caused by uteroplacental inefficiency. Early deceleration is caused by head compression. Variable deceleration is caused by umbilical cord compression. Prolonged deceleration has a variety of either benign or critical causes.

A teen asks a nurse, "What is physical dependence in substance abuse?" Which is the correct response by the nurse? a. Problem that occurs in conjunction with addiction b. Involuntary physiologic response to drug c. Culturally defined use of drugs for purposes other than accepted medical purposes d. Voluntary behavior based on psychosocial needs

B Physical dependence is an involuntary response to the pharmacologic characteristics of drugs such as opioids or alcohol. A person can be physically dependent on a narcotic/drug without being addicted; for example, patients who use opioids to control pain need increasing doses to achieve the same effect. Dependence is a physiologic response; it is not culturally determined or subject to voluntary control.

A 4-year-old boy is hospitalized with a serious bacterial infection. He tells the nurse that he is sick because he was "bad." The nurse's best interpretation of this comment is that it is: a. A sign of stress. b. Common at this age. c. Suggestive of maladaptation. d. Suggestive of excessive discipline at home.

B Preschoolers cannot understand the cause and effect of illness. Their egocentrism makes them think that they are directly responsible for events, making them feel guilt for things outside of their control. Children of this age show stress by regressing developmentally or acting out. Maladaptation is unlikely. This comment does not imply excessive discipline at home.

What three measures should the nurse implement to provide intrauterine resuscitation? Select the response that best indicates the priority of actions that should be taken. a. Call the provider, reposition the mother, and perform a vaginal examination. b. Reposition the mother, increase intravenous (IV) fluid, and provide oxygen via face mask. c. Administer oxygen to the mother, increase IV fluid, and notify the care provider. d. Perform a vaginal examination, reposition the mother, and provide oxygen via face mask.

B Repositioning the mother, increasing intravenous (IV) fluid, and providing oxygen via face mask are correct nursing actions for intrauterine resuscitation. The nurse should initiate intrauterine resuscitation in an ABC manner, similar to basic life support. The first priority is to open the maternal and fetal vascular systems by repositioning the mother for improved perfusion. The second priority is to increase blood volume by increasing the IV fluid. The third priority is to optimize oxygenation of the circulatory volume by providing oxygen via face mask. If these interventions do not resolve the fetal heart rate issue quickly, the primary provider should be notified immediately.

The nurse is completing a health history with a 16-year-old male. He informs the nurse that he has started using smokeless tobacco after he plays baseball. Which information regarding smokeless tobacco would be most correct for the nurse to provide to this teen? a. Not addicting. b. Proven to be carcinogenic. c. Easy to stop using. d. A safe alternative to cigarette smoking.

B Smokeless tobacco is a popular substitute for cigarettes and poses serious health hazards to children and adolescents. Smokeless tobacco is associated with cancer of the mouth and jaw. Smokeless tobacco is just as addictive as cigarettes. Although teens believe that it is easy to stop using smokeless tobacco, this is not the case. A popular belief is that smokeless tobacco is a safe alternative to cigarettes; this has been proven incorrect. Half of all teens who use smokeless tobacco agree that it poses significant health risks.

A 17-year-old tells the nurse that he is not having sex because it would make his parents very angry. This response indicates that the adolescent has a developmental lag in which area? a. Cognitive development c. Psychosocial development b. Moral development d. Psychosexual development

B The appropriate moral development for a 17-year-old would include evidence that the teenager has internalized a value system and does not depend on parents to determine right and wrong behaviors. Adolescents who remain concrete thinkers may never advance beyond conformity to please others and avoid punishment. Cognitive development is related to moral development, but it is not the pivotal point in determining right and wrong behaviors. Identity formation is the psychosocial development task. Energy is focused within the adolescent, who exhibits behavior that is self-absorbed and egocentric. Although a task during adolescence is the development of a sexual identity, the teenager's dependence on the parents' sanctioning of right or wrong behavior is more appropriately related to moral development.

During labor a fetus with an average heart rate of 135 beats/min over a 10-minute period would be considered to have: a. Bradycardia. c. Tachycardia. b. A normal baseline heart rate. d. Hypoxia.

B The baseline heart rate is measured over 10 minutes; a normal range is 110 to 160 beats/min. Bradycardia is a fetal heart rate (FHR) below 110 beats/min for 10 minutes or longer. Tachycardia is an FHR over 160 beats/min for 10 minutes or longer. Hypoxia is an inadequate supply of oxygen; no indication of this condition exists with a baseline heart rate in the normal range.

Why is continuous electronic fetal monitoring usually used when oxytocin is administered? a. The mother may become hypotensive. b. Uteroplacental exchange may be compromised. c. Maternal fluid volume deficit may occur. d. Fetal chemoreceptors are stimulated.

B The uterus may contract more firmly, and the resting tone may be increased with oxytocin use. This response reduces entrance of freshly oxygenated maternal blood into the intervillous spaces, thus depleting fetal oxygen reserves. Hypotension is not a common side effect of oxytocin. All laboring women are at risk for fluid volume deficit; oxytocin administration does not increase the risk. Oxytocin affects the uterine muscles.

Parents tell the nurse that they found their 3-year-old daughter and a male cousin of the same age inspecting each other closely as they used the bathroom. Which is the most appropriate recommendation the nurse should make? a. Punish children so this behavior stops. b. Neither condone nor condemn the curiosity. c. Allow children unrestricted permission to satisfy this curiosity. d. Get counseling for this unusual and dangerous behavior.

B Three-year-olds become aware of anatomic differences and are concerned about how the other "works." Such exploration should not be condoned or condemned. Children should not be punished for this normal exploration. Encouraging the children to ask questions of the parents and redirecting their activity are more appropriate than giving permission. Exploration is age-appropriate and not dangerous behavior.

Which statement, made by a 4-year-old child's father, is true about the care of the preschooler's teeth? a. "Because the 'baby teeth' are not permanent, they are not important to the child." b. "My son can be encouraged to brush his teeth after I have thoroughly cleaned his teeth." c. "My son's 'permanent teeth' will begin to come in at 4 to 5 years of age." d. "Fluoride supplements can be discontinued when my son's 'permanent teeth' erupt."

B Toddlers and preschoolers lack the manual dexterity to remove plaque adequately, so parents must assume this responsibility. Deciduous teeth are important because they maintain spacing and play an important role in the growth and development of the jaws and face and in speech development. Secondary teeth erupt at about 6 years of age. If the family does not live in an area where fluoride is included in the water supply, fluoride supplements should be continued.

The nurse providing care for the laboring woman realizes that variable fetal heart rate (FHR) decelerations are caused by: a. Altered fetal cerebral blood flow. c. Uteroplacental insufficiency. b. Umbilical cord compression. d. Fetal hypoxemia.

B Variable decelerations can occur any time during the uterine contracting phase and are caused by compression of the umbilical cord. Altered fetal cerebral blood flow would result in early decelerations in the FHR. Uteroplacental insufficiency would result in late decelerations in the FHR. Fetal hypoxemia would result in tachycardia initially and then bradycardia if hypoxia continues.

When is a child with chickenpox considered to be no longer contagious? a. When fever is absent c. 24 hours after lesions erupt b. When lesions are crusted d. 8 days after onset of illness

B When the lesions are crusted, the chickenpox is no longer contagious. This may be a week after onset of disease. The child is still contagious once the fever has subsided and after the lesions erupt, and may or may not be contagious any time after 6 days as long as all lesions are crusted over.

The emergency department nurse is assessing a pregnant trauma victim who just arrived at the hospital. What are the nurse's MOST appropriate actions? (Select all that apply.) A. Place the patient in a supine position. B. Assess for point of maximal impulse at fourth intercostal space. C. Collect urine for urinalysis and culture. D. Frequent vital sign monitoring. E. Assist with ambulation to decrease risk of thrombosis.

B, C, D (Passive regurgitation may occur if patient is supine, leading to high risk for aspiration. Placental perfusion is decreased when the patient is in a supine position as well. The heart is displaced upward and to the left in pregnant patients. During pregnancy, there is dilation of the ureters and urethra, and the bladder is displaced forward placing the pregnant trauma patient at higher risk for urinary stasis, infection, and bladder trauma. The trauma patient can suffer blood loss and other complications, necessitating frequent monitoring of vital signs. While the pregnant patient is at risk for thrombus formation, the patient must be cleared by the health care provider before ambulating. The pregnant trauma patient is at higher risk for pelvic fracture, and therefore this condition must be ruled out first as well.)

A pregnant patient with severe preeclampsia who is being transported to a tertiary care center needs to be administered magnesium sulfate injection for seizure activity. What actions does the nurse take when administering the drug? Select all that apply. A. A 10-g dose is administered in the buttock. B. A local anesthetic is added to the solution. C. The Z-track technique is used to inject the drug. D. The injection site is massaged after the injection. E. The subcutaneous route is used to inject the drug.

B, C, D (The nurse adds a local anesthetic to the solution to reduce pain that is caused by the injection. The Z-track technique is used to inject the drug so that the drug is injected in the intramuscular (IM) tissue safely. The nurse gently massages the site after administering the injection to reduce pain. The nurse administers two separate injections of 5 g in each buttock. Magnesium sulfate injections are administered in the IM layer and not the subcutaneous layer.)

What are the possible causes of miscarriage during early pregnancy? Select all that apply. A. Premature dilation of cervix B. Chromosomal abnormalities C. Endocrine imbalance D. Hypothyroidism E. Antiphospholipid antibodies

B, C, D, E (Chromosomal abnormalities account for 50% of all early pregnancy losses. Endocrine imbalance is caused by luteal phase defects, hypothyroidism, and diabetes mellitus in pregnant patients and results in miscarriage. Antiphospholipid antibodies also increase the chances of miscarriage in pregnant patients. Premature dilation of the cervix may cause a second-trimester loss and is usually seen in patients between 12 and 20 weeks' gestation.)

What does the nurse include in the plan of care of a pregnant patient with mild preeclampsia? Select all that apply. A. Ensure prolonged bed rest. B. Provide diversionary activities. C. Encourage the intake of more fluids. D. Restrict sodium and zinc in the diet. E. Refer to Internet-based support group

B, C, E (Activity is restricted in patients with preeclampsia, so it is necessary to provide diversionary activities to such patients to prevent boredom. The nurse encourages the patient to increase fluid intake to enhance renal perfusion and bowel function. The nurse can suggest Internet-based support groups to reduce boredom and stress in the patient. Patients need to restrict activity, but complete bed rest is not advised because it may cause cardiovascular deconditioning, muscle atrophy, and psychological stress. The patient needs to include adequate zinc and sodium in the diet for proper fetal development.)

A pregnant woman presents to the emergency department complaining of persistent nausea and vomiting. She is diagnosed with hyperemesis gravidarum. The nurse should include which information when teaching about diet for hyperemesis? (Select all that apply.) A. Eat three larger meals a day. B. Eat a high-protein snack at bedtime. C. Ice cream may stay down better than other foods. D. Avoid ginger tea or sweet drinks. E. Eat what sounds good to you even if your meals are not well-balanced.

B, C, E (The diet for hyperemesis includes: • Avoid an empty stomach. Eat frequently, at least every 2 to 3 hours. Separate liquids from solids and alternate every 2 to 3 hours. • Eat a high-protein snack at bedtime. • Eat dry, bland, low-fat, and high-protein foods. Cold foods may be better tolerated than those served at a warm temperature. • In general eat what sounds good to you rather than trying to balance your meals. • Follow the salty and sweet approach; even so-called junk foods are okay. • Eat protein after sweets. • Dairy products may stay down more easily than other foods. • If you vomit even when your stomach is empty, try sucking on a Popsicle. • Try ginger tea. Peel and finely dice a knuckle-sized piece of ginger and place it in a mug of boiling water. Steep for 5 to 8 minutes and add brown sugar to taste. • Try warm ginger ale (with sugar, not artificial sweetener) or water with a slice of lemon. • Drink liquids from a cup with a lid.)

A pregnant woman presents to the emergency department complaining of persistent nausea and vomiting. She is diagnosed with hyperemesis gravidarum. The nurse should include which information when teaching about diet for hyperemesis? Select all that apply. A. Eat three larger meals a day. B. Eat a high-protein snack at bedtime. C. Ice cream may stay down better than other foods. D. Avoid ginger tea or sweet drinks. E. Eat what sounds good to you even if your meals are not well-balanced.

B, C, E (The diet for hyperemesis includes: (1) Avoid an empty stomach. Eat frequently, at least every 2 to 3 hours. Separate liquids from solids and alternate every 2 to 3 hours. (2) Eat a high-protein snack at bedtime. (3) Eat dry, bland, low-fat, and high-protein foods. Cold foods may be better tolerated than those served at a warm temperature. (4) In general eat what sounds good to you rather than trying to balance your meals. (5) Follow the salty and sweet approach; even so-called junk foods are okay. (6) Eat protein after sweets. (7) Dairy products may stay down more easily than other foods. (8) If you vomit even when your stomach is empty, try sucking on a Popsicle. (9) Try ginger tea. Peel and finely dice a knuckle-sized piece of ginger and place it in a mug of boiling water. Steep for 5 to 8 minutes and add brown sugar to taste. (10) Try warm ginger ale (with sugar, not artificial sweetener) or water with a slice of lemon. (11) Drink liquids from a cup with a lid.)

The nurse finds that a child spends several hours each day playing video games and lives in a home environment with limited access to safe playgrounds and parks. What health risks does the nurse expect based on these findings? Select all that apply. A Tooth decay B Hypertension C Diabetes D Growth delays E Hypercholesterolemia

B, C, E A child's home environment with limited access to safe outdoor play areas and abundant access to television and video games is a major contributing factor for childhood obesity. Childhood obesity increases the risk for hypertension, diabetes, and hypercholesterolemia, among other conditions. Tooth decay is associated with poor dental care, not lack of exercise contributing to childhood obesity. Growth delays are associated with malnutrition, not lack of exercise contributing to childhood obesity.

A tiered system of categorizing FHR has been recommended by regulatory agencies. Nurses, midwives, and physicians who care for women in labor must have a working knowledge of fetal monitoring standards and understand the significance of each category. These categories include (Select all that apply): a. Reassuring. b. Category I. c. Category II. d. Nonreassuring. e. Category III.

B, C, E The three tiered system of FHR tracings include Category I, II, and III. Category I is a normal tracing requiring no action. Category II FHR tracings are indeterminate. This category includes tracings that do not meet Category I or III criteria. Category III tracings are abnormal and require immediate intervention.

A pregnant patient with chronic hypertension is at risk for placental abruption. Which symptoms of abruption does the nurse instruct the patient to be alert for? Select all that apply. A. Weight loss B. Abdominal pain C. Vaginal bleeding D. Shortness of breath E. Uterine tenderness

B, C, E (The nurse instructs the pregnant patient to be alert for abdominal pain, vaginal bleeding, and uterine tenderness as these indicates placental abruption. Weight loss indicates fluid and electrolyte loss and not placental abruption. Shortness of breath indicates inadequate oxygen, which is usually seen in a patient who is having cardiac arrest.)

In terms of language and cognitive development, a 4-year-old child would be expected to have which traits (select all that apply)? a. Think in abstract terms. b. Follow directional commands. c. Understand conservation of matter. d. Use sentences of eight words. e. Tell exaggerated stories.

B, E Children ages 3 to 4 years can give and follow simple commands and tell exaggerated stories. Children cannot think abstractly at age 4 years. Conservation of matter is a developmental task of the school-age child. Five-year-old children use sentences with eight words with all parts of speech.

A pregnant woman in her first trimester with a history of epilepsy is transported to the hospital via ambulance after suffering a seizure in a restaurant. The nurse expects which health care provider orders to be included in the plan of care? (Select all that apply.) A. valproate (Depakote). B. Serum lab levels of medications. C. Abdominal ultrasounds. D. Prenatal vitamins with vitamin D. E. carbamazepine (Tegretol).

B. Serum lab levels of medications. C. Abdominal ultrasounds. D. Prenatal vitamins with vitamin D. Carbamazepine (Tegretol) and valproate (Depakote) should be avoided if possible during pregnancy, especially during the first trimester, because their use is associated with NTDs in the fetus. Checking lab levels of medications, performing abdominal ultrasounds to assess fetal growth, and taking prenatal vitamins with vitamin D are all expected interventions for a pregnant woman diagnosed with epilepsy.

A pregnant woman with cardiac disease is informed about signs of cardiac decompensation. She should be told that the earliest sign of decompensation is most often: A. orthopnea. B. decreasing energy levels. C. moist frequent cough and frothy sputum. D. crackles (rales) at the bases of the lungs on auscultation.

B. decreasing energy levels. Orthopnea is a finding that appears later when a failing heart reduces renal perfusion and fluid accumulates in the pulmonary interstitial space, leading to pulmonary edema. Decreasing energy level (fatigue) is an early finding of heart failure. Care must be taken to recognize it as a warning rather than a typical change of the third trimester. Cardiac decompensation is most likely to occur early in the third trimester, during childbirth, and during the first 48 hours following birth. A moist, frequent cough appears later when a failing heart reduces renal perfusion and fluid accumulates in the pulmonary interstitial space, leading to pulmonary edema. Crackles and rales appear later when a failing heart reduces renal perfusion and fluid accumulates in the pulmonary interstitial space, leading to pulmonary edema.

C (9 months Most infants can pull themselves to a standing position at age 9 months. Any infant who cannot pull to a standing position by age 11 to 12 months should be referred for further evaluation for developmental dysplasia of the hip. At 6 months, the infant has just obtained coordination of arms and legs. By age 8 months, infants can bear full weight on their legs.)

By what age should the nurse expect that an infant will be able to pull to a standing position? a. 6 months b. 8 months c. 9 moths d. 11 to 12 months

The nurse is preparing to discharge a 30-year-old woman who has experienced a miscarriage at 10 weeks of gestation. Which statement by the woman would indicate a correct understanding of the discharge instructions? A. "I will not experience mood swings since I was only at 10 weeks of gestation." B. "I will avoid sexual intercourse for 6 weeks and pregnancy for 6 months." C. "I should eat foods that are high in iron and protein to help my body heal." D. "I should expect the bleeding to be heavy and bright red for at least 1 week."

C ("I should eat foods that are high in iron and protein to help my body heal." After a miscarriage a woman may experience mood swings and depression from the reduction of hormones and the grieving process. Sexual intercourse should be avoided for 2 weeks or until the bleeding has stopped and should avoid pregnancy for 2 months. A woman who has experienced a miscarriage should be advised to eat foods that are high in iron and protein to help replenish her body after the loss. The woman should not experience bright red, heavy, profuse bleeding; this should be reported to the health care provider.)

A woman is documented on the labor and delivery board to be 7cm dilated. Her family wants to know how long she will be in labor. The nurse should provide which information to the family? a) "She is doing well, in the second stage and it could be anytime now." b) "She is in the transition phase of labor and it will be with in 2 to 3 hours, might be sooner." c) "She is in active labor, she is progressing at this point and we will keep you posted." d) "She is still in early latent labor and has much too long to go to tell when she will deliver."

C ("She is in active labor, she is progressing at this point and we will keep you posted." Rationale: At 7cm dilated, she is considered in Active Phase of labor. There is no science that can predict the length of labor. She is progressing in labor and it is not best to give the family a specific time frame.)

The nurse is preparing to discharge a 30-year-old woman who has experienced a miscarriage at 10 weeks of gestation. Which statement by the woman indicates a correct understanding of the discharge instructions? A. "I will not experience mood swings since I was only at 10 weeks of gestation." B. "I will avoid sexual intercourse for 6 weeks and pregnancy for 6 months." C. "I should eat foods that are high in iron and protein to help my body heal." D. "I should expect the bleeding to be heavy and bright red for at least 1 week."

C (A woman who has experienced a miscarriage should be advised to eat foods that are high in iron and protein to help replenish her body after the loss. After a miscarriage, a woman may experience mood swings and depression from the reduction of hormones and the grieving process. Sexual intercourse should be avoided for 2 weeks or until the bleeding has stopped and should avoid pregnancy for 2 months. The woman should not experience bright red, heavy, profuse bleeding; this should be reported to the health care provider.)

The nurse should initially implement which intervention when a nulliparous woman telephones the hospital to report that she is in labor. a) Emphasize that food and fluid should stop or be light b) Tell the woman to stay home until her membranes rupture c) Ask the woman to describe why she believes that she is in labor d) Arrange for the woman to come to the hospital for labor evaluation

C (Ask the woman to describe why she believes that she is in labor Rationale: The nurse needs further information to assist in determining if the woman is in true or false labor. She will need to ask the patient questions to seek further assessment and triage information. Having her wait until membranes rupture may be dangerous, as she may give birth before reaching the hospital. She should continue fluid intake until it is determined whether or not she is in labor. She may be in false labor, and more information should be obtained before she is brought to the hospital.)

There are four essential components of labor. The first is the passageway. It is composed of the bony pelvis and soft tissues. What is one component of the passageway? a) Uterus b) False pelvis c) Cervix d) Perineum

C (Cervix Rationale: The cervix and vagina are soft tissues that form the part of the passageway known as the birth canal.)

Which nursing assessment indicates that a woman who is in second-stage labor is almost ready to give birth? a. The fetal head is felt at 0 station during vaginal examination. b. Bloody mucus discharge increases. c. The vulva bulges and encircles the fetal head. d. The membranes rupture during a contraction.

C (During the active pushing (descent) phase, the woman has strong urges to bear down as the presenting part of the fetus descends and presses on the stretch receptors of the pelvic floor. The vulva stretches and begins to bulge encircling the fetal head. Birth of the head occurs when the station is +4. A 0 station indicates engagement. Bloody show occurs throughout the labor process and is not an indication of an imminent birth. Rupture of membranes can occur at any time during the labor process and does not indicate an imminent birth.)

The nurse observes that maternal hypotension has decreased uterine and fetal perfusion in a pregnant patient. What does the nurse need to assess further to understand the maternal status? A. D-dimer blood test B. Kleihauer-Betke (KB) test C. Electronic fetal monitoring D. Electrocardiogram reading

C (Electronic fetal monitoring reflects fetal cardiac responses to hypoxia and hypoperfusion and helps to assess maternal status after a trauma. The D-dimer blood test is used to rule out the presence of a thrombus. The KB test is used to evaluate transplacental hemorrhage. Electrocardiogram reading is more useful to assess the cardiac functions in nonpregnant cardiac patients.)

While providing care to a patient in active labor, the nurse should instruct the woman that: a. The supine position commonly used in the United States increases blood flow. b. The "all fours" position, on her hands and knees, is hard on her back. c. Frequent changes in position will help relieve her fatigue and increase her comfort. d. In a sitting or squatting position, her abdominal muscles will have to work harder.

C (Frequent position changes relieve fatigue, increase comfort, and improve circulation. Blood flow can be compromised in the supine position; any upright position benefits cardiac output. The "all fours" position is used to relieve backache in certain situations. In a sitting or squatting position, the abdominal muscles work in greater harmony with uterine contractions.)

At 37 weeks of gestation, the patient is in a severe automobile crash where her abdomen was hit by the steering wheel and her seat belt. What actions would the emergency room nurse expect to perform upon the patient's arrival at the hospital? A. Stay with the patient, assure a patent airway is present, and keep the patient as calm as possible. B. Move the patient's skirt to determine if any vaginal bleeding is present, find out who to call, and monitor the level of consciousness. C. Assess the patient's vital signs, determine location and severity of pain, and establish continual fetal heart rate monitoring. D. Obtain arterial blood gases, obtain a hemoglobin and hematocrit, and oxygen saturation rate.

C (Full assessment of the patient and her fetus are essential and include vital signs, continual fetal heart rate monitoring, determining the location and severity of pain, whether any vaginal bleeding is dark red or bright red, and the status of the abdomen, which would be expected to be rigid or "board like." Staying with the patient, assuring a patent airway is present, and keeping the patient as calm as possible would be appropriate at the crash site before the arrival of emergency medical services (EMS). The current status of the patient and fetus are thepriority. The health care provider would prescribe the arterial blood gases and other laboratory work after the patient is assessed and stabilized.)

If a fetus is in an ROA position during labor, you would interpret this to mean the fetus is a) Presenting with the face as the presenting part b) In a common breech delivery position c) In a longitudinal lie facing the left posterior d) Facing the right anterior abdominal quadrant

C (In a longitudinal lie facing the left posterior Rationale: ROA (right occiput anterior) means the occiput of the fetal head points toward the mother's right anterior pelvis; the head is the presenting part.)

The four essential components of labor are known as the "four P's". Which of the four P's involves the pelvis? a) Psyche b) Powers c) Passageway d) Passenger

C (Passageway Rationale: The passageway is one of the 4 P's and involves the pelvis, both bony pelvis and the soft tissues, cervix, and vagina. The "passenger" refers to the fetus. The primary powers are the involuntary contractions of the uterus, whereas the secondary powers come from the maternal abdominal muscles. The psyche refers to the mother's mental state.)

What does the nurse advise a pregnant patient who is prescribed phenazopyridine (Pyridium) for cystitis? A. "Avoid sweet foods in diet." B. "Limit exposure to sunlight." C. "Do not wear contact lenses." D. "Restrict oral fluids to 125 mL per hour."

C (Phenazopyridine (Pyridium) colors the tears orange. Therefore the nurse instructs the patient to avoid wearing contact lenses. Sweet foods are avoided in patients with diabetes mellitus, because they can cause fluctuating glucose levels, which may harm the fetus. Exposure to sunlight is avoided when the patient is receiving methotrexate therapy, because it causes photosensitivity. Oral fluids are restricted in patients who are at risk for pulmonary edema.)

What instruction does the nurse provide to a pregnant patient with mild preeclampsia? A. "You need to be hospitalized for fetal evaluation." B. "Nonstress testing can be done once every month." C. "Fetal movement counts need to be evaluated daily." D. "Take complete bed rest during the entire pregnancy."

C (Preeclampsia can affect the fetus and may cause fetal growth restrictions, decreased amniotic fluid volume, abnormal fetal oxygenation, low birth weight, and preterm birth. Therefore the fetal movements need to be evaluated daily. Patients with mild preeclampsia can be managed at home effectively and need not be hospitalized. Nonstress testing is performed once or twice per week to determine fetal well-being. Patients need to restrict activity, but complete bed rest is not advised because it may cause cardiovascular deconditioning, muscle atrophy, and psychological stress.)

Signs of a threatened abortion (miscarriage) are noted in a woman at 8 weeks of gestation. What is an appropriate management approach for this type of abortion? A. Prepare the woman for a dilation and curettage (D&C). B. Place the woman on bed rest for at least 1 week and reevaluate. C. Prepare the woman for an ultrasound and blood work. D. Comfort the woman by telling her that if she loses this baby, she may attempt to get pregnant again in 1 month.

C (Prepare the woman for an ultrasound and blood work. D&C is not considered until signs of the progress to an inevitable abortion are noted or the contents are expelled and incomplete. Bed rest is recommended for 48 hours initially. Repetitive transvaginal ultrasounds and measurement of human chorionic gonadotropin (hCG) and progesterone levels may be performed to determine if the fetus is alive and within the uterus. If the pregnancy is lost, the woman should be guided through the grieving process. Telling the client that she can get pregnant again soon is not a therapeutic response because it discounts the importance of this pregnancy.)

Which intervention will help prevent the risk of pulmonary edema in a pregnant patient with severe preeclampsia? A. Assess fetal heart rate (FHR) abnormalities regularly. B. Place the patient on bed rest in a darkened environment. C. Restrict total intravenous (I.V.) and oral fluids to 125 mL/hr. D. Ensure that magnesium sulfate is administered as prescribed.

C (Pulmonary edema may be seen in patients with severe preeclampsia. Therefore the nurse needs to restrict total intravenous (I.V.) and oral fluids to 125 mL/hr. FHR monitoring helps assess any fetal complications. The patient is placed on bed rest in a darkened environment to prevent stress. Magnesium sulfate is administered to prevent eclamptic seizures.)

Signs of a threatened abortion (miscarriage) are noted in a woman at 8 weeks of gestation. What is an appropriate management approach for this type of abortion? A. Prepare the woman for a dilation and curettage (D&C). B. Place the woman on bed rest for at least 1 week and reevaluate. C. Prepare the woman for an ultrasound and bloodwork. D. Comfort the woman by telling her that if she loses this baby, she may attempt to get pregnant again in 1 month.

C (Repetitive transvaginal ultrasounds and measurement of human chorionic gonadotropin (hCG) and progesterone levels may be performed to determine if the fetus is alive and within the uterus. If the pregnancy is lost , the woman should be guided through the grieving process. D&C is not considered until signs of the progress to an inevitable abortion are noted or the contents are expelled and incomplete. Bed rest is recommended for 48 hours initially. Telling the woman that she can get pregnant again soon is not a therapeutic response because it discounts the importance of this pregnancy.)

In preparing for the actual birth, which fetal presentation would a nurse be least likely to find? a) Transverse lie b) Breech c) Shoulder d) Oblique lie

C (Shoulder Rationale: Shoulder presentations are the least likely to occur in less than 0.3 percent of all births. Approximately 97 percent of fetuses are in a cephalic presentation at the end of pregnancy. A longitudinal lie, in which the long axis of the fetus is parallel to the long axis of the mother, is the most common. When the fetus is in a transverse lie, the long axis of the fetus is perpendicular to the long axis of the woman. An oblique lie is in between the two.)

A nurse is reviewing hormone changes that occur during adolescence. The hormone that is responsible for the growth of beard, mustache, and body hair in the male is: a. Estrogen. c. Androgen. b. Pituitary hormone. d. Progesterone.

C Beard, mustache, and body hair on the chest, upward along the linea alba, and sometimes on other areas (e.g., back and shoulders) appears in males and is androgen dependent. Estrogen and progesterone are produced by the ovaries in the female and do not contribute to body hair appearance in the male. The pituitary hormone does not have any relationship to body hair appearance in the male.

With regard to factors that affect how the fetus moves through the birth canal, nurses should be aware that: a. The fetal attitude describes the angle at which the fetus exits the uterus. b. Of the two primary fetal lies, the horizontal lie is that in which the long axis of the fetus is parallel to the long axis of the mother. c. The normal attitude of the fetus is called general flexion. d. The transverse lie is preferred for vaginal birth.

C (The normal attitude of the fetus is general flexion. The fetal attitude is the relation of fetal body parts to one another. The horizontal lie is perpendicular to the mother; in the longitudinal (or vertical) lie the long axes of the fetus and the mother are parallel. Vaginal birth cannot occur if the fetus stays in a transverse lie.)

A blunt abdominal trauma causes fetal hemorrhage in a pregnant patient. The nurse finds that the patient is Rh negative. What action does the nurse take? A. Initiate magnesium sulfate per protocol. B. Administer oxytocin (pitocin). C. Administer prescribed Rho (D) immunoglobulin. D. Prepare the patient for magnetic resonance imaging (MRI).

C (The nurse administers the prescribed Rho(D) immunoglobulin to the patient to protect the patient from isoimmunization. The nurse needs to obtain a prescription for magnesium sulfate if there are eclamptic seizures in a patient with preeclampsia. Oxytocin (Pitocin) is administered to prevent bleeding after birth or the evacuation of the uterus. Magnetic resonance imaging (MRI) is used to assess injuries in a patient after trauma.)

With regard to the turns and other adjustments of the fetus during the birth process, known as the mechanism of labor, nurses should be aware that: a. The seven critical movements must progress in a more or less orderly sequence. b. Asynclitism sometimes is achieved by means of the Leopold maneuver. c. The effects of the forces determining descent are modified by the shape of the woman's pelvis and the size of the fetal head. d. At birth the baby is said to achieve "restitution" (i.e., a return to the C-shape of the womb).

C (The size of the maternal pelvis and the ability of the fetal head to mold also affect the process. The seven identifiable movements of the mechanism of labor occur in combinations simultaneously, not in precise sequences. Asynclitism is the deflection of the baby's head; the Leopold maneuver is a means of judging descent by palpating the mother's abdomen. Restitution is the rotation of the baby's head after the infant is born.)

The nurse knows that the second stage of labor, the descent phase, has begun when: a) The amniotic membranes rupture b) The cervix cannot be felt during a vaginal examination c) The woman experiences a strong urge to bear down d) The presenting part is below the ischial spines

C (The woman experiences a strong urge to bear down)

The maternity nurse understands that as the uterus contracts during labor, maternal-fetal exchange of oxygen and waste products: a. Continues except when placental functions are reduced. b. Increases as blood pressure decreases. c. Diminishes as the spiral arteries are compressed. d. Is not significantly affected.

C (Uterine contractions during labor tend to decrease circulation through the spiral electrodes and subsequent perfusion through the intervillous space. The maternal blood supply to the placenta gradually stops with contractions. The exchange of oxygen and waste products decreases. The exchange of oxygen and waste products is affected by contractions.)

During the preschool period, the emphasis of injury prevention should be placed on: a. Constant vigilance and protection. b. Punishment for unsafe behaviors. c. Education for safety and potential hazards. d. Limitation of physical activities.

C Education for safety and potential hazards is appropriate for preschoolers because they can begin to understand dangers. Constant vigilance and protection is not practical at this age since preschoolers are becoming more independent. Punishment may make children scared of trying new things. Limitation of physical activities is not appropriate.

The parents of a 15-year-old girl are concerned that their adolescent spends too much time looking in the mirror. Which statement is the most appropriate for the nurse to make? a. "Your teenager needs clearer and stricter limits about her behavior." b. "Your teenager needs more responsibility at home." c. "During adolescence this behavior is not unusual." d. "The behavior is abnormal and needs further investigation."

C Egocentric and narcissistic behavior is normal during this period of development. The teenager is seeking a personal identity. Stricter limits are not an appropriate response for a behavior that is part of normal development. More responsibility at home is not an appropriate response for this situation. The behavior is normal and needs no further investigation.

Vitamin A supplementation may be recommended for the young child who has: a. Mumps. c. Measles (rubeola). b. Rubella. d. Erythema infectiosum.

C Evidence shows that vitamin A decreases morbidity and mortality associated with measles. Vitamin A will not lessen the effects of mumps, rubella, or fifth disease.

Fetal bradycardia is most common during: a. Intraamniotic infection. b. Fetal anemia. c. Prolonged umbilical cord compression. d. Tocolytic treatment using terbutaline.

C Fetal bradycardia can be considered a later sign of fetal hypoxia and is known to occur before fetal death. Bradycardia can result from placental transfer of drugs, prolonged compression of the umbilical cord, maternal hypothermia, and maternal hypotension. Intraamniotic infection, fetal anemia, and tocolytic treatment using terbutaline would most likely result in fetal tachycardia.

Young people with anorexia nervosa are often described as being: a. Independent. c. Conforming. b. Disruptive. d. Low achieving.

C Individuals with anorexia nervosa are described as perfectionist, academically high achievers, conforming, and conscientious. Independent, disruptive, and low achieving are not part of the behavioral characteristics of anorexia nervosa.

In providing anticipatory guidance to parents whose child will soon be entering kindergarten, which is a critical factor in preparing a child for kindergarten entry? a. The child's ability to sit still b. The child's sense of learned helplessness c. The parent's interactions and responsiveness to the child d. Attending a preschool program

C Interactions between the parent and child are an important factor in the development of academic competence. Parental encouragement and support maximize a child's potential. The child's ability to sit still is important to learning; however, parental responsiveness and involvement are more important factors. Learned helplessness is the result of a child feeling that he or she has no effect on the environment and that his or her actions do not matter. Parents who are actively involved in a supportive learning environment will demonstrate a more positive approach to learning. Preschool and day care programs can supplement the developmental opportunities provided by parents at home, but they are not critical in preparing a child for entering kindergarten.

The nurse caring for the woman in labor should understand that maternal hypotension can result in: a. Early decelerations. c. Uteroplacental insufficiency. b. Fetal dysrhythmias. d. Spontaneous rupture of membranes.

C Low maternal blood pressure reduces placental blood flow during uterine contractions and results in fetal hypoxemia. Maternal hypotension is not associated with early decelerations, fetal dysrhythmias, or spontaneous rupture of membranes.

The nurse caring for the woman in labor should understand that increased variability of the fetal heart rate may be caused by: a. Narcotics. c. Methamphetamines. b. Barbiturates. d. Tranquilizers.

C Narcotics, barbiturates, and tranquilizers may be causes of decreased variability; methamphetamines may cause increased variability.

Imaginary playmates are beneficial to the preschool child because they: a. Take the place of social interactions. b. Take the place of pets and other toys. c. Become friends in times of loneliness. d. Accomplish what the child has already successfully accomplished.

C One purpose of an imaginary friend is to be a friend in time of loneliness. Imaginary friends do not take the place of social interactions but may encourage conversation. Imaginary friends do not take the place of pets or toys. They accomplish what the child is still attempting, not what has already been accomplished.

A 4-year-old child tells the nurse that she does not want another blood sample drawn because "I need all my insides, and I don't want anyone taking them out." Which is the nurse's best interpretation of this? a. Child is being overly dramatic. b. Child has a disturbed body image. c. Preschoolers have poorly defined body boundaries. d. Preschoolers normally have a good understanding of their bodies.

C Preschoolers have little understanding of body boundaries, which leads to fears of mutilation. The child is not capable of being dramatic at 4 years of age. She truly has fear. Body image is just developing in the school-age child. Preschoolers do not have good understanding of their bodies.

The nurse caring for a woman in labor understands that prolonged decelerations: a. Are a continuing pattern of benign decelerations that do not require intervention. b. Constitute a baseline change when they last longer than 5 minutes. c. Usually are isolated events that end spontaneously. d. Require the usual fetal monitoring by the nurse.

C Prolonged decelerations usually are isolated events that end spontaneously. However, in certain combinations with late and/or variable decelerations, they are a danger sign that requires the nurse to notify the physician or midwife immediately. A deceleration that lasts longer than 10 minutes constitutes a baseline change.

Which statement by the nurse is most appropriate to a 15-year-old whose friend has mentioned suicide? a. "Tell your friend to come to the clinic immediately." b. "You need to gather details about your friend's suicide plan." c. "Your friend's threat needs to be taken seriously and immediate help for your friend is important." d. "If your friend mentions suicide a second time, you will want to get your friend some help."

C Suicide is the third most common cause of death among American adolescents. A suicide threat from an adolescent serves as a dramatic message to others and should be taken seriously. Adolescents at risk should be targeted for supportive guidance and counseling before a crisis occurs. Instructing a 15-year-old to tell a friend to come to the clinic immediately provides the teen with limited information and does not address the concern. It is important to determine whether a person threatening suicide has a plan of action; however, the best information for the 15-year-old to have is that all threats of suicide should be taken seriously and immediate help is important. Taking time to gather details or waiting until the teen discusses it a second time may be too late.

The nurse is explaining Tanner staging to an adolescent and her mother. Which statement best describes Tanner staging? a. Predictable stages of puberty that are based on chronologic age b. Staging of puberty based on the initiation of menarche and nocturnal emissions c. Predictable stages of puberty that are based on primary and secondary sexual characteristics d. Staging of puberty based on the initiation of primary sexual characteristics

C Tanner sexual-maturing ratings are based on the development of stages of primary and secondary sexual characteristics. Tanner stages are not based on chronologic age. The age at which an adolescent enters puberty is variable. The puberty stage in girls begins with breast development. The puberty stage in boys begins with genital enlargement. Primary sexual characteristics are not the sole basis of Tanner staging.

According to standard professional thinking, nurses should auscultate the fetal heart rate (FHR): a. Every 15 minutes in the active phase of the first stage of labor in the absence of risk factors. b. Every 20 minutes in the second stage, regardless of whether risk factors are present. c. Before and after ambulation and rupture of membranes. d. More often in a woman's first pregnancy.

C The FHR should be auscultated before and after administration of medications and induction of anesthesia. In the active phase of the first stage of labor, the FHR should be auscultated every 30 minutes if no risk factors are involved; with risk factors it should be auscultated every 15 minutes. In the second stage of labor the FHR should be auscultated every 15 minutes if no risk factors are involved; with risk factors it should be auscultated every 5 minutes. The fetus of a first-time mother is automatically at greater risk.

The uterine contractions of a woman early in the active phase of labor are assessed by an internal uterine pressure catheter (IUPC). The nurse notes that the intrauterine pressure at the peak of the contraction ranges from 65 to 70 mm Hg and the resting tone range is 6 to 10 mm Hg. The uterine contractions occur every 3 to 4 minutes and last an average of 55 to 60 seconds. On the basis of this information, the nurse should: a. Notify the woman's primary health care provider immediately. b. Prepare to administer an oxytocic to stimulate uterine activity. c. Document the findings because they reflect the expected contraction pattern for the active phase of labor. d. Prepare the woman for the onset of the second stage of labor.

C The nurse is responsible for monitoring the uterine contractions to ascertain whether they are powerful and frequent enough to accomplish the work of expelling the fetus and the placenta. In addition, the nurse would document these findings in the client's medical record. This labor pattern indicates that the client is in the active phase of the first stage of labor. Nothing indicates a need to notify the primary care provider at this time. Oxytocin augmentation is not needed for this labor pattern; this contraction pattern indicates adequate active labor. Her contractions eventually will become stronger, last longer, and come closer together during the transition phase of the first stage of labor. The transition phase precedes the second stage of labor, or delivery of the fetus.

The parents of a 14-year-old girl express concerns about the number of hours their daughter spends with her friends. The nurse explains that peer relationships become more important during adolescence because: a. Adolescents dislike their parents. b. Adolescents no longer need parental control. c. They provide adolescents with a feeling of belonging. d. They promote a sense of individuality in adolescents.

C The peer group serves as a strong support to teenagers, providing them with a sense of belonging and strength and power. During adolescence, the parent-child relationship changes from one of protection-dependency to one of mutual affection and quality. Parents continue to play an important role in personal and health-related decisions. The peer group forms the transitional world between dependence and autonomy.

What is an advantage of external electronic fetal monitoring? a. The ultrasound transducer can accurately measure short-term variability and beat-to-beat changes in the fetal heart rate. b. The tocotransducer can measure and record the frequency, regularity, intensity, and approximate duration of uterine contractions (UCs). c. The tocotransducer is especially valuable for measuring uterine activity during the first stage of labor. d. Once correctly applied by the nurse, the transducer need not be repositioned even when the woman changes positions.

C The tocotransducer is especially valuable for measuring uterine activity during the first stage of labor, particularly when the membranes are intact. Short-term changes cannot be measured with this technology. The tocotransducer cannot measure and record the intensity of UCs. The transducer must be repositioned when the woman or fetus changes position.

The school nurse is discussing testicular self-examination with adolescent boys. Why is this important? a. Epididymitis is common during adolescence. b. Asymptomatic sexually transmitted diseases may be present. c. Testicular tumors during adolescence are generally malignant. d. Testicular tumors, although usually benign, are common during adolescence.

C Tumors of the testes are not common, but when manifested in adolescence, they are generally malignant and demand immediate evaluation. Epididymitis is not common in adolescence. Asymptomatic sexually transmitted disease would not be evident during testicular self-examination. The focus of this examination is on testicular cancer. Testicular tumors are most commonly malignant.

The nurse providing care for the laboring woman should understand that late fetal heart rate (FHR) decelerations are the result of: a. Altered cerebral blood flow. c. Uteroplacental insufficiency. b. Umbilical cord compression. d. Meconium fluid.

C Uteroplacental insufficiency would result in late decelerations in the FHR. Altered fetal cerebral blood flow would result in early decelerations in the FHR. Umbilical cord compression would result in variable decelerations in the FHR. Meconium-stained fluid may or may not produce changes in the fetal heart rate, depending on the gestational age of the fetus and whether other causative factors associated with fetal distress are present.

When educating a group of nursing students about the theories of onset of labor, the nurse identifies which of the following factors as the possible causes for onset of labor? Select all that apply. a) Increase in the production of progesterone b) Fall in the estrogen at 34-35 weeks of pregnancy c) Prostaglandin production in the myometrium d) Increase in the fetal cortisol levels e) Release of oxytocin by the pituitary

C, D, E (Rationale: The possible causes for the onset of labor include increase in the fetal cortisol levels, release of oxytocin by the posterior pituitary and the production of prostaglandins. Progesterone withdrawal, and not an increase, initiates labor. There is a rise in the estrogen levels at 34-35 weeks of pregnancy. Estrogen stimulates prostaglandin production and also promotes the release of oxytocin.)

The baseline fetal heart rate (FHR) is the average rate during a 10-minute segment. Changes in FHR are categorized as periodic or episodic. These patterns include both accelerations and decelerations. The labor nurse is evaluating the patient's most recent 10-minute segment on the monitor strip and notes a late deceleration. This is likely to be caused by which physiologic alteration (Select all that apply)? a. Spontaneous fetal movement b. Compression of the fetal head c. Placental abruption d. Cord around the baby's neck e. Maternal supine hypotension

C, E Late decelerations are almost always caused by uteroplacental insufficiency. Insufficiency is caused by uterine tachysystole, maternal hypotension, epidural or spinal anesthesia, IUGR, intraamniotic infection, or placental abruption. Spontaneous fetal movement, vaginal examination, fetal scalp stimulation, fetal reaction to external sounds, uterine contractions, fundal pressure and abdominal palpation are all likely to cause accelerations of the FHR. Early decelerations are most often the result of fetal head compression and may be caused by uterine contractions, fundal pressure, vaginal examination, and placement of an internal electrode. A variable deceleration is likely caused by umbilical cord compression. This may happen when the umbilical cord is around the baby's neck, arm, leg, or other body part or when there is a short cord, a knot in the cord, or a prolapsed cord.

Maternal and neonatal risks associated with gestational diabetes mellitus are: A. maternal premature rupture of membranes and neonatal sepsis. B. maternal hyperemesis and neonatal low birth weight. C. Maternal preeclampsia and fetal macrosomia. D. maternal placenta previa and fetal prematurity.

C. Maternal preeclampsia and fetal macrosomnia Premature rupture of membranes and neonatal sepsis are not risks associated with gestational diabetes. Hyperemesis is not seen with gestational diabetes, nor is there an association with low birth weight of the infant. Women with gestational diabetes have twice the risk of developing hypertensive disorders such as preeclampsia, and the baby usually has macrosomia. Placental previa and subsequent prematurity of the neonate are not risks associated with gestational diabetes.

D, E (D. sphincter control is achieved. E. primary dentition is complete.)

Characteristics of physical development of a 30-month-old child are the: (Select all that apply.) A. anterior fontanel is open. B. birth weight has doubled. C. genital fondling is noted. D. sphincter control is achieved. E. primary dentition is complete.

The patient is having a routine prenatal visit and asks the nurse what the childbirth education teacher meant when she used the term zero station. What is the best response by the nurse? a) "This is just a way of determining your progress in labor." b) This indicates that you start labor within the next 24 hours." c) "This means +1 and the baby is entering the true pelvis." d) "The presenting part is at the true pelvis and is engaged."

D ("The presenting part is at the true pelvis and is engaged." Rationale: Zero station is when the fetus is engaged in the pelvis, or has dropped. This is an encouraging sign for the patient. This sign is indicative that labor may be beginning, but there is no set time frame regarding when it will start. Labor has not started yet, and the fetus has not begun to move out of the uterus.)

A pregnant woman, multipara, has been in labor for several hours. She cries out that her contractions are getting harder and that she can't do this. The patient is really irritable, nauseated, annoyed, and fearful of being left alone. Considering the client's behavior, the nurse would expect the cervix to be dilated how many centimeters? a) 5-6 b) 0-2 c) 3-5 d) 8-10

D (8-10 Rationale: The reaction of the patient is indicative of entering or being in the transition phase of labor, stage 1. The dilation would be 8-10 cm. Before that, when dilation is 0-7 cm, the patient has an easier time using positive coping skills.)

Which basic type of pelvis includes the correct description and percentage of occurrence in women? a. Gynecoid: classic female; heart shaped; 75% b. Android: resembling the male; wider oval; 15% c. Anthropoid: resembling the ape; narrower; 10% d. Platypelloid: flattened, wide, shallow; 3%

D (A platypelloid pelvis is flattened, wide, and shallow; about 3% of women have this shape. The gynecoid shape is the classical female shape, slightly ovoid and rounded; about 50% of women have this shape. An android, or malelike, pelvis is heart shaped; about 23% of women have this shape. An anthropoid, or apelike, pelvis is oval and wider; about 24% of women have this shape.)

Which of the following would be least effective in promoting a positive birth outcome for a woman in labor? a) Promoting the woman's feelings of control b) Providing clear information about procedures c) Encouraging the woman to use relaxation techniques d) Allowing the woman time to be alone

D (Allowing the woman time to be alone Rationale: Positive support, not being alone, promotes a positive birth experience. Being alone can increase anxiety and fear, decreasing the woman's ability to cope. Feelings of control promote self-confidence and self-esteem, which in turn help the woman to cope with the challenges of labor. Information about procedures reduces anxiety about the unknown and fosters cooperation and self-confidence in her abilities to deal with labor. Catecholamines are secreted in response to anxiety and fear and can inhibit uterine blood flow and placental perfusion. Relaxation techniques can help to reduce anxiety and fear, in turn decreasing the secretion of catecholamines and ultimately improving the woman's ability to cope with labor.)

What does the nurse administer to a patient if there is excessive bleeding after suction curettage? A. Nifedipine (Procardia) B. Methyldopa (Aldomet) C. Hydralazine (Apresoline) D. Ergonovine (Methergine)

D (Ergonovine (Methergine) is an ergot product, which is administered to contract the uterus when there is excessive bleeding after suction curettage. Nifedipine (Procardia) is prescribed for gestational hypertension or severe preeclampsia. Methyldopa (Aldomet) is an antihypertensive medication indicated for pregnant patients with hypertension. Hydralazine (Apresoline) is also an antihypertensive medication used for treating hypertension intrapartum.)

What term is used to describe the position of the fetal long axis in relation to the long axis of the mother? a) Fetal position b) Fetal presentation c) Fetal attitude d) Fetal lie

D (Fetal lie Rationale: Fetal lie describes the position of the long axis of the fetus in relation to the long axis of the pregnant woman.)

The third stage of labor is considered to be expulsion of the placenta. This stage can last anywhere from five to 20 minutes. What is a sign that the placenta is separating from the wall of the uterus? a) Expulsion of blood clots b) Firm fundus c) Shortening of the umbilical cord d) Globular shape to the fundus

D (Globular shape to the fundus Rationale: Signs that indicate the placenta is separating from the uterine wall include a gush of blood, lengthening of the umbilical cord, and a globular shape to the fundus.)

Assessment reveals that the fetus of a client in labor is in the vertex presentation. The nurse determines that the presenting part is which of the following? a) Brow b) Buttocks c) Shoulders d) Occiput

D (Occiput Rationale: With a vertex presentation, a type of cephalic presentation, the fetal presenting part is the occiput. The shoulders are the presenting part when the fetus is in a shoulder presentation. The brow or sinciput is the presenting part when a fetus is in a brow presentation. The buttocks are the presenting part when a fetus is in a breech presentation.)

A client in labor has been admitted to the labor and birth suite. The nurse assessing her notes that the fetus is in a cephalic presentation. Which of the following should the nurse identify by the term presentation? a) Relation of the different fetal body parts to one another b) Relationship of the presenting part to the maternal pelvis c) Relation of the fetal presenting part to the maternal ischial spine d) Part of the fetal body entering the maternal pelvis first

D (Part of the fetal body entering the maternal pelvis first Rationale: The term presentation is the part of the fetal body that is entering the maternal pelvis first. Relationship of the presenting part to the sides of the maternal pelvis is called the position. Attitude is the term that describes the relation of the different fetal body parts to one another. Relation of the fetal presenting part to maternal ischial spine is termed the station.)

A 32-year-old woman presents to the labor-and-delivery suite in active labor. She is multigravida, relaxed, and talking with her husband. When examined by the nurse, the fetus is found to be in a cephalic presentation. His occiput is facing toward the front and slightly to the right of the mother's pelvis, and he is exhibiting a flexed attitude. How does the nurse document the position of the fetus? a) ROP b) LOP c) LOA d) ROA

D (ROA Rationale: Document the fetal position in the clinical record using abbreviations (Box 8-1). The first letter describes the side of the maternal pelvis toward which the presenting part is facing ("R" for right and "L" for left). The second letter or abbreviation indicates the reference point ("O" for occiput, "Fr" for frontum, etc.). The last part of the designation specifies whether the presenting part is facing the anterior (A) or the posterior (P) portion of the pelvis, or whether it is in a transverse (T) position.)

In order to accurately assess the health of the mother accurately during labor, the nurse should be aware that: a) The woman's blood pressure increases during contractions and falls back to prelabor normal between contractions b) Use of the Valsalva maneuver is encouraged during the second stage of labor to relieve fetal hypoxia c) Having the woman point her toes reduces leg cramps d) The endogenous endorphins released during labor raise the woman's pain threshold and produce sedation

D (The endogenous endorphins released during labor raise the woman's pain threshold and produce sedation.)

In order to evaluate the condition of the patient accurately during labor, the nurse should be aware that: a. The woman's blood pressure will increase during contractions and fall back to prelabor normal between contractions. b. Use of the Valsalva maneuver is encouraged during the second stage of labor to relieve fetal hypoxia. c. Having the woman point her toes will reduce leg cramps. d. The endogenous endorphins released during labor will raise the woman's pain threshold and produce sedation.

D (The endogenous endorphins released during labor will raise the woman's pain threshold and produce sedation. In addition, physiologic anesthesia of the perineal tissues, caused by the pressure of the presenting part, decreases the mother's perception of pain. Blood pressure increases during contractions but remains somewhat elevated between them. Use of the Valsalva maneuver is discouraged during second stage labor because of a number of unhealthy outcomes, including fetal hypoxia. Pointing the toes can cause leg cramps, as can the process of labor itself.)

Which is a priority nursing action when a pregnant patient with severe gestational hypertension is admitted to the health care facility? A. Prepare the patient for cesarean delivery. B. Administer intravenous (I.V.) and oral fluids. C. Provide diversionary activities during bed rest. D. Administer the prescribed magnesium sulfate.

D (The nurse administers the prescribed magnesium sulfate to the patient to prevent eclamptic seizures. I.V. oral fluids are indicated when there is severe dehydration in the patient. It is important to provide diversionary activities during bed rest, but it is secondary in this case. A patient who has experienced a multisystem trauma is prepared for cesarean delivery if there is no evidence of a maternal pulse, which increases the chance of maternal survival.)

A primigravida at 39 weeks of gestation is observed for 2 hours in the intrapartum unit. The fetal heart rate has been normal. Contractions are 5 to 9 minutes apart, 20 to 30 seconds in duration, and of mild intensity. Cervical dilation is 1 to 2 cm and uneffaced (unchanged from admission). Membranes are intact. The nurse should expect the woman to be: a. Admitted and prepared for a cesarean birth. b. Admitted for extended observation. c. Discharged home with a sedative. d. Discharged home to await the onset of true labor.

D (This situation describes a woman with normal assessments who is probably in false labor and will probably not deliver rapidly once true labor begins. These are all indications of false labor without fetal distress. There is no indication that further assessment or cesarean birth is indicated. The patient will likely be discharged; however, there is no indication that a sedative is needed.)

A woman at 39 weeks of gestation with a history of preeclampsia is admitted to the labor and birth unit. She suddenly experiences increased contraction frequency of every 1 to 2 minutes; dark red vaginal bleeding; and a tense, painful abdomen. The nurse suspects the onset of: A. eclamptic seizure. B. rupture of the uterus. C. placenta previa. D. placental abruption.

D (placental abruption. Eclamptic seizures are evidenced by the presence of generalized tonic-clonic convulsions. Uterine rupture presents as hypotonic uterine activity, signs of hypovolemia, and in many cases the absence of pain. Placenta previa presents with bright red, painless vaginal bleeding. Uterine tenderness in the presence of increasing tone may be the earliest finding of premature separation of the placenta (abruptio placentae or placental abruption). Women with hypertension are at increased risk for an abruption.)

The nurse is guiding parents in selecting a day care facility for their child. When making the selection, it is especially important to consider: a. Structured learning environment. b. Socioeconomic status of children. c. Cultural similarities of children. d. Teachers knowledgeable about development.

D A teacher knowledgeable about development will structure activities for learning. A structured learning environment is not necessary at this age. Socioeconomic status is not the most important factor in selecting a preschool. Preschool is about expanding experiences with others; cultural similarities are not necessary.

The most common cause of decreased variability in the fetal heart rate (FHR) that lasts 30 minutes or less is: a. Altered cerebral blood flow. c. Umbilical cord compression. b. Fetal hypoxemia. d. Fetal sleep cycles.

D A temporary decrease in variability can occur when the fetus is in a sleep state. These sleep states do not usually last longer than 30 minutes. Altered fetal cerebral blood flow would result in early decelerations in the FHR. Fetal hypoxemia would be evidenced by tachycardia initially and then bradycardia. A persistent decrease or loss of FHR variability may be seen. Umbilical cord compression would result in variable decelerations in the FHR.

By what age would the nurse expect that most children could understand prepositional phrases such as "under," "on top of," "beside," and "in back of"? a. 18 months c. 3 years b. 24 months d. 4 years

D At 4 years, children can understand directional phrases. Children 18 to 24 months and 3 years of age are too young.

Which characteristic best describes the language of a 3-year-old child? a. Asks meanings of words b. Follows directional commands c. Can describe an object according to its composition d. Talks incessantly, regardless of whether anyone is listening

D Because of the dramatic vocabulary increase at this age, 3-year-olds are known to talk incessantly, regardless of whether anyone is listening. A 4- to 5-year-old asks lots of questions and can follow simple directional commands. A 6-year-old can describe an object according to its composition.

By what age should concerns about pubertal delay be considered in boys? a. 12 to 12.5 years c. 13 to 13.5 years b. 12.5 to 13 years d. 13.5 to 14 years

D Concerns about pubertal delay should be considered for boys who exhibit no enlargement of the testes or scrotal changes by 13.5 to 14 years of age. Ages younger than 13.5 years are too young for initial concern.

Which is probably the most important criterion on which to base the decision to report suspected child abuse? a. Inappropriate parental concern for the degree of injury b. Absence of parents for questioning about child's injuries c. Inappropriate response of child d. Incompatibility between the history and injury observed

D Conflicting stories about the "accident" are the most indicative red flags of abuse. Inappropriate response of caregiver or child may be present, but is subjective. Parents should be questioned at some point during the investigation.

When assessing the relative advantages and disadvantages of internal and external electronic fetal monitoring, nurses comprehend that both: a. Can be used when membranes are intact. b. Measure the frequency, duration, and intensity of uterine contractions. c. May need to rely on the woman to indicate when uterine activity (UA) is occurring. d. Can be used during the antepartum and intrapartum periods.

D External monitoring can be used in both periods; internal monitoring can be used only in the intrapartum period. For internal monitoring the membranes must have ruptured, and the cervix must be sufficiently dilated. Internal monitoring measures the intensity of contractions; external monitoring cannot do this. With external monitoring, the woman may need to alert the nurse that UA is occurring; internal monitoring does not require this.

Increasing the infusion rate of nonadditive intravenous fluids can increase fetal oxygenation primarily by: a. Maintaining normal maternal temperature. b. Preventing normal maternal hypoglycemia. c. Increasing the oxygen-carrying capacity of the maternal blood. d. Expanding maternal blood volume.

D Filling the mother's vascular system makes more blood available to perfuse the placenta and may correct hypotension. Increasing fluid volume may alter the maternal temperature only if she is dehydrated. Most intravenous fluids for laboring women are isotonic and do not provide extra glucose. Oxygen-carrying capacity is increased by adding more red blood cells.

Which is the causative agent of scarlet fever? a. Enteroviruses b. Corynebacterium organisms c. Scarlet fever virus d. Group A -hemolytic streptococci (GABHS)

D GABHS infection causes scarlet fever. Enteroviruses do not cause the same complications. Corynebacterium organisms cause diphtheria. Scarlet fever is not caused by a virus.

Which statement is most descriptive of bulimia during adolescence? a. Strong sense of control over eating behavior b. Feelings of elation after the binge-purge cycle c. Profound lack of awareness that the eating pattern is abnormal d. Weight that can be normal, slightly above normal, or below normal

D Individuals with bulimia are of normal weight or more commonly slightly above normal weight. Those who also restrict their intake can become severely underweight. Behavior related to this eating disorder is secretive, frenzied, and out of control. These cycles are followed by self-deprecating thoughts and a depressed mood. These young women are keenly aware that this eating pattern is abnormal.

Which fetal heart rate (FHR) finding would concern the nurse during labor? a. Accelerations with fetal movement c. An average FHR of 126 beats/min b. Early decelerations d. Late decelerations

D Late decelerations are caused by uteroplacental insufficiency and are associated with fetal hypoxemia. They are considered ominous if persistent and uncorrected. Accelerations in the FHR are an indication of fetal well-being. Early decelerations in the FHR are associated with head compression as the fetus descends into the maternal pelvic outlet; they generally are not a concern during normal labor.

As a perinatal nurse you realize that a fetal heart rate that is tachycardic, is bradycardic, or has late decelerations or loss of variability is nonreassuring and is associated with: a. Hypotension. c. Maternal drug use. b. Cord compression. d. Hypoxemia.

D Nonreassuring heart rate patterns are associated with fetal hypoxemia. Fetal bradycardia may be associated with maternal hypotension. Fetal variable decelerations are associated with cord compression. Maternal drug use is associated with fetal tachycardia.

When planning care for adolescents, the nurse should: a. Teach parents first, and they, in turn, will teach the teenager. b. Provide information for their long-term health needs because teenagers respond best to long-range planning. c. Maintain the parents' role by providing explanations for treatment and procedures to the parents only. d. Give information privately to adolescents about how they can manage the specific problems that they identify.

D Problems that teenagers identify and are interested in are typically the problems that they are the most willing to address. Confidentiality is important to adolescents. Adolescents prefer to confer privately (without parents) with the nurse and health care provider. Teenagers are socially and cognitively at the developmental stage where the health care provider can teach them. The nurse must keep in mind that teenagers are more interested in immediate health care needs than in long-term needs

An adolescent teen has bulimia. Which assessment finding should the nurse expect? a. Diarrhea c. Cold intolerance b. Amenorrhea d. Erosion of tooth enamel

D Some of the signs of bulimia include erosion of tooth enamel, increased dental caries from vomited gastric acid, throat complaints, fluid and electrolyte disturbances, and abdominal complaints from laxative abuse. Diarrhea is not a result of the vomiting. It may occur in patients with bulimia who also abuse laxatives. Amenorrhea and cold intolerance are characteristics of anorexia nervosa, which some bulimics have. These symptoms are related to the extreme low weight.

Which symptoms should the nurse expect to observe during the physical assessment of an adolescent girl with severe weight loss and disrupted metabolism associated with anorexia nervosa? a. Dysmenorrhea and oliguria b. Tachycardia and tachypnea c. Heat intolerance and increased blood pressure d. Lowered body temperature and brittle nails

D Symptoms of anorexia nervosa include lower body temperature, severe weight loss, decreased blood pressure, dry skin, brittle nails, altered metabolic activity, and presence of lanugo hair. Amenorrhea, rather than dysmenorrhea, and cold intolerance are manifestations of anorexia nervosa. Bradycardia, rather than tachycardia, may be present.

While evaluating an external monitor tracing of a woman in active labor whose labor is being induced, the nurse notes that the fetal heart rate (FHR) begins to decelerate at the onset of several contractions and returns to baseline before each contraction ends. The nurse should: a. Change the woman's position. b. Discontinue the oxytocin infusion. c. Insert an internal monitor. d. Document the finding in the client's record.

D The FHR indicates early decelerations, which are not an ominous sign and do not require any intervention. The nurse should simply document these findings.

Steve, 14 years old, mentions that he now has to use deodorant but never had to before. The nurse's response should be based on knowledge that: a. Eccrine sweat glands in the axillae become fully functional during puberty. b. Sebaceous glands become extremely active during puberty. c. New deposits of fatty tissue insulate the body and cause increased sweat production. d. Apocrine sweat glands reach secretory capacity during puberty.

D The apocrine sweat glands, nonfunctional in children, reach secretory capacity during puberty. They secrete a thick substance as a result of emotional stimulation that, when acted on by surface bacteria, becomes highly odoriferous. They are limited in distribution and grow in conjunction with hair follicles in the axillae, genital and anal areas, and other areas. Eccrine sweat glands are present almost everywhere on the skin and become fully functional and respond to emotional and thermal stimulation. Sebaceous glands become extremely active at this time, especially those on the genitals and the "flush" areas of the body, such as face, neck, shoulders, upper back, and chest. This increased activity is important in the development of acne. New deposits of fatty tissue insulate the body and cause increased sweat production, but this is not the etiology of apocrine sweat gland activity.

Anorexia nervosa may best be described as: a. Occurring most frequently in adolescent males. b. Occurring most frequently in adolescents from lower socioeconomic groups. c. Resulting from a posterior pituitary disorder. d. Resulting in severe weight loss in the absence of obvious physical causes.

D The etiology of anorexia remains unclear, but a distinct psychologic component is present. The diagnosis is based primarily on psychologic and behavioral criteria. Anorexia nervosa is observed more commonly in adolescent girls and young women. It does not occur most frequently in adolescents from a lower socioeconomic group. In reality, anorexic adolescents are often from families of means who have high parental expectations for achievement. Anorexia is a psychiatric disorder

The most common cause of death in the adolescent age-group involves: a. Drownings. c. Drug overdoses. b. Firearms. d. Motor vehicles.

D The leading cause of all adolescent deaths in the United States is motor vehicle accidents. Drownings, firearms, and drug overdoses are major concerns in adolescence but do not cause the majority of deaths.

A nurse may be called on to stimulate the fetal scalp: a. As part of fetal scalp blood sampling. b. In response to tocolysis. c. In preparation for fetal oxygen saturation monitoring. d. To elicit an acceleration in the fetal heart rate (FHR).

D The scalp can be stimulated using digital pressure during a vaginal examination. Fetal scalp blood sampling involves swabbing the scalp with disinfectant before a sample is collected. The nurse would stimulate the fetal scalp to elicit an acceleration of the FHR. Tocolysis is relaxation of the uterus. Fetal oxygen saturation monitoring involves the insertion of a sensor.

An adolescent boy tells the nurse that he has recently had homosexual feelings. The nurse's response should be based on knowledge that: a. This indicates that the adolescent is homosexual. b. This indicates that the adolescent will become homosexual as an adult. c. The adolescent should be referred for psychotherapy. d. The adolescent should be encouraged to share his feelings and experiences.

D These adolescents are at increased risk for health-damaging behaviors, not because of the sexual behavior itself, but because of society's reaction to the behavior. The nurse's first priority is to give the young man permission to discuss his feelings about this topic, knowing that the nurse will maintain confidentially, appreciate his feelings, and remain sensitive to his need to talk it. In recent studies among self-identified gay, lesbian, and bisexual adolescents, many of the adolescents report changing their self-labels one or more times during their adolescence.

In terms of fine motor development, what could the 3-year-old child be expected to do? a. Tie shoelaces. b. Use scissors or a pencil very well. c. Draw a person with seven to nine parts. d. Copy (draw) a circle.

D Three-year-olds are able to accomplish the fine motor skill of drawing a circle. Tying shoelaces, using scissors or a pencil very well, and drawing a person with multiple parts are fine motor skills of 5-year-old children.

Which accomplishment would the nurse expect of a healthy 3-year-old child? a. Jump rope b. Ride a two-wheel bicycle c. Skip on alternate feet d. Balance on one foot for a few seconds

D Three-year-olds are able to accomplish the gross motor skill of balancing on one foot. Jumping rope, riding a two-wheel bike, and skipping on alternate feet are gross motor skills of 5-year-old children.

You are evaluating the fetal monitor tracing of your client, who is in active labor. Suddenly you see the fetal heart rate (FHR) drop from its baseline of 125 down to 80. You reposition the mother, provide oxygen, increase intravenous (IV) fluid, and perform a vaginal examination. The cervix has not changed. Five minutes have passed, and the fetal heart rate remains in the 80s. What additional nursing measures should you take? a. Scream for help. b. Insert a Foley catheter. c. Start Pitocin. d. Notify the care provider immediately.

D To relieve an FHR deceleration, the nurse can reposition the mother, increase IV fluid, and provide oxygen. If oxytocin is infusing, it should be discontinued. If the FHR does not resolve, the primary care provider should be notified immediately. Inserting a Foley catheter is an inappropriate nursing action. If the FHR were to continue in a nonreassuring pattern, a cesarean section could be warranted, which would require a Foley catheter. However, the physician must make that determination. Pitocin may place additional stress on the fetus.

Which medication may be given to high risk children after exposure to chickenpox to prevent varicella? a. Acyclovir b. Vitamin A c. Diphenhydramine hydrochloride d. Varicella zoster immune globulin (VZIG)

D VZIG is given to high risk children to help prevent the development of chickenpox. Immune globulin intravenous may also be recommended. Acyclovir is given to immunocompromised children to reduce the severity of symptoms. Vitamin A reduces morbidity and mortality associated with the measles. The antihistamine diphenhydramine is administered to reduce the itching associated with chickenpox.

The weight loss of anorexia nervosa is often triggered by: a. Sexual abuse. c. Independence from family. b. School failure. d. Traumatic interpersonal conflict.

D Weight loss may be triggered by a typical adolescent crisis such as the onset of menstruation or a traumatic interpersonal incident; situations of severe family stress such as parental separation or divorce; or circumstances in which the young person lacks personal control, such as being teased, changing schools, or entering college. There may in fact be a history of sexual abuse; however, this is not the trigger. These adolescents are often overachievers who are successful in school, not failures in school. The adolescent is most often enmeshed with his or her family.

A pregnant woman at 28 weeks of gestation has been diagnosed with gestational diabetes. The nurse caring for this client understands that: A. oral hypoglycemic agents can be used if the woman is reluctant to give herself insulin. B. dietary modifications and insulin are both required for adequate treatment. C. glucose levels are monitored by testing urine 4r times a day and at bedtime. D. dietary management involves distributing nutrient requirements over three meals and two or three snacks.

D. dietary management involves distributing nutrient requirements over three meals and two or three snacks. Oral hypoglycemic agents can be harmful to the fetus and less effective than insulin in achieving tight glucose control. In some women gestational diabetes can be controlled with dietary modifications alone. Blood, not urine, glucose levels are monitored several times a day. Urine is tested for ketone content; results should be negative. Small frequent meals over a 24-hour period help decrease the risk for hypoglycemia and ketoacidosis.

A (transfer objects from one hand to the other By age 7 months, infants can transfer objects from one hand to the other, crossing the midline. The crude pincer grasp is apparent at about age 9 months. The infant can scribble spontaneously at age 15 months. At age 12 months, the infant can release cubes into a cup.)

In terms of fine motor development, the infant of 7 months should be able to: a. Transfer objects from one hand to the other. b. Use thumb and index finger in a crude pincer grasp. c. Hold a crayon and make a mark on paper. d. Release cubes into a cup.

A, B (Rolling from abdomen to back and placing the feet in the mouth when supine are developmentally appropriate for a 5-month-old infant. Rolling from back to abdomen is developmentally appropriate for a 6-month-old infant. An 8-month-old infant should be able to sit erect without support. A 10-month-old infant can usually move from a prone to a sitting position.)

In terms of gross motor development, what would the nurse expect a 5-month-old infant to do (select all that apply)? a. Roll from abdomen to back. b. Put feet in mouth when supine. c. Roll from back to abdomen. d. Sit erect without support. e. Move from prone to sitting position.

A (roll from abdomen to back Rolling from abdomen to back is developmentally appropriate for a 5-month-old infant. The ability to roll from back to abdomen usually occurs at 6 months old. Sitting erect without support is a developmental milestone usually achieved by 8 months. The 10-month-old infant can usually move from a prone to a sitting position.)

In terms of gross motor development, what would the nurse expect a 5-month-old infant to do? a. Roll from abdomen to back. b. Roll from back to abdomen. c. sit erect without support d. Move from prone to sitting position.

C (This is a good parent-child interaction. The 18-month-old is capable of building a tower of 3 or 4 blocks. The ability to build towers of blocks usually begins at age 15 months. With ongoing development, the child is able to build taller towers. At this age, children imitate others around them and no longer throw blocks.)

In the clinic waiting room, a nurse observes a parent showing an 18-month-old child how to make a tower out of blocks. In this situation the nurse should recognize that: a. Blocks at this age are used primarily for throwing. b. Toddlers are too young to imitate the behavior of others. c. Toddlers are capable of building a tower of blocks. d. Toddlers are too young to build a tower of blocks.

37. Identify a goal of a patient with the following nursing diagnosis: Imbalanced Nutrition: Less Than Body Requirements related to diet choices inadequate to meet nutrient requirements of pregnancy. a. Gain a total of 30 lb. b. Take daily supplements consistently. c. Decrease intake of snack foods. d. Increase intake of complex carbohydrates.

a. Gain a total of 30 lb.

C (reassure the mother that this is very normal at this age Sucking is an infant's chief pleasure, and she may not be satisfied by bottle-feeding or breastfeeding alone. During infancy and early childhood, there is no need to restrict nonnutritive sucking. Dental damage does not appear to occur unless the use of the pacifier or finger persists after age 4 to 6 years. The nurse should explore with the mother her feelings about pacifier vs. thumb. This is a normal behavior to meet nonnutritive sucking needs. No data support that Latasha has sensory deprivation.)

Latasha is a breastfed infant being seen in the clinic for her 6-month checkup. Her mother tells the nurse that Latasha recently began to suck her thumb. The best nursing intervention is to: a. Recommend that the mother substitute a pacifier for Latasha's thumb. b. Assess Latasha for other signs of sensory deprivation. c. Reassure the mother that this is very normal at this age. d. Suggest that the mother breastfeed Latasha more often to satisfy sucking needs.

A, C, E (Toddlers younger than 2 years should be secured in a rear-facing, upright, approved car safety seat. After the age of 2 years, a forward-facing car seat can be used. Harness straps should be adjusted to provide a snug fit. The car safety seat should be placed in the middle of the rear seat. Children younger than 13 years should not ride in a front passenger seat that is equipped with an air bag.)

Motor vehicle injuries are a significant threat to young children. Knowing this, the nurse plans a teaching session with a toddler's parents on car safety. Which will she teach (select all that apply)? a. Secure in a rear-facing, upright, car safety seat. b. Place the car safety seat in the rear seat, behind the driver's seat. c. Harness safety straps should be fit snugly. d. Place the car safety seat in the front passenger seat equipped with an air bag. e. After the age of 2 years, toddlers can be placed in a forward-facing car seat.

Which action best facilitates lipreading by the hearing-impaired child? a. Speaking at an even rate b. Exaggerating pronunciation of words c. Avoiding using facial expressions d. Repeating in exactly the same way if child does not understand

a. Speaking at an even rate

A, B, E (A good pacifier should be easily grasped by the infant. One-piece construction is necessary to avoid having the nipple and guard separate. The material should be sturdy and flexible. An attached ribbon or string and soft, pliable material are not characteristics of a good pacifier.)

Pacifiers can be extremely dangerous because of the frequency of use and the intensity of the infant's suck. In teaching parents about appropriate pacifier selection, the nurse should explain that a pacifier should have which characteristics (select all that apply)? a. Easily grasped handle b. One-piece construction c. Ribbon or string to secure to clothing d. Soft, pliable material e. Sturdy, flexible material

D (This is a common and accepted practice, especially in some cultural groups. Co-sleeping or sharing the family bed, in which the parents allow the children to sleep with them, is a common and accepted practice in many cultures. Parents should evaluate the options available and avoid conditions that place the infant at risk. Population-based studies are currently underway; no evidence at this time supports or abandons the practice for safety reasons. This is the age at which children are just beginning to individuate. Increased daytime activity may help decrease sleep problems in general, but co-sleeping is a culturally determined phenomenon.)

Parents tell the nurse that their 1-year-old son often sleeps with them. They seem unconcerned about this. The nurse's response should be based on the knowledge that: a. Children should not sleep with their parents. b. Separation from parents should be completed by this age. c. Daytime attention should be increased. d. This is a common and accepted practice, especially in some cultural groups.

A (Most toddlers exhibit a physiologic anorexia in response to the decreased nutritional requirement associated with the slower growth rate. Parents should assist the child to develop healthy eating habits. The toddler is often unable to sit through a meal. Frequent nutritious snacks are a good way to ensure proper nutrition. To help with developing healthy eating habits, food should not be used as positive or negative reinforcement for behavior. The child may develop habits of overeating or eat nonnutritious foods in response.)

Parents tell the nurse that their toddler daughter eats little at mealtimes, only sits at the table with the family briefly, and wants snacks "all the time." The nurse should recommend that the parents: a. Give her planned, frequent, and nutritious snacks. b. Offer rewards for eating at mealtimes. c. Avoid snacks so she is hungry at mealtimes. d. Explain to her in a firm manner what is expected of her.

A (infants' temperaments are part of their unique characteristics Infant temperament has a strong biologic component. Together with interactions with the environment, primarily the family, the biologic component contributes to the infant's unique temperament. Children perceived as difficult may respond better to scheduled feedings and structured caregiving routines than to demand feedings and frequent changes in routines. Sara's temperament has been created by both biologic and environmental factors. The nurse should provide guidance in parenting techniques that are best suited to Sara's temperament.)

Sara, age 4 months, was born at 35 weeks' gestation. She seems to be developing normally, but her parents are concerned because she is a "more difficult" baby than their other child, who was term. The nurse should explain that: a. Infants' temperaments are part of their unique characteristics. b. Infants become less difficult if they are not kept on scheduled feedings and structured routines. c. Sara's behavior is suggestive of failure to bond completely with her parents. d. Sara's difficult temperament is the result of painful experiences in the neonatal period.

A (Allow to splash in bath The feel of the water while the infant is splashing provides tactile stimulation. Various colored blocks provide visual stimulation for a 4- to 6-month-old infant. A music box, tapes, and CDs provide auditory stimulation. Swings and strollers provide kinesthetic stimulation.)

The best play activity to provide tactile stimulation for a 6-month-old infant is to: a. Allow to splash in bath. b. Give various colored blocks. c. play music box, tapes, or CDs d. Use infant swing or stroller.

B (Rear facing in back seat The rear-facing car seat provides the best protection for an infant's disproportionately heavy head and weak neck. Infants should face the rear from birth to 20 pounds and as close to 1 year of age as possible. The middle of the back seat provides the safest position. Severe injuries and deaths in children have occurred from air bags deploying on impact in the front passenger seat.)

The clinic is lending a federally approved car seat to an infant's family. The nurse should explain that the safest place to put the car seat is: a. Front facing in back seat. b. Rear facing in back seat. c. Front facing in front seat if an air bag is on the passenger side. d. Rear facing in front seat if an air bag is on the passenger side.

B (For young children, the most effective cleaning of teeth is done by the parents. Different positions can be used if the child's back is to the adult. The adult should use one hand to stabilize the chin and the other to brush the child's teeth. The child can participate in brushing, but for a thorough cleaning adult intervention is necessary.)

The most effective way to clean a toddler's teeth is for the: a. Child to brush regularly with toothpaste of his or her choice. b. Parent to stabilize the chin with one hand and brush with the other. c. Parent to brush the mandibular occlusive surfaces, leaving the rest for the child. d. Parent to brush the front labial surfaces, leaving the rest for the child.

A (Flame burns from matches and lighters represent one of the most fatal types of burns in the toddler age-group. Scald burns from water, hot object burns from cigarettes or irons, and electric burns from outlets are all significant causes of burn injury. The child should be protected from these causes by reducing the temperature of the hot water in the home, keeping objects such as cigarettes and irons away from children, and placing protective guards over electrical outlets when not in use.)

The most fatal type of burn in the toddler age-group is: a. Flame burn from playing with matches. b. Scald burn from high-temperature tap water. c. Hot object burn from cigarettes or irons. d. Electric burn from electrical outlets.

B (Toddlers need small, frequent meals. Nutritious selection throughout the day, rather than quantity, is more important with this age-group. Physiologically, growth slows and appetite decreases during the toddler period. Milk consumption should not exceed 16 to 24 oz daily. Juice should be limited to 4 to 6 oz per day. Increasing the amount of milk will only further decrease solid food intake. Supplemental vitamins are important for all children, but they do not increase appetite.)

The mother of a 14-month-old child is concerned because the child's appetite has decreased. The best response for the nurse to make to the mother is: a. "It is important for your toddler to eat three meals a day and nothing in between." b. "It is not unusual for toddlers to eat less." c. "Be sure to increase your child's milk consumption, which will improve nutrition." d. "Giving your child a multivitamin supplement daily will increase your toddler's appetite."

A (fluids in addition to breast milk are not needed.)

The mother of a 3-month-old breastfed infant asks about giving her baby water since it is summer and very warm. The nurse should recommend that: A. fluids in addition to breast milk are not needed. B. water should be given if the infant seems to nurse longer than usual. C. water once or twice a day will make up for losses caused by environmental temperature. D. clear juices would be better than water to promote adequate fluid intake.

B (15 pounds Birth weight doubles at about age 5 to 6 months. At 6 months, an infant who weighed 7 pounds at birth would weigh approximately 15 pounds. Ten pounds is too little; the infant would have gone from the 50th percentile at birth to below the 5th percentile. Twenty pounds or more is too much; the infant would have tripled the birth weight at 6 months.)

The nurse is assessing a 6-month-old healthy infant who weighed 7 pounds at birth. The nurse should expect the infant to now weigh approximately: a. 10 pounds. b. 15 pounds. c. 20 pounds d. 25 pounds.

C (developmental/neurologic evaluation is needed.)

The nurse is assessing a 6-month-old infant who smiles, coos, and has a strong head lag. The nurse should recognize that: A. this assessment is normal. B. the child is probably cognitively impaired. C. developmental/neurologic evaluation is needed. D. the parent needs to work with the infant to stop head lag.

A, B, D, E (Temper tantrums are a common response to anger and frustration in toddlers. They occur more often when toddlers are tired, hungry, bored, or excessively stimulated. A nap prior to fatigue or a snack if mealtime is delayed will be helpful in alleviated the times when tantrums are most likely to occur. Tantrums may include screaming, kicking, throwing things, biting themselves, or banging their head. Effective management of tantrums includes safely isolating and ignoring the child. The child should learn that nothing is gained by having a temper tantrum. Giving in to the child's demands only increases the behavior.)

The nurse is assessing parental knowledge of temper tantrums. Which are true statements regarding temper tantrums (select all that apply)? a. Temper tantrums are a common response to anger and frustration in toddlers. b. Temper tantrums often include screaming, kicking, throwing things, and head banging. c. Parents can effectively manage temper tantrums by giving in to the child's demands. d. Children having temper tantrums should be safely isolated and ignored. e. Parents can learn to anticipate times when tantrums are more likely to occur.

B, E (B. Place iron toward the back side of the mouth with a dropper. E. Apply barrier ointment if needed to buttocks.)

The nurse is providing education to a parent of a 10-month-old infant receiving iron supplements. What will be included in the teaching? (Select all that apply.) A. Administer iron with meals. B. Place iron toward the back side of the mouth with a dropper. C. Mix iron with milk for greater absorption. D. Report black, tarry stools to health care provider. E. Apply barrier ointment if needed to buttocks.

D (encourage the toddler to do things for himself or herself when he or she is capable of doing them.)

The nurse is discussing toddler development with a parent. Which intervention will foster the achievement of autonomy? A. help the toddler complete tasks. B. provide opportunities for the toddler to play with other children. C. help the toddler learn the difference between right and wrong. D. encourage the toddler to do things for himself or herself when he or she is capable of doing them.

D (The decision about fluoride supplementation cannot be made until it is known whether the water supply contains fluoride and the amount. It is difficult to teach this age-group to spit out the mouthwash. Swallowing fluoridated mouthwashes can contribute to fluorosis. Fluoridated toothpaste is still indicated, but very small amounts are used. Fluoride supplementation is not recommended until after age 6 months.)

The nurse is discussing with a parent group the importance of fluoride for healthy teeth. The nurse should recommend that the parents: a. Use fluoridated mouth rinses in children older than 1 year. b. Have children brush teeth with fluoridated toothpaste unless fluoride content of water supply is adequate. c. Give fluoride supplements to breastfed infants beginning at age 1 month. d. Determine whether water supply is fluoridated.

A (a normal finding Because the anterior fontanel normally closes between ages 12 and 18 months, this is a normal finding, and no further intervention is required.)

The nurse is doing a routine assessment on a 14-month-old infant and notes that the anterior fontanel is closed. This should be interpreted as: a. A normal finding. b. A questionable finding—the infant should be rechecked in 1 month. c. An abnormal finding—indicates the need for immediate referral to a practitioner. d. An abnormal finding—indicates the need for developmental assessment.

A (is old enough to understand the word "No." By age 10 months, children are able to associate meaning with words. The child should be old enough to understand the word "No." The 10-month-old is too young to understand the purpose of an electrical outlet. The father is using both verbal and physical cues to teach safety measures and alert the child to dangerous situations. Physical discipline should be avoided.)

The nurse is interviewing the father of 10-month-old Megan. She is playing on the floor when she notices an electrical outlet and reaches up to touch it. Her father says "No" firmly and removes her from near the outlet. The nurse should use this opportunity to teach the father that Megan: a. Is old enough to understand the word "No." b. Is too young to understand the word "No." c. Should already know that electrical outlets are dangerous. d. Will learn safety issues better if she is spanked.

D (sudden infant death syndrome (SIDS).)

The nurse is interviewing the parents of a 4-month-old male infant brought to the hospital emergency department. The infant is dead on arrival, and no attempt at resuscitation is made. The parents state that the baby was found in his crib with a blanket over his head, lying face down in bloody fluid from his nose and mouth. They say he was "just fine" when they put him in his crib already asleep. The nurse should suspect his death was caused by: A. suffocation. B. child abuse. C. infantile apnea. D. sudden infant death syndrome (SIDS).

D (The 17-month-old is in the fifth stage of the sensorimotor phase: tertiary circular reactions. The child uses active experimentation to achieve previously unattainable goals. Trust is Erikson's first stage. Preoperations is the stage of cognitive development, usually present in older toddlers and preschoolers. Secondary circular reactions last from about ages 4 to 8 months.)

The nurse is planning care for a 17-month-old child. According to Piaget, in what stage would the nurse expect the child to be? a. Trust b. Preoperations c. Secondary circular reaction d. Tertiary circular reaction

C (Brush teeth with plain water if child does not like toothpaste.)

The nurse is teaching the parent of a 2-year-old child how to care for the child's teeth. Which of the following should be included? A. Flossing is not recommended at this age. B. The child is old enough to brush teeth effectively. C. Brush teeth with plain water if child does not like toothpaste. D. Toothbrush should be small and have hard, rounded, nylon bristles.

B (The toddler's activities begin to produce purposeful results.)

The nurse is teaching the parents of a 24-month-old about motor skill development. The nurse should include which statement in the teaching? A. The toddler walks alone but falls easily. B. The toddler's activities begin to produce purposeful results. C. The toddler is able to grasp small objects but cannot release them at will. D. The toddler's motor skills are fully developed but occur in isolation from the environment.

A (soft and flexible shoes are generally better The main purpose of the shoe is protection. Soft, well-constructed, athletic-type shoes are best for infants and children. High-top shoes are not necessary for support but may be helpful keeping the child's foot in the shoe. Inflexible shoes can delay walking, aggravate in-toeing and out-toeing, and impede development of the supportive foot muscles.)

The nurse notices that a 10-month-old infant being seen in the clinic is wearing expensive, inflexible, high-top shoes. The nurse should explain that: a. Soft and flexible shoes are generally better. b. High-top shoes are necessary for support. c. Inflexible shoes are necessary to prevent in-toeing and out-toeing. d. This type of shoe will encourage the infant to walk sooner.

D (Foreign-body aspiration is common during the second year of life. Although they chew well, children at this age may have difficulty with large pieces of food such as meat and whole hot dogs and with hard foods such as nuts or dried beans. Peanuts have many beneficial nutrients but should be avoided because of the risk of aspiration in this age-group. The sodium level may be a concern, but the risk of aspiration is more important. Many foods pass through the gastrointestinal tract incompletely digested. This is not necessarily detrimental to the child.)

The nurse recommends to parents that peanuts are not a good snack food for toddlers. The nurse's rationale for this action is that: a. They are low in nutritive value. b. They are very high in sodium. c. They cannot be entirely digested. d. They can be easily aspirated.

A, C, D, E (A. "I only smoke in the kitchen." C. "I have my baby sleep with me instead of alone in the crib." D. "I make sure my baby wears a flannel sleeper and has two blankets to keep warm in her crib." E. "I always leave my baby's favorite stuffed bunny rabbit in the crib to keep her from crying at night.")

The nurse should provide further teaching about sudden infant death syndrome (SIDS) prevention when hearing the mother of an 8-week-old make which statement? (Select all that apply.) A. "I only smoke in the kitchen." B. "I put my baby to sleep on her back." C. "I have my baby sleep with me instead of alone in the crib." D. "I make sure my baby wears a flannel sleeper and has two blankets to keep warm in her crib." E. "I always leave my baby's favorite stuffed bunny rabbit in the crib to keep her from crying at night."

A (The major reason for poisoning in the home is improper storage. Toddlers can climb, unlatch cabinets, and obtain access to high-security places. For medications, only a locked cabinet is safe. Toddlers can climb by using furniture. High places are not a deterrent to an exploring toddler. Toddlers are not able to generalize as dangerous all of the different forms of medications that may be available in the home. Teaching them not to touch medicines is not feasible. Many parents require medications for chronic illnesses. They must be taught safe storage for their home and when they visit other homes.)

The parent of 16-month-old Chris asks, "What is the best way to keep Chris from getting into our medicines at home?" The nurse should advise that: a. "All medicines should be locked securely away." b. "The medicines should be placed in high cabinets." c. "Chris just needs to be taught not to touch medicines." d. "Medicines should not be kept in the homes of small children."

C ("She may need to begin taking them at age 6 months." Fluoride supplementation is recommended by the American Academy of Pediatrics beginning at age 6 months if the child is not drinking adequate amounts of fluoridated water. The recommendation is to begin supplementation at 6 months, not at 2 weeks. The amount of water that is ingested and the amount of fluoride in the water are evaluated when supplementation is being considered.)

The parent of 2-week-old Sarah asks the nurse if Sarah needs fluoride supplements because she is exclusively breastfed. The nurse's best response is: a. "She needs to begin taking them now." b. "They are not needed if you drink fluoridated water." c. "She may need to begin taking them at age 6 months." d. "She can have infant cereal mixed with fluoridated water instead of supplements."

D ("Feeding himself will help foster his growth and development. Perhaps we can discuss ways to make the messes more tolerable.")

The parent of a 12-month-old infant says to the nurse, "He pushes the teaspoon right out of my hand when I feed him. I can't let him feed himself; he makes too much of a mess." The nurse's BEST response is: A. "It's important not to give in to this kind of temper tantrum at this age. Simply ignore the behavior and the mess." B. "You need to try different types of utensils, bowls, and plates. Some are specifically designed for young children." C. "It's important to let him make a mess. Just try not to worry about it so much." D. "Feeding himself will help foster his growth and development. Perhaps we can discuss ways to make the messes more tolerable."

D (Hot dogs must be cut into small, irregular pieces to prevent aspiration. Hot dogs are of a consistency, diameter, and round shape that may cause complete obstruction of the child's airway. If given to young children, the hot dog should be cut into small irregular pieces rather than served whole or in slices. The child's digestive system is mature enough to digest hot dogs. To eat the hot dog safely, the child should be sitting down, and the hot dog should be appropriately cut into irregularly shaped pieces.)

The parents of a 12-month-old child ask the nurse if the child can eat hot dogs. The nurse's reply should be based on knowing that: a. The child is too young to digest hot dogs. b. The child is too young to eat hot dogs safely. c. Hot dogs must be sliced into sections to prevent aspiration. d. Hot dogs must be cut into small, irregular pieces to prevent aspiration.

A (The baby talk is a sign of regression in the toddler. It should be ignored, while praising the child for developmentally appropriate behaviors. Regression is children's way of saying that they are expressing stress. The parents should not introduce new expectations and should allow the child to master the developmental tasks without criticism.)

The parents of a 2-year-old tell the nurse that they are concerned because the toddler has started to use "baby talk" since the arrival of their new baby. The nurse should recommend that the parents: a. Ignore the "baby talk." b. Explain to the toddler that "baby talk" is for babies. c. Tell the toddler frequently, "You are a big kid now." d. Encourage the toddler to practice more advanced patterns of speech.

D (Acceptable to encourage head control and turning over. These parents are implementing the guidelines to reduce the risk of SIDS. Infants should sleep on their backs and then be placed on their abdomens when awake to enhance development of milestones such as head control. The face-down position while awake and positioning on the back for sleep are acceptable because they reduce risk of SIDS and allow achievement of developmental milestones. These position changes encourage gross motor, not fine motor, development.)

The parents of a 3-month-old infant report that their infant sleeps supine (face up) but is often prone (face down) while awake. The nurse's response should be based on knowledge that this is: a. Unacceptable because of the risk of sudden infant death syndrome (SIDS). b. Unacceptable because it does not encourage achievement of developmental milestones. c. Unacceptable to encourage fine motor development. d. Acceptable to encourage head control and turning over.

A (Never heating a bottle in a microwave oven. Neither infant formula nor breast milk should be warmed in a microwave oven as this may cause oral burns as a result of uneven heating in the container. The bottle may remain cool while hot spots develop in the milk. Warming expressed milk in a microwave decreases the availability of antiinfective properties and causes separation of the fat content. Milk should be warmed in a lukewarm water bath.)

The parents of a 4-month-old infant tell the nurse that they are getting a microwave oven and will be able to heat the baby's formula faster. The nurse should recommend: a. Never heating a bottle in a microwave oven. b. Heating only 10 ounces or more. c. Always leaving the bottle top uncovered to allow heat to escape. d. Shaking the bottle vigorously for at least 30 seconds after heating.

B (Beginning to put her to bed while still awake)

The parents of a 5-month-old girl complain to the nurse that they are exhausted because she still wakes up as often as every 1 to 2 hours during the night. When she awakens, they change her diaper, and her mother nurses her back to sleep. What should the nurse suggest to help them deal with this problem? A. Putting her in parents' bed to cuddle B. Beginning to put her to bed while still awake C. Letting her cry herself back to sleep D. Giving her a bottle of formula instead of breastfeeding her so often at night

D (There is no need to restrain nonnutritive sucking during infancy.)

The parents of a 9-month-old infant tell the nurse that they are worried about their baby's thumb-sucking. What is the nurse's BEST reply? A. A pacifier should be substituted for the thumb. B. Thumb-sucking should be discouraged by age 12 months. C. Thumb-sucking should be discouraged when the teeth begin to erupt. D. There is no need to restrain nonnutritive sucking during infancy.

D (this is normal because of the immaturity of digestive processes at this age. The immaturity of the digestive tract is evident in the appearance of the stools. Solid foods are passed incompletely broken down in the feces. An excess quantity of fiber predisposes the child to large, bulky stools. This is a normal part of the maturational process, and no further investigation is necessary.)

The parents of a 9-month-old infant tell the nurse that they have noticed foods such as peas and corn are not completely digested and can be seen in their infant's stool. The nurse bases her explanation on knowing that: a. Children should not be given fibrous foods until the digestive tract matures at age 4 years. b. The infant should not be given any solid foods until this digestive problem is resolved. c. This is abnormal and requires further investigation. d. This is normal because of the immaturity of digestive processes at this age.

D (The arrival of a new infant represents a crisis for even the best-prepared toddler. Toddlers have their entire schedule and routines disrupted because of the new family member. The nurse should work with parents on ways to involve the toddler in the newborn's care and help focus attention on the toddler. The toddler does not hate the infant. This is an expected response to the changes in routines and attention that affect the toddler. This is a normal response. The toddler can be provided with a doll to tend to its needs when the parent is performing similar care for the newborn.)

The parents of a newborn say that their toddler "hates the baby . . . he suggested that we put him in the trash can so the trash truck could take him away." The nurse's best reply is: a. "Let's see if we can figure out why he hates the new baby." b. "That's a strong statement to come from such a small boy." c. "Let's refer him to counseling to work this hatred out. It's not a normal response." d. "That is a normal response to the birth of a sibling. Let's look at ways to deal with this."

C (Select an area that is safe and nonstimulating, such as a hallway.)

The parents of a toddler ask the nurse for suggestions about discipline. When discussing the use of timeouts, which of the following suggestions should the nurse include? A. Send the child to his or her room. B. If the child cries, refuses, or is more disruptive, try another approach. C. Select an area that is safe and nonstimulating, such as a hallway. D. The general rule for length of time is 1 hour per year of age.

B, C, D, E (B. Suggest that parents ignore the behavior as long as child is not harming self. C. Encourage the parents to provide comfort once the child has calmed down. D. Ask parents to praise the child for positive behavior when not having a tantrum. E. Tell parents not to give in to the original request that started the temper tantrum.)

The parents of a toddler ask the nurse how to handle their child's increasing number of temper tantrums. The nurse should include which positive reinforcement methods of reducing the number of tantrums? (Select all that apply.) A. Suggest that parents provide the child an "all or none" position. B. Suggest that parents ignore the behavior as long as child is not harming self. C. Encourage the parents to provide comfort once the child has calmed down. D. Ask parents to praise the child for positive behavior when not having a tantrum. E. Tell parents not to give in to the original request that started the temper tantrum.

D ("Becoming a fussy eater is expected during the toddler years.")

The parents of a toddler express frustration to the nurse because their child is a "fussy eater." The nurse's BEST response is: A. "You should provide larger servings of different foods. B. "Provide more bland food varieties as toddlers have few food preferences." C. "Table manners will improve if you provide finger foods." D. "Becoming a fussy eater is expected during the toddler years."

A (Using a transitional object.)

The parents of a toddler state their child is having trouble sleeping. What is the nurse's BEST suggestion to improve sleep habits? A. Using a transitional object. B. Varying the bedtime ritual. C. Restricting stimulating activities during the day. D. Explaining away fears.

D (If the need for basic trust has been satisfied, toddlers can give up dependence for control, independence, and autonomy. One of the tasks that the toddler is concerned with is the ability to withstand delayed gratification. Development of a conscience occurs during the preschool years. The recognition of sex differences occurs during the preschool years. The ability to get along with age mates develops during the preschool and school-age years.)

The psychosocial developmental tasks of toddlerhood include: a. Development of a conscience. b. Recognition of sex differences. c. Ability to get along with age mates. d. Ability to withstand delayed gratification.

B (At this age the child is in the final sensorimotor stage. Children will now search for an object in several potential places, even though they saw only the original hiding place. Children have a more developed sense of objective permanence. They will search for objects even if they have not seen them hidden. Putting an object in a container but being unable to take it out indicates tertiary circular reactions. An embryonic sense of time exists; although toddlers may behave appropriately to time-oriented phrases, their sense of timing is exaggerated.)

What describes a toddler's cognitive development at age 20 months? a. Searches for an object only if he or she sees it being hidden b. Realizes that "out of sight" is not out of reach c. Puts objects into a container but cannot take them out d. Understands the passage of time such as "just a minute" and "in an hour"

D (The preschooler has a great imagination. Sounds and shadows can have a negative effect on sleeping behavior. Night-lights provide the child with the ability to visualize the environment and decrease the fear felt in a dark room. A dark, quiet room may be scary to a preschooler. High-carbohydrate snacks increase energy and do not promote relaxation. Most 2-year-olds take one nap each day. Many give up the habit by age 3. Insufficient rest during the day can lead to irritability and difficulty sleeping at night.)

What is helpful to tell a mother who is concerned about preventing sleep problems in her 2-year-old child? a. Have the child always sleep in a quiet, darkened room. b. Provide high-carbohydrate snacks before bedtime. c. Communicate with the child's daytime caregiver about eliminating the afternoon nap. d. Use a night-light in the child's room.

D (placing the car seat in the back seat of the car facing forward.)

When explaining the proper restraint of toddlers in motor vehicles to a group of parents, the nurse should include: A. wearing safety belts snugly over the toddler's abdomen. B. placing the car seat in the front passenger seat if there is an airbag. C. using lap and shoulder belts when the child is over 3 years of age. D. placing the car seat in the back seat of the car facing forward.

C (Decreasing anxiety, particularly separation anxiety, is the function of a transitional object; it provides comfort to the toddler in stressful situations and helps make the transition from dependence to autonomy. A decrease in parental guilt (distress) is an indirect benefit of a transitional object. A transitional object may be part of a bedtime ritual, but it may not keep the child quiet at bedtime. A transitional object does not significantly affect negativity and tantrums, but it can comfort a child after tantrums.)

What is the primary purpose of a transitional object? a. It helps the parents deal with the guilt they feel when they leave the child. b. It keeps the child quiet at bedtime. c. It is effective in decreasing anxiety in the toddler. d. It decreases negativism and tantrums in the toddler.

A (Give the toddler a doll on which he or she can imitate parenting.)

What should the nurse recommend to help a toddler cope with the birth of a new sibling? A. Give the toddler a doll on which he or she can imitate parenting. B. Discourage the toddler from helping with care of new sibling. C. Prepare the toddler for upcoming changes about 1 to 2 weeks before birth of the sibling. D. Explain to the toddler that a new playmate will soon come home.

A (engage in parallel play.)

When completing the health assessment for a 2-year-old child, the nurse should expect the child to: A. engage in parallel play. B. fully dress self with supervision. C. have a vocabulary of at least 500 words. D. be one third of the adult height.

B (4 to 6 months Physiologically and developmentally, the 4- to 6-month-old is in a transition period. The extrusion reflex has disappeared, and swallowing is a more coordinated process. In addition, the gastrointestinal tract has matured sufficiently to handle more complex nutrients and is less sensitive to potentially allergenic food. Infants of this age will try to help during feeding. Two to 3 months is too young. The extrusion reflex is strong, and the infant will push food out with the tongue. No research base indicates that the addition of solid food to bottle-feeding has any benefit. Tooth eruption can facilitate biting and chewing; most infant foods do not require this ability.)

When is the best age for solid food to be introduced into the infant's diet? a. 2 to 3 months b. 4 to 6 months c. when birth weight has tripled d. When tooth eruption has started

A, B, C, E (A. initiate an immunization record. B. confirm the hepatitis B status of the newborn's mother. C. obtain a syringe with a 25-gauge, 5/8-inch needle. E. confirm that the newborn's mother has signed the informed consent.)

When preparing to administer Hepatitis B vaccine to a newborn, the nurse should: (Select all that apply.) A. initiate an immunization record. B. confirm the hepatitis B status of the newborn's mother. C. obtain a syringe with a 25-gauge, 5/8-inch needle. D. assess the dorsogluteal muscle as the preferred site for injection. E. confirm that the newborn's mother has signed the informed consent.

C (actively searches for a hidden object During the first 6 months of life, infants believe that objects exist only as long as they can see them. When infants search for an object that is out of sight, this signals the attainment of object permanence, whereby an infant knows that an object exists even when it is not visible. Between ages 8 and 12 weeks, infants begin to respond differentially to their mothers. They cry, smile, vocalize, and show distinct preference for their mothers. This preference is one of the stages that influence the attachment process, but it is too early for object permanence. Recognizing familiar objects is an important transition for the infant, but it does not signal object permanence. The ability to understand cause and effect, such as pulling on a string to secure an object, is part of secondary schema development.)

Which behavior indicates that an infant has developed object permanence? a. Recognizes familiar face such as the mother b. Recognizes familiar object such as a bottle c. Actively searches for a hidden object d. Secures objects by pulling on a string

C (Children should first see the dentist 6 months after the first primary tooth erupts and no later than age 30 months. Toddlers need fluoride supplements when they use a water supply that is not fluorinated. Toddlers also require supervision with dental care. The parent should finish brushing areas not reached by the child. A small nylon bristle brush works best for cleaning toddlers' teeth.)

Which comments indicate that the mother of a toddler needs further teaching about dental care? a. "We use well water so I give my toddler fluoride supplements." b. "My toddler brushes his teeth with my help." c. "My child will not need a dental checkup until his permanent teeth come in." d. "I use a small nylon bristle brush for my toddler's teeth."

D (Toddlers continue to have the short, straight internal ear canal of infants. The lymphoid tissue of the tonsils and adenoids continues to be relatively large. These two anatomic conditions combine to predispose the toddler to frequent infections. The abdominal respirations and lowered pulse and respiratory rate of toddlers do not affect their susceptibility to infection. The defense mechanisms are more efficient compared with those of infancy.)

Which factor is most important in predisposing toddlers to frequent infections such as otitis media, tonsillitis, and upper respiratory tract infections? a. Respirations are abdominal. b. Pulse and respiratory rates are slower than those in infancy. c. Defense mechanisms are less efficient than those during infancy. d. Short, straight internal ear/throat structures and large tonsil/adenoid lymph tissue are present.

A, C, D (An 18-month-old child can jump in place with both feet, throw a ball overhand without falling, and pull and push toys. Taking a few steps on tiptoe and standing on one foot momentarily are not acquired until 30 months of age.)

Which gross motor milestones should the nurse assess in an 18-month-old child (select all that apply)? a. Jumps in place with both feet b. Takes a few steps on tiptoe c. Throws ball overhand without falling d. Pulls and pushes toys e. Stands on one foot momentarily

A (Give large push-pull toys for kinesthetic stimulation The 12-month-old child is able to pull to a stand and walk holding on or independently. Appropriate toys for a child of this age include large push-pull toys for kinesthetic stimulation. A cradle gym should not be placed across the crib. Finger paints are appropriate for older children. A 12-month-old child does not have the stability to use a stick horse.)

Which information could be given to the parents of a 12-month-old child regarding appropriate play activities for this age? a. Give large push-pull toys for kinesthetic stimulation. b. Place cradle gym across crib to facilitate fine motor skills. c. Provide child with finger paints to enhance fine motor skills. d. Provide stick horse to develop gross motor coordination

A, C, E (An important aspect of compassionate care for parents experiencing a SIDS incident is allowing them to say good-bye to their infant. These are the parents' last moments with their infant, and they should be as quiet, meaningful, peaceful, and undisturbed as possible. Because the parents leave the hospital without their infant, it is helpful to accompany them to the car or arrange for someone else to take them home. A debriefing session may help health care workers who dealt with the family and deceased infant to cope with emotions that are often engendered when a SIDS victim is brought into the acute care facility. An autopsy may clear up possible misconceptions regarding the death. When the parents return home, a competent, qualified professional should visit them after the death as soon as possible.)

Which interventions should the nurse implement when caring for a family of a sudden infant death syndrome (SIDS) infant (select all that apply)? a. Allow parents to say goodbye to their infant. b. Once parents leave the hospital, no further follow-up is required. c. Arrange for someone to take the parents home from the hospital. d. Avoid requesting an autopsy of the deceased infant. e. Conduct a debriefing session with the parents before they leave the hospital.

A (Establish a structured routine and follow it consistently. The infant with failure to thrive should have a structured routine that is followed consistently. Disruptions in other activities of daily living can have a great impact on feeding behaviors. Bathing, sleeping, dressing, playing, and feeding are structured. The nurse should talk to the infant by giving directions about eating. This will help the infant maintain focus. Young children should be held while being fed, and older children can sit at a feeding table. The infant should be fed in the same manner at each meal. The infant can engage in sensory and play activities at times other than mealtime.)

Which is an important nursing consideration when caring for an infant with failure to thrive? a. Establish a structured routine and follow it consistently. b. Maintain a nondistracting environment by not speaking to the infant during feeding. c. Place the infant in an infant seat during feedings to prevent overstimulation. d. Limit sensory stimulation and play activities to alleviate fatigue.

D (Gently stimulate trunk by patting or rubbing. If the infant is apneic, the infant's trunk should be gently stimulated by patting or rubbing. If the infant is prone, turn onto the back. The infant should not be shaken vigorously, have the head rolled side to side, or be held by the feet upside down with the head supported. These actions can cause injury.)

Which is the most appropriate action when an infant becomes apneic? a. Shake vigorously. b. Roll head side to side. c. Hold by feet upside down with head supported. d. Gently stimulate trunk by patting or rubbing.

B (Large plastic blocks are appropriate for a toddler in isolation. Play objects for toddlers must still be chosen with an awareness of danger from small parts. Large, sturdy toys without sharp edges or removable parts are safest. Small plastic toys such as Legos can cause choking or can be aspirated. Balloons can cause significant harm if swallowed or aspirated. Coloring book and crayons would be too advanced for a toddler.)

Which play item should the nurse bring from the playroom to a hospitalized toddler in isolation? a. Small plastic Legos b. Set of large plastic building blocks c. Brightly colored balloon d. Coloring book and crayons

B (During the second year of life, level of comprehension and understanding of speech increases and is far greater than the child's vocabulary. This is also true for bilingual children, who are able to achieve this linguistic milestone in both languages. The 18-month-old child has a vocabulary of 10 or more words. At this age, the child does not use one-word sentences or phrases. The child has a limited vocabulary of single words that are comprehensible.)

Which should the nurse expect for a toddler's language development at age 18 months? a. Vocabulary of 25 words b. Increasing level of comprehension c. Use of phrases d. Approximately one third of speech understandable

B, C, E (To prevent accidental poisoning, parents should be taught to promptly discard empty poison containers, to know the number of the nearest poison control center, and to caution the child against eating nonedible items, such as plants. Parents should place all potentially toxic agents, including cosmetics, personal care items, cleaning products, pesticides, and medications, in a locked cabinet, not in the garage. Parents should be taught to never remove labels from containers of toxic substances.)

Which should the nurse teach to parents of toddlers about accidental poison prevention (select all that apply)? a. Keep toxic substances in the garage. b. Discard empty poison containers. c. Know the number of the nearest poison control center. d. Remove colorful labels from containers of toxic substances. e. Caution child against eating nonedible items, such as plants.

B (Voluntary control of the anal and urethral sphincters is achieved sometime after the child is walking. The child must be able to recognize the urge to let go and to hold on. The child must want to please the parent by holding on rather than pleasing self by letting go. Bowel training precedes bladder training. Watching older siblings provides role modeling and facilitates imitation for the toddler. The child should be introduced to the potty chair or toilet in a nonthreatening manner.)

Which statement about toilet training is correct? a. Bladder training is usually accomplished before bowel training. b. Wanting to please the parent helps motivate the child to use the toilet. c. Watching older siblings use the toilet confuses the child. d. Children must be forced to sit on the toilet when first learning.

C (birth weight doubles by age 5 months and triples by age 1 year Growth is very rapid during the first year of life. The birth weight approximately doubles by age 5 to 6 months and triples by age 1 year. The anterior fontanel closes at age 12 to 18 months. Binocularity is not established until age 15 months. Maternal iron stores are usually depleted by age 6 months.)

Which statement best describes the infant's physical development? a. Anterior fontanel closes by age 6 to 10 months. b. Binocularity is well established by age 8 months. c. Birth weight doubles by age 5 months and triples by age 1 year. d. Maternal iron stores persist during the first 12 months of life.

The nurse is talking to the parent of a 13-month-old child. The mother states, "My child does not make noises like 'da' or 'na' like my sister's baby, who is only 9 months old." Which statement by the nurse would be most appropriate to make? a. "I am going to request a referral to a hearing specialist." b. "You should not compare your child to your sister's child." c. "I think your child is fine, but we will check again in 3 months." d. "You should ask other parents what noises their children made at this age."

a. "I am going to request a referral to a hearing specialist."

14. After you complete your nutritional counseling for a pregnant woman, you ask her to repeat your instructions so you can assess her understanding of the instructions given. Which statement indicates that she understands the role of protein in her pregnancy? a. "Protein will help my baby grow." b. "Eating protein will prevent me from becoming anemic." c. "Eating protein will make my baby have strong teeth after he is born." d. "Eating protein will prevent me from being diabetic."

a. "Protein will help my baby grow."

19. Which nutritional recommendation about fluids is accurate? a. A woman's daily intake should be eight to ten glasses (2.3 L) of water, milk, or juice. b. Coffee should be limited to no more than two cups, but tea and cocoa can be consumed without worry. c. Of the artificial sweeteners, only aspartame has not been associated with any maternity health concerns. d. Water with fluoride is especially encouraged because it reduces the child's risk of tooth decay.

a. A woman's daily intake should be eight to ten glasses (2.3 L) of water, milk, or juice.

When should children with cognitive impairment be referred for stimulation and educational programs? a. As young as possible b. As soon as they have the ability to communicate in some way c. At age 3 years, when schools are required to provide services d. At age 5 or 6 years, when schools are required to provide services

a. As young as possible

Prevention of hearing impairment in children is a major goal for the nurse. This can be achieved through: a. Being involved in immunization clinics for children. b. Assessing a newborn for hearing loss. c. Answering parents' questions about hearing aids. d. Participating in hearing screening in the community.

a. Being involved in immunization clinics for children.

17. Which statement regarding acronyms in nutrition is accurate? a. Dietary reference intakes (DRIs) consist of recommended dietary allowances (RDAs), adequate intakes (AIs), and upper limits (ULs). b. RDAs are the same as ULs except with better data. c. AIs offer guidelines for avoiding excessive amounts of nutrients. d. They all refer to green leafy vegetables, whole grains, and fruit.

a. Dietary reference intakes (DRIs) consist of recommended dietary allowances (RDAs), adequate intakes (AIs), and upper limits (ULs).

A nurse would suspect possible visual impairment in a child who displays: a. Excessive rubbing of the eyes. b. Rapid lateral movement of the eyes. c. Delay in speech development. d. Lack of interest in casual conversation with peers.

a. Excessive rubbing of the eyes.

35. A pregnant patient would like to know a good food source of calcium other than dairy products. Your best answer is: a. Legumes b. Yellow vegetables c. Lean meat d. Whole grains

a. Legumes

18. With regard to protein in the diet of pregnant women, nurses should be aware that: a. Many protein-rich foods are also good sources of calcium, iron, and B vitamins. b. Many women need to increase their protein intake during pregnancy. c. As with carbohydrates and fat, no specific recommendations exist for the amount of protein in the diet. d. High-protein supplements can be used without risk by women on macrobiotic diets.

a. Many protein-rich foods are also good sources of calcium, iron, and B vitamins.

12. To prevent gastrointestinal upset, clients should be instructed to take iron supplements: a. On a full stomach. b. At bedtime. c. After eating a meal. d. With milk.

b. At bedtime.

27. Three servings of milk, yogurt, or cheese plus two servings of meat, poultry, or fish adequately supply the recommended amount of protein for a pregnant woman. Many patients are concerned about the increased levels of mercury in fish and may be afraid to include this source of nutrients in their diet. Sound advice by the nurse to assist the client in determining which fish is safe to consume would include: a. Canned white tuna is a preferred choice. b. Avoid shark, swordfish, and mackerel. c. Fish caught in local waterways are the safest. d. Salmon and shrimp contain high levels of mercury.

b. Avoid shark, swordfish, and mackerel.

22. With regard to nutritional needs during lactation, a maternity nurse should be aware that: a. The mother's intake of vitamin C, zinc, and protein now can be lower than during pregnancy. b. Caffeine consumed by the mother accumulates in the infant, who may be unusually active and wakeful. c. Critical iron and folic acid levels must be maintained. d. Lactating women can go back to their prepregnant calorie intake.

b. Caffeine consumed by the mother accumulates in the infant, who may be unusually active and wakeful.

38. In teaching the pregnant adolescent about nutrition, the nurse should: a. Emphasize the need to eliminate common teen snack foods because they are too high in fat and sodium. b. Determine the weight gain needed to meet adolescent growth and add 35 lb. c. Suggest that she not eat at fast-food restaurants to avoid foods of poor nutritional value. d. Realize that most adolescents are unwilling to make dietary changes during pregnancy.

b. Determine the weight gain needed to meet adolescent growth and add 35 lb.

5. A pregnant woman experiencing nausea and vomiting should: a. Drink a glass of water with a fat-free carbohydrate before getting out of bed in the morning. b. Eat small, frequent meals (every 2 to 3 hours). c. Increase her intake of high-fat foods to keep the stomach full and coated. d. Limit fluid intake throughout the day.

b. Eat small, frequent meals (every 2 to 3 hours).

33. The most important reason for evaluating the pattern of weight gain in pregnancy is to: a. Prevent excessive adipose tissue deposits b. Identify potential nutritional problems or complications of pregnancy c. Assess the need to limit caloric intake in obese women d. Determine cultural influences on the woman's diet

b. Identify potential nutritional problems or complications of pregnancy

13. Women with an inadequate weight gain during pregnancy are at higher risk of giving birth to an infant with: a. Spina bifida. b. Intrauterine growth restriction. c. Diabetes mellitus. d. Down syndrome.

b. Intrauterine growth restriction

16. Maternal nutritional status is an especially significant factor of the many factors that influence the outcome of pregnancy because: a. It is very difficult to adjust because of people's ingrained eating habits. b. It is an important preventive measure for a variety of problems. c. Women love obsessing about their weight and diets. d. A woman's preconception weight becomes irrelevant.

b. It is an important preventive measure for a variety of problems.

Appropriate interventions to facilitate socialization of the cognitively impaired child include to: a. Provide age-appropriate toys and play activities. b. Provide peer experiences such as Special Olympics when older. c. Avoid exposure to strangers who may not understand cognitive development. d. Emphasize mastery of physical skills because they are delayed more often than verbal skills.

b. Provide peer experiences such as Special Olympics when older.

A nurse is preparing to perform a dressing change on a 6-year-old child with mild cognitive impairment (CI) who sustained a minor burn. Which strategy should the nurse use to prepare the child for this procedure? a. Verbally explain what will be done. b. Have the child watch a video on dressing changes. c. Demonstrate a dressing change on a doll. d. Explain the importance of keeping the burn area clean.

c. Demonstrate a dressing change on a doll.

32. When providing care to the prenatal patient, the nurse understands that pica is defined as: a. Intolerance of milk products b. Iron deficiency anemia c. Ingestion of nonfood substances d. Episodes of anorexia and vomiting

c. Ingestion of nonfood substances

20. Which minerals and vitamins usually are recommended to supplement a pregnant woman's diet? a. Fat-soluble vitamins A and D b. Water-soluble vitamins C and B6 c. Iron and folate d. Calcium and zinc

c. Iron and folate

What should the nurse keep in mind when planning to communicate with a child who has autism? a. The child has normal verbal communication. b. The child is expected to use sign language. c. The child may exhibit monotone speech and echolalia. d. The child is not listening if she is not looking at the nurse.

c. The child may exhibit monotone speech and echolalia.

The teaching plan for the parents of a 3-year-old child with amblyopia ("lazy eye") should include what instruction? a. Apply a patch to the child's eyeglass lenses. b. Apply a patch only during waking hours. c. Apply a patch over the "bad" eye to strengthen it. d. Cover the "good" eye completely with a patch.

d. Cover the "good" eye completely with a patch.

11. A woman has come to the clinic for preconception counseling because she wants to start trying to get pregnant in 3 months. She can expect the following advice: a. "Discontinue all contraception now." b. "Lose weight so that you can gain more during pregnancy." c. "You may take any medications you have been taking regularly." d. "Make sure that you include adequate folic acid in your diet."

d. "Make sure that you include adequate folic acid in your diet."

30. The major source of nutrients in the diet of a pregnant woman should be composed of: a. Simple sugars b. Fats c. Fiber d. Complex carbohydrates

d. Complex carbohydrates

24. When counseling a client about getting enough iron in her diet, the maternity nurse should tell her that: a. Milk, coffee, and tea aid iron absorption if consumed at the same time as iron. b. Iron absorption is inhibited by a diet rich in vitamin C. c. Iron supplements are permissible for children in small doses. d. Constipation is common with iron supplements.

d. Constipation is common with iron supplements.

10. A woman in week 34 of pregnancy reports that she is very uncomfortable because of heartburn. The nurse would suggest that the woman: a. Substitute other calcium sources for milk in her diet. b. Lie down after each meal. c. Reduce the amount of fiber she consumes. d. Eat five small meals daily.

d. Eat five small meals daily.

28. Nutrition is one of the most significant factors influencing the outcome of a pregnancy. It is an alterable and important preventive measure for various potential problems, such as low birth weight and prematurity. While completing the physical assessment of the pregnant client, the nurse can evaluate the client's nutritional status by observing a number of physical signs. Which sign would indicate that the client has unmet nutritional needs? a. Normal heart rate, rhythm, and blood pressure b. Bright, clear, shiny eyes c. Alert, responsive, and good endurance d. Edema, tender calves, and tingling

d. Edema, tender calves, and tingling

36. To determine the cultural influence on a patient's diet, the nurse should first: a. Evaluate the patient's weight gain during pregnancy b. Assess the socioeconomic status of the patient c. Discuss the four food groups with the patient d. Identify the food preferences and methods of food preparation common to that culture

d. Identify the food preferences and methods of food preparation common to that culture

A child with autism spectrum disorder is hospitalized for a treatment that will last about 1 week. How should the nurse make the child comfortable? 1 Ask the parents to accompany the child. 2 Modify the room according to the child's needs. 3 Explain the surroundings of the room. 4 Help the child perform daily routine tasks.

1 Children with autism spectrum disorders often are uncomfortable in a new environment and may not like to be with strangers. Therefore children with an autism spectrum disorder must be accompanied by their parents during hospitalization. While caring for a visually impaired child, the nurse modify the room according to the needs of the child. This helps prevent accidents. Because the child is not visually impaired, the nurse need not explain the surroundings of the room. Children with autism spectrum disorders often do not like assistance and prefer to perform their daily chores by themselves. Therefore the nurse should not help the child with such activities.

The nurse is speaking with the parents of a child with a very high fever. Which statement by the child's parent indicates a need for additional teaching? 1 "The temperature is quite high. It's life-threatening." 2 "I guess chills are common during high fever." 3 "Antipyretics should bring down the temperature." 4 "Fever has its own advantages for the body."

1 Contrary to popular belief, neither the rise in temperature nor its response to antipyretics indicates the severity or etiology of the infection. This casts doubt on the value of using fever as a diagnostic or prognostic indicator. Chills are common as the body tries to conserve heat and raise the central temperature to a new set point. Antipyretics bring down the temperature but they do not treat the condition. Fever has physiologic benefits, including increased white blood cell activity, interferon production and effectiveness, and antibody production and enhancement of some antibiotic effects.

A 10-year-old female child requires daily medications for a chronic illness. Her mother tells the nurse that she is always nagging her to take her medicine before school. What is the most appropriate nursing action to promote the child's compliance? 1 Establishing a contract with her, including rewards 2 Suggesting time-outs when she forgets her medicine 3 Discussing with her mother the damaging effects of nagging 4 Asking the child to bring her medicine containers to each appointment so they can be counted

1 For school-age children, behavior contracting associated with desirable rewards is an effective method for achieving compliance. Time-outs should be used only if the behavioral contracting is not successful. Although nagging is not an effective strategy, the nurse needs to assist the mother in problem-solving rather than criticize the actions. Monitoring the medicine supply may be tried if the contracting is not successful.

The nurse is assessing a newborn with Down syndrome. The newborn's parent tells the nurse, "We are having a hard time holding our baby. We didn't have this hard of a time with our other children." What would be the nurse's best response? 1 "Children with Down syndrome have lower muscle tone." 2 "This happens in some children because of undeveloped bonding." 3 "Are you more apprehensive because your child has Down syndrome?" 4 "You should see a counselor to help you cope with your child's condition."

1 Newborns with Down syndrome have joint hyperflexibility and low muscle tone. This can make it difficult to hold the newborn because he or she can go limp like a rag doll. This makes it difficult for the parents to embrace and provide warmth to their newborn. This may make parents feel that the newborn is not bonding with them, but difficulty holding the child does not indicate impaired bonding between the child and parents. Inability to understand the child's needs and nonverbal communication indicates undeveloped bonding. Asking the parents whether they are more apprehensive does not answer their question. It is also a closed-ended question, which is not therapeutic communication. Telling the parents they need to see a counselor is not appropriate. They just need support and teaching.

The best explanation for why pulse oximetry is used on young children is that it: 1 is noninvasive. 2 is better than capnography. 3 is more accurate than arterial blood gases. 4 provides intermittent measurements of O2.

1 Pulse oximetry is a noninvasive method to determine oxygen saturation. Capnography measures carbon dioxide exhalation. It does not reflect oxygen perfusion. Pulse oximetry is less invasive and easier to test than arterial blood gases. Pulse oximetry provides continuous or intermittent measurements of oxygen saturation.

The nurse finds that a patient has developed tachycardia and tachypnea after administration of a muscle relaxant. What is an appropriate nursing action? 1 Administer dantrolene sodium intravenously. 2 Use hot compresses on the neck and axillae. 3 Assess the patient's history of surgical procedures. 4 Administer an inhaled anesthetic.

1 The nurse should administer dantrolene sodium intravenously as the patient is showing signs of malignant hyperthermia (MH). Symptoms of MH include hypercarbia, elevated temperature, tachycardia, tachypnea, acidosis, muscle rigidity, and rhabdomyolysis. The nurse uses ice packs on the groin, axillae, and neck as MH is usually accompanied by hyperthermia. A family or previous history of sudden high fever associated with a surgical procedure and myotonia increase the risk for MH. But the patient will not be assessed for it now as MH has already set in. Use of inhaled anesthetics increase the risk of MH; therefore, they should not be administered as the patient is exhibiting symptoms of MH.

The nurse is educating new parents about the prevention of sudden infant death syndrome (SIDS). What position does the nurse tell the parents is the best sleeping position for their infant? 1 Supine 2 Prone 3 On the side 4 On a chair

1 The safest sleeping position to prevent SIDS is wholly supine. No pillows should be placed in a young infant's crib while the infant is sleeping. Lying prone or lying on the side can raise the risk of sudden infant death syndrome as the infant's nose may get covered by the bed. Putting the child on a chair may be unsafe as the child may fall off of the chair.

The nurse is administering an antipyretic medication to a child with a high fever. What action does the nurse take in the first hour after giving the medication? 1 Check the temperature again. 2 Administer another dose. 3 Check the child's weight. 4 Check for aspirin toxicity.

1 The temperature is usually retaken 30 minutes after the antipyretic is given to assess its effect but should not be measured repeatedly. Another dose is not administered before 4 hours but no more than 5 times in 24 hours. The child's weight is taken if fluid imbalance is suspected. Aspirin toxicity can cause hyperthermia and is only assessed for if such toxicity is suspected.

The nurse is caring for a child recovering from ankle surgery. The child was administered an anesthetic for pain relief. What does the nurse do to prevent respiratory complications during the postoperative care? 1 Encourage respiratory movement with incentive spirometers. 2 Conduct hyperventilation with 100% oxygen. 3 Initiate cooling measures such as ice packs to the groin and axillae. 4 Change the child's position every 24 hours.

1 To prevent pneumonia and other respiratory complications after surgery, the nurse encourages respiratory movement with incentive spirometers or other motivating activities. Hyperventilation with 100% oxygen and cooling measures such as applying ice packs to the groin and axillae are done when malignant hyperthermia (MH) is diagnosed. The child's position is changed every 2 hours and deep breathing is encouraged.

The nurse is assessing a child with autism for prognostic factors. What findings in the child suggest a better prognosis? Select all that apply. 1 Male sex 2 Early recognition 3 Functional speech 4 Lower intelligence 5 Behavioral impairment

1, 2, 3 Male sex carries a more favorable prognosis than female sex. Early recognition allows early intervention to help the child recover. Children with functional speech have a better prognosis than those who do not have functional speech. Children with higher intelligence have a more favorable prognosis than children with lesser intelligence. Children who do not have behavioral impairment have a better prognosis than children with behavioral impairment.

The nurse is caring for a child who is scheduled to undergo an ostomy procedure. What are possible causes for a child to need undergo an ostomy procedure? Select all that apply. 1 Necrotizing enterocolitis 2 Hirschsprung disease 3 Crohn disease 4 Diseases of the bladder 5 Difficulty urinating

1, 2, 3, 4 A stoma is an opening, either natural or surgically created, which connects a portion of the body cavity to the outside environment. Children may require stomas for various health problems such as necrotizing enterocolitis, imperforate anus, and Hirschsprung disease. In older children, the most frequent causes are Crohn's disease and ureterostomies due to bladder defects. Difficulty in urinating is not reason enough for an ostomy unless the health care provider has diagnosed an underlying disorder that requires an ostomy.

A 9-year-old patient is scheduled for a surgical procedure next week. What teachings will the nurse include to ensure the patient's assent? Select all that apply: 1 Inform the patient about the nature of the condition. 2 Tell the patient what can be expected. 3 Inform the patient how consent is obtained. 4 Assess patient understanding. 5 Solicit an expression of the patient's willingness.

1, 2, 4, 5 Assent of the patient should include four key teachings. The nurse should help the patient achieve a developmentally appropriate awareness of the nature of their condition. The nurse should tell the patient what can be expected. The nurse should also make a clinical assessment of the patient's understanding, and solicit an expression of the patient's willingness to accept the proposed procedure. Such measures help reduce anxiety in the patient and are important for their assent. Information about the legal procedures of obtaining consent is not related to the surgical procedure and will not be as helpful in reducing anxiety.

A pregnant woman is diagnosed with a rubella infection during a prenatal checkup. What does the nurse expect the health care provider will tell the patient? Select all that apply. "The newborn may: 1 Have vision difficulties." 2 Have growth impairment." 3 Have difficulty hearing." 4 Develop breathing problems." 5 Not be able to concentrate."

1, 3 Rubella infections during pregnancy may cause hearing and visual loss in the newborn. However, these impairments may disappear as the child grows. Rubella infections do not cause growth retardation. Growth hormone deficiency or Turner syndrome can lead to growth impairment. Respiratory disorders or allergic reactions can result from hypersensitivities and can cause difficulty breathing in the newborn. A decreased ability to concentrate indicates impaired cognition. It usually results from inadequate intake of omega-3 fatty acids by the mother during pregnancy.

A 4-year-old child is seen in a clinic for a hearing impairment. What action does the nurse observe in the child to confirm hearing impairment? Select all that apply. The child: 1 Screeches happily when looking at a toy. 2 Has difficulty trying to read a book. 3 Does not respond when an alarm sounds. 4 Points at his tummy to indicate hunger. 5 Speaks fast, stutters, and has speech delay.

1, 3, 4 A child with a hearing impairment yells or screeches in pleasure because the child cannot hear how loud these sounds are. The child also does not respond to loud sounds and prefers nonverbal communication such as pointing. A child who has difficulty reading a book may have a visual impairment. Rapid speech with stuttering and speech delay are symptoms of fragile X syndrome.

A nurse is preparing a teaching session for parents on prevention of childhood hearing loss. The nurse should include that the most common cause of hearing impairment in children is: a. Auditory nerve damage. c. Congenital rubella. b. Congenital ear defects. d. Chronic otitis media.

d. Chronic otitis media.

The nurse is caring for a child after surgery. The child refuses to eat any food for lunch. What is an appropriate intervention by the nurse? 1 Insist that the child eat some more food. 2 Give the child a favorite food. 3 Call the child's parents. 4 Refer the matter to the dietician.

2 Although it is best to provide high-quality, nutritious foods, the child may desire foods and liquids that contain mostly empty or non-nutritional calories. Some well-tolerated foods include gelatin, diluted clear soups, carbonated drinks, flavored ice pops, dry toast, and crackers. Even though these substances are not nutritious, they can provide necessary fluid and calories. The nurse should not force the child to eat. Forcing a child to eat meets with rebellion and reinforces the behavior as a control mechanism. Calling the child's parents may make the child more irritated. The dietician is referred to before meal planning, and is consulted only if the child refuses to have any hospital food.

Which antipyretic is associated with Reye syndrome in children? 1 Acetaminophen (Tylenol) 2 Aspirin (Bayer) 3 Ibuprofen (Advil) 4 Norfloxacin (Noroxin)

2 Aspirin should not be given to children because of its association in children with influenza virus or chickenpox and Reye syndrome. Other antipyretics include acetaminophen (Tylenol), and nonsteroidal antiinflammatory drugs (NSAIDs). Acetaminophen (Tylenol) is the preferred drug. One nonprescription NSAID, ibuprofen (Advil), is approved for fever reduction in children as young as 6 months of age. Norfloxacin (Noroxin) is an antibiotic and is usually prescribed for bacterial infection of the gastrointestinal system.

The nurse assesses an infant at birth for height, weight, and other vital signs. What should the nurse include in the assessment to identify a conductive hearing disorder? The nurse assesses: 1 To see whether the infant's eyes move toward a flashlight. 2 The infant's response to an auditory stimulus. 3 The infant's vocal expressions during vocal communication. 4 The infant's physical activity toward a large moving object.

2 Assessment of an infant's response to auditory stimulation is used to detect a conductive hearing impairment in the newborn. Because the nurse suspects hearing impairment, the infant might have failed to respond to auditory stimulus. Eye movement following a flashlight helps in assessing the infant's vision. Vocal expressions or sounds produced in response to communication may indicate whether the infant has a speech or hearing impairment. Assessment of infant's physical activity toward a large moving object is useful to assess the child's muscle coordination. It also tests the infant's visual abilities.

A visually impaired child is hospitalized for eye surgery. What nursing intervention should be included in the plan of care to encourage the child to be independent? The nurse: 1 Does not keep a stool or small desk near the bed. 2 Instructs the cleaner not to move the furniture around. 3 Gives the child work to do while the child is in the hospital. 4 Does not educate the child about the treatment procedures.

2 Changing furniture positions can result in accidents, so this must be avoided. A small stool or a desk should be placed near the bed to support the child so that he or she can climb into bed easily. The child is a patient in the hospital and should not be asked to work while there. Educating the child about the procedures that will be carried out for the treatment will help the child understand and mentally prepare for them.

The relative of a child receiving oxygen therapy brings the child a remote-controlled airplane as a gift. What safety risk does the toy present to the patient? 1 The toy may cause suffocation. 2 The toy can cause fire. 3 The toy may have toxic lead paint. 4 The toy may distract the child.

2 Electrical or friction toys are not safe because sparks can cause oxygen to ignite. Bags made of plastic or other material are more likely to cause suffocation. Toxic lead paint is usually a concern for toys from unknown manufacturers. Distraction can be an advantage for the child as it is likely to bring some joy and solace.

The nurse finds that a child under care for a gastrostomy experienced fecal incontinence. What is a priority intervention by the nurse? 1 Use a disinfectant immediately on the skin. 2 Gently clean the skin and remove moisture. 3 Elevate the bed no more than 30 degrees. 4 Use an adhesive remover to remove fecal matter

2 Fecal incontinence, especially when mixed with urine, wound drainage, or gastric drainage around gastrostomy tubes can erode the epidermis. The nurse should gently clean the skin and remove the excess moisture. A disinfectant is used when there is a risk of infection and the skin barrier is damaged. Elevating the bed to no more than 30 degrees for short periods prevents friction injuries. Adhesive remover is used to remove adhesives and may not be effective for removing fecal matter.

The nursing instructor is teaching a group of students about gastrostomy feeding. Which statement by the student indicates a need for additional teaching? "It is used: 1 "...when tube passage is not possible through the mouth." 2 "...when tube passage is possible through the pharynx." 3 "...when tube passage is not possible through the cardiac sphincter." 4 "....to avoid the constant irritation of an NG tube."

2 Feeding by way of gastrostomy, or G tube, is often used for children in whom passage of a tube through the pharynx, mouth, esophagus, and cardiac sphincter of the stomach is contraindicated or impossible. It is also used to avoid the constant irritation of an NG tube in children who require tube feeding over an extended period.

What does the nurse keep in mind while administering an enema to a child? 1 The nurse should not give details about the procedure. 2 The buttocks of the child should be held together briefly. 3 Pillows should not be used during the procedure. 4 Administration of enemas should be noninvasive in children

2 Infants and young children are unable to retain the solution after it is administered, so the buttocks must be held together for a short time to retain the fluid. A careful explanation may help ease any concerns or fears the child may have about the procedure. The enema is administered and expelled while the child is lying with the buttocks over the bedpan and with the head and back supported by pillows. An enema is an intrusive procedure.

The nurse is caring for an infant on gastrostomy feeding. The nurse gives the infant a small and safe pacifier to suck on. What is the rationale behind this? 1 Nutritive sucking is essential during gastrostomy feeding. 2 It keeps the child from crying too much. 3 It prevents the risk of aspiration during gastrostomy feeding. 4 It enhances the nutritive value of gastrostomy feeding.

2 Infants are giving a pacifier as non-nutritive sucking can help decrease crying and aid in weight gain. Pacifiers do not increase the risk of aspiration as children suck on them. Pacifiers do not mix with gastrostomy feedings and cannot enhance the nutritive value of gastronomy feeding.

Early detection of a hearing impairment is critical because of its effect on areas of a child's life. The nurse should evaluate further for effects of the hearing impairment on: 1 reading development. 2 speech development. 3 relationships with peers. 4 performance at school

2 The ability to hear sounds is essential for the development of speech. Babies imitate the sounds that they hear. The child will have greater difficulty learning to read, but the primary issue of concern is the effect on speech. Relationships with peers and performance at school will be affected by the child's lack of hearing. The effect will be augmented by difficulties with oral communication.

Bronchial (postural) drainage generally is performed: 1 immediately before all aerosol therapy. 2 before meals and at bedtime. 3 immediately on arising and at bedtime. 4 thirty minutes after meals and at bedtime.

2 The most effective time for bronchial drainage is before meals and at bedtime. It is more effective after other respiratory therapy, such as bronchodilators or nebulizer treatments. The procedure should be done 3 to 4 times each day. When drainage is done after meals, it may cause the child to vomit.

The nurse is caring for a teenager scheduled for surgery. What criteria does the nurse use to obtain valid consent from the patient? The patient should: Select all that apply. 1 Be intelligent. 2 Be over the age of majority. 3 Be well-informed. 4 Act voluntarily. 5 Be healthy

2, 3, 4 To obtain valid informed consent, health care providers must meet three conditions. The person must be capable of giving consent; he or she must be over the age of majority. The person must receive the information needed to make an intelligent decision. The person must act voluntarily when exercising freedom of choice. The intelligence of the patient is not measured; the patient is only required to be competent enough to make decisions.

The nurse is preparing a child for an endotracheal tube (ET) placement. How does the nurse verify the placement of the tube? Select all that apply. 1 Visualization of unilateral chest expansion 2 Auscultation over the epigastrium 3 Examination of water vapor in the tube 4 Waveform verification with continuous capnography 5 Examination using a chest radiography

2, 3, 4, 5 ET tube placement should be verified by at least one clinical sign and at least one confirmatory technology. Such technologies include auscultation over the epigastrium and the lung fields bilaterally in the axillary region, examination of water vapor in the tube, and waveform verification with continuous capnography. Chest radiography can also be used to verify placement. Visualization of bilateral (and not unilateral) chest expansion is used to verify placement of the ET tube.

The nurse is preparing to insert a nasogastric (NG) tube for a child with impaired swallowing capacity. Arrange the steps of the procedure in the correct order. 1. Flush the tube with sterile water. 2. Place child supine with head slightly hyperflexed. 3. Measure the tube for approximate length. 4. Stabilize the tube by holding or taping it to the cheek. 5. Warm the formula to room temperature.

2, 3, 4, 5, 1 The child should be in a supine position with head slightly hyperflexed. The tube is measured for approximate length of insertion and marked. After insertion, the tube is stabilized by holding or taping it to the cheek. Warm formula to room temperature before starting the flow. The tube is flushed with sterile water after the feeding.

What should the nurse do to communicate with a patient who is cognitively impaired and speaks a foreign language? 1 Insert the patient's hearing aids. 2 Use verbal expressions. 3 Use a language translator. 4 Use visual aids and drawings.

3 A language translator should be used when any patient speaks a foreign language, no matter what his or her cognitive level is. Inserting the patient's hearing aids will help the patient hear, but it will not break the language barrier. Verbal expressions can be helpful, but they are not as effective as having a translator speaking the patient's language. Visual aids and drawings may be helpful, but not everything is easily communicated with these methods.

The parents of a 3-year-old child report seeing a whitish glow in the child's eyes. The nurse begins to examine the child. What information should the nurse give to the parents before assessment? "The child: 1 Needs hematologic assessment for confirmation of diagnosis." 2 May not be able to distinguish between colors." 3 May not see clearly for some time after the examination." 4 Needs immediate hospitalization after the examination."

3 A strange light in the eyes indicates that the child may have retinoblastoma. It is diagnosed by ophthalmoscopic examination, which involves dilation of the pupil. During this procedure, the eyes become sensitive, and the child may not be able to see clearly for some time. Informing the parents about it will reduce anxiety. Hematologic assessment is not used to diagnose retinoblastoma. A retinoblastoma can be diagnosed by ophthalmoscopic examination under general anesthesia and with imaging studies, including ultrasonography and computed tomography. A child with visual impairment may have difficulty distinguishing between colors, but it is not helpful for the parents to learn about it before the child's illness is diagnosed. The child may need immediate hospitalization after the examination depending on the severity of tumor, but the nurse should not tell the parents about hospitalization because it could make them panic.

A 10-year-old child is diagnosed with an autism spectrum disorder (ASD). The parents ask the nurse about the cause of the disorder. Which answer given by the nurse is most appropriate? 1 "Autism is caused by a high intake of proteins during pregnancy." 2 "The disorder is caused by vaccines that contain thimerosal." 3 "The exact cause of autism spectrum disorders is unknown." 4 "Alcohol consumption during pregnancy is linked to autism."

3 Although the exact cause of ASD is not known, the nurse should always help parents understand that they are not responsible for the child's condition. There are many theories about the cause of ASD, but nothing is definitive. High intake of proteins is necessary during pregnancy because it promotes proper growth and development of the fetus. Vaccines containing thimerosal are not associated with ASD. Thimerosal is a preservative found in some vaccines. Consumption of alcohol during pregnancy leads to fetal alcohol syndrome, not autism.

Several types of long-term central venous access devices are used. A benefit of using an implanted port (e.g., Port-a-Cath) is that it: 1 is easy to use for self-administered infusions. 2 does not need to pierce the skin for access. 3 does not need to limit regular physical activity, including swimming. 4 cannot dislodge from the port, even if child plays with port site.

3 Because this device is totally under the skin, there are no activity limitations for the child. The port has to be accessed with a special needle. Because the port is totally under the skin, a needle must be used to access the port. The port site is under the skin, so there is nothing for the child to play with.

The parents of a cognitively impaired child ask the nurse for guidance with discipline. The nurse's best response is: 1 "Discipline is ineffective with cognitively impaired children." 2 "Discipline is not necessary for cognitively impaired children." 3 "Behavior modification is an excellent form of discipline." 4 "Physical punishment is the most appropriate form of discipline."

3 Behavior modification with positive reinforcement is effective in children with cognitive impairment. Discipline is essential in assisting the child in developing boundaries. Positive behaviors and desirable actions should be reinforced. Most children with cognitive impairment will not be able to understand the reason for the physical punishment; consequently behavior will not change as a result of the punishment.

A neonate had corrective surgery 3 days ago for esophageal atresia. The nurse notices that after the child receives his gastrostomy feeding, there is often a backup of formula feeding into the tube. As a result, the nurse should: 1 position the child in a supine position after feedings. 2 position the child on his or her left side after feedings. 3 leave the gastrostomy tube open and suspended after feedings. 4 leave the gastrostomy tube clamped after feedings.

3 By keeping the tube open to air, the buildup of pressure on the operative site will be prevented. The child should be positioned on the right side with head elevated at approximately 30 degrees. The formula is backing up into the tube because of the delayed emptying. Leaving the tube clamped will create pressure on the operative site.

A child with fragile X syndrome was prescribed clonidine (Catapres) to improve attention and decrease hyperactivity. What other intervention may improve the child's cognitive ability? 1 Aromatherapy and hydrotherapy 2 Protein replacement and gene therapy 3 Language and occupational therapy 4 Hormone and biologic therapy

3 Children with fragile X syndrome have impaired cognitive development and may be prescribed clonidine (Catapres) to improve attention span and decrease hyperactivity. Other interventions that can improve cognitive ability in these children include speech and language therapies, occupational therapy, and special educational programs. Aromatherapy and hydrotherapy are useful for reducing stress and anxiety. Protein and gene replacement involves replacing the defected gene. It does not improve cognitive ability. Hormone therapy can be given to treat endocrine disorders. Biotherapy is given to strengthen the patient's immunity.

A 10-year-old child has moderate cognitive impairment. With which activity would a teacher expect the child to need help? 1 Copying information from the board 2 Learning safe and healthy habits 3 Performing arithmetic calculations 4 Communicating with classmates

3 Students with moderate cognitive impairment (IQ of 50-55) have difficulty with functional reading and arithmetic calculations. The student can perform simple manual skills, such as copying information from the board, learning safe and healthy habits, and communicating with classmates.

The nurse is caring for a child with cognitive impairment. Which statement made by the nurse to the parents is a reason for concern? 1 "I need to know more about cognitive impairment." 2 "I will ask the other staff to help with the child's care." 3 "I do not know what is going on with this child's health." 4 "I'll ask the health care provider to clarify my question."

3 The statement "I do not know what is going on with this child's health" is inappropriate because it indicates that the nurse does not understand the child's needs. The statement "I need to know more about cognitive impairment" shows the nurse's desire to learn about the disorder and help treat the child better. The statement "I will ask the other staff to help with the child's care" shows the nurse's desire to increase his or her skill level. The statement "I will ask the health care provider to clarify my question" is an inappropriate statement made by the nurse. It implies that the nurse is not aware of the care that should be given to a cognitively impaired child.

A child in the clinic exhibits reduced visual acuity in one eye despite appropriate optical correction. The nurse expects the child's health care provider to diagnosis the child with: 1 myopia. 2 hyperopia. 3 amblyopia. 4 astigmatism.

3 Visual acuity in one eye despite appropriate optical correction is amblyopia. Myopia is nearsightedness, which is the ability to see objects up close but not clearly at a distance. Hyperopia is farsightedness, which is the ability to see distant objects clearly but not those up close. Astigmatism is an alteration in vision caused by unequal curvature in the refractive apparatus of the eye.

The nurse is removing the tape of an intravenous catheter in a child. What is the most appropriate instruction given by the nurse? 1 "Stay calm while I remove the catheter." 2 "This will cause just a little bit of pain." 3 "Let's remove the tape together." 4 "You are the bravest kid in the world."

3 When it is time to discontinue an intravenous infusion, many children are distressed by the thought of catheter removal. Encouraging children to remove or help remove the tape from the site provides them with a measure of control and often fosters their cooperation. The nurse should not remove the tape without first asking the child for cooperation, so asking he child to remain calm and then remove the catheter is inappropriate. The nurse should not use the word pain; saying that it would cause discomfort is more appropriate. Telling children that they are bravest in the world may actually make them fear removal of the catheter as children are used to such statements during distress.

Parents have learned that their 6-year-old child has autism. The nurse may help the parents to cope by explaining that the child may: a. Have an extremely developed skill in a particular area. b. Outgrow the condition by early adulthood. c. Have average social skills. d. Have age-appropriate language skills.

a. Have an extremely developed skill in a particular area.

A couple visits the hospital for a prenatal checkup. On reviewing the genetic analysis report, the nurse finds that the male partner has fragile X syndrome. What should the nurse interpret from these findings? Select all that apply. 1 All of their sons will have a 50% chance of being affected. 2 All of their sons will be carriers for fragile X syndrome. 3 The chance of a daughter being affected is 50%. 4 All daughters will be carriers for fragile X syndrome. 5 All sons will be carriers and will have fragile X syndrome.

3, 4 Fragile X syndrome is an X-linked dominant syndrome with reduced penetrance. About 50% of daughters with fathers affected by fragile X syndrome will be affected because the dominant X chromosome can be from the affected father. All daughters with an affected father will be carriers. The sons get Y chromosomes from the father, so they are not necessarily carriers of the syndrome or affected by the syndrome. The sons can be carriers or affected if the syndrome is passed from the mother.

The nurse is assessing a child with Down syndrome. What findings in the child should alert the nurse to report to the health care provider immediately? Select all that apply. 1 Loss of pain sensation 2 Loss of impulse control 3 Loss of established motor skill 4 Loss of established bowel control 5 Loss of established bladder control

3, 4, 5 Loss of established motor skill and bowel and bladder control indicate spinal cord compression and must be reported immediately. The child with Down syndrome may have persistent neck pain caused by spinal cord compression. These children do not have impaired pain sensation. Children with Down syndrome are not aggressive. Loss of impulse control is not seen in such children.

Informed consent is valid when: Select all that apply. 1 universal consent is used. 2 it is completed only for major surgery. 3 a person is over the age of majority and competent. 4 information is provided to make an intelligent decision. 5 the choice exercised is free of force, fraud, duress, or coercion.

3, 4, 5 The age of majority is usually 18 years. The term competent is defined as possessing the mental capacity to make choices and understand their consequences. Enough information is provided so that the person can make an intelligent decision. The person giving consent does so voluntarily; that is, freely without coercion, any form of constraint, force, fraud, duress, or deceit. Universal consent is not sufficient. Informed consent must be obtained for each surgical or diagnostic procedure. Informed consents must be obtained for major and minor surgery, diagnostic tests, medical treatments, release of medical information, postmortem examination, removal of a child from the health care provider against medical advice, and photographs for medical, educational, or public use.

When a child with mild cognitive impairment reaches the end of adolescence, what characteristic would be expected? a. Achieves a mental age of 5 to 6 years b. Achieves a mental age of 8 to 12 years c. Is unable to progress in functional reading or arithmetic d. Acquires practical skills and useful reading and arithmetic to an eighth-grade level

b. Achieves a mental age of 8 to 12 years

The nursing instructor is teaching a group of students about using chest tubes in children. Which statement by the student indicates a need for additional teaching? 1 Excess fluid is removed by chest tubes. 2 Chest tubes clear air from lungs. 3 Pneumothorax may need chest tubes. 4 Chest tubes eliminate leaked blood.

4 Before chest tube insertion, hematologic and coagulation studies are assessed for any risk of bleeding during the procedure. Chest tubes therefore do not remove blood. They are used to remove fluid or air from the pleural or pericardial space. Chest tube placement is often a necessity for pneumothorax, hemothorax, chylothorax, empyema, and pleural or pericardial effusion.

The parent of a visually impaired infant says to the nurse, "I am afraid that my child may not be able to bond with me because my child cannot maintain eye contact with me." The nurse teaches the parent about other signs that indicate that the child is responding. What should the nurse include in the explanation? 1 "The child compensates by increasing listening to your voice and smiling." 2 "The child's attention span decreases when a parent is trying to communicate." 3 "The child does not make throaty sounds when a parent is trying to communicate." 4 "The child's breathing or activity increases when the child is in contact with or near a parent."

4 Changes in respiratory patterns and increasing activity reflect the child's excitement about being close to the parent. The child is able to hear the parent's voice but is not able to smile when hearing the parent's voice at this age. This happens after about 2 years of age. Decreased attention span could be caused by lack of interest in the communication. When a child makes sounds in response to the parent's communication, it is sign of intimate bonding between the child and parent.

The parents brought their child to the emergency department after a needle penetrated the child's eye. Which action should the nurse perform while caring for the child? 1 Examine the eye to look for foreign bodies. 2 Irrigate the eye to remove the needle from the eye. 3 Evert the upper eyelid to wash the eye thoroughly. 4 Observe for hyphema and reaction of the pupil to light.

4 If a child has a penetrating eye injury of any kind, the nurse should examine the eye to determine whether any aqueous humor has leaked from the penetration site. The nurse should observe the presence of hyphema, or bleeding from the eye. The nurse should also assess for pupillary reaction to light because it helps assess the functioning of the pupil. The nurse does not need to examine the eye for foreign bodies because there is already a foreign body in the eye. If the child is experiencing a penetrating eye injury, the nurse does not irrigate the eye to remove the object because this can further damage the cornea. In the case of chemical burns, the nurse rinses the eye by everting the upper eyelid.

The nurse needs to take the blood pressure of a preschool boy for the first time. Which action is best in gaining his cooperation? 1 Take his blood pressure when a parent is there to comfort him. 2 Tell him that this procedure will help him get well more quickly. 3 Explain to him how the blood flows through the arm and why the blood pressure is important. 4 Permit him to handle equipment and see the dial move before putting the cuff in place.

4 Permitting the child to handle the equipment and see the dial move allows him to play out the experience ahead of time. The parent's presence will be helpful, but it will not alleviate fear of the unknown. The child will not be able to understand the relationship between blood pressure and feeling better. Additionally, there is no evidence that this procedure will help him get well more quickly. Explaining how blood flows through the arm is too complex for this age group.

The diagnosis of cognitive impairment is based on the presence of: 1 intelligence quotient (IQ) of 75 or less. 2 IQ of 70 or less. 3 subaverage intellectual functioning, deficits in adaptive skills, and onset at any age. 4 subaverage intellectual functioning, deficits in adaptive skills, and onset before 18 years of age.

4 The diagnosis of cognitive impairment includes subaverage intellectual functioning and deficits in adaptive skills, including an onset before age 18. IQ is only one component of the diagnosis of cognitive impairment. The onset of the deficit must be before age 18 to meet the diagnosis of cognitive impairment.

The nurse is evaluating a child for suspected autism. Which finding in the child suggests autism? 1 Limited functional play 2 Avoidance of body contact 3 Language delay at an early age 4 Inability to maintain eye contact

4 The hallmark of autism is an inability to maintain eye contact with another person. Limited functional play may be seen in children with autism, but it is not a hallmark of autism. Autistic children also avoid body contact, but it is not a hallmark finding. Language delay at an early age is not a hallmark of autism; however, children with autism may exhibit language delay at an early age.

What nursing care should be provided to a school-aged child with cognitive impairment? 1 Periodic testing of thyroid function 2 Education on sexuality 3 Education on self-care skills for the child 4 Speech therapy referral for the child

4 The nurse should refer the child with cognitive impairment for speech therapy. It helps improve communication and promotes social behavior of the child. Periodic testing of the thyroid function is done if the child has Down syndrome. Thyroid function is not altered in all children with cognitive impairment. Sexual information is given to adolescents with cognitive impairment. Younger school-age children with cognitive impairment may not understand information regarding sexuality. Self-care skills should be taught by the parents to children with cognitive impairment.

A 13-year-old patient with an ankle injury requires minor surgery. The parents of the patient have given their consent but are unable to wait during the procedure. What is the best nursing action in this context? 1 Persuade the parents to be with the patient. 2 Ask the patient's school teacher to be present. 3 Conduct the surgery when either parent is available. 4 Adhere to the parents' wishes of not participating

4 The nurse should support parents who do not want to be present in their decision and encourage them to remain close by so they can be available to support the child immediately after the procedure. The nurse should not insist that the parents sit through the procedure as it can create tension between the parents and the child. Reaching the patient's school teacher is inappropriate if parents are around and have decided not to stay. Waiting for either parent to sit through the procedure may delay treatment unnecessarily.

A child with autism spectrum disorder (ASD) is admitted to the hospital with pneumonia. The nurse should plan which priority intervention when caring for the child? a. Maintain a structured routine and keep stimulation to a minimum. b. Place the child in a room with a roommate of the same age. c. Maintain frequent touch and eye contact with the child. d. Take the child frequently to the playroom to play with other children.

a. Maintain a structured routine and keep stimulation to a minimum.

The major consideration when selecting toys for a child who is cognitively impaired is: a. Safety. b. Age appropriateness. c. Ability to provide exercise. d. Ability to teach useful skills.

a. Safety

The most common clinical manifestation of retinoblastoma is: a. Glaucoma. c. White eye reflex. b. Amblyopia. d. Sunken eye socket.

c. White eye reflex.

Which assessment findings indicate to the nurse a child has Down syndrome (select all that apply)? a. High-arched, narrow palate b. Protruding tongue c. Long, slender fingers d. Transverse palmar crease e. Hypertonic muscle tone

A, B, D

A nurse is providing a parent information regarding autism. Which statement made by the parent indicates understanding of the teaching? a. "Autism is characterized by periods of remission and exacerbation." b. "The onset of autism usually occurs before 3 years of age." c. "Children with autism have imitation and gesturing skills." d. "Autism can be treated effectively with medication."

b. "The onset of autism usually occurs before 3 years of age."

The most common type of hearing loss, which results from interference of transmission of sound to the middle ear, is called: a. Conductive. c. Mixed conductive-sensorineural. b. Sensorineural. d. Central auditory imperceptive.

a. Conductive.

A child with autism is hospitalized with asthma. The nurse should plan care so that the: a. Parents' expectations are met. b. Child's routine habits and preferences are maintained. c. Child is supported through the autistic crisis. d. Parents need not be at the hospital.

b. Child's routine habits and preferences are maintained.

Mark, a 9-year-old with Down syndrome, is mainstreamed into a regular third-grade class for part of the school day. His mother asks the school nurse about programs such as Cub Scouts that he might join. The nurse's recommendation should be based on knowing that: a. Programs such as Cub Scouts are inappropriate for children who are cognitively impaired. b. Children with Down syndrome have the same need for socialization as other children. c. Children with Down syndrome socialize better with children who have similar disabilities. d. Parents of children with Down syndrome encourage programs such as scouting because they deny that their children have disabilities.

b. Children with Down syndrome have the same need for socialization as other children.

An implanted ear prosthesis for children with sensorineural hearing loss is a(n): a. Hearing aid. c. Auditory implant. b. Cochlear implant. d. Amplification device.

b. Cochlear implant.

A newborn assessment shows separated sagittal suture, oblique palpebral fissures, depressed nasal bridge, protruding tongue, and transverse palmar creases. These findings are most suggestive of: a. Microcephaly. c. Cerebral palsy. b. Down syndrome. d. Fragile X syndrome.

b. Down syndrome

A parent whose child has been diagnosed with a cognitive deficit should be counseled that intellectual impairment: a. Is usually due to a genetic defect. b. May be caused by a variety of factors. c. Is rarely due to first-trimester events. d. Is usually caused by parental intellectual impairment.

b. May be caused by a variety of factors.

Distortion of sound and problems in discrimination are characteristic of which type of hearing loss? a. Conductive b. Sensorineural c. Mixed conductive-sensorineural d. Central auditory imperceptive

b. Sensorineural

A parent asks the nurse why a developmental assessment is being conducted for a child during a routine well-child visit. The nurse answers based on the knowledge that routine developmental assessments during well-child visits are: a. Not necessary unless the parents request them. b. The best method for early detection of cognitive disorders. c. Frightening to parents and children and should be avoided. d. Valuable in measuring intelligence in children.

b. The best method for early detection of cognitive disorders.

Which statement by a parent about a child's conjunctivitis indicates that further teaching is needed? a. "I'll have separate towels and washcloths for each family member." b. "I'll notify my doctor if the eye gets redder or the drainage increases." c. "When the eye drainage improves, we'll stop giving the antibiotic ointment." d. "After taking the antibiotic for 24 hours, my child can return to school."

c. "When the eye drainage improves, we'll stop giving the antibiotic ointment."

A 10-year-old patient is talking to the nurse about wanting to try contact lenses instead of wearing glasses. She states that the other children at her school call her "four-eyes." Contact lenses should be prescribed for a child who is: a. At least 12 years of age. b. Able to read all the written information and instructions. c. Able to independently care for the lenses in a responsible manner. d. Confident that she really wants contact lenses.

c. Able to independently care for the lenses in a responsible manner.

The child with Down syndrome should be evaluated for what characteristic before participating in some sports? a. Hyperflexibility b. Cutis marmorata c. Atlantoaxial instability d. Speckling of iris (Brushfield's spots)

c. Atlantoaxial instability

Which term refers to opacity of the crystalline lens that prevents light rays from entering the eye and refracting on the retina? a. Myopia b. Amblyopia c. Cataract d. Glaucoma

c. Cataract

When caring for a newborn with Down syndrome, the nurse should be aware that the most common congenital anomaly associated with Down syndrome is: a. Hypospadias. c. Congenital heart disease. b. Pyloric stenosis. d. Congenital hip dysplasia.

c. Congenital heart disease.

Which action is contraindicated when a child with Down syndrome is hospitalized? a. Determine the child's vocabulary for specific body functions. b. Assess the child's hearing and visual capabilities. c. Encourage parents to leave the child alone for extended periods of time. d. Have meals served at the child's usual mealtimes.

c. Encourage parents to leave the child alone for extended periods of time.

The nurse is discussing sexuality with the parents of an adolescent girl with moderate cognitive impairment. Which should the nurse consider when dealing with this issue? a. Sterilization is recommended for any adolescent with cognitive impairment. b. Sexual drive and interest are limited in individuals with cognitive impairment. c. Individuals with cognitive impairment need a well-defined, concrete code of sexual conduct. d. Sexual intercourse rarely occurs unless the individual with cognitive impairment is sexually abused.

c. Individuals with cognitive impairment need a well-defined, concrete code of sexual conduct.

A father calls the emergency department nurse saying that his daughter's eyes burn after getting some dishwasher detergent in them. The nurse recommends that the child be seen in the emergency department or by an ophthalmologist. What should the nurse recommend before the child is transported? a. Keep the eyes closed. b. Apply cold compresses. c. Irrigate eyes copiously with tap water for 20 minutes. d. Prepare a normal saline solution (salt and water) and irrigate eyes for 20 minutes.

c. Irrigate eyes copiously with tap water for 20 minutes

The pediatric nurse understands that fragile X syndrome is: a. A chromosome defect affecting only females. b. A chromosome defect that follows the pattern of X-linked recessive disorders. c. The second most common genetic cause of cognitive impairment. d. The most common cause of noninherited cognitive impairment.

c. The second most common genetic cause of cognitive impairment.

Which teaching guideline helps prevent eye injuries during sports and play activities? a. Restrict helmet use to those who wear eyeglasses or contact lenses. b. Discourage the use of goggles with helmets. c. Wear eye protection when participating in high-risk sports such as paintball. d. Wear a face mask when playing any sport or playing roughly.

c. Wear eye protection when participating in high-risk sports such as paintball.

An adolescent male visits his primary care provider complaining of difficulty with his vision. When the nurse asks the adolescent to explain what visual deficits he is experiencing, the adolescent states, "I am having difficulty seeing distant objects; they are less clear than things that are close." What disorder does the nurse suspect the adolescent has? a. Hyphema b. Astigmatism c. Amblyopia d. Myopia

d. Myopia

An adolescent gets hit in the eye during a fight. The school nurse, using a flashlight, notes the presence of gross hyphema (hemorrhage into anterior chamber). The nurse should: a. Apply a Fox shield. b. Instruct the adolescent to apply ice for 24 hours. c. Have adolescent rest with eye closed and ice applied. d. Notify parents that adolescent needs to see an ophthalmologist.

d. Notify parents that adolescent needs to see an ophthalmologist.


Related study sets

English Language A level: Word Classes

View Set

Managerial Accounting Chapter 1 hw 1

View Set

Chapter 17 - Business Organizations

View Set

Science Biology Test 2- Body Systems

View Set

Elsevier Adaptive Quizzing: Safety and Infection Control

View Set

Material Chapter 6 and Part 1 of Chapter 7

View Set